Naked Science Forum

On the Lighter Side => New Theories => Topic started by: timey on 16/02/2017 03:18:52

Title: Is there a discrepancy with the equivalence principle?
Post by: timey on 16/02/2017 03:18:52
This thread is an offshoot from Mike's thread that can be found here:

https://www.thenakedscientists.com/forum/index.php?topic=69764.0 (https://www.thenakedscientists.com/forum/index.php?topic=69764.0)

Ever since I saw this information breaking in the national news:

https://www.nist.gov/news-events/news/2010/09/nist-pair-aluminum-atomic-clocks-reveal-einsteins-relativity-personal-scale (https://www.nist.gov/news-events/news/2010/09/nist-pair-aluminum-atomic-clocks-reveal-einsteins-relativity-personal-scale)

I have been researching every aspect, and am well read on both caesium atomic clocks and quantum clocks - so the information that I have given is good, as a synopsis, although admittedly after reading the available info now, rather than back in 2010, I see they have seriously overcome the problems that I read about concerning the unreliability of the quantum clock.

Having been analysing the situation, my analysis further compounded through having discussed these matters here at this site for last 2 years, my diagnosis is that there seems to be some confusion surrounding the interpretation of the equivalence principle.

On the one hand what I observe is that there is a school of thought that states that a caesium atomic clock placed at a higher gravity potential only 'appears' to have a higher frequency from the perspective of the lower gravity potential...
And that if one places oneself at the higher gravity potential with the clock, then the frequency of the clock will be the same as it was in the lower gravity potential, and that it will now 'appear' to you that the lower gravity potential clock has a lower frequency.

This is a direct consequence of the equivalence principle, and the concept that a caesium atom will be equivalent in each reference frame.

...and it would seem that the same school of thought is prevalent regarding SR time dilation...
This being that one's atomic clock aboard a rocket in relative motion will also be observed by oneself to be ticking 'normally', and it is the stationary rocket who's clock is observed to be running slow.
But if one were to place oneself on the stationary rocket, the stationary rocket's atomic clock would be ticking normally, and you would observe the rocket in relative motion's clock as ticking slow.

Then on the other hand - there is the school of thought that a person will age in keeping with their time dilated clock as described in the NIST link above, and the link below.

http://science.howstuffworks.com/humans-age-in-space.htm (http://science.howstuffworks.com/humans-age-in-space.htm)

Physically it just isn't possible for both concepts to reside in 'sensible' physics.

Of course there is a way of turning the equivalence principle on its head to describe why a person ages in keeping with their clock, that also retains the speed of light in each reference frame, but it requires that one take the school of thought that what one observes of the other reference frame is not just an appearance and that the other clock really is running at a differing rate.

If a clock is indeed running at different rates in differing gravity potentials, then a clock with respect to gravitational time dilation can be calibrated at sea level, and all differences mapped for an absolute reference frame in which SR effects will be infinitely more calculable.
(This being relevant to your thread)

Therefore my fascination for the standard second stems from the above.

But, far more significantly in my view, also because the speed of light, frequency, energy, and just about every action/time aspect of physics is indeed held relative to the standard second.

I really cannot understand why nobody seems to recognise the significance of this.
Title: Re: Is there a discrepancy with the equivalence principle?
Post by: Mike Gale on 16/02/2017 05:26:18
But if one were to place oneself on the stationary rocket, the stationary rocket's atomic clock would be ticking normally, and you would observe the rocket in relative motion's clock as ticking slow.
That's a common mistake with SR. Your interpretation of GR is correct. An observer at lower elevation sees the clock running faster whereas an observer at higher elevation sees the clock running slower. If you climb up or down, you will find that the clock has gained or lost time respectively. In SR, each observer perceives the other's clock to run slower. If you accelerate into the other reference frame, you will find that the clock has lost time. If the clock decelerates into your reference frame, you will find that you have lost time. It seems nonsensical, but that's the nature of the beast and you have to get your head around the concept before you delve into GR. The Twins Paradox is very instructive in that regard and Viascience (on YouTube) does a good job on it.
Title: Re: Is there a discrepancy with the equivalence principle?
Post by: timey on 16/02/2017 07:55:20
I've read at least 5 books entirely dedicated to the subject of SR and GR, and Einstein's own papers, while practically every other physics book I've read also is inclusive of the subject.
And... I am totally sick to death of the twin paradox.  What more is there to say about it?  It's not complicated to understand.

Yes it does seem nonsensical under the current remit of physics, because it is nonsensical under the remit of current physics.  And where something is nonsensical it's generally because it is wrong.

Therefore, because the observations stand as sound of time being lost under the remit of SR, and time being gained under the remit of GR at h from M, where a person with the clock will age in keeping with the clock, the equivalence principle is challenged.
One cannot state the other reference frame as equivalent by stating that the atom does not have a lower, or a higher frequency via SR and GR.

The only means available to uphold the equivalence principle under this remit, is to state 'all' atoms as having a lesser or higher frequency under SR and GR circumstance, and this gives 'physical cause' for the concept of a person ageing in keeping with their clock, and also the physical cause for the clock one is ageing in keeping with ticking 'normally'.

This can be reflected in this statement:

As the body rises it loses kinetic energy and gains potential energy. As it descends it loses pe and gains ke.

One could mention that the balance between pe and ke in this manner will ensure that the energy of the atom will remain the same in the higher reference frame as it is in the lower reference frame, and there is a consideration to support this having an effect on a clock.

This being that the moving clock has a greater ke than the stationary clock.  If a greater ke causes a slower tick rate in a clock in relative motion, then a clock in a higher gravity potential will tick faster at a 'higher frequency' with less ke.

But then one must look at what light is doing when it is approaching M and the fact of light gaining ke...
This being because lights frequency increases with the increased ke and decreased pe as it moves into the lower gravity potential, which is the opposite of what is occurring for the atom.

Leading me to my theory of an additional contra directional gravitational time dilation for open space in relation to M, and the fact that light has no rest mass, and therefore the possibility arises that light is not affected by pe.  That light might be being affected solely by this additional time dilation that my model adds.

And adding this contra directional gravitational time dilation then gives a physical cause for the acceleration of gravity, and the fact that all m in free fall accelerates at the 'same rate', in that as m or light approaches M, the increasingly shortening length of seconds of this contra directional gravitational time dilation of 'open space' will accelerate all value of mass in free fall at the same rate.

Because this added contra directional time dilation is the exact value of GR time dilation at h from M only negative, the equivalence principle is upheld, in that the speed of light will be observed as constant at 299 792 458 metres per second at any gravity potential by the observer in the reference frame, due to the fact that if the speed of light travels a metre in a slower time that is equal in value to the faster rate of time of the observer, the distance travelled by the light will be equal under the remit of both rates of time.

As shown in line 2 and 3 of this diagram:

(https://s1.postimg.org/53hr6kpl7/image.jpg) (https://postimg.org/image/53hr6kpl7/)

The m will see the light travelling by the remit of its 10% faster second, and the light will be travelling under the remit of open space's 10% slower second, but the distance travelled by the light will be the same distance.

Because the 90% light speed will take 10% longer to get there, and 'appear' to be travelling 10% longer metres per second of 'space'.

And the 110% light speed will take 10% shorter to get there, and will be appear to be travelling 10% shorter metres per the second of the m.

But according to m at h, and m's clock, the speed of light will be travelling 299 792 458 metres per its rate of second, and according to the rate of time of 'open space', the speed of light will be travelling at 299 792 458 metres per open space's second, and the equivalence principle is upheld.

Now isn't that a more interesting conversation than the twin paradox?
I'll bet you haven't heard anything like that put forward before...
Honestly Mike, with all due respect to you, but as far as I'm concerned the twin paradox conversation is about as welcome to me, as "Stairway to Heaven" is to a guitar shop floor attendant.

Im not suggesting changes to current physics because I do not understand the current physics, I'm doing it because I do.
Have you read "The Trouble with Physics" by Lee Smolin?
Title: Re: Is there a discrepancy with the equivalence principle?
Post by: GoC on 16/02/2017 11:58:25
We all interpret relativity with our own key. If you think relativity is incorrect you are using the wrong key. You cannot take what you are told in a book or by anyone here and interpret it to your understanding. Consider your in a maze and have to remember all the dead ends to reach the true understanding. You do not actually recognize the dead ends so you can continue on a path of misunderstanding. I can tell by not giving enough information about the statements and in some cases being incorrect in the statements where the confusion comes. I can tweak those statements to follow observations to follow relativity. When you understand relativity and its math completely its like looking at the maze from up top viewing the dead ends. I have to go to work but will be back and try to clear some confusion.
Title: Re: Is there a discrepancy with the equivalence principle?
Post by: alancalverd on 16/02/2017 12:16:56
"Current physics" is what actually happens, as far as we can measure it.

It happens that photon frequency shifts correlate exactly with expectation as gravitational potential energy is converted to kinetic energy.

It's fun to play Stairway to Heaven, Milestones, and the intro to Surrey with the Fringe on Top, and ask who first wrote that chord sequence.  I think it was CPE Bach.
Title: Re: Is there a discrepancy with the equivalence principle?
Post by: GoC on 16/02/2017 16:10:31
Physics has always been the same. Our current understanding of physics keep changing with new discoveries. Relativity and its math are a fixed state of relationships between mass, the photon and time.

I've read at least 5 books entirely dedicated to the subject of SR and GR, and Einstein's own papers, while practically every other physics book I've read also is inclusive of the subject.
And... I am totally sick to death of the twin paradox.  What more is there to say about it?  It's not complicated to understand.

Yes it does seem nonsensical under the current remit of physics, because it is nonsensical under the remit of current physics.  And where something is nonsensical it's generally because it is wrong.

Not in the case of relativity. It's the understanding of relativity that is incorrect.

 
Quote
Therefore, because the observations stand as sound of time being lost under the remit of SR, and time being gained under the remit of GR at h from M, where a person with the clock will age in keeping with the clock, the equivalence principle is challenged. 

Than I will challenge your understanding of relativity equivalence principle.

 
Quote
One cannot state the other reference frame as equivalent by stating that the atom does not have a lower, or a higher frequency via SR and GR.

True

 [/quote]
The only means available to uphold the equivalence principle under this remit, is to state 'all' atoms as having a lesser or higher frequency under SR and GR circumstance, and this gives 'physical cause' for the concept of a person ageing in keeping with their clock, and also the physical cause for the clock one is ageing in keeping with ticking 'normally'.  [/quote]

True.

This can be reflected in this statement:

As the body rises it loses kinetic energy and gains potential energy. As it descends it loses pe and gains ke.

Not accurate. Potential and kinetic both lose energy. He is confusing dilation with an increase in mass. While gamma expansion would normally be seen as an increase in mass by those who do not understand what gamma represents, the mass remains the same while the density of energy is reduced. This allows mass to be more attracted to mass mimicking more gravity but the reduction of pe causes attraction of gravity. g=a weight issues from the energy spectrum only and not more mass. Current physics treat gamma as an increase in mass. That is incorrect subjective thinking because they do not know how gravity attraction actually works. Dilation causes a loss of energy density. It is to expanded space the mass is attracted causing less friction to spacetime.
 

Quote
One could mention that the balance between pe and ke in this manner will ensure that the energy of the atom will remain the same in the higher reference frame as it is in the lower reference frame, and there is a consideration to support this having an effect on a clock.

Your clock is affected by the dilation of potential energy by having the photon have to go further because pe is dilated. Your clock tick rate is a measurement of the potential energy state of your current position. Your reaction rate and biological clock are matched to your tick rate yes. And we haven't gotten past GR for equivalency.

 [/quote]
This being that the moving clock has a greater ke than the stationary clock.  If a greater ke causes a slower tick rate in a clock in relative motion, then a clock in a higher gravity potential will tick faster at a 'higher frequency' with less ke. [/quote]

Ok this is the part where you have to understand the nature of equivalence in relativity. You are confusing motion with equivalence same as confusing gravity attraction with equivalence. Motion and gravity attraction are not what is necessarily equivalent. Acceleration and deceleration both cause gravity. Deceleration your clock ticks faster acceleration the clock runs slower. Motion is not what is equivalent necessarily but it can be. Lets take the Earth and say there is a tube through the center. The gravitational center your clock ticks the slowest and you are not attracted to another position. No gravity. The dilation of pe is the greatest. Your kinetic energy is also zero as stationary with no direction to go. What is equivalent is taking a spaceship and going with earths dilated state in space and traveling up to 32 ft/s/s linearly in 8,000 feet and stop accelerating instantly. The clock in the center of earth and on your ship will tick at the same energy state by tick rate. Now SR does not have physical dilation. It is the change in energy state c by vector motion. The electron cycles (this is a subjective path as a cycle term) slower because some of the electron cycle is used for vector motion for total available c. It is the reduction of energy between SR and GR that has equivalency. Frequency is controlled by the energy state for both SR and GR. Both change the angle of wave creation by their equivalence ratio.


 
Quote
But then one must look at what light is doing when it is approaching M and the fact of light gaining ke...
This being because lights frequency increases with the increased ke and decreased pe as it moves into the lower gravity potential, which is the opposite of what is occurring for the atom.
 

And there is the confusion.

 
Quote
Leading me to my theory of an additional contra directional gravitational time dilation for open space in relation to M, and the fact that light has no rest mass, and therefore the possibility arises that light is not affected by pe.  That light might be being affected solely by this additional time dilation that my model adds. 

Leading you away from relativity. It's all explained by the correct interpretation of relativity.

 
Quote
And adding this contra directional gravitational time dilation then gives a physical cause for the acceleration of gravity, and the fact that all m in free fall accelerates at the 'same rate', in that as m or light approaches M, the increasingly shortening length of seconds of this contra directional gravitational time dilation of 'open space' will accelerate all value of mass in free fall at the same rate.

Dilation and reduction in pe is the cause of gravity attraction. While free fall is indistinguishable they are not actually at the same rate. It is a ratio of the weight of the earth's dilation vs. a ball or feather in a vacuum. The difference is meaningless but there is a difference.


 
Quote
Because this added contra directional time dilation is the exact value of GR time dilation at h from M only negative, the equivalence principle is upheld, in that the speed of light will be observed as constant at 299 792 458 metres per second at any gravity potential by the observer in the reference frame, due to the fact that if the speed of light travels a metre in a slower time that is equal in value to the faster rate of time of the observer, the distance travelled by the light will be equal under the remit of both rates of time.

The speed of light is measured to be the same in every frame. In GR dilation the physical measuring stick increases without mass being added and energy being lost per volume. In SR speed energy ratio between cycle and total energy for vector motion c ratio which light has to travel further due to motion causes a visual lengthening of your measuring stick and an actual lengthening of light in a light clock. Mechanical and light clocks both tick the same rate in a frame proving equivalence between GR and SR.

 
Quote
As shown in line 2 and 3 of this diagram:

(https://s1.postimg.org/53hr6kpl7/image.jpg) (https://postimg.org/image/53hr6kpl7/)

The m will see the light travelling by the remit of its 10% faster second, and the light will be travelling under the remit of open space's 10% slower second, but the distance travelled by the light will be the same distance.

Because the 90% light speed will take 10% longer to get there, and 'appear' to be travelling 10% longer metres per second of 'space'.

And the 110% light speed will take 10% shorter to get there, and will be appear to be travelling 10% shorter metres per the second of the m.

But according to m at h, and m's clock, the speed of light will be travelling 299 792 458 metres per its rate of second, and according to the rate of time of 'open space', the speed of light will be travelling at 299 792 458 metres per open space's second, and the equivalence principle is upheld.

Now isn't that a more interesting conversation than the twin paradox?
I'll bet you haven't heard anything like that put forward before...
Honestly Mike, with all due respect to you, but as far as I'm concerned the twin paradox conversation is about as welcome to me, as "Stairway to Heaven" is to a guitar shop floor attendant.

Im not suggesting changes to current physics because I do not understand the current physics, I'm doing it because I do.
Have you read "The Trouble with Physics" by Lee Smolin?

There is no trouble with physics. It's the understanding of physics that prove difficult for some.
Title: Re: Is there a discrepancy with the equivalence principle?
Post by: timey on 16/02/2017 16:37:17
"Current physics" is what actually happens, as far as we can measure it.

It happens that photon frequency shifts correlate exactly with expectation as gravitational potential energy is converted to kinetic energy.

It's fun to play Stairway to Heaven, Milestones, and the intro to Surrey with the Fringe on Top, and ask who first wrote that chord sequence.  I think it was CPE Bach.

Yes - and any different way of interpreting these measurements, will 'have' to use the same measurements, otherwise the new interpretation won't be proportional to that which we observe, and therefore will be wrong.

If a new interpretation of the measurements doesn't require inventing Dark Energy and Dark Matter in order to render the interpretation of observed measurements as valid, and gives physical cause for measurements that are at present physically unaccounted for, then the new interpretation, as an experimental venture, is 'worthy' of calculating.

You say:  "...as far as we can measure"

I say: "...you can't put a clock into 'open space' and measure what time is doing in 'open space', because by placing any'thing' into an 'open space' that space is no longer open, but has some'thing' in it!

By Bach aye!  It was in playing Stairway to Heaven at a guitar shop (Andy's) that I found out about guitar shop attendant aversion!

(GoC - don't bother!  I actually think your interpretation of relativity to be 'pants'.  But I would never in a million years get on your threads and talk to you like you talk to me.  As far as I am concerned you are welcome to your ideas, and your right to express them...
And... I am not saying relativity is wrong, I am saying it is incomplete, which is exactly how Einstein himself described his own theory!  If Einstein could accept this as fact then why can't you?)
Title: Re: Is there a discrepancy with the equivalence principle?
Post by: GoC on 16/02/2017 17:33:58
I believe it to be complete. It describes energy and mass perfectly in ratio with observation and math. I am sorry for your emotions and I believe you are further along then most. Just thinking for yourself wins that prize. Einstein was far from perfect but relativity by postulates has never missed an observation.

I am only giving you relativity issues in favor of relativity. If you can show that relativity is incorrect I sure would be interested. Your ideas move results out of the GR and SR equivalency. All I am doing is showing equivalency is correct and observed. v0 for dilation can never be measured to be v0. Different tick rates prove different distances in GR and dilation of length.

Your conjecture is not strong enough. I can only help you make it stronger by challenging it. You need to think your way around my arguments. My arguments are relativity is correct and complete. If you cannot chip away at me how well are you going to do against the truly intelligent. I am only of average intelligence. We only grow through adversity. I find you to be somewhat interesting and I am trying to help you grow. Lose the emotion. You might be more intelligent than I am and end up being one of the greats. Grow or stagnate its always a choice.
Title: Re: Is there a discrepancy with the equivalence principle?
Post by: timey on 16/02/2017 18:06:35
Relativity is not complete because Dark Energy and Dark Matter are 'not' physically observed.  Only the necessity for their observation is observed.

Different tick rates prove different distances in GR and dilation of length.

Only because these different tick rates are being measured via the speed of light being held relative to a standard second.

Hold the speed of light relative to the tick rate in that reference frame, and the dilation, or contraction, of length does not occur, and the metre can remain constant.
Title: Re: Is there a discrepancy with the equivalence principle?
Post by: GoC on 16/02/2017 18:21:11
Constant to what? There is no standard frame of reference. Relativity keeps everything referenced to c as a constant. If you say the speed of light changes in every frame ratio's no longer follow observations.
Title: Re: Is there a discrepancy with the equivalence principle?
Post by: timey on 16/02/2017 18:29:52
If there is no frame of reference with which to hold anything relative to, then the fact that the speed of light is held constant to the time period of a standard second is challenged.

All physics measurements of time in relation to action are held relative to a standard second.

Simply calculate the speed of light via the tick rate of the differing reference frame and the metre remains constant.
Title: Re: Is there a discrepancy with the equivalence principle?
Post by: Mike Gale on 17/02/2017 01:16:47
Wow! That was a flurry of ideas. I see that you (timey) are a non-believer when it comes to the Twin's Paradox. You're not alone in that belief. The crux of the dissent is that the moving clock slows down during the acceleration phase, drifts during the cruising phase and speeds up during the declaration phase. SR says the same is true from the other perspective, but it's a bit paradoxical because only one of the clocks actually feels the force of acceleration or deceleration. There's no good answer to that, but you can make a good case for the SR interpretation, as Viascience does, and it is compelling that other consequences of SR such as relativistic mass are consistent with observation. The nay-sayers have all their work ahead of them.
Title: Re: Is there a discrepancy with the equivalence principle?
Post by: timey on 17/02/2017 01:44:42
Mike (chuckle)... I have my sights set a great deal higher, and in a much wider picture than you seem to be appreciating.

I am not a non believer in the twin paradox, and I don't have a problem with SR.

What I do think is that there is a missing component to physics, I think I have found it, and when GR and SR are superimposed into the fact of this missing component - this component being a phenomenon that is already observed but currently has no 'physical cause' - that a fully described cyclic universe emerges, and a theory of everything that unites the standard model with gravity for a continuum in quantum.

I am not a naysayer.  What I am is indeed a yaysayer to everything concerning current physics except Dark Energy, Dark Matter, and the notion of expansion, where all aspects apart from these are remixed into another arrangement in relation to the added component.

I am talking GR as Einstein wrote it, inclusive of the retraction of his cosmological constant (that ensured a steady state), minus the concept of the Hubble effect, (ie: a currently contracting universe), while turning the equivalence principle upside down.

My model is basically just current physics turned back to front, inside out, and upside down.

I lifted this from somewhere else I'm chatting:

what Einstein said (http://einsteinpapers.press.princeton.edu/vol7-trans/156?highlightText=%22speed%20of%20light%22): "the curvature of light rays occurs only in spaces where the speed of light is spatially variable".

It can work just as equally if one says:

"The curvature of light rays occurs only in spaces where the speed of light is 'temporally' variable"

To far better and more sensible results!

Title: Re: Is there a discrepancy with the equivalence principle?
Post by: Mike Gale on 17/02/2017 02:40:15
I might add that the accepted response to the "who feels the force?" argument is that feelings have nothing to do with it. The force is simply an artifact of changing reference frames. Acceleration serves to change velocity but it is velocity that dictates ones perception of time and space. Be that as it may, this only takes you further down the rabbit hole because GR deals in accelerated reference frames. It's still too early to panic though because free fall in GR is in perfect agreement with that premise. It is only when you consider reference frames that are stationary within a gravitational field that things get muddy again. My take on that is this: holding your position in a gravitational field takes additional energy, which is not accounted for in the SC metric. In other words, the SC metric tells you how things look from afar, but it doesn't tell you much about what's going on in your neighbourhood. That's why there's no GR equivalent to the Lorentz transform.
Title: Re: Is there a discrepancy with the equivalence principle?
Post by: Mike Gale on 17/02/2017 02:58:21
GR does make some predictions about your local universe though. For one thing, light can go around in circles so you can see the back of your head if you manage to reach the horizon (where SC will hold you in place with no added energy.) The problem is, you have to wait until the end of time for light to complete a round trip.
Title: Re: Is there a discrepancy with the equivalence principle?
Post by: timey on 17/02/2017 03:46:33
And how do you see the situation working with 3 time dilations?

Because if one adds a contra directional gravitational time dilation for open space, this being contra directional to GR gravitational time dilation - and also includes GR gravitational time dilation - then you can superimpose SR time dilation on top of this picture for a 3 dimensional time matrix annexed to the space time matrix, where the 3 space dimensions in relation to the 3 time dimensions result in the time aspect of the space time matrix.

With mass being directly affected by GR and SR time dilation, and indirectly affected by the contra directional time dilation of open space in that mass must move through it, while light is only affected by the contra directional time dilation, because it has no mass. (rest mass)

And there you have an alternative to using the Lorentz Transformations for the SC metric, where a metre is held constant, and it is these 3 dimensions of time that are the variables.

If you apply this remit to the universe, you will find that black holes are not at-all how current physics believes, and that a cyclic universe emerges.

You will also find that the equivalence principle will require being turned upside down as explained in post 2.
Title: Re: Is there a discrepancy with the equivalence principle?
Post by: Mike Gale on 17/02/2017 08:20:29
I think you're talking about directional time dilation, in which space is invariant and time passes at different rates in different directions of space. That's the variable light speed approach to SR, which is debunked by Michelson and Morley.
Title: Re: Is there a discrepancy with the equivalence principle?
Post by: timey on 17/02/2017 15:13:51
The Michelson Morley was conducted inline motion in a uniform gravity field rather than the gravitational gradient that I am talking about...

However, my model makes an a addition to the equivalence principle that light cannot exceed the rate of local time.  So my model would suggest that that the inline light of the Michelson Morley 'is' slowing down because it cannot travel faster than 299 792 458 metres per second of the reference frame, and subsequently the arm of the experiment does not contract.

The LIGO results are treated to the same school of thought, where the light in the tubes is blue shifted by the gravity wave, for the duration of the gravity wave, and that the light is travelling at the speed of light relative to a 'shorter' second than occurs at that reference frame's usual gravity field, without the addition of the gravity wave's gravity field...  And, we can find that the tubes of the LIGO do not contract.

This being due to the contra directional gravitational time dilation that my model adds for open space.

Point your experiment arm, or tube, upwards' into the gravitational gradient, and the light will redshift on it's outward journey, where my model states that the light is moving into reference frames with 'longer seconds, and it will blueshift on the return journey, where the seconds will become shorter on the way back down...

This is mathematically indistinguishable from applying GR gravitational time dilation to red shift/blue shift under the same circumstance, because one gravitational time dilation is the equal to the other linearly, but in opposing direction.
(ie: where the 'clock' ticks x amount faster with GR time dilation, because light is 'not' affected by the GR time dilation, the light is then only affected by the contra directional gravitational time dilation which is the equal to GR x faster time, but negative, so it is x amount slower time)

In my model the 'motion' related time dilation of SR isn't used to calculate the fact of light travelling across space.  It is only the contra directional gravitational time dilation that is used.
And all this contra directional gravitational time dilation is doing, is transferring the notion of 'spatially' variable space, into 'temporally' variable space, where light speed calculated relative to the 'longer' seconds of these temporally variable reference frames, remains a speed of 299 792 458 constant metres, per variable seconds.

Mass is what defines the rate of this contra directional gravitational time dilation of open space, and the contra directional gravitational time dilation defines how Mass moves through open space.
Title: Re: Is there a discrepancy with the equivalence principle?
Post by: Mike Gale on 18/02/2017 01:39:08
I'm having trouble following your arguments because you use unconventional terms like contra-directional dilation without explanation. Dilation is not directional in SR so I presume you're talking about GR, in which case you are suggesting that time speeds up as you approach a black hole. That would fly in the face of gravitational lensing, even if space is invariant. Can you clarify?
Title: Re: Is there a discrepancy with the equivalence principle?
Post by: timey on 18/02/2017 02:54:56
The reason why the terminology 'contra directional gravitational time dilation' is unfamiliar to you is because I am the person responsible for coining this terminology, and the notion of it's existence as a third phenomenon of time dilation that now gives us 'physical cause' for the acceleration of gravity.

Yes, yes, yes, it does mean that time speeds up when you approach a black hole!
(But hark on the fact that you being a piece of mass yourself, that you would experience your time as being faster than that of the black hole itself, ie: GR gravitational time dilation, and as you are accelerated towards the black hole by the increasingly faster rates of time of the contra directional gravitational time dilation, your own clock will be becoming increasingly slower in its rate of time.  On Earth, the time dilations would converge at ground, sea level, to then carry on down into the ground, where your clock would run slower still, and the contra directional time dilation gets faster still...*Not at-all sure about a black hole having a ground level though, ;) ...)

*'you', being a body of mass, would also be experiencing SR time dilation effects due to the acceleration.

Oh, I'm so pleased...  This shows you have given the matter some thought.  Yay!

Gravitational lensing - yes I can clarify:

Remembering that the contra directional time dilation is the equal in value to GR gravitational time dilation at h from M, only negative, and is the mechanism that accelerates mass, or light towards a body of mass, or decelerates mass or light away from a body of mass...

When light approaches mass it blue shifts into faster time...
If the light source is directly behind a big mass located in-between observation point and light source, the light that is travelling past this mass on its way to the observation point, will bend towards the mass because of the faster time around this mass.  From the observation point this will appear like a brighter halo of light, partly due to the bending, and partly due to the blue shifting.

If that sounds exactly like the current explanation, it's because it is exactly like the current explanation.
It matters not if the time near the mass, or the mass cluster, the light is bending around is running fast or slow, so long as the value of the time dilations are equal in value, (positive and negative), the bend and the blueshift remain as observed by calculation using either.
Title: Re: Is there a discrepancy with the equivalence principle?
Post by: Mike Gale on 18/02/2017 16:19:11
I still don't understand why you think a black hole might be white, but if that was indeed the case then a gravitational lens would be convex rather than concave and we would see a starkly different picture in the Eddington and Hubble (telescope) images. How do you account for that?
Title: Re: Is there a discrepancy with the equivalence principle?
Post by: timey on 18/02/2017 18:29:52
Nope - not a white hole.  The black hole remains a black hole.

Reversing the concept of time running faster in 'open space' than it does for bodies of mass, does not reverse the parameters for the black hole itself.  All it does is give 'physical cause' for the observed acceleration of gravity caused by the mass of the black hole.

Yes - this does mean that a black hole is full of energy, but it remains black because light cannot shine there.

What I am suggesting is that a black hole is inherent with the same properties as are attributed to the 'current physics' Big Bang, pre inflation.  That all matter in the black hole is so compressed, and hot, that the conditions there are a plasma, are opaque, and that light cannot 'shine'.

This is relevant to my model, in that my model's contraction will eventually result in all matter being contained in a singular black hole, that without any gravitational counterpart to cause a stabilisation, it will explode all of its content via its super luminal jets, and spew particles across space to create a sea, that is formed of these particles, which then start forming into clumps during this next cycle of a period of slow contraction.

I am suggesting that the black holes observed today are miniature representations of the Big Bang phenomenon, that will not explode and empty 'all' of their content, as in the Big Bang 'proper', but are partially displaying the super luminal jets of a 'proper' Big Bang when consuming big masses.  Only partially, because these black holes still have gravitational counterparts in the universe holding them stable, and when consuming a big mass, such as a star, they become partially unstable, causing them to partially spew particles via super luminal jets due to being rendered partially unstable by the sudden change in gravitational field as the star is consumed.

The event horizon of a black hole is simply the point at which the plasma begins and light can no longer shine...

I don't really understand why you would think that the gravitational lensing would be resulting as convex instead of concave.
At present it is the gravitational attraction/acceleration of the mass the light is bending around that causes the phenomenon.  All I've done is give the accelerative force a physical cause...  Why would it change the observation?
Title: Re: Is there a discrepancy with the equivalence principle?
Post by: Mike Gale on 18/02/2017 21:10:55
You're missing my point. The black hole is white if time speeds up on approach. What goes on inside is irrelevant.
The difference between a concave lens and a convex lens is that one concentrates light rays onto a focal point and the other diverges them away from a focal point. If a black hole was equivalent to a convex lens, objects in the background would appear to be farther away from the gravitating mass rather than closer to it. That would be contrary to Eddington's observations and images from the Hubble telescope.
Title: Re: Is there a discrepancy with the equivalence principle?
Post by: timey on 18/02/2017 22:04:44
No - the black hole would only be a white hole if you reversed its time under current physics remit, where space is 'spatially' variable.

http://www.iflscience.com/physics/what-white-hole/

It's unclear to me quite what is meant by 'a time reversal of a black hole'. Do they mean gravitational variable time, or are they talking about reversing sequential time?

In either case, this is not what I am suggesting at-all.  I'm suggesting that these 'spatially' variable spaces in space are 'temporally' variable instead.

With Eddington, and gravitational lensing, what we are observing is light being bent towards the gravitational mass as it passes this mass on its way to us.

Light travels in straight lines, not curves, unless it is curved due to space being curved by the gravitation of a mass.  When the light source is behind the in-between mass in relation to the observation point, then the halo of light is basically just a 'kink' in the straight line that light is taking.  Looking at a kink in a straight line of light who's trajectory is head on to observation, will result in a band of light, or a halo, as the gravitation of the mass pulls the light inwards.

Stating that the gravitational force pulling that light inwards is 'contra directional gravitational time dilation' related does not change the observation.
Title: Re: Is there a discrepancy with the equivalence principle?
Post by: timey on 19/02/2017 00:44:11
Good grief - it's like the Marie Celeste on this site at mo...

Anyway - if someone were here, then they might ask me:

"If black holes are so plasma hot that conditions are opaque and light can't shine, then how come we don't observe the relevant heat signature?"
Title: Re: Is there a discrepancy with the equivalence principle?
Post by: Mike Gale on 19/02/2017 02:25:34
That guy on IFL is talking through his hat. GR does not predict white holes. It just doesn't disprove them. They are certainly not the time reversal of black holes. That's the problem with Internet articles like that. No peer review. I expect he'll get a dressing down from his editor if anyone of influence gets wind of his ramblings.
A white hole is simply repulsive gravity. It pushes instead of pulling. Although it is the basis for Big Bang theory, it is entirely speculative. There is circumstantial evidence of the Big Bang, but I would place both theories in that category. They're akin to the celestial teapot because you can't prove that they don't exist.
Title: Re: Is there a discrepancy with the equivalence principle?
Post by: Mike Gale on 19/02/2017 02:34:22
As for light traveling in straight lines, that's a bit of a misnomer. A light ray is an abstraction of a light wave. It describes the path of constant phase. It's tempting to think of a photon as a particle that travels through space, but that's not quite right. It is the QM wave function that travels through space. The particle doesn't actually exist until it is detected. In the context of GR, you will be better served to think of light as an extended wave or a whole bunch of particles surfing on a wave.
Feynman's book QED is an approachable account of this concept. The proof of the pudding is that, although the majority of photons obey the laws of optics, the range of reflection angles that are actually observed cannot be explained in terms of surface defects or imperfect collimation.
Title: Re: Is there a discrepancy with the equivalence principle?
Post by: timey on 19/02/2017 02:57:53
Well thanks for clearing that up!
And yes - there is a lot of miss-information on the net, which is why I rather read books by respected and qualified physicists.
In any case white holes have bog all to do with my model, nor repulsive gravity.

Btw, I am still reading your thread, however not being mathematically proficient, any input from me would be entirely redundant, but I'm still reading and waiting to hear what any person who is mathematically proficient says... given that anyone logs in!   It's dead as a door nail round here at the mo.  I've never seen it so quiet!

I'm pretty well read on quantum as well, although to say so my understanding of in depth particle physics is lacking.  My model does apply itself in that region where the added contra directional gravitational time dilation applied to quantum results in a continuum that unites the standard model with gravity...
But there's little point in talking about this until the cosmological aspects are understood.

Wave, photon, or both, the trajectory of light does follow the curvature of space though doesn't it?
Title: Re: Is there a discrepancy with the equivalence principle?
Post by: timey on 19/02/2017 02:59:39
Just saw your edit

QED - I've read it
Title: Re: Is there a discrepancy with the equivalence principle?
Post by: Mike Gale on 19/02/2017 03:00:58
Yes. Light rays follow geodesics.
Title: Re: Is there a discrepancy with the equivalence principle?
Post by: timey on 19/02/2017 03:06:43
So an observation of a 'head on' (for want of better terminology) trajectory of light that has a 'kink' in it caused by curvature of space surrounding a mass located between light source and observation point will result in a band, or halo of light?
Title: Re: Is there a discrepancy with the equivalence principle?
Post by: Mike Gale on 19/02/2017 06:41:19
Kink is not the right word. Space bends, but it doesn't fold. And there is no halo effect. Light rays do not disperse as they travel through curvy space. If I am sufficiently far removed to perceive the curvature of space in your neighbourhood, we will simply disagree about the location of the emitter when a light ray strikes your eye. You will use the angle of incidence to project the light ray backwards in a straight line. I can see that your perception is in error, but if we both walk towards where we think the emitter is located, we will end up at the same place.
That's not to say that gravitational lensing is innocuous. It creates some fantastic optical illusions. The image of space beyond the black hole is distorted and objects can appear to be duplicated on either side. Viascience gives some excellent examples of this.
Title: Re: Is there a discrepancy with the equivalence principle?
Post by: timey on 19/02/2017 07:43:46
Ok, kink is not the correct word.

To explain:

Without the mass in-between, what we are observing is the portion of the stars 360 degree in all directions light shine that arrives at our location, which is just a tiny, minuscule portion of this stars light.

To create something to visualise, I now draw a straight line from us to the star. (I appreciate this is a vast over simplification, of course there are other geodesic considerations going on, but to simplify).
Now we will circle this initial line with 9 more lines between us and the star.
Looking 'head on' at these lines arriving at us, we can now think of our end of these lines being points, where we cannot see the lines themselves, only the ends of the lines.

Now looking at the light source as being behind the mass that is in-between, with the mass covering the middle point, and our surrounding 9 points being visible.

These points, when represented as lines will be bent towards this mass.
Now time dilation comes into play.  Calculate this time dilation as slower time or faster time for same observation.

As slower time near mass:
The light is slowing near the mass, with faster light arriving more quickly than light is leaving, therefore as the light is bent towards the mass by the unknown processes of gravitational force, there is also more of it there as it is arriving faster than it leaves.  The light will also blueshift near the mass and have more energy.
Represented as a line, the line is bent inwards towards the mass and carries on out of the gravitational field of the mass towards us.
As a point, the point will become stretched a little.  Lots and lots of points being bent inwards will create a band of light, or halo.

As faster time:
The light is speeding up as it approaches the mass and is bent inwards by the process of time getting faster near mass, but slows down when it has passed, creating a build up of the faster moving light that is now slowing when leaving the gravitational field.  The light is blue shifted and has more energy.
Represented as a line, again the light is bent inwards and then carries on towards us.
As a point, again the point will become stretched a little.  Lots and lots of points being bent inwards will create a band of light, or halo.

In both instances, what is occurring is that when the light is bent inwards towards the mass in-between, light that would otherwise be passing by our own location on other trajectories, unseen by us, is directed towards us by this bending inwards, resulting in us seeing more of the light.

I'll have a watch of that Viascience to see what he is saying, but to say so, if the time dilation surrounding the in-between mass is of faster time, rather than slower, I can easily visualise why duplicate images would be apparent behind a black hole...
Title: Re: Is there a discrepancy with the equivalence principle?
Post by: Mike Gale on 20/02/2017 01:56:17
Maybe I misunderstood your concept of a halo. An extended bright object directly behind a black hold does indeed appear as a halo around the black hole. It's called an Einstein ring. An off-axis object appears as an arc, which may be duplicated on the opposite side. This can be interpreted in terms of time dilation, but you can't have it both ways. Light rays are bent towards the black hole if time slows down towards the centre, in which case the background image is stretched outwards. They are deflected away from the black hold if time speeds up towards the centre, in which case the background image is squashed inwards.
Title: Re: Is there a discrepancy with the equivalence principle?
Post by: timey on 20/02/2017 02:16:51
I'm sorry, but as far as I have read, the reason why light is pulled inwards towards a gravitational mass is due to gravity.
The bigger the mass, the greater the bend.

...and this bend will be inwards towards the mass.

By stating gravity as being for the greater part due to 'contra directional gravitational time dilation', all I'm doing is giving a physical cause for the phenomenon of gravitational acceleration.
Title: Re: Is there a discrepancy with the equivalence principle?
Post by: timey on 20/02/2017 02:30:44
P.S.  By stating time as running faster closer to mass than it does for open space, this will not cause light to bend outwards.  I just can't see how you are arriving at this conclusion...
Title: Re: Is there a discrepancy with the equivalence principle?
Post by: GoC on 20/02/2017 03:14:20
Maybe I misunderstood your concept of a halo. An extended bright object directly behind a black hold does indeed appear as a halo around the black hole. It's called an Einstein ring. An off-axis object appears as an arc, which may be duplicated on the opposite side. This can be interpreted in terms of time dilation, but you can't have it both ways. Light rays are bent towards the black hole if time slows down towards the centre, in which case the background image is stretched outwards. They are deflected away from the black hold if time speeds up towards the centre, in which case the background image is squashed inwards.

Light bends away from a BH. There is no energy in a BH from the surface to the center. There is no gravitational dilation within a BH. There is no motion in a BH. Relativity does not work within a BH. Time measurements are severely slowed near a BH as dilation is at its greatest. Light can not reach a BH because light is energy while a BH retains no time energy. It just becomes a mass sucker with no internal motion. Images bend completely around a BH causing the Einstein ring which is the threshold between the BH and fundamental energy.
Title: Re: Is there a discrepancy with the equivalence principle?
Post by: timey on 20/02/2017 03:46:13
GoC - That is blatantly wrong!  Black holes are gravitational masses.  Please stop talking shite on my threads!

Quote
When astronomers refer to lensing, they are talking about an effect called gravitational lensing. Normal lenses such as the ones in a magnifying glass or a pair of spectacles work by bending light rays that pass through them in a process known as refraction, in order to focus the light somewhere (such as in your eye).

Light will not be focused towards one's eye if it is being bent outwards, end of story.

All the info you need is on the net. What's the matter with you?

https://en.m.wikipedia.org/wiki/Einstein_ring
Title: Re: Is there a discrepancy with the equivalence principle?
Post by: GoC on 20/02/2017 15:16:23
GoC - That is blatantly wrong!  Black holes are gravitational masses.  Please stop talking shite on my threads


Light will not be focused towards one's eye if it is being bent outwards, end of story.


The sun sets behind the horizon And we still view the sun above the horizon. Light is bent towards your position of view in this case. I am just correcting you to follow observations in reality. Please look it up for yourself. Einstein proved light is bent around massive objects in space to view the same star in two different apparent positions in space. There is enough wrong with the internet not to add more.
Title: Re: Is there a discrepancy with the equivalence principle?
Post by: timey on 20/02/2017 15:50:27
And... all of your post above is due to light being bent 'inwards' 'towards' the gravitational mass.

Light bends away from a BH.

No it doesn't!

A black hole that is gravitationally lensing a light source located at a distance directly behind, will be bending the light inwards to create the effect of an Einstein ring, as is quite clearly remarked upon in the link I provided.

Are you telling me Wikapedia is miss-informed?
Title: Re: Is there a discrepancy with the equivalence principle?
Post by: Mike Gale on 20/02/2017 17:37:14
Maybe I misunderstood your concept of a halo. An extended bright object directly behind a black hold does indeed appear as a halo around the black hole. It's called an Einstein ring. An off-axis object appears as an arc, which may be duplicated on the opposite side. This can be interpreted in terms of time dilation, but you can't have it both ways. Light rays are bent towards the black hole if time slows down towards the centre, in which case the background image is stretched outwards. They are deflected away from the black hold if time speeds up towards the centre, in which case the background image is squashed inwards.

Light bends away from a BH. There is no energy in a BH from the surface to the center. There is no gravitational dilation within a BH. There is no motion in a BH. Relativity does not work within a BH. Time measurements are severely slowed near a BH as dilation is at its greatest. Light can not reach a BH because light is energy while a BH retains no time energy. It just becomes a mass sucker with no internal motion. Images bend completely around a BH causing the Einstein ring which is the threshold between the BH and fundamental energy.
You are speculating when you talk about what goes on inside a black hole because the SC metric is silent in that domain. It predicts nonsense, as you pointed out when you said relativity doesn't work in that domain. Images are not bent "completely" around a black hole. The light ray that goes directly towards the centre is not diverted at all for example. It simply stalls at the horizon at the end of time. Otherwise you are correct. Conventional wisdom says time slows down on approach and that is indeed consistent with observation. Timey contends that the reverse may be true and she is vehemently unconvinced that that is contrary to observation. I thought my analogy to a conventional lens might clear things up, but I probably just confused things even more because a gravitational lens does not focus light like a regular lens does. In deference to Einstein's adage that failure to explain physical concepts like this in everyday language demonstrates a lack of understanding, the onus is on us (the physicists) to convince Timey that she's wrong. To that end, I still think the root of the problem is a lack of understanding about optics, not GR. The question is, what would the background image look like if time speeds up on approach?
Title: Re: Is there a discrepancy with the equivalence principle?
Post by: timey on 20/02/2017 18:23:33
Of course a gravitational lens does not work like a magnifying glass lens, as the big mass is not a concave piece of glass!

However, there is a reason why it is called a lens, in that gravitational lensing does bend light via refraction, that otherwise would not arrive in ones eye, so that it does arrive in ones eye.

Now listen up!

****I am quite 'clear' that conventional physics states that a black hole is running on slow or stopped time.****

This model that I am putting forward, is an experimental model that is looking at the remit of time running slower in open space than it does for mass.
This being on the basis that GR gravitational time dilation is observed as time running faster at h from M, and that a clock in space cannot measure what time is doing in open space, because once you put a clock
In open space to measure time in open space, the space with the clock in it is no longer open.

Therefore open space cannot be measured by a clock.

I am neither right nor wrong in experimenting with the notion of time running slower for open space than it does for mass.  What I am is ***experimental***
May I make a reminder that this is the board for "New Theories".  There is no need to convince me that I am wrong.  I am fully aware that my ideas are NOT current physics.

Now that we have cleared this up, please let us get back to that which we were talking about...

By stating time as running slower for open space than it does for mass, it will be observed that seconds get shorter and shorter the closer to mass one gets, and this will cause an acceleration.
Therefore - my model attributes this notion to the 'observed' acceleration of gravitational attraction.

As current physics attributes gravitational lensing as light being pulled inwards towards a mass by gravity, stating that time is running faster for mass than it does in open space will NOT change the observation of gravitational lensing.

However one makes description of the physical cause for the observation of the phenomenon of gravitational acceleration, it will not change the observation of the effect of gravitational acceleration.

Therefore there is a physical possibility that time running slower in space and faster for mass could exist.

If it does, then this concept leads us to a fully described cyclic universe and a theory of everything that unites gravity with the standard model for a continuum in quantum.
Which is why I am looking for a competent mathematician to calculate this experimental model - ok?
Title: Re: Is there a discrepancy with the equivalence principle?
Post by: Mike Gale on 20/02/2017 18:32:11
Hmmm. I think you made my point, Riddle me this then. What happens to that on-axis light ray in your model? It certainly doesn't stall at the horizon. It would impact the black hole at an astronomical speed.
Title: Re: Is there a discrepancy with the equivalence principle?
Post by: timey on 20/02/2017 18:43:45
No it won't.  This being because both GR and SR time dilations must also be brought to bear.

And my model is inclusive of a 3 way time matrix annexed to the space time matrix which I have explained in the thread "My model of a cyclic universe continued'.


Edit:  An analogy of this time matrix using some simplified analogous mathematics can be found on that thread in post 33, and 34
Title: Re: Is there a discrepancy with the equivalence principle?
Post by: Mike Gale on 20/02/2017 18:48:38
Okay, I was ignoring spatial dilation because I understood that (or rather the lack of that) to be part of your model. If you take that into account, the light ray would impact the black hole at light speed in a finite amount of time.
Title: Re: Is there a discrepancy with the equivalence principle?
Post by: timey on 20/02/2017 18:52:22
Nope - there is no 'spatial' dilation in my model, only 'temporal'.

See edit above, and post 33 and 34 of my model of a cyclic universe continued.
Title: Re: Is there a discrepancy with the equivalence principle?
Post by: Mike Gale on 20/02/2017 18:56:20
That's my point. Without spatial dilation, the light ray exceeds light speed.
Title: Re: Is there a discrepancy with the equivalence principle?
Post by: timey on 20/02/2017 19:02:29
Not if the speed of light is travelling at 299 792 458 meters per second of that reference frame.

The equivalence principle is upheld.
Title: Re: Is there a discrepancy with the equivalence principle?
Post by: Mike Gale on 20/02/2017 19:06:48
Speed is distance over time. If one varies and the other does not then speed must change.
Title: Re: Is there a discrepancy with the equivalence principle?
Post by: timey on 20/02/2017 19:10:21
If you travel one metre at a constant speed that is held relative to a longer or shorter 'variable' second.  Then the distance remains the same, and it just takes a longer or shorter amount of 'time' to travel that metre.
Title: Re: Is there a discrepancy with the equivalence principle?
Post by: Mike Gale on 20/02/2017 19:18:15
Exactly. The light ray therefore impacts the black hole at light speed in a finite amount of time. There's nothing to stop it from penetrating the horizon and it exceeds light speed on the other side. In the other direction, it stalls at a certain distance from the horizon. That makes absolutely no sense because time does not stall in free space.
Title: Re: Is there a discrepancy with the equivalence principle?
Post by: timey on 20/02/2017 19:21:59
That's correct.  Now you can measure inside the black holes reference frame where the speed of light 'would be' 299 792 458 metres per second of BH reference frame, which would be dependent on BH mass size, and time in open space will only stall at 0 gravity
Title: Re: Is there a discrepancy with the equivalence principle?
Post by: Mike Gale on 20/02/2017 19:23:20
BTW, the mathematical formulation you seek is:
ds^2 = A*(cdt)^2 - dr^2 - r^2*d(angle)^2
where A is positive and a function of r.
I might add that there is no way to reconcile this equality with SR or GR.
Title: Re: Is there a discrepancy with the equivalence principle?
Post by: timey on 20/02/2017 19:28:05
Hey thanks!  Although it will take me quite some hours to crawl at my mathematical pace to understand that.

Also with regards to last post of mine:
I say 'would be', because actually light will not be able to shine in a black hole of this description.  It will be plasma super hot, opaque, and light cannot shine.
Title: Re: Is there a discrepancy with the equivalence principle?
Post by: timey on 20/02/2017 19:30:29
BTW, the mathematical formulation you seek is:
ds^2 = A*(cdt)^2 - dr^2 - r^2*d(angle)^2
where A is positive and a function of r.
I might add that there is no way to reconcile this equality with SR or GR.

Yes there is, (or there should be) because both are super imposed into the CDG time dilation for 'mass only'!

Edit: minus the SR spatial considerations, which are then re-interpreted and shuffled over to describe why we don't register massive heat signature for BH's.
Title: Re: Is there a discrepancy with the equivalence principle?
Post by: Mike Gale on 20/02/2017 20:28:07
Right you are. I stand corrected. That formula is in fact the one I'm advocating on my "New Theories" thread (https://www.thenakedscientists.com/forum/index.php?topic=69764.0 (https://www.thenakedscientists.com/forum/index.php?topic=69764.0)). The difference between your model and mine is whether A increases or decreases on approach.
The reason why people balk at this idea is that it seems to neglect spatial dilation. However, that is only true for a suspended reference frame (i.e. one that is not in free fall.) It is perfectly consistent with SR if A=1 in free space and it is identical to the SC metric in the free fall case if A is the SC scaling factor. Any other case requires additional energy, which is not available in the metric.
Having said that, I can't imagine any argument that would justify A being proportional to r. The only way A can increase on approach is if gravity is repulsive, as in a white hole. That would mean A=(1+rs/r) rather than A=(1-rs/r). I think your model involves something like A=(1+r/rs) and that blows up in free space (where rs=0.)
Title: Re: Is there a discrepancy with the equivalence principle?
Post by: timey on 20/02/2017 21:37:48
One of the reasons why I think that you will be able to calculate my model Mike, is because of your variable speeds of light.
It's a not such a big leap in thinking to get from what you are doing, to what I am trying to do.

The only way A can increase on approach is if gravity is repulsive, as in a white hole.

You are missing the fact that in my model, A 'is' gravity, for the greater part that is.  My model splits G or g into an attraction that will have a very much reduced value, and the rest is the CDG time dilation related 'acceleration' that accelerates the attraction.

We don't need to add this CDG time dilation 'as well' as gravity, as per how current remit calculates gravity.
GR can then be added for m in relation to M, and SR for relative motion sits on top of this, with the CDG time dilation being what defines the speed of light of the reference frame m is moving through, and SR time dilation is affected because the percentage of the speed of light that the speed that m is travelling at becomes variable.

Have a read of the last four posts on 'my model of a cyclic universe continued again'.  You will see more what I am thinking then...
Title: Re: Is there a discrepancy with the equivalence principle?
Post by: Mike Gale on 21/02/2017 01:38:04
Variable light speed was not my idea. Einstein wrote about it at length (http://einsteinpapers.press.princeton.edu/vol7-trans/156?highlightText=%22spatially%20variable%22 (http://einsteinpapers.press.princeton.edu/vol7-trans/156?highlightText=%22spatially%20variable%22)). It just never caught on as a useful way of thinking about GR. Not that it's invalid, he just didn't find it helpful when explaining GR to non-believers. I expect it was hard enough to convince them that the speed of light is invariant in an SR context and chose to let that sleeping dog lie.
It's not really fair to say that the SC scaling factor is gravity, although it certainly involves the concept. Gravity is a force whereas the scaling factor represents energy. (A force is the rate of change of energy.) The scaling factor is in fact an expression of conservation of energy for the free fall case. GR theorists cringe at that definition. They contend that it is something completely different and any resemblance to classical concepts like conservation of energy is purely coincidental. I contend that they are confused because they have forgotten to account for SR effects when formulating the metric. But I digress.
Splitting gravity into two parts is a new one on me. Can you give me the executive overview or do I have to tackle four more discussion threads?
Title: Re: Is there a discrepancy with the equivalence principle?
Post by: Spring Theory on 21/02/2017 02:26:42
If you look at matter as photons in orbit, it makes the nature of space time easier to explain. Also, the bending of space by matter is really a compressing of space that leaves decompressed space at its perimeter.

Light travels slower in decompressed space (gravity wells) so the orbital cycles of matter travel slower which makes time appear to slow down.

The gravity effect is the bending of the photon path due to speed gradients in the direction of decompressed space. It does not matter how massive the "particle" (photon orbital) is, the effect is equivalent for all photon systems, hence the equivalence principle...
Title: Re: Is there a discrepancy with the equivalence principle?
Post by: timey on 21/02/2017 03:15:18
Variable light speed was not my idea. Einstein wrote about it at length (http://einsteinpapers.press.princeton.edu/vol7-trans/156?highlightText=%22spatially%20variable%22 (http://einsteinpapers.press.princeton.edu/vol7-trans/156?highlightText=%22spatially%20variable%22)). It just never caught on as a useful way of thinking about GR. Not that it's invalid, he just didn't find it helpful when explaining GR to non-believers. I expect it was hard enough to convince them that the speed of light is invariant in an SR context and chose to let that sleeping dog lie.
It's not really fair to say that the SC scaling factor is gravity, although it certainly involves the concept. Gravity is a force whereas the scaling factor represents energy. (A force is the rate of change of energy.) The scaling factor is in fact an expression of conservation of energy for the free fall case. GR theorists cringe at that definition. They contend that it is something completely different and any resemblance to classical concepts like conservation of energy is purely coincidental. I contend that they are confused because they have forgotten to account for SR effects when formulating the metric. But I digress.
Splitting gravity into two parts is a new one on me. Can you give me the executive overview or do I have to tackle four more discussion threads?

Of course I know what Einstein said.  I've read those papers.  What I meant was that it is because you can recognise Einstein's reference, and with respect that in your own interpretation of what you are doing that you appear to be open to alternative, (alternative that makes sense to me under current physics remits btw), that it will be easier for you to make the mental leap to understand and calculate these differences that I add...
And to say so it's really quite heartening to have a conversation that is progressive.

At present in physics we have this force called gravity.  We know everything about it mathematically down to the very last minute detail, apart from 'why' it does what it does...
So far there is no physical cause that can be attributed to the fact that gravity accelerates objects towards the greater mass.
So far there is no physical cause that can be attributed to the fact that m, no matter its value, will free fall to towards M at the same rate.

Also - I think it worth mentioning that physics does not have a fully coherent theory of time.  This is well documented in all of the books I've read.

Ok look - Just for the time being banish all thought's of GR time dilation and SR time dilation from your mind completely, and just think of what I'm saying in terms of attributing a physical cause for the fact that objects are accelerated towards the greater mass, and that m no matter its value, free falls at the same rate towards M.

Now place into this picture - of objects being accelerated towards the greater mass, and that m, no matter it's value, free falls towards M at the same rate - a time dilation that is inherent to the g-field surrounding M, where the seconds get progressively longer at h from M...

... Thinking upon this you will find that a physical mechanism as such would account for both the observation of objects being accelerated towards the greater mass, and the observation that m, no matter it's value, will free fall towards M at the same rate...

And after scratching your head, and rubbing your chin while you hmmm and ahh for a while, you would realise that this is 'well cool' because this means that all the maths for this concept already exist!

But then there are a few hurdles, such as that GR time dilation states clocks as ticking faster at h from M...
But...  It occurs to one, or perhaps it was dawns upon, that it is quite possible for both to be happening simultaneously, because in relation to what is occurring on M, each will be the opposite of each other equally, and just because m's time is as such doesn't negate m from being accelerated towards the greater mass.
SR time dilation* can then be added to the picture to move GR time dilated m through this time dilated related g-field.

* Not length contraction.

The remit of quite how this could work is laid out (only a few paragraphs) in posts 33 and 34 of this thread:

https://www.thenakedscientists.com/forum/index.php?topic=69592.0

Unfortunately there is no 'find post' function on this site, so if you want me to copy and paste those posts to this thread I will, but if you read the last 4 most recent posts, if you have already understood what I have said above, then these last 4 posts should suffice.
Title: Re: Is there a discrepancy with the equivalence principle?
Post by: Mike Gale on 21/02/2017 04:24:57
If you look at matter as photons in orbit, it makes the nature of space time easier to explain. Also, the bending of space by matter is really a compressing of space that leaves decompressed space at its perimeter.

Light travels slower in decompressed space (gravity wells) so the orbital cycles of matter travel slower which makes time appear to slow down.

The gravity effect is the bending of the photon path due to speed gradients in the direction of decompressed space. It does not matter how massive the "particle" (photon orbital) is, the effect is equivalent for all photon systems, hence the equivalence principle...
You seem to be advocating some kind of aether theory, but you are way off topic on this thread. Let's try to stay focused on the topic at hand, shall we?
Title: Re: Is there a discrepancy with the equivalence principle?
Post by: Spring Theory on 21/02/2017 04:51:08
If you look at matter as photons in orbit, it makes the nature of space time easier to explain. Also, the bending of space by matter is really a compressing of space that leaves decompressed space at its perimeter.

Light travels slower in decompressed space (gravity wells) so the orbital cycles of matter travel slower which makes time appear to slow down.

The gravity effect is the bending of the photon path due to speed gradients in the direction of decompressed space. It does not matter how massive the "particle" (photon orbital) is, the effect is equivalent for all photon systems, hence the equivalence principle...
You seem to be advocating some kind of aether theory, but you are way off topic on this thread. Let's try to stay focused on the topic at hand, shall we?
Equivalence principle!!????
Title: Re: Is there a discrepancy with the equivalence principle?
Post by: Mike Gale on 21/02/2017 04:52:42
I think I get the gist of what you're saying. You're suggesting that the phenomenon of time dilation is separate and distinct from gravity. I couldn't agree more. If you interpret GR in terms of variable light speed, time dilation is entirely due to velocity. Same goes for spatial dilation, but that one is a bit of a mixed bag because light speed defines the relationship between space and time. You could turn it around by making space squishy and time rigid, but SR teaches us that it's really a mixture of the two and that is the view I am advocating. Time and space are variable in SR, but light speed is constant. GR (or at least my interpretation of it) extends that concept by making light speed variable in a gravitational field.
BTW - you can link directly to a comment by copying the link above it, which reads "Re: topic".
Title: Re: Is there a discrepancy with the equivalence principle?
Post by: Mike Gale on 21/02/2017 04:56:08
If you look at matter as photons in orbit, it makes the nature of space time easier to explain. Also, the bending of space by matter is really a compressing of space that leaves decompressed space at its perimeter.

Light travels slower in decompressed space (gravity wells) so the orbital cycles of matter travel slower which makes time appear to slow down.

The gravity effect is the bending of the photon path due to speed gradients in the direction of decompressed space. It does not matter how massive the "particle" (photon orbital) is, the effect is equivalent for all photon systems, hence the equivalence principle...
You seem to be advocating some kind of aether theory, but you are way off topic on this thread. Let's try to stay focused on the topic at hand, shall we?
Equivalence principle!!????
The equivalence principle, which states that observers in free fall do not feel their own weight, is not in contention here.
Title: Re: Is there a discrepancy with the equivalence principle?
Post by: Spring Theory on 21/02/2017 04:57:59
I think I get the gist of what you're saying. You're suggesting that the phenomenon of time dilation is separate and distinct from gravity. I couldn't agree more. If you interpret GR in terms of variable light speed, time dilation is entirely due to velocity. Same goes for spatial dilation, but that one is a bit of a mixed bag because light speed defines the relationship between space and time. You could turn it around by making space squishy and time rigid, but SR teaches us that it's really a mixture of the two and that is the view I am advocating. Time and space are variable in SR, but light speed is constant. GR extends that concept by making light speed variable in a gravitational field.
BTW - you can link directly to a comment by copying the link above it, which reads "Re: topic".

Agreed to a point, but I hold to the side that time is rigid and space is squishy...
Title: Re: Is there a discrepancy with the equivalence principle?
Post by: Spring Theory on 21/02/2017 04:58:54
If you look at matter as photons in orbit, it makes the nature of space time easier to explain. Also, the bending of space by matter is really a compressing of space that leaves decompressed space at its perimeter.

Light travels slower in decompressed space (gravity wells) so the orbital cycles of matter travel slower which makes time appear to slow down.

The gravity effect is the bending of the photon path due to speed gradients in the direction of decompressed space. It does not matter how massive the "particle" (photon orbital) is, the effect is equivalent for all photon systems, hence the equivalence principle...
You seem to be advocating some kind of aether theory, but you are way off topic on this thread. Let's try to stay focused on the topic at hand, shall we?
Equivalence principle!!????
The equivalence principle, which states that observers in free fall do not feel their own weight, is not in contention here.
Topic: Is there a discrepancy with the equivalence principle?
Title: Re: Is there a discrepancy with the equivalence principle?
Post by: Mike Gale on 21/02/2017 05:00:51
I think I get the gist of what you're saying. You're suggesting that the phenomenon of time dilation is separate and distinct from gravity. I couldn't agree more. If you interpret GR in terms of variable light speed, time dilation is entirely due to velocity. Same goes for spatial dilation, but that one is a bit of a mixed bag because light speed defines the relationship between space and time. You could turn it around by making space squishy and time rigid, but SR teaches us that it's really a mixture of the two and that is the view I am advocating. Time and space are variable in SR, but light speed is constant. GR extends that concept by making light speed variable in a gravitational field.
BTW - you can link directly to a comment by copying the link above it, which reads "Re: topic".

Agreed to a point, but I hold to the side that time is rigid and space is squishy...
My comment was in regard to timey's theory. I don't know what yours is and I strongly suggest that you take it up in a new thread. Otherwise this one will become incomprehensible.
Title: Re: Is there a discrepancy with the equivalence principle?
Post by: Mike Gale on 21/02/2017 05:10:50
Reply #65 appears to be a typo. Authors are allowed to withdraw their comments so I would ask Spring Theory to do so and I will then delete this one.
Title: Re: Is there a discrepancy with the equivalence principle?
Post by: GoC on 21/02/2017 15:39:31
Variable light speed was not my idea. Einstein wrote about it at length (http://einsteinpapers.press.princeton.edu/vol7-trans/156?highlightText=%22spatially%20variable%22 (http://einsteinpapers.press.princeton.edu/vol7-trans/156?highlightText=%22spatially%20variable%22)). It just never caught on as a useful way of thinking about GR. Not that it's invalid, he just didn't find it helpful when explaining GR to non-believers. I expect it was hard enough to convince them that the speed of light is invariant in an SR context and chose to let that sleeping dog lie.
It's not really fair to say that the SC scaling factor is gravity, although it certainly involves the concept. Gravity is a force whereas the scaling factor represents energy. (A force is the rate of change of energy.) The scaling factor is in fact an expression of conservation of energy for the free fall case. GR theorists cringe at that definition. They contend that it is something completely different and any resemblance to classical concepts like conservation of energy is purely coincidental. I contend that they are confused because they have forgotten to account for SR effects when formulating the metric. But I digress.
Splitting gravity into two parts is a new one on me. Can you give me the executive overview or do I have to tackle four more discussion threads?

Of course I know what Einstein said.  I've read those papers.  What I meant was that it is because you can recognise Einstein's reference, and with respect that in your own interpretation of what you are doing that you appear to be open to alternative, (alternative that makes sense to me under current physics remits btw), that it will be easier for you to make the mental leap to understand and calculate these differences that I add...
And to say so it's really quite heartening to have a conversation that is progressive.

At present in physics we have this force called gravity.  We know everything about it mathematically down to the very last minute detail, apart from 'why' it does what it does...
So far there is no physical cause that can be attributed to the fact that gravity accelerates objects towards the greater mass.
So far there is no physical cause that can be attributed to the fact that m, no matter its value, will free fall to towards M at the same rate.

Also - I think it worth mentioning that physics does not have a fully coherent theory of time.  This is well documented in all of the books I've read.

Ok look - Just for the time being banish all thought's of GR time dilation and SR time dilation from your mind completely, and just think of what I'm saying in terms of attributing a physical cause for the fact that objects are accelerated towards the greater mass, and that m no matter its value, free falls at the same rate towards M.

Now place into this picture - of objects being accelerated towards the greater mass, and that m, no matter it's value, free falls towards M at the same rate - a time dilation that is inherent to the g-field surrounding M, where the seconds get progressively longer at h from M...

... Thinking upon this you will find that a physical mechanism as such would account for both the observation of objects being accelerated towards the greater mass, and the observation that m, no matter it's value, will free fall towards M at the same rate...

And after scratching your head, and rubbing your chin while you hmmm and ahh for a while, you would realise that this is 'well cool' because this means that all the maths for this concept already exist!

But then there are a few hurdles, such as that GR time dilation states clocks as ticking faster at h from M...
But...  It occurs to one, or perhaps it was dawns upon, that it is quite possible for both to be happening simultaneously, because in relation to what is occurring on M, each will be the opposite of each other equally, and just because m's time is as such doesn't negate m from being accelerated towards the greater mass.
SR time dilation* can then be added to the picture to move GR time dilated m through this time dilated related g-field.

* Not length contraction.

The remit of quite how this could work is laid out (only a few paragraphs) in posts 33 and 34 of this thread:

https://www.thenakedscientists.com/forum/index.php?topic=69592.0

Unfortunately there is no 'find post' function on this site, so if you want me to copy and paste those posts to this thread I will, but if you read the last 4 most recent posts, if you have already understood what I have said above, then these last 4 posts should suffice.

   I understand your reference to the cars in different lanes. That is the prelude to understanding Relativity GR. What you need to understand is your measuring stick (your meter stick) becomes physically larger and you measure distances with a larger (longer) measuring stick. As an example the distance to the sun would be say 92.9 million miles distance with a larger Earth vs. 93 million miles on the present size of our Earth. You measure a shorter distance in a slower clocks frame. This will feel counter intuitive until you work it out correctly. You need to get over this hump to recognize relativity to be correct.

SR is a visual measuring stick change and GR is a physical measuring stick change that has equivalence between SR and GR. SR being the hypotenuse for light travel and GR being dilation of space. Both have there equivalencies in being relative to c energy state.

You can understand relativity as motion, time or energy. I have found the easiest for me is through energy.  Geometry of motion for SR and energy state of dilation for GR. The clock measures dilation in GR and speed in SR. The equivalency is through the clock tick rate.

It's complicated until you understand it properly. Once that happens you wonder what was so difficult to understand. I can see both you and Mike are on the path. Relativity is the correct fork in the road. Gravity is explained by dilation of energy for the Pe state. There is an increase in dilation to the center of mass which can be followed orthogonally by the reduction of Pe to the center of mass. Mass has no fundamental energy. Electrons are moved by c. BH's have no time energy motion at all. No c period. Energy actually bends around a BH and the photon is a wave on energy. No direct access to a BH like you can with normal mass. Anyone suggesting light can reach a BH does not understand Relativity properly.

Normal mass (the atom) has a ratio of a marble to a football field where the electron travels 100 yards to a marble. A black Hole is a football field full of marbles. The fundamental energy of time and motion are excluded from a BH. The surface to the center is the same Pe=0. The fundamental energy c curves around the BH because of this. BH's are out of the relativity nature of normal mass and space.

So the equivalence principle is alive and well in normal mass.
Title: Re: Is there a discrepancy with the equivalence principle?
Post by: Spring Theory on 21/02/2017 16:34:21
Reply #65 appears to be a typo. Authors are allowed to withdraw their comments so I would ask Spring Theory to do so and I will then delete this one.
Not a typo.
Title: Re: Is there a discrepancy with the equivalence principle?
Post by: timey on 21/02/2017 18:04:37
Mike, to retain continuity I have posted my post that you answered, your answer, and my answer to your answer below...

At present in physics we have this force called gravity.  We know everything about it mathematically down to the very last minute detail, apart from 'why' it does what it does...
So far there is no physical cause that can be attributed to the fact that gravity accelerates objects towards the greater mass.
So far there is no physical cause that can be attributed to the fact that m, no matter its value, will free fall to towards M at the same rate.

Also - I think it worth mentioning that physics does not have a fully coherent theory of time.  This is well documented in all of the books I've read.

Ok look - Just for the time being banish all thought's of GR time dilation and SR time dilation from your mind completely, and just think of what I'm saying in terms of attributing a physical cause for the fact that objects are accelerated towards the greater mass, and that m no matter its value, free falls at the same rate towards M.

Now place into this picture - of objects being accelerated towards the greater mass, and that m, no matter it's value, free falls towards M at the same rate - a time dilation that is inherent to the g-field surrounding M, where the seconds get progressively longer at h from M...

... Thinking upon this you will find that a physical mechanism as such would account for both the observation of objects being accelerated towards the greater mass, and the observation that m, no matter it's value, will free fall towards M at the same rate...

And after scratching your head, and rubbing your chin while you hmmm and ahh for a while, you would realise that this is 'well cool' because this means that all the maths for this concept already exist!

But then there are a few hurdles, such as that GR time dilation states clocks as ticking faster at h from M...
But...  It occurs to one, or perhaps it was dawns upon, that it is quite possible for both to be happening simultaneously, because in relation to what is occurring on M, each will be the opposite of each other equally, and just because m's time is as such doesn't negate m from being accelerated towards the greater mass.
SR can then be added to the picture to move GR time dilated m through this time dilated related g-field.

The remit of quite how this could work is laid out (only a few paragraphs) in posts 33 and 34 of this thread:

https://www.thenakedscientists.com/forum/index.php?topic=69592.0 (https://www.thenakedscientists.com/forum/index.php?topic=69592.0)

Unfortunately there is no 'find post' function on this site, so if you want me to copy and paste those posts to this thread I will, but if you read the last 4 most recent posts, if you have already understood what I have said above, then these last 4 posts should suffice.

I think I get the gist of what you're saying. You're suggesting that the phenomenon of time dilation is separate and distinct from gravity. I couldn't agree more. If you interpret GR in terms of variable light speed, time dilation is entirely due to velocity. Same goes for spatial dilation, but that one is a bit of a mixed bag because light speed defines the relationship between space and time. You could turn it around by making space squishy and time rigid, but SR teaches us that it's really a mixture of the two and that is the view I am advocating. Time and space are variable in SR, but light speed is constant. GR (or at least my interpretation of it) extends that concept by making light speed variable in a gravitational field.
BTW - you can link directly to a comment by copying the link above it, which reads "Re: topic".

Ok - going at it one step at a time...

No - I am suggesting the exact opposite.  I am suggesting that gravity is 'almost' entirely time dilation related*.
Not GR time dilation related.
Not SR time dilation related.
But due to a time dilation that is inherent to 'open spaces' in relation to M, that 'cannot' be measured by a clock...
(*This time dilation can be thought of as kinetic energy related)

This being because when you place a clock in an open space, that space is no longer open, and what is being measured is not what time is doing in the open space g-field surrounding M, but is measuring what time does for m in relation to M.  The measuring of a clock ticking faster at h from M being GR time dilation.*
(*This can be thought of as being gravity potential related)

The above is indeed the entire premises for my model.

If you have any questions about the above, ask now...

If you understand the above, and recognise that this 'new' time dilation can give physical cause for the observed actions of gravity, we can move on to looking at the mathematical structure of SR, the fact that these maths can describe physical observation, and examine 'actual' physical cause for both SR time dilation, and its associated length contraction maths.
Title: Re: Is there a discrepancy with the equivalence principle?
Post by: Mike Gale on 21/02/2017 23:49:09
I think it's a mistake to attribute time dilation to GR so I'm with you there. My argument is that SR has that phenomenon covered for the free fall case and all other cases involve additional energy, which is not available in the GR metric. But it's certainly a mistake to disregard SR and attribute time dilation to something new. SR is a definitive account of KE. GR is all about PE.
Title: Re: Is there a discrepancy with the equivalence principle?
Post by: timey on 22/02/2017 00:13:58
My model doesn't disregard SR time dilation in the slightest, and gives physical description of why SR length contraction mathematics work in the current maths to describe observation.
Title: Re: Is there a discrepancy with the equivalence principle?
Post by: Mike Gale on 22/02/2017 03:20:09
Oh yes. I see. I read your list of nots backwards. You're conceding SR dilation and GR dilation (if there is such a thing), but you think there may be something more to gravity. I am reminded of Laplace's response to Napoleon's question about the absence of God in his physics. He is purported to have said, "I had no need of that hypothesis." Religious connotations aside, I think the same thing applies here. Perhaps I should address the question that sparked this discussion (i.e. the title of this thread.)
The answer to that question is no. There is no discrepancy. The equivalence principle is perfectly consistent with GR. It was in fact the founding principle, which inspired Einstein to seek answers. He called it the happiest thought of his life. It's disappointing that he didn't go to greater lengths to tie it into SR, but it's understandable because GR was a stunning breakthrough in and of itself. We stand in awe and despite what some egos claim, we still struggle to comprehend. I get that you seek comprehension, but I don't see the need to invent new causes for gravity. What's the motivation? The scientific method dictates that we have faith in SR and GR until they are proven wrong by observation. As the optimist said when she fell past the 20th storey, "So far, so good."
Title: Re: Is there a discrepancy with the equivalence principle?
Post by: timey on 22/02/2017 06:26:49
Well she said - that would be all very well, but last time I checked the situation, GR and SR, in order to describe, so brilliantly and adequately, these observations of our universe, (under the remit of current physics model), cannot make this description without the 'added' and as of yet 'unobserved' dimensions of Dark Matter, and Dark Energy.

... And this model that I am trying to describe, by altering the dimensions of the equivalence principle ever so slightly, just uses what we 'do' observe.

And what physical cause is given to gravity as per current model?

What exactly is it that you are calculating when you calculate using G or g?

And isn't it a direct consequence of the equivalence principle that you are unsure if SR or GR time dilation is real or not?
On the one hand it is said that these time dilations only appear to happen from the other reference frame, and on the other hand a person ages in keeping with their time dilated clock...
How does one go about physically reconciling this anomaly?
Title: Re: Is there a discrepancy with the equivalence principle?
Post by: Mike Gale on 23/02/2017 02:34:25
I see. You're trying to defeat the Dark Side, which is GR's answer to galaxy rotation curves. Good point. That would certainly qualify as a faith-shaking observation. You could have made that more clear in the topic title though. If you ask me, dark matter is a contrived solution. It points to a flaw in the physics as opposed to a new form of matter. I suspect it's an artifact of the weak field approximation, which is used to calibrate the SC metric. I think I know how to fix the metric, but there's a still lot of math to be done in order to debunk DM conclusively. It may well be that my theory is the same as yours, but it's too early to tell.
As to your probing questions:
Q1: what physical cause is given to gravity as per current model?
A1: Variable speed of light, which could be due to time dilation or spatial dilation, but not both.
Q2: what exactly is it that you are calculating when you calculate using G or g?
A2: Those constants represent coupling between mass and space. They are essentially unit conversion factors. You can actually measure distance in terms of mass or express mass in terms of distance. That's what the SC scaling distance is all about. Mass is essentially a hole in space. The same is true of the speed of light. It is essentially a unit conversion factor between space and time.
Q3: isn't it a direct consequence of the equivalence principle that you are unsure if SR or GR time dilation is real or not?
A3: I wouldn't say that we're unsure about SR or GR dilation. Given the equivalence principle, they are in perfect agreement with almost all observations of the solar system. (I say almost because precession of a planet's axis of rotation remains unexplained. That may be a lack of mathematical skill as opposed to a defect in the model though.) The model doesn't seem to hold for galaxies, but the metric is demonstrably dubious for strong fields. It therefore seems likely that the problem is in the metric as opposed to the principle itself. For now we have to stick with the DM explanation in order to compile evidence. It is an obvious cop out, but it's a workable thesis until something better comes along. It's annoying when its proponents talk like it's a done deal though. Ego hinders progress.
Q4: how does one go about physically reconciling this anomaly [with regards to reference frames]?
A4: I think SR and GR have us covered there. One's perception of the universe depends on one's velocity and the local field strength. Both of those factors are observer-dependent. The relationship between reference frames is difficult to comprehend, but the logic is infallible.
Title: Re: Is there a discrepancy with the equivalence principle?
Post by: timey on 23/02/2017 03:47:04
My model is a fully described cyclic universe that makes all of its development from particle form to what we see today in a 'contraction' period that was initiated as soon as the inflation period of my model's Big Bang* stopped inflating...

...Therefore I am challenging Hubble's velocity related interpretation of the red shift distance correlation...
... stating the acceleration of gravity as contra directional gravitational time dilation related...

And in a contracting universe where the acceleration of gravity is time dilation related the necessity for Dark Energy, and Dark Matter is negated.

*The Big Bang of my cyclic model is due to the eventual contraction of all mass in the universe into one super super massive black hole.which will explode via its super luminal jets, where all the mass and energy of this universe are inflated into a sea of particles that start clumping together in a slow and prelonged contraction that accelerates in that contraction as matter further clumps.  ie: cyclic
(I'm pretty certain that I've posted this info in this thread)

1/ variable speed of light is not a physical cause for gravity.  It can be a physical consequence of gravity, but cannot be describing a physical cause for gravity.
So again - what physical cause are you giving gravity?

2/ Those constants are mathematical numbers based on measurement and observation.
You are calculating a phenomenon of which you know what it does, but not what it is, nor what causes it.

3/  I would contend and state that the equivalence principle is not making adequate description, because according to the equivalence principle the clock in the other reference frame is ticking with the same energy and frequency as the clock in your own reference frame, and this is in direct contention to the notion that one ages in keeping with their time dilated clock.

4/ I disagree...  As per in 3/, the logic stinks!  My rendition of the equivalence principle ensures that all remains equivalent, and unifies gravity with the standard model for a continuum in quantum.
Title: Re: Is there a discrepancy with the equivalence principle?
Post by: timey on 23/02/2017 04:03:29
https://www.thenakedscientists.com/forum/index.php?topic=69830.0 (https://www.thenakedscientists.com/forum/index.php?topic=69830.0)

Did you see this thread (only 4 short posts so far)
Title: Re: Is there a discrepancy with the equivalence principle?
Post by: Mike Gale on 23/02/2017 04:19:06
Ah, yes. Hubble expansion. That's a bit of a kerfuffle. The root of that phenomenon is the white hole known as the Big Bang. It seems to suggest dark energy, but I suspect that is an artifact of the metric defect, just like DM.
You are speculating when you talk about masses combining to form bigger black holes. The SC metric does not permit that without invoking QM and that's an entirely new kettle of fish. Despite Hawking's best efforts, the relationship between GR and QM continues to elude us. I am hopeful that correcting the defect in the metric will shed some light on that subject, but I don't have anything to hang my hat on yet. Note that the inflationary period of Big Bang theory is entirely contrived, just like DM and DE. It is an observation, not a causal explanation.
1) Variable light speed is the cause of gravity. Dilation is the cause of variable light speed. Mass is the cause (or a cause) of dilation. The theory has nothing to say about the cause of mass. It just is.
2) Call them what you will. The fact remains that they are unit conversion factors. The laws of physics are unchanged if you set both of them equal to one. All that changes is your units of measure. The GR interpretation of mass is the size of a hole in space. The SR interpretation of space is distance traveled in a unit of time at light speed. Another way of thinking about that is the number of wave cycles between events.
3) Incorrect. Energy is observer-dependent.
4) Many people have tried to poke holes in Einstein's logic. All (including me) have failed.
Title: Re: Is there a discrepancy with the equivalence principle?
Post by: timey on 23/02/2017 04:59:08
Einstein himself said his logic was incomplete, and illogical.

I will watch you and your metric with interest.  And if you get a bit bored at-all while reconciling length contraction to the percentage of the coordinate speed of light your mass or light is travelling at in relation to distance travelled - just for laughs try setting time set at stopped in a 0 gravity field, and then re-calculate a prediction for the cosmological constant...

With regards to my own model, I've given it a great deal of incredibly deep thought for many years now, having read a lot of books that have told me all of the matters that need addressing in order to produce a theory of everything, of which my model addresses them all.

However - to understand my model, it would take the same type of paradigm shift in thinking that a person conditioned to think in terms of the geocentric model would have had to employ to start thinking in terms of the heliocentric model.

I'm not going to argue about relativity here, I find doing so incredibly boring.  It's all anyone ever does, back and forth, back and forth.
Many theoretical physicists are looking at the possibility of current physics being wrong, trying new ideas, new metric, MOND, MOG, etc, and if I can't find a progressive discussion that isn't going to result in the words 'incorrect because relativity says so', I'd rather put my energy into learning the maths that need to be employed to describe what I am describing in my model.
Title: Re: Is there a discrepancy with the equivalence principle?
Post by: Mike Gale on 24/02/2017 00:02:30
https://www.thenakedscientists.com/forum/index.php?topic=69830.0 (https://www.thenakedscientists.com/forum/index.php?topic=69830.0)

Did you see this thread (only 4 short posts so far)
I did not. Thanks for pointing that out. They seem to be struggling with the relativistic version of KE. Not sure what they're after.
Title: Re: Is there a discrepancy with the equivalence principle?
Post by: Mike Gale on 24/02/2017 01:17:23
Einstein himself said his logic was incomplete, and illogical.

I will watch you and your metric with interest.  And if you get a bit bored at-all while reconciling length contraction to the percentage of the coordinate speed of light your mass or light is travelling at in relation to distance travelled - just for laughs try setting time set at stopped in a 0 gravity field, and then re-calculate a prediction for the cosmological constant...

With regards to my own model, I've given it a great deal of incredibly deep thought for many years now, having read a lot of books that have told me all of the matters that need addressing in order to produce a theory of everything, of which my model addresses them all.

However - to understand my model, it would take the same type of paradigm shift in thinking that a person conditioned to think in terms of the geocentric model would have had to employ to start thinking in terms of the heliocentric model.

I'm not going to argue about relativity here, I find doing so incredibly boring.  It's all anyone ever does, back and forth, back and forth.
Many theoretical physicists are looking at the possibility of current physics being wrong, trying new ideas, new metric, MOND, MOG, etc, and if I can't find a progressive discussion that isn't going to result in the words 'incorrect because relativity says so', I'd rather put my energy into learning the maths that need to be employed to describe what I am describing in my model.
I've never read anything that suggests Einstein was in doubt. He was certainly aware of purported paradoxes like the tale of two twins and Bell's spaceship, but those have since been solved as far as I know.
I don't know what MOG is, but I've read about MOND. It's is an interesting twist on gravity that involves variable G. I haven't looked at it since I noticed that the SC metric permits a variable light speed interpretation. It may be the same theory as mine, but it's hard to tell because they confuse the bejeebers out of G.
It is not relativity that makes you incorrect about energy, or at least it's not SR or GR. Energy is observer dependent in Galilean relativity, too. Viascience gives a good example involving a ping-pong ball and a bat. From the ball's perspective, the bat is moving. From the bat's perspective, the ball is moving. The energy associated with each depends on your point of view.
Title: Re: Is there a discrepancy with the equivalence principle?
Post by: timey on 24/02/2017 01:26:53
Yes - but both bat and ball are moving with respect to space - and who cares about the bat or the ball's point of view?

If there are no spatially variable spaces, then one can know where both the bat and the ball are with respect to space, and not bother with what the bat or the ball is perceiving.

I really recommend you reading Lee Smolin's 'The Trouble with Physics'.  It takes you to all the places in current physics that do not correlate with each other, and makes great description in brief of all kinds of recognised, Wikapedia documented, alternatives that people have been working on.
Title: Re: Is there a discrepancy with the equivalence principle?
Post by: Mike Gale on 24/02/2017 01:32:07
The weird thing about the bat and ball scenario is that momentum is not conserved unless they are of equal mass. Einstein solved that problem by making mass depend on velocity. There's a similar thought experiment involving a mass that emits light. Conserving momentum in that scenario is tricky.
Title: Re: Is there a discrepancy with the equivalence principle?
Post by: timey on 24/02/2017 01:40:07
But in the case of free fall velocity is not mass dependent, so how can mass be velocity dependent?
Title: Re: Is there a discrepancy with the equivalence principle?
Post by: Mike Gale on 24/02/2017 01:46:04
I just noticed another common misconception in your last response. It's pointless to talk about things moving with respect to space unless you specify whose space you're talking about. There is no absolute reference frame. If you find a reference frame in which you are standing still, I can find another in which you are moving. It's all relative.
Title: Re: Is there a discrepancy with the equivalence principle?
Post by: Mike Gale on 24/02/2017 01:48:26
But in the case of free fall velocity is not mass dependent, so how can mass be velocity dependent?
Velocity is not mass dependent, but mass is velocity dependent. It increases with velocity. The classical equation of motion (mv=mat) neglects this effect. The relativistically correct version is:
mv*gamma=mat
Title: Re: Is there a discrepancy with the equivalence principle?
Post by: timey on 24/02/2017 01:57:10
Post 85:

So what?  It's quite clear that nothing in the universe is at rest.

Post 86:

Quote

"Many contemporary authors such as Taylor and Wheeler avoid using the concept of relativistic mass altogether:

"The concept of "relativistic mass" is subject to misunderstanding. That's why we don't use it. First, it applies the name mass - belonging to the magnitude of a 4-vector - to a very different concept, the time component of a 4-vector. Second, it makes increase of energy of an object with velocity or momentum appear to be connected with some change in internal structure of the object. In reality, the increase of energy with velocity originates not in the object but in the geometric properties of spacetime itself."

So if we are talking about the time component of a 4 vector that is a geometric property of space time itself?
Title: Re: Is there a discrepancy with the equivalence principle?
Post by: Mike Gale on 24/02/2017 02:05:49
Lots of things are at rest. It depends on your on your own state of motion.
I don't know where you got that quote, but it's a non-sequitur. GR theorists certainly prefer 4-momentum over relativistic mass, but it's just a bookkeeping trick. They are one and the same phenomenon. As I said before, mass is a measure of distance and distance is an observer-dependent quantity.
Title: Re: Is there a discrepancy with the equivalence principle?
Post by: timey on 24/02/2017 02:19:46
Mass is a measure of distance?

Under what premise can you say that?

Time is a measure of distance, and if the speed of light is to be held variable to variable distances, then the speed of light can also be held variable to variable seconds where distance then remains constant, and frequency is then the observer dependent phenomenon.

Edit: What exactly is at rest?
Title: Re: Is there a discrepancy with the equivalence principle?
Post by: Mike Gale on 24/02/2017 02:24:25
rs = 2GM/c2
If you set 2G/c2=1 then M=rs.
i.e. 1 kg = c2/2G meters.
That's the old metric of course. For the new one:
1 kg = c2/G meters.
Title: Re: Is there a discrepancy with the equivalence principle?
Post by: timey on 24/02/2017 02:34:17
Ok

rs = 2GM/c2
If you set 2G/c2=1 then M=rs

r*s
r is radius
s is the the Swartzchild radius?

Why does one set 2G/c^2 = 1 ?
Title: Re: Is there a discrepancy with the equivalence principle?
Post by: Mike Gale on 24/02/2017 02:36:52
No. rs is rs. I'm just a lazy typer. You can set G and c to whatever you want. All that changes is the units of measure for mass and distance. In fact, setting c=1 is a common practice because E=m instead of E=mc2. It saves you a lot of typing, but it's easy to lose track of the relationship between G and c for example.
Title: Re: Is there a discrepancy with the equivalence principle?
Post by: timey on 24/02/2017 02:43:52
But what is rs?

And what is 1?

This is where I got the quote btw:

https://en.m.wikipedia.org/wiki/Mass_in_special_relativity
Title: Re: Is there a discrepancy with the equivalence principle?
Post by: Mike Gale on 24/02/2017 02:47:10
Sorry. rs is the Schwarzschild scaling distance. It's the radius of a black hole.
Title: Re: Is there a discrepancy with the equivalence principle?
Post by: timey on 24/02/2017 02:51:07
Good, good.  I'm learning!

And what is the relevance of 1?
Title: Re: Is there a discrepancy with the equivalence principle?
Post by: Mike Gale on 24/02/2017 07:24:54
An arbitrary choice for illustrative purposes. Like I said, you can set G and c to any values you like. The only thing that changes is the units of measure for mass and distance.
Title: Re: Is there a discrepancy with the equivalence principle?
Post by: puppypower on 24/02/2017 11:41:06
But if one were to place oneself on the stationary rocket, the stationary rocket's atomic clock would be ticking normally, and you would observe the rocket in relative motion's clock as ticking slow.
That's a common mistake with SR. Your interpretation of GR is correct. An observer at lower elevation sees the clock running faster whereas an observer at higher elevation sees the clock running slower. If you climb up or down, you will find that the clock has gained or lost time respectively. In SR, each observer perceives the other's clock to run slower. If you accelerate into the other reference frame, you will find that the clock has lost time. If the clock decelerates into your reference frame, you will find that you have lost time. It seems nonsensical, but that's the nature of the beast and you have to get your head around the concept before you delve into GR. The Twins Paradox is very instructive in that regard and Viascience (on YouTube) does a good job on it.

With GR, reference is energy dependent based on gravitational potential; up and down the space-time well. With SR there are two affects going on. One affect is a reference mirage, while the other affect is also energy dependent. In the twin paradox, only one twin will age slower, even though, when in motion, both references see each other moving via the principle of equivalency. One reference sees reality, while the other reference sees a mirage. In SR, Einstein added a relativistic mass term, which works as an energy balance, analogous to mass and gravity. It provides the energy balance which separates the mirage from the real affect.  However, it not easy to measure relativistic mass.

A mirage is not a hallucination. A mirage, like a lake in the desert can be photographed, because it is based on the bending of light. There is physical basis for a mirage, It is not just the imagination.  However, even though it may look and photograph, like water, it is still an illusion. This is why we need to be careful about bold claims the universe, by mistaking mirages for the real thing. That would be the basis for a magic trick, since you can show hard data that appears to make the mirage, real. Mirages can photograph well.


(https://www.thenakedscientists.com/forum/proxy.php?request=http%3A%2F%2Fwww.crystalinks.com%2Fmirage506.jpg&hash=a939e3904a7e72280a58d8b898caa236)
Title: Re: Is there a discrepancy with the equivalence principle?
Post by: GoC on 24/02/2017 12:01:27
Mass is a measure of distance?

Under what premise can you say that?

Time is a measure of distance, and if the speed of light is to be held variable to variable distances, then the speed of light can also be held variable to variable seconds where distance then remains constant, and frequency is then the observer dependent phenomenon.

Edit: What exactly is at rest?

Yes you could but then the reaction time as a rate would suffer between frames. Remember your measuring stick also changes distances. This is what equivalence is all about. Not just time as you understand it but distance for reaction rates measured by a frames clock. Your position puts reaction rates and all physics different between frames. What a mess. Curvature of space and lensing of galaxies show dilation. Gravity follows dilation and Pe to the center of mass. We can measure the dilation and Pe differences as reaction rates but the reaction rates always follow the frames tick rate. So the physics are the same in every frame.

You want to change the speed of light? The speed is the same as a constant where energy dilation increases the distance light has to travel at a constant. This is the easiest way to follow physics. Your way causes more confusion and is inaccurate because dilation is viewable in space. Light is constant!!
Title: Re: Is there a discrepancy with the equivalence principle?
Post by: Mike Gale on 24/02/2017 15:04:40
With GR, reference is energy dependent based on gravitational potential; up and down the space-time well. With SR there are two affects going on. One affect is a reference mirage, while the other affect is also energy dependent. In the twin paradox, only one twin will age slower, even though, when in motion, both references see each other moving via the principle of equivalency. One reference sees reality, while the other reference sees a mirage. In SR, Einstein added a relativistic mass term, which works as an energy balance, analogous to mass and gravity. It provides the energy balance which separates the mirage from the real affect.  However, it not easy to measure relativistic mass.
Ah yes. Einstein called that a stubbornly persistent illusion. In other words, what you see is literally what you get. The point he was trying to make is that it doesn't matter if the effect is real or not. What matters is the observables. Relativistic mass is a case in point. It is not an add-on. It emerges from SR when you apply conservation of momentum and it is borne out by observation.
Title: Re: Is there a discrepancy with the equivalence principle?
Post by: timey on 24/02/2017 15:14:09
Again GoC, you are completely missing the fact that having a temporally dilated g-field would result in the measuring stick remaining unchanged!
Constant metres!

The speed of light does remain constant as it is held relative to the dilated second of the g-field.
The equivalence principle is upheld in that in each reference frame, the speed of light remains 299 792 458 metres held relative to the length of second of that reference frame.

Then for particles with rest mass, all m at h from M experiences an equal addition of gravity potential energy that is derived from its position at h from M, and is not calculated with respect to mass value, (this notion is synonymous to free fall back to front), and again, the equivalence principle is upheld...
All m at h from M remains the equivalent to each other in any reference frame, and this gives us a physical cause for a person ageing in keeping with their time dilated clock, and reciprocally a physical cause as to why m at h from M does not register that their clock is time dilated.

A clever person might note that the light and the mass will be subject to differing rates of time, and they would be correct.  The lights time is negative to the m at h from M's time at that coordinate.
However, m at h from M would not be able to distinguish this, because if the m at h from M's rate of time is of equal difference to that of the g-field, with respect to ground level M, but positive rather than negative, the light will appear to be travelling 299 792 458 metres from the point of view of both rates of time.

My way gives us a fully described cyclic universe.
...And it is current physics that is the MESS!
Title: Re: Is there a discrepancy with the equivalence principle?
Post by: Mike Gale on 24/02/2017 15:21:10
Remember your measuring stick also changes distances. This is what equivalence is all about. Not just time as you understand it but distance for reaction rates measured by a frames clock. Your position puts reaction rates and all physics different between frames. What a mess. Curvature of space and lensing of galaxies show dilation. Gravity follows dilation and Pe to the center of mass. We can measure the dilation and Pe differences as reaction rates but the reaction rates always follow the frames tick rate. So the physics are the same in every frame.

You want to change the speed of light? The speed is the same as a constant where energy dilation increases the distance light has to travel at a constant. This is the easiest way to follow physics. Your way causes more confusion and is inaccurate because dilation is viewable in space. Light is constant!!
You must be careful with the language. Equating mass with distance doesn't change the physics because both are subject to the same dilation. It is nothing more than a change of units, no different than the concept of light years. Variable light speed on the other hand is an alternative way to interpret the Schwarzschild metric. Spatially dependent dilation is an equally valid approach. It's just a matter of preference. If you find one way easier to understand, then stick with it by all means. But keep in mind that they are in fact equivalent and be prepared to mentally translate when conversing with those in the other camp.
Title: Re: Is there a discrepancy with the equivalence principle?
Post by: timey on 24/02/2017 15:36:11
Mike - you are working on the basis of SR time dilation alone.  I haven't included SR time dilation to the picture at-all (for the moment).

I am using 'only' the attractive force of the g-field when I say the the g-field is temporally derived, rather than spatially.

As far as I am concerned the way that you are using time dilation is 2 dimensional and places the responsibility for changes in space at the foot of mass changes due to velocity and a 'stretchy' space.
My way treats space as a third dimension that is caused by the g-field that is determined by M.  The stretchy-ness of space is caused by temporal changes within the g-field itself.  It takes a longer or shorter amount of time to travel distance at 'any' velocity.

(I am the only person in this camp!)
Title: Re: Is there a discrepancy with the equivalence principle?
Post by: Mike Gale on 24/02/2017 15:37:36
Again GoC, you are completely missing the fact that having a temporally dilated g-field would result in the measuring stick remaining unchanged!
Constant metres!

The speed of light does remain constant as it is held relative to the dilated second of the g-field.
The equivalence principle is upheld in that in each reference frame, the speed of light remains 299 792 458 metres held relative to the length of second of that reference frame.

Then for particles with rest mass, all m at h from M experiences an equal addition of gravity potential energy that is derived from its position at h from M, and is not calculated with respect to mass value, (this notion is synonymous to free fall back to front), and again, the equivalence principle is upheld...
All m at h from M remains the equivalent to each other in any reference frame, and this gives us a physical cause for a person ageing in keeping with their time dilated clock, and reciprocally a physical cause as to why m at h from M does not register that their clock is time dilated.

A clever person might note that the light and the mass will be subject to differing rates of time, and they would be correct.  The lights time is negative to the m at h from M's time at that coordinate.
However, m at h from M would not be able to distinguish this, because if the m at h from M's rate of time is of equal difference to that of the g-field, with respect to ground level M, but positive rather than negative, the light will appear to be travelling 299 792 458 metres from the point of view of both rates of time.

My way gives us a fully described cyclic universe.
...And it is current physics that is the MESS!
You have revealed a flaw in your theory. The passage of time cannot depend on who you are or how much mass you comprise. Well, it can I suppose, but it's a solution to a problem that doesn't exist. There is no evidence that big people perceive time any differently than smaller ones. Unless your mass is comparable to your planet of course. That's why GR is formulated in terms of a negligibly small test mass.
Title: Re: Is there a discrepancy with the equivalence principle?
Post by: timey on 24/02/2017 15:42:40
Actually there is!  Everyone has an ideal weight for their height, and if one is overweight, one won't live as long as one would if one isn't.

Also - bodies are a direct result of the gravitational force.  If we go somewhere where gravity is differing, bodies suffer a consequence.
Title: Re: Is there a discrepancy with the equivalence principle?
Post by: Mike Gale on 24/02/2017 15:48:15
Biology is not going to make this any easier. Let's stick with the test mass.
Title: Re: Is there a discrepancy with the equivalence principle?
Post by: timey on 24/02/2017 15:50:12
Biology is a direct result of energy, so I don't see why there shouldn't be a connection.
Title: Re: Is there a discrepancy with the equivalence principle?
Post by: Mike Gale on 24/02/2017 15:54:12
I am using 'only' the attractive force of the g-field when I say the the g-field is temporally derived, rather than spatially.
Yes, but surely you understand that gravity is a function of position, not time. It is only a function of time when you're in motion with respect to the gravitational field.
Title: Re: Is there a discrepancy with the equivalence principle?
Post by: timey on 24/02/2017 15:57:53
No - I am suggesting that gravitational acceleration is a function of time dilation at that position, and that it is this time dilation that causes the directional accelerated motion.
Title: Re: Is there a discrepancy with the equivalence principle?
Post by: Mike Gale on 24/02/2017 15:59:41
Biology is a direct result of energy, so I don't see why there shouldn't be a connection.
That's an oversimplification. My point is that biological concepts (such as the relationship between weight and lifespan) are emergent phenomena. The line between physics and biology is a long and winding road.
Title: Re: Is there a discrepancy with the equivalence principle?
Post by: timey on 24/02/2017 16:01:50
It's not if one turns the equivalence principle upside down like I am.
Title: Re: Is there a discrepancy with the equivalence principle?
Post by: Mike Gale on 24/02/2017 16:03:23
No - I am suggesting that gravitational acceleration is a function of time dilation at that position, and that it is this time dilation that causes the directional accelerated motion.
But that's my point. We are arguing about the validity of an interpretation. Both views are valid. Whatever gets you through the night. Just don't confuse an interpretation with a law of physics.
Title: Re: Is there a discrepancy with the equivalence principle?
Post by: Mike Gale on 24/02/2017 16:12:21
It's not if one turns the equivalence principle upside down like I am.
Okay. Case in point then. There are many reasons why obesity is correlated with a shorter lifespan. Excess fat in the body puts pressure on the organs, making them less efficient. The heart has to move more blood to keep the fat cells alive. Fat cells consume energy that would otherwise be available for other bodily functions. Fat cells reproduce so an obese person rolls the genetic dice more often. Thinking about this, I am reminded that I must go on a diet. The point is, these are emergent phenomena. It's more than just observer-dependent energy.
Title: Re: Is there a discrepancy with the equivalence principle?
Post by: timey on 24/02/2017 16:16:49
But by turning the equivalence principle upside down, energy is NOT observer dependent!
Title: Re: Is there a discrepancy with the equivalence principle?
Post by: timey on 24/02/2017 16:21:41
No - I am suggesting that gravitational acceleration is a function of time dilation at that position, and that it is this time dilation that causes the directional accelerated motion.
But that's my point. We are arguing about the validity of an interpretation. Both views are valid. Whatever gets you through the night. Just don't confuse an interpretation with a law of physics.

The view that the g-field itself being inherent with a contra directional gravitational time dilation without considering m at h from M, but just considering the coordinate of h from M doesn't exist in current physics.

This is an entirely new interpretation of gravitational acceleration in the g-field, and would 'change' the interpretation of the laws of physics, to great advantage I'll add, because in doing so this solves all current physics conundrums!
Title: Re: Is there a discrepancy with the equivalence principle?
Post by: Mike Gale on 24/02/2017 16:31:53
Contra directional dilation (CDD) is certainly a new idea, but it has no basis in physics unless you can establish a need and a cause. Note that the SC metric divorces itself from m by making it negligibly small. That's one of the rules of the game. If you invoke m, you change the game and all bets are off. I understand that you are indeed trying to change the game, but your motive is baseless as far as I can tell. There is no observation or experiment that contradicts GR or SR principles. The equivalence principle is alive and well. Anomalies like galaxy rotation curves demand a refinement of the law, not necessarily a new one (like CDD, DM or DE.)
Title: Re: Is there a discrepancy with the equivalence principle?
Post by: timey on 24/02/2017 16:56:16
Currently Mike, you are calculating G and g without knowing what it is that you are calculating.

Adding the contra directional gravitational time dilation for the g-field surrounding M gives G and g physical cause.

By adding this contra directional gravitational time dilation, in addition to both GR gravitational time dilation, and SR motion related time dilation, this model can provide a fully described cyclic universe where the Big Bang, inflation, and contraction are given physical cause.

How does your relativistic correction go on to give physical cause for the Big Bang?  It doesn't!  And can't!

P.S.  If you want to lose weight eat more protein inclusive of the fat, and lay off the carbs.
...and since you are in USA (?) avoid anything sweetened with corn syrup.  Apart from it likely being genetically modified, which causes inflammation within the digestive system, leading to a whole host of associated health problems, including auto immune disorders, even when not genetically modified cannot be digested into energy form and causes the spare tyre syndrome.
Title: Re: Is there a discrepancy with the equivalence principle?
Post by: Mike Gale on 24/02/2017 17:44:02
I don't calculate them. They are observed quantities. They are certainly not defined by any law of physics, not even yours. The Big Bang has no cause in any stretch of the imagination. It is purported to be a quantum fluke, which is just a fancy version of the anthropic principle. It directs us to shut up and calculate. Note that there are plenty of plausible alternatives to the Big Bang. They just can't get any traction because they are all consistent with observation. There's a book called "The Big Bang Never Happened", which is worthwhile if you're interested in that sort of thing.
My proposed correction to the SC metric is equivalent to the cosmic constant, although it would be more aptly described as a cosmic variable.
Title: Re: Is there a discrepancy with the equivalence principle?
Post by: timey on 24/02/2017 17:55:08
Gravitational acceleration is an observed quantity that at present has no physical causation in current physics.  This is s well documented fact.

My model's rendition of the Big Bang does have physical cause, although if one traced my model's universe's cycles back in time to the first Big Bang, which my model places in the microscopic region, in that each cycle of my model's universe is bigger than the last, admittedly my model is also at a loss to describe how something was created out of nothing!
Title: Re: Is there a discrepancy with the equivalence principle?
Post by: Mike Gale on 24/02/2017 17:58:17
Gravitational acceleration is an observed quantity that at present has no physical causation in current physics.  This is s well documented fact.

My model's rendition of the Big Bang does have physical cause, although if one traced back my model's universe's cycles back in time to the first Big Bang, which my model places in the microscopic region, in that each cycle of my model's universe is bigger than the last, admittedly my model is also at a loss to describe how something was created out of nothing!
Depends what you consider to be a causal explanation. Mass is the cause of gravity. Nobody knows what causes mass. Have you ever heard the expression that "it's turtles all the way down"? If not, Google it.
Title: Re: Is there a discrepancy with the equivalence principle?
Post by: timey on 24/02/2017 18:02:46
Yes indeed, mass is the cause of gravity, but gravitational acceleration is yet to be given a physical cause.
My model states that mass is created by virtual particles, under the remit of this contra directional gravitational time dilation which renders quantum as a continuum, and all that 'complex' space being time dilation, a virtual particle will have 'the time' in which to become real.
Title: Re: Is there a discrepancy with the equivalence principle?
Post by: Mike Gale on 24/02/2017 18:04:30
Gravitational acceleration is caused by variable light speed. Variable light speed is caused by dilation. Dilation is caused by mass. Back to square one.
You are invoking several physical concepts out of context. Virtual particles for example are a way of thinking about energy and momentum transport in QM. They have no basis in relativity and no objective reality (hence the name.)
Title: Re: Is there a discrepancy with the equivalence principle?
Post by: timey on 24/02/2017 18:08:54
Ah - well...  variable light speed is caused by the contra directional gravitational time dilation inherent to the g-field, the g-field is caused by mass, and we are back to square 2 - square root 2 in fact, and Pythagorus!

The contra directional gravitational time dilation and the change to the equivalence principle cause QM to be a continuum that can be united with this altered rendition of relativity.
Title: Re: Is there a discrepancy with the equivalence principle?
Post by: Mike Gale on 24/02/2017 18:15:59
Now that you bring it up, GR is really nothing more than an extension of Pythagorus for curved space.
Title: Re: Is there a discrepancy with the equivalence principle?
Post by: timey on 24/02/2017 18:19:46
Have you seen this:

http://www.mrelativity.net/MBriefs/Relativistic%20Escape%20Velocity%20using%20Special%20Relativity.htm

...of interest to me only in that he is using golden ratio in relation to square root 2.

(Great conversation btw, I'm really enjoying it!)
Title: Re: Is there a discrepancy with the equivalence principle?
Post by: Mike Gale on 24/02/2017 18:26:44
Yes. I quoted that article before. Jeffrey argues that it's not peer reviewed, but I can't find any fault with it.
BTW - QM is utterly incompatible with a continuum. That would lead us back to the ultraviolet catastrophe.
Title: Re: Is there a discrepancy with the equivalence principle?
Post by: GoC on 24/02/2017 18:30:11
And SR is the hypotenuse of Pythagoras. GR is a dilation radius issue and physical.
Title: Re: Is there a discrepancy with the equivalence principle?
Post by: Mike Gale on 24/02/2017 18:32:27
I'm going to be out of touch of a while. I'm off to Australia. I may pick this up when I get there if time permits.
Title: Re: Is there a discrepancy with the equivalence principle?
Post by: timey on 24/02/2017 18:35:32
Not with the change I make to the equivalence principle.
Planck's h constant becomes a time dilation related phenomenon, where the energy being added to the atoms of the black body are causing the atoms to be running at a faster rate of time and emit higher frequency photons.

If you take this time dilation factor into account, the ultraviolet catastrophe doesn't occur.

Hope you have good time in Oz, are you going West or East?
Title: Re: Is there a discrepancy with the equivalence principle?
Post by: Mike Gale on 24/02/2017 19:58:47
I'm back sooner than I thought. WiFi is.a wonderful thing.
And SR is the hypotenuse of Pythagoras. GR is a dilation radius issue and physical.
No. SR extends the concept of a right triangle by projecting it onto a curved surface. There's more to it of course, but that's more or less how it starts once you realize that spacetime is curved.
Title: Re: Is there a discrepancy with the equivalence principle?
Post by: Mike Gale on 24/02/2017 20:13:34
Not with the change I make to the equivalence principle.
Planck's h constant becomes a time dilation related phenomenon, where the energy being added to the atoms of the black body are causing the atoms to be running at a faster rate of time and emit higher frequency photons.

If you take this time dilation factor into account, the ultraviolet catastrophe doesn't occur.

Hope you have good time in Oz, are you going West or East?
The only thing that saves us from the UV catastrophe is quantization. That's what Planck's h is all about. It defines the granularity of space (or time if you prefer.) It's an outlandish idea and Planck himself didn't actually believe it at first. He offered it up as an anecdote to highlight the discrepancy between the classical model of a black body radiator and the experimental result. One would think it should be subject to relativity since it involves energy, but it's actually an invariant. You don't need to mess with it to conserve energy in a relativistic context. Again though, it's really just another one of those unit conversion factors. It relates frequency to energy. You can set it equal to 1 if it gets in the way, but you have to compensate in all of your other conversion factors.
Title: Re: Is there a discrepancy with the equivalence principle?
Post by: Mike Gale on 24/02/2017 20:20:32
West
Title: Re: Is there a discrepancy with the equivalence principle?
Post by: timey on 24/02/2017 20:54:56
If you calculate the energy and frequency increases per shorter length seconds as energy is increased then the quantised nature of the experiment is a continuum.

West aye?  I hung out in Freemantle, Perth for a while back in 88.  If you're there on holiday, (rather than business) then, although my info is just a tad out of date, I found Margaret River to be lovely, great surf if you like that sort of thing, and the petrified forest is amazing.
Title: Re: Is there a discrepancy with the equivalence principle?
Post by: GoC on 25/02/2017 13:56:54
I'm back sooner than I thought. WiFi is.a wonderful thing.
And SR is the hypotenuse of Pythagoras. GR is a dilation radius issue and physical.
No. SR extends the concept of a right triangle by projecting it onto a curved surface. There's more to it of course, but that's more or less how it starts once you realize that spacetime is curved.

You are looking at two different issues
Ok Mike here is the data:

The Lorentz contraction for half the speed of light is 0.866025. Where does that come from?  Lets look at half the speed of light as a right triangle. It creates a 30,60,90 triangle. When we look at Cos 30 we get 0.866025 relative to the speed of light as 1 for the motion of light. It takes about 13.4% longer to travel the distance as perpendicular. That is your Lorentz contraction. Your light clock is affected by that same issue to reduce the tick rate. When you relate it to reaction time of a frame the cycle time of the electron also is affected by that same 13.4% reduction. Light and mechanical clocks tick at the same rate in every frame. So the biological reaction rate is slowed and affects your biological aging clock

If you drew a line between two points of a circle you would be correct it does not include the additional circle radius. That is your confusion. All vector speeds give the same hypotenuse issue but you cannot draw a line between two points of a circle or sphere for an accurate distance of light. 

So what exactly is your answer as No mean mathematically?

In GR your curve is a two dimensional understanding of a three dimensional issue. Dilation is a sphere from a round object with a gradient of its dilation of energy. Dilation of energy being its greatest in the center of mass where energy is the least dense (dilated the most) out. As the difference between the most dilated position increases as a dilation gradient, attraction to the center of mass increases while your clock tick rate also increases measuring the energy gradient state increase away from the center of mass.

The physical energy dilation in GR matches the Pythagoras hypotenuse travel distance in SR for equivalency. Relativity is just a plain beautiful mathematical representation of the observed universe.
Title: Re: Is there a discrepancy with the equivalence principle?
Post by: timey on 25/02/2017 15:52:12
I'm back sooner than I thought. WiFi is.a wonderful thing.
And SR is the hypotenuse of Pythagoras. GR is a dilation radius issue and physical.
No. SR extends the concept of a right triangle by projecting it onto a curved surface. There's more to it of course, but that's more or less how it starts once you realize that spacetime is curved.

In my model Mike, the right triangle is extended by the time dilation of the g-field itself via G or g, of which their values are 'already' being used in the current mathematics.
The concept of this dilation, or curvature of space being time dilation related means that everything 'appears' to curve through space because it is taking a longer time to travel a metre in the weaker g-field, and a shorter time to travel a metre in the stronger g-field.
Therefore any acceleration, or deceleration, that m, or M, or light, is subject to in the g-field can be calculated as time dilation related, and the triangle isn't extended spatially, it is extended temporally.

If one now adds GR time dilation, and SR time dilation on top of this g-field, as is 'already' occurring in the current maths, one will find that the SR 'action' of extending the right triangle for a curvature of space has 'already' been taken care of by the time dilated g-field.

No need to add curvature twice!
Title: Re: Is there a discrepancy with the equivalence principle?
Post by: GoC on 25/02/2017 22:43:47

No need to add curvature twice!

Of course not. Physical dilation of GR and hypotenuse of speed for SR. With the photon time and distance are related. So temporal speed and distance are interchangeable.

Because of dilation and no ability to observe 90 degrees or even 180 degree reflection time is never fixed. There is no preferred frame.
Title: Re: Is there a discrepancy with the equivalence principle?
Post by: timey on 25/02/2017 23:06:11
No preferred frame for what exactly?

Why is that relevant?

P.S.
Space is dilated.
GR contracts time, so how does GR cause space dilation?
Faster seconds cause acceleration in the decelerated g-field, not deceleration.
Title: Re: Is there a discrepancy with the equivalence principle?
Post by: timey on 26/02/2017 00:04:53
The result of what you are defending so vigorously, on my New Theories thread GoC, is that SR is making space longer, and GR is making things go faster, over a longer space, ie: metres comprised of 'empty nothing' that stretch, and this stretching of nothing then serves to retain the desired notion of constant speed...

How can 'nothing' be stretched?

Magic perhaps?

Or do you wish to take the view of there being a 'fabric' of space that is stretched?
A fabric that is imperceivable, and to all intents and purposes, apart from the stretchiness, is entirely indistinguishable from being anything but empty tracts of distance between clumps of mass.

...and you declare GR and SR to be a brilliant and perfect mathematical description of our universe?

I don't think so...
Title: Re: Is there a discrepancy with the equivalence principle?
Post by: Mike Gale on 26/02/2017 12:35:55
Ok Mike here is the data:

The Lorentz contraction for half the speed of light is 0.866025. Where does that come from?  Lets look at half the speed of light as a right triangle. It creates a 30,60,90 triangle. When we look at Cos 30 we get 0.866025 relative to the speed of light as 1 for the motion of light. It takes about 13.4% longer to travel the distance as perpendicular. That is your Lorentz contraction. Your light clock is affected by that same issue to reduce the tick rate. When you relate it to reaction time of a frame the cycle time of the electron also is affected by that same 13.4% reduction. Light and mechanical clocks tick at the same rate in every frame. So the biological reaction rate is slowed and affects your biological aging clock

If you drew a line between two points of a circle you would be correct it does not include the additional circle radius. That is your confusion. All vector speeds give the same hypotenuse issue but you cannot draw a line between two points of a circle or sphere for an accurate distance of light. 

So what exactly is your answer as No mean mathematically?

In GR your curve is a two dimensional understanding of a three dimensional issue. Dilation is a sphere from a round object with a gradient of its dilation of energy. Dilation of energy being its greatest in the center of mass where energy is the least dense (dilated the most) out. As the difference between the most dilated position increases as a dilation gradient, attraction to the center of mass increases while your clock tick rate also increases measuring the energy gradient state increase away from the center of mass.

The physical energy dilation in GR matches the Pythagoras hypotenuse travel distance in SR for equivalency. Relativity is just a plain beautiful mathematical representation of the observed universe.
It's getting hard to follow a conversation on this thread because we're pursuing a number of different topics in parallel. This one started with Reply#126 in which you seemed to imply that Pythagoras is only partially accounted for in SR and it is not until you consider GR that the concept is fully embraced. That is patently false. SR introduces a twist on Pythagoras in which c2=a2-b2 rather than c2=a2+b2. (It's a pinch as opposed to a rotation.) What GR brings to the table is the idea of geodesics in curved space.
Title: Re: Is there a discrepancy with the equivalence principle?
Post by: GoC on 26/02/2017 13:39:59
The result of what you are defending so vigorously, on my New Theories thread GoC, is that SR is making space longer, and GR is making things go faster, over a longer space, ie: metres comprised of 'empty nothing' that stretch, and this stretching of nothing then serves to retain the desired notion of constant speed...
Your assumptions dictate your understanding.  Incorrect assumptions predict incorrect results.

Quote
How can 'nothing' be stretched?

Magic perhaps?
It would have to be magic to stretch zero. Go back to your assumptions for the proper answer.
Quote
Or do you wish to take the view of there being a 'fabric' of space that is stretched?
Its either that or magic. We all have that choice. I prefer fabric myself. How do you stretch zero?

Quote
A fabric that is imperceivable, and to all intents and purposes, apart from the stretchiness, is entirely indistinguishable from being anything but empty tracts of distance between clumps of mass.

Anything at the speed of light would need something faster than the speed of light to allow detection other than orthogonally. How do you stretch zero?

Quote
...and you declare GR and SR to be a brilliant and perfect mathematical description of our universe?
Yes because I took the time to understand relativity and not the shortcuts to misunderstanding.
Quote
I don't think so...

Many that go down the rabbit hole of relativity are confused by the logic controlling the rabbit hole. When you study the rabbit hole a pattern forms. Then you can leave the rabbit hole and view the universe in a different light. No pun intended.

No preferred frame for what exactly?
To measure with of course. When you understand that is no longer a valid question.
Quote
Why is that relevant?
Because there is no standard to use as a calibration.
Quote
P.S.
Space is dilated.
Ok what is being dilated? Nothing? I would disagree.
Quote
GR contracts time, so how does GR cause space dilation?
GR does not contract time. This is your assumption. GR expands energy and a clock measures the energy  density of space. Spacetime is an energy state.

Quote
Faster seconds cause acceleration in the decelerated g-field, not deceleration.

You are confused in the rabbit hole. Energy is expanded and light has further to go through space at c. Clocks slow as they descend a gravity hole. Pe decreases as it descends a gravity well. Energy expands as it descends a gravity well. GR expansion of energy matches SR hypotenuse travel distance to match equivalence of energy c for clock tick rates.

Learn the patterns before you make judgments about relativity.

Mike has a good grasp of relativity. He does not understand it the way I do but still understands it his way. We both understand it needs a matrix I take the matrix further to create conditions that cause relativity. I may be correct or incorrect but relativity is correct by all observations.
Title: Re: Is there a discrepancy with the equivalence principle?
Post by: Mike Gale on 26/02/2017 19:56:42
Relativity involves a lot of ratios and when a ratio changes, it can be hard to figure out whether that is due to the numerator changing or the denominator. Sometimes it's important. Sometimes not. The trick is to learn the difference. A convention or preference is not worth arguing about.
It's also easy to get trapped in the aether argument. Space and time are ultimately just units of measure. They have no objective meaning in and of themselves. They certainly must measure something though and that could be described as an aether. The nature of the aether is elusive. It seems to have intrinsic energy (akin to tension in a guitar string.) It is not the rigid stage that Newton envisioned, but a squishy stage doesn't really clarify anything. In order to make any progress, you have to put all that on the shelf (as Newton did) and think about what the math is trying to tell you.
Title: Re: Is there a discrepancy with the equivalence principle?
Post by: timey on 26/02/2017 20:41:30
If one considers this aether to be a time dilation phenomenon it solves the problem in all respects.

But I'm getting really bored of saying so... Either someone here is going to take the notion seriously and calculate it, or their not.

In the mean time (excuse the pun), how you finding Oz, and have you seen Tombstone?

https://www.thenakedscientists.com/forum/index.php?topic=69840.0
Title: Re: Is there a discrepancy with the equivalence principle?
Post by: Mike Gale on 27/02/2017 07:59:43
Before we can formulate a mathematical description of your model we have to separate the philosophy from the physics. I don't think we're there yet.
Title: Re: Is there a discrepancy with the equivalence principle?
Post by: Mike Gale on 27/02/2017 12:03:24
I keep getting hung up on the term "standard second" because, in my mind at least, that translates to a preferred reference frame. I think you mean to say "proper time", which is what you measure on a stationary clock. Does that sound right? If so, you would be well served to adopt that terminology when conversing with physicists. The concept of proper time gets fuzzy in GR because it's not invariant like it is in SR unless you are talking about a free fall reference frame.
Title: Re: Is there a discrepancy with the equivalence principle?
Post by: timey on 27/02/2017 13:42:17
The standard second is what all units of measurement that involve time are held relative to.

Speed of light = 299 792 458 metres per standard second...
Frequency = wave cycles per standard second...
Joules per standard second...
Etc...

So it is good news that you are hung up on the term standard second translating to a preferred reference frame, because that is exactly and entirely my point!

My point being that if one measures all the above as per the rate of time as per the clock in the 'other' reference frame, then distance, frequency, and energy measurements will be exactly the same in each and every reference frame.*

The equivalence principle currently states that this is the case, but for the reason that the clock is ticking normally in the other reference frame, as per a standard second measurement, and only appears different from the other reference frame.
The difference I make just states that the clock is actually physically ticking at a differing rate to the standard second, but it will appear normal if you go to that other reference frame for the reason that you and your atoms will also be ticking at the differing rate of that reference frame, therefore giving a physical reason for the notion of ageing in keeping with ones time dilated clock.

*But... 'a clock' cannot measure what time is doing in open space.
Therefore it is my suggestion that there is something different going on with time in open space than there is for a clock, (and m in general), when placed in open space.

So 'proper time' is the time on the stationary clock.  But what rate is 'proper time' running at?

Why do you say that SR time is invariant?
Clearly the clock in the frame that is in relative motion to the stationary clock is ticking at a slower rate.
But how can you state the stationary clock as stationary?
A stationary clock can be considered stationary held relative to a clock that is not in motion with respect to the stationary clock, but neither clocks are actually stationary.  Both will be in motion relative to a clock somewhere else.

Furthermore, both clocks may be stationary with respect to each other, but be placed in differing gravity potentials.
For instance:  Place your first clock on ground, and your second clock on a helicopter at a height of 100 metres, and then rather than the helicopter maintain its position directly above the ground clock, whereas the clock on the helicopter would be in motion faster than the ground clock, ask your helicopter to be in motion as per the clock on the ground, where the helicopter will observe the ground clock to be moving away with the rotation of the earth.
Both clocks are now stationary with respect to each other, but the helicopters clock will be ticking faster.
Which clock is displaying the 'proper time' in this instance?
Title: Re: Is there a discrepancy with the equivalence principle?
Post by: GoC on 27/02/2017 15:26:19
The standard second is what all units of measurement that involve time are held relative to.

There is no standard second.
Quote
Speed of light = 299 792 458 metres per standard second...
Frequency = wave cycles per standard second...
Joules per standard second...
Etc...

So it is good news that you are hung up on the term standard second translating to a preferred reference frame, because that is exactly and entirely my point!
You can mathematically chose to vary the speed of light. I am not hung up on a preferred frame. If you vary the speed of light there is still no preferred frame. Mike explained that.

Quote
My point being that if one measures all the above as per the rate of time as per the clock in the 'other' reference frame, then distance, frequency, and energy measurements will be exactly the same in each and every reference frame.*

No the ratios of speed of light to reaction and distance measurements are the same in every frame. There is a distinct difference you may not be understanding.
Quote
The equivalence principle currently states that this is the case, but for the reason that the clock is ticking normally in the other reference frame, as per a standard second measurement, and only appears different from the other reference frame.
Yes observed to be physically different with equivalent ratios of reaction, SOL and distance within that frame but not between frames.

Quote
The difference I make just states that the clock is actually physically ticking at a differing rate to the standard second, but it will appear normal if you go to that other reference frame for the reason that you and your atoms will also be ticking at the differing rate of that reference frame, therefore giving a physical reason for the notion of ageing in keeping with ones time dilated clock.

Except for the standard second reference that is essentially correct. There is no standard reference frame of a standard second.
Quote
*But... 'a clock' cannot measure what time is doing in open space.
It can measure the energy density of the space it occupies. We can then compare spacetime energy by tick rate. A clock only measures the energy of a frame in terms of tick rate.

Quote
Therefore it is my suggestion that there is something different going on with time in open space than there is for a clock, (and m in general), when placed in open space.
Yes but the dilation of mass is insignificant for a ratio of masses we are measuring against. You suggest all mass falls at the same rate in a vacuum. The truth is we can not measure the difference in attraction because it is also insignificant. M is attracted to m and m is attracted to M. Divide the mass of the earth by the mass of  either item  and there is no significant difference. We cannot measure the difference.

Quote
So 'proper time' is the time on the stationary clock.  But what rate is 'proper time' running at?
There is no such thing as a stationary clock.

Quote
Why do you say that SR time is invariant?
Because we do not view objects as perpendicular. It's always an angle greater than 90 degrees by vector. c is constant while geometry of motion is corrected by relativity. If relativity was wrong than geometry of motion is wrong. I trust math.

Quote
Clearly the clock in the frame that is in relative motion to the stationary clock is ticking at a slower rate.
No such thing as a stationary clock.
Quote
But how can you state the stationary clock as stationary?

Before understanding relativity if I did I was wrong. There is no such thing as a stationary clock.
Quote
A stationary clock can be considered stationary held relative to a clock that is not in motion with respect to the stationary clock, but neither clocks are actually stationary.  Both will be in motion relative to a clock somewhere else.

So we can only measure the ratio's of the energy state of a frame. What is your issue with relativity's equivalency principle?
Quote
Furthermore, both clocks may be stationary with respect to each other, but be placed in differing gravity potentials.
So we are back to measuring the energy ratios based on how much of c is being used by each frame. Nothing to do with being stationary. There is no stationary. That is a local illusion.

Quote
For instance:  Place your first clock on ground, and your second clock on a helicopter at a height of 100 metres, and then rather than the helicopter maintain its position directly above the ground clock, whereas the clock on the helicopter would be in motion faster than the ground clock, ask your helicopter to be in motion as per the clock on the ground, where the helicopter will observe the ground clock to be moving away with the rotation of the earth.
Both clocks are now stationary with respect to each other, but the helicopters clock will be ticking faster.
This means the clocks are in different energy frames. Clocks measure the energy state of the frame it occupies.
Quote
Which clock is displaying the 'proper time' in this instance?
Both are displaying the 'proper energy state' of the frame they are occupying. Can you define time properly?
Title: Re: Is there a discrepancy with the equivalence principle?
Post by: timey on 27/02/2017 16:24:46
If a clock is displaying the energy state of the reference frame it is occupying, then one cannot state that energy is observer dependent.

If a clock measures the energy state of the reference frame of space it is occupying, this statement in itself separates the clock from the reference frame of the space it is measuring.  Now you have 2 reference frames.  The reference frame of the clock, and the reference frame of space the clock is measuring.
As the clock is now occupying the reference frame of the space, how can you say the clock is making a measurement of the space it is occupying?
The clock is measuring the reference frame of its own self when in that space, not the other way round.

Define time:
Time is not a measurement to be imposed upon the universe, but is a reactive phenomenon inherent 'within' the universe.
Title: Re: Is there a discrepancy with the equivalence principle?
Post by: GoC on 27/02/2017 22:15:31
If a clock is displaying the energy state of the reference frame it is occupying, then one cannot state that energy is observer dependent.
True, energy needs no observer to exist.
Quote
If a clock measures the energy state of the reference frame of space it is occupying, this statement in itself separates the clock from the reference frame of the space it is measuring. 

No it's in the frames energy state to measure. As in a thermometer for temperature.
Quote
Now you have 2 reference frames. 
Yes energy and mass. E=mc^2. mc for energy moving electrons and c for energy occupying space. Electrons and photons are confounded in every frame by c.
Quote
The reference frame of the clock, and the reference frame of space the clock is measuring.
They are confounded in the same frame to measure the same speed of light in every frame.
Quote
As the clock is now occupying the reference frame of the space, how can you say the clock is making a measurement of the space it is occupying?
By measuring the same speed of light in every frame. Once again mechanical and light clocks tick at the same rate in every frame so the photon and the electron are confounded in every frame.
Quote
The clock is measuring the reference frame of its own self when in that space, not the other way round.
Yes and the electron mechanical clock ticks at the same rate as the photon clock. The photon is pure energy displayed as distance.
Quote
Define time:
Time is not a measurement to be imposed upon the universe,
Correct man has no power over fundamental energy.
Quote
but is a reactive phenomenon inherent 'within' the universe.
The reaction phenomena is moving the electrons confounded with the photons in every frame. Time=Motion =Energy c. Relativity explains it all already. It even explains gravity as a difference in energy states and mass being attracted to the lowest energy state by dilation in GR.

Learn relativity mathematics. The gamma term is dilation of energy.
Title: Re: Is there a discrepancy with the equivalence principle?
Post by: timey on 27/02/2017 22:31:08
Go and find me the Dark Energy and Dark Matter that are required for Relativity to be a viable theory...
Without them Relativity is dead!

Until such time please excuse me while I discuss this alternative that describes all observation without them.
Title: Re: Is there a discrepancy with the equivalence principle?
Post by: Mike Gale on 27/02/2017 23:20:55
A clock can be stationary with respect to an observer. Your biological clock is a case in point. It just doesn't make sense to talk about the reference frame of space because space and time are observer dependent quantities. The standard second is whatever you perceive it to be in your reference frame. Everybody has a different perspective on that. The clock on the helicopter runs faster than one on the ground because they are in different reference frames. SR can't cope with that scenario because, even though the observers are stationary with respect to one another, they are accelerated to different extents. In classical terms, they have different PEs. PE can be interpreted as variable light speed in GR.  You can attribute that to time dilation or space dilation, but not both (as in SR.)
Title: Re: Is there a discrepancy with the equivalence principle?
Post by: timey on 28/02/2017 00:15:13
Mike - it doesn't make sense that space and time are observer dependent.
... And this is where the staunch Relativist will tell me I don't understand relativity, which is wrong.  I do understand, and it doesn't make physical sense.

My objective is to make sense of the observations without the need to invoke any unobserved quantities.
My model can do this - but it requires that one assume that the energy and frequency of a GR time dilated clock, and the energy and frequency of an SR time dilated clock are 'not' observer dependent, and that gravitational acceleration/deceleration are a third time dilation phenomenon.

Yes - a second may be whatever one perceives it to be, however the maths describing all observations hold the NIST standard second relative to that observation via measurement.  Therefore the standard second is being used as the preferred reference frame whether one recognises the fact or not.
If one measures a differing rate of time via the standard second, energy and frequency will be greater, or lesser than a reference frame where one is stating a standard second.

What physics comprises of currently is a bunch of very different mathematical systems of which one may employ one or the other of a choice.  But none of these systems can describe the whole shebang, only portions of it.

A unified theory of everything will employ one system of mathematics that can describe all observations without relying on unobserved quantities.

This has been my goal...

I think my model and the changes that I suggest to the current mathematical systems are capable of doing such.

The clock on the helicopter that is maintaining the same speed as the clock on the ground, observing the clock on the ground moving away with the rotation of the planet is relevant.
This being because when NIST conducted their ground level relativity tests, they measured a clock that was at a greater radius than the clock on the ground, (albeit just 1 metre greater radius) therefore the elevated clock is experiencing a greater speed than the lower clock is, and will be subject to greater SR effects.
Title: Re: Is there a discrepancy with the equivalence principle?
Post by: Mike Gale on 28/02/2017 01:44:56
Relativity may not make sense in your world view, but it is a perfectly consistent account of observation (except perhaps galaxy rotation curves.). The mathematical logic is infallible.
BTW - I am enjoying a sunny day in Perth and I can report that the laws of physics are alive and well down here, too.
Title: Re: Is there a discrepancy with the equivalence principle?
Post by: Mike Gale on 28/02/2017 01:50:26
Note also that SR cannot cope with acceleration. It describes KE in the context of uniform motion. GR describes PE  in a stationary context, although it is also valid for free fall because PE=KE in that case.
Title: Re: Is there a discrepancy with the equivalence principle?
Post by: timey on 28/02/2017 03:43:01
In answer to post 151:

I remember that the sky looks much higher in Perth than it does in UK. Observer dependency, or more Dark Energy down under.  Wotcha reckon?  ;)

Look - I agree that relativity gives a very good, in fact the best representation of the universe that physics currently has.  There is no doubt about this, I am not contending the fact, and indeed am in awe of its brilliance myself...
Especially considering how Einstein was so very much less informed than we are today.

But come on Mike, the very author of this theory declared it incomplete, was sure that he was missing something, and spent the rest of his life trying to figure out what and why...

There are only so many ways of fitting observation together into an architecture of mechanical viability.  My model's alterations are all merely a transference of the current concepts and mathematics into an altered arrangement.  Mostly just switching polarities really.

There are many problems with current physics.  They are well documented in all the physics books I've read - and I have spent the last 9 years giving it a lot of in-depth thought and research.

There are many observations that do not agree with current theory.
For one - since the advent of the Hubble telescope, a team of astrophysicists led by Eric Lerner from Lawrenceville plasma physics have conducting a surface brightness test first suggested by Richard Tolman.
When observing closer galaxies in relation to very distant galaxies they have found that there is an anomaly with luminosity magnitude:

http://www.sci-news.com/astronomy/science-universe-not-expanding-01940.html

Quote
Therefore if the Universe is not expanding, the redshift of light with increasing distance must be caused by some other phenomena – something that happens to the light itself as it travels through space.

“We are not speculating now as to what could cause the redshift of light,” Mr Lerner said.

Title: Re: Is there a discrepancy with the equivalence principle?
Post by: timey on 28/02/2017 03:51:17
In addition to my answer to post 151

Note also that SR cannot cope with acceleration. It describes KE in the context of uniform motion. GR describes PE  in a stationary context, although it is also valid for free fall because PE=KE in that case.

Yes - and PE is related to GR gravitational time dilation...
... And I'm suggesting that this description of KE is related to my model's contra directional gravitational time dilation, that gives physical cause to the acceleration, or deceleration, of gravity.

Edit: This also gives a counterpart reason for the dilation of space that is equivalent to SR dilation of the metre, this being a temporal dilation to replace SR's spatial dilation.
Title: Re: Is there a discrepancy with the equivalence principle?
Post by: Mike Gale on 28/02/2017 10:09:48
Einstein certainly suspected that something was amiss in his theory. That's why he was playing around with the cosmological constant. The problem is that, without a cosmological constant, the white hole origin theory (aka. Big Bang) predicts an expanding universe. Einstein was trying to stop the expansion so the universe would be static. Hubble's discovery compelled him to abandon that effort. His theory was in perfect agreement with observation at the time of his death. Since then we have discovered accelerated expansion and anomalous orbits, which are attributed to DE and DM respectively. These are admittedly contrived, but they do not necessarily challenge relativity at its core as you contend.
Title: Re: Is there a discrepancy with the equivalence principle?
Post by: GoC on 28/02/2017 11:56:45
Einstein certainly suspected that something was amiss in his theory. That's why he was playing around with the cosmological constant. The problem is that, without a cosmological constant, the white hole origin theory (aka. Big Bang) predicts an expanding universe. Einstein was trying to stop the expansion so the universe would be static. Hubble's discovery compelled him to abandon that effort. His theory was in perfect agreement with observation at the time of his death. Since then we have discovered accelerated expansion and anomalous orbits, which are attributed to DE and DM respectively. These are admittedly contrived, but they do not necessarily challenge relativity at its core as you contend.
DM and DE actually give cause to Relativity. The two different descriptions are actually one thing. The point structure of the Aether with spin for energy c creating Relativity. The Aether spin is Quantum Mechanics creating c. A planet has pe gradient. A solar system has pe gradient. A galaxy has a pe gradient. When the voyagers left our solar system they appeared to slow down. To this day scientists are scratching their heads wondering why? The threshold of the solar systems pe decrease was crossed and the clock tick rate increased. That signaled a slow down in their model not truly understanding relativity affects in the correct model.

Main streams model is red shift from SR only for galaxies. There is also a GR red shift not being accounted for by our pe position of observation further out in our galaxy. Light from galaxies mostly come from 25% in the center of galaxies where dilation is the greatest. We are out in the 75% distance from center. Einstein was probably correct in his steady state. We make up the BB due to main streams model of no medium of gravitational potential (pe). Now reason for a BB. BH's actually prove the BB time line wrong. There whole model needs revamping not relativity. Aether, Spacetime, Energy, DM and DE call it anything you like but transfer has to have a medium and that medium is what we call the spectrum.
Title: Re: Is there a discrepancy with the equivalence principle?
Post by: timey on 28/02/2017 12:32:56
Einstein certainly suspected that something was amiss in his theory. That's why he was playing around with the cosmological constant. The problem is that, without a cosmological constant, the white hole origin theory (aka. Big Bang) predicts an expanding universe. Einstein was trying to stop the expansion so the universe would be static. Hubble's discovery compelled him to abandon that effort. His theory was in perfect agreement with observation at the time of his death. Since then we have discovered accelerated expansion and anomalous orbits, which are attributed to DE and DM respectively. These are admittedly contrived, but they do not necessarily challenge relativity at its core as you contend.

No - Einstein added his cosmological constant to GR that ensured that his rendition of a steady state did not CONTRACT!

Henrietta Swan Leavitt noticed that there was a correlation between red shift and distance, which Hubble then calculated as being velocity related.
This is what began the trend to the wide scale acceptance of the Big Bang notion and an expanding universe, and therefore Einstein retracted his cosmological constant as folly.

It is since we have discovered that the red shifts of the red shift distance correlation are getting redder, that Einstein's cosmological constant has come back into the forefront to become significant to an accelerated expansion of the universe, because in conjunction with Hubble's velocity related red shift distance correlation, the cosmological constant has become relevant to Dark Energy as a means of describing this accelerated expansion.

No - his theory was not in perfect agreement with observation at his death.  In consideration of QM and the fact that GR and QM are incompatible, Einstein was dissatisfied.

It is not I personally who contends relativity.  Nor did I invent any of the contentions to relativity that I discuss.  These are all contentions that qualified and respected physicists, who for the most part hold positions in respected Universities, who's books I've read, who's research I've investigated, etc, have put forward.

The DES, a project designed to identify Dark Energy, a supposed 5 year project, supposedly concluding in 2016, has found no sign of the stuff, and Dark Matter itself also remains elusive.
Title: Re: Is there a discrepancy with the equivalence principle?
Post by: GoC on 28/02/2017 14:40:26
Nothing can be measured directly at the speed of light. It will remain elusive until we recognize the orthogonal measurements are all related to c as a constant fundamental energy. It's out there just not recognized for what it is they are measuring.
Title: Re: Is there a discrepancy with the equivalence principle?
Post by: timey on 28/02/2017 14:54:08
If mass, or space, or time, or a combination of all 3, are contracted or dilated - then a change in energy occurs, either as a result, or due to being the cause...

And this c you are on about is 'just' speed.  A speed GoC...comprende? Just a speed!
Ok?
Title: Re: Is there a discrepancy with the equivalence principle?
Post by: timey on 28/02/2017 15:13:47
I agree that c is the universal speed limit, and that despite dilation or contraction of time, that c remains constant in each and every reference frame.

My model clearly adheres to this axiom within the changes I make to the equivalence principle.
Title: Re: Is there a discrepancy with the equivalence principle?
Post by: timey on 28/02/2017 15:24:24
... And not only does my model strictly adhere to the axiom of c being the universal speed limit, it actually manages to do so far better and more adhesivly than conventional relativity, which has a reference frame known as 'the fabric of space' that conventional relativity states as exempt from this axiom.

So as far as conventional relativity goes, relativity contravenes its own self stated axiom, this axiom being the very foundation that relativity itself is built upon!
Title: Re: Is there a discrepancy with the equivalence principle?
Post by: GoC on 28/02/2017 21:21:54
To suggest there is nothing controlling the electron and photon to be confounded is a violation of logic in the first order. But that in no way interferes with Relativity based on its postulates. There has to be a substance creating the spectrum. The substance confirms mechanics over the magic of main stream beliefs in no fabric.
Title: Re: Is there a discrepancy with the equivalence principle?
Post by: timey on 28/02/2017 21:48:04
Who suggested that there is nothing controlling the electron and photon to be confounded?

I'm trying to describe a new interpretation of relativity here.  I really don't appreciate your input.  I think your grasp on conventional relativity is lacking.  You continually bleat on about confounded this and energy dilation that, fundamental c, and so on, while not engaging with the topic or responding to that which is being discussed.

I am suggesting that gravitational acceleration is time dilation related, and outlining a change to the equivalence principle.
Unfortunately I'm also having to point out the 'documented' flaws in conventional relativity as well.
If you even responded in kind to the 'current physics relativity' flaws that I'm pointing out, rather than engaging in the changes I suggest - I could just about forgive you - but you do neither.  You just clog up the thread with your confoundedness notion making it difficult to retain topical continuity.

Again - with all due respect, I do really wish you would open your own thread to talk about confounded particles upon.
Title: Re: Is there a discrepancy with the equivalence principle?
Post by: Mike Gale on 28/02/2017 23:46:05
I agree with Timey about staying on topic. It's always tempting to interject your own hypotheses. I could be accused of that too, but I only do so to illustrate alternative points of view when the discussion ventures into areas of overlap.
As for the cosmological constant, timey is right that it was supposed to stop the universe from contracting, but it has the opposite effect for a white hole. The point is that Hubble's discovery alleviated the need for any such thing. But accelerated expansion suggests that Einstein was actually on the right track. The cosmological variable is a better way to account for that because it is not an ad hoc factor. There is actually a physical explanation for it.
Title: Re: Is there a discrepancy with the equivalence principle?
Post by: timey on 01/03/2017 01:04:56
Only if Hubble's velocity related red shift distance correlation interpretation is valid.

If not then Einstein was on the right track but blind-sided by his steady state preconceptions...

What did you think about the findings of Eric Lerner and his team of astrophysics concerning the magnitude of very distant galaxies in relation to those much closer?

Btw - point of contention, the evidence for expansion, and accelerated expansion is not physical, nor fact, but is merely an interpretation of an observation!
Title: Re: Is there a discrepancy with the equivalence principle?
Post by: Mike Gale on 01/03/2017 05:13:56
I would say accelerated red shift is an observation, which is by definition a fact. As such, it demands an explanation and the best one going comes from SR and GR. The only sticking point is that GR doesn't quite match observation unless you invoke DE. We're working on that.
As for fading galaxies, there are many possible explanations. Estimating galactic mass is an inexact science for example. Intergalactic dust is not out of the question either. It's worth investigating, but it's too early to draw any conclusions.
Title: Re: Is there a discrepancy with the equivalence principle?
Post by: GoC on 01/03/2017 12:01:15
Who suggested that there is nothing controlling the electron and photon to be confounded?

I'm trying to describe a new interpretation of relativity here.  I really don't appreciate your input.  I think your grasp on conventional relativity is lacking.  You continually bleat on a to relativity about confounded this and energy dilation that, fundamental c, and so on, while not engaging with the topic or responding to that which is being discussed.

The confounding of the electron and photon is the very essence of equivalence between SR and GR. Nothing wrong with Relativity issues related to equivalency. I am trying to show you that. Your frustration is due to not being able to find a conjecture to invalidate relativity.
Quote
I am suggesting that gravitational acceleration is time dilation related, and outlining a change to the equivalence principle.

Gravitational acceleration is time related. Its in Einstein's gamma factor.

 
Quote

Unfortunately I'm also having to point out the 'documented' flaws in conventional relativity as well.
Your pointing out flaws in subjective interpretation only and not flaws in relativity equivalence.

 
Quote
If you even responded in kind to the 'current physics relativity' flaws that I'm pointing out, rather than engaging in the changes I suggest - I could just about forgive you - but you do neither. 

There are no flaws in relativity for those who understand relativity correctly. Equivalency is a corner stone of relativity. Forgive your self if you cannot really find a flaw.

 
Quote
 
 You just clog up the thread with your confoundedness notion making it difficult to retain topical continuity.
 
Confounded electron and photon in every frame is the very definition of equivalency. This is observed as fact!!!!!! The subjective nature of cause is the only issue.
 
Quote

Again - with all due respect, I do really wish you would open your own thread to talk about confounded particles upon.
 
Prove relativity observations of physics not being the same in every frame. Then there is a equivalency issue. Until then you need to understand relativity is correct.
[/quote]
Title: Re: Is there a discrepancy with the equivalence principle?
Post by: Mike Gale on 01/03/2017 12:55:18
Perhaps I missed something. What's this business about confounded electrons and photons? And how is it relevant to timey's theory?
Note that the equivalency principle is that objects in free fall do not feel their own weight. You seem to be talking about something else entirely.
Title: Re: Is there a discrepancy with the equivalence principle?
Post by: GoC on 01/03/2017 15:05:17
Perhaps I missed something. What's this business about confounded electrons and photons? And how is it relevant to timey's theory?
Note that the equivalency principle is that objects in free fall do not feel their own weight. You seem to be talking about something else entirely.
Or perhaps you are not including something. Equivalency is between SR and GR. Acceleration in SR and gravity in GR. Speed, time and distance are interrelated within a frame. When you measure the same speed of light in every frame the time, speed measured and distance measured are the same in every frame. This also has to do with equivalency. You said the same earlier in the thread. So for there to be a discrepancy in equivalency between SR and GR she would have to prove the electron and photon are not adjusted by the same amount to maintain the same observation of the SOL. Do you understand all of the ramifications of equivalency?

Timey needs to prove the observations of the SOL of every frame is not the same. Just like a sentence in a paragraph you can not pick words that change the meaning of the paragraph. If she understood enough about the entire equivalence issue she would understand where her objections are incorrect. I cannot help her frustration trying to change the physical nature of the universe. She is correct about the subjective reasoning of main stream. But not about the equivalency which is much more than free fall.
Title: Re: Is there a discrepancy with the equivalence principle?
Post by: timey on 01/03/2017 15:37:53
The observation of red shift could just as easily be attributed to the light having taken a longer amount of time to travel, than distance/c, which is exactly what the velocity related interpretation is saying...
And therefore if one takes the 'speed' attributed to a red shift, and the 'distance' to the light source, and says:
distance/speed=time*c=distance/c=time
...one can know by how much of a longer time the light took to travel, and that will be by how much time has slowed in 'open space' due to a gravitational deceleration caused by the weaker g-fields of open space.

Anyway I OFFICIALLY give up.

It is the mark of an educated person to be able to consider an idea that they do not believe in themselves, and physics is not a belief, it's a system of mathematics based on interpretation of observation.

Anything can be calculated, but I dare say that it's too much for any mathematician who may be among the thousands and thousands of people who have been viewing my many threads.

Like everything else in my life this is done - finished - and OVER.
I cannot carry on anymore.
Title: Re: Is there a discrepancy with the equivalence principle?
Post by: Mike Gale on 02/03/2017 05:56:15
Don't despair. It is the quest for knowledge that is exciting, not the knowing. Feynman himself said he was content to die not knowing all the answers. He just enjoyed discovering things for himself.
My last post was directed at GoC by the way and I didn't understand anything he said in Reply #169. It would be nice to know peoples' credentials so we can tell if they're talking from a position of authority. I have a master's degree in engineering physics and no formal training in GR.
Title: Re: Is there a discrepancy with the equivalence principle?
Post by: GoC on 02/03/2017 11:02:16
I am a bum on the streets can't you tell?

Authority is an illusion in theoretical issues involved with the universe. I started out in engineering and switched to chemistry. My father was interested in quantum mechanics and we discussed it going to and from DuPont together. That was where my interest began. The past 15 years of my life I have had an obsession with relativity learning all I could. Many times I thought relativity was just plain wrong. But each time I found the path to show relativity to be correct. Equivalency was the most interesting and the same time most difficult between GR and SR. Einstein could not explain this well for GR. SR is simple angles in geometry of motion.
   My biggest obstacle was to discard the subjective opinions from the past so called authorities. No one is an authority of the universe. It is all speculation. I do consider myself to have a fair amount of intelligence but not exceptional. I learned relativity by math, motion, clock tick rate and as energy related to c. One more postulate I used was the cycle rate of the electron. Whatever that actually might be. At c there is no energy for cycling of the electron. From there I related relativity to fundamental energy c. That was my breakthrough for understanding relativity. The math is taught by the "authorities" and that is the easy part. How do you understand dilation in GR when the authorities claim there is nothing to dilate. As an engineering student you must have understood mechanics was always involved in all aspects of motion. Electrons do not move themselves. If you learned anything in engineering nothing can only create nothing. The electron and the photon measure the same distance in every frame (equivalency) between SR (photon) and GR (electron). The authorities will grant that to be true. Physics are the same in every frame. Tick rate of a clock is not the same in every frame. Energy c and distance c measure total available energy. The clock tick rate measures the energy of a frame by the distance between ticks.
Dilation in GR is an increase in distance for energy. Less energy per volume of space. This is engineering logic and needs no authority. The "authorities" will tell you there is nothing out there to dilate. That's when I think what moves the electrons? Mathematicians can violate mechanics but how can a mechanical engineer violate mechanics? All energy (QM) comes from c of space and mass occupies space within the sea of energy. The electrons are a conduit for that energy. Mechanics of mass is based on kinetic energy supplied by fundamental energy that move the electrons. You have that or magic. As an engineer I would hope you chose mechanics over magic. E= kinetic mass c * fundamental c. Without the energy for motion there would be no motion (QM).

Although I ended up in pharmaceuticals as an analytical supervisor my passion has been analyzing relativity.
Title: Re: Is there a discrepancy with the equivalence principle?
Post by: Mike Gale on 02/03/2017 12:24:03
Thanks for that. I advocate disclosure of credentials not to brag, but rather to clarify the source of my lack of understanding. If I don't understand what you're talking about, there are two possibilities: (1) you are using technical jargon to describe a mainstream concept, in which case I can refer to the literature to get up to speed; or (2) you are conveying your own ideas in your own language, in which case you are my only source of information.
I think you (Goc) are confusing reference frames with frames of a movie (i.e. snapshots in time.) They are related concepts, but quite different. A reference frame in SR is a coordinate system that moves with respect to another with constant velocity. GR deals with special cases of accelerated motion, particularly those involving free fall and rotation (i.e. orbital motion.)  In that case, reference frames are snapshots in time.
Also, what you refer to as kinetic mass seems to be related to Newton's concept of inertial mass. I think you are suggesting that E=mc1c2 where c1 and c2 are two different speeds of light. I can't imagine what the physical justification for that would be, but maybe I don't understand what you're trying to convey.
Title: Re: Is there a discrepancy with the equivalence principle?
Post by: jeffreyH on 02/03/2017 13:22:48
What you are suggesting here is that there is an absolute background frame of reference. This cannot be reconciled with relativity.
Title: Re: Is there a discrepancy with the equivalence principle?
Post by: jeffreyH on 02/03/2017 13:33:34
The only way to proceed with your ideas is to start with SR and a sphere 1 light second in radius. Then consider the sphere to have photon emitters all around the surface set to fire all at the same time. Obviously just for this thought experiment. If they are all set to converge at the origin of the sphere then barring any unseen events they should all converge at the same time.
Then for GR do the same for a sphere near but no centred on a large mass. Now the photons will not all converge at once.

Title: Re: Is there a discrepancy with the equivalence principle?
Post by: GoC on 02/03/2017 16:09:42
What you are suggesting here is that there is an absolute background frame of reference. This cannot be reconciled with relativity.

Energy can be reconciled with Relativity being the very cause of Relativity. There is no absolute background!!!!! There is absolute c total energy available. The energy density is not constant but the energy c is constant. Equivalence proves c to be constant. Measurement by a clock measures the density of c in your frame.

The only way to proceed with your ideas is to start with SR and a sphere 1 light second in radius. Then consider the sphere to have photon emitters all around the surface set to fire all at the same time.


Relativity would negate that approach because of equivalence in SR and GR. How would you measure one second and where? Would you use light for measured distance to the center? If you did you already fouled your test. You cannot use the thing you are measuring for distance. Light cannot be part of the measurement.
 
Quote
Obviously just for this thought experiment. If they are all set to converge at the origin of the sphere then barring any unseen events they should all converge at the same time.
Simultaneity of relativity measurements will be an issue also.. This is a failed experiment before it even starts. Use a better design!!
 
Quote
Then for GR do the same for a sphere near but no centred on a large mass. Now the photons will not all converge at once.
If you measure the distance with light and then use light for that distance the thought experiment would work for being the same distance light would travel to the center.  But that would be ridicules. Uniform time is not possible in a dilated frame. Just like acceleration does not allow uniform time. Understand time before using time in an experiment.

Relativity will not allow the point you are trying to make. I understand your point.

You have been taught subjective ideas and you carry them with you into discussions of Relativity.

Main stream is in a maze not understanding their model hit a dead end. There is a mechanical reason for Relativity. Main stream threw away their tools so they have no ability to work on the mechanics.
Title: Re: Is there a discrepancy with the equivalence principle?
Post by: timey on 02/03/2017 19:36:45
Mike - please accept my apologies.  My outburst wasn't directed specifically at you.  It is born of sheer frustration that I cannot seem to get through to anyone concerning what I'm saying, and existence itself is not that appealing to me since I lost my horses and my way of life.

Ok - so you are an engineer first and foremost.  This is good, and I can work with you from the basis of an engineering point of view, and will elaborate.  I find your understanding of relativity to be very good, but if you want to get a really good grip on current state of physics, then I beseech you to read Lee Smolin's book 'The trouble with physics'.  Then we can talk from the basis of a professional theoretical physicist, and his professional equivalents observations of current physics and where relativity falls short.

Meanwhile, I have been discussing Joe Newman's energy motor, posts 82, 83, and 87 here:

https://www.thenakedscientists.com/forum/index.php?topic=69522.50

Copy and paste back to my thread here if you can find any part of what I say there is something you can relate to, not to clog up James's thread.

*

Jeff - Mike, in his relativistic correction to the Swartschild metric is using GR time dilation as a means to variable speeds of light.
My theory attributes the speed of light as variably derived via a time dilation that gives physical cause for the acceleration of gravity.
Mike is using the PE value.
I am using the KE value, this being to describe the absolute reference frame.*  Where I then also use the PE value for GR time dilation for m only.
This ensures that m, and the location of space that m is in, are always the positive and negative of each other in relation to M, and therefore light, travelling at the speed of light via either the negative value rate of time, or the positive value rate of time will ALWAYS travel 299 792 458 metres from either point of view...
And also describes how m and M and light move through the absolute reference frame of open space.

Then SR time dilation is drawn on top of this picture to describe relative motion, for m and M's experience of their own time, *but there is no need to use the length contraction/dilation concept of SR, because this has already been taken care of as a temporal dilation in the absolute reference frame.

It's a different way of calculating the same observations.

*

GoC - I've read your 'define time' thread.  I kind of understand what you are talking about, but the basic physics references are lacking.  You make good points, but more from the philosophy based perspective.
If you want to talk on 'my' threads you 'must' speak from a physics based perspective.  I advise that you also read 'The Trouble with Physics' to understand where relativity falls short.
I don't mean to be horrible to you, but I am indeed trying to describe something extremely specific here that is important to me.
You are welcome to join in with regards to the topic, but please get up to speed on the subject matter, I have little patience otherwise.
Title: Re: Is there a discrepancy with the equivalence principle?
Post by: GoC on 03/03/2017 12:00:30
timey- Relativity does not fall short no matter what your patience level might be for it to fall short. Its always the understanding that falls short. Even with so called physicists.
Title: Re: Is there a discrepancy with the equivalence principle?
Post by: timey on 03/03/2017 12:57:15
timey- Relativity does not fall short no matter what your patience level might be for it to fall short. Its always the understanding that falls short. Even with so called physicists.

In that case you have no business posting on this thread.

This is the type of post that makes me feel like deleting my profile here!

Title: Re: Is there a discrepancy with the equivalence principle?
Post by: Mike Gale on 03/03/2017 12:59:09
I still don't have the gist of Goc's theory, but I think timey's theory involves an absolute reference frame, which she calls the reference frame of free space. The problem with that approach is that motion is relative. If you and I are moving past one another in free space, far removed from everything else, we have no point of reference to determine which of us is in motion and which of us is stationary. Even Galileo would have to admit that the answer is observer dependent. Maxwell proposed some kind of average of the fixed stars, but there's no way to know how many stars are currently beyond one's visible horizon in any given direction and as they become visible over time, I may see more or fewer than you do, depending on our vantage points. That makes it observer dependent again. There's just no way to pin it down.
Having said that, there is in fact a preferred reference frame in a GR scenario, specifically that of the gravitating mass. In that sense, timey is correct.
Title: Re: Is there a discrepancy with the equivalence principle?
Post by: Mike Gale on 03/03/2017 13:01:42
timey- Relativity does not fall short no matter what your patience level might be for it to fall short. Its always the understanding that falls short. Even with so called physicists.
This is the type of post that makes me feel like deleting my profile here!
You have already provided an authoritative reference to debunk this opinion.
Title: Re: Is there a discrepancy with the equivalence principle?
Post by: timey on 03/03/2017 13:20:12
I still don't have the gist of Goc's theory, but I think timey's theory involves an absolute reference frame, which she calls the reference frame of free space. The problem with that approach is that motion is relative. If you and I are moving past one another in free space, far removed from everything else, we have no point of reference to determine which of us is in motion and which of us is stationary. Even Galileo would have to admit that the answer is observer dependent. Maxwell proposed some kind of average of the fixed stars, but there's no way to know how many stars are currently beyond our visible horizon in any given direction and I may see more or fewer than you do, depending on our vantage points, and that makes it observer dependent again. There's just no way to pin it down.

Oh thank goodness. A post I can respond to. :)

Mike - I do get what you are saying, but you present an unlikely if not impossible scenario.  If you or I were moving in space relative to each other, we would have both arrived at our positions 'from' somewhere.
Unless we were extremely remiss, or unaccountably forgetful we would both know where that somewhere was that we had departed from, and be able to work out where we were in relation to each other.
If we were able to observe each other for an observer dependent perspective, we would be in close proximity to each other, and even if we could not observe each other, being in radio contact with each other, we could work out our relative speeds via both of our g-field trajectories in much the same way as all space trajectories are calculated.

Therefore by knowing where one is in the g-field, and understanding how M is moving to create the g-field, if there is no actual dilation going on in spatial terms and the metre always remains constant, then our relative speeds are now easily definable.
Title: Re: Is there a discrepancy with the equivalence principle?
Post by: GoC on 03/03/2017 15:06:26
I still don't have the gist of Goc's theory, but I think timey's theory involves an absolute reference frame, which she calls the reference frame of free space. The problem with that approach is that motion is relative. If you and I are moving past one another in free space, far removed from everything else, we have no point of reference to determine which of us is in motion and which of us is stationary. Even Galileo would have to admit that the answer is observer dependent. Maxwell proposed some kind of average of the fixed stars, but there's no way to know how many stars are currently beyond our visible horizon in any given direction and I may see more or fewer than you do, depending on our vantage points, and that makes it observer dependent again. There's just no way to pin it down.

Oh thank goodness. A post I can respond to. :)

Mike - I do get what you are saying, but you present an unlikely if not impossible scenario.  If you or I were moving in space relative to each other, we would have both arrived at our positions 'from' somewhere.
Unless we were extremely remiss, or unaccountably forgetful we would both know where that somewhere was that we had departed from, and be able to work out where we were in relation to each other.
If we were able to observe each other for an observer dependent perspective, we would be in close proximity to each other, and even if we could not observe each other, being in radio contact with each other, we could work out our relative speeds via both of our g-field trajectories in much the same way as all space trajectories are calculated.

Therefore by knowing where one is in the g-field, and understanding how M is moving to create the g-field, if there is no actual dilation going on in spatial terms and the metre always remains constant, then our relative speeds are now easily definable.
My business in the threads is to challenge ideas to either make them stronger or weaker.

I do not think you actually understand what Mike is telling you. You read his words but not his understanding.

Somewhere is always moving in space and the meter never remains constant. The meter is observer dependent. A meter is a meter in every frame but not between frames. Here I am challenging your use of the word meter and its limitations of use in our understanding of relativity. There is no meter we can use as a reference. There is no position in space we can use as stationary for a reference. We cannot know which one of us are moving the most compered to the other. To bring it home there is no rest frame.

If you do not want to have your ideas challenged by relativity all you have is faith. Science is not about faith. Once you start with the equivalency principle there are many obstacles of math and observation you have to overcome to challenge relativity's explanation. Relativity can be explained as energy, motion or time. Mathematically they line up perfectly with each other. This is what you have to overcome. I chose energy to challenge your ideas because that is where you are challenging Relativity.
Title: Re: Is there a discrepancy with the equivalence principle?
Post by: timey on 03/03/2017 15:58:19
I still don't have the gist of Goc's theory, but I think timey's theory involves an absolute reference frame, which she calls the reference frame of free space. The problem with that approach is that motion is relative. If you and I are moving past one another in free space, far removed from everything else, we have no point of reference to determine which of us is in motion and which of us is stationary. Even Galileo would have to admit that the answer is observer dependent. Maxwell proposed some kind of average of the fixed stars, but there's no way to know how many stars are currently beyond our visible horizon in any given direction and I may see more or fewer than you do, depending on our vantage points, and that makes it observer dependent again. There's just no way to pin it down.

Oh thank goodness. A post I can respond to. :)

Mike - I do get what you are saying, but you present an unlikely if not impossible scenario.  If you or I were moving in space relative to each other, we would have both arrived at our positions 'from' somewhere.
Unless we were extremely remiss, or unaccountably forgetful we would both know where that somewhere was that we had departed from, and be able to work out where we were in relation to each other.
If we were able to observe each other for an observer dependent perspective, we would be in close proximity to each other, and even if we could not observe each other, being in radio contact with each other, we could work out our relative speeds via both of our g-field trajectories in much the same way as all space trajectories are calculated.

Therefore by knowing where one is in the g-field, and understanding how M is moving to create the g-field, if there is no actual dilation going on in spatial terms and the metre always remains constant, then our relative speeds are now easily definable.

Mike - I wrote the text below to Jeff earlier on in the thread...  In vain no doubt as he only makes seagul posts where he does not respond, nor probably even read the replies...

But:

"Jeff - Mike, in his relativistic correction to the Swartschild metric is using GR time dilation as a means to variable speeds of light.
My theory attributes the speed of light as variably derived via a time dilation that gives physical cause for the acceleration of gravity.
Mike is using the PE value.
I am using the KE value, this being to describe the absolute reference frame.*  Where I then also use the PE value for GR time dilation for m only.
This ensures that m, and the location of space that m is in, are always the positive and negative of each other in relation to M, and therefore light, travelling at the speed of light via either the negative value rate of time, or the positive value rate of time will ALWAYS travel 299 792 458 metres from either point of view...
And also describes how m and M and light move through the absolute reference frame of open space.

Then SR time dilation is drawn on top of this picture to describe relative motion, for m and M's experience of their own time, *but there is no need to use the length contraction/dilation concept of SR, because this has already been taken care of as a temporal dilation in the absolute reference frame."

Therefore one can know exactly where one is in the g-field in relation to anything else,

(btw, I am ignoring GoC from now on)
Title: Re: Is there a discrepancy with the equivalence principle?
Post by: Mike Gale on 04/03/2017 01:40:55
If you or I were moving in space relative to each other, we would have both arrived at our positions 'from' somewhere. Unless we were extremely remiss, or unaccountably forgetful we would both know where that somewhere was that we had departed from, and be able to work out where we were in relation to each other.
Exactly. "Somewhere" is the missing point of reference. Without that third point in space, we can't tell who's moving and who's not. Even if you add a third party, all we can say is that one of us (or both of us) is in motion with respect that observer. Note also that the concept of rotation doesn't make sense if you and I are alone in the universe. All of this comes from Galilean relativity. SR is the next logical step if you invoke conservation of (kinetic) energy.
Title: Re: Is there a discrepancy with the equivalence principle?
Post by: timey on 04/03/2017 02:03:50
The point I'm making is that in reality, that 'somewhere', the third reference point 'always' exists.
It is only in a hypothetical scenario that it doesn't.  So why bother with it at-all?

One must use triangulation wherever one is.  It's just that in space stuff gets fuzzy because of time shift.  And with SR it gets fuzzy because a spatial shift.  Overlaying one on top of the other gets complicated, as the time shift coordinates change with spatial dilation.

In my model the spatial dilation is taken care of as a temporal dilation that gives physical cause for the acceleration of gravity.  This renders your g-field coordinates as static, well not quite because M is moving in relation to m, but it is now possible to triangulate taking only these two opposing time shifts into consideration.
And SR time dilation and conservation of kinetic energy then becomes a very interesting prospect indeed!
Title: Re: Is there a discrepancy with the equivalence principle?
Post by: Mike Gale on 04/03/2017 07:07:09
Building blocks. You have to understand the 2-body problem (SR) before you can make any sense of the 3-body problem (GR.) You can't do an end run around SR and jump straight into GR because the latter is based on the former.
BTW - I downloaded Smolin's book. Looks like a good read (thanks for that), but from the TOC and Intro, it is clear that he is well beyond SR and GR and deep into string theory. The theme seems to be that, despite standing on the shoulders of giants, we have made scant little progress in theoretical physics during the last 4 decades. It must have been published before the vindication of Higgs though because that was pretty good.
Title: Re: Is there a discrepancy with the equivalence principle?
Post by: Mike Gale on 04/03/2017 07:37:41
I don't contest the idea that GR can be interpreted as dilation of time in rigid space, but I do contest the idea that GR time dilation is a substitute for SR. They are complementary effects. GR dilation is due to PE. SR dilation is due to KE.
Title: Re: Is there a discrepancy with the equivalence principle?
Post by: timey on 04/03/2017 14:44:51
But I am not suggesting that one consider that GR time dilation is a substitute for SR...
What I am suggesting is that there is a 'third' phenomenon of time dilation inherent to the g-field itself, exactly and equal to GR time dilation at h from M, but negative in value.  And that this causes the gravitational acceleration that causes SR time dilation.  And that under this remit there is no need to add the spatial dilation that is the resulting companion of the maths of SR.  This being because the third time dilation of the g-field has already added this effect as a temporal dilation of space to replace the SR spatial description, and that this suggested temporal description can equally describe the observed curvature of space.

Furthermore, I am suggesting that the current maths can be considered to be already calculate this acceleration caused by the third time dilation in the use of G and g.

As G and g do not have physical cause in current physics, all I have done is give physical cause to mathematics that are already being used, and in that the curvature of space is now represented as a temporal representation, the spatial geometry is back to Newtonian, and the right hand triangle is now not extended as a spatial extension.

It's a differing means of describing observation, that does not require dark energy, or dark matter, to balance the books.
Title: Re: Is there a discrepancy with the equivalence principle?
Post by: Mike Gale on 05/03/2017 01:08:08
I don't get it. If the 3rd dilation cancels GR dilation, there is no PE. That's just the free fall reference frame, which is fully described by SR.
Title: Re: Is there a discrepancy with the equivalence principle?
Post by: timey on 05/03/2017 02:05:50
Ah - but this third time dilation for open space doesn't cancel out GR time dilation...
What it does is separate the concept of m at h from M, from the concept of h from M, where h from M is inherent with open space time dilation that m at h from M experiencing GR time dilation, is in or moving through...
With the third time dilation of open space being the cause of any change in gravitational acceleration or deceleration that m experiences at h from M.

This provides another description of free fall that gives physical cause the the description of the acceleration of gravity, and also gives a physical cause for the curvature of space as a 'temporal' dilation.

Then SR time dilation becomes a description of how motion will 'further' affect the value of GR time dilation for m and M.

Mass experiences the positive value of GR time dilation, and the effects of motion related SR dilation, while moving through the negative value of the third time dilation of open space.

And big bodies of mass, and galaxy clusters will be changing the rate of the negative value of the time dilation of open space 'positively' as they move through the negative value open space time dilation...
This concept is exactly synonymous to the GR remit of mass telling spacetime how to curve, and spacetime telling mass how to move.
Title: Re: Is there a discrepancy with the equivalence principle?
Post by: Mike Gale on 05/03/2017 11:58:34
It's not clear from your description whether the 3rd dilation cancels GR completely or only in part. Either way, it's an ad hoc factor unless you can justify it with a new or existing law of physics (e.g. conservation of energy or momentum.) In other words, how does one go about calculating it?
The distinction between m at h from M and nothing at h from M is moot because GR is formulated in terms of an infinitesimally small test mass. That means m=0 and volume=0 for all intents and purposes. Note that the coordinate system for the metric is the one in which the gravitating mass (M) is at rest. Proper time is local time. Proper space is more complicated, but Flamm's paraboloid is a good way to think about it. In that view, the local radial dimension is stretched compared to that perceived by a distant observer. Angles around the gravitating mass stay the same. (I contend that the stretch is due to SR, not GR, but I think I am alone in that belief so far.) Note also that SR does not depend on M or G or g or m except insofar as they determine free fall velocity.
Title: Re: Is there a discrepancy with the equivalence principle?
Post by: timey on 05/03/2017 14:54:54
Mike -the hypothetical third time dilation of open space that I suggest can be thought of as an aether type scenario that M and m are moving through.

You are right that GR time dilation will be affected by this third time dilation, but not in the sense of cancelling it out.  You are still thinking in terms of 2 dimensional mathematics.
The GR time dilation will be affected by the hypothetical third time dilation of open space for the reason that the value of M is creating the open space g-field, and the g-field is inherent with the third time dilation that runs counter directional in value to GR time dilation for m at h from M, where the GR time dilation of m at h from M is also related to the value of M.

These concepts can be calculated as a 3 way time dilation matrix annexed to a spacetime matrix, where ones space coordinates in the g field relate
a) to the g-field hypothetical time dilation of h from M
b) to the GR time dilation of m at h from M
c) to the SR time dilation experienced by m due to m's velocity at h from M
...and result a calculation for the time aspect of the spacetime matrix as sequential time.

a) will be causing the curvature of space as a temporal dilation, and m will move as to this curvature
b) will be the time m experiences at h from M
c) will cancel out the GR time dilation effects that m experiences at h from M.

Equivalence principle:
The speed of light is held constant at each coordinate at h from M via the length of second of the hypothetical third time dilation of open space.
Light will cover 299 792 458 metres held relative to the 'longer' second...
But - because the hypothetical time dilation is the negative value of GR time dilation - m at h from M will also observe that the light is travelling at 299 792 458 metres per the 'shorter' second experienced by m at h from M.
Now we implement the part 2 change to the equivalence principle which incorporates stating all m at h from M as gaining energy at gravity potential as an addition that 'doesn't' incorporate calculating in the value of m.  In other words, simply add the energy at gravity potential as a blanket addition, where all m is affected to that value equally.
Now all remains equivalent for all at any h from M.
One has a physical reason for the fact of a person ageing in keeping with their time dilated clock.
And this concept of a blanket addition of potential energy for m at h from M can be thought of as free fall upside down.

Lastly - one then must then use the speed of light value as per held relative to the hypothetical third time dilation timing in order to calculate SR time dilation for m in motion in a changing g-field.
This means that as m moves into g-field coordinates where the speed of light is held relative to the negative value 'longer' seconds, the constant speed that m is travelling at will become a greater percentage of the speed of light of that coordinate, and m will experience an escalation of the slowing of its own experience of time, and take a longer time to cover a metre.
Interestingly Mike, under this remit -if m then regulates its speed to remain a constant percentage of the speed of light as held relative to the increasingly longer seconds of the coordinates in the weaker g-field m is travelling through, ie: slows its speed down, SR time dilation effects will be lesser, and m will experience itself as covering a metre faster...
This being because the curvature of space is a temporal dilation where a metre always remains a constant metre, and curvature is simply a matter of there being longer seconds in the weaker g-field.*

We can then see that under this remit  of regulating SR time dilation effects via speed control, this can cause m to avoid some of the temporal dilation of space, and therefore avoid at least some of the curvature to get 'there' wherever that may be, quicker.

* If you really examine this structure, you will realise that it is just using the currents maths in a rearranged fashion.

(Even the maths for the concept of length contraction/dilation are used in my model, switched to a concept of an observational time frame dependency that insures the fact of a universal absolute 'now' that all coordinates have in common despite their rates of time differing)
Title: Re: Is there a discrepancy with the equivalence principle?
Post by: Mike Gale on 06/03/2017 00:03:55
Your recipe for the 3rd dilation is missing ingredients and quantities. On one hand you say it is "the negative value of GR time dilation". On the other you say it "doesn't cancel out GR time dilation." Isn't that contradictory?
Title: Re: Is there a discrepancy with the equivalence principle?
Post by: timey on 06/03/2017 01:13:46
Only if you are looking at GR time dilation being inherent to space at h from M, rather than to m at h from M.

I am making a clear separation of m at h from M, from the space of h at M, and stating that m at h from M is experiencing its own time, this being GR, and SR time dilation effects, while moving through a space inherent with a separate time dilation that affects how m moves through this space.

This is stating that both GR and SR time dilation effects occurring for m, and the hypothetical third time dilation occurring for space, are both occurring simultaneously as separate issues, where the value of GR time dilation, and the value of the hypothetical third time dilation both relate back to the value of M via the g-field, and the value of m's SR motion related time dilation effects also relate back to the value of M via the g-field.
Title: Re: Is there a discrepancy with the equivalence principle?
Post by: timey on 06/03/2017 06:09:35
As a foot note:

This structure is rendering the concept of time itself as a reactive phenomenon that is related to energy.
Title: Re: Is there a discrepancy with the equivalence principle?
Post by: Mike Gale on 06/03/2017 12:30:29
SR does not depend on M or m, although it does have implications for both (because they are observer dependent quantities.) There is no distinction between the field for m at h from M and that for nothing at h from M because the field is calculated under the premise that m is vanishingly small. It is certainly true that a finite value of m will alter spacetime in the same manner as M, but that is an unnecessary complication and teaches us nothing new. Note that SR and GR treat time as a function of energy.
Title: Re: Is there a discrepancy with the equivalence principle?
Post by: timey on 06/03/2017 13:51:44
What you say is correct in conventional relativity, no doubt about it, and I am not contending this as being the case in conventional relativity.

But...  What I am describing above is an altered relativity that describes my contracting cyclic model.

Yes, you are right, conventional relativity is based on the fact that energy and frequency are observer dependent...
My model differs in that while it holds that the observation of light is observer dependent because one can only see light when it arrives at ones location, it holds that the observation of an atom's energy and frequency, that is not at one's location, is not observer dependent but 'is' actually the differing frequency and energy that it is observed to have.

Yes you are correct in the fact that relativity does not consider space as anything but a distance, or fabric, that is extended in the right triangle.
My model differs in that it says that the distance of space itself is not spatially extended in the right triangle, but is instead temporally extended in the right triangle, and it uses the third time dilation of the g-field-field itself that applies to space 'only,' as a means of a physical cause for the acceleration of gravity in order to achieve this extension of the right triangle as a temporal phenomenon.

This structure that I have put forward can describe a a fully described model of the universe without adding any unknowns that are not observed, unites the standard model with gravity for a continuum in quantum, and gives mechanics that describe Big Bangs both before and after the fact.

Conventional relativity cannot do this.  Therefore I am looking for a mathematician who can create the mathematics to describe my model.
These mathematics will be of the same values as conventional relativity mathematics, but these values are employed in a differing alignment for alternate reasons...
One could consider that what I have done here by considering space as a temporal dilation is give complex numbers a physical reason for existing.  A physicality that ensures that one can calculate both the position and velocity of an electron simultaneously.

Please understand that I have made the changes from conventional relativity, that you note, purposefully and intentionally.  These are not misunderstandings of conventional relativity, but are indeed consciously derived alterations.
Title: Re: Is there a discrepancy with the equivalence principle?
Post by: Mike Gale on 07/03/2017 01:39:16
OK, but you still have to be accurate about what conventional theory does and doesn't do. And you have to demonstrate a need for the 3rd dilation. As far as I can tell, it's nothing more than a dodge around spatial dilation in SR or a re-interpretation of it. In any case, a mathematician is not going to be of any help unless you can specify a recipe for the 3rd dilation, along with ingredients and quantities.
Title: Re: Is there a discrepancy with the equivalence principle?
Post by: Mike Gale on 07/03/2017 12:52:07
My model differs in that while it holds that the observation of light is observer dependent because one can only see light when it arrives at ones location, it holds that the observation of an atom's energy and frequency, that is not at one's location, is not observer dependent but 'is' actually the differing frequency and energy that it is observed to have.
Are you proposing that a receding clock runs slower and an approaching clock runs faster, such that the round trip time is the same in both reference frames? It is possible to balance the space time books on that premise. The relative velocity of reference frames becomes observer dependent. Spatial dilation is more complicated because time passes at different rates in off-axis directions. The problem is that it is in stark disagreement with the extended longevity of muons in flight. It is also inconsistent with the Twins paradox, which is evidenced by GPS and (more directly) by flying atomic clocks on airplanes.
Title: Re: Is there a discrepancy with the equivalence principle?
Post by: timey on 07/03/2017 14:19:18
No - a clock or particle of mass will never be able to measure the hypothetical third time dilation because this time dilation is inherent to space only and the clock or particle will only ever itself experience a combination of GR coordinate time dilation, and SR motion time dilation.

For a clock or particle at rest with respect to the g-field, the third time dilation will have no effect whatsoever.  But if the clock or particle is not at rest with respect to the g-field, then the hypothetical third time dilation will affect how the clock or particle moves with respect to changes in the g-field.

The ingredients and quantities can be directly derived via the acceleration/deceleration of gravity, where additional kinetic energy then DOES NOT increase mass and mass value remains constant.
Where, (scratches head, as not sure and still thinking about it):
m/a = time dilation...
- to show direction of thoughts, but perhaps better derived:
p/a = time dilation (?)
Because
p = mv
and the hypothetical third time dilation is then an integral over force, causing acceleration...
Title: Re: Is there a discrepancy with the equivalence principle?
Post by: pasala on 12/03/2017 11:47:35
Friends,

I would like to add few lines, regarding my opinion on time dilation and c.

Well, it is true that time dilates for two reasons gravity and velocity. 

Gravitational time dilation is an actual difference of elapsed time between two events as measured by observers differently situated from gravitational masses, in regions of different gravitational potential.  As per Einstein huge masses such as Earth also influences tick rate of the clock.  Here it is wonder to me, it is not for criticising, but to discuss facts only, how huge masses influences clock.  As per Newton's inverse square law, gravity mainly depends on the size of mass.  But as per Einstein, uneven distribution of matter and energy in space time plays key role.  Actually, Einstein invented that gravity influences our clocks.

Ok, here the tick rate of clock is influenced by the gravity.

As per the theory proposed by me 'what exactly is gravity' it is the huge energy, which i termed it as 'energy base' is playing key role in gravity.  This potential energy is turning out as kinetic energy due to the pressure/force of hydrosphere.  Gravity on any planet mainly depends on the 'standard energy' of that planet. Gravity strength at any place mainly depends on the gap between Earth and hydrosphere. However it is also influenced by the energy dilated by Earth which works as anti gravity waves. 

The other important reason why time dilates is velocity.  In the case of gravity, the clock is influenced by the variations in the gravity.  It's ok.  But here, in the case of velocity, the source or how and what exactly is influencing the functioning of clock is not known. 

There is a big gap here, it is true that if the velocity increases, the ticking rate of clock decreases and of course it is proved.  But what exactly is influencing the clock is not known.    In the case of gravity, time is dilated due to variations in the gravity.  Here, if the velocity increases what happens to clock, and why it dilates.  Ok, we are using cesium clock, and there is no mechanical fault, still time dilates. 

Ok, in the case of gravity, it is not the fault of clock used, but it is due to gravity.  In the case  of  velocity also, there is no fault of the clock used by us, but some thing outside force is influencing the clock.  Here velocity is not an object or thing and some thing is influencing our clocks.  When the velocity increases, it is starting influencing our clocks. 

As said by timey, a clock dilating on the head of a person, never dilates age of that person. 

There is every need for us to re-think, 'what exactly is time dilation'.  Our clocks shows only elapsed time due to gravity or velocity.  In fact our clocks have no capacity to dilate the universal time.  They got to be adjusted to universal time.  It is the universal time which is standard time.

Universal time can be dilated due to various reasons.  Suppose for any reason, sun started dilating huge energy into the open area.  EMF force against the Earth increases and Earth start rotating speedily, completes sun rise and sun set within 11.55 minutes, thereby dilating 5 minutes.  In other words, in case if the Sun started dilating small amount of energy, EMF decreases and rotation of Earth slows down and sun rise and sun set will be completed by 12.05, thereby by dilating 5 minutes.  Universal time also depends on the gravity of a planet.  Suppose if the gravity of a planet weakens, sun rise and sun set also slows down.

Speed of light:
There are number of invisible things influencing speed of light.  At present we are taking two mirrors, focusing light on them and measuring the time taken to travel.

But we are feeling that outside is empty and that fresh ray is created each time.  This is not at all correct and no fresh ray is created. 

Ok, let us study it with simple example.  Let us assume that there is a water pipe and it is completely filled with water.  When the motor is switched on, fresh water coming from the well pushes the water at the beginning of the pipe and due to force/pressure water start flowing.  Here, suppose water pipe is empty,  then fresh water first accumulates in the pipe and than start flowing.  At present we are taking that water pipe is empty and it is being filled each time and than flowing out. 

This is incorrect.

In the case of our universe also, it is not empty and it is not being filled by the Sun energy.  In fact Sun energy has no such capacity.  Due to Big Bang or whatever it may, all the planets have dilated huge fundamental energy and it led to formation of  'stock of fundamental energy'.  Here on this Earth, due to formation of climate 'Energy Base' is developed and it is existing with lot of pressure. 

When we switch on the torch light, very small amount of energy, potential energy stored in the battery cell has no capacity to create a Electro magnetic force.  It simply creates a force/pressure on the existing energy ray and flow of electrons propogates light.  In the example, water pipe is not empty, when the fresh water joins, a force/pressure is created and the water at the end moves out.  This is the way and how  light travels from one place to other place.  Propagation of light at any place mainly depends on the outside energy. 

Suppose if we are on the moon, where climate is weak, naturally energy base is also weak and there fore propagation of light is weak.  Well, for that on the Earth also light varies according to gravity. 

We have tied science to certain limitations.  It is like an Elephant trainer, who ties the Elephant with a simple chain.  When compared to the strength of the Elephant this chain is nothing.  But Elephant feels that it cannot and remains within the boundary.  If what exactly is gravity is solved, all these issued will be solved and science will take a straight line of research.

Yours
Psreddy
Title: Re: Is there a discrepancy with the equivalence principle?
Post by: timey on 12/03/2017 14:45:34
This thread is exploring the possibility that potential energy is related to GR time dilation, and the possibility of a third time dilation existing that applies only to the g-field itself, where the third time dilation is an integral over force and is the cause of gravitational acceleration.
Title: Re: Is there a discrepancy with the equivalence principle?
Post by: timey on 12/03/2017 21:55:13
Hey Mike - shiiiiit, I just had a realisation...
Firstly, my apologies because you were right...and I was wrong.  Einstein did have to add his lambada constant to stop the universe expanding, not contracting as I said 'in error' however many posts ago.
I was muddling my own thinking somehow with the facts, but it is from realising my error that I think I've finally figured out how to go about calculating my contracting model.

I find formula easier to understand than numerical calculations because the relationships are proportional, however my understanding of symbols is lacking.  I wonder, would you help me out...?

Could you please help me run through the symbols of these maths.

G*u*v + lambada*u*v = -8*pi*G/? Tuv

G, v, and pi are clear.  Lambada value is 0.5, but could you please give me a physical description of u, which I think means 'change' and what is the symbol that 8*pi*G is divided by?  I've used a question mark, but the symbol looks like a question mark upside down.  What does it mean?
And could you clarify for me what T is symbolising here?
Title: Re: Is there a discrepancy with the equivalence principle?
Post by: Mike Gale on 12/03/2017 22:37:31
Looks like a modified version of the Einstein field equation. The variables u and v are subscripted indices. They refer to rows and columns of a 2D tensor, which is just a fancy name for a matrix in this context. Summation over u and v is implied. (That shorthand was invented by Einstein.) Each index ranges from 1 to 4, corresponding to the 4 dimensions of spacetime. G with no subscripts is likely Newton's constant. G with subscripts is an element of the Einstein tensor, which is an expression of the curvature of spacetime. Tuv is an element of the energy tensor. I'm not sure what lambdauv is. Probably a proposed extension to the cosmological constant. The funny symbol (?) would have to be c4. Probably a malfunction in the equation renderer. Note that Tuv belongs in the numerator, not the denominator.
Title: Re: Is there a discrepancy with the equivalence principle?
Post by: timey on 12/03/2017 23:12:56
Lambda*u*v as I understand it is the cosmological constant that Einstein added to his equation, and then retracted in light of Hubble's velocity related interpretation of the red shift distance correlation.

As to the rest of what you said, I'll have to get myself some translations off the net for some of it.
I'll be back after some thinking time passes.

Thanks Mike!
Title: Re: Is there a discrepancy with the equivalence principle?
Post by: timey on 13/03/2017 02:22:43
Ok - Einstein added the cosmological constant Lambada*u*v to his equation to stop the universe from expanding...

Einstein retracted his cosmological constant that stopped his rendition of GR expanding the universe when Hubble's velocity related interpretation of the red shift distance correlation gave cause for the phenomenon of expansion.

In that my model re-interprets Hubble's velocity related red shift distance correlation interpretation as being a time dilation related red shift distance correlation via a third time dilation to give physical cause for the acceleration of gravity, the spatial dimensions are now back to being Newtonian, and the curvature of space is now inherent to the open space g-field via this third time dilation.

The first expression of Einstein's equation G*u*v, and the 8*pi*G, I think can already be describing this third time dilation because of the use of G.

So without Hubble's velocity related interpretation, and without the cosmological constant, Einstein's equation leaves us with a universe that is still expanding.
Add the constant back into the equation and we have a steady state.

Physicists describe how Einstein's cosmological constant may be relevant in value as describing the acceleration of the universe via Dark Energy, but as Einstein was using the constant to stop the universe from expanding, how can this constant be used to describe an accelerated expansion?
To describe an accelerated expansion by using the value of Einstein's constant, are they considering the constant in the negative?

Like this:
Guv - lambada*u*v = - 8*pi*G/c^4 Tuv
Title: Re: Is there a discrepancy with the equivalence principle?
Post by: Mike Gale on 13/03/2017 03:28:37
Ah yes. I didn't notice the missing symbol in the 2nd term. It should be Lambda * guv, not Lambdauv. Lambda is Einstein's cosmological constant. (He called it cosmological because it is the same at every point in space and constant because it doesn't change over time.) guv is an element of the metric tensor, which is the curved space extension of Pythagorus. The metric tensor is also part of the Einstein tensor.
Don't confuse G with Guv. They are completely different objects. One is a scalar (i.e. a 1 dimensional matrix with 1 element.) The other is an element of a 2D matrix (comprising 4 rows and 4 columns) and usually represents a function involving several variables. Same goes for g and guv.
Note that the T in Tuv is shorthand for total energy.
Title: Re: Is there a discrepancy with the equivalence principle?
Post by: timey on 13/03/2017 03:56:31
I just lifted the equation as written in hand writing off a horizon program about about Dark Energy, supposedly as Einstien wrote it.
I daresay that the equation has been reduced to a minimal expression as per esthetics.  Less is more beautiful so I hear, which is why everyone fell in love with the Dirac equation...

Are you sure about the extra g?
In the program they were quite clear about how Einstien was annoyed because of having to add the middle term that includes lambada.  It had been simply:

Guv = 8+pi*G/c4 Tuv

And Einstien shook Hubble's hand in relief that he could ditch the lambada*u*v, although admittedly it may be my mistake and that I copied the equation down wrong.  Not impossible at-all.

Oh - and when you said numerator and denominator, which bit of the equation were you referring to please?

P.S.  When saying that G 'can' already be describing my model's third time dilation this is only because G is the gravitational constant.  Surely any maths that use G are juggling that constant?
Title: Re: Is there a discrepancy with the equivalence principle?
Post by: Mike Gale on 13/03/2017 04:02:54
A positive value for the cosmological constant makes the universe expand over time. A negative value makes it collapse. Zero corresponds to a static universe.
Title: Re: Is there a discrepancy with the equivalence principle?
Post by: Mike Gale on 13/03/2017 04:04:54
If Lambda = 0 then Guv = (8*pi*G/c4) * Tuv.
Title: Re: Is there a discrepancy with the equivalence principle?
Post by: Mike Gale on 13/03/2017 04:11:34
You might be right about how Lambda affects expansion and contraction. A positive value is likely necessary to stop the expansion. Otherwise it wouldn't make sense that Einstein abandoned the idea (by setting Lambda=0) when Hubble discovered that the universe was indeed expanding. In that case, a negative value would make it expand faster. A positive value would slow the expansion and a larger positive value would make it collapse.
Title: Re: Is there a discrepancy with the equivalence principle?
Post by: timey on 13/03/2017 04:13:45
But that surely is on the basis of Hubble's velocity related red shift distance correlation.
Einstien was able to remove the constant in the face of Hubble's discovery because Hubble had provided a cause for expansion, but had to relinquish his steady state preconceptions.
Therefore the equation without the constant describes expansion, and without the constant this would be 0 right?

So - when looking at the value of Einstiens constant to describe the acceleration of this expansion, you can't add the constant to describe this acceleration, adding it to the equation causes a steady state, so it must surely be subtracted... No?



Ah yes - just saw your post.  Exactly, so subtracting it would cause acceleration
Title: Re: Is there a discrepancy with the equivalence principle?
Post by: Mike Gale on 13/03/2017 04:15:28
Hubble didn't find a cause, just evidence. But otherwise I think you are correct. See my edit to my last comment.
Title: Re: Is there a discrepancy with the equivalence principle?
Post by: Mike Gale on 13/03/2017 04:17:11
I don't think you can get accelerated expansion out of a constant though. You'd have to change it to a variable of some sort.
Title: Re: Is there a discrepancy with the equivalence principle?
Post by: timey on 13/03/2017 04:30:12
The expansion of the universe without the Dark Energy notion wasn't thought to be a constant speed.  It was thought to be slowing down.
However - you do have a point there...

I wrote this before seeing your post:
So in order to describe a contracting universe it would be necessary to add the constant twice right?
But would that be the whole term Lambada*u*v, or Lambada*g*u*v as you say, or would it be a case of doubling the Lambada value?
The Lambada value is 0.5 so doubling that would be in as much as saying +1*u*v or +1*g*u*v (I'll have to look at the program again tomorrow to see if I wrote it down wrong)

if Einstein's equation minus the constant caused an expanding universe, there must be a rate to this expansion within the maths of the equation surely?

And... If the constant stops this rate of expansion, surely there must be an equal and equalising negative rate within that constant?
Title: Re: Is there a discrepancy with the equivalence principle?
Post by: Colin2B on 13/03/2017 08:41:51
I don't think you can get accelerated expansion out of a constant though. You'd have to change it to a variable of some sort.
I assume you are familiar with ρvac = ρΛ   ≡ Λ/8πG


Sorry, equation fn on here isn't working yet so bit laborious to type out with alt codes.
Didn't mean to interrupt your discussion, I'll leave you to it.

Title: Re: Is there a discrepancy with the equivalence principle?
Post by: jeffreyH on 13/03/2017 13:36:08
Combining the moment of inertia tensor with the stress energy momentum tensor is fun.
Title: Re: Is there a discrepancy with the equivalence principle?
Post by: timey on 13/03/2017 13:55:50
Ah - Colin, well firstly so say so, it's starting to look like I may have to eat my hat on YouTube for you concerning G and g, but I do stand firmly by my position on the marks, in the context that I was using them.

Mike of course will no doubt recognise the maths of your interjection to understand the context, but I on the other hand could use a few words of explanation to ensure that my guess work is correct.

This is relevant to Einstein's constant as a variable, and p is what renders variability?

Jeff - Yes, thanks for that!  I'm sure it is fun for those in the know.  So what moment of inertia do you refer to, and is that relevant to Einstein's constant in relation to Dark Energy? Enigmatic-ism's aren't particularly the remit of being informative I'll have you know. 
Title: Re: Is there a discrepancy with the equivalence principle?
Post by: timey on 13/03/2017 13:57:03
Hubble didn't find a cause, just evidence. But otherwise I think you are correct. See my edit to my last comment.

Mike - 'evidence' is a bit of a strong term for velocity related red shift interpretation.

Evidence was found to support a correlation between red shift and distance, but there is no evidence to support the velocity related interpretation. The 'speed' of a red shift could just as easily be interpreted as a 'rate of time'.
Title: Re: Is there a discrepancy with the equivalence principle?
Post by: pasala on 13/03/2017 15:25:50
Mr. timey
Well, about dark energy:
Dark energy is unknown form of energy presumed to be exist througt the space and  It is also said to be causing expansion of universe.  Dark energy is  homogeneous, not very dense and is not known to interact through any of the fundamental forces other than gravity. 

Dark energy is said to be slowing down.

Do dark energy causing expansion of universe:

Dark energy is also known as vacuum energy.  Do this vacuum is having power. 

Ok, let us study it with simple example, basing on which our predecessors estimated strength of  dark energy.

Let us take a plastic bucket filled with water.  Take another plastic bucket, close the first one using this bucket. Now create vacuum in between these two buckets.  Water in the bucket start raising.  It is said when vacuum is created, dark energy started creating pressure on the water.

This is incorrect.  When vacuum is created, pressure on the water is lost and it is just relaxed and start raising.  Here, pressure in between two buckets is low, but pressure inside water is high and it tries to occupy the gap.

Here power of vacuum is possible only when:
01  Pressure outside is high

Actually it is not the power or strength of the Dark energy but outside energy.

So, naturally Dark energy has no role in the expansion or contraction of universe.

Yours
Psreddy
Title: Re: Is there a discrepancy with the equivalence principle?
Post by: Colin2B on 13/03/2017 21:50:10
 
Ah - Colin, well firstly so say so, it's starting to look like I may have to eat my hat on YouTube for you concerning G and g, but I do stand firmly by my position on the marks, in the context that I was using them.
I'm not foolish enough to get drawn into this discussion, but as you (I hope) persevere with learning more maths you will find that more of your views change with an understanding of what I was saying.

 
This is relevant to Einstein's constant as a variable, and p is what renders variability?
A constant is a constant not a variable, but you have to determine the correct value of the constant and equivalences like this can help. However, you are dropping in to the borderland between standard model and GR and one of the more interesting areas of speculation. I'm sure you are already aware of the quantum fluctuation problem and it's link to dark energy, the cosmological constant, etc and no doubt Mike will be happy to discuss it.
 
Just a final comment. As I look at Alan's recent posts I note you commented:
Yet physics states the frequency of the clock observed in the other gravity potential as observer dependent, and that if one places oneself with the clock in the other gravity potential that this observation disappears like some sort of mirage and the clock in the other gravity potential is ticking normally.
The observation doesn't just disappear like a mirage. Last year I took the trouble to write out a simplified explanation of how this works. I note that Pete has also linked to an explanation using a diagrammatic approach, but he also seems convinced you haven't read it.
I'm not going to go over old ground & repeat myself here, because I think further discussion is unlikely to be productive. However, the effect is real but like many measurements eg speed, energy, momentum, it is dependant on where the measurement is being made.
Title: Re: Is there a discrepancy with the equivalence principle?
Post by: Mike Gale on 13/03/2017 23:04:09
Hubble didn't find a cause, just evidence. But otherwise I think you are correct. See my edit to my last comment.

Mike - 'evidence' is a bit of a strong term for velocity related red shift interpretation.

Evidence was found to support a correlation between red shift and distance, but there is no evidence to support the velocity related interpretation. The 'speed' of a red shift could just as easily be interpreted as a 'rate of time'.
My point was simply that Hubble made the measurements and noticed the correlation between distance and speed. He left it up to theorists (like Einstein) to explain the cause of that correlation.
Title: Re: Is there a discrepancy with the equivalence principle?
Post by: Mike Gale on 13/03/2017 23:12:30
I don't think you can get accelerated expansion out of a constant though. You'd have to change it to a variable of some sort.
I assume you are familiar with ρvac = ρΛ   ≡ Λ/8πG
Sorry, equation fn on here isn't working yet so bit laborious to type out with alt codes.
Didn't mean to interrupt your discussion, I'll leave you to it.
I am not. That would be the energy density of free space for c=1 I think. Nicely done though. I hadn't thought of using alt codes to render Greek letters.
Wikipedia says "A positive vacuum energy density resulting from a cosmological constant implies a negative pressure, and vice versa. If the energy density is positive, the associated negative pressure will drive an accelerated expansion of the universe, as observed." I speculated in a previous comment that the constant would have to be changed to a variable in order to account for accelerated expansion because that wasn't discovered until 1998. Why would Einstein drop the constant if GR predicts accelerated expansion and Hubble's data did not? The answer must be that Hubble did observe accelerated expansion and the new data just suggests a faster rate. The literature could be much clearer on this point, but Wikipedia also notes that the level of confidence in the 1998 analysis has dropped from 5-sigma to 3-sigma so the cosmological constant (and the dark energy from whence it is presumed to originate) may be doomed again.
Title: Re: Is there a discrepancy with the equivalence principle?
Post by: timey on 13/03/2017 23:15:31
Ah - Colin, well firstly so say so, it's starting to look like I may have to eat my hat on YouTube for you concerning G and g, but I do stand firmly by my position on the marks, in the context that I was using them.
I'm not foolish enough to get drawn into this discussion, but as you (I hope) persevere with learning more maths you will find that more of your views change with an understanding of what I was saying.

 
This is relevant to Einstein's constant as a variable, and p is what renders variability?
A constant is a constant not a variable, but you have to determine the correct value of the constant and equivalences like this can help. However, you are dropping in to the borderland between standard model and GR and one of the more interesting areas of speculation. I'm sure you are already aware of the quantum fluctuation problem and it's link to dark energy, the cosmological constant, etc and no doubt Mike will be happy to discuss it.
 
Just a final comment. As I look at Alan's recent posts I note you commented:
Yet physics states the frequency of the clock observed in the other gravity potential as observer dependent, and that if one places oneself with the clock in the other gravity potential that this observation disappears like some sort of mirage and the clock in the other gravity potential is ticking normally.
The observation doesn't just disappear like a mirage. Last year I took the trouble to write out a simplified explanation of how this works. I note that Pete has also linked to an explanation using a diagrammatic approach, but he also seems convinced you haven't read it.
I'm not going to go over old ground & repeat myself here, because I think further discussion is unlikely to be productive. However, the effect is real but like many measurements eg speed, energy, momentum, it is dependant on where the measurement is being made.


I wasn't trying to draw you into a conversation of such type.  Just an answer as to why you referred to the sum would have been fine.

Something along the lines of:
"Yes timey, if you add the equation I mention like this, for this reason, this resulting in maths that can describe an accelerative or decelerative aspect to adding, or subtracting the lambada constant equation to or from Einstein's equation"
...i.e. putting your input into context, would be just fine.


The only reason that I mentioned to you that my understanding had been altered is a) to try and break the ice a little, and b) because I realise that the system within the current maths by which my model can be calculated is much simpler than I first thought.
Edit: And it was in talking to Alan that I realised this actually..

I know very well that the cosmological constant is related to Dark Energy, and the standard model, and to quantum.
And... as my model, via a subtle change to the equivalence principle, unites the standard model with gravity for a continuum in quantum, I very much hope the conversation gets that far, where conversations with Mike are enjoyable because although he too is a fairly staunch relative fan, he actually thinks what I'm saying through and the conversation is progressive.

As to some of my conversations you refer to, I really do not need anybody to explain to me how conventional relativity views the situation.  I am saying to try viewing it a differing way.  And quoting current physics, saying that that is just relativity and pointing me to A-level physics, or making enigmatic remarks doesn't cut it for me, but maths explanations 'are' very helpful.
I already know that the results of viewing the situation in the way that conventional relativity does are that the standard model is not united with gravity, there is no formal theory on time, and physics cannot get behind the Big Bang or the event horizon of a black hole.
You may prefer to go down the observer dependent rabbit hole, and be my guest, but what happens when there is no observer?  sh1t still happens man!  And it's what's happening that interests me...
And in any case, the measuring of anywhere is entirely dependent on which time you hold measurement relative to.  Make measurements over there by over there's rate of time, all measurements remain the same.

Nobody is ever going to 'tell me' what to think under any circumstances, in any field.  I question everything without fail, think it through, make investigation...and hey, do you know what Colin, my investigations tell me that physics is looking for a theory that can unite the current physics.
Has this information reached your ears, Colin?

Or are you prejudice to the possibility that an uneducated non-mathematician is capable of inspired thought, and your commentary trending towards the notion that I will somehow be struck by the foolishness of my notions in the face of the infallible logic of current physics, and that having read many books by cutting edge professional physicists, this will occur due to commentary posted upon this site?

If so I will duly apologise...  I always do if I'm in the wrong.
Title: Re: Is there a discrepancy with the equivalence principle?
Post by: Mike Gale on 13/03/2017 23:35:02
The expansion of the universe without the Dark Energy notion wasn't thought to be a constant speed.  It was thought to be slowing down.
However - you do have a point there...

I wrote this before seeing your post:
So in order to describe a contracting universe it would be necessary to add the constant twice right?
But would that be the whole term Lambada*u*v, or Lambada*g*u*v as you say, or would it be a case of doubling the Lambada value?
The Lambada value is 0.5 so doubling that would be in as much as saying +1*u*v or +1*g*u*v (I'll have to look at the program again tomorrow to see if I wrote it down wrong)

if Einstein's equation minus the constant caused an expanding universe, there must be a rate to this expansion within the maths of the equation surely?

And... If the constant stops this rate of expansion, surely there must be an equal and equalising negative rate within that constant?
Wikipedia says Λ=1.19×10-52 m-2
Note that u and v are indices, not multiplicative factors.
Title: Re: Is there a discrepancy with the equivalence principle?
Post by: timey on 14/03/2017 00:04:10
Mike - a woman who's name I posted earlier this thread (edit: Henrietta Swan Leavitt, as she should be remembered), noticed that there was a correlation between red shift and distance.*  Needless to say a speed would not be noticeable due to the photographic nature of the observation, impossible to observe in real time anyway, and is indeed more to the point entirely experimentally unverifiable.

*Actually to be more precise Leavitt discovered the luminosity-time period to determine distance, and:
quote wiki:
Quote
After Leavitt's death, Edwin Hubble used the luminosity–period relation for Cepheids together with spectral shifts first measured by fellow astronomer Vesto Slipher at Lowell Observatory to determine that the universe is expanding (see Hubble's law).

Hubble's interpretation added the velocity related aspect...
This interpretation can be interpreted differently to the same dimensions by  interpreting the observation as being related to this third time dilation, rather than being velocity related, where the rate of velocity of a red shift can be directly re-calculated as a rate of time.  Where we are now (of course) considering that galaxies are not expanding away from each other.

By knowing the mass of the star, and the distance of star, one can know the frequency the light started out with and the value of the g-field between.
The third time dilation I propose is inherent to the g-field where light will shift in frequency, as has been experimentally proven, and the third time dilation of the g-field will cause the light to take a longer time to travel.  i:e. physical cause for curvature.

Edit:  In reply to your post.   What I do understand is that the right hand expression of Einstein's equation is equal to the left side.  How it gets equal doesn't concern me.  What I'm interested in is the fact that without the added expression on the left side, Einstein's equation describes an expanding universe, so there must be some multiplying variable over time inclusive within the left side.
And that by adding the mathematical expression of his constant on the left side, to counteract this expansion over time, there must be an action that is negative with respect to this multiplying variable factor of the left side expression, within the additional expression of the constant to the left side in order to achieve the desired steady state universe that Einstein first sought.

Edit 2: The the multiplying factor over time may well occur on the right side for all I know, but that the equation without the added constant expression causes expansion, and that with it is stopping this expansion is where I'm coming from.
Title: Re: Is there a discrepancy with the equivalence principle?
Post by: timey on 14/03/2017 02:04:21
In addition to post above:

Don't get me wrong Mike - I have watched the GR Susskind lectures and am well aware of the complexity that is summed up in those mathematical expressions.
I also understand that I can view the summing up of that complexity into the form of Einstein's equation as the bigger picture of that complexity.

So yes the logic of adding or not adding, or indeed subtracting this mathematical expression of Einstein's constant interests me for the reasons I've said above.
Title: Re: Is there a discrepancy with the equivalence principle?
Post by: Mike Gale on 14/03/2017 04:13:12
Leavitt's story is unfortunately typical. Credit rarely goes where credit is due. Galileo, Kepler and Darwin are all guilty of the same crime. (I only recently learned of Kepler's guilt in that book you recommended by Smolin. A top rate book. It should be mandatory reading for all would be scientists.)
Title: Re: Is there a discrepancy with the equivalence principle?
Post by: Mike Gale on 14/03/2017 04:27:18
Note that you can put the cosmological constant on either side of the equation. In fact, the energy density representation is traditionally placed on the RHS with a corresponding change in sign. The equality expressed in Einstein's field equation is that between spacetime curvature and energy.
Title: Re: Is there a discrepancy with the equivalence principle?
Post by: jeffreyH on 14/03/2017 06:24:29
Contravariance, covariance, basis vectors and dual basis vectors. Vectors and 1-forms. Keep reading timey.
Title: Re: Is there a discrepancy with the equivalence principle?
Post by: McQueen on 14/03/2017 08:20:55
Quote
  pasala:  Friends, I would like to add few lines, regarding my opinion on time dilation and c. Well, it is true that time dilates for two reasons gravity and velocity. 


It is almost axiomatic that gravity will affect not time, but a clock, this is especially so when a single second is equal to 9,192,631,770  ticks as in an atomic clock. Hence even raising the clock by a few centimetres will result in a difference in the gravitational potential and the loss of whatever fractions of a tick or tock that that translates to. But to say that this is a proof of General Relativity is absolutely nonsensical. It is understandable that if the tick (or tock) was really due to gravity that there should be a quantifiable difference in the time shown by clocks near the sun and clocks near the earth due to the gravitational force of the sun being proportionately greater and with the degrees of measurement available today this is probably what will be found if an experiment is done.     

On the other hand if the difference in time were really due to General Relativity the difference in time would not be measured in individual ticks or tocks or fractions of ticks and tocks but in whole minutes, this of course palpably does not happen. So to treat any slight variations in ticks and tocks as a proof of General Relativity is absurd.

Secondly any difference in time in clocks subjected to velocity is adequately explained using purely classical physics.  A clock on a plane automatically acquires the velocity of the plane itself and this might also cause a difference in a tick or a tock again nowhere near what is needed to satisfy relativity.
Quote
pasala : Speed of light: There are number of invisible things influencing speed of light.  At present we are taking two mirrors, focusing light on them and measuring the time taken to travel.  But we are feeling that outside is empty and that fresh ray is created each time.  This is not at all correct and no fresh ray is created. 


I agree with this assessment, it is another way of stating that looking at an object from a different point of  view does not necessarily result in a new frame of reference. As I have repeatedly pointed out the existence of a Universal aether would sort out all of these problems in a very prosaic, day to day manner, without any of the exoticism of GR or SR. Also if the speed of light is made constant and everything else is variable what can't you change? You can morph a kitten into a tiger and do anything else you want.

Coming back to the main gist of the argument,  it is a great , a very great pity,  that not enough attention is given to an investigation of the first causes surrounding the rise of special and general relativity. If the situation is examined it is found to be the ultimate absurdity. Michelson and Morley were looking for an aether that was several millions of time more rigid  than steel  yet permeable enough for the sun and the planets to pass easily through and at the same time was so elusive as to evade any attempts at detection. What is even more titillating or shocking, depending on one's point of view, is the fact that the Michelson & Morley non-experiment is still referred to today as if it were the ultimate proof that General Relativity and Special Relativity do in fact exist and are real tangible proofs of the way the Universe functions.  As the great GBS said: 'Politics is the last resource of the scoundrel.' It could be equally true to say that "Relativity (whether special or general) is the last resort of a desperate scientist."

All of the greatest Scientific minds including Poincare, Lorentz, Maxwell and indeed Einstein himself  truly believed that an aether especially one with electromagnetic properties would solve every problem in physics. Unfortunately no such aether model was forthcoming and as stated above special relativity and general relativity were accepted in an act of desperation since there was no other alternative. 

Today Neo-Classical physics ( also known as Gestalt Aether Theory) offers a model of the aether which accounts for every known phenomena in physics from electromagnetism, to radio waves to gravity and neutrinos.  This theory deserves greater notice.
 
Title: Re: Is there a discrepancy with the equivalence principle?
Post by: timey on 14/03/2017 13:23:20
Leavitt's story is unfortunately typical. Credit rarely goes where credit is due. Galileo, Kepler and Darwin are all guilty of the same crime. (I only recently learned of Kepler's guilt in that book you recommended by Smolin. A top rate book. It should be mandatory reading for all would be scientists.)

Oh good Mike, I'm glad you rated the book.  I found it to be incredibly informative about where current theories do correspond with each other.  Smolin gives specific instruction about precisely which issues are causing problem, and provides a template within his book concerning the matters that need to be examined marked out in the 5 big questions that cause the current theories not to be unified.
Now I know you have read it, I will get my copy out and reread it that I may talk with you in more precise terms about these questions.

Smolin also talks about the relationship between non-mathematicians and mathematicians concerning theoretical physics, making interesting psychological observations, also giving description of the type of religious dogma that both mathematicians and non-mathematicians can fall foul of when trying to put forward new ideas, and the fact that this attitude towards new ideas can only hamper the progress of physics.

Where he has already laid out an examination of time itself within the remit of current physics, making observation of how SR, GR and the standard model regard the phenomenon of time and what those differences are, concludes the book in discussing his notion that the underlying fault in physics may lie with our perception of time, and that he would have to get his thinking cap on.

My model is derived straight from these observations that Smolin makes about time, where I have taken the view that time is a reactive phenomenon that is inherent to the universe itself, and that this time phenomenon is energy related.

So Einstein's equation and an equality between spacetime curvature and energy is exactly what I want to be looking at, and because my model changes the remit of Hubble's velocity related red shift interpretation, and the model is contracting from just after my model's differing rendition of an inflation period, the fact of adding in, not adding in, or subtracting the mathematical expression of Einstein's  constant to his original equation is also of relevance.  This being because re-interpreting the red shift distance correlation in the way I suggest results in a non-expanding universe, however because the universe must be either expanding or contracting, I look towards it contracting and am looking for a way to calculate that within the current maths.

But...there is a possibility within the remit of the reinterpretation my model makes of Hubble's velocity related red shifts that the universe may be a steady state of sorts.  This could only be so if there is a precise balance between the formation of black holes from matter, and the formation of stars and planets associated with the 'emissions' either from a black hole, or from the actions surrounding the event horizon that give rise to clouds of high energy particles.  There is some evidence to support this type of balance within the data concerning thunder clouds, in that positrons have been observed at high altitude (where there shouldn't be any) and high energy particle transitions are occurring.
In a steady state balanced universe what we would be looking at is a bit of a 'convection (?)' system of transference from one state to the other, and the vague possibility of particle creation within Compton scattering and high energy particle interaction.

This steady state balance does not give description of the Big Bang though. Therefore I prefer the contracting cyclic model that I'm proposing which suggests a bigger picture balance of an eventual transference over sequential time of all clumped matter back to individual particles via a universal singularity of 'all' matter in the form of a singular black hole.  Which without having any counterpart gravitational partner to be a balancing factor will explode. (This black hole of the final throws of a universes cycle could be described as the 'white hole' that you have mentioned)

However Mike (chuckle)... Although my mathematical understanding gets better by the day, it really will be quite some time before I will be able to juggle the complexities of current mathematics to competently describe a contracting model as I propose.
Therefore I am looking for a mathematician, in much the same way one might look for an architect to design the type of house one has in mind, who would be up for producing the mathematics that will describe my contracting model.
I know it can be done.  Anything can be calculated, (for better or worse, I appreciate).  It's just a case of my finding someone who wants to.
Title: Re: Is there a discrepancy with the equivalence principle?
Post by: timey on 14/03/2017 15:55:24
Test post:

This topic is not refreshing to recent topics page.
Title: Re: Is there a discrepancy with the equivalence principle?
Post by: LB7 on 14/03/2017 16:11:03
Not at all, the mass doesn't exist, so what we called a mass is only an electromagnetic rotor, each piece of matter is an electromagnetic rotor. If the distance is low, the rotors can be in phase, so there is an attraction. If the distance is higher, (very far in Universe, not km !) the rotors can't be in phase and the gravity can be negative. The inertia is the energy needed to deform matter, and when a matter is moving there is a deformation, when an object moves in translation for example, the deformation gives a potential energy what we called "kinetic energy". So, the inertia mass is the same than the gravity, because the mass doesn't exist, there is only the inertia.

My theory needed only to create the energy and my device do it.
Title: Re: Is there a discrepancy with the equivalence principle?
Post by: Mike Gale on 15/03/2017 03:50:56
Pasala, LB7 and McQueen seem to be peddling new (and incomplete) theories of their own. The format of this forum is not conducive to that level of complexity. Let's try to stay focused, shall we? We're trying to figure out how timey's theory differs from mainstream physics. Comparisons with other fringe theories is not helpful. Here's the state of affairs as I understand them. Timey can correct me if I'm wrong.
She is proposing a new source of time dilation that is due to the gravitating mass itself as opposed to an interaction between masses. I don't understand the distinction yet and we have yet to establish the recipe, but I think she is trying to address the dark energy dilemma. She thinks that conventional relativity will be unaffected, but some aspects of it (spatial dilation I think) may have to be interpreted differently.
Title: Re: Is there a discrepancy with the equivalence principle?
Post by: LB7 on 15/03/2017 08:32:38
Quote
She is proposing a new source of time dilation that is due to the gravitating mass itself
I'm agree with that, for me, to change the clock I need a velocity or a mass (not 2 masses). It is explained easily: the matter (each part of matter) is an electromagnetic rotor. The is a high electromagnetic attraction followed by a high electromagnetic repulsion, the mean is an attraction when the distance is low (at level of Universe) and can be a repulsion at far distance when it is not possible to synchronized the rotors because there is an inertia and there is the problem of synchronized N rotors.

How the clock can change with a mass: the electromagnetic rotor (a planet for example) is synchronized with the "clock-particle" of the clock. The clock-particle of the clock rotate around itself at a velocity 'c' (light speed). Attraction: the clock-particle will accelerate because there is an attraction, but it is not possible because the clock-particle is already at 'c'. Repulsion: the clock-particle will decelerate and it can. So the clock-particle need more time to make its round. And a round is necessary to change a step of the matter.

I don't know what is the clock-particle but maybe it has no mass, in fact I don't need a mass for that particle.

I don't know exactly how it operate the attraction/repulsion of the electromagnetic field, especially in 3 dimensions, I would appreciate the Universe more in 2 dimensions because it could be easier to understand my theory and less maths. 

The attraction is very (very !) high like the repulsion, it is not a small attraction like gravity.

Title: Re: Is there a discrepancy with the equivalence principle?
Post by: Colin2B on 15/03/2017 12:00:40
Or are you prejudice to the possibility that an uneducated non-mathematician is capable of inspired thought.
You really don't know me well enough to make that accusation. Those who do know me suggest I might lean in the opposite direction. May be partly due to in my early schooling being mis-streamed so spent a great deal of time with people the school judged to be 'below average'. In many cases below average does not mean lower intelligence, I saw many reasons – prolonged childhood illness (more common then), frequent school moves, lack of home support for learning are just some. I have remained friends with some of those school friends and despite their not having paper qualifications they are more than capable of inspired thought and it is interesting how many who, not having the paper qualifications for a job to match their capabilities, have set up and run their own very successful businesses.
I was fortunate in that my problem was diagnosed and treated, but the slow start meant I left school with no real qualifications. So I had to rely on evening classes to get the qualifications I needed to go into full-time study, but again in those evening classes I met intelligent thinkers who had missed out.
I have also spend quite a lot of time supporting adult numeracy courses, probably because of what I saw at school, and again I have met very intelligent people who have just not been well served by the school system they were in, or by personal circumstances. We had educational psychologists test our intake to see whether it was possible to tailor teaching technique to individual students, they commented that it  was interesting that, despite this being a self-selecting group, a large number scored well above average on both verbal and non-verbal reasoning tests. So no, I don't link intelligence and level of education.
And before you leap to a different conclusion, we have raised 3 daughters and I'm married to a very intelligent woman, so I have no illusions about the intelligence and technical ability of women.

To be honest I don't think you stopped to think before writing this, if you had you might have wondered why I previously suggested you were capable of an OU degree if you lack the ability to think. We might disagree on some ideas and calculations, but that doesn't mean I downrate your intelligence.

At the risk of causing even more misunderstanding:

No, I don't expect you to have a Damascus moment and decide current physics is perfect – even I don't think it is. What I did mean is based on my own experience anything new I learn is reflected back and may change my previous understanding – a sort of 'Ah that's what was meant'.

Neither do I intend to try and teach you relativity. The two items I mentioned show, in different ways, that the way Pete & I think about time dilation (and I'm sure Jeff as well) has common ground with some of your thoughts. Pete has some good diagrams that show why the usual textbook explanation  can be misleading, but for some reason they aren't showing in the browser, I've pm'd him about it.

I might remove this in a few days so as not to block the thread.


Pasala, LB7 and McQueen seem to be peddling new (and incomplete) theories of their own. The format of this forum is not conducive to that level of complexity. Let's try to stay focused, shall we? We're trying to figure out how timey's theory differs from mainstream physics. Comparisons with other fringe theories is not helpful. Here's the state of affairs as I understand them. Timey can correct me if I'm wrong.
She is proposing a new source of time dilation that is due to the gravitating mass itself as opposed to an interaction between masses. I don't understand the distinction yet and we have yet to establish the recipe, but I think she is trying to address the dark energy dilemma. She thinks that conventional relativity will be unaffected, but some aspects of it (spatial dilation I think) may have to be interpreted differently.
Mike
The open nature of the thread title makes it difficult to limit contributions from others. I've said before that the way the theory is presented and the analogies used don't help to make the ideas clear to a reader, particularly as they are spread over a large number of posts stretching back years.
Can I suggest that you agree with Timey a thread title something like 'Scoping and defining Inverted Time Theory'. I think it would be useful to have all the propositions, assumptions, effects eg how is light affected vs test mass, inputs eg evidence, etc and to do so in a brainstorming format ie no critique or analysis at this stage, just clarification.
That way it would be possible limit off topic contributions. I'm sure Jeff would agree to join me in policing any off topic input.




 
Title: Re: Is there a discrepancy with the equivalence principle?
Post by: timey on 15/03/2017 18:00:43
Colin - it was a question not an accusation, but I am conscious of how others reading may perceive conversations that occur on this site, so thanks for your post above. Appreciated!
Title: Re: Is there a discrepancy with the equivalence principle?
Post by: timey on 15/03/2017 18:01:33
Pasala, LB7 and McQueen seem to be peddling new (and incomplete) theories of their own. The format of this forum is not conducive to that level of complexity. Let's try to stay focused, shall we? We're trying to figure out how timey's theory differs from mainstream physics. Comparisons with other fringe theories is not helpful. Here's the state of affairs as I understand them. Timey can correct me if I'm wrong.
She is proposing a new source of time dilation that is due to the gravitating mass itself as opposed to an interaction between masses. I don't understand the distinction yet and we have yet to establish the recipe, but I think she is trying to address the dark energy dilemma. She thinks that conventional relativity will be unaffected, but some aspects of it (spatial dilation I think) may have to be interpreted differently.

Mike - yes, that is the perspective I am examining as being a physical possibility.

I am getting into these temporally spatial considerations at present with Alan on this thread, where I have been looking at potential energy being responsible for an m in relation to M interpretation of GR time dilation, and am now attempting to set that concept into a temporally dilated open space, rather than a spatially dilated open space:

https://www.thenakedscientists.com/forum/index.php?topic=69875.new;topicseen#top
Title: Re: Is there a discrepancy with the equivalence principle?
Post by: alancalverd on 15/03/2017 18:23:15
Bad starting point. The gravitational potential sign convention is there because there's no theoretical upper limit to M, so deep space is the zero reference point and gravitational potential is always negative towards M, hence if you like the concept, near space is "temporally constricted". But why bother when the conventional GR dilatation equation works perfectly in practice?   
Title: Re: Is there a discrepancy with the equivalence principle?
Post by: timey on 15/03/2017 19:23:32
Because the standard model isn't united wth gravity.

Gravity potential energy being converted into kinetic energy to bounce 2 cannonballs of vastly differing mass values off a trampoline to same height implies that potential energy has physical consequences and therefore cannot be 'just' a convention.

And as a side issue - a temporal constriction implies shorter seconds.
Title: Re: Is there a discrepancy with the equivalence principle?
Post by: Mike Gale on 15/03/2017 23:29:59
Mike
The open nature of the thread title makes it difficult to limit contributions from others. I've said before that the way the theory is presented and the analogies used don't help to make the ideas clear to a reader, particularly as they are spread over a large number of posts stretching back years.
Can I suggest that you agree with Timey a thread title something like 'Scoping and defining Inverted Time Theory'. I think it would be useful to have all the propositions, assumptions, effects eg how is light affected vs test mass, inputs eg evidence, etc and to do so in a brainstorming format ie no critique or analysis at this stage, just clarification.
That way it would be possible limit off topic contributions. I'm sure Jeff would agree to join me in policing any off topic input.
When you open a new thread, you are instructed to ask a question in the title. Maybe the instructions should be extended to make an exception for this section of the forum.
Policing sounds a bit aggressive. Moving them into new threads and leaving links behind would be kinder, but you (and other referees) would then have to coin new titles. Maybe you could use PMs to encourage offenders to do that on their own.
Title: Re: Is there a discrepancy with the equivalence principle?
Post by: Mike Gale on 15/03/2017 23:45:25
But why bother [with timey's theory] when the conventional GR dilatation equation works perfectly in practice?   
Good question. I think the reason why timey wants to abandon spatial dilation is that it's a hard pill to swallow. Bell's spaceship paradox exemplifies the issue. I think she may be jumping the gun though. It's not immediately clear (to me at least) that the new temporal dilation will necessarily invalidate spatial dilation. However, it's hard to argue the point one way or the other without a recipe.
Title: Re: Is there a discrepancy with the equivalence principle?
Post by: timey on 16/03/2017 01:23:13
Mike - the recipe for the third time dilation 'is' the 'action' of the acceleration of gravity.
Because the third time dilation of the g-field itself is inherent with faster seconds closer to M, any m in that field being attracted towards M will be accelerated in its trajectory towards M by the fact of shorter seconds closer to M.
Other than m having to move through this open space time dilation, the third time dilation has no connection whatsoever to conventional GR gravitational time dilation, other than GR time dilation being an m in relation to M phenomenon, and that both time dilation a relate back to M via the g-field.  It is important to take on board that both of these time dilation are occurring simultaneously to each other.
(When SR time dilation is added in, this architecture then negates the necessity for relativistic mass)

SR time dilation will need to be added in, and that gets more complex (and very interesting), but one would find that SR spatial dilation is no longer necessary to describe curvature.
The third time dilation is already causing this curvature as a temporal dilation, rather than a spatial dilation, and also gives a physical cause for the acceleration of gravity, where at present no-one has a clue 'why' gravity accelerates as it does, physics only knows that it does.
Title: Re: Is there a discrepancy with the equivalence principle?
Post by: Mike Gale on 16/03/2017 02:06:25
The problem is, GR dilation has nothing to do with m. It's all about M. The recipe for a stationary observer is dT = dt * sqrt(1-2GM/rc2), where dT is the change in proper (i.e. local) time and dt is the change is coordinate time (i.e. as perceived by a distant observer.) If both observers are at intermediate locations then dt1/dt2 = sqrt((1-2GM/r1c2)/(1-2GM/r2c2)).
Title: Re: Is there a discrepancy with the equivalence principle?
Post by: timey on 16/03/2017 02:17:27
That is because GR time dilation is attributed as an open space phenomenon.  Current physics states time as running faster at h from M.

My model switches GR time dilation to being an m in relation to M phenomenon, where all m will be subject to time dilation in the g-field regardless of mass value in the same way that all m is subject to accelerating towards M in the g-field regardless of mass value.
My model then attributes the third time dilation to the open space that m is in, and states this as the cause of the acceleration that m experiences in the g-field.
Title: Re: Is there a discrepancy with the equivalence principle?
Post by: Mike Gale on 16/03/2017 02:22:04
OK, but what is the recipe? How does m affect dilation? Is dT proportional to m or m2 for example? Does it depend on the distance to the observer (i.e. h from m)?
Title: Re: Is there a discrepancy with the equivalence principle?
Post by: timey on 16/03/2017 02:30:59
Well actually I think that we don't need to include an observer.
A clock will tick faster at elevation due to gravity potential energy.  How does one calculate an acceleration for all values of mass?
However one calculates that an acceleration can be equal for all values of mass in the g-field is how one can calculate that time dilation will be equal for all values of mass at any h from M.

The third time dilation can be calculated directly from the red shift blue shift maths where light frequency increases closer to mass as it arrives, and reduces further away from mass as it leaves.
Title: Re: Is there a discrepancy with the equivalence principle?
Post by: Mike Gale on 16/03/2017 02:35:34
The clock is the observer. Acceleration due to gravity (in a Newtonian context) is simply GM/r2. This is true for all possible values of m and M, but not all possible values of r.
Title: Re: Is there a discrepancy with the equivalence principle?
Post by: timey on 16/03/2017 02:46:01
Well G in that equation would be describing the third time dilation which is now attributed to open space g-field.

Now all we need to do is figure out an equation that holds time dilation for m at h from M equal for all values of mass.
Potential energy gets converted into kinetic energy in free fall where a bigger mass will bounce off a trampoline to the same height as a smaller mass.  A bigger mass has more potential energy than a smaller mass if held at rest with respect to the g-field, but more potential energy being converted into kinetic energy in free fall doesn't cause the bigger mass to fall faster.
What I'm looking for here is an equation that causes all values of mass to experience an equal degree of time dilation at h from M.
Title: Re: Is there a discrepancy with the equivalence principle?
Post by: Mike Gale on 16/03/2017 02:51:44
G is nothing more than a unit conversion factor. The point of the equation is that acceleration is proportional to M/r2. The time dilation associated with that relationship is the one I gave above: dT=dt*sqrt(1-2GM/rc2). These are the equations for m at r from M.
Title: Re: Is there a discrepancy with the equivalence principle?
Post by: timey on 16/03/2017 03:00:05
Yes - under current physics remit this is so.

Let me rephrase, the equation using G in relation to M and r^2 describing gravitational acceleration is already describing the third time dilation that my model attributes to open space.  The point being that the third time dilation is proportional to M/r^2.

The phenomenon of GR time dilation is then removed from the domain of the open space g-field and is attributed to being an m with respect to M time dilation that all m in relation to radius from M will experience equally.
Title: Re: Is there a discrepancy with the equivalence principle?
Post by: Mike Gale on 16/03/2017 03:03:23
If the 3rd time dilation depends on M, it is not an attribute of open space. If you hijack GR dilation as the 3rd dilation, you are left lacking a recipe for GR dilation.
Title: Re: Is there a discrepancy with the equivalence principle?
Post by: timey on 16/03/2017 03:05:00
It only depends on M for a value in the g-field surrounding M.
Title: Re: Is there a discrepancy with the equivalence principle?
Post by: Mike Gale on 16/03/2017 03:07:06
The g-field around M extends to infinity. Open space is just locations that are sufficiently removed from M that the g-force is effectively zero.
Title: Re: Is there a discrepancy with the equivalence principle?
Post by: timey on 16/03/2017 03:08:30
And therefore the seconds of the 3rd time dilation would get longer to infinity, and time would stop in a 0 gravity field.
Title: Re: Is there a discrepancy with the equivalence principle?
Post by: Mike Gale on 16/03/2017 03:09:47
Yes, as perceived by a local observer. The distant observer perceives local time to stop at the horizon. The local observer perceives distant time to speed up towards infinity. GPS is a prime example of this.
Title: Re: Is there a discrepancy with the equivalence principle?
Post by: timey on 16/03/2017 03:11:23
And what relevance does that actually have?

In fact a local observer in a 0 gravity field is an impossibility because the observer is mass, and if the observer was at rest with respect to the gravity field, his time would be running very fast with respect to the nearest M which would be a great distance away indeed if the gravity field was near 0.
Title: Re: Is there a discrepancy with the equivalence principle?
Post by: Mike Gale on 16/03/2017 03:17:49
The point is that GR time dilation makes perfect sense. What's missing in your theory is a recipe for the new dilation. The only thing we have established so far is that it depends on the distance of the observer from M, but not on M itself. (Because, if it depended on M then it would be one and the same with GR dilation.)
Note that 0-gravity means M=0. There is no GR dilation in that case: dT=dt at all points in space.
Title: Re: Is there a discrepancy with the equivalence principle?
Post by: timey on 16/03/2017 03:30:33
No - the 3rd time dilation is an ether type scenario and isn't observed by anyone except in the form of an accelerative force that increases at reduced h from M.
It doesn't need to be described as it already is being described within the current maths.

What does need to be re-described (for the purpose of my model), is GR time dilation, where the values remain the same, but for the reason that all m at h from M experiences this increased rate of time equally at any given elevation of h from M.

This differs from conventional relativity in that GR time dilation is no longer implying that time gets faster at h from M.  It states that time will only be getting faster for m at h from M.
The 3rd time dilation will be getting slower at h from M.
Each will be the equal and opposite of each other at any h from M.
Title: Re: Is there a discrepancy with the equivalence principle?
Post by: Mike Gale on 16/03/2017 03:35:49
But it is already true that all m at h from M experience the same GR time dilation for a given value of M. You can make m as big or small as you like, as long as m>=0. I think you're talking about another dilation that depends on both m and M when the observer is colocated with m.
Title: Re: Is there a discrepancy with the equivalence principle?
Post by: timey on 16/03/2017 03:42:53
But physics makes sums of m at h from M on the basis of a caesium atoms frequency always being 9,192,631,770Hz at any gravity potential...

No - I'm looking to unite the standard model with gravity.
Title: Re: Is there a discrepancy with the equivalence principle?
Post by: Mike Gale on 16/03/2017 03:45:51
No. The number of oscillations per second (or the number of seconds per oscillation) depends on altitude. You can interpret that as time dilation or variable light speed. The net effect is the same.
Title: Re: Is there a discrepancy with the equivalence principle?
Post by: timey on 16/03/2017 03:47:42
So why this obsession with observer dependency?
And if the atom really has a higher frequency at altitude it must have a higher energy...
Title: Re: Is there a discrepancy with the equivalence principle?
Post by: Mike Gale on 16/03/2017 03:49:47
You need an observer to experience time. A clock is an observer. An observer is a clock. A clock at altitude runs faster because it has more potential energy.
Title: Re: Is there a discrepancy with the equivalence principle?
Post by: timey on 16/03/2017 03:50:46
But time does not need an observer to operate.  Observer or not it's still does its thing.
Title: Re: Is there a discrepancy with the equivalence principle?
Post by: Mike Gale on 16/03/2017 03:52:18
No. Time is meaningless without an observer to experience it. The observer doesn't have to be sentient though. Anything that oscillates will do. An atom for example.
Title: Re: Is there a discrepancy with the equivalence principle?
Post by: timey on 16/03/2017 04:00:44
A clock at altitude runs faster because it has more potential energy.
Exactly... And all other mass at altitude has more potential energy which is why it experiences time at the same rate that the clock is ticking.  All particles will have more energy and higher frequency at altitude.

So this GR time dilation does not have to be associated with the open space surrounding mass that is the g-field being caused by M.


In reply to in between post, yes - all particles will oscillate.  The observation of this oscillation will be dependent on where the observer is observing from, but only because an observer holds his observation relative to the rate that his own clock is ticking.
Title: Re: Is there a discrepancy with the equivalence principle?
Post by: timey on 16/03/2017 04:07:49
You may now mention that the pe=m times something or another equation, (it would take me time to look it up,) would cause different values of mass to have more or less potential energy at higher altitude than they would at lower altitude, and if the time dilation was potential energy related differing mass values would be subject to different degrees of time dilation at same altitude, which is not what we observe...
...and occilations must remain proportional to each other in every coordinate at h from M, to satisfy the equivalence principle.
Title: Re: Is there a discrepancy with the equivalence principle?
Post by: Mike Gale on 16/03/2017 04:09:31
You don't need an observer to make the atom oscillate (at least not in a classical sense - some QM theorists would beg to differ.) The oscillation has consequences for the atom independent of any external observer. The point is that the atom is an observer of time and its mass can be arbitrarily small so that it has no significant impact on the passage of time as perceived by any other observer.
Title: Re: Is there a discrepancy with the equivalence principle?
Post by: timey on 16/03/2017 04:13:37
Fair do's.  I am in agreement.
However it is the m in relation to M that causes it to have additional potential energy.  I don't see the atom gaining energy on its own steam, but as per being in a relationship with M.

But as per post above?
Title: Re: Is there a discrepancy with the equivalence principle?
Post by: Mike Gale on 16/03/2017 04:18:28
The equation you're thinking of is PE=mgh. An object's experience of time at altitude does not depend on its mass. Time dilation depends on the ratio of PE to m (i.e. gh.) Note that g=GM/h2. (These are all classical approximations of course.)
Title: Re: Is there a discrepancy with the equivalence principle?
Post by: timey on 16/03/2017 04:28:20
Well in that case all the math can remain more or less the same.
But the interpretation of the reasons for the math have changed with the time dilation interpretation of the acceleration of gravity, where now one calculates time as stopped in the 0 gravity field, and we have a temporally deduced dilation of space where Hubble's velocity related interpretation of the red shift distance correlation is replaced by this temporal description were wavelength in the g-field is time dilation related, not distance related, and we can go look at adding energy to the particles of the black body of the black body experiment and apply the increase in occilations time dilation being experienced by the black body as being the cause of the emissions of higher energy and frequency photons...
Title: Re: Is there a discrepancy with the equivalence principle?
Post by: Mike Gale on 16/03/2017 04:38:07
But time does not stop when g=0 (indeed, what would it stop relative to?) and concepts like velocity, frequency and energy are meaningless if time stands still. (We're talking about stationary reference frames of course so we can ignore SR.)
Title: Re: Is there a discrepancy with the equivalence principle?
Post by: timey on 16/03/2017 04:45:54
My model would agree with you in that time does not stop for m in a 0 gravity field, for the reason that if m is there it is not a 0 gravity field, and you can't measure a 0 gravity field of m is not there.
m in a near non zero gravity field would have a fast rate, while the g-field itself would have a slow rate.
Title: Re: Is there a discrepancy with the equivalence principle?
Post by: Mike Gale on 16/03/2017 04:48:10
What about light, for which m=0? You are suggesting that the passage of time depends on the value of m. That is demonstrably not true.
Title: Re: Is there a discrepancy with the equivalence principle?
Post by: timey on 16/03/2017 04:52:34
Ah - well you see where lights frequency increases where it gets closer to M, where an m's frequency decreases when it gets closer to M is going in the opposing direction, therefore my model's says that light doesn't have potential energy because it has no mass, and it only experiences the acceleration or deceleration of the 3rd time dilation otherwise known as the acceleration, or deceleration of gravity.

The light gravitationally shifts and the extra/lesser length of wavelength is 3rd time dilation related,
Title: Re: Is there a discrepancy with the equivalence principle?
Post by: Mike Gale on 16/03/2017 05:07:08
PE is a classical concept. It doesn't make sense for objects travelling at or near light speed or for objects that have no mass.
An observer perceives lower frequencies (i.e. red shift) when peering down a gravity well. You can interpret that as observer time speeding up due to its own mass rather than local time slowing down due to the remote object's proximity to the gravitating mass, but a third party of different mass perceives the same red shift as long as all observers are much lighter than the gravitating mass.
Title: Re: Is there a discrepancy with the equivalence principle?
Post by: Colin2B on 16/03/2017 09:00:27
When you open a new thread, you are instructed to ask a question in the title. Maybe the instructions should be extended to make an exception for this section of the forum.
Policing sounds a bit aggressive. Moving them into new threads and leaving links behind would be kinder, but you (and other referees) would then have to coin new titles. Maybe you could use PMs to encourage offenders to do that on their own.
Can't remember where 'it is written' but the question rule is relaxed for new theories and chat etc. It's only enforced in the 3 main sections.
On policing. If it is clear what the purpose of the thread is and stated up front I would expect people to respect that. Although we do our best to be sympathetic in moderating, the moderators are unpaid volunteers and we all have day jobs or projects so we have limited time to manage people who decide to be awkward - and there are quite a few of those.
Have a think how best to set it up - if at all - and we'll do what we can to help it run.
Timey will need to write all this up in a concise form anyway if she's going to publish.

PS Don't see why Bell's is a problem for distance, great teaching aid on accelerated frames. Probably a side issue at this stage anyway, get the system analysis done 1st, detail later.
Title: Re: Is there a discrepancy with the equivalence principle?
Post by: timey on 16/03/2017 15:28:57
Quote
PE is a classical concept. It doesn't make sense for objects travelling at or near light speed or for objects that have no mass.
If we look at potential energy as a time accelerant for mass, and kinetic energy as a time decelerant for mass, then mass held at rest with respect to the g-field of M will have more potential energy, where pe=mgh, and pe/m = time accelerant, which under that remit will render the time accelerant as equal for all values of m and non-existent for 0 mass phenomenon such as light, and the g-field itself.

Converting pe into ke will decrease the time accelerant as in free fall.  Changing a position at rest with respect to the g-field, to a lower potential at rest with respect to the g-field will not increase ke, but it will decrease the pe and therefore the value of the time accelerant will decrease.

Add pe to a body in the form of an accelerant (fuel) to be converted into pe that is converted into acceleration, where acceleration converts pe into ke resulting in less time accelerant and time slows down for moving body.

Quote
An observer perceives lower frequencies (i.e. red shift) when peering down a gravity well. You can interpret that as observer time speeding up due to its own mass rather than local time slowing down due to the remote object's proximity to the gravitating mass,

..."you can interpret that as observer time speeding up due to its own mass"...

I'm suggesting that the observer in this scenarios time would be speeding up not as a product of its own mass, but as a product of its proximity to M.  Get another value M and m in same proximity, i.e: at same m at same h from a different value M, will have a differing rate of time.  It is M that denotes how much potential energy m will experience at h from M, and pe=mgh, where pe/m then renders the time accelerant as being equal at that h from M for all value of m.

Quote
An observer perceives lower frequencies (i.e. red shift) when peering down a gravity well.

Yes and an observer will perceive higher frequencies (i.e. blue shift) when peering up a gravity well.
But light is a differing observation for 2 reasons. Firstly one can only see the light when it arrives at one's observation point, so one is only viewing what the light looks like at their gravity potential.
And secondly that an observer peering down a gravity well, his clock's frequency (and the frequency of all the atoms that make up his body and that of his craft) is increased at that h from M compared to how it would be at a lower gravity potential.  Whereas lights frequency is reduced at that h from M compared to what it was in the lower gravity potential.
Therefore my model states that the light being of 0 m doesn't have any pe to increase its frequency at h from M, and is only subject to the 3rd time dilation, this 3rd time dilation being otherwise and more commonly known as the accelerative and declarative force of gravity in the g-field of M that all m and light must move through when experiencing changing positions in the g-field with respect to M.
Title: Re: Is there a discrepancy with the equivalence principle?
Post by: alancalverd on 16/03/2017 15:55:30
Except that time dilation applies in the absence of m.
Title: Re: Is there a discrepancy with the equivalence principle?
Post by: timey on 16/03/2017 17:10:07
And how would you know that without measuring it?

And how would you measure it without involving mass?
Title: Re: Is there a discrepancy with the equivalence principle?
Post by: alancalverd on 16/03/2017 17:17:44
Use a photon.

The interesting point is that we know F = GMm/r2 and F = ma if m<<M, but we don't know why mgrav = minert
Title: Re: Is there a discrepancy with the equivalence principle?
Post by: timey on 16/03/2017 18:06:48
But the photon can only be observed at one's location in the gravity potential.
And the photon emitter in the gravity potential is a mass.

Back to my Google investigations for full translation of some of those maths terms...and some thinking.
Title: Re: Is there a discrepancy with the equivalence principle?
Post by: timey on 16/03/2017 19:18:28
mgrav=minert is part of the equivalence principle.

Quote
It is only when there is numerical equality between the inertial and gravitational mass that the acceleration is independent of the nature of the body.

...and if you consider the possibility that the acceleration is caused by a 3rd time dilation factor in the g-field itself, then acceleration is independent of the nature of the accelerated body - but will be dependent upon the g-field value in relation to the nature of M.
Title: Re: Is there a discrepancy with the equivalence principle?
Post by: Mike Gale on 16/03/2017 23:01:06
mgrav=minert is not only part the equivalence principle, it is the equivalence principle in its entirety. In Einstein's own words, "observers falling freely in a gravitational field do not feel their own weight." The only caveats are that they do feel tidal forces and it's not clear if the principle holds when escape velocity approaches light speed.
Title: Re: Is there a discrepancy with the equivalence principle?
Post by: timey on 16/03/2017 23:16:37
Yes - the equivalence principle, or concepts close to it, also imply that the caesium atom will be of the same frequency in each and every reference frame, and states the speed of light as constant.
The tidal forces being gravitational direction, are, in my model, associated with the magnetic moment/s of an atom, and accelerated in that direction by the 3rd time dilation, otherwise known as the acceleration of gravity.
If it is a time dilation related acceleration this would afford a weightless feeling, and all value of mass would accelerate at same rate.

*

Going back to the Einstein equation and the fact that without adding the mathematical expression of the constant, Einstein's equation describes a universe that is expanding...
Logically speaking, as I mentioned before, this would require that the equation without the expression of the constant has a factor that describes an expansion.
Why does this equation describe expansion?
What is the expanding factor within those mathematics?
For instance, is it distance?

*

In answer to edit:  Speed of light held relative to what rate of time?
Title: Re: Is there a discrepancy with the equivalence principle?
Post by: alancalverd on 16/03/2017 23:41:10
Yes - the equivalence principle, or concepts close to it, also imply that the caesium atom will be of the same frequency in each and every reference frame, and states the speed of light as constant.

So that's all right, then. And the constancy of c can be demonstrated with a variant of the Michelson Morley experiment without invoking any time measurement. 
Title: Re: Is there a discrepancy with the equivalence principle?
Post by: timey on 16/03/2017 23:50:37
Unless the Michealson Morley experiment's inline arm does not shrink for the reason that the speed of light cannot exceed the local rate of time...
Title: Re: Is there a discrepancy with the equivalence principle?
Post by: timey on 16/03/2017 23:56:18
In which case, as an inverse effect, the LIGO tubes do not shrink either for the reason that the local rate of time has been affected for the duration of the gravity wave.
Title: Re: Is there a discrepancy with the equivalence principle?
Post by: Mike Gale on 17/03/2017 04:06:11
I'm not entirely clear how one gets an expanding universe from the SC metric. The only formal derivation I know of is the Friedmann (FD) metric, which is something entirely different: ds2 = a(t)2ds32 - c2dt2
The quantity ds3 comprises the spatial displacements and a(t) is an arbitrary function of time. You can use GR principles to compute a(t) for any given spatial topology (i.e. flat space, expanding space or contracting space.) The result is then parameterized to align with Hubble's data and, without a cosmological constant, expanding space is presumably the best fit. (I've never seen that calculation, but it has presumably convinced many experts.)
This calculation is entirely contrived because, unlike the SC metric, which is anchored on a gravitating mass, the FD metric is anchored on the observer. SC curvature has a physical explanation, which is the energy of the gravitating mass. FD curvature is empirical. It doesn't depend on the mass of the observer, only Hubble's data. We just give it a name - the Big Bang - and presume that it applies for all points in space.
It seems to me that a more reasonable approach would be to use the SC metric to predict how the universe looks when viewed from inside a gravity well. If you adopt the variable light speed interpretation, the speed of light (and everything else) increases towards infinity as you peer deeper and deeper into space. In this view, the expansion that we perceive from our vantage point here on Earth is primarily due to our proximity to our galactic core. An observer in intergalactic space would perceive flat space.
The reason I bring this up is that timey is trying to reconcile Hubble's data with the SC solution and that's essentially what Friedmann was trying to do.
Title: Re: Is there a discrepancy with the equivalence principle?
Post by: timey on 17/03/2017 15:52:22
Ok - it just came back to me why I though Euler's method in relation to Katherine Johnson was relevant to your thread.
You are using GR time dilation to hold your coordinate speed of light relative to.  But when adding in SR spatial dilation, this fudges up your position within those GR coordinates, **unless you state your coordinate times as occurring at these variable spatial dimensions, i.e. that the GR time shifts occur not at every h from M, but occur as per extended radius held relative to variable distance**(I'll come back to this thought later)
If you go back and add more GR coordinates to the spatial dilation, then, by using the variable speeds of light within that spatial dilation to calculate, this then again affects the SR spatial dilation and you have to re-assess your coordinates...again, and so on.
Right?
It would seem to me that you can hold the speed of light relative to the rate of time at a coordinate, or you can use SR to dilate spatial dimensions, but you cannot use SR to dilate space 'and' hold the speed of light relative to coordinate time because you get a catch 22 going on. 

However, in calculating the above, you are already using a calculation that is describing the acceleration of gravity at each coordinate with an inclusion of a rendition of GM/r^2 operating within the maths.
My model has attributed the nature of this acceleration to a physical cause.  This being that a 3rd time dilation is inherent to the g-field where seconds are getting shorter and shorter nearer M, and getting longer and longer out in space.
Current maths are already calculating this phenomenon as being gravitational acceleration.

So moving on - anything with mass will be affected by gravity potential, where pe=mgh.  Where we can say that pe/m=time dilation equal for all value of m at h from M.
(It is appreciated that there may be a squaring that needs to occur in that equation, I am just illustrating that having arrived at a pe value using the mass value, that by dividing that pe value by the mass value we have an equal distribution of pe for all mass values that will result in an equal time dilation for all mass values at h from M)

Now that we have disassociated GR time dilation from the concept of time running faster for open space and the g-field itself, and attributed this phenomenon as being inherent to m in relation to M only, we can go back to looking at the acceleration of gravity and the attributed 3rd time dilation that gives physical cause for acceleration.
We know that this 3rd time dilation does not directly affect the rate of time for m,**although the acceleration that it causes does**(I'll come back to this thought later), but we can see that m is accelerated in the g-field with respect to M.

Now back to my earlier thought:
Quote
**unless you state your coordinate times as occurring at these variable spatial dimensions, i.e. that the GR time shifts occur not at every h from M, but occur as per extended radius held relative to variable distance**
Which will also give rise to problems but serves as a platform for me to say this:

If one considers that the 3rd time dilation, otherwise known as the acceleration of gravity is causing a temporal dilation...
When one holds the speed of light relative to GR time dilation, the GR coordinates will not be affected by the temporal dilation of the 3rd time dilation and the catch 22 is negated.
But... one won't need to add SR spatial dilation to describe curvature, it's already described.

But applying the principle of SR time and spatial dilation to this new picture as a phenomenon experienced by the traveller, rather than as a phenomenon observed by an observer, gets very interesting indeed!
Where I now come back to the thought:
Quote
The 3rd time dilation does not affect the the rate of time for m,**although the acceleration that it causes does**
SR (in your rendition), would be holding it's measurements relative to the coordinate speed of light.
So as the coordinate speed of light increases at h from M via being held relative to a GR time dilated shorter second, the 'constant' speed a traveler is travelling at becomes a lesser percentage of that coordinate speed of light, and the time dilation and resulting 'appearance' of spatial dilation that the traveler himself observes will trend to being shorter seconds and shorter distance.
**You may find that there isn't enough head room within a distance travelled for a traveler to experience the distance he is travelling to be lesser, or for the seconds on his clock to become shorter, unless his speed is being constantly decelerated that is...**(I'll come back to this thought)

If you add a decelerating phenomenon to the travellers speed, i.e: outbound into the decelerating force of gravity (3rd time dilation), the speed becomes a lesser percentage of the rising coordinate speed of light, resulting in the traveler experiencing faster seconds, resulting in a perceived shorter distance.
When adding an acceleration to the travellers speed on the inbound into the accelerating force of gravity (3rd time dilation), the speed traveled becomes a greater percentage of the coordinate speed of light.  The traveller will perceive a lengthening of his seconds and will perceive distance travelled as being longer.
(There is an argument here that SR time dilation effects cancel out GR time dilation effects)

So, not withstanding the matter of travelling at **constant** speed at an accelerated or decelerated coordinate, whereby it would be possible to equate the factor of a form of additional energy to action conversion that might serve to balance any anomaly in the books, I don't see any reason why an expanding universe cannot be calculated as such, using the 3rd time dilation as an accelerative/decelerative force, and also as a temporally derived curvature of distance in space...
No reason, apart from the fact that lights frequency decreases and the wavelength gets longer at distance from M, which we have put down to the light being stretched by the velocity that light sources are receding away from us, so that's ok then...

But this does nothing to explain why the expansion was caused in the first place, nor the energy required to propel this expansion.

So - to calculate my contracting model, one simply holds the speed of light relative to the 3rd time dilation instead of GR time dilation, while holding SR measurement relative to the speed of light held relative to this 3rd time dilation.
Now the travellers 'constant' speed on the outbound is an escalating percentage of the coordinate speed of light, his personal time becomes increasingly dilated with longer seconds, and he experiences a distance travelled as being longer as a result.  (A result he could just as easily attribute to GR time dilation btw)
If adding a decelerating phenomenon to the travellers speed, i.e. the decerative force of gravity (3rd time dilation), the speed becomes a lesser percentage of the speed of light of that coordinate... And if the value of the speed remains a constant percentage of the coordinate speed of light, the traveler will experience his SR effects as constant.
When adding an acceleration to the travellers speed on the inbound into an acceleration of gravity (third time dilation), the speed travelled becomes a lesser percentage of the speed of light of that coordinate...  If the value of the speed remains a constant percentage of the coordinate speed of light, the traveller will experience his SR effects as constant.
(I'm not sure if this balance can be achieved via holding the coordinate speed of light relative to GR time dilation, but perhaps it can)

I think my model works better...
By separating GR time dilation for mass from the concept of time running faster in space, we have shown that light which has no mass will not have any potential energy and will not be subject to GR time dilation.
Unlike mass, light's frequency is known to reduce in the weaker g-field, and the longer length in wavelength can be thought to be 3rd time dilation related, which re-interprets Hubble's law as being a temporally deduced red shift distance correlation rather than Hubble's velocity related interpretation.

Now we have established possible cause, we could look to a contracting universe and my model's alternative rendition of Big Bang theory...
Title: Re: Is there a discrepancy with the equivalence principle?
Post by: Colin2B on 17/03/2017 22:19:43
I'm not entirely clear how one gets an expanding universe from the SC metric. The only formal derivation I know of is the Friedmann (FD) metric, which is something entirely different: ds2 = a(t)2ds32 - c2dt2

You are aware ratio observed density to critical density inversely proportional to Hubble parameter squared. You can link Hubble to cosmological via one of the Friedman Eqs using that equation. A while since I went through this so would need to work through again, but you should be able to get there.
Pity Pete won't join in, he's the expert, worked with Alan Guth and still has access for checking ideas.
Title: Re: Is there a discrepancy with the equivalence principle?
Post by: Mike Gale on 17/03/2017 23:49:19
THE Alan Guth? Isn't he famously brilliant? I imagine he would be loathe to drudge through this discussion thread because we are still waxing philosophical about timey's theory. We have yet to establish the (mathematical) recipe for her 3rd dilation for example. The relativistic correction to the SC metric thread might be more up his alley: https://www.thenakedscientists.com/forum/index.php?topic=69764.0 (https://www.thenakedscientists.com/forum/index.php?topic=69764.0).

The density ratio you (Colin2B) speak of goes into the specification of the spatial topology in the Friedmann analysis. It's one of the parameters you fiddle around with to get the FD metric to match Hubble's data.

I encourage timey to study the FD solution though. I think his observer-centric metric might be the same as her free space dilation. (https://en.wikipedia.org/wiki/Friedmann_equations (https://en.wikipedia.org/wiki/Friedmann_equations))
Title: Re: Is there a discrepancy with the equivalence principle?
Post by: timey on 18/03/2017 00:09:10
When you say a recipe for the 3rd time dilation what do you mean?

Is it not enough to describe that this time dilation is the cause of gravitational acceleration.  That a body travelling at constant speed in a vacuum can be accelerated towards an M by seconds becoming progressively shorter closer to M, to the accelerative value of GM/r^2(if that is the equation that describes acceleration?)...?

I'll look at the link..
Title: Re: Is there a discrepancy with the equivalence principle?
Post by: timey on 18/03/2017 00:20:48
No, those maths are too complex for me, and it sounds like a calculation of an expanding universe. So no, there may be similarities from a mathematical perspective if one were to try and calculate a contracting model, but Freidman is not calculating a contracting model, nor is he stating the accelerative force of gravity as being time dilation related, far as I can see.
Title: Re: Is there a discrepancy with the equivalence principle?
Post by: timey on 18/03/2017 00:32:30
Sorry, but it took me over 3 hours to write post 293, gutted really...
Title: Re: Is there a discrepancy with the equivalence principle?
Post by: timey on 18/03/2017 01:35:39
Anyway - perhaps someone can help me...
If we were to say that the value of GR time dilation, associated with M Earth, caused an acceleration to an object travelling at a constant speed, i.e. being accelerated by shorter seconds, negating any gravitational attraction or other phenomenon completely, what value of an acceleration would GR time dilation cause in metres per second squared?
Title: Re: Is there a discrepancy with the equivalence principle?
Post by: Mike Gale on 18/03/2017 05:34:09
When you say a recipe for the 3rd time dilation what do you mean?

Is it not enough to describe that this time dilation is the cause of gravitational acceleration.  That a body travelling at constant speed in a vacuum can be accelerated towards an M by seconds becoming progressively shorter closer to M, to the accelerative value of GM/r^2(if that is the equation that describes acceleration?)...?

I'll look at the link..
You are describing an existing interpretation of the SC solution. (The SC solution can be interpreted as variable light speed or variable time speed. The net effect is the same.) In a previous post you had postulated a different source of time dilation, which does not involve a gravitating mass. That's what Friedmann was on about.
Title: Re: Is there a discrepancy with the equivalence principle?
Post by: Mike Gale on 18/03/2017 05:42:46
Anyway - perhaps someone can help me...
If we were to say that the value of GR time dilation, associated with M Earth, caused an acceleration to an object travelling at a constant speed, i.e. being accelerated by shorter seconds, negating any gravitational attraction or other phenomenon completely, what value of an acceleration would GR time dilation cause in metres per second squared?
GR is consistent with Newton in the scenario you describe (i.e. here on Earth) regardless of whether you interpret it in terms of variable time speed or variable light speed. In other words, acceleration due to gravity is GM/r2 if the ratio of M to r2 is small enough. GR corrections for other cases are hard to pin down because M loses its meaning in a sufficiently strong field. The SC metric gives something like GM/r2*(1-rs/r) where rs=2GM/c2. I don't think that's quite right though because the validity of the SC metric in close proximity to the event horizon is questionable.
Title: Re: Is there a discrepancy with the equivalence principle?
Post by: timey on 18/03/2017 12:04:20
If one can understand that an acceleration towards an M can be considered as time dilation related, and the deceleration away from M can be considered time dilation related, then this time dilation is occurring in the opposite direction to GR time dilation. (which my model retains as an m at h from M phenomenon)
It also means that time only runs faster in space for m in relation to M.  Open space h from M is running slower time.

This is NOT the current view.  The current view is that time runs slower nearer M and faster out in space.  My idea completely changes that notion and the consequences of doing so are immense.

Now when a bigger M is involved such as a black hole, the same will apply, and the M of a black hole will have a much, much faster rate of time than Earth.  And where there is m at h from M of black hole, time will run even faster for m only.
And this is what considering gravitational acceleration being time dilation related results in.

Calculating the SC under the remit of gravitational acceleration being time dilation related would result in a completely new picture of the physics for a black hole.  No time stopped, no energy/information loss.
Just normal physics on a larger scale.

I don't know where I said that the 3rd time dilation was not related to the gravitational mass, but a g-field surrounding M is obviously caused by M.  And the 3rd time dilation is caused by the g-field surrounding M, where the g-field is weaker at greater h from M, and the 3rd time dilation has longer seconds in the weaker g-field.
Title: Re: Is there a discrepancy with the equivalence principle?
Post by: Colin2B on 18/03/2017 14:40:53
 
THE Alan Guth? Isn't he famously brilliant?
Yes Prof at MIT, the leading light on inflation, but free thinking and able to adapt it if necessary.

 
I imagine he would be loathe to drudge through this discussion thread because we are still waxing philosophical about timey's theory. We have yet to establish the (mathematical) recipe for her 3rd dilation for example. The relativistic correction to the SC metric thread might be more up his alley:
Like most busy people he is unlikely to look unless someone he trusts eg Pete says 'hey this is interesting'.
Some top phyicists do go onto some of the dedicated physics sites.
 
The density ratio you (Colin2B) speak of goes into the specification of the spatial topology in the Friedmann analysis. It's one of the parameters you fiddle around with to get the FD metric to match Hubble's data.
I thought you would understand

 
No, those maths are too complex for me, and it sounds like a calculation of an expanding universe. So no, there may be similarities from a mathematical perspective if one were to try and calculate a contracting model, but Freidman is not calculating a contracting model, nor is he stating the accelerative force of gravity as being time dilation related, far as I can see.
You can get to a contracting universe solution via these linking cosmo constant. However, big sticking point is, as you say, g being time related.

 
Mike, I think overall the issue is that Timey doesn't want an alternative that 'looks similar' unless it has certain specifications. In this respect although current maths solutions might suggest some techniques to use, it's not going to provide an answer.

 
Timey, somewhere else you mention time quantisation and filtering of time observed. Have you looked at chronons – particles of time?

 

 
Title: Re: Is there a discrepancy with the equivalence principle?
Post by: timey on 18/03/2017 15:00:46
I have read about the concept of time being quantised, but not recently.

However when I am referring to time in the context of quantisation, I am referring to an observer dependency concept.  That an observation of a rate of time that is differing from one's own will be proportional to the difference in rate, and that the observation is then a quantised, or inversely quantised representation.

My model states that if one consider that the temperature energy added to the black body increases the rate of time for the atoms of the black body, that then radiate higher frequency photons, that one can hold the frequency of the photon relative to the rate of time of the emitting atom and all quantised effects will then be negated.
Title: Re: Is there a discrepancy with the equivalence principle?
Post by: Mike Gale on 18/03/2017 17:47:25
If one can understand that an acceleration towards an M can be considered as time dilation related, and the deceleration away from M can be considered time dilation related, then this time dilation is occurring in the opposite direction to GR time dilation. (which my model retains as an m at h from M phenomenon)
Accelerating towards something is the same as decelerating away from it. The distinction has to do with initial velocity. Acceleration describes how the velocity of an object changes over time. Velocity describes how its location changes over time. It is possible for an object to experience acceleration yet remain stationary. In that case, it experiences a force, which is proportional to its inertial mass, and acceleration must then be interpreted as a change in the speed of time or, since m=E/c2, a change in the speed of light. In either case, the force changes the object's perception of space (in all directions) because, as Einstein pointed out, light is the only unambiguous way to measure distances in space.
Title: Re: Is there a discrepancy with the equivalence principle?
Post by: timey on 18/03/2017 18:26:42
Ok - well here you are saying about an acceleration that is caused by a force that is proportional to inertial mass...
My model differs as to what is the cause of this force, in that my model is saying that the accelerative force is caused not by a force that is proportional to inertial mass, but is caused by the 3rd time dilation of the g-field, which is denoted by h from M.
Title: Re: Is there a discrepancy with the equivalence principle?
Post by: Mike Gale on 18/03/2017 18:29:00
The force depends on inertial mass, but acceleration (due to gravity) does not. Gravity imposes constant acceleration at a given location. The resulting force depends on the observer's mass. By comparison, a rocket engine imposes constant force so acceleration depends on mass in that case.
Title: Re: Is there a discrepancy with the equivalence principle?
Post by: timey on 18/03/2017 18:35:11
In my model - the force depends on value of M, and h from M, and has nothing to do with m.
But there is the question of directional force, and that is why I was asking how many metres per second squared a constant velocity would be accelerated by GR time dilation increases in time in a gravity field.
Title: Re: Is there a discrepancy with the equivalence principle?
Post by: Mike Gale on 18/03/2017 18:37:26
No. Acceleration depends on M and h. Force depends on acceleration and m. In mathematical terms: F=ma and a=GM/h2. This is true for weak fields. The GR correction for strong fields is tricky, as I pointed out previously, and there is no consensus on it. One way to approach the problem is to recognize that the SC metric can be interpreted in terms of variable light speed. In that case, m and M are variables (because E=mc2.) Even so though, the SC metric breaks down at the horizon so it is unreliable when you get up close and personal.
Title: Re: Is there a discrepancy with the equivalence principle?
Post by: jeffreyH on 18/03/2017 18:49:50
No. Acceleration depends on M and h. Force depends on acceleration and m. In mathematical terms: F=ma and a=GM/h2. This is true for weak fields. The GR correction for strong fields is tricky, as I pointed out previously, and there is no consensus on it.

In strong fields very small changes in h are more significant. Hence tidal forces. This tends towards a singularity.
Title: Re: Is there a discrepancy with the equivalence principle?
Post by: timey on 18/03/2017 18:52:41
Yes - I understand how the current view is held, but I'm looking at a different means of description that attributes a physical cause for the force occurring, and this is a description that doesn't involve m.

A 100kg cannonball and a 10kg cannonball in a vacuum will both bounce off a perfect reflector to the same height that they were dropped from.
Both cannonballs will accelerate at same rate, and decelerate at same rate...
It is the conversion of potential energy into kinetic energy that ensures the bigger ball bounces as high as the smaller ball, more mass=more potential energy for conversion.
But there is no physical description as to why mass should experience acceleration in the g-field, and my rendition gives a physical description where none is being currently given.

And in my model - because time runs slower in space, bigger masses than Earth are not going to have slower rates of time than Earth, as I said before.  This negates all trickyness in the strong field.
Title: Re: Is there a discrepancy with the equivalence principle?
Post by: Mike Gale on 18/03/2017 18:57:29
In strong fields very small changes in h are more significant. Hence tidal forces. This tends towards a singularity.
True dat. Tidal forces are due to the spherically symmetric nature of the field. If you stand upright in a gravitational field and extend your arms, your hands experience less accleration than your head (because they are farther away from the centre of mass of the gravitating body.) This has dire consequences for your integrity when you are free falling in a strong field. Susskind calls it spaghettification.
Title: Re: Is there a discrepancy with the equivalence principle?
Post by: timey on 18/03/2017 19:00:45
And in my model - because time runs slower in space, bigger masses than Earth are not going to have slower rates of time than Earth, as I said before.  This negates all trickyness in the strong field.
Title: Re: Is there a discrepancy with the equivalence principle?
Post by: Mike Gale on 18/03/2017 19:08:57
GPS demonstrates that time runs faster (or light goes faster) in free space.
Title: Re: Is there a discrepancy with the equivalence principle?
Post by: timey on 18/03/2017 19:11:38
No - GPS demonstrates that time runs faster for m in space.  You can't measure open space, and light that covers distance slower will look exactly the same as light that covers distance quicker.  If the positive time and the negative time are equal, the distance travelled will be the same.
Title: Re: Is there a discrepancy with the equivalence principle?
Post by: Mike Gale on 18/03/2017 19:15:33
The mass of the satellite clock does not factor into the time dilation. And observers who disagree about the speed of time or the speed of light must necessarily disagree about distances in space and therefore simultaneity.
Title: Re: Is there a discrepancy with the equivalence principle?
Post by: timey on 18/03/2017 19:20:56
No - and the factor of the mass of any body of mass at that h will not factor into the the time dilation.

pe=mgh.  And we can say pe/m so that all atoms within m experience equal pe.

I don't see any reason why time dilation for m at h from M should be any different...
Title: Re: Is there a discrepancy with the equivalence principle?
Post by: timey on 18/03/2017 19:28:54
In my model no-one has to disagree about anything.  Distances are constant, and just by understanding which rate of time one is using to measure, everything is abundantly clear to everyone no matter where they are, although they may not be observing the entire picture where rates of time differ, because the observation will be proportional to the difference in rate of time and observations will result in being descrete, or quantised.  This will be more noticeable between vastly differing rates of time, but is indeed apparent in an observation of an atom at elevation to an observer as a change in the frequency of the atom.  Measure the atom via the rate of time the atom is experiencing, and the observation of the elevated atom will have the same frequency as the atom on the ground, when measuring the atom on the ground via the rate of time on the ground.
Title: Re: Is there a discrepancy with the equivalence principle?
Post by: Mike Gale on 18/03/2017 19:30:22
Even so, you are not entirely wrong. One's mass does in fact affect one's perception of the passage of time. That's why GR is formulated in terms of an infinitesimally small test mass. It's a simplifying assumption to make the math easier. You can certainly take that effect into account, but you're not going to learn anything new unless you fully understand the small mass scenario. Nobody can claim that status yet.
Title: Re: Is there a discrepancy with the equivalence principle?
Post by: Mike Gale on 18/03/2017 19:34:46
In my model no-one has to disagree about anything.  Distances are constant, and just by understanding which rate of time one is using to measure, everything is abundantly clear to everyone no matter where they are, although they may not be observing the entire pis tire where rates of time differ vastly, because the observation will be proportional to the difference in rate of time and observations will result in being descrete, or quantised.
If space is invariant then SR dictates that light speed depends on the velocity of the observer and the direction of the light ray. That leads to nonsense because the inertial mass of an object would depend on its direction of travel relative to the observer. It also flies in the face of the Michelson-Morley result.
Title: Re: Is there a discrepancy with the equivalence principle?
Post by: timey on 18/03/2017 19:48:46
Well SR might dictate that in current physics, but my model attributes SR as a mass in relation to velocity phenomenon experienced only by the traveller, and that light with no m is not affected.  Light travels as per the 3rd time dilation.

Did you not read my post of 293?

I mentioned before about how my model views that light cannot exceed the local rate of time and therefore the M&M experiment's inline motion arm does not shrink.
Title: Re: Is there a discrepancy with the equivalence principle?
Post by: timey on 18/03/2017 20:21:43
Look - my model makes a very subtle difference to the equivalence principle by stating the observation of one's own clock as observer dependent, instead of the other clock in a differing gravity potential, and an addition that states the speed of light cannot exceed the local rate of time.

All of the other changes occur as a result of doing so.

Frequency and energy changes 'do' occur at differing gravity potentials for mass, due to pe, and lights frequency and energy changes are occurring due to the 3rd time dilation.

Time does not run fast in space it runs slow.
Black holes and any value of M greater than Earth have a faster rate of time than Earth, not slower.

GR time dilation only affects mass in relation to M
SR affects m in relation to m's GR time dilation via m's velocity, where the speed of light is held relative to the 3rd time dilation, and is 'only' experienced by the traveller, although the observation of the traveler will be proportional to the rate of time of the observer and rendered quantised.
The 3rd time dilation only affects space and light, where the 3rd time dilation is caused by value of M via the g-field, and all m's and M's motion is affected by it.

And this 3rd time dilation gives physical cause for the acceleration of gravity and physical cause for a temporally derived curvature of space.
Title: Re: Is there a discrepancy with the equivalence principle?
Post by: Mike Gale on 18/03/2017 21:41:54
Well SR might dictate that in current physics, but my model attributes SR as a mass in relation to velocity phenomenon experienced only by the traveller, and that light with no m is not affected.  Light travels as per the 3rd time dilation.
Then you are proposing to throw SR out the window. I can't help you with that. Many have tried. All have failed. Note that, although SR has consequences for the concept of inertial mass (which are confirmed by experiment), it does not depend on it.
Title: Re: Is there a discrepancy with the equivalence principle?
Post by: Mike Gale on 18/03/2017 21:53:18
Time does not run fast in space it runs slow.
This is demonstrably untrue. I understand that you are proposing that time also runs faster due to one's own mass, but that is also demonstrably untrue. Einstein's light clock, which is the time keeper in the Michelson-Morley experiment, is massless for example.
Title: Re: Is there a discrepancy with the equivalence principle?
Post by: timey on 18/03/2017 22:07:45
Well SR might dictate that in current physics, but my model attributes SR as a mass in relation to velocity phenomenon experienced only by the traveller, and that light with no m is not affected.  Light travels as per the 3rd time dilation.
Then you are proposing to throw SR out the window. I can't help you with that. Many have tried. All have failed. Note that, although SR has consequences for the concept of inertial mass (which are confirmed by experiment), it does not depend on it.

I'm proposing that SR as a means of describing spatial space curvature be placed on the window ledge, in favour of the 3rd time dilation describing a temporal space curvature, and then aback off the ledge for a re-instating of SR time dilation as a rate of time experienced by the traveler, and the travellers perception of spacial dilation as being a result of the slower time.
The observer will observe length contraction due to a proportional observation that will be proportional to the difference between rate of time of observer and observed.

Hardy throwing SR out the window verbatim is it.  More like subjecting its    position of relevance to a minor remix I think...

This adheres to experiment that confirms SR velocity related time dilation for m in relative motion.
Title: Re: Is there a discrepancy with the equivalence principle?
Post by: Mike Gale on 18/03/2017 22:08:07
Did you not read my post of 293?
Yes, but you covered a lot of ground in that post. We can't debate everything at once.
Title: Re: Is there a discrepancy with the equivalence principle?
Post by: Mike Gale on 18/03/2017 22:15:47
I'm proposing that SR as a means of describing spatial space curvature be placed on the window ledge, in favour of the 3rd time dilation describing a temporal space curvature, and then aback off the ledge for a re-instating of SR time dilation as a rate of time experienced by the traveler, and the travellers perception of spacial dilation as being a result of the slower time.
Your 3rd dilation depends on M. SR describes the case where M=0. They are not interchangeable.
Title: Re: Is there a discrepancy with the equivalence principle?
Post by: Mike Gale on 18/03/2017 22:27:54
I'm sure you misspoke, but temporal space curvature is on the verge of an oxymoron. It would mean that space curvature changes over time. That's a horse of a completely different colour (viz. gravitational waves.) I think you were trying to attribute space curvature to time dilation and that means light speed depends on the direction of travel in the absence of gravity. I think I mentioned before that you can actually balance the space-time books with a variable light speed interpretation of SR, but it leads to contradiction when you apply that geometry to real life scenarios. Wikipedia has a good article on that (https://en.wikipedia.org/wiki/Variable_speed_of_light (https://en.wikipedia.org/wiki/Variable_speed_of_light).) Interestingly, Einstein was the first to propose the idea.
Title: Re: Is there a discrepancy with the equivalence principle?
Post by: timey on 19/03/2017 12:08:01
Where M=0 that is a 0 gravity field with no M in it, and the 3rd time dilation will stop with an infinitely long length of second.

You seem to be applying the 3rd time dilation to mass here.

You don't seem to understand that GR time dilation for m at h from M and the 3rd time for open space at h from M are the exact opposite of each other at each h from M.
Both are being caused by the g-field of M and the values will converge with each other at sea level (Earth), or equivalent to sea level for any other M.
So at sea level Earth 3rd time dilation and GR time dilation are equal.  All m at h from sea level earth will experience an increase in rate of time, i.e. GR time dilation, and all open space at h from sea level Earth will experience a decrease in rate of time, i.e. the 3rd time dilation, which will cause a body of m, any value, thrown upwards to decelerate at same rate, and cause the body of m, any value, to accelerate at same rate on the way back down.

The 'rate' of acceleration/deceleration is the 3rd time dilation I refer to.

The directional force in the gravity field is then held as due to the magnetic moments of m.
Where GR time dilation is increasing the rate of time for m, more magnetic moments occur, and where SR time dilation effect is decreasing as the mass is decelerated by the 3rd time dilation, again more magnetic moments are occurring.
And on the way back down the opposite occurs.

This is why I am trying to find out what the value of an acceleration would be in metres per second squared for near Earth parameters, using GR time dilation time increases as the means for acceleration, while negating all other influences.
Would this result in 9.807 m/s^2?
Or would it be a lesser value?

*

Next post - why would a temporally derived space curvature change over time?  The 3rd time dilation is caused by value of M, because it is the value of M that denotes the value of the g-field, and the value of the g-field denotes the value of the 3rd time dilation.
Consider your m travelling in open space over a period of sequential time.  As your m moves into the weaker g-field it simply takes a longer amount of sequential time to cover the same distance.
Now we may be on the ground observing the journey, and because (let's just say) we know the mass can only travel at 1 speed, we would see the mass moving at this constant speed being decelerated in the g-field.
We can call the traveler up on his mobile phone, he can tell us that our massive clock that we have with us appears to him to be running slow compared to his clock, and we can tell him not to worry, that's just GR time dilation increasing the rate of time for his clock, and that SR effects will be slowing his clock down, but as he is decelerating at mo, that these SR effects will also be decreasing.
The deceleration in speed that his craft is experiencing, being due to the 3rd time dilation.
If our travellers constant speed is being decelerated by longer seconds in open space, this will cause a temporal curvature of space.  If one did not realise that there were longer seconds in the weaker g-field, one would perhaps think that distances had become dilated...
Which 'is' what current physics thinks!

*
Holding light speed relative to the 3rd time dilation is geometrically possible and is synonymous to observation and experiment.
Title: Re: Is there a discrepancy with the equivalence principle?
Post by: Mike Gale on 19/03/2017 15:32:20
I think I do understand what you are proposing. Your idea is that open space is maximally dilated and the presence of mass energy pulls it back in a way that emulates the SC solution of GR. What I don't understand is your distinction between m at h from M and nothing at h from M. That implies a dependency between open space dilation and M, which you propose to counter with dilation from observer mass. I can't see how that would work because observer mass can be arbitrarily small compared to M.
Now that I say it out loud, I realize we are probably confusing the issue by bringing Hubble and SR into the conversation. By the same token, I would caution you against the use of technical terms like magnetic moment because the concept you have in mind is probably quite different from the one it invokes in the mind of a physicist.
My point about the terminology was that "temporal space curvature" could be misconstrued as gravity waves. Temporally-derived space curvature is more clear and that, as I pointed out, leads to the variable light speed hypothesis.
Title: Re: Is there a discrepancy with the equivalence principle?
Post by: timey on 19/03/2017 16:11:28
Yes - whereas you are achieving variable speeds of light for your relativistic correction to the SC by holding the speed of light relative to GR time dilation, whereby this will cause the problems I described in post 293 concerning SR related spatial dilation, and the defining of the specific coordinate location where time shifts are occurring that the speed of light may then be held relative to..
... And my model is holding the speed of light relative to the 3rd time dilation, where the curvature of space is a temporal dilation and the coordinate speed of light is definable at coordinates that are held as constant.  i.e. The distance traveled is a straight line comprised of constant metres, but the amount of time it takes to travel this straight line at a constant speed is variable, where it takes a longer time in the weaker gravity field to travel a constant metre.

*

When speaking in terms of a magnetic moment I am referring to that which one may read when googling 'magnetic moment', whereas the wiki and resulting pages are indeed written by physicists.

When looking at the wiki gravity potential page there is a comment:
"Where mass can be held as analogous to charge"...
Title: Re: Is there a discrepancy with the equivalence principle?
Post by: Mike Gale on 19/03/2017 16:29:26
In that case you are completely off base with magnetic moments because they have nothing to do with GR. Even if you have a theory that relates gravity to magnetic moments, it's probably best to take that off the table for now.
Title: Re: Is there a discrepancy with the equivalence principle?
Post by: Mike Gale on 19/03/2017 16:32:01
I presume your bystander perceives the traveller to be moving away from the gravitating mass with constant velocity. That's a very complicated scenario. The thrust of the rocket engine has to exceed the force gravity for a while in order to achieve cruising speed. It then has to be reduced abruptly to match the force of gravity at the point where cruising speed is achieved and backed off gradually thereafter to maintain cruising speed as the force of gravity recedes. The rocket engine adds energy to the traveller at a variable rate and that alters the space-time continuum in an unknown manner. Furthermore, the rocket loses mass over time as it generates thrust.
Title: Re: Is there a discrepancy with the equivalence principle?
Post by: pasala on 19/03/2017 16:40:40
Mr timey
As said by you:
"Next post - why would a temporally derived space curvature change over time?  The 3rd time dilation is caused by value of M, because it is the value of M that denotes the value of the g-field, and the value of the g-field denotes the value of the 3rd time dilation.
Consider your m travelling in open space over a period of sequential time.  As your m moves into the weaker g-field it simply takes a longer amount of sequential time to cover the same distance.
Now we may be on the ground observing the journey, and because (let's just say) we know the mass can only travel at 1 speed, we would see the mass moving at this constant speed being decelerated in the g-field.
We can call the traveler up on his mobile phone, he can tell us that our massive clock that we have with us appears to him to be running slow compared to his clock, and we can tell him not to worry, that's just GR time dilation increasing the rate of time for his clock, and that SR effects will be slowing his clock down, but as he is decelerating at mo, that these SR effects will also be decreasing".

I think there is every need for broad discussion on space time.  It is true that it is the base for Einstein gravity.   As per Einstein even objects "at rest"  are actually moving through space time.  Actually space time is not just space, but also time.  The "velocity" through space time is called a four-velocity and it is always equal to the speed of light.  Space time in gravitation field is curved.  The apple moving first only in the time direction  starts accelerating in space and the velocity in time becomes velocity in space. The acceleration happens because the time flows slower when the gravitational potential is decreasing. Apple is moving deeper into the gravitational field, thus its velocity in the "time direction" is changing .

Here everything is possible through space time.    As per Einstein space time is completely filled with matter and energy and the matter tells space time how to curve.  Suppose if the space breaks, naturally time also breaks.  It appears to be unbelievable yet it is true.  In case Earth is not having space, what happens matter and energy moves out freely without any obstruction. Can we imagine any planet, without space. No. it is impossible and there is no scope for gravity and and it is the death of a planet.  So, naturally the stronger the space time the stronger gravity.   

It is also said that M decides curvature of space time.  Ok, let us see  Moon, it is not a small one, but why its space is weak and thus gravity.  If the mass of a planet decides gravity on a planet than different planets have to experience different gravity fields, but in real terms it is not happening so.

It is true that Earth is rotating against its own axis and also moving round the sun and for that total universe is moving to an unknown place.  Actually, we have to remember one important point that Earth is not rotating and is being made to rotate by the gravity winds.  It is not Earth alone, things on the Earth, space and everything is moving. 

The apple is not moving, but is being made to move by the gravity winds.  Strong gravity winds, hooked Earth so strongly that nothing can escape that much easily.  Even matter and energy are not raising abnormally as assumed by Einstein.  So, Apple never changes gravity field and it remains  in static position, at any time frame. 

Gravity continue to accelerate against apple right from its inception.  Actually, there is no motion of apple and it is in inertial position permanently.  During time frame, apple becomes weak, looses control of the stem and gets into the hands of gravity.

Yours
Psreddy



Title: Re: Is there a discrepancy with the equivalence principle?
Post by: timey on 19/03/2017 17:28:11
I presume your bystander perceives the traveller to be moving away from the gravitating mass with constant velocity. That's a very complicated scenario. The thrust of the rocket engine has to exceed the force gravity for a while in order to achieve cruising speed. It then has to be reduced abruptly to match the force of gravity at the point where cruising speed is achieved and backed off gradually thereafter to maintain cruising speed as the force of gravity recedes. The rocket engine adds energy to the traveller at a variable rate and that alters the space-time continuum in an unknown manner. Furthermore, the rocket loses mass over time as it generates thrust.

Let's say the constant speed is 0.5 light speed, where light speed is held relative to ground clock...
The observer on the ground would see this speed decelerate in the longer seconds of the g-field.
The observer in the craft would observe his speed as decelerated by the longer seconds of the g-field, or he may come to the conclusion that gravity has reduced his speed, and at further distance from M may think that a metre has become longer in the weaker gravity field.
If the observer on the ground then thought that a metre had become longer in the weaker gravity field, then he would think that the craft had become length contracted.
(I will add thrust to this scenario at later date.  Keeping it simple for now)

So moving on - the observer in the craft's rate of time will have become increased in the weaker gravity field, (GR), and also decreased by the relative motion (SR)... Where the SR measurement of the decrease in rate of time is held relative to the percentage of the speed of light that the craft's speed is moving at, where the speed of light is still being held relative to the ground clock's time.

You suggest that the SR measurement of the speed of light be held relative to the GR time, or that is what I have deduced that you suggest,  Maybe I have this wrong...?
But if I am right, then - when you hold the SR measurement relative to the speed of light held relative to the GR time dilation, the GR time dilation is held relative to a particular coordinate. But then by making the SR calculation this involves a spatial dilation and your GR coordinate changes.  So back to SR to recalculate with the changed GR coordinate and the spatial dilation changes again, and so on...
Am I not correct that this catch 22 exists within the remit of holding light speed variable to GR time dilated seconds?
Title: Re: Is there a discrepancy with the equivalence principle?
Post by: Mike Gale on 19/03/2017 20:57:37
I see. The traveller is in free fall with an initial velocity vector that points away from the gravitating mass. That's a bit simpler, but the scenario can be further simplified (with no loss of generality) by setting initial velocity to zero. We should also stipulate that the entire episode takes place in a weak field so we don't have to worry about the validity of the SC metric near the horizon. In that case, Newtonian dynamics will suffice for the mathematics and all that remains is the philosophical interpretation of gravity, which Newton famously left as an exercise for the reader.
The equivalence principle dictates that traveller time is the same as in open space (i.e. far removed from the gravitating mass) so, if light speed is invariant, traveller space is the same as open space. That's the first point of contention because Flamm's paraboloid indicates that local space is compressed compared to open space. Light speed must therefore decrease as gravity increases. The experts disagree on this point, but we can impose that the observers are colocated at some instant of time so the light speed issue is moot as long as we limit ourselves to radial distance measurements.
GR predicts that bystander time runs slower because the bystander is suspended in the gravitational field, being held in place by a planetary crust or a rocket engine with constant thrust. Slower time with invariant light speed means space is compressed, as per Flamm's paraboloid.
That's the picture painted by GR. The next step is to describe this scenario in the context of your theory.
Title: Re: Is there a discrepancy with the equivalence principle?
Post by: timey on 19/03/2017 21:26:47
We cannot set the outbound velocity to 0 because without an initial speed that moves the craft in order to cover the distance that is inherent with slower seconds, a) one will not be moving, and b) there is no apparent speed for these slower seconds to decelerate.
Therefore we have an escape velocity where the outbound speed must be equal to the deceleration.  If we have the craft moving at a constant speed, it will decelerate at a certain rate.  If we want to upkeep a constant speed, we must accelerate at a certain rate.
Free fall near Earth accelerates at 9.807m/s^2.
In order to upkeep a constant outbound speed, would the craft have to accelerate at 9.807m/s^2?

And why must light speed decrease as gravity increases?
If local space is compressed compared to open space, why would seconds be elongated in a compressed space?
A higher energy, higher frequency, compressed wavelength would be more synonymous with faster time and shorter seconds wouldn't it?
A clock's frequency, and energy is higher with shorter wavelengths when it's tick rate is increased...
Title: Re: Is there a discrepancy with the equivalence principle?
Post by: Mike Gale on 20/03/2017 04:27:40
Zero velocity is a perfectly valid initial state. That's how Newton's apple got started for example. Newton makes no hypothesis about how the force of gravity gets things moving. The GR view is that a stationary object moves through time at light speed and acceleration simply rotates the velocity vector into the spatial domain. It's a change of heading in the spacetime continuum.

In my simplified scenario, the traveller is in free fall and the bystander is accelerating. That change eliminates the need for variable thrust, which is a significant complication.

Variable light speed is one possible interpretation of GR. Some people prefer to think of it as spacetime dilation, akin to SR. In either case, we have eliminated the effect by placing the observers at the same altitude at some instant of time.

If mass, space and light speed are invariant then higher energy (potential or kinetic) equates to shorter seconds.

Anyway, that's the picture painted by GR. The question is, what does it look like in your theory?
Title: Re: Is there a discrepancy with the equivalence principle?
Post by: timey on 20/03/2017 12:45:07
I am talking about Newtons Apple headed outbound, with an observer on the ground, and thrust is exactly what I want to be discussing.

I'm also not talking about what the universe 'looks like' to an observer when measured via differing rates of time.
I want to be talking about what the universe 'is like' with the differing rates of time, and then go back and check that what an observer will observe from his own rate of time is a match to experimentally verified observation.

Apart from an addition and a subtle change to the equivalence principle, the only change that my model makes to current remit is that the accelerative/decelerative force called gravity is a time dilation phenomenon.  Everything else stays the same, although the necessity for the 'dark stuff' is negated...
So what GR and SR looks like, is what my model looks like, only the reasons for it looking that way are differing.
Therefore I can't paint you a picture of what my model 'looks like' in relation to GR, as all observation will 'look' as they have always looked, just for differing reason.

So firstly, my model does not use SR to describe a spatial curvature of space, it uses the 3rd time dilation to describe a temporal dilation of space.
The metre always remains constant in my model, it just takes a longer or shorter amount of time for mass or light to travel a metre where the speed of light, or the speed of the mass is now held relative to the 3rd time dilation.
Now we have an observation of curved space that is not spatially dilated.
This is the equal to a Newtonian geometry that is now temporally dilated to an equal of relativity's spatial curvature.

Are you with me so far?

What I'm saying is that the deceleration a constant speed outbound from M will experience is time dilation related.  That when holding the slower seconds in space relative to the constant speed, this will decelerate the constant speed.
The same can be said for free fall, where there is an acceleration of 9.807m/s^2 near Earth, and 2 radii distance away from Earth, there is an acceleration of 4.25m/s^2, where my model states the accelerative force as time dilation related.

What we do see in action though is a directional force that determines that a body will fall towards a mass, and that when a mass's speed is decelerated away from mass,  when decelerated to a standstill, the directional force will then cause the mass to fall back towards the Earth.
In GR this is called the attraction of gravity.
In my model the acceleration of gravity is time dilation related, but this cannot describe the directional force.

Therefore I am interested in knowing by how many metres per second squared a body of mass must accelerate by to upkeep a constant speed on the outbound.
Is the acceleration needed to upkeep a constant speed on the outbound, the same acceleration that is experienced in free fall on the inbound?
Title: Re: Is there a discrepancy with the equivalence principle?
Post by: GoC on 20/03/2017 21:08:06
Timey I know you are not going to respond to me but your distances and values are incorrect. The atmosphere is only about 300 miles and 2 radii from the Earths surface is 16,000 miles out in space where you would be weightless and not necessarily attracted to the Earth at all depending on your position with the Earth. Your mass would be to relatively small for the dilation to be affected by the Earth and its vector speed away.

Now lets take half the distance to the center of the Earth. The attraction is about 2.45 m/s^2 until you reach the center where everything on earth is attracted to your position. Gravity like many physical issues is the inverse square of the distance. So like twice the diameter four times the amount. Sight would be twice the distance gives one quarter the viewable size of the object. Distance is not linear.

Title: Re: Is there a discrepancy with the equivalence principle?
Post by: Mike Gale on 20/03/2017 23:45:19
The initial velocity is a red herring. It can be set to zero without loss of generality. You need to understand that before we can make any progress. Are we agreed that the traveller has no rocket engine?
Title: Re: Is there a discrepancy with the equivalence principle?
Post by: timey on 21/03/2017 00:24:05
But this is where direction considerations come into play Mike...
If we set an outbound velocity at 0 the craft won't be going anywhere.

On the other hand, when dropping a craft from a 0 velocity we see the acceleration increases that velocity from 0.
I suppose that we could possibly set velocity at 0 for outbound, and then calculate the acceleration needed to move outwards as a negative velocity.
In any case - is the outbound rate of necessary acceleration, calculated in the negative if you like, the equal of free fall acceleration, being the question?

It should be for the reason that 2 cannonballs of differing mass values (in vacuum) bounce off perfect reflector to the same height they were dropped from.

It is appreciated that current physics has an explanation for this phenomenon, but it should also be appreciated that current physics does not describe 'why' this phenomenon does what it does.

I am looking at this acceleration and deceleration phenomenon as being caused by a 3rd time dilation factor in the g-field.
Which then begs the question of the necessity for a directional force, which I am looking at being due to an electromagnetic phenomenon.
Title: Re: Is there a discrepancy with the equivalence principle?
Post by: Mike Gale on 21/03/2017 03:18:34
It's true that acceleration vectors are additive. If your rocket accelerates upwards at 'a', the net acceleration will be 'a' minus 'g'. It will hover if a=g, fall if a<g and climb if a>g, but 'a' increases over time because the rocket must lose mass in order to generate thrust. There's no easy way to compute GR dilation for an arbitrary value of 'a', let alone one that varies over time. It gets even more complicated if you equip the rocket with an automatic thrust controller to maintain constant velocity or constant acceleration. In any of these cases, the best you can hope for is a numerical approximation. Note that the planetary crust accelerates the bystander upwards at 'g' so there's no point putting the traveller into that reference frame.
Title: Re: Is there a discrepancy with the equivalence principle?
Post by: timey on 21/03/2017 03:34:26
So in my model's scenario where this acceleration/deceleration is caused solely by the 3rd time dilation of the g-field caused by M, now what I am looking for is a minute fractional directional force that increases linearly with the higher gravity potential energy, which is why I look to the magnetic moment of the atom, or rather the rate they occur at, the electron transitions in relation to the quantum energy level and the analogy of gravity potential to the electric field where mass is analogous to charge.

Btw, not that it's immediately important, but adding thrust to rocket can be analogous to carrying potential energy onboard that is converted at 20% or so conversion to energy loss to kinetic energy.  As potential energy is converted mass value decreases and less onboard energy is required to achieve same kinetic energy.

In reply to edit:  I don't think it matters about an observer, we have the info we need in the value of M.
Title: Re: Is there a discrepancy with the equivalence principle?
Post by: Mike Gale on 21/03/2017 04:04:08
I'm ignoring your references to magnetic moments, electron state transitions, QM and electric fields as per my last comment on that subject. (i.e. They have nothing to do with GR.)

I assume a "minute fractional force" refers to an infinitesimally small change, in which case you are invoking calculus. That's really just a change in scale. It doesn't change the scenario or make the math any easier.

As for the thrust controller, it is not hard to design an algorithm that will result in approximately constant velocity or approximately constant acceleration. The problem is, the proliferation of variables precludes an exact solution in GR and obfuscates the dilation result.

Regarding the observers, they are nothing more than ideal (i.e. infallible) clocks in space. For our purposes, they are massless and non-sentient.
Title: Re: Is there a discrepancy with the equivalence principle?
Post by: timey on 21/03/2017 04:15:40
Well the whole point Mike is that GR only has nothing to do with magnetic moments, and quantum energy levels because the standard model is not united with gravity...
Gravity is a continuum, and quantum is not.  The idea that I propose is designed to unite the two.  Therefore I am looking at electron transitions in the gravity potential.  Electron transitions are directly related to quantum energy levels.  Give a physical cause  (time dilation), to the rate of the transitions, where the physical cause for the time dilation is potential energy related, and energy is the cause of your time dilation.  This negates the quantised nature and quantum is a continuum united with gravity...
Title: Re: Is there a discrepancy with the equivalence principle?
Post by: Mike Gale on 21/03/2017 04:27:16
QM has nothing to say about the duration of state transitions. They are presumed to occur instantaneously or retroactively (depending on who you ask.) Furthermore, you have not proposed a model to relate gravity to QM or EM phenomena. If your theory relies on an unspecified relationship like that, it is completely useless.
Title: Re: Is there a discrepancy with the equivalence principle?
Post by: timey on 21/03/2017 04:31:51
I don't think it is.

Considering the blackbody:
If one takes the frequency of the emitted light to be time related - then rather than measure the energy increase via an invariant time, one measures the increase in energy as per the rate of time synonymous to that increased frequency of light, I am quite sure that the quantised nature of the data can be rendered a continuum.
Title: Re: Is there a discrepancy with the equivalence principle?
Post by: Mike Gale on 21/03/2017 04:40:27
I think you're talking about the ultraviolet catastrophe, which implies quantization of space and/or time, but you have not related that concept to gravity. A theory based on an unspecified hypothesis is an unspecified theory.
Title: Re: Is there a discrepancy with the equivalence principle?
Post by: timey on 21/03/2017 04:49:39
Well because Planck's h constant relates to the shift in frequency of light, and light shifts in the g-field, there is an 'energy' relationship going on there with the g-field energy.
And because a clock's electron transitions shift in frequency in the gravity potential there is a potential 'energy' relationship there... (excuse the pun)
Where +energy=shorter seconds in my model.
Title: Re: Is there a discrepancy with the equivalence principle?
Post by: alancalverd on 21/03/2017 07:17:13
Well because Planck's h constant relates to the shift in frequency of light, .

Except it doesn't. The fractional change in wavelength is

L /L0= 1-GM/rc2

Any mention of h cancels out, nothing on the right-hand side is quantised, and the effect is continuous. If you wish you can substitute f0/f or E0/E for L/L0 but the h's still cancel.
Title: Re: Is there a discrepancy with the equivalence principle?
Post by: timey on 21/03/2017 14:21:50
Yes - but:

Wavelength = h/p

.... and where the value of p changes for already emitted light (in the gravitational field*) in relation to the h constant, this extra or lesser length in the wavelength isn't distance related anymore, it's 3rd time dilation related, as per my model that is...
(*in the scenario of the constant gravity potential where +energy to emitting source=higher frequency of emitted light, the frequency of the emitted light is GR time dilation related, and the length of wave is related to the 'rate' at which the photon is emitted from the emitting source)
Title: Re: Is there a discrepancy with the equivalence principle?
Post by: GoC on 21/03/2017 15:16:47
Well because Planck's h constant relates to the shift in frequency of light, .

Except it doesn't. The fractional change in wavelength is

L /L0= 1-GM/rc2

Any mention of h cancels out, nothing on the right-hand side is quantised, and the effect is continuous. If you wish you can substitute f0/f or E0/E for L/L0 but the h's still cancel.

Yes according to the standard model. Is it possible the physics community is wording their responses with half truths to maintain their model? Although your h cancels out of your equation r is variable * c in different frames while your equation does not address the gamma term between frames in relativity mathematics. Timey is addressing this fact while denying  its cause. Physics relativity is the same in every frame but not between frames.

Planck's length is directly dependent on Planck's time and visa versa. c is plank's time but r is relativity's distance. r can change while c remains the same. This would create photon's of different lengths in different gravity potentials. And create different tick rates.
Title: Re: Is there a discrepancy with the equivalence principle?
Post by: alancalverd on 21/03/2017 16:16:00
and where the value of p changes for already emitted light (in the gravitational field*) in relation to the h constant, this extra or lesser length in the wavelength isn't distance related anymore,
Your obsession with h is worrying. p doesn't change "in relation to h". p changes because the photon's kinetic energy changes as it travels through a gravitational field gradient. The value of h is irrelevant to the fractional change in p, E, L or f, which are all related through c and  given by the same equation (or its inverse, obviously).     
Title: Re: Is there a discrepancy with the equivalence principle?
Post by: alancalverd on 21/03/2017 16:22:55


Yes according to the standard model. Is it possible the physics community is wording their responses with half truths to maintain their model? Although your h cancels out of your equation r is variable * c in different frames while your equation does not address the gamma term between frames in relativity mathematics.

Nothing to do with models. It's a prediction from general relativity, which is based solely on the discovery that c is invariant, and is borne out by experiment. There is no room for half-truths in physics. Either your equation predicts what happens, or it doesn't.
Title: Re: Is there a discrepancy with the equivalence principle?
Post by: GoC on 21/03/2017 17:15:02
Alan:

   I agree c is invariant but space distance is not relative to mass gradient dilation. The clock tick rate measures the gamma difference in dilation (expansion).

With dilation the mass expands to measure a longer measuring stick length to match the clocks new tick rate. The ground state of the electron follows a longer volume in space matching the visual increase in SR. The SR angle of light in a light clock changes with speed changes for equivalency with GR reduction of available c.
Title: Re: Is there a discrepancy with the equivalence principle?
Post by: timey on 21/03/2017 17:24:43
and where the value of p changes for already emitted light (in the gravitational field*) in relation to the h constant, this extra or lesser length in the wavelength isn't distance related anymore,
Your obsession with h is worrying. p doesn't change "in relation to h". p changes because the photon's kinetic energy changes as it travels through a gravitational field gradient. The value of h is irrelevant to the fractional change in p, E, L or f, which are all related through c and  given by the same equation (or its inverse, obviously).     

Yes - that's right, p doesn't change in relation to h, but wavelength changes in relation to p in relation to h.

p is changing in relation to v, and v is changing in relation to a, where a is either increasing or decreasing due to a potential energy to kinetic energy conversion, (where relativistic mass in relation to kinetic energy for light in current physics remit ensures 0 acceleration/deceleration), but it is indeed p in relation to h that denotes the change in the wavelength...
Title: Re: Is there a discrepancy with the equivalence principle?
Post by: alancalverd on 21/03/2017 22:28:22
v doesn't change. We're talking about photons.
Title: Re: Is there a discrepancy with the equivalence principle?
Post by: timey on 21/03/2017 23:13:10
Which is why I said:
Quote
where a is either increasing or decreasing due to a potential energy to kinetic energy conversion
...for mass accelerated or decelerated in the gravity potential.

... And added this:
Quote
(where relativistic mass in relation to kinetic energy for light in current physics remit ensures 0 acceleration/deceleration)
For light in the gravity potential..

In either case:
Wavelength = h/p
where
p = mv
or
p = h*vbar
where
vbar = v/a

And I am looking at acceleration in the gravity potential being 3rd time dilation related, where I am also looking at GR time dilation being potential energy related, and considering that temperature energy added to the blackbody is increasing the rate that an atom of the blackbody emits a photon at, i.e. an increase in rate of electron transitions, and consequently the quantum energy level of the atom that then emits a higher energy, higher rate/frequency of photon.

Planck calculated the energy increases via an invariant second.  Calculate the temperature energy added via the rate (second) of the emitted frequency and the quantum nature is negated.
Title: Re: Is there a discrepancy with the equivalence principle?
Post by: Mike Gale on 21/03/2017 23:28:23
I'm lost. What happened to our free-falling traveller? We need to establish timey's prediction for dilation in that case to ensure that her theory is consistent with experiment. If that test fails, then all other debates (like this one about h) are moot.

I think we have the scenario boiled down to brass tacks. In order to ensure we are not pushing the boundaries of GR, we are analyzing a time slice in which a free falling traveller and a stationary bystander are at the same altitude and sufficiently removed from the event horizon that Newton's laws are valid. If the velocity vector is radial, GR says the bystander's clock runs slower by a factor of (1-2GM/c2r). This is consistent with SR if the traveller is falling at the escape velocity, which means v=0 at infinity. (Any other velocity invalidates the SC solution.) I think timey's theory predicts that the clocks run at the same rate because she attributes time dilation to altitude rather than velocity.
Title: Re: Is there a discrepancy with the equivalence principle?
Post by: timey on 21/03/2017 23:34:17
The prediction for the 3rd time dilation 'is' the acceleration or deceleration of gravity in relation to M.
Observed in action experimentally, and in everyday life.  Everyone knows that gravity accelerates, but current physics doesn't have a physical cause for the phenomenon.
I'm just giving a physical cause to an observation that we already make, that has previously been given no physical cause.
Title: Re: Is there a discrepancy with the equivalence principle?
Post by: Mike Gale on 22/03/2017 00:43:53
You're dodging the question. These observers are at the same altitude. Never mind what causes gravity to motivate things. What does your theory predict in terms of clock rates? Faster, slower or the same?
Title: Re: Is there a discrepancy with the equivalence principle?
Post by: Mike Gale on 22/03/2017 03:16:14
BTW - I described the GR cause (of accelerated motion) in Reply #338.
Title: Re: Is there a discrepancy with the equivalence principle?
Post by: timey on 22/03/2017 18:44:34
No - my model does add in SR time dilation.
You do make interesting point though, because I'm not sure what the answer is to your free faller versus bystander consideration, so having a look at it, but you'll have to realise this look of 'mine' won't involve the math.

However, if we place the bystander on the radial orbital velocity where SR time dilation equalises GR time dilation and the clock reads the same rate as a ground clock, the third time dilation would be inherent to the g-field at that radius and would be the equal but opposite value to GR time dilation, whatever that would be if SR time dilation at that velocity, at that radial orbit, hadn't cancelled GR effects out.
The craft+bystander them-self would not be registering this 3rd time dilation on the onboard clock, but the motion of the craft will be affected.

Now the question arises as to whom's clock the velocity of the craft is held relative to...
Is it the ground clock?
Is it the crafts clock?
Both are running at the same rate, but if we are holding the speed relative to the crafts clock, then we must appreciate that we are stating that both GR time dilation and SR time dilation are affecting the speed of that craft.
SR measurements can't affect the speed of the craft because if they did the speed of the craft would affect the time dilation, and the time dilation the speed of the craft, and we have a catch 22, so the question is, are we holding the speed of the craft relative to the ground clock, or to the GR time dilation portion of the crafts time?

On the basis that when time is held relative to the ground clock, the acceleration of 9.807m/^2 is a measurement held relative to the ground clock, when accelerating at 9.807m/s^2 on the ground in the unchanging gravity potential the distance covered per second squared is +9.807m...
The speed required to maintain an orbital at our bystanders radius, i.e. orbital velocity, held relative to the 3rd time dilation, which is the exact opposite of what GR time dilation would be at that radius relative to the ground clock, i.e. negative value, is causing the craft to cover 9.807m in a longer amount of time.
But both the ground clock and the bystander understand that that the bystander's clock is running faster, so  if both held the speed of the craft relative to the GR time dilation the craft's clock is experiencing, then both would think that the bystander was covering 9.807m in a quicker time.

Ok - so on the basis that near Earth approximation of gravitational acceleration is 9.807m/s^2, and at 2 radii from centre Earth it is around 4.25m/s^2, whatever the value is at the particular radial we are discussing, the radial where SR effects at that orbital velocity cancel out GR effects at h from M, this value of m/s^2 will be relevant to the orbital velocity required for that radial...
So the orbital velocity for each radius will be proportional to the m/s^2 value at that radius, for the reason that the crafts motion has been slowed by the longer seconds there, and it must increase its velocity in each increasing radius in order to counter these slower seconds, and this calculation must take into addition the extra distance involved in travelling the greater distance of that radius.

So according to both the ground clock and the bystander the bystander's speed may have been affected as an increased rate, where the craft is covering 9.807m of the radial distance at a faster rate to the exact value that the craft is actually covering 9.807m of radial distance at a slower rate...
If holding the speed of the craft relative to the 3rd time dilation, this would be a slower speed.
If holding the speed relative to the GR time dilation, this would be a faster speed.
But in both cases, an equal amount of distance, i.e. 9.807m would be travelled at those speeds in both of those rates of time because we are talking about it taking a longer or shorter amount of time to travel a distance.
However... according to the ground clock the bystander and his craft would be covering 9.807m of radial distance at a speed held relative to the ground clock's rate of time, where 9.807m of radial distance covered in a faster time, or covered in an equal value slower time is being calculated as a distance covered by a speed as per the ground clock. i.e. orbital velocity.
So the difference between escape velocity for a particular radius and orbital velocity for that particular radius would be mathematically significant in relation to the 3rd time dilation.
For instance, if we were to say the bystander was orbiting the radius of 2 radii from centre of Earth, the acceleration of gravity at that radius is around 4.25m/s^2.  Where my model is saying that if we were to calculate this acceleration as per the second squared of that radius, this would be equal to an acceleration of 9.807m/s^2 at a near Earth radius at near Earth's rate of time.
In other words it just takes a longer amount of time to travel same distance, and it is only by calculating using an invariant time to measure an acceleration, that the distance travelled is reduced to being a 4.25m/s^2 acceleration.
Therefore by remit of the 3rd time dilation, the orbiting craft would only be covering 4.25m of radial distance for every second squared as per held relative to the ground clock and the speed of the craft as held relative to the ground clock would need to be increased in order to still be covering 9.807m of radial distance in the same amount of ground clock time.

It is also worth noting that where current physics holds the SR time dilation effects experienced by the craft as a speed measurement held relative to the ground clock, via the speed of the craft being a percentage of the speed of light held relative to the ground clock - my model holds the SR time dilation effects held relative to the 3rd time dilation, where the speed of the craft is a percentage of the speed of light held relative to the 3rd time dilation.
This will change the value of the SR effects being experienced by the bystander, but does not affect the rate of the speed that the bystander is covering distance at, not from an observer's point of view.  It will be the rate that the observer's own clock is ticking at that will influence his assessment of the crafts speed.
This also means that any spatial dilation that the free faller and bystander experience is a product of their own rate of time, and all actual distances remain constant.

So to sum up, the bystander at the 'current physics' radius where SR effects held relative to the ground clock are cancelling out GR effects, in my model - will this be at a radius that is closer to M?
This being for the reason that when holding the SR time dilation effects of that radius's orbital velocity as a percentage of the speed of light held relative to the 3rd time dilation at that velocity, the speed will be a higher percentage of the speed of light at that radius, and SR effects will be increased.
But... If we hold the speed the craft is moving at relative to the 3rd time dilation at that radius, the speed itself is reduced...
By calculating the reduced speed as a percentage of the speed of light held relative to the 3rd time dilation, we should end up with the same proportion of SR effects as when calculating the speed held relative to the ground clock as a percentage of the speed of light held relative to the ground clock, so the SR effects in my model would in fact be at the original radius, and synonymous to that which relativity predicts.

But enough of the bystander aye ;)
The free falling observer would, in my model, have to start from somewhere tangible.  Let's say we dropped him at a radius where GR time dilation is twice what it is at the radius where orbital velocity SR effects cancel out the GR effects...
The free falling observer's clock is running 'that much' faster than the bystanders clock, and the bystanders clock is running at the same rate as the ground clock.

Now the question is, what speed will the free falling observer be travelling at by the time he reaches the bystanders radius?
The 3rd time dilation decrease in length of seconds at each decreasing radius in the g-field will have accelerated his speed...
Therefore whatever the value of m/s^2 acceleration existing at the radius we dropped him from in relation to what value m/s*2 acceleration exists at the bystander's radius is relevant.
However, we must recognise that when we say m/s^2, we are holding that measurement relative to the ground clock. 

The free faller's clock will have been being decreased at each decreasing radius in its rate of time as per GR time dilation...  And at the bystander's radius the free faller's clock will be the GR time dilation at that radius, minus the SR effects due to the velocity he is travelling at.
The bystander and the free faller will be experiencing exactly the same GR time dilation effects, but the experience of their own time will differ due to travelling at differing velocities.*
However, the SR effects that the free faller will be subject to will be greater at the more distant radius.  This being because the SR effects related to his speed will be being held relative to his speed being a percentage of the speed of light held relative to the 3rd time dilation length of second at that radius.  But again - we can calculate the speed the free faller is falling at as held relative to the 3rd time dilation length of second  at each radius, and this will result in the same value as holding the speed of the craft relative to the ground clock, and the speed of the craft as being a percentage of the speed of light held relative to the ground clock.

*A scenario that would be apparent if we dropped the free faller from a radius that was only a few metres above the bystander's radius, where the sequential time involved in a shorter distance for the free faller to accelerate within would be in shorter supply, resulting in a lesser acceleration.

Here we come to the crux of the matter...
Who's time are we holding the sequential time relative to?
If we are not holding speeds and distance subsequently travelled relative to time dilated clock's, then we are not holding sequential time relative to time dilated clock's, and on this consideration we must remember that the ground clock is ideas itself a time dilated clock.  There is nothing special about the ground clock's time, other than the fact that a lot of measurements are held relative to it.
We did make it simple for ourselves because the bystanders clock is running at same rate as the ground clock, but it is doing so for different physical reasons.
So - if we slice the moment in time that the free faller happens to pass through the bystanders radius at the moment the bystander is passing the free faller's location at that radius, the bystander will see that the free faller's clock is likely running a tad faster, dependent on the value of the radius he was dropped from, and the free faller will see the bystanders clock running a little slower, again dependent on the radius he was dropped from.
As far as the bystander is concerned the free faller is falling at a speed that is increasing by m/s^2.
As far as the free faller is concerned, the distance that he has fallen is increasing by m/s^2...
Again we must note that this m/s^2 measurement is being held relative to the ground clock, and that the bystander's clock is running at same rate as the ground clock.
Both the ground and the bystander are in agreement with each other regarding speed, sequential time, and m/s^2, and therefore distance travelled, but they are doing so for differing reasons.

The case for the free faller on the other hand is that both the ground clock and the bystander's clock agree that the free faller's clock is running fast.  The free faller when at the bystander's radius on the other hand agrees that both the clock on the ground and the bystander's clock are running at same rate, this being a bit slower than his own.
Now the question here is raised in how one interprets the speed the free faller is covering distance at...?
Are we saying that the free faller is covering distance by the remit of a second as held relative to the ground clock?
Or are we saying that the free faller is covering distance by his speed held relative to his own clock?
My model is saying that the free faller is covering distance by his speed held relative to the 3rd time dilation.
The free faller covering distance as per his speed held relative to the 3rd time dilation, would be travelling distance at a constant speed, in variable times that cause the speed to appear accelerated when measured via an invariant time, such as the ground clock's time, and also in this case the bystanders time when at that radius, at that orbital velocity.  When holding both speed and SR effects relative to the 3rd time dilation, we arrive at a constant speed to deduce the SR effects from*, (this being the same result as calculating an increased speed with the speed being held relative to a percentage of the speed of light held relative to ground clock), and we may now deduct these SR effects from the GR time dilation effects at that radius for a clock time for the free faller.
(*the free faller's acceleration from a starting point of 0 speed will be 0, and this can't be correct because he will be going nowhere, but his clock's rate will be decreasing, so by holding the speed of 0 relative to his decreasing clock rate, this negative approach to motion will result in a distance travelled, and therefore a speed)

So to sum up - a slice of the free faller's time may be, dependent on the distance between the radius that he was dropped from, slightly slower, or slightly faster than the bystander's clock, where in that instant of sequential time both the free faller and the bystander observe each other.
My model states that it is of no significance that each observes the other as having a differing rate of time because they both share a universally common present moment, where the observation of 1 rate of time from the other is proportional to the difference in rate and will appear quantised, or discrete.
For instance, if we dropped our free faller into free fall at 0.806c as per held relative to the ground clock, the ground clock would observe that speed to be accelerating as the radius decreased, the bystander would observe that speed to be accelerated as per the ground clock does due to the clock's running at same rate, but the free faller would not experience any acceleration of his speed, his speed would remain constant as far as his accelerometer is concerned* but his rate of time will be decreasing as per GR time dilation, and as his speed becomes a lesser percentage of the speed of light held relative to the increasing length of seconds at decreasing radius where the SR effects affecting the free faller will then be reducing.  GR time dilation is elf will be reducing into longer seconds, and SR time dilation effects that cause longer seconds will be reducing...

So we have a GR phenomenon that speeds a clock's time up observed as reducing, and an SR phenomenon that when held relative to the 3rd time dilation, results in slowing a clock down, that is reducing.  One can see a balance in that scenario where it is possible that the speed of the free faller may be held relative to the rate of time of the free faller's clock for a constant speed.
Lastly, when the bystander observes the free faller who is now travelling 0.806c as per held relative to the ground clock, in the slice of the moment of time that they observe each other, the bystander will observe that the free faller has become length contracted, because the bystander can only view a proportion of the free faller's rate of time.  And the free faller will observe that the bystander has become length contracted because the free faller can only view a proportion of the bystander's time.  What they are actually viewing of each other is just portions of each other's time, but the motion will blur these discrete packages of the image.  If neither were actually moving in relation to each other, which would of course change the remit of the time dilation difference, but let's say that this impossible situation of an observation of 2 very differing rates of time could be observed at close proximity to each other, then the slower time would observe gaps between action of the faster time, and the faster time would observe action missing of the slower time.
The important bit being that we can consider that sequential time is like a light cone scenario, where different rates of time can share the same present moment.
This is not so different a perspective than that of calculating quantum states...

Continued in next post...
Title: Re: Is there a discrepancy with the equivalence principle?
Post by: timey on 22/03/2017 18:45:17
And this is already far too long a post despite my edits, but to go the whole hog:

We can now examine what one observer would observe of the other if we were to send a light signal from each observer to the other, but the free faller being dropped into free fall at 0.806c is unrealistic, I only used that to illustrate an exaggerated proportional observation, but again we can use this exaggeration as an illustration of such.

According to my model both the free faller's and the bystander's light emitters will be emitting a photon as per the electron transitions caused by the sum of SR and GR time dilation effects that are affecting each of them.
At the point in sequential time, (which we have established is the same for both free faller and bystander because both agree that the same amount of distance is being travelled by the free faller, no matter if they cannot agree on the speed, or a time unless referring back to the ground clock(current physics), or 3rd time dilation (my model), where we now have the bystander and the free faller both emit a single photon.
The distance between the free faller and the bystander is 1000 metres, they share the same gravity potential at their locations at this radius, their GR time dilation is equal but they have a vast difference in SR effects going on, due to travelling at differing speeds.
In my model the 2 respective photon's will travel the 1000 metre distance between the bystander and the free faller, and the free faller and the bystander, at a speed of light held relative to the 3rd time dilation of that radius.
This doesn't really matter very much over a distance of 1000 metres within  an unchanged gravity potential, the light will just take that much longer to travel the distance...
But the ground clock, the bystander and the free faller will deduce that the distance between the bystander and the free faller is greater than 1000 metres if they are holding the speed of light relative to the ground clock.
If the bystander holds the speed of light relative to his own clock he will agree with that the distance is greater than 1000 metres to the same proportion that the ground clock is, but this is taking into account both the GR and SR effects on his clock.  If he were at a differing radius he would have a perspective differing from the ground clock...
The free faller is experiencing a lot more SR time dilation on his clock than the bystander, so according to his clock, the bystander is a much greater distance away than the distance that both the bystander and the ground clock agree upon, but only if he measures the speed of light as held relative to the ground clock with respect to distance.  If he holds the speed of light relative to his own clock with respect to distance, the bystander will be much closer.  Perhaps closer than the 1000 metres of actual distance between them in fact...
If we hold the speed of light relative to the GR time dilation only, then both the bystander and the free faller will agree on the distance between them, but they will as a result have to hold their own speeds relative to their GR time dilation.  The ground clock will disagree on the distance travelled, unless the ground clock also holds the speed of light relative to the GR time dilation, in which case the ground clock would also have to hold the speeds travelled relative to the GR time dilation.  When applying SR to achieve curvature of space, as per current physics remit, under the remit of holding the speed of light and speed of craft/free faller relative to GR time dilation, SR must also hold its measurements to the speed of light relative to the GR time dilation, and the GR time dilation coordinate becomes a catch 22 under the remit of spatial dilation.

My model, by attaching the speed of light to the 3rd time dilation, and the speed of the craft to the 3rd time dilation, ensures that distance remains constant where perceived spatial dilation is a temporal perception due to the rate of time on one's clock.  The g-field remains unchanged in relation to M and the 3rd time dilation also remains unchanged with relation to M, so it doesn't really matter what anybody's clock is doing really, the important thing is that no matter who's time one is measuring from, the distance covered always remains the same.

Finally - while noting that the gravity potential between bystander and free faller is equal at the moment in time we ask each to emit photons, the photon's emitted by the emitter in the faster rate of time of the bystander will be of higher energy and frequency than the photon's emitted by the emitter in the free faller's slower rate of time.
What one observes of the others photon's will be a proportional observation, where missing action, or gaps between action will cause each to observe a frequency of the others photon's that is not the frequency the photon's were emitted at, but in reality, there would be no such case as a free faller free falling from a start speed of 0.806c, and the difference in frequency between the photon's emitted in locations of differing rates of time between bystander and free faller would be minimal and barely tangible.

Crikes Mike... (chuckle) you got me right at it there let me tell you...
Title: Re: Is there a discrepancy with the equivalence principle?
Post by: timey on 22/03/2017 18:54:43
In addition to my 2 posts above outlining my model's remit of the bystander and the free faller...

BTW - I described the GR cause (of accelerated motion) in Reply #338.

Here is post 338:

Zero velocity is a perfectly valid initial state. That's how Newton's apple got started for example. Newton makes no hypothesis about how the force of gravity gets things moving. The GR view is that a stationary object moves through time at light speed and acceleration simply rotates the velocity vector into the spatial domain. It's a change of heading in the spacetime continuum.

In my simplified scenario, the traveller is in free fall and the bystander is accelerating. That change eliminates the need for variable thrust, which is a significant complication.

Variable light speed is one possible interpretation of GR. Some people prefer to think of it as spacetime dilation, akin to SR. In either case, we have eliminated the effect by placing the observers at the same altitude at some instant of time.

If mass, space and light speed are invariant then higher energy (potential or kinetic) equates to shorter seconds.

Anyway, that's the picture painted by GR. The question is, what does it look like in your theory?

I don't see that you have given a physical cause for the acceleration of gravity here... What is the physical cause of the acceleration?
...and if higher kinetic energy leads to shorter seconds then a) why is light of higher kinetic energy near M than further away, and b) why does a clock at rest with respect to the g-field tick faster at h from M?
Title: Re: Is there a discrepancy with the equivalence principle?
Post by: timey on 22/03/2017 22:02:48
On the basis that this conversation wasn't continued where you have said this in response to my post:
and where the value of p changes for already emitted light (in the gravitational field*) in relation to the h constant, this extra or lesser length in the wavelength isn't distance related anymore,
Your obsession with h is worrying. p doesn't change "in relation to h". p changes because the photon's kinetic energy changes as it travels through a gravitational field gradient. The value of h is irrelevant to the fractional change in p, E, L or f, which are all related through c and  given by the same equation (or its inverse, obviously).     
...and I have replied:
Yes - that's right, p doesn't change in relation to h, but wavelength changes in relation to p in relation to h.

p is changing in relation to v, and v is changing in relation to a, where a is either increasing or decreasing due to a potential energy to kinetic energy conversion, (where relativistic mass in relation to kinetic energy for light in current physics remit ensures 0 acceleration/deceleration), but it is indeed p in relation to h that denotes the change in the wavelength...
And you replied:
v doesn't change. We're talking about photons.
Where I then said:
Which is why I said:
Quote
where a is either increasing or decreasing due to a potential energy to kinetic energy conversion
...for mass accelerated or decelerated in the gravity potential.

... And added this:
Quote
(where relativistic mass in relation to kinetic energy for light in current physics remit ensures 0 acceleration/deceleration)
For light in the gravity potential..

In either case:
Wavelength = h/p
where
p = mv
or
p = h*vbar
where
vbar = v/a

And I am looking at acceleration in the gravity potential being 3rd time dilation related, where I am also looking at GR time dilation being potential energy related, and considering that temperature energy added to the blackbody is increasing the rate that an atom of the blackbody emits a photon at, i.e. an increase in rate of electron transitions, and consequently the quantum energy level of the atom that then emits a higher energy, higher rate/frequency of photon.

Planck calculated the energy increases via an invariant second.  Calculate the temperature energy added via the rate (second) of the emitted frequency and the quantum nature is negated.

Do you have anything to say?
Title: Re: Is there a discrepancy with the equivalence principle?
Post by: Mike Gale on 22/03/2017 23:17:57
I haven't nodded off. Well, technically I did, but I haven't abandoned this thread. I think we are in a different time zone or there was a delay in the forum engine.
Title: Re: Is there a discrepancy with the equivalence principle?
Post by: timey on 22/03/2017 23:37:37
Hey no problem - I am on UK time, but sometimes I stay up to talk to you.  My reply to your 'dodging an answer' post is somewhat long winded, I'll admit...

P.S.  The site isn't refreshing New Theiries threads to the recent topic's page properly, so if you are looking there to see if anyone has posted... it may be that they have but it isn't showing on that page.
Title: Re: Is there a discrepancy with the equivalence principle?
Post by: Mike Gale on 22/03/2017 23:39:36
The reason I'm trying to change your scenario is that GR gives ambiguous results when the observer is using a rocket engine to overcome (or augment) gravity and SR can't cope with orbital motion. You may have a clear picture of how your theory deals with those cases, but mainstream physics does not. It is therefore impossible to make comparisons unless you concede the simplifications I propose.
Title: Re: Is there a discrepancy with the equivalence principle?
Post by: timey on 22/03/2017 23:47:55
I am not sure what the simplifications you wish to make are...
Is it setting speed at 0 on the outbound?
Title: Re: Is there a discrepancy with the equivalence principle?
Post by: Mike Gale on 22/03/2017 23:57:47
As to the cause of spooky action at a distance, GR and SR interpret acceleration as a change of heading in the spacetime continuum. How a force like gravity changes one's heading is not known. QM postulates an exchange of particles called gravitons, which are essentially photons going backwards in time. However, that explanation only leads to more questions. You ask, what causes 'A'? The answer is 'B'. Then you ask, what causes 'B'? The answer is 'C'. Ad infinitum. At the end of the day, it all boils down to observables and causal relationships. We can observe that your clock is running slower than mine for example. We propose a relationship that explains that observable in terms of another, such as our relative velocity. Then we do experiments to see if there are cases where the relationship doesn't hold. That's the best we can do. You can imagine a cause for the 2nd observable (e.g. goblins and fairies), but there is no way to determine if it is true. That's the dilemma of string theory for example. It seems plausible, but there's no way to test it.
Title: Re: Is there a discrepancy with the equivalence principle?
Post by: timey on 23/03/2017 00:15:54
GR is the best theory we have of gravity, but all it does is give a mathematical representation that is the best fit to observation.
This is why my model sets out to use the same mathematics remixed into a differing arrangement that also describes observation.

In very simple terms all my model is doing is trading the spatial dilation that an observer observes to being a temporal dilation that is caused by the attributing the force of the acceleration of gravity to being 3rd time dilation related, where the 3rd time dilation is caused by the strength of the g-field in relation to M, and the curvature of space is a temporal dilation of flat space.
The result of this change knocks on to GR time dilation which becomes a phenomenon that is only affecting m in relation to M, and this is gravity potential energy related.
We then take this change to GR time dilation back to the blackbody and note that temperature energy is causing emissions of photon's that get higher in frequency as more energy is added.  Frequency is held relative to an invariant time.  Hold frequency relative to variable time, as the frequency escalates, seconds get shorter, and if you calculate the energy added per second held relative to the shorter seconds, the quantum nature deduced via the ultra violet catastrophe is negated.
Title: Re: Is there a discrepancy with the equivalence principle?
Post by: Mike Gale on 23/03/2017 02:24:29
I am not sure what the simplifications you wish to make are...
Is it setting speed at 0 on the outbound?
As I described in Reply #360, a free falling traveller and a stationary bystander (the latter supported by a planetary crust) are at the same altitude and sufficiently removed from the event horizon that Newton's laws are valid. The velocity vector points radially inwards and is equal to zero if the world line (i.e. flight path) is extrapolated back to an infinite distance, in which case traveller speed at the altitude in question is the so-called escape velocity: sqrt(2GM/r). As a further clarification, traveller velocity is nowhere near light speed.

GR and SR predict that the traveller perceives the bystander clock to run slower by a factor of (1-2GM/c2r) or, more accurately (if you buy into my relativistic correction to the SC solution), sqrt(1-v2/c2). I think your theory predicts that the clocks run at the same rate because it attributes time dilation to altitude rather than velocity.

Note that many GR theorists (I dare say most) make that mistake, too. They usually compensate for their error by invoking SR again and then adding that dilation to the (null) GR result. (I say "again" because GR already incorporates SR if it is interpreted correctly.) You'll see shenanigans like that in the GPS literature for example. It's hard to talk them down because they do get the right answer. It's a case of two wrongs making a right.
Title: Re: Is there a discrepancy with the equivalence principle?
Post by: timey on 23/03/2017 03:36:54
I was talking about this on 'my model of a cyclic universe continued', where I had been studying this:

https://en.m.wikipedia.org/wiki/Parabolic_trajectory

If
v = square root 2* v orbital
and
d = v*0.4142
where
d/v = 3rd time dilation

Where d is the distance that 'hasn't' been travelled due to the 3rd time dilation, i.e. flat space, temporal curvature...
I'm not so strong on maths though Mike...
The idea is that the speed is constant, it's just the time dilation that accelerates, or decelerates the speed.

If the craft started out at 0 speed, then any acceleration it experiences is 3rd time dilation related.

According to a diagram I made on the square to diagonal, if we hold distance and time constant on the square, and run time dilation up the diagonal, I worked out via dividing the geometry of my diagram into constants, that a temporal dilation can be placed into a Newtonian geometry, and that the time dilation is the value of 0.4142.

So the equation you post where you say
Quote
in which case traveller speed at the altitude in question is sqrt(2GM/r).
This speed at each radius can be treated to the maths above (I think, scratches head)

In reply to your edit:
No - you keep on overlooking the fact that my model, despite describing the curvature of space as a temporal dilation of a flat space, this temporal dilation rendering space coordinates as infinitely more definable, still retains the SR effects as a phenomenon that the traveler travelling at that velocity experiences himself.
SR effects are held relative to the 3rd time dilation where the velocity is a percentage of the speed of light held relative to the 3rd time dilation length of second at that radius.
SR time dilation effects will cancel out GR time dilation effects, and SR spatial dilation will be experienced by the traveler due to his slower rate of time, but the velocity he is travelling at, and the speed of light his SR time dilation is held relative to, are both being held relative to the 3rd time dilation length of second at that coordinate.
Title: Re: Is there a discrepancy with the equivalence principle?
Post by: Mike Gale on 23/03/2017 03:43:58
OK, if I understand correctly, your theory predicts no time dilation for these observers until you invoke SR. I can't fault you for that because, as I said, GR theorists make the same mistake and I am probably in the minority when I express the opinion that they're wrong. It would be nice to have a 2nd opinion from a scholar on this subject (where's Alan Guth when you need him?), but I guess it's off the table for now. I'll have to do some more noodling.
Title: Re: Is there a discrepancy with the equivalence principle?
Post by: timey on 23/03/2017 03:54:31
No - I'm saying that GR time dilation and the 3rd time dilation are separate phenomenon that occur simultaneously.
That GR time dilation affects mass only at h from M, and at that h from M the 3rd time dilation will be the exact and equal negative of GR time dilation, where the 3rd time dilation has no effect on the time dilation for mass, but it will affect how that mass moves through space.
SR is then added in for mass only, where SR effects will cancel out GR effects and vice versus.
Title: Re: Is there a discrepancy with the equivalence principle?
Post by: Mike Gale on 23/03/2017 03:57:37
Understood, but the test mass 'm' does not factor into the GR equations so I'm still at a loss to explain your distinction between dilation for m at h from M and dilation for nothing at h from M. That's the missing recipe I was talking about earlier. As I said, I need to do some more noodling.
Title: Re: Is there a discrepancy with the equivalence principle?
Post by: timey on 23/03/2017 04:14:34
Near Earth the acceleration of gravity is 9.807m/s^2.
At 2 radii from centre of Earth, at this radius the acceleration of gravity is 4.25m/s^2.
If you held the measurement of this acceleration relative to the length of a 3rd time dilation second at that radius, then the acceleration would be 9.807m/s^2.
(roughly speaking)

The current GR time dilation perception is that this GR time dilation is inherent to open space where the rate of time gets faster at the higher gravity potential, and that mass in that open space at a particular gravity potential will be affected by that time.
My model changes this and states that GR time dilation only affects the mass via gravity potential energy, and that it is the addition of this potential energy that increases the rate of the electron transitions. i.e. frequency.
Then it states that the g-field of open space itself is inherent with the 3rd time dilation that affects how this mass moves through open space.

I'll leave you to it though, and happy noodlings...
Title: Re: Is there a discrepancy with the equivalence principle?
Post by: alancalverd on 23/03/2017 08:14:04
So let's put a clock at 2R. Now put one observer on earth and another on the moon. What do they see?
Title: Re: Is there a discrepancy with the equivalence principle?
Post by: timey on 23/03/2017 13:12:58
The clock at 2r will be GR time dilated, so conventional relativity describes what the moon observer will see in relation to what the Earth observer will see - is the answer to your question.

The observation of the 3rd time dilation would be in that the clock without any energy input to maintain its position at 2r, would then be changing position from 2r back to Earth, where the increasingly shorter seconds of the g-field are increasing the speed the clock moves at through the g-field.
Title: Re: Is there a discrepancy with the equivalence principle?
Post by: GoC on 23/03/2017 14:46:27
So let's put a clock at 2R. Now put one observer on earth and another on the moon. What do they see?

2R from the center of the Earth, from the surface of the Earth or from the atmosphere of the Earth?


The clock at 2r will be GR time dilated, so conventional relativity describes what the moon observer will see in relation to what the Earth observer will see - is the answer to your question.

The observation of the 3rd time dilation would be in that the clock without any energy input to maintain its position at 2r, would then be changing position from 2r back to Earth, where the increasingly shorter seconds of the g-field are increasing the speed the clock moves at through the g-field.

What does GR dilated mean in space. Where is the threshold of attraction for the difference in the inverse square of the distance? There is one for inside the parameter of mass and one for outside the parameter of mass.
Title: Re: Is there a discrepancy with the equivalence principle?
Post by: alancalverd on 23/03/2017 17:38:34
R is always taken from the centre of gravity, provided that R > the radius at mean sea level. It's a lot easier than asking where the atmosphere ends, because it doesn't.
Title: Re: Is there a discrepancy with the equivalence principle?
Post by: timey on 23/03/2017 18:56:32
So let's put a clock at 2R. Now put one observer on earth and another on the moon. What do they see?

The clock at 2r will be GR time dilated, so conventional relativity describes what the moon observer will see in relation to what the Earth observer will see - is the answer to your question.

The observation of the 3rd time dilation would be in that the clock without any energy input to maintain its position at 2r, would then be changing position from 2r back to Earth, where the increasingly shorter seconds of the g-field are increasing the speed the clock moves at through the g-field.

Any response?
Title: Re: Is there a discrepancy with the equivalence principle?
Post by: Mike Gale on 24/03/2017 00:02:42
So let's put a clock at 2R. Now put one observer on earth and another on the moon. What do they see?
That's a tough one because the moon's gravity is not trivial and it's orbital speed is variable. The GPS scenario is similar and simpler because the satellite's gravity is trivial and it's orbital speed is effectively constant. A GEO satellite is even easier because it is stationary with respect to the Earth observer. In that case, GR says the satellite clock runs faster or slower by a factor of sqrt(1-2GM/c^2/R)/sqrt(1-2GM/c^2/r) and you can take that result to the bank. (https://en.wikipedia.org/wiki/Gravitational_time_dilation (https://en.wikipedia.org/wiki/Gravitational_time_dilation))
Title: Re: Is there a discrepancy with the equivalence principle?
Post by: timey on 24/03/2017 01:32:03
Ok - so if we could imagine for the purpose of the calculation that a constant speed - never mind the cause of the movement or consideration of that which is moving - to be moving in the direction of r, away from M, would it be possible for you to calculate the acceleration that a constant speed would accelerate at if the speed were held relative to the faster seconds of each r between R and r?

Edit: This assuming that R is the closer radius, I've had a moment of doubt, but if I've got it wrong way round...
Title: Re: Is there a discrepancy with the equivalence principle?
Post by: Mike Gale on 24/03/2017 03:05:34
R is indeed the shorter radius, but I've had my own 2nd thoughts. Even though the satellite is stationary with respect to the Earth observer, both are rotating within the SC coordinate system. It's not valid to assume that the SC coordinate system rotates with the Earth unless the satellite is replaced with a hovering rocket (co-rotating with the Earth observer) or another terrestrial observer at the top of a tall tower. Centripetal acceleration does not amount to the same thing because the satellite does not feel its own weight. We should revise our scenario accordingly to avoid that wrinkle. Constant linear velocity is even more complicated because you have to accelerate in a gravitational field in order to achieve that.
Title: Re: Is there a discrepancy with the equivalence principle?
Post by: GoC on 24/03/2017 10:57:38
R is always taken from the centre of gravity, provided that R > the radius at mean sea level. It's a lot easier than asking where the atmosphere ends, because it doesn't.

Yes but the density of mass has a threshold where attraction to the Earth for your weight to the center of mass changes abruptly. Being basically weightless in space. There is an abrupt change in tick rate of your clock that is not linear to the center of mass. Basically insignificant change but a change non the less. This is where the gradient dilation changes its acceleration to the center of the Earth.

There probably is no place in the universe where the tick rate remains the same so while we can be somewhat accurate we can never be precise. A mathematicians goal of precision in relativity is a moving target.
Title: Re: Is there a discrepancy with the equivalence principle?
Post by: timey on 24/03/2017 11:15:29
R is indeed the shorter radius, but I've had my own 2nd thoughts. Even though the satellite is stationary with respect to the Earth observer, both are rotating within the SC coordinate system. It's not valid to assume that the SC coordinate system rotates with the Earth unless the satellite is replaced with a hovering rocket (co-rotating with the Earth observer) or another terrestrial observer at the top of a tall tower. Centripetal acceleration does not amount to the same thing because the satellite does not feel its own weight. We should revise our scenario accordingly to avoid that wrinkle. Constant linear velocity is even more complicated because you have to accelerate in a gravitational field in order to achieve that.

Even the top of tower will be rotating faster than the bottom of tower...
The only way, as you say, is to have a hovering rocket that when maintaining height above the Earth, also maintains the same speed of rotation, where the observer on the craft will observe the observer on the ground rotating away from the crafts position, and therefore the speed of both will remain the same.

And yes if we take the gravitational field into consideration this does complicate matters for a trajectory away from M...
However - this is why I said to just consider the calculation as a constant speed - no matter what is causing the motion, or consideration of that which is being moved - to remove gravitational and SR aspects from the calculation.
What I'm interested in is by how much GR time dilation would increase a constant speed between R and r if we held the speed relative to the shorter seconds at each h from M.
What I'm looking for is a m/s^2 acceleration - so having taken into consideration that the shorter seconds of GR time dilation have accelerated this constant speed, clearly the 'acceleration' part described by m/s^2 would then be being held relative to the rate of an 'Earth second' at R, or 'standard second' as per the physics remit of holding metres per second squared relative to the standard second.

Edit:  It would be pertinent to my intended direction as to the use of this calculation if you considered r to be 2 radii from centre of Earth...
Title: Re: Is there a discrepancy with the equivalence principle?
Post by: Mike Gale on 25/03/2017 00:11:36
The point is to get both observers attached to the planet by gravity at different altitudes. That's the simplest case for the SC solution because the metric reduces to: dT/dt = sqrt(1 - 2GM/c2r). Anything else involves the addition of more variables, which only serve to obfuscate the result. This effect can be measured directly and is actually used to maintain the accuracy of the clocks that track universal time (during seismic events for example.) It can be interpreted as a change in light speed, but most people prefer to think of it as a change in time speed (i.e. time dilation.) The net effect is the same either way because clock speed depends on light speed as well as time speed. Note that acceleration due to gravity is GM/r2.
Title: Re: Is there a discrepancy with the equivalence principle?
Post by: Mike Gale on 25/03/2017 03:47:03
...just consider the calculation as a constant speed - no matter what is causing the motion, or consideration of that which is being moved - to remove gravitational and SR aspects from the calculation.
SR describes constant velocity in the absence of gravity. GR describes free fall in a gravitational field. It can be extrapolated to describe suspension in a gravitational field, but there is no exact GR solution for any other case, only numerical approximations.

What I'm interested in is by how much GR time dilation would increase a constant speed between R and r if we held the speed relative to the shorter seconds at each h from M.
What I'm looking for is a m/s^2 acceleration - so having taken into consideration that the shorter seconds of GR time dilation have accelerated this constant speed, clearly the 'acceleration' part described by m/s^2 would then be being held relative to the rate of an 'Earth second' at R, or 'standard second' as per the physics remit of holding metres per second squared relative to the standard second.
I think you're talking about free falling from the top of the tower. (Although that's accelerated motion, not constant velocity.) It's easy to compute time dilation for that case relative to an infinitely removed observer, but not so easy when the bystander is on the ground. The SC solution can cope with two free falling observers or two suspended observers, but not one of each. That's the GPS scenario and there is no consensus on the correct way to solve that problem.
Title: Re: Is there a discrepancy with the equivalence principle?
Post by: timey on 25/03/2017 23:39:15
No you misunderstand my intention - I'm not referring to any 'actual' physical event in wanting to know by how much a constant speed would be accelerated by in m/s^2, if the constant speed were held relative to GR time dilation at every r, from R to r=2radii.

Excluding the other physical considerations, all I want to know is - if we reduce the length of a second, how much faster will the speed be?
At each and every r, from R to r=2radii, GR time dilated seconds get shorter. (or appear as if they do from the lower potential)
What would the acceleration be in m/s^2 if we held a constant speed relative to the shorter seconds at each radius.

Just to be absolutely clear:
When referring to a speed it is no mystery that a speed is defined as per a distance held relative to a time period.  In this case metres travelled per second, and it should be pretty obvious that metres per second are being held relative to the time period of a standard second measurement.
So on this basis we can now hold a constant speed relative to a series of increasingly shorter seconds found to be shorter at each and every r between R and r=2radii, and this will give us an acceleration.
It is the value of this mathematically contrived acceleration that I need to know.

At 2 radii from centre of Earth the acceleration of gravity is 4.25m/s^2, or thereabouts.
At near Earth it is around 9.807m/s^2.
At each and every r between R and r=2 radii, the acceleration decreases.

I want to examine the difference in value between the 'gravitational acceleration' of m/s^2 occurring between r=2radii and R, in relation to this mathematically contrived 'time dilation' acceleration between R and r=2radii - that I'm trying to get you to calculate the value of...

This is actually pertinent to the calculation of my model, where the 'acceleration' of gravity is 3rd time dilation related... In order to calculate my model, each component of the GR field equations must be dissembled and reassembled in an alternative arrangement.  Once I know the value of this 'mathematically contrived' acceleration, I'll be in a better position to proceed.
Title: Re: Is there a discrepancy with the equivalence principle?
Post by: Mike Gale on 27/03/2017 00:03:51
Here's a cheat sheet I made for calculating GR time dilation with the Schwarzschild metric for cases that involve only radial motion. Of particular interest to this thread are the stationary and constant velocity cases, but I'm not entirely satisfied with those two results because they involve additional energy, beyond that which is available in the metric. I think that invalidates the interpretation of proper time as local time. (That's why GPS is so tricky for example.)
Note also that the validity of the Schwarzschild metric in the vicinity of the event horizon is questionable, as I pointed out in this thread: https://www.thenakedscientists.com/forum/index.php?topic=69764.0
If you accept my proposed correction to the metric, the scaling factor changes from (1-rs/r) to (1-rs/2r)2.
I should also point out that my free fall solutions are only valid for escape velocity (i.e. when KE cancels PE.) Anything else exceeds the energy in the metric.
Title: Re: Is there a discrepancy with the equivalence principle?
Post by: Mike Gale on 27/03/2017 04:04:35
Do you (Timey) think this is a fair assessment of your theory?
https://www.thenakedscientists.com/forum/index.php?topic=69882.msg510239#msg510239
Title: Re: Is there a discrepancy with the equivalence principle?
Post by: timey on 27/03/2017 17:53:46
Do you (Timey) think this is a fair assessment of your theory?
https://www.thenakedscientists.com/forum/index.php?topic=69882.msg510239#msg510239

I think Timey is proposing that spacetime is intrinsically dilated and that's what determines the location and movement of objects in space and time. It's an anthropic argument (i.e. chicken vs. egg) so it's really a matter of philosophy, not physics. The same could be said of string theory of course, but physics is supposed to be about the observables. It seeks to explain how one observable changes in relation to another. An explanation is worthless if it invokes undefined concepts like photon-to-electron ratio, electron cycle, electron travel distance, and oscillating mass. You might as well be talking about gnomes and fairies (or strings.)
I expect you're eluding to wave-particle duality. If so, you are way off base because the amplitude of a matter wave is a distance in probability space, not conventional space. That is, matter wave power, which is proportional to amplitude squared, equates to a probability of finding a mass at a given location in space and time. GR and SR have nothing to say about any of that.
At the risk of adding to your confusion, I should add that probability space is only one possible interpretation of QM. There is a respectable theory that the waves are actually electromagnetic in nature. It's incomplete and far from mainstream, though.

Ok - Mike, bringing your post on GoC's thread back here, as I wouldn't want to clog up his thread with my posts, I have more respect...

Firstly, I'm sorry but I do not have access to a computer and am just conducting myself from a phone's operating system, whereby I can't view pdf's, so for the moment I haven't seen the info you posted in pdf format.

Secondly, I am still observing that you are having difficulty stepping outside of the view of conventional GR and SR.
Clearly the conventional view describes a universe that is presently expanding...
My model seeks to describe a universe that is presently contracting, a contraction that has been slowly accelerating from the point that my model's rendition of the universe's inflation period ceased.

This involves a completely different rendition of GR and SR principles, therefore what I am attempting to describe 'will be' different.
However, the differences that my model makes are an alternative means of describing experimentally confirmed observations, without the necessity for an inclusion of Dark matter, or Dark Energy.

I now know more or less exactly how my model can be calculated, bar not having the required value of the information that I was requesting from you...
And what I am seeking is someone well versed in manipulating mathematics, who can can let go of their conventional relativity/quantum preconceptions in order to take specific instructions as how GR and SR and quantum principles can be very subtly changed in order to describe how my model is put together.
Nothing philosophical about it what-so-ever...

With regards to what you surmise about my model, the closest you come is in regarding probability space.  My model includes a time dilation factor to the quantum region of physics that adds probability space before the fact rather than after, where my model states that the concept of probability space is born of the fact of current physics not taking into account that where frequency is a timing function of energy, +energy = shorter seconds.

My model is based on a theory of time which renders time itself as a reactive phenomenon caused within and as part of the mechanics of the universe, where the rate of time anywhere is a reaction to energy, and different rates of timing are occurring simultaneously as a reaction to their energy state.
As opposed to the current view where time is 'only' a measurement of sequential events, as observed by an observer.

Because your relativistic correction to the SC metric includes holding the speed of light relative to GR time dilation, I had thought that it wouldn't be too much trouble to transpose the amount by which the speed of light would be accelerated by these increases in the rate of time into a m/s^2 value... (between EarthR and r=2radii)
This is all I'm asking...
Title: Re: Is there a discrepancy with the equivalence principle?
Post by: timey on 28/03/2017 00:37:38
OK, I stand corrected. You are proposing a new twist on the aether theory. I assume you are aware that aether was the prevailing view before Einstein. You should be intimidated by the fact that he was able to sway so many of the greatest minds of the modern era, but even if you are not, you should study his arguments. Chances are he has already debunked your theory. For starters, you need to account for the Michelson-Morley result.

My model does account for the Michelson-Morley result in that it makes an additional axiom that the speed of light cannot exceed the local rate of time.  As I have said before.

https://en.m.wikipedia.org/wiki/Hubble's_law

Quote
After Hubble's discovery was published, Albert Einstein abandoned his work on the cosmological constant, which he had designed to modify his equations of general relativity to allow them to produce a static solution, which he thought was the correct state of the universe. The Einstein equations in their simplest form model generally either an expanding or contracting universe, so Einstein's cosmological constant was artificially created to counter the expansion or contraction to get a perfect static and flat universe.[41] After Hubble's discovery that the Universe was, in fact, expanding, Einstein called his faulty assumption that the Universe is static his "biggest mistake".[41] On its own, general relativity could predict the expansion of the Universe, which (through observations such as the bending of light by large masses, or the precession of the orbit of Mercury) could be experimentally observed and compared to his theoretical calculations using particular solutions of the equations he had originally formulated.

Here we can see that the Einstein equations will describe an expanding universe or a contracting universe.

If Hubble's red shift velocities are re-interpreted as being due to slower time in space, then the aether type scenario is that motion is affected by slower time in space.

It's a logical proposition for a contracting universe of the cyclic type that my model describes. 
Title: Re: Is there a discrepancy with the equivalence principle?
Post by: Mike Gale on 28/03/2017 01:46:44
My bias towards mainstream physics is irrelevant. I'm not trying to debunk your theory. (At least not yet.) I'm only trying to identify how it differs from mainstream. Because you are unable to provide a formal derivation, our only recourse is to juxtapose predictions. You obviously have the constant velocity scenario worked out in your mind, but GR can't solve that one without generating controversy. The approach I'm advocating is to define a scenario which is unambiguous in GR and then ask what your theory has to say about it.
Title: Re: Is there a discrepancy with the equivalence principle?
Post by: Mike Gale on 28/03/2017 02:00:34
I noted earlier that the expanding universe hypothesis comes from the Friedmann solution, not the Schwarzschild solution. Although both are consistent with GR principles, they are distinctly different. Friedmann dilation is exclusively temporal so it is the same for all points in space. Schwarzschild dilation may or may not be temporal, but it most definitely has a spatial dependency.
Title: Re: Is there a discrepancy with the equivalence principle?
Post by: Mike Gale on 28/03/2017 02:12:12
I cannot lay claim to the variable speed of light (VSL) hypothesis. Einstein floated that idea for several years before he conceded the dilation view. Although VSL has fallen out of favour, neither view is free of controversy.
Title: Re: Is there a discrepancy with the equivalence principle?
Post by: timey on 28/03/2017 03:02:04
Well we can start with Dark Energy:
Dark Energy isn't necessary in my model because the universe is not expanding.  Although distances between masses, i.e. galaxy clusters are becoming greater, this is because the masses themselves are slowly becoming closer together.

Now for Dark Matter:
Dark matter isn't necessary in my model because gravitational acceleration is 3rd time dilation related.
This completely changes the outlook on gravitational attraction.
With just 1 or 2 bodies m orbiting a central body M at a particular radius, at that radius their movement will be slowed down by the 3rd time dilation.  If one calculated the orbital velocity via the 3rd time dilation, comparatively one would say that the m's velocity was increased at radius from main body M, not decreased.
When there are multiple bodies of m at each radius from main body M, as in the galaxy spiral, the 3rd time dilation will not be increasing the length of a second at same rate due to a greater mass distribution.
Again - if we calculated velocity speeds via the 3rd time dilation at that radius, we would find that the orbital speeds at each radius from main body M would be proportional to the 3rd time dilation of open space caused by mass distribution at that radius.

If you are all good with above, next we should address my model's rendition of Big Bang theory, and how my model gives physical mechanics for an inflation period that is analogous to Alan Guth's rendition but gives actual physical cause.

In reply to your further posts:
If Freidmann considered a temporal dilation that was equal everywhere then this differs from my model in that the temporal dilation of my model is dilated in space and contracted near mass, and it is the physical cause of gravitational acceleration.  i.e. already included in the Einstein equation.
Title: Re: Is there a discrepancy with the equivalence principle?
Post by: Mike Gale on 28/03/2017 03:10:26
...the rate of time anywhere is a reaction to energy, and different rates of timing are occurring simultaneously as a reaction to their energy state.
OK, you are proposing that clock speed depends on clock mass as well as gravity and velocity. I presume Einstein's light clock defines the speed limit since light is massless. SR is therefore sacrosanct. That leaves GR, which is consistent with observation. You have to discard some part of it in order to accommodate this new effect without changing the result. The discarded part would have to depend on clock mass in the opposite way. It's a zero sum game. If I say A=B+C and you say A+D=B+C+D, we have said the same thing.
Title: Re: Is there a discrepancy with the equivalence principle?
Post by: timey on 28/03/2017 03:21:33
Erm no - I'm saying that mass energy is causing time dilation, and that temperature energy, or potential energy increases will increase that rate of time.  All masses will run at differing rates of time, but that when subject to external energy, i.e. temperature energy or potential energy, that all different mass values will be affected equally, thereby upholding the equivalence principle that all is equal in every frame.

So a ceasium atom, or an ion, or an FE57 will all be equal in relation to each other at each gravity potential.

The 3rd time dilation will not register on any clock, and is inherent to the g-field to the value of M, surrounding M, and this time dilation affects how m moves in the g-field surrounding M, and in outer space.

This effect that affects movement in the g-field is not a new effect.  This 3rd time dilation is just giving a physical cause to an effect that is being accounted for already in the Einstien equation, this being gravitational acceleration.
Title: Re: Is there a discrepancy with the equivalence principle?
Post by: timey on 28/03/2017 03:36:06
I should add that velocity will also affect the time dilation for mass, and that any value of mass will be affected equally by a velocity, as per how temperature energy and potential energy will affect all mass equally, where this velocity related time dilation is connected to kinetic energy.
Title: Re: Is there a discrepancy with the equivalence principle?
Post by: Mike Gale on 28/03/2017 03:39:01
A temporal dilation that eludes every conceivable clock is no different than the celestial teapot. It may well be that the timing mechanism, which is responsible for atomic decay, is subject to a mass-dependent temporal dilation. That doesn't change the observable, which is that atoms of different masses decay at different rates. I am reminded of Feynman's account of a Q&A session with some famous physicist of yore. I can't remember who it was, but he was fielding questions on Kepler's laws of planetary motion. The prevailing view of the time was that angels pushed the planets around in the orbits of Copernicus and someone asked what happened to them. In deference to Newton's spooky action at a distance, he pointed out that Kepler's theory was consistent with the angel hypothesis except that they pulled instead of pushing.
Title: Re: Is there a discrepancy with the equivalence principle?
Post by: timey on 28/03/2017 03:47:33
It is not a celestial teapot if one can observe the physical results, these being the motions/velocities in space.

And it would be impossible to measure an 'open space' with a clock because a clock is mass.
If one were looking to measure the scenario with light, just look to the frequency of light reducing with the longer seconds of space at increasing h from M, and the increasing frequency of light in the shorter seconds at decreasing h from M.

A clock at rest with respect to the g-field, it's frequency increases in the higher gravity potential.
Lights frequency decreases in the higher gravity potential.
Light has no mass and will not gain potential energy at h from M.
Title: Re: Is there a discrepancy with the equivalence principle?
Post by: Mike Gale on 28/03/2017 03:57:30
But you're proposing an untestable mechanism for an observable, which is already fully accounted for. Remember Laplace's retort to Napoleon when asked why he neglected to credit God in his book on cosmology? (https://en.wikipedia.org/wiki/Pierre-Simon_Laplace#I_had_no_need_of_that_hypothesis)
Title: Re: Is there a discrepancy with the equivalence principle?
Post by: timey on 28/03/2017 04:12:33
Gravity is very far from being fully accounted for.  Everyone knows 'what' it does, but not 'how' it does it.

Giving the acceleration of gravity a physical cause that then negates the need for Dark Energy and Dark Matter is a double bonus surely?
Title: Re: Is there a discrepancy with the equivalence principle?
Post by: Mike Gale on 28/03/2017 04:16:02
Einstein's cause is spacetime dilation. You can propose a mechanism by which mass causes dilation, but it turns into an infinite regression like the turtles hypothesis. (https://en.wikipedia.org/wiki/Turtles_all_the_way_down)
Title: Re: Is there a discrepancy with the equivalence principle?
Post by: timey on 28/03/2017 04:26:21
No - the g-field causes the temporal dilation of space and the g-field is caused by M, where the g-field of M affects the motions of m...

The moon is an m in relation to Earth's M, and the planets are m's to the Sun's M.
The Sun is an m in relation to distribution of mass on the radial to central M of Galaxy.
All M's of Galaxy clusters are slowly being pulled together.
Title: Re: Is there a discrepancy with the equivalence principle?
Post by: Mike Gale on 28/03/2017 04:33:03
That's what Einstein is saying. The difference is that he makes no hypothesis about how mass energy incurs dilation. He just observes that it does and that it doesn't depend depend on 'm'.
Title: Re: Is there a discrepancy with the equivalence principle?
Post by: timey on 28/03/2017 04:38:30
That's right, and gravitational acceleration doesn't depend on m either.
Title: Re: Is there a discrepancy with the equivalence principle?
Post by: Mike Gale on 28/03/2017 04:40:23
Then what do you mean by "m in relation to M"?
Title: Re: Is there a discrepancy with the equivalence principle?
Post by: timey on 28/03/2017 04:41:28
Because M is denoting the value of gravity potential at h from M, and the value of the g-field at h from M.
Title: Re: Is there a discrepancy with the equivalence principle?
Post by: Mike Gale on 28/03/2017 04:42:32
Sure, but what's 'm' got to do with it?
Title: Re: Is there a discrepancy with the equivalence principle?
Post by: timey on 28/03/2017 04:44:38
pe=mgh, where pe/m then makes pe equally distributed to m.
Now you have an energy to frequency of electron transitions relationship for m at h from M.
Title: Re: Is there a discrepancy with the equivalence principle?
Post by: Mike Gale on 28/03/2017 04:50:58
Gravitational potential and gravitational field are the same thing. They are measures of potential energy per unit mass. The value of 'gh' from GR applies to all masses.
Title: Re: Is there a discrepancy with the equivalence principle?
Post by: timey on 28/03/2017 05:02:15
That's right - the difference being in my model that anything that is massless is not affected by gravity potential energy.  Therefore space itself and light are only affected by the 3rd time dilation, and do not experience GR time dilation.

The third time dilation is what causes m to decelerate m<<<M, and accelerate m>>>M.
The 3rd time dilation is what causes energy to become stretched out over a longer wave length L<<<M, and causes energy to become compressed into shorter wavelength L>>>M.
(L being for light)
Title: Re: Is there a discrepancy with the equivalence principle?
Post by: Mike Gale on 28/03/2017 05:04:11
But light is demonstrably affected by gravity.
Title: Re: Is there a discrepancy with the equivalence principle?
Post by: timey on 28/03/2017 05:09:51
The 3rd time dilation is caused by gravity.
Anywhere where light is demonstrably affected by gravity can be described in terms of a 3rd time dilation related acceleration/deceleration.
Title: Re: Is there a discrepancy with the equivalence principle?
Post by: Mike Gale on 28/03/2017 05:14:09
There is no need for that hypothesis. The effects of gravity on light are fully accounted for in GR.
Title: Re: Is there a discrepancy with the equivalence principle?
Post by: timey on 28/03/2017 05:20:09
...and GR in the form of Einstein's equation can precisely describe a contracting model, but only if red shifts aren't velocity related.
Are you sure that GR describes light's passage across space independently of SR?
Title: Re: Is there a discrepancy with the equivalence principle?
Post by: Mike Gale on 28/03/2017 05:35:00
Absolutely. And as I said previously, the SC solution has nothing to say about Hubble's data. That's the Friedmann solution.
Title: Re: Is there a discrepancy with the equivalence principle?
Post by: timey on 28/03/2017 12:50:41
So if the Friedmann equations are describing the Hubble data in relation to the Einstein equation, then what is Freidmann using as a means to the description?
You mentioned something about him using time to constantly dilate space...
Can you give me a more detailed description?
For instance when you say 'to constantly dilate space', do you mean a constantly time dilated space, or a space that is dilated by constant time?
Title: Re: Is there a discrepancy with the equivalence principle?
Post by: alancalverd on 28/03/2017 16:50:13

A clock at rest with respect to the g-field, it's frequency increases in the higher gravity potential.
Lights frequency decreases in the higher gravity potential.
Light has no mass and will not gain potential energy at h from M.

Why do you keep repeating (and now embellishing) this nonsense? We're talking physics, not Mozart! Stic k tot he observed facts: The frequency of any repetitive process appears higher when observed from a lower gravity potential. The frequency of any photon is higher, when received at a lower gravity potential, than observed at the source.  The two phenomena are identical.
Title: Re: Is there a discrepancy with the equivalence principle?
Post by: alancalverd on 28/03/2017 16:51:51

A clock at rest with respect to the g-field, it's frequency increases in the higher gravity potential.
Lights frequency decreases in the higher gravity potential.
Light has no mass and will not gain potential energy at h from M.

Why do you keep repeating (and now embellishing) this nonsense? We're talking physics, not Mozart! Stic k tot he observed facts: The frequency of any repetitive process appears higher when observed from a lower gravity potential. The frequency of any photon is higher, when received at a lower gravity potential, than observed at the source.  The two phenomena are identical. An observer at the same gravitational potential sees no frequency shift. 
Title: Re: Is there a discrepancy with the equivalence principle?
Post by: timey on 28/03/2017 18:09:40
There is nothing that I have said that renders the observations that you speak of as anything other than the observations you speak of.

Clearly Alan, I am looking to a differing mathematical description as to calculating these observations.  A differing means that results in a description of our universe that does not require Dark Energy, or Dark Matter, that will unify the standard model with gravity, and provide mechanics for Big Bang and Inflation.

If you think that is nonsense, then don't participate.
There are many physicists who write about the fact of the standard model and gravity not being unified and perhaps someone else who has read these same books will be more interested in my model than you are.
Title: Re: Is there a discrepancy with the equivalence principle?
Post by: Mike Gale on 29/03/2017 00:39:50
So if the Friedmann equations are describing the Hubble data in relation to the Einstein equation, then what is Freidmann using as a means to the description?
You mentioned something about him using time to constantly dilate space...
Can you give me a more detailed description?
For instance when you say 'to constantly dilate space', do you mean a constantly time dilated space, or a space that is dilated by constant time?
I think I said exclusively, not constantly. I'm no expert on the subject, but it is presumably energy from the Big Bang that drives Friedmann dilation. The reason why it is exclusively temporal is entirely empirical. Hubble expansion appears to be the same in every direction of space no matter where you're standing or how you're moving. It's really just a way to guesstimate Einstein's cosmological constant.
Title: Re: Is there a discrepancy with the equivalence principle?
Post by: timey on 29/03/2017 01:04:47
I thought that red shift velocities are proportional to distance of source.
This meaning that more distant light sources are receding away from us faster.

I've just started a discussion on this subject here:

https://www.thenakedscientists.com/forum/index.php?topic=70013.0
Title: Re: Is there a discrepancy with the equivalence principle?
Post by: timey on 29/03/2017 01:22:30
In addition to post above...

https://en.m.wikipedia.org/wiki/Hubble's_law
Quote
In 1922, Alexander Friedmann derived his Friedmann equations from Einstein's field equations, showing that the Universe might expand at a rate calculable by the equations.[35] The parameter used by Friedmann is known today as the scale factor which can be considered as a scale invariant form of the proportionality constant of Hubble's law.

...whatever that means (chuckle)

In any case Friedmann deduced this from the Einstein equation which can just as adequately describe a contracting universe, where to re-interpret Hubble's velocity related red shifts one would have to look to the parameter of the 'scale factor'.

What is the scale factor?

https://en.m.wikipedia.org/wiki/Scale_factor_(cosmology)

In the link where they are talking about Hubble's parameter, can you tell me anything about this?
Quote
where the dot represents a time derivative.
...and the fact that I am seeing dots over a and d?
Title: Re: Is there a discrepancy with the equivalence principle?
Post by: Mike Gale on 29/03/2017 01:45:03
It's misleading to say that Friedman's solution emerged from GR. It is certainly consistent with GR, but that is by design. It's really just a curve fit to Hubble's data. Variables like the Hubble scaling factor are entirely contrived. They are nothing more than convenient knobs that you can twiddle to get the equation to fit the data. They hardly qualify as constants of nature.
One of the inputs to the analysis is the desired geometry, which can be flat, oblate or obtuse so the question of expanding, contracting or static is pre-determined. The expanding option is the most plausible because it can make the cosmological constant go away (if you twiddle the other knobs in an appropriate manner.) I'm not sure, but I think accelerated expansion requires some more knobs.
The dot denotes rate of change over time: x-dot=dx/dt for example.
Title: Re: Is there a discrepancy with the equivalence principle?
Post by: timey on 29/03/2017 02:10:17
Well I was just quoting the wiki link.

The Einstien equation can describe a contracting universe where the constant also 'goes away'.
If the Hubble data curve can be transposed into a time dilated curve relative to distance that is not expanding over time, rather than being a velocity curve relative to distance that is expanding over time...this constitutes another method of knob twiddling resulting in a description of a contracting universe.
Title: Re: Is there a discrepancy with the equivalence principle?
Post by: Mike Gale on 29/03/2017 04:01:56
The Einstein field equations are part and parcel to GR so they are included in the Schwarzschild and Friedmann solutions. Without those equations, GR is just cartography in 4 dimensions. (I say "just", but the mathematics of that are incredibly complex. We stand on the shoulders of giants.)
I think you meant to say that Schwarzschild (not Einstein) can describe a contracting universe, but that's not quite right either. Schwarzschild (or rather Newton, on which it is based) predicts clumping of mass because everything is attracted to everything else. The cosmological constant can counteract gravity with expanding space. I've never seen the mathematical proof, but it presumably does so at large scales so that, although local groups of masses tend to coalesce, highly dispersed ones do not.
It might be possible to tweak Friedmann's knobs to produce a constant rate of expansion, but that wouldn't fit Hubble's data.
Title: Re: Is there a discrepancy with the equivalence principle?
Post by: Mike Gale on 29/03/2017 04:26:50
It is worth noting that Hubble expansion is far from a done deal. Reckoning distances at cosmological scales is an inexact science. There are also plenty of anomalies (including blue-shifted galaxies like Andromeda.) It is entirely possible that red-shifted galaxies are simply moving away from us at different, but constant velocities. One smart aleck (I can't remember who) noted that we should not be surprised if the horse that ran the fastest is farthest from the gate half way through the race. Accelerated expansion is not so easily dismissed though.
Title: Re: Is there a discrepancy with the equivalence principle?
Post by: timey on 29/03/2017 04:52:56
No - I meant what I posted.

https://en.m.wikipedia.org/wiki/Hubble%27s_law
Quote
The Einstein equations in their simplest form model generally either an expanding or contracting universe, so Einstein's cosmological constant was artificially created to counter the expansion or contraction to get a perfect static and flat universe.

The Einstein equations in their simplest form can model an expanding or contracting universe.  The cosmological constant in either case can be added to counter the expansion, or the contraction to get a steady state.

Therefore if Hubble's velocity related red shifts were re-interpreted as being due to a curve of slower time in space, then Galaxy clusters would be (more or less) maintaining their orbital positions in a space that is slowly contracting at an accelerating rate.
Red shifts and blueshifts as observed would be due to g-fields in space becoming weaker (red shift) because distance between light source and observation point are slightly increasing as mass clusters pull closer together, and the g-field is becoming stronger (blueshift) because distance between light source and observation point is closing as mass clusters pull closer together.
(same explanation but without velocity related expansion)

To understand this outlook, one has to consider the initial conditions.
The initial conditions for this contraction in my model are afforded immediately after the period of my model's rendition of an Inflation period which leaves us with a sea of individual particles.
If you can imagine how much space a truckload of tree trunks would take up if we wood chipped it...
Imagine all the clumped mass of the universe in particle form...
These are the initial conditions for contraction in my model where open distances in space are expanding in length because particles are vacating their former positions as mass clumps.
Title: Re: Is there a discrepancy with the equivalence principle?
Post by: Mike Gale on 29/03/2017 04:59:48
Einstein's field equations are meaningless in isolation of the GR framework. They are simply conservation of energy and momentum constraints expressed in a form that is compatible with 4D cartography.
Title: Re: Is there a discrepancy with the equivalence principle?
Post by: timey on 29/03/2017 05:02:40
I am referring to the equation that I posted earlier this thread which is Einstein's equation that describes GR.
Title: Re: Is there a discrepancy with the equivalence principle?
Post by: Mike Gale on 29/03/2017 05:05:56
I must have missed that, but I expect you were quoting the Schwarzschild solution because Einstein doesn't have one of his own.
Title: Re: Is there a discrepancy with the equivalence principle?
Post by: timey on 29/03/2017 05:06:38
Quote from: Mike Gale link=topic=69800.msg509381#msg50938t1 date=1489377894
If Lambda = 0 then Guv = (8*pi*G/c4) * Tuv.

This equation
Title: Re: Is there a discrepancy with the equivalence principle?
Post by: Mike Gale on 29/03/2017 05:07:22
That's Schwarzschild.
Title: Re: Is there a discrepancy with the equivalence principle?
Post by: timey on 29/03/2017 05:09:50
Really?  Well I never.  The program I lifted that equation from said it was the Einstien equation.
I didn't know the SC was Einstien's equation.
Title: Re: Is there a discrepancy with the equivalence principle?
Post by: Mike Gale on 29/03/2017 05:12:24
I suppose it's fair to credit Einstein because Schwarzschild's solution is based on his field equations, but it's ambiguous because there are several other solutions, all of which are based on Einstein's constraints.
Title: Re: Is there a discrepancy with the equivalence principle?
Post by: timey on 29/03/2017 05:13:59
In any case, that equation can describe a contracting universe.
Title: Re: Is there a discrepancy with the equivalence principle?
Post by: Mike Gale on 29/03/2017 05:14:35
Agreed, but that equation is only one part of a larger set.
Title: Re: Is there a discrepancy with the equivalence principle?
Post by: timey on 29/03/2017 05:20:37
Only when you start adding the Hubble parameter surely?
The program I watched said that Einstien was upset to have had to have added the constant lambda because it ruined the simplistic beauty of his equation.
Title: Re: Is there a discrepancy with the equivalence principle?
Post by: Mike Gale on 29/03/2017 05:22:19
No wait. That equation is actually part of Einstein's field equations, but it doesn't tell you anything about expansion until you apply it to a scenario like the Schwarzschild solution.
Title: Re: Is there a discrepancy with the equivalence principle?
Post by: Mike Gale on 29/03/2017 05:26:05
It's true that Einstein was disgusted with his cosmological constant (because it's ad hoc) and was happy to see the end of it when Hubble's data emerged on the scene. But observations of accelerated expansion suggest that he may have been too hasty.
Title: Re: Is there a discrepancy with the equivalence principle?
Post by: timey on 29/03/2017 05:28:32
As I understood it, that equation is describing GR in full, and it does describe expansion, or contraction without any additions.  This being why Einstien included the constant, to stop his equation from doing either in order to maintain a steady state.

There are no observations of expansion.  There are only observations that are interpreted as expansion.
Title: Re: Is there a discrepancy with the equivalence principle?
Post by: timey on 29/03/2017 05:36:43
In addition to post above...

The GR field equations, if I remember correctly from the Susskind lectures, include the use of the Lorentz transformations.
Title: Re: Is there a discrepancy with the equivalence principle?
Post by: Mike Gale on 29/03/2017 05:39:40
The Einstein tensor (denoted Guv) is a fundamental part of GR, but there are lots of other moving parts. I have to admit that I don't really understand how the Schwarzschild solution leads to an expanding universe, but that is indeed what the literature says. I suppose you have to envision an accelerated object as a stationary one with space flowing in the opposite direction. It hurts my brain.
Title: Re: Is there a discrepancy with the equivalence principle?
Post by: timey on 29/03/2017 05:44:41
So that is the first expression of the equation, and it is equal to the latter part.  The latter part also being moving parts?
Title: Re: Is there a discrepancy with the equivalence principle?
Post by: timey on 29/03/2017 05:58:15
...and aren't the GR field equations different from the Einstien equation?
Where Einstein's equation describes GR, but the GR field equations describe moving stuff around under the remit of GR?
Title: Re: Is there a discrepancy with the equivalence principle?
Post by: Mike Gale on 29/03/2017 06:10:40
The GR field equations, if I remember correctly from the Susskind lectures, include the use of the Lorentz transformations.
Quite right. Lorentz invariance is one of the constraints.
Title: Re: Is there a discrepancy with the equivalence principle?
Post by: Mike Gale on 29/03/2017 06:15:43
So that is the first expression of the equation, and it is equal to the latter part.  The latter part also being moving parts?
The other "moving parts" come from the cartography part of GR.The Einstein tensor is part of an equality that relates energy and momentum to spacetime curvature. The idea is that accelerated motion in flat space is equivalent to uniform motion in curved space.
Title: Re: Is there a discrepancy with the equivalence principle?
Post by: timey on 29/03/2017 15:17:10
Quote
The Einstein tensor is part of an equality that relates energy and momentum to spacetime curvature. The idea is that accelerated motion in flat space is equivalent to uniform motion in curved space.

Yes - that is why my model's change to the structure is physically possible.
My model states that space is flat, and accelerated motion is uniform motion that is accelerated by a 3rd time dilation that gives physical cause for the acceleration of gravity...
Where the energy of the g-field causes the 3rd time dilation, the differing lengths of seconds this energy of the g-field causes result in non uniform motion for any m in the g-field.

Gravity potential energy is increased for m at h from M, which will increase the frequency of the electron transitions of any m.
And where m=0, the decreasing g-field value at any h from M will decrease any m=0 energy levels, such as the energy of the g-field itself, and the energy levels of already emitted light.
Where we can say that pe=mgh, and that where m=0 we are talking about there being no additional potential energy at h from M...
Could a calculation such as something like e=gh describe the decreasing values of the g-field at any h from M?
And if so - can a calculation such as e=gh result in values that can be matched to the reduced energy levels for already emitted light<<<M at h from M?
Title: Re: Is there a discrepancy with the equivalence principle?
Post by: timey on 29/03/2017 16:52:08
http://homepages.rpi.edu/~newbeh/gersonaguiblava.pdf

Mike - here is a link plotting supernova light curves to the time dilation factor.  As they are talking about time dilation slowing the passage of light across space, this must be an SR application.

As per my model, it would be the  3rd time dilation of open space that would be the cause of the light's curve of slower motion across space.

As an aside to add to the basis of my model's 3rd time dilation consideration - in current physics if we apply SR to mass travelling at a percentage of the speed of light, we cannot state the time dilation effects of SR as slowing the velocity of that mass, because if we did then the lesser velocity wouldn't be the same percentage of the speed of light and the time dilation factor would consequently be altered.
It is not possible to calculate a time dilation effect caused by a velocity if the time dilation effect calculated changes the velocity.  This is a catch 22.
Title: Re: Is there a discrepancy with the equivalence principle?
Post by: Mike Gale on 30/03/2017 05:03:07
"Light curve" is insider jargon for temporal variation in intensity. You measure brightness over the course of several days. The idea is that we know how the brightness of a type Ia supernova changes over time so, if you're confident that you're looking at that type of object, the time line can be used as a clock and you can use red shift measurements to figure out how fast that clock should run relative to your own. SR is only involved insofar as it is incorporated into GR. The "tired light" theory, which they purport to have debunked, is an alternative to GR.
Title: Re: Is there a discrepancy with the equivalence principle?
Post by: Mike Gale on 30/03/2017 05:27:11
Einstein taught us that the only unambiguous way to measure distances in space is to measure the flight time of a light ray. If time slows down in flat space and the speed of light is invariant then space must be compressed equally in all directions. That scenario is incompatible with a spherically symmetric gravitational field and cannot be reconciled with the constant velocity scenario of SR.
Title: Re: Is there a discrepancy with the equivalence principle?
Post by: jeffreyH on 30/03/2017 12:39:05
You do not need the Einstein field equations to describe spacetime.
Title: Re: Is there a discrepancy with the equivalence principle?
Post by: timey on 30/03/2017 13:43:53
They don't sell red diesel at Birchanger Green....
Title: Re: Is there a discrepancy with the equivalence principle?
Post by: timey on 30/03/2017 13:44:50
Quote
If time slows down in flat space and the speed of light is invariant then space must be compressed equally in all directions.

If time slows down in flat space and the speed of light is invariant, by holding the speed of light relative to the longer seconds space will be temporally dilated.
Title: Re: Is there a discrepancy with the equivalence principle?
Post by: guest4091 on 30/03/2017 17:18:57
Timey;
In SR a force is applied to a clock, resulting in motion, resulting in time dilation.
In GR a force is applied to a clock, resulting in motion (free fall), resulting in time dilation. In SR we can identify the source of energy applied to the clock. In GR, it's the g-field, but we have no theory to explain how the energy is transferred from the dominant mass M to the field.
In both cases, time dilation results from motion regardless of the energy source.
Why do you propose a theory that reverses cause and effect?
(If I interpreted your idea correctly)
Title: Re: Is there a discrepancy with the equivalence principle?
Post by: timey on 30/03/2017 22:00:55
Quote
In SR a force is applied to a clock, resulting in motion, resulting in time dilation.
... Agreed.
Quote
In GR a force is applied to a clock, resulting in motion (free fall), resulting in time dilation.
... Agreed, it results in SR motion related time dilation.
Quote
In SR we can identify the source of energy applied to the clock. In GR, it's the g-field,
Ok well - In my model the action of the force of the g-field causing motion is 3rd time dilation related.  The 3rd time dilation comprises of seconds that get longer in the weaker g-field.
Quote
but we have no theory to explain how the energy is transferred from the dominant mass M to the field.
In order to describe how the 3rd time dilation is caused we must look to GR time dilation.
With the addition of the 3rd time dilation for the g-field, this renders GR time dilation as a phenomenon that is experienced by m in relation to M, where the frequency of m's electron transitions are then linked to potential energy on the basis that a higher frequency usually is accompanied by a higher level of energy.  pe=mgh where pe/m ensures that all value of m is affected equally.
Where m=0 we are now looking to the value of the g-field for energy, where a weaker g-field has less energy.
In this manner we can say that free energy, (i.e. energy not bound to mass), such as the value of the g-field at any h from M, is the cause of the 3rd time dilation, and that this 3rd time dilation is the cause of the observed frequency and energy changes occurring for already emitted light (also energy not bound to mass) in the g-field.

Now SR time dilation must be accounted for... (not so simple)
A clock in relative motion has a lower frequency.  I am now looking for a reason for motion to reduce energy to account for a lower frequency of electron transitions.
In the case of free fall we could say that potential energy is being converted into kinetic energy where kinetic energy does not affect the frequency of the constituents of body m, but this isn't sufficient.  In a constant potential, additional energy input would be required to initiate movement, and it would be the additional energy that would be being converted into kinetic energy.  A fuel to energy conversion would have bog all to do with an onboard clock's frequency.
Ok - so we know that at any location in the universe a body will have its own rest mass energy + the potential energy associated with its position.  The potential energy that a body will have is reliant on its position with respect to the bigger body Mass.
If we consider that a smaller body at rest with respect to the bigger body has a a value of potential energy, then we could say that any motion with respect to the bigger body can reduce this value of potential energy and that this will reduce the energy and frequency of the workings of the clock.
This remit will also work in the free fall case, (where the acceleration is 3rd time dilation related), and also in the case of deceleration experienced by m<<<M. (where the deceleration experienced is 3rd time dilation related)
... I get frustrated here (as a non-mathematician) because I suspect that, not relativistic mass itself, but the value of relativistic mass and the way it is calculated is pertinent to this SR time dilation remit I suggest, where I understand that relativistic mass is negligible at non-relativistic speeds, but note that SR time dilation is also negligible at non-relativistic speeds.

The goal of the mission is to describe the phenomenon of time itself as a reactive within the universe rather than a measurement of sequential events, and when this remit is then applied to quantum, by understanding that frequency is time dilation related, (GR time dilated point of fact), when re-calculating the blackbody under the remit of +energy=shorter seconds, the quantum nature of the energy additions should be negated as a continuum.
Title: Re: Is there a discrepancy with the equivalence principle?
Post by: Mike Gale on 31/03/2017 00:12:09
You do not need the Einstein field equations to describe spacetime.
That is true for the case of uniform motion (i.e. SR.) Otherwise EFE (i.e. GR) is your best bet.
Title: Re: Is there a discrepancy with the equivalence principle?
Post by: Mike Gale on 31/03/2017 00:15:05
They don't sell red diesel at Birchanger Green....
The relevance of that reference is lost on me.
Title: Re: Is there a discrepancy with the equivalence principle?
Post by: Mike Gale on 31/03/2017 00:20:00
If time slows down in flat space and the speed of light is invariant, by holding the speed of light relative to the longer seconds space will be temporally dilated.
We are saying the same thing. There is no distinction between temporally dilated space and (unqualified) dilated space: d=ct, where 'c' is light speed and 't' is the flight time of a light ray as perceived by the observer.
Title: Re: Is there a discrepancy with the equivalence principle?
Post by: Mike Gale on 31/03/2017 00:23:30
In SR a force is applied to a clock, resulting in motion, resulting in time dilation.
Incorrect. There are no forces in SR. SR dilation is entirely due to velocity. Acceleration due to a force is just the means by which a velocity is achieved. More acceleration gets you there faster than less acceleration, but the net effect is the same.
Title: Re: Is there a discrepancy with the equivalence principle?
Post by: Mike Gale on 31/03/2017 00:30:55
In GR a force is applied to a clock, resulting in motion (free fall), resulting in time dilation.
GR does not necessarily involve motion. It works when the observers are held in place by a planetary crust for example. It fails if the observers are experiencing different forces from external sources (i.e. rocket engines), but it is possible to (approximately) compensate with SR in certain variations of that case (e.g. orbital free fall.)
Title: Re: Is there a discrepancy with the equivalence principle?
Post by: Mike Gale on 31/03/2017 00:41:09
In SR we can identify the source of energy applied to the clock. In GR, it's the g-field, but we have no theory to explain how the energy is transferred from the dominant mass M to the field.
GR explains how the field is influenced by mass (or light) energy. It just doesn't explain why because we don't know exactly what's going on inside the mass (or the light.) You can speculate all day about that, but it's pointless because we can't probe the universe at that scale. That's the string theory dilemma.
Title: Re: Is there a discrepancy with the equivalence principle?
Post by: Mike Gale on 31/03/2017 01:13:49
In both cases [GR and SR], time dilation results from motion regardless of the energy source.
Again, this is true of SR, but not GR.
Title: Re: Is there a discrepancy with the equivalence principle?
Post by: Mike Gale on 31/03/2017 01:17:39
Why do you [timey] propose a theory that reverses cause and effect?
(If I interpreted your idea correctly)
I don't think Timey addressed this question in her response so allow me to intervene. Her theory does not violate causality, at least no more so than GR already does (e.g. in the vicinity of the Schwarzschild event horizon.) She is only trying to link time dilation to observer mass.
Title: Re: Is there a discrepancy with the equivalence principle?
Post by: timey on 31/03/2017 02:07:21
No - I am not trying to link time dilation to observer mass.

What I'm trying to do is link energy to the phenomenon of time, where the universe has 3 separate time dilations.
1 caused by subtracting/adding pe from mass via motion/lack of motion = SR
1 caused by adding/subtracting pe to mass via position in g-field = GR
1 caused by adding/subtracting g-field energy for g-field of M where m=0 = 3rd time dilation...

Where the 3rd time dilation is the 'action of' acceleration/deceleration of gravity.

The intention here is to dispense with the need for the considerations of any observer and quite simply map out the g-field/s, and the motions of masses that occur in them just by knowing the value of M and operating under the remit of +energy=shorter seconds.
String theory, now you come to mention it, would indeed be infinitely more calculable under this remit.

As for the red diesel comment, well Birchanger Green is a motorway service station, and people don't drive tractors on the motorway.
... I made the comment because where Jeff said that the GR field equations don't describe space time
Quote
wiki
They do describe the fundamental interactionof gravitation as a result of spacetimebeing curved by mass and energy
Bit on the overly cryptic side perhaps...but it tickled me at the time. (chuckle).
It's just that Jeff didn't elaborate on his reason for posting that statement, and unless a statement is quantified as to context, one may as well be saying in as much as: 'They don't sell red diesel at Birchanger Green'...

Edit: It occurs Mike that perhaps because you are not from UK, it may be of some relevance to mention that red diesel is a lesser government taxed fuel for off road, and farm purposes only, that is identifiable from regular diesel in that the colour red has been added.
Title: Re: Is there a discrepancy with the equivalence principle?
Post by: timey on 31/03/2017 03:09:47
In GR a force is applied to a clock, resulting in motion (free fall), resulting in time dilation.
GR does not necessarily involve motion. It works when the observers are held in place by a planetary crust for example. It fails if the observers are experiencing different forces from external sources (i.e. rocket engines), but it is possible to (approximately) compensate with SR in certain variations of that case (e.g. orbital free fall.)

I think that Phyti was referring to the acceleration or deceleration that is described by GR that is causing motions that cause SR time dilation.
Title: Re: Is there a discrepancy with the equivalence principle?
Post by: Mike Gale on 31/03/2017 05:09:17
No - I am not trying to link time dilation to observer mass.

What I'm trying to do is link energy to the phenomenon of time, where the universe has 3 separate time dilations.
1 caused by subtracting/adding pe from mass via motion/lack of motion = SR
1 caused by adding/subtracting pe to mass via position in g-field = GR
1 caused by adding/subtracting g-field energy for g-field of M where m=0 = 3rd time dilation...
You contradict yourself because your "corrections" to GR and SR both involve observer mass. Also, I've made this point before, but your 3rd case is the GR case because GR is formulated in the context of an infinitesimally small observer mass.
BTW - we did away with red diesel in Canada because it's an overly complicated way to subsidize farmers.
Title: Re: Is there a discrepancy with the equivalence principle?
Post by: timey on 31/03/2017 13:46:47
Canada aye?  And there I was thinking you was in USA.

I suppose that your accusation of my contradicting myself depends very much on what you mean by observer mass.
In GR 'conventional', the time dilation at h from M, i.e. the increased frequency of electron transitions, is not thought to be occurring because of any change in external 'energy' additions experienced directly by the mass at h from M.
GR time dilation is thought to be occurring because time gets faster 'at' h from M, whereas it is not known why time gets faster at h from M when observed from the lower potential.

My model simply states mass at h from M as experiencing external energy additions at h from M due to pe=mgh where pe/m ensures that all m is affected equally, and it is these additions of pe that cause increase in the frequency of electron transitions for m at h from M.  My model gives this as the physical cause for GR time dilation that is observed of m at h from M from the lower potential.

Now which mass are you referring to as the observer mass please?
If you are referring to the mass at h from m, observed from the lower potential to have a higher frequency of electron transitions, as being the observer mass, then the observer mass at h from M will not observe any change in frequency of itself or any other body of mass at that h from M. All frequencies of electron transitions of any particle value m, although actually physically increased by their position at h from M will all retain their interactive relationships amongst themselves proportionally with an equal addition of potential energy for all.
And since we can just calculate an addition of pe at any h from M, we can remove ourselves from the necessity of being reliant on what an observer observes from any other potential.  It's no longer important because we know the cause of the observation.

Same with SR.

Quote
but your 3rd case is the GR case because GR is formulated in the context of an infinitesimally small observer mass.

An infinitesimally small mass is not m=0, but I do agree that the 3rd time dilation is the GR case, in that GR is the description of acceleration and deceleration of the motions of mass in the g-field.  It's just that GR has neglected to give its case an actual physical cause, and I have added an actual physical cause to GR's case.
Title: Re: Is there a discrepancy with the equivalence principle?
Post by: Mike Gale on 01/04/2017 00:19:32
My model simply states mass at h from M as experiencing external energy additions at h from M due to pe=mgh where pe/m ensures that all m is affected equally, and it is these additions of pe that cause increase in the frequency of electron transitions for m at h from M.
GR says exactly the same thing, although it applies to any clock (not just your "electron transitions", which I presume are atomic decay rates) and any field strength (not just gh, which is the weak field approximation.)
Title: Re: Is there a discrepancy with the equivalence principle?
Post by: Mike Gale on 01/04/2017 00:29:06
Now which mass are you referring to as the observer mass please?
If you are referring to the mass at h from m, observed from the lower potential to have a higher frequency of electron transitions, as being the observer mass, then the observer mass at h from M will not observe any change in frequency of itself or any other body of mass at that h from M. All frequencies of electron transitions of any particle value m, although actually physically increased by their position at h from M will all retain their interactive relationships amongst themselves proportionally with an equal addition of potential energy for all.
And since we can just calculate an addition of pe at any h from M, we can remove ourselves from the necessity of being reliant on what an observer observes from any other potential.  It's no longer important because we know the cause of the observation.
It is the mass at h from M. Observer mass is measured in the local reference frame. Observers at other locations in the field will perceive a different value. If A is observing B then observer mass is that of B as perceived by B. It is also known as "test mass" because it is the mass that you use to probe the field. It is taken to be arbitrarily small compared to M. Observer A is also presumed to be sufficiently removed from observer B that they don't have any significant influence on one another.
Title: Re: Is there a discrepancy with the equivalence principle?
Post by: timey on 01/04/2017 01:23:02
Thanks for clearing that up...

If A observes B, and B observes A, both will observe the other's clock to be of a differing electron transition frequency to their own.
We can just leave it at that now with the observers, because it is possible to predict the electron transition frequency in relation to the gravity potential of the location.

This is the case because at every gravity potential the electron transitions of a caesium atom, when observed from within the same gravity potential will always be observed to have the frequency of 9,192,631,770 Hz.
Calibrate the clock in one gravity potential and then raise it by a metre, from the lower potential it will be observed to have a higher frequency. Raise the clock by another metre and it will be observed from 2 metres below to have an even higher frequency than 1 metre clock.
On the basis that the gravitational shift equations for blueshift exactly match the time shift at h from M, where a light emitter at 2 metres emits a photon, and the observer 2 metres below observes the light as blue shifted, mathematically it is possible to map out these frequency of electron transition changes in the gravity potential based on the gravitational shift equations.

So I just don't see the necessity to involve observers because it doesn't matter what something looks like from where*, we only need to consider proportions and relationships.
(*if there is a rainbow in the sky and you are on one side of the valley, and I on the other, we can speak to each other on our mobile phone's about 'wow, do you see the rainbow over there', but your rainbow and my rainbow will be in differing positions in the sky.)

*

...and in answer to your post before the one above:
I am actually almost certain that while the GR time dilation effect is known to be due to position in the gravity potential, that additional potential energy is not thought to be the cause of the time dilation...
Edit: If it were then the standard model would be unified with gravity.

However, if you take the remit of additional pe causing GR time dilation back to the black body and consider that adding temperature energy is also increasing the frequency of electron transitions, which are a result of a higher quantum energy level and result in the emission of higher frequency photons, and then consider that energy added is shortening seconds... when re-calculating the temperature energy additions of the black body held relative to the shorter seconds, the quantum nature of these energy additions will be negated.
Title: Re: Is there a discrepancy with the equivalence principle?
Post by: Mike Gale on 01/04/2017 04:37:41
As a point of order, the atomic clock is based on nuclear decay, not electron state transitions. But as I said before, the observers are the clocks. The concept of time makes no sense in the absence of observers. If time slows down and nobody is there to measure it, there are no consequences so who cares? Imagine yourself as a point particle alone in the universe. How would you measure time? Indeed, how would you measure space? The answer is you can't. You need another point of reference and a means of communication. Communication requires oscillation so the simplest possible universe is one that contains two clocks and some light so they can see each other. The nature of the universe in the absence of those ingredients is moot.
Title: Re: Is there a discrepancy with the equivalence principle?
Post by: timey on 01/04/2017 12:55:55
Quote
Wiki:
A caesium standard or caesium atomic clock is a primary frequency standard in which electronic transitionsbetween the two hyperfine ground states of caesium-133 atoms are used to control the output frequency.

Point of fact, it 'is' the electron transitions that are used to control the output frequency.

Quote
The concept of time makes no sense in the absence of observers.
...This is only a valid perspective if one is viewing time as being simply a measurement imposed upon the universe.
I am taking the view that time is a phenomenon that is a reactive within the universe.  This view is supported by the remit of both GR and SR observed and experimentally verified time dilations, and is a view that is respectably noted in most physics books where the author is considering the concept of time.

Electron transitions are related to quantum energy levels, where electron transitions that are increasing in frequency are accompanied by a higher quantum energy...
Although there are theories about how particles become high energy in thunderstorms, sand storms, forest fires and volcanoes to result in lightening, quite how these particles become high energy is not fully understood.
But if you look to the caesium atomic clock, the electron transitions are observed to increase in frequency at h from M, so by remit of equivalency the electron transitions of any particle at h from M will also increase, and now you have physical cause for high energy particles at altitude.

Quote
If time slows down and nobody is there to measure it, there are no consequences
I just illustrate how time speeding up for particles at altitude has physical consequences...  Particles don't measure themselves, they interact with each other based on energy levels.
If particles are experiencing their time based on energy levels, then where there is no mass, i.e. the spaces between masses, the energy levels are much lower.
This is where the 3rd time dilation steps in, but by the nature of m=0, we don't have an observer of any kind to record or measure this time dilation.  All we can do is measure the consequences of it, which are that m in the g-field of M will be accelerated towards M by means of the shorter seconds as the g-field increases in strength near M, and decelerated away from M by means of the longer seconds as the g-field decreases in strength further away from M.

So again we see that wether anyone is measuring the situation or not, that the 3rd time dilation has consequences.
The 3rd time dilation has another much more important consequence though, because one must come to the conclusion that time does not run faster out in space, and a black holes time will be running at a far faster rate than anywhere else, which is absolutely fantastic news because GR under this remit can measure inside the black hole.
The mathematics won't break down.

The only portion of the picture that we are missing now is the directional force that tells m which direction to take in the g-field of M.
Here I now hark back to the increased frequency of electron transitions for the particle at h from M, and the magnetic moment of an electron.
Quote
:Wiki
From classical electrodynamics, a rotating electrically charged body creates a magnetic dipole with magnetic poles of equal magnitude but opposite polarity. This analogy holds as an electron indeed behaves like a tiny bar magnet. One consequence is that an external magnetic field exerts a torque on the electron magnetic moment depending on its orientation with respect to the field.
...where I think that an increased frequency of electron transitions will cause magnetic moments to occur more frequently, and this is what 'tells' m to be attracted directionally in the g-field.

Under this remit time is having physical consequences without observer of measurement, because time is part and parcel of 'why' and 'how' everything is moving in the universe.

However, this view only holds in a contracting universe, it won't work for the expanding model.

I am talking about an alternative view on space expansion here:

https://www.thenakedscientists.com/forum/index.php?topic=70013.new;topicseen#new

(only 16 posts so far)
Title: Re: Is there a discrepancy with the equivalence principle?
Post by: Mike Gale on 01/04/2017 14:58:47
I stand corrected on atomic clocks.
Title: Re: Is there a discrepancy with the equivalence principle?
Post by: Mike Gale on 01/04/2017 15:03:03
Fundamentally, a clock is nothing more than an oscillator. A caesium clock is no different than a pendulum clock in that regard. It's just more accurate and more reliable. Einstein's light clock is the simplest possible time keeper. You would be well advised to adopt that concept in your deliberations so you don't have to invoke QM concepts like electron state transitions, which are distractions.
As for the relationship between time and the observer, I recommend Sean Carroll's "From Eternity to Here". It's a thorough and entertaining treatment of the nature of time. He's got some pretty good YouTube videos if you can't be bothered with the book.
Title: Re: Is there a discrepancy with the equivalence principle?
Post by: timey on 01/04/2017 18:38:07
I've read it.

Quantum is exactly where I want to be because yes you are right, the clock is oscillating faster at h from M, where it's increased frequency can be held relative to the increased potential energy that it gains at h from M.

...And I shall repeat, again and again and again, that if one takes the view that an increase in energy is causing a higher frequency of electron transitions, and that a higher frequency of electron transitions signifies an increase in the rate of time - when one takes this remit back to Planck's black body experiment and considers that temperature energy is increasing the energy level of the blackbody resulting in a higher frequency of electron transitions, resulting in the emission of higher frequency photons, where one includes that an increase in the frequency of electron transitions signifies an increase in the rate of time - when measuring the temperature energy input joules/second held relative to the shorter seconds caused by the increases in energy, the quantum nature of the data will be negated.
Title: Re: Is there a discrepancy with the equivalence principle?
Post by: Mike Gale on 02/04/2017 05:10:13
You are correct to say that higher frequency equates to faster time (i.e. shorter seconds), but quantum state transitions and black body radiators are red herrings. You can make the same point with a classical oscillator like a pendulum clock. The only difference is the manner in which the oscillation is observed, be it the position of the pendulum or the frequency of emitted photons. In the case of a black body radiator, which emits photons at all frequencies, it is the most popular frequency (i.e. highest intensity) that is of interest for a time keeper.
Title: Re: Is there a discrepancy with the equivalence principle?
Post by: timey on 02/04/2017 12:23:35
Again - this is entirely dependent on how one is viewing the phenomenon of time.
You are saying 'time keeping'.
I am saying 'energy reactive'.

The blackbody emits higher frequency photons as an energy reactive.
+energy=higher frequency of emitted photon
A higher frequency of photon emission requires that the emitting body be possessed of a higher frequency of electron transitions, and a higher frequency of electron transitions requires additional energy.

Therefore that which we observe of the electron transitions that control the output of the atomic clock in the higher gravity potential 'could' be viewed as the electron transitions increasing in frequency due to additional energy, this being potential energy, whereas the black body's observed increase in frequency of electron transitions is due to temperature energy.

If one were to apply DeBroglie wave functions to either scenario:
Wavelength =h/p
...via the Einstein Planck relation which should hold for both the function of the light emitter, and the emitted light.

However - if one views the increase in frequency of light emitted from the blackbody as being due to an increase in the electron transitions of the constituent atoms of the blackbody, and calculates this as an increase in the rate of time for the constituent atoms of the blackbody - if one then considers the addition of joules/second as +energy=joules per/shorter second, and then recalculates Planck's blackbody data curve under the remit of 'variable' seconds, then the quantum nature of the Planck ultra violet catastrophe is negated, and Planck's h constant is a function of variable time.

If one then takes this view back to the g-field, one must conclude:
a) that the atom at h from M is emitting a higher frequency photon
and...
b) that the already emitted photon is shifting frequency in the g-field due to the energy of the g-field
...where the g-field at a higher potential has lower energy, in the face of a particle mass at higher potential having a higher energy, where:
pe=mgh
and where m=0, no potential energy is added.
Title: Re: Is there a discrepancy with the equivalence principle?
Post by: timey on 03/04/2017 12:19:47
I really wouldn't mind at all if one were to tell me that they had applied the remit of adding joules as per variable second to the ultraviolet catastrophe, and that I am mistaken in thinking that the remit that I suggest results in a continuum that negates the quantum nature of Planck's interpretation of the  blackbody data...

However, it's pretty clear to me that I am not mistaken...
If one takes the increase in frequency of the blackbody's emission of photons as an indication of the emitting atoms increased rate of time, and then calculates the additional energy that caused the increase in frequency via the shorter second, the data will be a continuum.

This is exactly the same as the difference between measuring clock's that are in the higher gravity potential 'from' the lower potential via the lower gravity potentials rate of time, as opposed to measuring each clock that is raised into the higher potential 'at' the higher potential location of that clock.

In the 1st instance the rate of time is held as an invariant as per the lower potential clock's rate of time...
This being the same as measuring additions of joules as per invariant second to the blackbody.

In the 2nd instance the rate of time is held as a variable at each gravity potential, where the clock when measured via the rate of time 'at' that gravity potential is always measured as 9,192,631,770 Hz...
This being the same as measuring additions of joules as per variable seconds to the blackbody.

Planck himself would have welcomed this notion with unmitigated joy, as he tried everything to iron out his data, resulting in his h constant that he first described as being a fudge factor.
Title: Re: Is there a discrepancy with the equivalence principle?
Post by: timey on 04/04/2017 12:28:46
Can someone please tell me if my suggested treatment of the Planck blackbody data is misconceived or not...
Title: Re: Is there a discrepancy with the equivalence principle?
Post by: Mike Gale on 05/04/2017 04:31:53
By 'energy reactive', I presume you mean that the characteristic frequency of the atomic clock or black body radiator is affected by time dilation. That is true, but in either case, the radiator is an oscillator (i.e. a clock) and you need a time keeper (i.e. another clock) to measure the frequency of the light. If you need a mental picture of the mechanism by which an electron state transition produces a photon, think of a disk with an off-center hole rotating on edge in a gravitational field. There are two stable states, one with the hole on the top and another with the hole on the bottom. The transition from one state to the other involves a lot of wobbling. The wobble frequency corresponds to the photon frequency.
Title: Re: Is there a discrepancy with the equivalence principle?
Post by: timey on 05/04/2017 13:19:33
Well Mike - I am appreciating your description of oscillations, and point you towards the electron transitions (oscillations) in relation to energy...
But let me now return to the question of time itself.

If you have read Sean Carol's book 'from here to eternity', and you have also read Lee Smolin's book the trouble with physics', then you are aware that physics does not have a fully formed theory on time.
And you yourself must be aware from your own mathematical ventures that while GR is stating variable times, SR by default must hold the speed of light relative to 'a' clock somewhere.

It is a fully accepted notion via GR and SR that time flows at differing rates in the universe and that observers clock's under specified circumstances will not agree.

Now consider that there is actually an experimental app created that can tell a person how fast or slow they are ageing due to their longitude, height above sea level, and speed of travel.  This undoubtably being held relative to how a stationary (relative to Earth) person is ageing at a particular longitude and height above sea level...
Thousands of people in the UK were all moving around their daily business experiencing very slightly differing rates of time.
The interesting thing about this is that any of these people using this app will still agree that the day has passed by at the same rate.  They can meet up with each other having each spent their day at differing locations of differing height above sea level, differing longitudes, and differing relative motion, but meet to sit down at a specified 'dinner time', where everyone agrees that it is the specified time.
If we, for the purposes of illustration, wildly exaggerated the time differences between differing heights above sea level, longitude, and speed of motion, under the remit of the phone app it is possible to understand that these wildly differing rates of time that the app is registering due to people's differing ageing rates as they go about their daily business would not affect their ability to meet anyone at a specified time for dinner.  A person at the station may be ageing say 100 times faster than the person on the high speed train that is passing the station but both will still observe the sun to set when the sun sets.
Now in this case we can clearly work out that the phone app that is using the GPS on one's phone to register one's height above sea level, longitude, and speed of travel in order to apply the GR and SR calculations of time dilations is also providing everyone with a synchronised clock reading, and this clock reading is based on Greenwich Mean Time where all mobile phones, in UK, no matter their circumstances of location or speed, are all keeping the same time.
Therefore the rate a person ages at, even in this wildly exaggerated scenario, would not affect their ability to meet up for dinner at a specified time, or for the person at the station and the person on the high speed train passing through the station to view the sun setting at the same moment that the sun sets.

Let's look at this another way.  Continuing our wildly exaggerated differences in ageing scenario, lets say that the mobile phones of all the people using the app were to adjust each mobile phone's clock as per the apps time calculations as to height above sea level, longitude, and speed of travel.
The sun isn't going move at differing rates throughout the day for each differing clock rate.  The person at the station ageing 100 times faster than the person on the high speed train passing through the station, their mobile phone's variable clock readings do not agree, but if they both took a picture with their mobile phone of the sunset at the moment the train passed through the station, and then texted each other the picture, it would be clear that the sun would be in the same position in the sky for both people.

From this we can deduce that the rate that a clock runs at does not affect the passing of time as we know it...
People who's personal biological clock's, as per the relativity app, are running faster or slower, than the clock that is calibrated to serve to synchronise the nations mobile phone clock's, are ageing faster or slower, but this ageing faster or slower on a personal scale does not affect the observation of how fast the sun moves across the sky, and 'when' the sun will set.
If the mobile phones were set to display both the variable time, and the synchronised time, then the person on the station who's time was running 100 times faster than the person on the high speed train, and 10% faster than the synchronised mobile phone time, could calculate (in the Spring), that the sun sets 2.46 hours earlier each day that passes, relative to the synchronised time, where the person on the train who's time is running 90% slower than synchronised mobile phone time, could calculate that the sun sets 90 times in one day, relative to the synchronised clock.
The person on the train can see that his 1 and a half hour journey held relative to the synchronised mobile phone time will only take a minute held relative to his variable time clock.  Clearly he will not be observing the cows or rabbits in the fields, moving around any faster than cows and rabbits usually move.  If he times their movements via his variable clock, they will move at a faster speed when held relative to the variable clock, and if he times their movements via his synchronised mobile phone clock, they will move at slower speeds when held relative to the synchronised clock,
Therefore it is clearly obvious that despite the rate of time the clock's, and the people's biological body clock's are running at, that in each case the 'amount' of time is 'the same', and that each clock is making a differing measure of the 'same amount' of time.

We could now deduce that the clock's and biological body clock's running at differing rates at differing heights from sea level, longitudes, and relative speeds are doing so as a physical reaction to their locational conditions, and that these physical reactions to local conditions do not affect the actual sequential events of the local itself.

This is a description of relativity conventional.
Is there any point here that you disagree with?
Title: Re: Is there a discrepancy with the equivalence principle?
Post by: alancalverd on 06/04/2017 01:19:00
Can someone please tell me if my suggested treatment of the Planck blackbody data is misconceived or not...

Yes. The blackbody spectrum is a continuum, not a line spectrum, by definition.

h was never a "fudge factor" but a necessary dimensioned constant required to relate frequency to energy. The value of h depends on your units of measurement (imperial, metric, avoirdupois...) but its dimensions do not.
Title: Re: Is there a discrepancy with the equivalence principle?
Post by: timey on 06/04/2017 02:20:43
The point is that a higher frequency be associated with a faster rate of time for the emitting body.
If Planck, instead of using an invariant second, had used a variable second to measure his energy per second additions - where the increased energy causes a faster rate of time for the emitter, which then causes a higher frequency photon to be emitted - calculating the energy increases under the remit of +energy=shorter seconds, the ultra violet catastrophe would remain within the classical prediction under this remit.

It doesn't matter which units are used.  It only matters that the energy additions are observed to cause frequency increases, and that one consider the increase in frequency to be indicative of a shorter second.
When using the shorter length of second indicated by the increased frequency to measure the energy input that caused the frequency increase, the quantum nature of the energy additions will be negated.

I'm pretty certain that I am not mistaken in the mechanics of this notion.

As an aside, it was Planck himself who was quoted as having referred to his h constant as a fudge factor.
Yes - it was necessary for Planck to add the h constant to make sense of his data and relate energy to frequency, but only because he couldn't figure out any other means by which to form the relationship...
If I am not mistaken the remit I suggest affords another means by which to form the relationship.
Title: Re: Is there a discrepancy with the equivalence principle?
Post by: Mike Gale on 06/04/2017 03:49:55
Can someone please tell me if my suggested treatment of the Planck blackbody data is misconceived or not...

Yes. The blackbody spectrum is a continuum, not a line spectrum, by definition.

h was never a "fudge factor" but a necessary dimensioned constant required to relate frequency to energy. The value of h depends on your units of measurement (imperial, metric, avoirdupois...) but its dimensions do not.
It's not entirely inaccurate to call it a fudge factor. It was a shot in the dark that paid off. He called it an act of despair. It is admittedly nothing more than a unit conversion factor, but the point is that photon energy depends on frequency and nothing else. That means all light waves have the same amplitude. As such, photon energy is affected by time dilation, but not space dilation (except insofar as the two are related.) It's really just another way of saying that the speed of light is invariant.
Title: Re: Is there a discrepancy with the equivalence principle?
Post by: timey on 06/04/2017 13:05:00
Mike - Do you disagree at any point with post 488?
Title: Re: Is there a discrepancy with the equivalence principle?
Post by: Mike Gale on 07/04/2017 00:56:32
Some, but not all:

1) Time dilation between Earth dwellers does not depend on longitude, only altitude and ground speed.
2) If all of the busybodies meet at the pub at sunset, each will have aged by a different amount. Everyone will agree that the sun has indeed set; they just won't agree about the elapsed time since sunrise.
3) The app would have to involve futuristic physics because we don't currently know how to calculate time dilation as the observers speed up, slow down and change directions. Problems like that have to be solved in a piecemeal fashion using numerical approximation techniques.
Title: Re: Is there a discrepancy with the equivalence principle?
Post by: timey on 07/04/2017 01:18:55
At a more southern longitude (in UK) gravity is weaker than it is at a more northern longitude of same altitude.  This being because of the equatorial bulge.
However, the time dilation difference at the more southern longitude is exactly cancelled out by the SR effects of the more southern longitude's greater speed of rotation, so you are correct that longitude does not cause a difference in time dilation and that it is only speed and altitude causing a difference.

As to the mobile phone app I refer to, it exists in reality.  It was developed by Professor of physics Jim Al-Khalilli, and you can watch a program called "Gravity and Me: The force that shapes us"... that refers to this app and is well worth a watch.

Number 2:  Yes - precisely, which suggests that time dilation for mass is a physical reaction caused by the conditions of the local, and that this time dilation that is affecting mass is not related to the sequential events of the local.
Correct?
Title: Re: Is there a discrepancy with the equivalence principle?
Post by: Mike Gale on 07/04/2017 02:18:01
The bulge is a red herring because it doesn't affect PE. The physics of rotating reference frames are a quagmire. You should simplify your scenario by assuming a spherical, non-rotating Earth.
Jim's app will employ numerical approximations. It will also suffer from lack of precision in altitude and ground speed.
Your experience of time has nothing to do with your mass, only your altitude and ground speed.
Title: Re: Is there a discrepancy with the equivalence principle?
Post by: timey on 07/04/2017 03:13:46
It does affect PE because at the equator the bulge renders one further from the centre of mass.  However, as said, and I am quoting Professor Jim Al-Khalilli directly, the rotational speed as one gets closer to the equator increases and SR time dilation exactly cancels out the GR time dilation caused by the bulge.
...and Jims app will actually be as precise at determining position, altitude and speed as GPS is.

However this is not the point I am making here.
Quote
2) If all of the busybodies meet at the pub at sunset, each will have aged by a different amount. Everyone will agree that the sun has indeed set; they just won't agree about the elapsed time since sunrise.
Quote
Number 2:  Yes - precisely, which suggests that time dilation for mass is a physical reaction caused by the conditions of the local, and that this time dilation that is affecting mass is not related to the sequential events of the local.
Correct?

Title: Re: Is there a discrepancy with the equivalence principle?
Post by: Mike Gale on 07/04/2017 04:04:55
Jim is wrong on that point. Polar time at sea level is the same as equatorial time at sea level. It's a case of two wrongs making a right. (The change in gravity due to the bulge is canceled by centripetal acceleration. SR doesn't come into play because these observers are stationary with respect to one another and with respect to the center of mass. A spherically symmetric, non-rotating Earth is the correct model for calculating GR time dilation on the ground.) The GPS literature paints a more accurate picture. See for example: http://physics.stackexchange.com/questions/126919/does-time-move-slower-at-the-equator
Title: Re: Is there a discrepancy with the equivalence principle?
Post by: Mike Gale on 07/04/2017 04:08:16
With regards to point #2, mass has nothing to do with it. It's all about ground speed (i.e. SR) and altitude (i.e. GR.)
Title: Re: Is there a discrepancy with the equivalence principle?
Post by: Mike Gale on 07/04/2017 04:43:39
Note that the Kerr metric describes how to calculate dilation from the perspective of an observer who is not rotating with the Earth. It's not recommended for the faint of heart though.
Title: Re: Is there a discrepancy with the equivalence principle?
Post by: timey on 07/04/2017 12:56:30
Quote
Jim is wrong on that point.
Polar time at sea level is the same as equatorial time at sea level. It's a case of two wrongs making a right.

Yes that is what Jim said, and what I wrote.

You cannot say this...
Quote
(The change in gravity due to the bulge is canceled by centripetal acceleration.
...and then say this:
Quote
SR doesn't come into play because these observers are stationary with respect to one another and with respect to the center of mass.
...because you have contradicted yourself.
The centripetal acceleration  (SR) at the equator exactly cancels out the change in gravity (GR) at the equator *for an observer* at the equator because *the observer* at sea level at the equator is moving in space faster than *the observer* at sea level at a polar location is.  This is 'how' the change in gravity (GR) is cancelled by the centripetal acceleration (SR).

I have *highlighted* the term *the observer* above for a reason:
Quote
With regards to point #2, mass has nothing to do with it. It's all about ground speed (i.e. SR) and altitude (i.e. GR.)
Without mass being involved, what exactly are we stating as having a ground speed (SR) and altitude (GR)?
It would be impossible to measure either SR or GR effects without a mass being involved.

Therefore:
Suggesting that time dilation for mass is a physical reaction caused by the conditions of the local, and that these SR and GR time dilation effects that are affecting mass are not related to the sequential events of the local...
Because, as you have said:
Quote
If all of the busybodies meet at the pub at sunset, each will have aged by a different amount. Everyone will agree that the sun has indeed set; they just won't agree about the elapsed time since sunrise.
Where the elapsed time between sunrise and sunset (on any particular day) is an invariant amount of time, but the busybodies have experienced that invariant amount of time (associated with that calendar day) differently to each other.

The point being that despite the busybodies experience of their own time, the *actual amount* of time that has passed from sunrise to sunset (that day) remains the same.
This is illustrated in post 488 by asking the mobile phone app to display both the Relativity app time as to the phones location and speed, and the synchronised time that all mobile phones in UK display as a norm.

To give an idea of direction, I am discussing these matters with a view to examining what the rate of time is doing where m=0, such as the spaces in the universe where mass is absent.
Title: Re: Is there a discrepancy with the equivalence principle?
Post by: jeffreyH on 07/04/2017 19:30:22
I am proposing a 4th time dilation that completely cancels the 3rd time dilation via vacuum energy. I can't decide yet whether it is Dyson, Hoover or Henry.
Title: Re: Is there a discrepancy with the equivalence principle?
Post by: timey on 07/04/2017 19:43:36
Please see your linear space vector thread whereas I am pleased to be adding to your understanding of conventional physics.

No need for an apology!
Title: Re: Is there a discrepancy with the equivalence principle?
Post by: jeffreyH on 07/04/2017 20:02:33
I agree. You don't need to apologise. I forgive you.
Title: Re: Is there a discrepancy with the equivalence principle?
Post by: timey on 07/04/2017 21:50:25
... I've now changed my mind Jeff and will be expecting an apology from you...

And for anyone else reading, please know that I always start out from the viewpoint that the other person has a better understanding of conventional physics than I do - and it is surprising to me how very rarely this is actually the case...
Title: Re: Is there a discrepancy with the equivalence principle?
Post by: jeffreyH on 07/04/2017 22:14:07
Was it Dyson, Hoover or Henry that gave me away?
Title: Re: Is there a discrepancy with the equivalence principle?
Post by: jeffreyH on 07/04/2017 22:20:04
That is Freeman Dyson, William G Hoover and Joseph Henry. You can google any one of them and learn a bit of science history.
Title: Re: Is there a discrepancy with the equivalence principle?
Post by: timey on 07/04/2017 22:25:01
No - it is your lack of understanding of conventional physics that gives you away.
I suggest that you actually learn about a subject before embarrassing yourself by posing as an authority.
Title: Re: Is there a discrepancy with the equivalence principle?
Post by: jeffreyH on 07/04/2017 22:54:46
I am definitely not an authority and have an awful lot to learn yet. I have gotten through 26 mathematics and physics books. On a variety of different subjects. I will very likely double the number eventually. I don't profess to have a theory of anything.
Title: Re: Is there a discrepancy with the equivalence principle?
Post by: timey on 07/04/2017 23:05:27
But you do profess to know that GR and SR time dilations are one and the same thing...

Which book did you learn that from please?
Title: Re: Is there a discrepancy with the equivalence principle?
Post by: Ethos_ on 07/04/2017 23:12:01
I think we would all be better served if each and everyone of us would leave our egos at the front door.................Just sayin
Title: Re: Is there a discrepancy with the equivalence principle?
Post by: timey on 07/04/2017 23:20:16
Ethos - In my case this is a matter of honour not ego...
I do not have a problem if someone ridicules my posts on the basis of an informed position, but Jeff is basing his response on incorrect information that he has posted in response to my posts on his 'is there a linear vector space that can be used with gravitational fields' thread.
I never have a problem admitting my weak spots or mistakes, but I do have a problem with a person posing as an authority and ridiculing me based on an understanding of conventional physics that is incorrect.
Title: Re: Is there a discrepancy with the equivalence principle?
Post by: Ethos_ on 07/04/2017 23:57:13
Ethos - In my case this is a matter of honour not ego...
I do not have a problem if someone ridicules my posts on the basis of an informed position, but Jeff is basing his response on incorrect information that he has posted in response to my posts on his 'is there a linear vector space that can be used with gravitational fields' thread.
I never have a problem admitting my weak spots or mistakes, but I do have a problem with a person posing as an authority and ridiculing me based on an understanding of conventional physics that is incorrect.
My friend, I wasn't singling out anyone in particular.

I've been following all your threads Timey and consider you to be very knowledgeable even though you sometimes stray. But of course, we all do some of that from time to time. So please don't consider this an attack Timey, I wish to only encourage you.

I don't offer my views that often but I do like considering new thoughts our members at various times offer. And I do think you may be on to something but providing empirical evidence is going to be very difficult. You see, the problem with defining shorter or longer seconds hinges upon what you call the "standard second" and establishing this standard requires a universal point of reference. And sadly, no such universal point of reference is attainable according to current understanding.

Please don't consider this an attack my friend, I'm not here to exercise my ego. But there are a few problems you'll need to overcome if your theory is ever to succeed.

Again, I commend you on your logic and suspect you might be right. But to prove your case, you'll need to establish what you call the "universal common now", and I doubt that this is possible. And, you will also need to establish a "universal frame of reference" and that is also going to fail as well.

Do you have a thought on an experiment we could pursue that might navigate around these impediments and effectively come at your theory from a different angle?



Title: Re: Is there a discrepancy with the equivalence principle?
Post by: alancalverd on 08/04/2017 00:04:03
The point is that a higher frequency be associated with a faster rate of time for the emitting body.
Then if we take a single spectral line, we'd expect it to shift according to the temperature of the emitter. It doesn't. Lines appear and disappear according to whether the temperature is high enough to bring bound electrons into particular energy states, but the inter-state energy, and hence the frequency of the photon emitted or absorbed by that state change, is not temperature-dependent. Therefore time is not temperature-dependent. http://www.astronomynotes.com/starprop/s12.htm has good diagrams.
Title: Re: Is there a discrepancy with the equivalence principle?
Post by: alancalverd on 08/04/2017 00:06:55
Was it Dyson, Hoover or Henry that gave me away?

I'm sure there is a "sucker" joke lurking in the cybersphere.
Title: Re: Is there a discrepancy with the equivalence principle?
Post by: timey on 08/04/2017 00:16:35
Ethos

If one considers that GR and SR time dilations are only affecting mass and do not affect the open space around the mass, and then considers that the acceleration of gravity in the g-field surrounding M is due to a 3rd time dilation that is contra directional to GR time dilation in the g-field, then not only can one calibrate a clock to run as per standard second anywhere on Earth at sea level, as long as one remembers the centripetal speed of that longitude from which to calculate SR, one has a standard second to measure further GR and SR effects from, one will also find one has an absolute frame that is determined by the gravity field of M, or more realistically from the multiple gravity fields of M's, from which to do so.
Title: Re: Is there a discrepancy with the equivalence principle?
Post by: timey on 08/04/2017 00:20:24
The point is that a higher frequency be associated with a faster rate of time for the emitting body.
Then if we take a single spectral line, we'd expect it to shift according to the temperature of the emitter. It doesn't. Lines appear and disappear according to whether the temperature is high enough to bring bound electrons into particular energy states, but the inter-state energy, and hence the frequency of the photon emitted or absorbed by that state change, is not temperature-dependent. Therefore time is not temperature-dependent. http://www.astronomynotes.com/starprop/s12.htm has good diagrams.

You are correct Alan, time is not temperature dependent...
But temperature is energy dependant, electron transitions are energy dependent, and if one applies temperature to the blackbody, when it gets hot it emits photons.
Title: Re: Is there a discrepancy with the equivalence principle?
Post by: timey on 08/04/2017 02:02:05
In addition to post above:

The photons that the blackbody emits as it gets hotter are of higher frequency, and a higher frequency of photon is associated with a higher energy level and also a higher frequency of electron transitions.

Where else do we observe a higher frequency of electron transitions?
Oh yes, that's right, an atomic clock is observed to have a higher frequency of electron transitions in the higher gravity potential, when observed from a lower potential.
We say 'when observed from a lower potential' because if one observes the clock in the higher gravity potential when one is with the clock, it will be the lower potential clock that appears to be running slow.
This being in keeping with GR prediction that clock's will run at progressively faster rates in the progressively higher potentials.

But what is causing the higher frequency of electron transitions for the atomic clock in the higher potential?
A higher frequency of electron transitions requires additional energy.  But that's Ok because at the higher gravity potential pe=mgh, where pe/m=an equal energy addition for any value mass, and therefore all mass will be affected equally at each potential.

Now going back to measuring the clocks in differing potentials:
The higher potential clock appears to run fast from the lower potential, and the lower potential clock appears to run slow from the higher potential.  Clearly if one is measuring the frequency of electron transitions via variable time then one gets differing results dependant on the clock one decides to measure with.
Bingo!
Take that notion back to the blackbody, (and stars of differing temperatures), and where we observe the results of higher frequency electron transitions (emitted photons), we apply the remit of measuring via variable time, where just like observed with the atomic clock, we say that an increase in the frequency of electron transitions is indicative of a faster rate of time, and then we go back to the ultraviolet catastrophe and apply the +energy equals shorter seconds to the energy increases applied to the blackbody.

To explain further:
When Planck increased the energy input to the blackbody that increased the temperature, he measured these increases as a per second measurement.
Planck used an invariant second...
By measuring these energy increases via a second that is getting shorter in keeping with the electron transition frequency increases, the quantum nature of the energy increase relationship to frequency output will be negated.
Title: Re: Is there a discrepancy with the equivalence principle?
Post by: Colin2B on 08/04/2017 09:13:14

To follow up what Ethos is saying and offered in the same spirit and hope of bringing a common understanding:



But you do profess to know that GR and SR time dilations are one and the same thing...
Jeff didn't actually say that, what he did say was:


The time dilation of SR is a special case of the time dilation of GR hence why it I called special relativity. It simply omits the gravitational field and uses flat spacetime exclusively. This is contained within the framework of general relativity. They are only distinguished to show differences caused by the presence or absence of a gravitational field. To make them distinct entities is an artificial device.



You are both 'correct'. SR is a subset (special case) of GR - which can be used to calculate both dilations as one. However, to aid teaching we (artificially?) separate them and usually find it easier to treat them as separate calculations.

Title: Re: Is there a discrepancy with the equivalence principle?
Post by: timey on 08/04/2017 14:15:09
Well Colin, of course it is clearly set out in physics that SR is used to describe motion in 'flat' space, but it is also clear that SR is also used to calculate time dilation for orbitals, or centripetal rotations, or relativity apps, that quite clearly also include a GR altitude related time dilation where the SR effects cancel out some, or all of the GR altitude time dilation effects, resulting in a clock that runs slower than a 'ground clock'... however please correct me if I am wrong, but...

The birds eye overview of the physics situation is that there are a multitude of differing means to accurately calculate a multitude of differing circumstances, but there isn't a means by which one method of calculation can describe all circumstance.

For instance:
SR is used in particle physics, but the standard model cannot be united with gravity..
The GR field equations can describe a gravitational field, but cannot describe multiple fields, therefore has trouble describing galaxies.
The GR field equations break down to infinities in a black hole.
Etc...

So to examine SR:
Quote
In the Special Theory of Relativity, published in his so-called “miraculous year” of 1905, Einstein had the audacity to turn the question around and ask: what must happen to our common notions of space and time so that when the distance light travels in a given time is measured, the answer is always 300,000 km/s? For example, if a spaceship fires a laser beam at a piece of space debris flying towards it at half the speed of light, the laser beam still travels at exactly the speed of light, not at one-and-a-half times the speed of light. He began to realize that either the measurement of the distance must be smaller than expected, or the time taken must be greater than expected, or both.

In a nutshell, the Special Theory of Relativity tells us that a moving object measures shorter in its direction of motion as its velocity increases until, at the speed of light, it disappears. It also tells us that moving clocks run more slowly as their velocity increases until, at the speed of light, they stop running altogether. In fact, it also tells us (as we will see in subsequent sections) that the mass of a moving object measures more as its velocity increases until, at the speed of light, it becomes infinite.

Thus, one person’s interval of space is not the same as another person’s, and time runs at different rates for different observers travelling at different speeds.

So - what I am doing is having the audacity to turn the calculations of the observations around and describe these observations under a remit that 'should' be able to describe all circumstance by means of the same method of calculation.

By remit of logic it is my thoughts that a remix is justified because GR holds that the rate of time, i.e. the length of the period of a second, is dependent upon one's coordinate, but SR holds the speed of light relative to a 'specific' period of time...  This is a contradiction, therefore logically speaking there will be an alternative method.

This method of calculation that I am suggesting will still be Einstein's equation of GR (describing a contracting universe), and be inclusive of the Lorentz transformation calculation, but I am attributing physical cause and effect mechanics for all phenomenon (inclusive of Big Bang and Inflation) during my remix of where and why these equations are relevant.

The first step is to attribute the phenomenon of the acceleration of gravity a physical cause.  A test particle m of any value is observed to accelerate >>>M, and decelerate <<<M.  By stating that there is a time dilation phenomenon where m=0, i.e: the g-field surrounding M, that is inherent with longer seconds as the g-field gets weaker with distance from M, what I have done is partially transferred some of the SR remit to the g-field itself, and this now becomes a means of describing space curvature.  The dimensions of space as per a Newtonian geometry are, at each coordinate, dilated or contracted by time, where the metres of distance traveled remain constant (no distance curvature), and it takes a longer or shorter amount of time to travel a metre.

Under this remit we have not involved any factor of m when describing the motion of m in relation to M, and this constitutes a means of a description of free fall.

It does not however constitute a description of directional force in the g-field, and for this we must now look to the electron transitions of m in the gravity potential, and the electron transitions in relation to energy increases such as the blackbody.

Within a Newtonian geometry of space that curves motions via an acceleration deceleration of the rate of time, we now have a compatibility with the remit of electrodynamics, and the conversation above concerning the ultraviolet catastrophe and re-calculating the energy increases via variable seconds that are getting shorter in keeping with electron transition frequency increases should unite the point particle model with the wave function model.

This then gives us another means of an alternative interpretation of Hubble's red shift velocities, and it becomes possible to consider the contracting model, and we can now go back to the GR observed time dilation phenomenon and add this in again for m in relation to M due to potential energy, and to the SR observed time dilation and add this in for m in relative motion to M, also due to potential energy, where these 2 time dilations and the 3rd time dilation of the g-field can be used in matrix maths annexed to the 3 dimensions of coordinate space to result in the 'proper time' of a four dimensional spacetime matrix.

I can explain the physical mechanics of how this calculation should be articulated, but have no idea how to go about describing such in mathematical format.

And it is the translating of these physical mechanics into mathematical format that I am here at this forum seeking help with.
Title: Re: Is there a discrepancy with the equivalence principle?
Post by: alancalverd on 08/04/2017 14:48:45
So we now have the suggestion that increasing the temperature of the groundlevel clock will remove the observed discrepancy between it and the clock at altitude. But it doesn't, because electron state transitions within an atom are not temperature-dependent for a given state - remember what I said about spectral lines in # 513 above.That's quantum mechanics.  Time is measured by quantum transitions of bound electrons.

Black body photons are a continuum produced by oscillation of free electrons. Each photon has a discrete energy but there are no forbidden or preferred energies - the definition of blackbody radiation! 

The reason for cooling cesium clocks is to reduce thermal (black body!) noise, Doppler jitter, and interference from nearby unwanted near-ground-state transitions that might be excited thermally: the hyperfine splitting is a very weak signal to be detected against a background of all sorts of irrelevant stuff.
Title: Re: Is there a discrepancy with the equivalence principle?
Post by: jeffreyH on 08/04/2017 15:26:01
In special relativity in a universe containing mass there is never a position within it where there is zero probability of a gravitational force operating on a test mass. Not even in deep space. Therefore special relativity is by necessity artificially contrived. The effect of the force may be infinitesimally small but its probability is never zero is the point.
Title: Re: Is there a discrepancy with the equivalence principle?
Post by: alancalverd on 08/04/2017 15:35:27
If there is more than one mass other than the test mass, there is at least one point where the net gravitational field is zero. Just sum the vectors and look carefully!
Title: Re: Is there a discrepancy with the equivalence principle?
Post by: jeffreyH on 08/04/2017 15:37:04
If there is more than one mass other than the test mass, there is at least one point where the net gravitational field is zero. Just sum the vectors and look carefully!

You got me gov. It's a fair cop.
Title: Re: Is there a discrepancy with the equivalence principle?
Post by: guest4091 on 08/04/2017 16:54:33
Will there be a conclusion/resolution concerning the question, in our lifetime?
Title: Re: Is there a discrepancy with the equivalence principle?
Post by: jeffreyH on 08/04/2017 17:25:59
Will there be a conclusion/resolution concerning the question, in our lifetime?

Quite probably not.
Title: Re: Is there a discrepancy with the equivalence principle?
Post by: timey on 08/04/2017 17:33:43
So we now have the suggestion that increasing the temperature of the groundlevel clock will remove the observed discrepancy between it and the clock at altitude. But it doesn't, because electron state transitions within an atom are not temperature-dependent for a given state - remember what I said about spectral lines in # 513 above.That's quantum mechanics.  Time is measured by quantum transitions of bound electrons.

Black body photons are a continuum produced by oscillation of free electrons. Each photon has a discrete energy but there are no forbidden or preferred energies - the definition of blackbody radiation! 

The reason for cooling cesium clocks is to reduce thermal (black body!) noise, Doppler jitter, and interference from nearby unwanted near-ground-state transitions that might be excited thermally: the hyperfine splitting is a very weak signal to be detected against a background of all sorts of irrelevant stuff.

The electrons of the blackbody emitted frequencies are free electrons.
The electron transitions occurring in the atoms of the blackbody that are associated with a photon being emitted are bound electrons.

In the case of the blackbody, all that 'jitter' caused by inputting energy causes the bound electrons to increase in the frequency of their transitions.
As the energy input is increased the fact that electron transition frequency is increasing is clearly illustrated in the fact the radiation emissions are increased in frequency.
Because of the 'jitter' it would be impossible to observe the electron transitions occurring within the atoms of the blackbody where the energy input is causing a transference of energy to frequency output, but the fact that they are occurring and the value of the frequency can be determined directly from the frequency of the emitted radiation.

The atomic clock on the other hand is being subject to a differing but otherwise incredibly similar energy input.  The caesium atoms are being excited by a microwave beam.
Needless to say a microwave beam causes an energy transfer in much the same way as any energy input where more energy will result in a higher temperature.

Therefore the fact is that yes, if one were to calibrate the microwave beam of the lower potential atomic clock differently, that one could cause the lower clock to be calibrated to run, 'in' the lower potential, at the same rate that the clock in a higher potential is running at.
This is an actual scientifically recognised fact, is used to calibrate satellite clocks for certain purposes, and is known as slanting a clock, where I remember Evan making a post about slanting clocks in this way last year.
Title: Re: Is there a discrepancy with the equivalence principle?
Post by: timey on 08/04/2017 18:50:48
Do you believe that by rambling incoherently you can mesmerise your victims, sorry audience, into submission?

You are really upsetting me now.  All you do is ridicule me because I am saying things that you haven't read in a physics book.
This is New Theories board Jeff.
If you cannot engage in the topic matter do not participate.
There are forum rules you know.
Title: Re: Is there a discrepancy with the equivalence principle?
Post by: timey on 08/04/2017 19:03:09

Chris - I watched a Horizon program on Dark Energy last year where physicists were saying in light of Dark Energy remaining a complete mystery, that perhaps a new approach is required...
Among those physicists was George Efstathiou from Cambridge University.
Do you know him?
Title: Re: Is there a discrepancy with the equivalence principle?
Post by: alancalverd on 08/04/2017 19:06:49


The electrons of the blackbody emitted frequencies are free electrons.
The electron transitions occurring in the atoms of the blackbody that are associated with a photon being emitted are bound electrons.
The first sentence is almost true (what we actually say is that blackbody radiation is emitted by free electrons - it makes more sense). The second sentence therefore cannot be true.

Quote
In the case of the blackbody, all that 'jitter' caused by inputting energy causes the bound electrons to increase in the frequency of their transitions.
There is no jitter in blackbody radiation. It is a continuum, not a line spectrum.
Quote
As the energy input is increased the fact that electron transition frequency is increasing is clearly illustrated in the fact the radiation emissions are increased in frequency.
Nothing to do with transitions, which are a feature of quantised energy levels. Free electrons can have any energy, and lose it in any quantity, thus radiating a continuum.

And be careful with phrases like "transition frequency" - it's ambiguous and can make you look foolish, which you aren't. "Creative writing" has no place in physics; the poetry is in the observation, not the description..   

We'll talk about the meaning of temperature another time.
Title: Re: Is there a discrepancy with the equivalence principle?
Post by: timey on 08/04/2017 21:39:26
Quote
(what we actually say is that blackbody radiation is emitted by free electrons - it makes more sense).
Why does it make more sense?
The energy has been added to the blackbody in order for radiation to occur.  What you suggest makes it sound as though the blackbody itself is superfluous to the experiment.  The blackbody is a conductor and photons are being emitted as a result of energy being conducted via the blackbody.

Energy is converted into heat.  I rub my hands together=adding energy, and the atoms of my hand get warmer.  If I rubbed them together super-fast the atoms in my hand would become heat damaged and could under some very extreme rubbing indeed potentially (although not very likely) cause my hands to burst into flames.

A blackbody is a much better energy conductor than my hands are.  One can add energy, where this energy will not cause the atoms of the blackbody to break down, and yes Alan, the blackbody emits a spectrum of frequency that is a continuum.
But the additions of energy it takes to cause this spectrum of frequency that is a continuum is not a continuum.  It's quantised, or it is under the remit that Planck measured by. (Light wavelength is h/p, where the wavelength and frequency of a wave are inversely proportional to each other.)
Therefore the radiation itself may not be a line spectrum, but the energy additions causing the radiation are synonymous to a line spectrum.

What I suggest is an alternative remit by which to measure the energy additions resulting in the energy increases also being a continuum to match the radiation continuum.

Quote
Nothing to do with transitions, which are a feature of quantised energy levels. Free electrons can have any energy, and lose it in any quantity, thus radiating a continuum.

The point is that the blackbody experiment results in the energy additions being quantised in order to make the data fit the observation of a continuum of radiation.
On the basis that the energy additions are quantised, then the frequency of electron transitions causing photons to be emitted are also relevant.

Quote
There is no jitter in blackbody radiation. It is a continuum, not a line spectrum.
Quote
The reason for cooling cesium clocks is to reduce thermal (black body!) noise,  Doppler jitter, and interference from nearby unwanted near-ground-state transitions that might be excited thermally:
Yes - and this is to enable an element of control as to the output frequency of the clock.
Quote
:Sci-tech
Microwave ovens use radio waves at a specifically set frequency to agitate water molecules in food. As these water molecules get increasingly agitated they begin to vibrate at the atomic level and generate heat.
This is a far cry from what the microwave beam is doing when it excites the cesium atoms of the atomic clock to produce a set frequency of photon but the principle is the same.
The microwave beam exciting the cesium atom causes the atom to vibrate (electron transitions?) at a set frequency by adding energy, and a frequency of photon is emitted by the atom.

The clock when placed in a higher potential has a higher frequency of electron transitions than the lower clock does when observed by the lower clock, and the higher clock observes that the clock in the lower potential is running slow.
In keeping with the GR prediction that time runs faster in the higher potential, and in keeping with the GR prediction that one's own clock will appear to be ticking normally.

However, a higher frequency of electron transitions requires an increase in energy level, unless one is just saying that time runs faster out in space, (why and how one just cannot explain), but I am suggesting, as an alternative, that an energy increase can be realised by considering potential energy.

One of the reasons for doing so (there are a few) stems back to the quantised nature of the blackbody data, where I suggest that one view the frequency increases observed in the radiation of the blackbody as energy input is increased to be indicative of a shorter second, and then recalculate the additions of the energy via the remit of this variable time which 'should' negate the quantum nature of the energy additions to frequency output relationship of the blackbody data.
Title: Re: Is there a discrepancy with the equivalence principle?
Post by: timey on 08/04/2017 21:46:09
Or alternatively (lol) the much shorter version:

But Alan - the radiation emitted from the blackbody may be a continuum, but the energy additions are not...
Planck had to quantise the energy additions to fit the data.
Title: Re: Is there a discrepancy with the equivalence principle?
Post by: Colin2B on 08/04/2017 23:38:16

please correct me if I am wrong,

No, I'm not going to do that. As you have pointed out before this is new theories and you are at liberty to put forward any ideas you have.
My only purpose in posting was to point out that there had been a misunderstanding.

There is another one here:


 (what we actually say is that blackbody radiation is emitted by free electrons - it makes more sense).

Quote
Why does it make more sense?
Compare what you said "emitted frequencies are electrons" with what Alan said "radiation is emitted by free electrons". I think you will agree there is a huge difference.
Title: Re: Is there a discrepancy with the equivalence principle?
Post by: timey on 09/04/2017 00:55:35
Quote
However, please correct me if I am wrong, but...

The birds eye overview of the physics situation is that there are a multitude of differing means to accurately calculate a multitude of differing circumstances, but there isn't a means by which one method of calculation can describe all circumstance.

This was what I asked you to correct me on if I was wrong.
This has nothing to do with a New Theory.  It is a fact that is widely commented upon by professional physicists who are all quite open about the fact that current physics is not adequate in that there is no uniting theory, many known unknowns, and therefore in all likely-hood a multitude of unknown unknowns.

I read books by physicists who are considering these known unknowns in relation to current physics and describing where and why different theories are not compatible, and these are the subjects that I wish to discuss, and explain where I see that the current theories can be calculated differently so that they do unite.

It's fine for Jeff to ask a question on New Theories such as "Is there a linear vector space that can be used with gravitational fields?", where clearly under the current remit of GR there isn't.  And it's fine for Jeff to chat with Mike who was experimenting with a variable speed of light...
But my wishing to discuss here on this forum the matters that I have read/watched/investigated for last 9 years is ridiculed, where I have to put up with Jeff pretending that he doesn't understand that it is an acceptable point of view that SR and GR time dilations be considered and calculated separately???
And you yourself would decline to comment on the fact that there is not a means of describing all circumstance under the remit of a unified theory???

Feels a bit like discrimination to me...

As to your edit:

This is what I said followed by what Alan said:
Quote
The electrons of the blackbody emitted frequencies are free electrons.
Quote
The first sentence is almost true (what we actually say is that blackbody radiation is emitted by free electrons -

This is what you said:

Quote
Compare what you said "emitted frequencies are electrons" with what Alan said "radiation is emitted by free electrons". I think you will agree there is a huge difference.

The mistake I made was in saying free electrons instead of free energy.
Clearly one would not get any phenomenon of radiation unless energy was being conducted by the blackbody, and the point is that although the spectrum of emitted radiation is a continuum, the measure of the energy additions to cause that continuum of spectrum of emissions is not a continuum, and the method of calculating that I suggest would render the energy additions as a continuum.

Why would you choose to point out a mistake like so, in favour of discussing how current physics doesn't have a unifying theory?

"Hey timey, I'm sure you didn't mean to say electrons there 'cos that don't make sense, but for sure there is a lacking of a unified theory, and that is an interesting take on a means to re-calculate the blackbody data.
But just to say, you are aware that if you apply that +energy=shorter seconds remit to the spectrum observed of stars, that because temperature and frequency of stars increases with mass size, this would mean that time would be running faster for the bigger stars than the smaller ones?  Which of course flies in the face of currently held theories..."

"Why yes, I am perfectly aware of this and it is good news indeed, because apart from all the other matters in physics that this notion unifies, now GR maths won't break down in black holes."
Title: Re: Is there a discrepancy with the equivalence principle?
Post by: Mike Gale on 09/04/2017 04:13:13
Jeffrey is hilarious. I had to read that twice because I didn't recognize the brand names at first glance.

Quote
Jim is wrong on that point.
Polar time at sea level is the same as equatorial time at sea level. It's a case of two wrongs making a right.
Yes that is what Jim said, and what I wrote.
But you both got the right result for the wrong reason. The Earth-fixed, Earth-centric reference frame is rotating from the perspective of a deep space observer, but surface-dwellers are not moving with respect to one another.
You cannot say this...
Quote
(The change in gravity due to the bulge is canceled by centripetal acceleration.
...and then say this:
Quote
SR doesn't come into play because these observers are stationary with respect to one another and with respect to the center of mass.
...because you have contradicted yourself.
It seems contradictory, but it's not because the reference frame in which the bulge manifests is rotating. You have to despin the Earth (to get rid of the equatorial bulge) before you can apply the SC solution. It's a valid approach because rotation is relative.
The centripetal acceleration  (SR) at the equator exactly cancels out the change in gravity (GR) at the equator *for an observer* at the equator because *the observer* at sea level at the equator is moving in space faster than *the observer* at sea level at a polar location is.  This is 'how' the change in gravity (GR) is cancelled by the centripetal acceleration (SR).
Moving in space relative to what? There are only 3 points of reference in this scenario: 2 surface dwellers and the center of mass. Each is stationary with respect to the others.
I have *highlighted* the term *the observer* above for a reason:
Quote
With regards to point #2, mass has nothing to do with it. It's all about ground speed (i.e. SR) and altitude (i.e. GR.)
Without mass being involved, what exactly are we stating as having a ground speed (SR) and altitude (GR)?
It would be impossible to measure either SR or GR effects without a mass being involved.
The observers are just clocks. They need not have mass.
Therefore:
Suggesting that time dilation for mass is a physical reaction caused by the conditions of the local, and that these SR and GR time dilation effects that are affecting mass are not related to the sequential events of the local...
Because, as you have said:
Quote
If all of the busybodies meet at the pub at sunset, each will have aged by a different amount. Everyone will agree that the sun has indeed set; they just won't agree about the elapsed time since sunrise.
Where the elapsed time between sunrise and sunset (on any particular day) is an invariant amount of time, but the busybodies have experienced that invariant amount of time (associated with that calendar day) differently to each other.

The point being that despite the busybodies experience of their own time, the *actual amount* of time that has passed from sunrise to sunset (that day) remains the same.
This is illustrated in post 488 by asking the mobile phone app to display both the Relativity app time as to the phones location and speed, and the synchronised time that all mobile phones in UK display as a norm.

To give an idea of direction, I am discussing these matters with a view to examining what the rate of time is doing where m=0, such as the spaces in the universe where mass is absent.
Solar time is invariant. Swatch time is not.
Title: Re: Is there a discrepancy with the equivalence principle?
Post by: Mike Gale on 09/04/2017 04:41:38
You are not wrong to say that physics doesn't have all the answers, but you must not take offense when people try to poke holes in your theories. Remember how Feynman defined the scientific process: guess, predict and compare. Fending off naysayers is part of step #3. It can test your patience, but at least they don't string you up like Galileo anymore.
Title: Re: Is there a discrepancy with the equivalence principle?
Post by: alancalverd on 09/04/2017 12:44:51
Or alternatively (lol) the much shorter version:

But Alan - the radiation emitted from the blackbody may be a continuum, but the energy additions are not...
Planck had to quantise the energy additions to fit the data.

No he didn't. Temperature is not quantised, nor is the thermal energy of a black body. 
Title: Re: Is there a discrepancy with the equivalence principle?
Post by: alancalverd on 09/04/2017 12:54:18

Quote
Microwave ovens use radio waves at a specifically set frequency to agitate water molecules in food. As these water molecules get increasingly agitated they begin to vibrate at the atomic level and generate heat.
This is a far cry from what the microwave beam is doing when it excites the cesium atoms of the atomic clock to produce a set frequency of photon but the principle is the same
The microwave beam exciting the cesium atom causes the atom to vibrate (electron transitions?) at a set frequency by adding energy, and a frequency of photon is emitted by the atom.
No! The frequency we are looklng for has nothing to do with bond stretching or motion of any nucleus  at all. It is a spin-spin transition of an electron in its ground, not excited, state. There is no "vibration" involved - it's a purely quantum phenomenon for which there is no appropriate mesoscopic model.

Quote
The clock when placed in a higher potential has a higher frequency of electron transitions than the lower clock does when observed by the lower clock, and the higher clock observes that the clock in the lower potential is running slow.
You will confuse everyone by talking abour "frequency of transitions", which means the number of transitions occuring per second, not the photon frequency associated with a single transition.
 
Quote
In keeping with the GR prediction that time runs faster in the higher potential, and in keeping with the GR prediction that one's own clock will appear to be ticking normally.
Absolutely correct. Why not leave it at that, since it works? 

Quote
However, a higher frequency of electron transitions requires an increase in energy level, unless one is just saying that time runs faster out in space, (why and how one just cannot explain), but I am suggesting, as an alternative, that an energy increase can be realised by considering potential energy.
Which is exactly how blue shift is calculated, without reinventing (actually disinventing) the cesium clock. All GR does is to give you a generalised formulation that works in accelerating frames as well as stationary gravitational fields.
Title: Re: Is there a discrepancy with the equivalence principle?
Post by: timey on 09/04/2017 14:16:37
Or alternatively (lol) the much shorter version:

But Alan - the radiation emitted from the blackbody may be a continuum, but the energy additions are not...
Planck had to quantise the energy additions to fit the data.

No he didn't. Temperature is not quantised, nor is the thermal energy of a black body. 

(Sigh)...

Planck's h constant emerged from his blackbody experiment.

https://en.m.wikipedia.org/wiki/Planck_constant#Black-body_radiation

Quote
Planck had imposed the quantization of the energy of the oscillators, "a purely formal assumption … actually I did not think much about it…" in his own words,[8]but one which would revolutionize physics. Applying this new approach to Wien's displacement law showed that the "energy element" must be proportional to the frequency of the oscillator, the first version of what is now sometimes termed the "Planck–Einstein relation":

E
h
f

Planck was able to calculate the value of h from experimental data on black-body radiation: his result, 6.55×10−34 J⋅s, is within 1.2% of the currently accepted value.[6] He was also able to make the first determination of the Boltzmann constant kB from the same data and theory.[9]


The h constant is a Joules per invariant second measurement.

I am saying that:
Quote
the quantization of the energy of the oscillators
can be avoided by calculating via a variable second where:
Quote
the "energy element" must be proportional to the frequency of the oscillator,
but the frequency of the oscillator is indicative of the length of second.

So - by measuring each frequency via the remit of its variable second, this will result in the mathematics of a constant frequency for all oscillations, and
Quote
the quantization of the energy of the oscillators
will be negated as a continuum.

Hopefully now that I've interjected the 'proper' terminology, what I'm saying will be more easily understood.
Title: Re: Is there a discrepancy with the equivalence principle?
Post by: alancalverd on 09/04/2017 14:32:02
h has nothing to do with quantisation. It is the arbitrary constant that calculates the energy of any photon whose wavelength is known. The quantised photon model simply talks about standing waves in a box, and delivers the black body spectrum whatever the value of h.

Whilst I'm on my high horse, the experimental data was established well before Planck - he "merely" explained it!

The only oscillator whose frequency indicates a second is, by definition, a cesium clock. The idea that the length of a second is determined by the local oscillator would be fine, except that the radio time signal doesn't depend on whether I'm listening on AM (198 kHz) or FM (98 MHz) - it's determined by the clock at MSF, running at umpteen gigahertz and transmitting at 60 kHz.

You might have some fun using your variable time concept to predict atomic orbitals from the gravitational field of a proton and an electron. If it works, I'll be impressed. 
Title: Re: Is there a discrepancy with the equivalence principle?
Post by: timey on 09/04/2017 15:08:28
Quote
The only oscillator whose frequency indicates a second is, by definition, a cesium clock.

...and the Relativity app that I spoke about in post 488 shows clearly that were these app users to carry portable cesium atomic clocks about their daily routines of change in location, and speed of doing so that all of these cesium atomic clocks would be running at differing rates.

So what was that about the definition of a second?
The point that I was making in post 488 was that
a) A cesium clock is subject to differing oscillations at differing speeds and altitudes where the people with these clocks will have aged faster or slower dependent on their clock's oscillations.
But:
b) That (where I wildly exaggerated the time differences between the person on the station and the person on the high speed train passing the station), because the person on the high speed train and the person sitting at the station can both take a photo of the sun setting at the same moment the train passes the station, where if they send each other a text of the picture they took, the sun would be in the same position in the sky in both pictures, the amount of time that has elapsed during the day will be the same period of time no matter how the person at the train station or the person on the high speed train ages in the experience of their clock's time.
Suggesting:
c) That the time that everyone using the Relativity app is ageing by and experiencing is a ***separate issue** from the time that they are moving around in.
Title: Re: Is there a discrepancy with the equivalence principle?
Post by: alancalverd on 09/04/2017 17:44:08
Your cesium clock measures time at the place where you are. We are quite used to thermometers and barometers telling us the local temperature and pressure, and there's no need for a universal frame of either.

Your two photographs will show the sun in the same position because "the amount of time that has elapsed during the day" is defined by the position of the sun relative to the earth. But "the amount of time that has elapsed since you set off on your journey" will depend on whether you are measuring it, or someone standing still on the earth, or moving in another train. The Haefle-Keating experiment showed just that. And since the clocks have no universal reference point, they are all correct. That's SR. GR extends the mathematics to an accelerating or gravitational reference frame and comes up with different, but entirely proven, numbers.       

So time is affected by relative speed and gravitation, to what appears to be an entirely predictable extent. What's the problem you are trying to solve by pretending otherwise?

Let's go back to thermometers and barometers. We expect that they will all read the same when next to each other, and we have a notion of absolute zero temperature and absolute vacuum, at which point we would hope they all read zero respectively.  "Deep space" gives us a notion of zero gravitational field, for what it's worth, and we can detect acceleration, so we can find a point where that is zero too. So in principle we can use a clock to measure speed, acceleration or gravitational field, by comparing it with a "deep space" clock . Problem is that we know it is affected by all three variables, so it isn't obvious which one we are measuring unless we can also measure our velocity relative to the reference clock.   
Title: Re: Is there a discrepancy with the equivalence principle?
Post by: timey on 09/04/2017 19:15:21
Quote
So time is affected by relative speed and gravitation, to what appears to be an entirely predictable extent. What's the problem you are trying to solve by pretending otherwise?

I am not pretending anything, and have not in any way disagreed with the predicted.

On the mobile phone relativity app the app is calculating how one's relative speed and position of altitude affects one's ageing throughout their daily routine.
(Clearly the differences are minuscule, but this wouldn't be the case in more extreme parts of the universe)
The differences can be held relative to the mobile phone's usual clock, and therefore the mobile phone's usual clock can be known as the reference time, where a 'set' amount of time elapses between sunrise and sun set on any given day.

Therefore the Relativity app clock's time that is displayed relative to the mobile phone's usual clock is a ***separate issue*** to the mobile phone's usual clock's time.

The mobile phone's usual clock is 'historically' based on the motions of the planet in relation to the sun, where a cesium atomic clock calibrated at sea level, despite the differing altitudes due to equatorial bulge and the differing centripetal speeds at differing longitudes, will run at the same rate at sea level Equator and sea level Polar, providing us with a standardisation of the length of a second.
(which I have, by stating the clock as calibrated at sea level Earth, attributed this standard to being the length of a second at the position of sea level Earth)

Clearly anyone using the relativity app anywhere on the planet will notice differing times from the standard as soon as they start moving around and changing altitude...
(We won't go into geological density differences as of yet)
...and these differing rates of time that the app users are registering due to their changes in relative speed and altitude are merely a faster or slower means of measuring the reference time.

However, the important bit of these considerations is that the people moving about at relative speeds relative to the centripetal speed of their longitude, and changing their positions of altitude held relative to sea level, will all be ***physically ageing*** in keeping with their relativity app clock.
They are doing so, as predicted by GR calculations.
(and these very same calculations can be used to calculate the gravitational shift of light, i.e. blueshift.)

So - this suggests that changes in oscillations are occurring not only for the atoms of the clock, but also for the atoms of the person.

Now I ask you to return to the blackbody where, never mind the physics of how temperature results in radiation, the spectrum of frequency shift is described by the Planck Einstein relation, and ask you to put aside your prejudices and consider that an increase in frequency is indicative of a shorter second...

(E=hf... Where Planck has used a standard reference second (mobile phone's usual clock) to measure Joules via)

...and recalculate Joules per second via a variable second that is in keeping with the frequency of the emitted radiation, where the quantised nature of Planck's treatment of the data will then not be necessary.
The energy additions causing higher frequency will, under this remit of measurement, fit the frequency spectrum as an energy continuum.
Title: Re: Is there a discrepancy with the equivalence principle?
Post by: alancalverd on 09/04/2017 20:58:45
Quote
[Now I ask you to return to the blackbody where, never mind the physics of how temperature results in radiation, the spectrum of frequency shift is described by the Planck Einstein relation, and ask you to put aside your prejudices and consider that an increase in frequency is indicative of a shorter second...
There is no "spectrum of frequency shift". The blackbody spectrum is temperature-dependent. Nothing "shifts" at the higher temperature, but the maximum energy of free electrons within the body has increased, so it is capable of releasing higher energy photons, and more photons at any other energy below that.

We know that time runs slower at a lower gravitational potential, so a clock at a higher potential appears to be generating "shorter seconds", and characteristic photons (i.e. line spectra) are blueshifted compared with those generated at the lower potential. Wholly different mechanism from the temperature dependence of blackbody radiation.
Title: Re: Is there a discrepancy with the equivalence principle?
Post by: timey on 09/04/2017 22:11:15
Quote
There is no "spectrum of frequency shift".

I am quite simply referring to the Planck Einstein relation where there is a relationship between Planck's h constant that emerged from Planck's treatment of energy within the data, as we have discussed as per a Wiki Link in the posts above, and the frequency of a photon.

I am quite well aware of how current physics views the time dilation phenomenon.
The temperature dependence of the blackbody is irrelevant to this discussion.  It's the resulting energy of the photons that is of relevance...
Where I will now quote myself:
Quote
: timey
never mind the physics of how temperature results in radiation, the spectrum of frequency shift is described by the Planck Einstein relation, and ask you to put aside your prejudices and consider that an increase in frequency is indicative of a shorter second."

...and know that where I have said spectrum of frequency shift, that I am referring to the fact that the frequency changes as a continuum from low frequency to higher frequency, where energy and frequency are proportional to each other and E=hf.

And that in asking that the increases in frequency be held relative to shorter seconds in order that frequency remain constant, and suggesting that measuring Joules per shorter second relative to the increased frequency will result in an energy continuum, one should realise that:
a) This has nothing to do with temperature and only has to do with the resulting energy producing photons.
b) This considering +energy=shorter seconds is a "New Theory" bit of the discussion.
c) This "New Theory" bit of the discussion, although not predicted by  GR, does follow GR mathematics for an energy continuum, where wave'length' becomes ***time dilation*** related where:
slower time=longer wavelength
faster time=shorter wavelength
and:
d) This results in describing a relationship between the energy of the g-field, i.e. gravitational force energy, and any other type of energy conversation resulting in oscillations of any type.
(see DeBroglie wavelengths)

However this remit, although compatible with the continuum of GR mathematics, has taken the notion of time running fast in 'space' and replaced it with the notion of time running faster for 'mass' in relation to the potential energy of the g-field of M, where the +energy=shorter seconds remit applied to the spectrum of stars of differing M results in time running faster for the bigger stars than it does for smaller stars...

...This directly relates to the next change to conventional physics that my model makes.
This being to attribute another separate phenomenon of time dilation to the g-field of M where m=0 and no potential energy is added, that gives cause for the accelerative force observed of the g-field relating to M.
This renders Einstein's laws of gravity as compatible with both Newtonian geometry, and the summing up of gravity fields as per Newtonian gravity, which means that this remit will also be compatible with electrodynamics.
Title: Re: Is there a discrepancy with the equivalence principle?
Post by: Mike Gale on 09/04/2017 23:48:52
You cannot pin time dilation to space because SR dictates that a moving observer passing through a point in space perceives time differently than one who is standing still at the same location. Furthermore, the discrepancy is not symmetric in the way you might expect. Each observer perceives the other's clock to run slower.
Title: Re: Is there a discrepancy with the equivalence principle?
Post by: timey on 10/04/2017 00:43:42
But I'm not pinning time dilation to space. 

I'm suggesting that motions in the g-field are accelerated or decelerated by a time dilation that is occurring in the g-field.
That this 'additional' time dilation of the g-field will not affect any clock because a clock is mass where the clock will gain or lose potential energy in the g-field, or in relative motion to the g-field and this gaining or losing of potential energy is what is causing the observed GR (altitude) and SR (relative motion) time dilations.

In accordance with the viewpoint that GR time dilation and SR time dilation may be considered as 2 parts of a singular time dilation phenomenon, this additional time dilation can be thought of as a 3rd part of this singular phenomenon in that it just states that pe=mgh and when m=0, no potential energy is added or subtracted, where the time dilation for m=0 can be calculated via the gravitational field strength equation (where we say strength is energy), where g(r)= GmE/r^2.
Title: Re: Is there a discrepancy with the equivalence principle?
Post by: timey on 10/04/2017 13:25:56
https://en.m.wikipedia.org/wiki/Wave–particle_duality

Quote
:Wiki
Wave–particle duality is the concept that every elementary particle or quanticentity may be partly described in terms not only of particles, but also of waves. It expresses the inability of the classicalconcepts "particle" or "wave" to fully describe the behavior of quantum-scaleobjects. As Albert Einstein wrote: "It seems as though we must use sometimes the one theory and sometimes the other, while at times we may use either. We are faced with a new kind of difficulty. We have two contradictory pictures of reality; separately neither of them fully explains the phenomena of light, but together they do."[1]

What this remit that I am suggesting does is unite the point particle model with the wave function model via a physical causation for the wave function of the particle.
Title: Re: Is there a discrepancy with the equivalence principle?
Post by: timey on 11/04/2017 12:54:46
You cannot pin time dilation to space because SR dictates that a moving observer passing through a point in space perceives time differently than one who is standing still at the same location. Furthermore, the discrepancy is not symmetric in the way you might expect. Each observer perceives the other's clock to run slower.

The problem (as far as I'm concerned) with conventional SR is, that despite the remit of GR stating that time is running at differing rates, i.e. differing lengths of seconds, at differing gravity potentials, all SR calculations are held relative to the speed of light, where the speed of light is universally being held relative to the invariant time period of a standard second.

By doing this one is defaulting SR to the remit of a preferred frame.  This preferred frame being the frame of reference where a second 'is' the length of a standard second.

Furthermore SR combines a time dilation factor with a length contraction factor, both of which are determined by the percentage of the speed of light that the travellers motion/speed constitutes, which means that SR is also holding the speed of the traveller relative to the preferred frame of a standard second.

As far as I'm concerned I don't have a problem with the notion of standardising a second, or using the preferred reference frame of a standard second, so long as one realises that is what one is doing...
An atomic cessium clock can be calibrated at sea level Earth to the time period of a standard second, and despite the differences in altitude and centripetal motion between sea level equator and sea level polar which cancel each other out, a cessium atomic clock will run at the same rate at sea level of any longitude on the planet, (leaving aside geological density considerations for now), where any differences from the from the gravity potential and relative motion of any longitude of sea level Earth may then be measured as differing from that length of second if one keeps track of the centripetal rotation and altitude of equatorial bulge at that longitude.*

But - I can see that in order to incorporate both GR and SR time dilation effects simultaneously in all given circumstance is not easily achieved under the current remit.
In some cases GR effects are included, as I have discussed regarding the Relativity app in previous posts, and as laid out in this post concerning equatorial bulge versus centripetal motion.
But when SR is used to describe the motions of light travelling across space, it uses a (non GR) flat space describing the (GR) curvature of space via length contraction and time dilation, where all measurements are by default being held relative to the preferred frame of a standard second.

*Ok - so we know via the remit of conventional physics mathematics that there is an orbital radius from Earth where the orbital speed required to maintain that radius exactly cancels out the GR effects of altitude, where a clock will run at the same rate at this orbital speed, at that radius, as a clock at sea level Earth does...
But SR is being calculated on the basis that the speed of light is held relative to this standard length of second at each and every radius, and is also holding orbital speeds relative to this standard length of second.

If we then take into consideration that SR is also incorporating a calculation of a differing length of measuring stick, this is where it all starts to get super interesting...
At this radius where orbital speed SR time dilation exactly cancels out GR altitude time dilation, the length of a metre will be differing according to conventional SR remit.
But hang on a mo - this is an exact symmetry of what we observe of a longitude on Earth isn't it?
As the distance from centre of Earth increases at longitudes closer to the equatorial bulge causing greater altitude, the speed of centripetal motion also increases and this exactly cancels out the GR altitude time dilation at each and every longitude...
Therefore the calculation of a differing length of measuring stick that SR incorporates within it's time dilation calculation will exactly replicate the proportional differences between altitude of equatorial bulge in relation to centripetal speed of the longitudes on Earth.

So my first question is:
When calculating the centripetal speed of a longitude in relation to altitude of equatorial bulge, is the calculation of the SR effects inclusive of the change in the length of a measuring stick, or have these SR changes in the length of a measuring stick been considered as negligible at these low speeds?

(It is my intention to put all this in the context of the premiss of my model, but will do so in stages, because I've found that longer posts are not responded to)
Title: Re: Is there a discrepancy with the equivalence principle?
Post by: Mike Gale on 12/04/2017 05:06:28
There is no preferred RF in SR because, as Galileo observed, all motion is relative. You may think I'm moving and you're standing still, but from my perspective you're moving and I'm standing still. GR puts a twist on that by introducing a 3rd reference point and a field, which distorts our perspectives.
Title: Re: Is there a discrepancy with the equivalence principle?
Post by: timey on 12/04/2017 13:42:30
Well absolutely...apart from the fact that being 'at rest' is a false scenario because no-where is at rest.
Things or people can only be at rest with respect to each other, and in the case of centripetal acceleration, a person at rest at a polar sea level, and a person at rest at an equatorial sea level may be at rest with respect to the centre of the Earth*, but they are in relative motion to each other with respect to their position of longitude, where the speed at which the equatorial sea level and the polar sea level longitudes are moving around the centre of the Earth through space is differing.
*(Edit: actually on second thoughts unless a person was directly on top of either pole, neither would be at rest with respect to the centre of Earth either, as all positions of longitude will be moving faster than the centre of the Earth.)

But... what about the fact that SR is using the speed of light as a reference point to make its calculations from?

The speed of light is using the reference point of a metre held relative to a set time period...
...and this 'set' time period is a second as denoted by the caesium standard, where it is observed that the caesium standard is not only distorted by relative motion, but also by position in the gravity potential.

You may be disinclined to consider the use of a 'set' time period as being a preferred reference frame, and if you like I shall refer to the use of a 'set' period of time as something else, but what I am wanting to talk about here is the fact that SR is using this caesium standard as a basis to make calculations from.
...And it is more the SR changes in the measuring stick that accompany the SR time dilation calculations that I am interested in discussing with respect to the 21.36km difference in altitude at sea level equator relative to sea level polar, and the fact that the SR time dilation effects caused by the extra rotational speed at each longitude from poles to equator exactly cancels out the GR time dilation effects of increase in altitude at each longitude.

Where I have said:
Quote
Therefore the calculation of a differing length of measuring stick that SR incorporates within it's time dilation calculation will exactly replicate the proportional differences between altitude of equatorial bulge in relation to centripetal speed of the longitudes on Earth.

So let's say that we are dispensing with the GR time dilation effects of altitude and we only use the SR effects of time dilation and change in length of measuring stick to calculate the time dilation differences of centripetal motion in relation to the equatorial bulge?

The inline motion of the centripetal acceleration will shorten the measuring stick for a measure of the radius right?
So as the longitudes increase in centripetal motion from poles to equator, is the shortening of the measuring stick proportional to the extra distance of the radius?
Title: Re: Is there a discrepancy with the equivalence principle?
Post by: timey on 13/04/2017 15:21:14
If we were to imagine for comparison purposes that there were no equatorial bulge, and that the mass of the Earth were to be redistributed so that the distance to centre of Earth was equal at equatorial sea level and polar sea levels - where we then calibrated our caesium atomic clock at sea level equator - we would observe that the equatorial sea level longitude would still be moving faster with respect to the polar sea levels because the earth is spinning from west to east, but is not spinning from north to south, or south to north... And because we are still a spherical shape, clearly a person at sea level equator and a person at polar sea level are still spinning through space at differing radius to each other.

So now we have a scenario where the  centripetal motion at equatorial bulge is still faster than that at the polar sea levels, but the altitudes from centre of earth are equal at every longitude...
As the centripetal motion is reduced at longitudes that are more southern or northern, where the 'reality model's' decreased altitude at more southern and northern longitudes would normally decrease the rate of the GR altitude time dilation at exactly the rate that the decrease in centripetal motion will increase the rate of SR motion time dilation, in this 'imagined model' there is no decrease in altitude at the more northern and more southern longitudes, so GR altitude time dilation will not cancel out the SR motion time dilation effects of the more southern and northern longitudes where the centripetal motion decreases...

So as opposed to the 'reality model' where we have a clock that will run at same rate at polar sea levels as it does at equatorial sea level, this 'imagined model' will have clocks at sea level that run progressively faster as longitudes become more southern or northern and the clocks at the poles will be running the fastest.

I'd be really interested to know the value of that data curve...
Title: Re: Is there a discrepancy with the equivalence principle?
Post by: timey on 14/04/2017 14:13:35
Because the 21.36km difference in altitude at sea level equatorial bulge relative to polar sea levels causes the GR increases in rate of time, due to the greater altitude as longitudes get closer to the equator, to be exactly cancelled out by the SR decreases in rate of time due to the increases in centripetal acceleration caused by the greater radial distance, the increases in distance from the centre of the Earth that the equatorial bulge causes and the increases in centripetal motion that the increase in radial distance of the bulge causes are special...
(This being no surprise wot-so-ever because the bulge is indeed caused by the centripetal motion.)
... It is possible to obtain the data curve of how the differences in altitude are affecting the rate of time in relation to a data curve of how the differences in centripetal speed are affecting the rate of time.

Therefore (as per my 'imagined model of the post above) by ironing out the equatorial bulge so that the distance to centre of earth is equal at sea level of equator and poles, where we can now dispense with GR altitude considerations, and (given that we calibrated the caesium atomic clock at sea level equator) calculate how the rate of time is increasing at each more southern or northern longitude from the equator purely due to the decreases in radial distance at the progressively more southern and northern longitudes of this (now) perfect sphere, where the resulting decreases in centripetal motion at these progressively more southern and northern longitudes are now increasing the SR motion related rate of time, this 'imagined model' is creating a means of examining the 'reality model's' data further.

By comparing the data from the 'imagined model' with the 'reality model', the differences in radial distance at each longitude, compared to the differences in centripetal speed at each longitude, compared to the differences in rate of time at each longitude should be illuminative of the relationship between distance and speed.

While I am of course aware of the ramifications of the Michelson Morley experiment, what I am interested in within this comparison of data curves between the 'reality model' and 'imagined model', is where in these SR time dilation considerations is this notion of a variable measuring stick significant?
Title: Re: Is there a discrepancy with the equivalence principle?
Post by: timey on 15/04/2017 12:05:32
I'm suggesting that motions in the g-field are accelerated or decelerated by a time dilation that is occurring in the g-field.
That this 'additional' time dilation of the g-field will not affect any clock because a clock is mass where the clock will gain or lose potential energy in the g-field, or in relative motion to the g-field and this gaining or losing of potential energy is what is causing the observed GR (altitude) and SR (relative motion) time dilations.

In accordance with the viewpoint that GR time dilation and SR time dilation may be considered as 2 parts of a singular time dilation phenomenon, this additional time dilation can be thought of as a 3rd part of this singular phenomenon in that it just states that pe=mgh and when m=0, no potential energy is added or subtracted, where the time dilation for m=0 can be calculated via the gravitational field strength equation (where we say strength is energy), where g(r)= GmE/r^2.


Chris - I watched a Horizon program on Dark Energy last year where physicists were saying in light of Dark Energy remaining a complete mystery, that perhaps a new approach is required...
Among those physicists was George Efstathiou from Cambridge University.
Do you know him?
Title: Re: Is there a discrepancy with the equivalence principle?
Post by: timey on 15/04/2017 14:17:48
Quote
: Einstein
However we select from nature a complex (of phenomena) using the criterion of simplicity, in no case will it's theoretical treatment turn out to be forever appropriate... But I do not doubt that the day will come when that description (the general theory of relativity), too, will have to yield to another one, for reasons which at present we do not yet surmise.  I believe that this process of deepening the theory has no limits.

'New Theories of Everything', page 109 'forces and particles' by John D Barrow
Quote
Does it not seem more probable that there exist additional forces of nature that are intrinsically very weak, or highly selective in the things they act upon, or which have a minute range?  Such forces may well exist.  They do not play any great role in the structure of the everyday world, or even the world of the present day high energy physicist, but their presence totally determines the form of the ultimate Theory of Everything that we seek.

***or highly selective in the things they act upon***

By considering, where m=0, a 3rd aspect to the time dilation phenomenon that gives physical cause for the observed accelerative/decelerative force of the g-field, the ramifications of doing so give rise to an alternative theory of time dilation being energy related and may provide the unifying theory of everything that will unite the point particle model with the wave function model unifying the standard model with gravity... and do so without detracting from
the premiss of Einstein's mathematics of general relativity, only requiring that one disregard the view that time runs faster at altitude in 'open space' and re-attribute this viewpoint to time running faster for m in relation to proximity to M via potential energy, where pe=mgh and gravitational field energy is g(r)=GmE/r^2...
... m will be subject to + or - potential energy due to position in, and relative motion to, the gravity field, increasing or decreasing the rate of time for m, and where m=0 we say that g(r) is increasing or decreasing the rate of time in the g-field causing the motions of m in relation to M to be accelerated or decelerated towards or away from M.

This means that Einstein's laws of gravity can be described within a Newtonian geometry, multiple gravity fields can be summed up as per Newtonian gravity, and a physical causation for the wave function has been established to unite the point particle model with the wave function model that will describe the quantum model as a continuum to be united with gravity.

This remit will describe all observations as per experimental evidence, but describes a contracting universe as per Einstein's equations of GR, where the difference between my contracting model and the conventional expanding model is that time runs faster for bigger M's than it does for smaller M's, there is now no mathematical requirement for the 'far away clock' in the impossible 0 gravity field running infinitely fast, and consequently the mathematics of Einstein's GR will not break down to infinities in black holes.

Edit: This contracting model does not require Dark Energy or Dark Matter to balance the books and therefore is worthy of consideration.
Title: Re: Is there a discrepancy with the equivalence principle?
Post by: alancalverd on 17/04/2017 08:57:12
there is now no mathematical requirement for the 'far away clock' in the impossible 0 gravity field running infinitely fast,

A multiple straw man!

Nothing impossible about a zero gravity field. It is the point at which a clock observed by someone else appears to run at its maximum (not infinite) rate.

Nor is there an infinity in a black hole. In order for the gravitational field in a black hole to be infinite, it must contain the entire mass of the universe in zero volume. If it were not so, we could increase its field by adding mass or squeezing it, so it wasn't infinite to begin with. As far as we know, several black holes exist, so none of them has any infinite properties.
Title: Re: Is there a discrepancy with the equivalence principle?
Post by: timey on 17/04/2017 13:41:53
It was my plan to remove all of my 1600+posts from this forum but apparently that is not aloud.
Please de-register my account as I requested on Saturday.
Title: Re: Is there a discrepancy with the equivalence principle?
Post by: Demolitiondaley on 17/04/2017 23:25:54
It was my plan to remove all of my 1600+posts from this forum but apparently that is not aloud.
Please de-register my account as I requested on Saturday.


What's up Timey, who's upset you?
Title: Re: Is there a discrepancy with the equivalence principle?
Post by: timey on 18/04/2017 04:54:27
there is now no mathematical requirement for the 'far away clock' in the impossible 0 gravity field running infinitely fast,

A multiple straw man!

Nothing impossible about a zero gravity field. It is the point at which a clock observed by someone else appears to run at its maximum (not infinite) rate.

Nor is there an infinity in a black hole. In order for the gravitational field in a black hole to be infinite, it must contain the entire mass of the universe in zero volume. If it were not so, we could increase its field by adding mass or squeezing it, so it wasn't infinite to begin with. As far as we know, several black holes exist, so none of them has any infinite properties.

Quote
: Nelson Mandela (almost)
Our deepest fear is not that we are inadequate.
Our deepest fear is that we are powerful beyond measure.
We ask ourselves, who am I to be brilliant, gorgeous, talented and fabulous?
Actually, who are you not to be?
Your playing small doesn't serve the world.
We were born to make manifest the glory that is within us.
And as we let our own light shine, we unconsciously give other people the permission to do the same.

Now then young Alan, I have had just about as much as I can take of your obtusity, be that purposeful or otherwise...

Physics does not know how fast the 'far away clock' is running, it has never been observed, nor would the gravity field be zero if it had a clock in it.
In any case there is no such thing as a zero gravity field.

http://www.yalescientific.org/2010/10/mythbusters-does-zero-gravity-exist-in-space/

Furthermore, I did not say that a gravity field could be infinite...
What I said was that the maths for conventional GR break down to infinities in a black hole, and that under the remit of my proposed alterations to GR the maths will not break down to infinities in a black hole.

As to my deviations from conventional terminologies that you as a constant bemoan so vociferously...
Quote
: Jessica Rabbit
I'm not bad, I'm just drawn that way.
...please note my profile info.

But perhaps it might be of significance that:
Quote
: J R Tolkien
Not all those who wander are lost.

I am proposing a model of the universe that is compatible with GR mathematics, that doesn't require Dark Energy or Dark Matter.
This model 'should' unify the standard model with gravity while providing fully described mechanics for a rendition of Big Bang and Inflation theory.

In short this model has all the criteria for a 'Theory of Everything' inclusive of simplicity and a suggested experiment to prove or disprove itself.

All that is required is for someone of more capability than I to make the calculations to check and see if this model that I am visualising as a geometrical architecture of cause and effect mechanics is actually mathematically viable.*

I don't see this, or my unconventional terminology for that matter, as being a problem for someone of your intelligence, so to be honest I am finding your responses out of keeping with my expectations of you.

*If the model is a mathematical viability, this does not mean that it is correct, it would only mean that this model is viable.

To be proven correct would fall to the outcome of my model's suggested experiment which, when NIST get their portable atomic clocks up and running as planned in next couple of years, it will be possible to conduct...
Only then will this model I suggest be actually proven or disproven... but in the mean time, for someone like yourself who is trained in mathematics, I really don't see a problem with making the calculations that I suggest.

So rather than treating me as though I am the forum's clown why not just do the calculations?
What have you got to lose?
Title: Re: Is there a discrepancy with the equivalence principle?
Post by: alancalverd on 18/04/2017 17:22:55
I have explained previously how it is entirely possible to have a zero gravity field, but just for you, here it is again.

Every massive body produces a gravitational field. The field vector is directed towards the center of mass of the body and its magnitude at any distance from the body is given by the usual 1/r2 expression. The net field vector at any point in space is the vector sum of all vectors at that point, So there will be a finite number of points in any universe where the net field vector is zero, and in an infinite universe, there will be an infinite number of such points.

The mass of the source clock is irrelevant. The search for a zero-field point is merely the search for the point at which the frequency of that clock, as observed from any other point, is at its maximum.

The problem with deviating from conventional terminology is exactly the same problem as deviating from the local language. It doesn't matter how often or how loudly you shout in English, you won't get fish and chips, but the merest whisper of "poisson et pommes frites" will have monsieur le chef at your service. He's not being cussid, just doing his job, which is not to read your mind but to feed Francophones - or to discuss physics, as the case may be.

A reference to the post where you describe your critical experiment would be much appreciated.
Title: Re: Is there a discrepancy with the equivalence principle?
Post by: timey on 18/04/2017 20:00:30
Look - I hate to be pernickety Alan, especially in light of my own shoddy use of terminology, but please allow me to explain, again, that there is no such thing as a zero gravity field.
Quote
: Yalescientific.org
Although gravity never reaches zero, it gets close.

What you are referring to (I think) is zero point field.

https://en.m.wikipedia.org/wiki/Zero-point_energy

Quote
A vacuum can be viewed not as empty space but as the combination of all zero-point fields. In QFT this combination of fields is called the vacuum state, its associated zero-point energy is called the vacuum energy and the average expectation value of zero-point energy is called the vacuum expectation value (VEV) also called its condensate. The term zero-point field(ZPF) is sometimes used when referring to a specific vacuum field. The QED vacuum is a part of the vacuum state which specifically deals with quantum electrodynamics (e.g. electromagnetic interactions between photons, electrons and the vacuum) and the QCD vacuumdeals with quantum chromodynamics(e.g. color charge interactions between quarks, gluons and the vacuum). Recent experiments advocate the idea that particles themselves can be thought of as excited states of the underlying quantum vacuum, and that all properties of matter are merely vacuum fluctuations arising from interactions of the zero-point field.[2]

However:

Quote
The idea that "empty" space can have an intrinsic energy associated to it, and that there is no such thing as a "true vacuum" is seemingly unintuitive. For many practical calculations (especially in QED) zero-point energy is dismissed by fiat in the mathematical model as a constant that may be canceled or as a term that has no physical effect. Such treatment causes problems however, as in Einstein's theory of general relativity the absolute energy value of space is not arbitrary and gives rise to the cosmological constant. Furthermore, many physical effects attributed to zero-point energy have been experimentally verified, such as spontaneous emission, Casimir force, Lamb shift, magnetic moment of the electron and Delbrück scattering,[3][4]these effects are usually called "radiative corrections".[5] In more complex nonlinear theories (e.g. QCD) zero-point energy can give rise to a variety of complex phenomena such as multiple stable states, symmetry breaking, chaos and emergence.

Physics currently lacks a full theoretical model for understanding zero-point energy, in particular the discrepancy between theorized and observed vacuum energy is a source of major contention.[6][7] Physicists John Wheelerand Richard Feynman calculated the zero-point radiation of the QED vacuum to be an order of magnitude greater than nuclear energy, with one teacup containing enough to boil all the world's oceans[8] while experimental evidence from both the expansion of the universe and the Casimir effect show any such force to be exceptionally weak. This discrepancy is known as the cosmological constant problem (or vacuum catastrophe) and is one of the greatest unsolved mysteries in physics.

Many physicists believe that "the vacuum holds the key to a full understanding of nature" [9] and that studying it is critical in the search for the theory of everything. Active areas of research include the effects of virtual particles,[10] quantum entanglement,[11]the difference (if any) between inertial and gravitational mass,[12][13] variation in the speed of light,[14][15] a reason for the observed value of the cosmological constant[16] and the nature of dark energy.[17][18]

The concept of zero-point energy was developed by Max Planck in Germany in 1911 as a corrective term added to a zero-grounded formula developed in his original quantum theory in 1900.


My model's treatment of the blackbody data in relation to the ultra violet catastrophe that led to Planck's introduction of the quanta (as we have discussed previously), recalculates E=hf by holding the frequency as constant in the face of variable length seconds...  By shortening the time period of a second in order to keep the frequency constant, the quantum nature of the energy levels becomes a continuum that can be united with gravity.

This unification with gravity is possible because the wave function has been given a physical cause, (i.e. that frequency is time dilation related via energy), uniting the wave function model with the point particle model, where the length of a wave is a function of time, the frequency is the timing, and the energy is causing the rate of the time.

Now we can go back to the 'far away clock'.

Quote
: Alan

The search for a zero-field point is merely the search for the point at which the frequency of that clock, as observed from any other point, is at its maximum.

... And the rate that a person with that clock will age is also at its maximum.  But what is that maximum and why?

Quote
The mass of the source clock is irrelevant.

No - the mass 'value' of the clock is irrelevant, but the fact of the mass itself is highly relevant because if there were no mass present at that location in space, then the pe=mgh would just be pe=gh, or more conventionally, the gravitational field strength (vacuum energy) would be g(r)=GM/r^2, (or whatever the equivalent equation for multiple fields is)

Now we go back to my model's treatment of the ultra violet catastrophe where +energy=shorter seconds, and we attribute the gravitational field strength, or vacuum energy, as the causation of a 3rd aspect of the time dilation phenomenon where the difference between the rate of time for a clock at h from M and the rate of time that is occurring for the location of h from M is simply the difference between pe=mgh*, and vacuum energy=g(r).

*(where a pe/mvalue equation distributes the pe equally over all particle constituents of m)

As the clock is placed further away from M it's rate of time will increase...  But at each further radius from M the rate of the field time will decrease.  This gives the accelerative and decelerative force of the g-field a physical causation.  Any body of m moving into the gravity field of >>>M will be observed as accelerated by the shorter seconds, and visa versus <<<M.

So let's look at what you were saying about the 'far away clock' running at maximum...  Where the clock is in the zero point field, it's own gravitational mass energy will outweigh the surrounding field energy.  When this happens the clock is no longer an m, it's an M...

As a thought experiment - If one were to then take a test particle m from the clock and position it at h from M(clock) in the gravitational field of the clock then the tiny little clock of the test particle will run faster than the rate of time running for M(clock).  And one may then repeat the procedure for test particle m when test particle m's own gravitational mass outweighs g(r)=GMclock/r^2 value and test particle m becomes M(test particle) where a tinsy tiny test particle m from M(test particle) will have a faster rate of time at h from M(test particle), etc, etc...

This means that bigger M's, i.e. of greater mass value, are running at faster rates of time than smaller M's.

(There are a multitude of posts over the last 2 years where I have given description of my model's suggested experiment, but it is easier just to write it out again than searching for these posts)

My model's suggested experiment to prove or disprove itself incorporates placing 2 identical atomic clocks at differing locations of the same gravity potential, i.e. altitude and longitude, but of know significant difference in geological density.

My model predicts that the clock in the denser location will run faster.
Title: Re: Is there a discrepancy with the equivalence principle?
Post by: nilak on 18/04/2017 20:49:18
Quote
My model predicts that the clock in the denser location will run faster.
The clock rate vary between different gravitational potential. If there is a difference in the geological density it is hard to find points of the same gravity potential.
If you choose two points A and B, in different locations, if an object than moves between them gains velocity then there is a difference in the gravitational potential. I think this velocity gain is responsible for the clock rate increase.
For a clock in free fall from A to B it's atoms will gain and keep energy even if at B you hold the clock from falling. That energy is equivalent to electrons gaining more velocity.


I think you need to describe the complete scenario that includes the shape of the maasive object that generates gravity.
Title: Re: Is there a discrepancy with the equivalence principle?
Post by: timey on 18/04/2017 21:39:58
To be clear Nilak, this experiment that I suggest is an Earthbound experiment.

At sea level Earth a stationary atomic clock will run at the same rate at all longitudes from polar to equator. 
The increased rate of time due to altitude of the equatorial bulge is exactly canceled out by the decreased rate of time due to the centripetal motion of the greater radius at equator relative to the poles.

My model's experiment suggests placing 2 identical atomic clocks at differing locations that are at the same altitude to equalise gravity potential, and at the same longitude if possible to ensure equal centripetal motion - (although sea level locations would ensure a natural equalisation of the rate of time of both) - but at locations of known inequality of geological density.

Neither of the clocks will be in motion other than the afore mentioned centripetal motion, and both should be placed upon the ground.

My model predicts that the clock in the denser location will run faster.
Title: Re: Is there a discrepancy with the equivalence principle?
Post by: nilak on 18/04/2017 21:54:15
In that case if you have a hollow massive sphere in deep space, an object inside it will not experience any gravity force. Do you think a clock closer to the surface will tick faster? I don't see any reason to do that.
Title: Re: Is there a discrepancy with the equivalence principle?
Post by: timey on 18/04/2017 22:12:39
Your reply illuminates the fact that you haven't quite grasped the premiss set out in post 560.  But hey, don't worry about it Nilak... go back, read it again, wait for someone else to respond perhaps.
Title: Re: Is there a discrepancy with the equivalence principle?
Post by: alancalverd on 18/04/2017 23:15:16
.........
My model predicts that the clock in the denser location will run faster.

In other words, that a clock at a lower gravitational potential will run faster than one at a higher gravitational potential. Which is exactly the opposite of what we observe, whether it is an atomic clock in a GPS satellite, the redshift of the solar spectrum, the Pound-Rebka experiment, or even the most recent work with aluminum atom clocks separated by a few meters of height.   But the important point is that you have identified the critical experiment. The question is whether it is feasible or whether we can find a feasible substitute.

Rather than look for places where the earth's radius is identical but g is significantly different, you can separate the clocks vertically at a single location and calculate the differential due to circular motion at different heights thus:

the gravitational field at a distance x from the center of gravity of a large body  is GM/x2 and the centrifugal field is v2/x where v is the tangential velocity.

Now x is around 6.4 x 106 meters at the surface and the tangential velocity at the equator (for simplicity) is 2 x pi x 6.4 x 106 /24 x 3600 = 465 meters/second.

I'll leave it to you to calculate the gravitational field and centrifugal field at, say x = 6.400001 x 106 meters (for the Al clock experiment) or 6.400023 x 106 for the PR experiment. I think you will find that the centrifugal effect is negligible in comparison with the calculated and observed frequency shift, rather as the original experimenters assumed (or calculated). The reason, of course, is that the centrifugal field varies with x whilst the gravitational field varies with x2, but you can apply the centrifugal correction anyway if you wish.

There's no easier experiment than one that has already been done!
Title: Re: Is there a discrepancy with the equivalence principle?
Post by: timey on 19/04/2017 01:03:12
I guess you didn't read post 562 which would have informed you that considerations of altitude and centripetal motion are an intrinsic consideration of the proposed experiment.

It is also clear that you did not fully understand the ramifications of the M(clock) and test particle thought experiment of post 560.

What you have understood is that what I am suggesting is the opposite of that which conventional physics would predict, but you are understanding it for the wrong reasons.
Conventional physics would state that in an experiment where 2 clocks are placed at the same altitude, and same centripetal motion as each other, whereas both clock's will run at same rate, if the locations were of differing geological density, the clock in the denser location would run slower.  This being because conventional physics believes that because a clock runs faster at altitude from M, and a clock runs slower closer to M, that time runs slower for bigger bodies of mass.

My model, without detracting from the observed and experimentally verified fact that a clock runs faster at altitude from M, and that a clock runs slower closer to M, states that a bigger value M will have a faster rate of time than a smaller value M, where all m at h from M will run faster than M.
As per the premiss set out in post 560 my model is stating that while an m(clock) will run faster in relation to h from M, and slower at decreased h from M, that this is potential energy related as per pe=mgh, where +energy= shorter seconds, but when the value of m's own measure of g outweighs the g at h from M, (such as in zero point field) that m(clock) becomes M(clock) where any test particle in relation to h from M(clock) will run at a faster rate of time than M(clock).

If you have a real good long deep think about this Alan, (and please do that, because you have said that you think me of at least some intelligence, and I really wouldn't be interested in just dicking around trying to tell you an idea I have about physics in favour of just having a laugh dicking around on the 'on the even lighter side' thread 'which was much more fun' if I didn't think I had something of importance to relate to you), you will realise that I am not transgressing any observation or experimentally verified phenomenon in my model's prediction that the clock in the denser location will run faster.

The difference 'at this juncture' between my model and the conventional model is that my model states that where m=0 that potential energy is not added or subtracted, and that where the g-field of M is of lesser energy at h's from M, that the rate of time is -energy=longer seconds.
The reason that this possibility has been overlooked by conventional physics (apart from Hubble's contribution) is because where m=0 a clock cannot measure.
However, because light is m=0, where light shifts in the gravitational field and the wave length gets longer <<<M, or shorter >>>M, that the change in the length of the wavelength is g-field '3rd aspect time dilation' related, and where the acceleration >>>M or deceleration <<<M of m's motions in relation to M are observed, this is also g-field '3rd aspect time dilation' related, and this is the means of observing the '3rd aspect time dilation' phenomenon that I am proposing, where the 3rd aspect time dilation is a part and parcel of the time dilation phenomenon entire of which SR and GR time dilation are the other 2 aspects.
Title: Re: Is there a discrepancy with the equivalence principle?
Post by: nilak on 19/04/2017 05:16:44
Quote
"No - the mass 'value' of the clock is irrelevant, but the fact of the mass itself is highly relevant because if there were no mass present at that location in space, then the pe=mgh would just be pe=gh, or more conventionally, the gravitational field strength (vacuum energy) would be g(r)=GM/r^2, (or whatever the equivalent equation for multiple fields is)"
I didn't understand this part.
Why if no mass present the pe=gh ? It looks as if m=1kg.
Title: Re: Is there a discrepancy with the equivalence principle?
Post by: alancalverd on 19/04/2017 07:29:41
But the clock doesn't sense M, only the gravitational field associated with it, so it makes no difference whether you increase M or reduce x. Unless you are proposing an entirely new force that looks like gravity, works backwards, only affects clocks, and has never been detected previously.That's pretty radical, especially as the red shift of the solar spectrum (large M compared with Mearth) is exactly what you would calculate without it.

As for the addition and subtraction of potential energy in the absence of rest mass, I think you will find this well covered by the explanation of gravitational lensing. It's also interesting that the PR result is exactly what you expect from a calculation of exchange between potential and kinetic energy for a photon.     

On a matter of linguistics, "conventional physics" doesn't "believe" anything. We make hypotheses and test them. Fact is that clocks run slower at a lower gravitational potential. The job of theoretical physics is to explain and extrapolate observation, not to dismiss it as belief. Now we have an observation and a theory which so far has predicted the next result, and you want to replace it with a theory that predicts exactly the opposite.  Difficult starting point, my friend! As Patrick said to Murphy: "if I was going to Cork I wouldn't start from Dublin" but if you really have to, it's not advisable to begin by assuming Cork to be east of Dublin.
Title: Re: Is there a discrepancy with the equivalence principle?
Post by: nilak on 19/04/2017 10:20:55
What do you mean by centripetal motion? I think that would mean a motion from a height h to a lower height on a straight line towards the center of rotation ( as viewed from outside the earth).
 I suppose you meant the circular motion around the earth center of mass.
Title: Re: Is there a discrepancy with the equivalence principle?
Post by: yor_on on 19/04/2017 10:24:02
That's an interesting definition Alan. Places where gravity is non existent. Would that mean that gravity negates itself? When I think of 'Black Holes' I see that definition of a 'singularity' as being of a 'infinite magnitude', actually I think that is what defines it. Mathematics use this idea of larger and smaller infinities, and using a argument of nothing being 'infinite', assuming one can continue to add to it, may become questionable from that point of view. I don't really like 'different magnitudes' of infinity's myself, have a problem imagining what it should mean, but I think you might agree to that 'filling up' a infinity shouldn't 'change it', presuming it already is infinite?
=

What I'm thinking is not about what one measure outside some event horizon, just that if a infinity ever got 'filled up' it couldn't have been a 'infinity' to start with.
Title: Re: Is there a discrepancy with the equivalence principle?
Post by: nilak on 19/04/2017 10:30:10
Inside a hollow sphere (perfect and alone in the universe) I think we can say gravity is nonexistent. The spacetime inside it is just flat Minkowski spacetime.
Title: Re: Is there a discrepancy with the equivalence principle?
Post by: yor_on on 19/04/2017 10:35:48
Well, yes, and that Is how I get Alan's argument. I think him to be correct. Although I will differ presumably on the definition of what can be called 'infinite'. That is if you're thinking of a perfect 'sphere' of a even density and referring to it's 'center'
==

Another interesting question might be, should you be able to measure a rotation in your universe?
Title: Re: Is there a discrepancy with the equivalence principle?
Post by: nilak on 19/04/2017 11:10:40
Quote
Another interesting question might be, should you be able to measure a rotation in your universe?
I think we are getting off topic with this question. You should be able to measure a centrifugal force. I remember asking this question but I don't remember the answer. You can view the sphere as made of individual atoms that move an a circular path so the relative motion still exists. The case simplifies to two balls rotating connected by a rod.
Title: Re: Is there a discrepancy with the equivalence principle?
Post by: jeffreyH on 19/04/2017 12:36:06
The gravity inside a hollow sphere only cancels completely  at the centre of mass. If the thickness of your shell equalled the diameter of Jupiter then the closer to an interior surface the greater the force.
Title: Re: Is there a discrepancy with the equivalence principle?
Post by: timey on 19/04/2017 13:26:22
Quote
But the clock doesn't sense M, only the gravitational field associated with it, so it makes no difference whether you increase M or reduce x.
What on earth are you on about?
A clock is observed to undergo a change in frequency of electron transitions due to changes in the gravitational field.  The gravitational field is due to M.
Change the value of M and the clock will be experiencing a changed gravitational field.

Quote
Unless you are proposing an entirely new force that looks like gravity, works backwards, only affects clocks, and has never been detected previously.

What I am proposing is that there is a 3rd aspect to the time dilation phenomenon that does not affect mass.  It has never been detected before because it doesn't affect mass.  Yes is works 'backwards' or 'contra directional' in the g-field to GR altitude time dilation, and what this means is that time runs at a slower rate in 'open space', where 'open space' is defined as the parts of the universe where there is no mass.

Quote
.That's pretty radical, especially as the red shift of the solar spectrum (large M compared with Mearth) is exactly what you would calculate without it.

Yes - exactly!  And a time related wavelength will give exactly the same mathematical results as a distance or velocity related wavelength which is why my model 'can' be mathematically viable via the time distance speed formula.

Quote
As for the addition and subtraction of potential energy in the absence of rest mass, I think you will find this well covered by the explanation of gravitational lensing.

Where the description of the force of gravity that bends the light is lacking a physical causality.

Quote
It's also interesting that the PR result is exactly what you expect from a calculation of exchange between potential and kinetic energy for a photon.

Yes - that is most interesting indeed, because clearly there is something in the gravitational field that is causing the light to be shifted, and this 'something' in the gravitational field is lacking a description of physical causation.

Quote
On a matter of linguistics, "conventional physics" doesn't "believe" anything. We make hypotheses and test them. Fact is that clocks run slower at a lower gravitational potential.

Wholeheartedly agreed!  My model states the exact same thing...
But do clocks run slower in the greater gravitational field?
Is there a difference between gravity potential energy and gravitational field energy?
Potential energy is added or subtracted for mass at differing gravity potentials.
What occurs for open space?
Is it a 'fact' that 'time' runs faster 'in' the weaker gravity field where there is no mass?

Quote
The job of theoretical physics is to explain and extrapolate observation, not to dismiss it as belief.

So - explain and extrapolate the observation of gravitational acceleration please...

Quote
Now we have an observation and a theory which so far has predicted the next result,

What we have is an observation of gravitational acceleration and a theory that does not provide physical causation for this phenomenon.

Quote
and you want to replace it with a theory that predicts exactly the opposite.

...and this is where you are misunderstanding my theory.
My theory does not re-predict any part of GR, it holds all GR predictions as sound.
However, it is the consequence of assuming that the 'fact' of a clock running faster at altitude from M, and the 'fact' of a clock running slower nearer to M means that time is running faster in 'open space' that leads theoretical physics to the 'belief' that bigger M's are running at slower rates of time than smaller M's.

... And this is a 'belief' based on an 'assumption' because it is not physically proven to be 'fact' that a bigger M is running at a slower rate than a smaller M.
All that is proven as 'fact' is that all m in M's gravitational field will age at differing rates due to position and relative motion with respect to M's gravitational field.

The experiment that I suggest is designed to equalise position and relative motion with respect to M's gravitational field and therefore measure what rate a clock will run at in a denser location compared to a less dense location.

I've never been to Ireland but as I have friends in Cork, Cork would be a great starting point!
Title: Re: Is there a discrepancy with the equivalence principle?
Post by: timey on 19/04/2017 15:47:42
why are you discussing hollow spheres on this thread?

Please stick to the topic matter.  If you wish to discuss hollow spheres open another thread.
Title: Re: Is there a discrepancy with the equivalence principle?
Post by: jeffreyH on 19/04/2017 17:25:41
The gravity inside a hollow sphere only cancels completely  at the centre of mass. If the thickness of your shell equalled the diameter of Jupiter then the closer to an interior surface the greater the force.
https://physics.stackexchange.com/questions/43626/is-spacetime-flat-inside-a-spherical-shell

Well I beg to differ.
Title: Re: Is there a discrepancy with the equivalence principle?
Post by: timey on 20/04/2017 00:13:32
Alan - I've reposted my reply to your post incase you missed it:

But the clock doesn't sense M, only the gravitational field associated with it, so it makes no difference whether you increase M or reduce x.

What on earth are you on about?
A clock is observed to undergo a change in frequency of electron transitions due to changes in the gravitational field.  The gravitational field is due to M.
Change the value of M and the clock will be experiencing a changed gravitational field.

Unless you are proposing an entirely new force that looks like gravity, works backwards, only affects clocks, and has never been detected previously.

What I am proposing is that there is a 3rd aspect to the time dilation phenomenon that does not affect mass.  It has never been detected before because it doesn't affect mass.  Yes is works 'backwards' or 'contra directional' in the g-field to GR altitude time dilation, and what this means is that time runs at a slower rate in 'open space', where 'open space' is defined as the parts of the universe where there is no mass.

.That's pretty radical, especially as the red shift of the solar spectrum (large M compared with Mearth) is exactly what you would calculate without it.

Yes - exactly!  And a time related wavelength will give exactly the same mathematical results as a distance or velocity related wavelength which is why my model 'can' be mathematically viable via the time distance speed formula.

As for the addition and subtraction of potential energy in the absence of rest mass, I think you will find this well covered by the explanation of gravitational lensing.

Where the description of the force of gravity that bends the light is lacking a physical causality.

It's also interesting that the PR result is exactly what you expect from a calculation of exchange between potential and kinetic energy for a photon.

Yes - that is most interesting indeed, because clearly there is something in the gravitational field that is causing the light to be shifted, and this 'something' in the gravitational field is lacking a description of physical causation.

On a matter of linguistics, "conventional physics" doesn't "believe" anything. We make hypotheses and test them. Fact is that clocks run slower at a lower gravitational potential.

Wholeheartedly agreed!  My model states the exact same thing...
But do clocks run slower in the greater gravitational field?
Is there a difference between gravity potential energy and gravitational field energy?
Potential energy is added or subtracted for mass at differing gravity potentials.
What occurs for open space?
Is it a 'fact' that 'time' runs faster 'in' the weaker gravity field where there is no mass?

The job of theoretical physics is to explain and extrapolate observation, not to dismiss it as belief.

So - explain and extrapolate the observation of gravitational acceleration please...

Now we have an observation and a theory which so far has predicted the next result,

What we have is an observation of gravitational acceleration and a theory that does not provide physical causation for this phenomenon.

and you want to replace it with a theory that predicts exactly the opposite.

...and this is where you are misunderstanding my theory.
My theory does not re-predict any part of GR, it holds all GR predictions as sound.

However, it is the consequence of assuming that the 'fact' of a clock running faster at altitude from M, and the 'fact' of a clock running slower nearer to M means that time is running faster in 'open space' that leads theoretical physics to the 'belief' that bigger M's are running at slower rates of time than smaller M's.

... And this is a 'belief' based on an 'assumption' because it is not physically proven to be 'fact' that a bigger M is running at a slower rate than a smaller M.
All that is proven as 'fact' is that all m in M's gravitational field will age at differing rates due to position and relative motion with respect to M's gravitational field.

The experiment that I suggest is designed to equalise position and relative motion with respect to M's gravitational field and therefore measure what rate a clock will run at in a denser location compared to a less dense location.
Title: Re: Is there a discrepancy with the equivalence principle?
Post by: alancalverd on 20/04/2017 09:10:10
Didn't miss it, just wondered whether it was worth the effort of going over old ground again! Glutton for punishment, o here goes......

Quote
But the clock doesn't sense M, only the gravitational field associated with it, so it makes no difference whether you increase M or reduce x.
What on earth are you on about?
A clock is observed to undergo a change in frequency of electron transitions due to changes in the gravitational field.  The gravitational field is due to M.
Change the value of M and the clock will be experiencing a changed gravitational field.

But you could also change x to get the same result, so the observed frequency isn't a function of M alone but of the gravitational potential -GM/x.

Henry Cavendish established in 1797, and practically every national laboratory and astronomical observation since then has confirmed,  that G is independent of the density of the attracting body.

So the clock result depends only on M/x, so you might as well vary x at one geographical location, which is a lot easier than looking for local variations in g (the effect of density variation) and correcting for latitude (not longitude!) . Which is exactly what P&R and practically everyone else have done. 

Now if I understand your hypothesis, moving a source closer to a massive body will gradually slow the apparent frequency of the source (which is our common observation) until it reaches a point where it speeds up again (which has not been observed). So you are postulating a very short-range force that hasn't yet been measured, that stops black holes collapsing to zero diameter (because it would take for ever to do so). Is that just the weak force? Or is it remotely possible that the collapse is prevented simply by Heisenberg's indeterminacy?
Title: Re: Is there a discrepancy with the equivalence principle?
Post by: jeffreyH on 20/04/2017 12:23:16
Alan's post above is one everyone should read through carefully. Make sure you understand it all.
Title: Re: Is there a discrepancy with the equivalence principle?
Post by: GoC on 20/04/2017 13:20:39

Henry Cavendish established in 1797, and practically every national laboratory and astronomical observation since then has confirmed,  that G is independent of the density of the attracting body.

A BH actually proves that statement incorrect.

The difference between a black hole and regular mass is frequency is not possible in a black hole. The frequency produced in a spherical planet is reduced to the center of a planet if atomic clocks follow frequency. A correlation has been established. Time continues to tick slower to the center of a planet if we extend the observed tests.
Title: Re: Is there a discrepancy with the equivalence principle?
Post by: timey on 20/04/2017 13:31:22
Alan - you seem to have fallen into the very common trap of believing that you have nothing left to learn.

Henry Cavendish established in 1797, and practically every national laboratory and astronomical observation since then has confirmed,  that G is independent of the density of the attracting body.

The program "Gravity and Me: The force that shapes us" is really very informative and entirely disagrees with you on this point.

It takes you on a journey around Britain with a set of laboratory scales and a laboratory weight and quite clearly shows one by how many Newtons that laboratory weight changes due to locational position and conditions.

The physicists are very clear that when they measure the gravity on Dartmoor, that the density of the granite at that position is having an effect on the measurement!

The program takes you to the Cavendish experiment site itself, giving explanation of this experiment.
It also takes you to a laboratory where gravity is measured every day by a highly sensitive machine where gravity is observed to differ on a daily basis, and these differences can be due to the simple fact of the extent of rain fall on that day.

Gravity mapping from space clearly shows that anomalies of gravity on Earth are present due to density differences.

https://earthobservatory.nasa.gov/Features/GRACE/page3.php

Atomic clocks are used to measure how differences in gravity effect the rate of time.
However, at present these clocks are laboratory bound and have only been able to measure the differences in the rate of time due to change in altitude and relative motion.
The reason why NIST are developing laboratory sensitive portable atomic clocks is actually with the intention of measuring locations of differing density.

You are correct that G is used for astronomical measurements, but really this is an approximation because near Earth gravity measurements differ for many reasons, and one of those reasons is the geological density of the location.

The program also discussed a mobile phone relativity app where the app records one's altitude and speed of motion during the day telling one by what rate they are ageing due to position of altitude and relative motion...
If you are going to be making comment as an authority on gravity, watching this program would be of immense benefit to you.

But you could also change x to get the same result, so the observed frequency isn't a function of M alone but of the gravitational potential -GM/x.

The experiment that I suggest is to designed to determine exactly whether the observed frequency of a (stationary to the gravity field) clock is just a function of gravity potential (position of altitude from centre of Earth), or if density of M (gravitational field) will also affect the observed frequency of the clock in any particular fashion, (i.e. increase or decrease the observed frequency).
Title: Re: Is there a discrepancy with the equivalence principle?
Post by: nilak on 20/04/2017 16:18:07
Your thought experiment seems ambiguous  to me. You say the gravitational potential is the same for two points of the same position of altitude from the centre of the earth. That is an aproximation that doesn't take into account local densities. It you add them to the equation, you will find a higher gravity then higher gravitational potential in the denser location. However, the difference is so small that I don't think it you can measure any effect on clocks. But you say that in the denser region clocks will tick faster. What is the reason and what would be the magnitude of the effect?
Title: Re: Is there a discrepancy with the equivalence principle?
Post by: timey on 20/04/2017 18:45:20
Your thought experiment seems ambiguous  to me.
It is your understanding of the suggested 'actual' (not thought) experiment that is ambiguous.

You say the gravitational potential is the same for two points of the same position of altitude from the centre of the earth.
This is dependant on what you mean by gravity potential...
Gravity potential energy is pe=mgh, where pe stands for potential energy, m is a body of mass in relation to Mearth, g is the strength of the g-field at h, and h 'can' be the height from the centre of the earth.
The g-field of Mearth is not the same thing as the gravity potential of the g-field Earth.
pe=mgh is not the same thing as g(r)=GMearth/r^2.

That is an aproximation that doesn't take into account local densities. It you add them to the equation, you will find a higher gravity then higher gravitational potential in the denser location.
Yes you are correct...
GMearth/r^2 is an approximation.  It does not take into account the gravity anomalies found in locations of earth that are at the same altitude from centre of earth, but of differing geological densities.
Taking into account the gravity anomalies of Earth, one will find that the 'g' of g(r)=GMearth/r^2 is differing according to the density of the location.
...And this is exactly what my suggested experiment seeks to measure.
The experiment seeks to place 2 identical atomic clocks in differing locations of the exact same altitude from centre of earth, (to equalise GR altitude time dilation), where these locations share the exact same radius of centripetal motion, (to equalise SR motion related time dilation), but are of known significant difference in geological density.

However, the difference is so small that I don't think it you can measure any effect on clocks.
You are very wrong about this.  If a laboratory scale can detect the difference in the weight caused by the anomaly, an atomic clock will be more than able to measure the difference in rate of time this will cause.

But you say that in the denser region clocks will tick faster.
Conventional physics will tell you that the clock in the denser location will run slower.  This is why BH's are thought to be being observed as running at very slow, or stopped rates of time.
Although a clock will run faster when it is subject to greater potential energy at greater heights from the centre of gravity, my model predicts that a clock will also run faster if subject to a greater gravitational field, therefore my model predicts that in comparing the 2 clocks under the conditions of the suggested experiment, that the clock in the denser location will run faster.

What is the reason and what would be the magnitude of the effect?
The reason for this is defined by the afore suggested re-calculation of the ultraviolet catastrophe under the remit of +energy=shorter seconds and the consequence that because stars of greater temperature are greater in mass value, that time will be running faster for bigger stars than for smaller stars...

In the suggested experiment the magnitude of the effect would be dependent on the anomaly of g.
Title: Re: Is there a discrepancy with the equivalence principle?
Post by: jeffreyH on 20/04/2017 19:50:54

Henry Cavendish established in 1797, and practically every national laboratory and astronomical observation since then has confirmed,  that G is independent of the density of the attracting body.

A BH actually proves that statement incorrect.

The difference between a black hole and regular mass is frequency is not possible in a black hole. The frequency produced in a spherical planet is reduced to the center of a planet if atomic clocks follow frequency. A correlation has been established. Time continues to tick slower to the center of a planet if we extend the observed tests.

That is incorrect. The gravitational potential of a black hole at radius r is the same as for any uncompressed mass of the same value. Assuming that r is outside the surface of the uncompressed mass. Therefore your view of the situation is in error.
Title: Re: Is there a discrepancy with the equivalence principle?
Post by: nilak on 20/04/2017 20:24:11
Quote
Gravity potential energy is pe=mgh, where pe stands for potential energy, m is a body of mass in relation to Mearth, g is the strength of the g-field at h, and h 'can' be the height from the centre of the earth.
Ok, g is the strength of the field at h, but when you use 198907897acbf1f704f74f1abb1e38ef.gif, g is needs to be constant for any height, otherwise you need the g function of h and to integrate the function over h.
Also you can use infinite as a reference point for gravitational potential if the field is not constant.
Title: Re: Is there a discrepancy with the equivalence principle?
Post by: timey on 20/04/2017 21:00:22
Your maths text has come out wonky and I can't see what equation you post, but why would this be relevant to a physical experiment?

Under the remit of a change to a greater altitude the equation pe=mgh will incorporate a decrease in value of m, a decrease in value of g, and an increase in value of h, resulting (via the equation pe=mgh) in an increased value of pe.

Under the remit of the suggested experiment and a difference in gravitational field due to density, the equation pe=mgh will incorporate h being held equal for both locations, and in the denser location, an increased value of m, and an increased value of g, resulting in the clock in the denser location (via the equation pe=mgh) having a greater value of pe.

The g of each location can be compared in each location via scale and weight, and therefore the magnitude of the difference in rate of time observed of each clock can be calculated held relative to measurement.
Title: Re: Is there a discrepancy with the equivalence principle?
Post by: nilak on 20/04/2017 22:02:32
I don't know why you vary m which for calculating gravitational potential it is replaced with 1.
To calculate the gravitational potential at those two points say A and B, I think you can measure the direction and strength of the gravitational field from A and B till a certain height where the difference between them becomes small say 1000km, then extrapolate the g function till infinity. Then integrate the gravitational potential function of h, using the measured g(h).
35280cef4de33c8d708afae86200b9e5.gif
But, this is for a hypothetical lonely planet with no rotation and no translational motion.
However, you talk about a real experiment and I don't think it is possible to determine the gravitational potential with the precision you want to test your prediction. Also I think the difference in clock rates will be much smaller than the measurement standard deviation.
Title: Re: Is there a discrepancy with the equivalence principle?
Post by: alancalverd on 20/04/2017 22:47:09
Alan - you seem to have fallen into the very common trap of believing that you have nothing left to learn.

Henry Cavendish established in 1797, and practically every national laboratory and astronomical observation since then has confirmed,  that G is independent of the density of the attracting body.

The program "Gravity and Me: The force that shapes us" is really very informative and entirely disagrees with you on this point.

It takes you on a journey around Britain with a set of laboratory scales and a laboratory weight and quite clearly shows one by how many Newtons that laboratory weight changes due to locational position and conditions.

I have a lot left to learn, but I can recognise the difference between G (the universal gravitational constant) and g (the local acceleration due to gravity). I learned that at the age of 16, and I haven't met any physicists (or geologists, who use the variation in g when prospecting for oil) who didn't know it. Maskeleyne's famous 1774 estimate of the mass of the earth was based on the variation of g around Schiehallion.  If the program misled you in that respect, you should complain.

As I have pointed out many times, scientists tend to be very pernickety about language, and even about symbols, and we sometimes make the  mistake of thinking that others have as much respect for precise communication.

And by the way you can't measure g with "laboratory scales" for the very reason I mentioned. You need to use a spring balance or an electromagnetic force balance. "Scales" compare the gravitational force gm1: gm2 on two lumps of matter, so g cancels out. Terminology matters.   

Title: Re: Is there a discrepancy with the equivalence principle?
Post by: alancalverd on 20/04/2017 23:00:41
Apropos Dartmoor, or more accurately Bodmin Moor, the engineers working on the A30 improvement scheme umpteen years ago asked for 1000 geological survey points along the proposed route before they would complete the design and fix a price, The first three boreholes hit granite at about 2 m depth so the accountants cancelled the geology contract and insisted on a quote for a design based on granite at 2 m. It turned out that the geologists had hit the only granite boulders in 30 miles of peat bog. Unsurprisingly, the project ran somewhat over time and budget.     
Title: Re: Is there a discrepancy with the equivalence principle?
Post by: timey on 20/04/2017 23:13:16
As always your diction is entertaining and interesting, but to say so it has nothing to do with placing 2 identical clocks at locations of differing geological density, but of equal height from centre of earth and equal radius of centripetal motion.

The part of the program where they measured the precision laboratory weight with the laboratory sensitive scales revealed that the mass of this weight registered as greater or lesser on the scales as per Newtons under differing gravity potential conditions.

Edit: When they measured Dartmoor because it is closer to the equator, the geological density there compared to a previous measurement on Snowdonia was mentioned as a factor.
Title: Re: Is there a discrepancy with the equivalence principle?
Post by: alancalverd on 21/04/2017 00:15:22
Not scales. Never. Scales don't "register newtons": they compare the gravitational force on two masses. Either the program was lying or you have incorrectly renamed a force balance. If the presenter said "scales" he should be taken outside and re-educated. And the precision of the test mass is irrelevant - any lump of nonmagnetic material would do.

But the point is that time depends on the local gravitational field, not the nature of the source of that field. All you have quoted is a demonstration that field depends on mass and distance, and that mass depends on density and volume, so dense rocks produce a stronger field than less dense rocks at a given distance. Nothing new there - it was known to Cavendish, which is why he used gold and lead in his experiments.

Title: Re: Is there a discrepancy with the equivalence principle?
Post by: timey on 21/04/2017 00:30:49
Ok - let me re-phrase.

A lump of metal of a particular mass was placed upon apparatus.
This was used to demonstrate that the lump of metal registered different values when it was placed on the apparatus at the base of Snowdonia, at the top of Snowdonia, and on Dartmoor closer to the equator.
When they measured Dartmoor, it was remarked upon that because the geological density of Dartmoor is lesser than Snowdonia, that this would be a factor as to the value registered when the lump of metal was placed on the apparatus.

Sticking to conventional physics for this post, if clocks were placed at locations of differing geological density but at the same height from centre of earth, and at same radius of centripetal motion, then as per the remit of conventional physics, the clock in the greater gravity field should run slower.

Under the remit of conventional physics a clock in the greater gravity field will be observed to run slower.
Title: Re: Is there a discrepancy with the equivalence principle?
Post by: alancalverd on 21/04/2017 07:41:44
......as predicted and proved by experiment, many times!

We know

gy  = gmax - w2Rcos2y

where gmax is g at the poles, w is the angular velocity of the earth, R the radius of the earth, and y is the latitude.

It is surprising that a shift of only 2 degrees of latitude was distinguishable from the subterranean density effect, but the above equation is sufficiently accurate over small variations in y that it can indeed be used for locating geological anomalies like ore and oil deposits.

Fortunately, national laboratories that maintain time standards are aware of the local value of g and hence can account for the difference between, say, WWV time signals (from Colorado) and MSF (from Cumbria).

Now it seems that your hypothesis is that two clocks operating in the same value of g would run at different rates if their local geology was different. Very spooky. What mechanism do you think tells the clock whether it is sitting over sand, or over granite but under a lead roof, if g is the same in both cases?   
Title: Re: Is there a discrepancy with the equivalence principle?
Post by: timey on 21/04/2017 13:14:41
Under the remit of conventional physics a clock in the greater gravity field will be observed to run slower.

......as predicted and proved by experiment, many times!

We know

gy  = gmax - w2Rcos2y

where gmax is g at the poles, w is the angular velocity of the earth, R the radius of the earth, and y is the latitude.

It is surprising that a shift of only 2 degrees of latitude was distinguishable from the subterranean density effect, but the above equation is sufficiently accurate over small variations in y that it can indeed be used for locating geological anomalies like ore and oil deposits.

Fortunately, national laboratories that maintain time standards are aware of the local value of g and hence can account for the difference between, say, WWV time signals (from Colorado) and MSF (from Cumbria).

Now it seems that your hypothesis is that two clocks operating in the same value of g would run at different rates if their local geology was different. Very spooky. What mechanism do you think tells the clock whether it is sitting over sand, or over granite but under a lead roof, if g is the same in both cases?   

if clocks were placed at locations of differing geological density but at the same height from centre of earth, and at same radius of centripetal motion,

Then g would 'not' be the same for each clock.

A clock will be observed to run faster at a greater altitude - where as per the equation pe=mgh, the difference between the lower clock and the higher clock is that for the higher clock m has decreased in value, g has decreased in value and h has increased in value resulting in an increase in potential energy for the higher clock.

For the 2 clocks of the suggested experiment - placed at locations of equal radius of centripetal motion, (to equalise SR effects) and at equal altitude, but at locations of differing geological density *and therefore at locations of differing g* - the clock in the denser location (compared to the clock in the less dense location) will have an increased m, and an increased g, where h remains the same for both clocks.

Will the clock that is subject to the greater g have a higher value of pe than the clock that is subject to the lesser g?
Title: Re: Is there a discrepancy with the equivalence principle?
Post by: alancalverd on 21/04/2017 17:55:38
For the 2 clocks of the suggested experiment - placed at locations of equal radius of centripetal motion, (to equalise SR effects) and at equal altitude, but at locations of differing geological density *and therefore at locations of differing g* - the clock in the denser location (compared to the clock in the less dense location) will have an increased m,



NO NO NO NO NO!

The whole point of the television experiment you quoted, and the entire lives of Galileo, Cavendish, Maskeleyne, Hoyt, Newton, Eotovos, Kater and umpteen others was that the mass of a body does not change with location. Mass and weight are different, which is why we use kilograms for one and newtons for the other. . Scales compare masses;  gravimeters, spring balances and suchlike measure weight. Weight = mg.

This is really the most elementary stuff. If you don't understand it, you are wasting your life discussing relativisitic quantum mechanics because you won't understand that either.

All you have said so far is that a clock in a stronger gravitational field will appear to run slow. Everybody agrees with that, because it has been demonstrated.
Title: Re: Is there a discrepancy with the equivalence principle?
Post by: timey on 21/04/2017 18:58:09
Excuse my lacking in knowledge of elementary mathematics.

So m stays the same in the equation which really does simplify the matter so...

The clock that is raised in altitude compared to the lower clock will be subject to a decrease in g and an increase in h, where the equation pe=mgh results in a greater value of pe for the higher clock.

On the basis that the above statement is true, is it?

2 identical clocks placed in locations of the same radius of centripetal motion (to equalise SR effects), and at the same altitude from centre of earth so that h is equal for both, but placed in locations of significant differing geological density - the clock in the denser location compared to the clock in the less dense location will have an increased g, where the equation pe=mgh will result in a greater value of pe for the clock in the denser location.

The clock in the greater gravity field will have more potential energy than the clock in the lesser gravity field.

Agreed?
Title: Re: Is there a discrepancy with the equivalence principle?
Post by: nilak on 21/04/2017 22:06:09
The gravitational acceleration g can vary across the globe [1,2] within approx. 120mGal which is e1650de16c1eca0fa398511d351c17d5.gif.
I understand that the chart represents the deviation from standard smooth value. For example at the equator it would be a deviation from 9.78 m/s^2
Therefore we can have at one place a1=9.80006 and at another 9.79994.
If we want to see the time dilation due to gravity at a1 and a2 we can use
809ccf61b271d8b854d7a4cbffc48595.gif
If we make the notation k as a dilation factor to mean T=T0 • k
For a1 we get k1=1.000 000 000 694 715
and for a2, k2=1.000 000 000 694 707
That is 8 seconds difference in 4ef44bbdb2a6214f5cd1aab50733b060.gif
That mean we need to be able to measure a time rate difference of 1 second over approx. 4 milion years.
Atomic clocks lose only 1 sec over 15 billion years, therefore I think you are right, we can measure the necessary time rate difference between the two locations.

https://astarmathsandphysics.com/a-level-physics-notes/173-forces-and-motion/2811-variation-of-the-acceleration-due-to-gravity-over-the-surface-of-the-earth.html [1]

https://en.m.wikipedia.org/wiki/Gravity_of_Earth [2]
Title: Re: Is there a discrepancy with the equivalence principle?
Post by: timey on 21/04/2017 22:52:20
Nilak - the difference between what is observed of a clock in the g-field concerning a comparison of an elevation of just one metre has already been measured.

The only factor that is negating such an experiment that I suggest is that so far laboratory precision atomic clocks are not portable and have so far only tested the parameters of altitude and relative motion within the lab.

Here is some great discussion on this chat site, where despite the fact that some of the posters, (who are all working in the area of some type of scientific research because that is part of the prerequisite necessary to be a member of this site), are making mistakes concerning potential energy being a contributor to mass value, and other such gaffs, people seem not to be problematic about expressing the understanding that conventional physics has not got all the answers and don't seem to have a problem in exploring the fact.

If I were able to join this site (which I'm not as I cannot prove that my job incorporates research), I would add to the discussion that a cesium atomic clock will be observed to have a higher frequency of electron transitions on top of the mountain as compared to the valley, and that in most areas of physics, a higher frequency is accompanied with a higher energy.

The conversation is interesting to me in that here we can see people who are professional or semi professionals, who have jobs at universities, who are making mistakes despite their obvious status of 'university education', all engaging in the premiss of an exploratory conversation without a hint of any derision being attributed to the OP, or to the mistakes made by some of the replies.  Or perhaps the mistakes I spot went unoticed by people replying because they were concentrating on the premiss of the question rather than searching for any means possible to be obtuse to the content and deride the OP, or any of the replies.

https://www.researchgate.net/post/Where_does_mgh_potential_energy_reside_exactly
Title: Re: Is there a discrepancy with the equivalence principle?
Post by: alancalverd on 21/04/2017 23:06:08
Excuse my lacking in knowledge of elementary mathematics.
Nothing to do with maths. Physics.

Quote
The clock in the greater gravity field will have more potential energy than the clock in the lesser gravity field.

Agreed?

Beware of imprecise or elliptical statements. The clock subject to the higher value of g will appear to run slower. Predicted and demonstrated fact.
Title: Re: Is there a discrepancy with the equivalence principle?
Post by: timey on 21/04/2017 23:26:35
Not being sure if mg=weight, or if m is incorporating the weight within the value of itself in the mgh is merely an non understanding of the construct of the math, nothing to do with physics which for me is all about shapes and curves and patterns which are indeed maths but aren't expressed in mathematical format.

*

But in the instance of the conditions of position that my experiment suggests, the clock in the denser location 'will' have a greater value of pe won't it?

If you can answer that question we might be able to discuss some unconventional physics, which is indeed my purpose here Alan, to explain to you my New Theory, right?

And given that you are engaging with me here, this is to the purpose of understanding what this New Theory incorporates, right?
Or do you have some other reason for posting in response to the proposition of a new idea?

Because if you aren't going to actually 'hear' the new idea, engage with the premiss of it, and explore the parameters, then what is it that you are doing?
Title: Re: Is there a discrepancy with the equivalence principle?
Post by: alancalverd on 22/04/2017 00:10:22
Once again: mass is an inherent and fixed property of a body. Weight is the gravitational force on that body in trhe presence of another, and varies with gravitational field.

Potential energy is measured with reference to some point. In the case of gravitational potential energy it is relative to the point on the earth's surface a distance h immediately below  the starting point. Now you have postulated two starting points a, b with different local values gb<gb because there is a lump of something dense at b.

Now move your test mass from b to a. There being an attractor at b, you have to do some work to move it away, so the potential energy at a must be greater than at b.

You can see how important it is to recognise the sign convention that gravitational potential is zero in deep space and negative close to an attractor.
Title: Re: Is there a discrepancy with the equivalence principle?
Post by: timey on 22/04/2017 01:52:53
Quote
Potential energy is measured with reference to some point. In the case of gravitational potential energy it is relative to the point on the earth's surface a distance h immediately below the starting point. Now you have postulated two starting points a, b with different local values gb<gbbecause there is a lump of something dense at b.

Now move your test mass from b to a. There being an attractor at b, you have to do some work to move it away, so the potential energy at a must be greater than at b.

I have read umpteen physics books and have just finished reading John D Barrow's book on Theories of Everything, again...
Do not confuse my non-understanding of mathematical format with a lack of understanding of conventional physics.

The experiment that I suggest is not moving anything anywhere.
The clocks are placed at 2 differing locations and measured relative to each other.  The radius of centripetal motion is the same for both clocks.  This ensures that the SR time dilation effects of centripetal motion are equal for both clocks.  It also ensures that sea level is the same distance from centre of earth.
Sea level is now the point that h will be measured from.
One clock is located at a dense location and the other is located at a significantly less dense location, where both locations are the same h from sea level.
pe=mgh, where g can be measured at both locations, and the clock in the denser location where the measure of g is greater will have a greater value of pe.

If we raised both clocks at their locations by a metre, they will both be observed to run faster, i.e. have a greater frequency of electron transitions, where both clocks will have an increased value of pe.
If we place clocks at each of the locations a metre below the original h, the clocks will be observed to run slower, i.e. have a lesser frequency of electron transitions, where both clocks will have a decreased value of pe.

Conventional physics states that potential energy is not the cause of the higher or lesser frequency of electron transitions observed of the clock...

My theory of time explores the notion that potential energy is indeed the cause of the higher or lesser frequency of electron transitions.
If my theory is correct then at these 2 locations each clock will be observed to have a higher frequency when raised a metre above its original location, and will be observed to have a lower frequency when placed a metre below its original location, as per GR predictions and experimental evidence, but the clock in the denser location will always be running faster than the clock in the less dense location when placed at the same h's.

In this manner no transgressions are made from the predictions of GR, but the 'interpretation' that bigger M's are running slower time than smaller M's, that is born of the observed fact of a clock running slower nearer to M is wrong.

If my theory is wrong then the clock at the denser location will run slower.

This is not a difficult concept to grasp, and one that I'm quite sure that you cannot possibly misunderstand...
(and as the ReasearchGate chat forum illustrates in its thread 'Where does mgh potential energy reside", I am far from being the first person in the world to consider that it resides and reacts within the mass)

Would you now like to engage in a discussion concerning the far reaching ramifications - that are inclusive of a physical cause for the wave function, a physical cause for gravitational acceleration/deceleration, a physical cause for Big Bang, a physical cause for Inflation period, the standard model being unified with gravity, and the negation of the necessity for Dark Matter and Dark Energy - of my theory being correct?
Title: Re: Is there a discrepancy with the equivalence principle?
Post by: timey on 22/04/2017 02:47:39
I did actually note a serious challenge to the premiss of my theory that Pete posted on Mikes thread involving the Shapiro delay...
I'm going to have to wait until I have access to a computer to investigate the exact parameters of this type of observation because I cannot view pdf's on my phone.
Title: Re: Is there a discrepancy with the equivalence principle?
Post by: timey on 22/04/2017 04:47:27
Actually this Shapiro delay doesn't pose a problem for my theory at-all.

According to my theory all motions in space are subject to being accelerated or decelerated by my model's addition of a 3rd aspect of the time dilation phenomenon that is caused by gravitational field energy where m=0. i.e 'open space'.
The radar beam between source Earth and target Venus will travel a curve of time that gets slower as it leaves the Earth and faster as it reaches Venus, but because the parameters are the exact opposite of the curve of time that gets faster as it leaves Earth and slower as it reaches Venus, what we are looking at is the same arc of curve, but for differing reasons.

Introduce the Sun onto the stage and we have an attractive body in the vicinity just off centre between source and target that causes the radar beam to bend...
In my model this bending of the radar beam is caused by time speeding up in the gravitational field of the Sun, which would lead one to think that the radar beam would take less time to complete the round trip than it would without the Sun being in the vicinity just off centre between source and target... it certainly had me thinking such until I thought it through properly...
However, the distance involved in the bend caused by the gravitational field of the Sun will add to the distance that the radar beam travels, as compared to the distance without the Sun in the vicinity, and the extra time it takes for the radar beam to travel the extra distance of this bend caused by the Sun and return to source will be the cause of the observed time delay.

Same arc of curves, same value of time delay, but a mirror image causation... where the maths and values that describe conventional physics can also describe my alternative theory if they are rearranged in their configuration,

Challenge to my theory negated!
Title: Re: Is there a discrepancy with the equivalence principle?
Post by: alancalverd on 22/04/2017 08:58:42

Conventional physics states that potential energy is not the cause of the higher or lesser frequency of electron transitions observed of the clock...

"Conventional" physics states that time runs slower at a lower gravitational potential, because that is what we observe. It doesn't matter how you obtain that lower potential, whether by moving closer to the attractor or increasing  its mass, g = GM/r^2 is the defining parameter. What's the problem? 
Title: Re: Is there a discrepancy with the equivalence principle?
Post by: alancalverd on 22/04/2017 09:01:27

The radar beam between source Earth and target Venus will travel a curve of time that gets slower as it leaves the Earth and faster as it reaches Venus, but because the parameters are the exact opposite of the curve of time that gets faster as it leaves Earth and slower as it reaches Venus, what we are looking at is the same arc of curve, but for differing reasons.

So your radar beam does not behave the same as everyone else's? How does it know it's yours? 

Even if the starting and finishing points were the same, the curves would be of different shape.
Title: Re: Is there a discrepancy with the equivalence principle?
Post by: alancalverd on 22/04/2017 09:07:23
And whilst I'm on my high horse, all clocks behave the same whether they involve the potential energy of electrons (and I can't think of a clock that does) or the escapement wheel of a wristwatch. Except of course for pendulum clocks which depend on the local value of g.
Title: Re: Is there a discrepancy with the equivalence principle?
Post by: timey on 22/04/2017 13:26:28
So your radar beam does not behave the same as everyone else's? How does it know it's yours? 

Even if the starting and finishing points were the same, the curves would be of different shape.

What on Earth are you on about?

If a straight line trajectory from (a) to (b) takes 2 minutes to travel at a constant speed as measured by a clock in the lab, and we then introduce the fact that there are time variants in the space in-between (a) and (b) that get steadily more extreme from (a) up to midway point, and then get less extreme from midway point to (b)...

1) It wouldn't be of any consequence to the time of 2 minutes that it had taken to travel between (a) and (b) as measured by the lab clock.

2) It would make no difference to the time measurement of 2 minutes that the lab has made if the time variant was faster or slower at the midway point, so long as the rate of time was faster or slower to the same value in the positive or the negative held relative to the lab clock.

3) Although the trajectory travelled between (a) and (b) is a straight line, if the lab were to create a data graph of the time variants between (a) and (b) held relative to the length of a second as measured by the lab clock, the data curve of the changes in rate of time would constitute the same value of curve be they describing either a faster or slower rate of time at the midway point, so long as the rate of time was faster or slower to the same value in the positive or the negative held relative to the lab clock.

Now introduce point (c) that is placed off centre in between (a) and (b).

1) Point (c) is an attractive body...
Due to the influence of this attractive body (c) the trajectory of the path travelled from (a) to (b) is no longer a straight line.  It becomes bent towards (c) and constitutes (with regards to Shapiro delay) a special case of gravitational lensing.

2) Again, It would make no difference to the time measurement the lab has made of more than 2 minutes, that is caused by this curved trajectory if the time variant was considered as faster or slower in the vicinity of (c), so long as the rate of time was faster or slower to the same value in the positive or the negative held relative to the lab clock.

Therefore the Shapiro delay observation does not pose a problem for my theory.

Quote
"Conventional" physics states that time runs slower at a lower gravitational potential, because that is what we observe. It doesn't matter how you obtain that lower potential, whether by moving closer to the attractor or increasing  its mass, g = GM/r^2 is the defining parameter. What's the problem?

Physics does observe that a clock runs slower at a lower gravity potential, but I observe that physics does not describe 'why'.
I don't have a problem Alan, but I've read that convention physics does have problems in describing some pretty crucial stuff.
Under the remit of my theory this 'pretty crucial stuff', that conventional physics has problems describing, can be described as cause and effect mechanics.
I quite clearly outlined how a clock can be observed to run slower in the lower gravity potential, faster in the higher gravity potential, and 'also' run faster in the greater gravity field.
What's your problem?

Quote
all clocks behave the same whether they involve the potential energy of electrons (and I can't think of a clock that does) or the escapement wheel of a wristwatch. Except of course for pendulum clocks which depend on the local value of g.

Actually the fact is that clocks that are basing their time measurement on the frequency of electron transitions are observed not to behave the same in differing gravity potentials...
The experiment that I suggest would determine whether or not potential energy is the cause of the energy changes that are usually required to initiate frequency changes.

Quote
And whilst I'm on my high horse

Have you ever considered getting a smaller horse?
Title: Re: Is there a discrepancy with the equivalence principle?
Post by: alancalverd on 22/04/2017 13:45:48
We ignorant earthlings use electromagnetic radiation for RAdio Detection And Ranging, in the naive belief that its propagation speed in vacuo is constant.   

Anyway, if it wasn't, it is clear that the radius of the path of any projectile passing an attractor, increases with the speed of the projectile, which is why bullets travel further than cricket balls.  So if your supposed radar beam speeded up as it left earth for venus, it would describe a quite different banana path from one that slowed down.

Quote
Actually the fact is that clocks that are basing their time measurement on the frequency of electron transitions are observed not to behave the same in differing gravity potentials...
A reference would be most helpful.
Title: Re: Is there a discrepancy with the equivalence principle?
Post by: timey on 22/04/2017 15:05:23
We ignorant earthlings use electromagnetic radiation for RAdio Detection And Ranging, in the naive belief that its propagation speed in vacuo is constant.

If a straight line trajectory from (a) to (b) takes 2 minutes to travel at a constant speed as measured by the lab clock.

I guess that I too am an earthling, how strange that you should miss the fact.

Anyway, if it wasn't, it is clear that the radius of the path of any projectile passing an attractor, increases with the speed of the projectile, which is why bullets travel further than cricket balls.

As said, this earthling keeps the speed constant held relative to the lab clock, as this is how experiments are measured.

So if your supposed radar beam speeded up as it left earth for venus, it would describe a quite different banana path from one that slowed down.

This earthling's supposed radar beam, at the speed of c held relative to the lab clock, travels into slower time (not faster) on it's way to Venus, where it reaches the point that it starts being subject to Venus's gravity field where it then travels into faster time to bounce off Venus and make the return trip to Earth. (where my model concludes that conventional physics relies on the use of SR to describe this effect)

Yes you are correct that the data curves would differ from each other...  One would be the exact inverse of the other where the changes of the rate of time in relation to distance travelled of either description are held relative to the lab clock, and both descriptions will result in the same physical observation.

With respect to the fact that clocks that are basing their time measurement on the frequency of electron transitions are observed not to behave the same in differing gravity potentials...

A reference would be most helpful.

See NIST relativity tests, where clocks are observed to have a higher frequency of electron transitions at 1 metre altitude as observed from the lower potential, and the maths that state that the clock in the lower potential will have a lower frequency of electron transition as observed from the higher potential.
Title: Re: Is there a discrepancy with the equivalence principle?
Post by: alancalverd on 22/04/2017 17:09:28
NIST established that clocks at altitude appear to run faster than those on the ground, as expected. They all behave the same way, regardless of the mechanism of their timekeeping. Your previous statement was ambiguous.
Title: Re: Is there a discrepancy with the equivalence principle?
Post by: timey on 22/04/2017 17:37:21
It is in fact relevant to the discussion that all clocks behave the same way regardless of their time keeping mechanism.

All clock mechanics operate on a frequency based system...
You can place different types of clocks at same height, and each different mechanism will be operating at a differing frequency, but the time keeping each clock is observed of will be the same.

When the clocks are raised into the higher potential they all are observed, from the lower potential, to have higher frequency.  The differences in each clocks frequency between different mechanisms of time keeping at the lower potential will all remain proportional to each other when the clocks are raised into the higher potential.
This must be a truth because otherwise the clocks would not be considered adequate time keepers.

If an increase in potential energy were the cause of the increase in frequency observed of elevated clocks from the lower potential, then this can be described for any type of clock as pe=mgh where pe/m= an equal increase of pe for any value m.
Title: Re: Is there a discrepancy with the equivalence principle?
Post by: alancalverd on 22/04/2017 23:44:29
But the frequency of photons is also observed to increase with gravitational potential, by exactly the same amount, even though m = 0. So it's nothing to do with the mass of the clock or any part thereof. 

The nice thing about GR is that is is based on one assumption only - the constancy of c - and predicts the observed blue shift and gravitational lensing to an exceptional degree of accuracy. So why invoke any other mechanism?
Title: Re: Is there a discrepancy with the equivalence principle?
Post by: timey on 23/04/2017 00:43:59
But the frequency of photons is also observed to increase with gravitational potential, by exactly the same amount, even though m = 0.

A photon emitted at h sea level will be measured at 1 metre elevation as having decreased in frequency.
A photon emitted at 1 metre h from sea level will be measured at sea level as having increased in frequency.
A clock (of any type sensitive enough to measure) placed at sea level will be observed from 1 metre h from sea level to have a lower frequency.
A clock (of any type sensitive enough to measure) placed at 1 metre h from sea level will be observed from sea level to have a higher frequency.

It would seem to me that there is a marked difference in how the frequency of light increases in the gravity potential...
This difference being that a clock has a higher frequency in the higher gravity potential, and that light emitted in the lower potential will have a lower frequency in the higher gravity potential.

So it's nothing to do with the mass of the clock or any part thereof.

That light has no mass constituting a g(r)=GMearth/r^2 equation, and the clock has mass constituting a mgh=pe/m=an equal addition of pe for any mass value, would suggest that the differences in how the frequency of a clock (any type) and how frequency of light are affected by gravity, as I illustrated above can be physically described as the difference between gravity potential energy and gravitational field energy.

The nice thing about GR is that is is based on one assumption only - the constancy of c - and predicts the observed blue shift and gravitational lensing to an exceptional degree of accuracy.

I agree that GR is great, where the fact is that the maths for GR can describe my contracting model...
My model also holds that c is a constant, and for an observer at any gravity potential anywhere on the universe c will always be 299 792 456 m/s^2, and as gravitational field time where m=0 changes from one potential to another, this will be descriptive of red shift and blueshift as per the values of the gravitational shift equation.

The inverted time curve (as described for Shapiro delay a few posts back) will also apply for gravitational lensing observations, where the inverse time curve will describe the observation of the light being curved towards the attracting body.

So why invoke any other mechanism?

In order to attribute physical cause and effect mechanics to the acceleration of gravity, give physical cause for the wave function of the point particle, unite the standard model with gravity, describe a continuum negating quantum, give physical cause and effect mechanics to the Big Bang and Inflation theory and provide physics with a unifying theory of everything.

That's why...
Title: Re: Is there a discrepancy with the equivalence principle?
Post by: alancalverd on 23/04/2017 13:33:46

A photon emitted at h sea level will be measured at 1 metre elevation as having decreased in frequency.
A photon emitted at 1 metre h from sea level will be measured at sea level as having increased in frequency.
A clock (of any type sensitive enough to measure) placed at sea level will be observed from 1 metre h from sea level to have a lower frequency.
A clock (of any type sensitive enough to measure) placed at 1 metre h from sea level will be observed from sea level to have a higher frequency.

It would seem to me that there is a marked difference in how the frequency of light increases in the gravity potential...
This difference being that a clock has a higher frequency in the higher gravity potential, and that light emitted in the lower potential will have a lower frequency in the higher gravity potential.


You must amaze yourself sometimes! You say A>B in both cases, so the mechanism must be different because the result is the same. Your future is in politics, not physics.

I admire your concluding chutzpah! Her Majesty's Loyal Opposition needs a new leader who can solve everything  with a stroke of the pen. Go for it!
Title: Re: Is there a discrepancy with the equivalence principle?
Post by: timey on 23/04/2017 15:01:49
You say A>B in both cases,

I don't know what you mean when you say A>B.
If physics says that time runs slower nearer M then it doesn't matter where the observer is located or what he observes because the maths are telling us that the clock runs slower nearer M, where a decrease in the frequency of the clock is indicative of a slower rate of time...
So a clock at 1 metre above sea level compared to a clock at sea level will have a HIGHER FREQUENCY.
If one records the frequency of a photon emitted at sea level and then records the frequency of that photon when it has reached 1 metre above sea level for comparison purposes, (which again is unnecessary as the maths are descriptive), that photon at 1 metre above sea level will have a LOWER FREQUENCY.

The clock will have a HIGHER FREQUENCY at 1 metre above sea level compared to the clock at sea level.
The photon will have a LOWER FREQUENCY at 1 metre above sea level compared to the frequency it had when emitted at sea level.

That is an observed FACT of physics.

so the mechanism must be different because the result is the same.

The result is not the same.
The result shows a clear difference in how the frequency of emitted light and the frequency of a clock behave in the gravity field.

But the mechanism I propose is the same...
It is a FACT of physics that frequency is proportional to energy.
So in looking at what energy 'may be' causing frequency to change:
mgh=pe
and where m=0:
gh=gravitational field energy.

I am simply adding a new suggestion, this being that the phenomenon of time is physically caused by energy, thus adding a 3rd aspect to the time dilation phenomenon - where the decrease in gravitational field energy is indicative of slower rates of time for anywhere where m=0, and that where mgh=pe and pe/m (so any value m will increase or decrease pe equally), m in the gravitational field will be running at a faster rate of time than the field itself where these rates of time occur independently from each other and simultaneously to each other, where the variable gravitational field energy time causes the acceleration or deceleration of m's motions in the gravitational field.

That is not politics.
That is a perfectly simple and tangible mechanism that gives physical cause to the phenomenon of time and to the acceleration of gravity.

I understand that this is not conventional physics, but it is a FACT that conventional physics is perfectly well aware that it does not have a fully formulated Theory of Time.

This Theory of Time results in fully described cause and effect mechanics for a cyclic universe that does not require Dark Energy, Dark Matter, or anything that is not already observed...
All it requires is that one calculate from the viewpoint that +energy=shorter seconds, i.e. a faster rate of time, where different rates of time can occur independently of each other. and simultaneously to each other.*
(*the FACT that different rates of time can occur independently of each other and simultaneously to each other is clearly illustrated by the relativity mobile phone app)

It's not 'hard' Alan...
It's a very simple concept that is highly calculable via GR mathematics. (for someone versed in mathematics).

The experiment that I suggest would serve to prove or disprove this Theory.
Title: Re: Is there a discrepancy with the equivalence principle?
Post by: nilak on 23/04/2017 17:59:25
Although your analogy with the photon that gets a higher frequency at a lower level, works with my hypothesis except, I get a higher wavelength, but also a higher frequency (because fermions don't travel at constant c, but v varies) for fermions only, I it ( the analogy)doesn't work with standard physics because if the atoms of a clock go lower they gain speed but when the clock stops at that lower level that kinetic energy is lost. My hypothesis says that the electrons kinetic energy will be retained although the atom itself stops.
Title: Re: Is there a discrepancy with the equivalence principle?
Post by: timey on 23/04/2017 18:54:14
Although your analogy with the photon that gets a higher frequency at a lower level, works with my hypothesis except, I get a higher wavelength, but also a higher frequency

I have not made an 'analogy'.

It is a physical fact that light, be it redshifted or blue shifted, has a higher frequency the closer it is to M.
It is a physical fact that the clock has a higher frequency the further away it is from M.

As to your theory, I'm sorry to say, but it is my understanding that a value of frequency is always 'inversely' proportional to wavelength.
Title: Re: Is there a discrepancy with the equivalence principle?
Post by: nilak on 23/04/2017 20:11:28
Although your analogy with the photon that gets a higher frequency at a lower level, works with my hypothesis except, I get a higher wavelength, but also a higher frequency

I have not made an 'analogy'.

It is a physical fact that light, be it redshifted or blue shifted, has a higher frequency the closer it is to M.
It is a physical fact that the clock has a higher frequency the further away it is from M.

As to your theory, I'm sorry to say, but it is my understanding that a value of frequency is always 'inversely' proportional to wavelength.
I said it is an analogy because if light blueshifts as it goes lower into the gravity well, I thought you said that a clock should do that as well, but I've seen now you have a different hypothesis. As for the frequency vs. wavelength, the general formula is f=v/λ. If v increases and the wavelength increases the result can be either a constant frequency, a decrease or an increase in frequency. For light v=c=constant therefore the frequency is always inversely proportional to the wavelength. Why would lambda increase with velocity? Just look at an OAM light beam and it just does that. It is a fact.
Title: Re: Is there a discrepancy with the equivalence principle?
Post by: alancalverd on 23/04/2017 21:35:23
Sorry, Timey, you are spouting absolute nonsense. We've been around this block before and I thought you understood the experimental results. Apparently not. I'm out.
Title: Re: Is there a discrepancy with the equivalence principle?
Post by: timey on 23/04/2017 21:51:25
Sorry, Timey, you are spouting absolute nonsense. We've been around this block before and I thought you understood the experimental results. Apparently not. I'm out.
Sorry, Timey, you are spouting absolute nonsense. We've been around this block before and I thought you understood the experimental results. Apparently not. I'm out.

It is a physical fact that light, be it redshifted or blue shifted, has a higher frequency the closer it is to M.
It is a physical fact that the clock has a higher frequency the further away it is from M.

That 'is' the experimental and predicted results.
You, as a physicist cannot state otherwise because you will be incorrect.

As to my contracting model and my model's interpretation of observations and the experimentally verified results, a contracting model will not interpret observation in the same way as an expanding model.

What seems to be the problem here Alan is your inability to consider a model that isn't exactly the same as the current model, and this, considering the board title "New Theories" is what 'is' nonsensical.
Title: Re: Is there a discrepancy with the equivalence principle?
Post by: nilak on 23/04/2017 23:05:35
Quote
mgh=pe
and where m=0:
gh=gravitational field energy.
Again, if m=0 then pe=0 in that equality. Or I'm just missing somethig?
gh is not energy but gravitational potential (because gravitational potential is per unit (1) mass) , unless there is a special case where m=1kg.

Quote
I am simply adding a new suggestion, this being that the phenomenon of time is physically caused by energy
I agree with you here. That is because it is also the conclusion following my hypothesis.

You say that if the potential energy of the clock gets lower its frequency goes down. If your clock has a Ep potential energy at h1 and it free falls till h0 it will loose(Epl is the amount lost) part of Ep which goes into kinetic energy now Ek.
E0=Ep=Ep-Epl+Ek
Epl=Ek
But at h0 the clock is held in place (by a net for example) and we measure its energy again. Now its kinetic energy is absorbed by the net and the new energy becomes E1=Ep-Epl therefore E1<E0.

Well, my hypothesis is exactly the opposite. I don't consider the Ep as intrinsic to the clocks energy but only as a energy gain possibility (potential) only if the clocks falls down but the clock doesn't have that energy yet. The clock falls and gains energy then its Ek is absorbed by the net. Therefore no kinetic energy change in the process and the clock should hold its frequency. However, my hypothesis says that although the atom stops, the kinetic energy increase (because even if the atom was initially at rest its electons were in motion) of electrons around the atom is retained. Futthermore as a comparison to your model, my hypothesis says a fermions at a higher velocity has a higher frequency but also a higher wavelength and the wavelength is responsible for the clock's frequency decrease. (I'm sorry I'm probably exaggerating with my model).
Title: Re: Is there a discrepancy with the equivalence principle?
Post by: timey on 23/04/2017 23:46:04
Quote
mgh=pe
and where m=0:
gh=gravitational field energy.
Again, if m=0 then pe=0 in that equality. Or I'm just missing somethig?
gh is not energy but gravitational potential (because gravitational potential is per unit (1) mass) , unless there is a special case where m=1kg.

Look Nilak, I am not versed in maths...
By saying gh=gravitational field energy what I am looking to describe is more conventionally termed (I think) as g(r)= GMearth/r^2.

I am trying to attribute the gravitational field itself with energy where the energy is greater at closer radius to M.
Title: Re: Is there a discrepancy with the equivalence principle?
Post by: GoC on 24/04/2017 15:26:10

I am trying to attribute the gravitational field itself with energy where the energy is greater at closer radius to M.
You have it backwards. Dilation increases to the center of mass. The energy expands to become less dense. It is the density of energy affecting the clock in GR. The density of energy decreases as the mass increases. At the speed of light attraction in a very large sun the expansion is so great particles can no longer remain apart and a BH is formed. The mass of our sun would occupy about 1.7 miles in diameter.
Title: Re: Is there a discrepancy with the equivalence principle?
Post by: timey on 24/04/2017 16:04:45
https://socratic.org/questions/explain-how-force-energy-and-work-are-related-1

Quote
: Socreatic.org
When a force acts on an object as the object travels some distance, the work done on the object is equal to the force times the distance. And when work is done on an object, its energyincreases by an amount equal to the work.

Therefore it stands to reason that as an objects energy increases by the work done by the force, that the force has an energy that it imparts to the object.

A gravitational field has more force nearer M than it does further away.
Title: Re: Is there a discrepancy with the equivalence principle?
Post by: alancalverd on 24/04/2017 18:57:38
It is a physical fact that light, be it redshifted or blue shifted, has a higher frequency the closer it is to M.
It is a physical fact that the clock has a higher frequency the further away it is from M.

For the last time, ever, light is observed to be blue shifted when seen from a lower gravitational potential than its source. Clocks are seen to run faster when observed from a lower gravitational potential than the signal source. Same phenomenon, same process.

Imagine a clock that generated a time signal by electrons moving between energy levels with a small energy difference. That would produce an electromagnetic (radiofrequency) wave whose  frequency defined time. Now imagine an electromagnetic wave generated by electrons moving between levels with a much larger energy difference - light. At what photon energy do you think the mechanism of blue shift  suddenly reverses, and why?

Or let's make it really simple. A mechanical clock operates a shutter that allows blue light to pass for 0.1 seconds every second. We observe it from a lower gravitational potential. Are you saying that the clock appears to run faster but the light gets redder? You know that isn't the case.
Title: Re: Is there a discrepancy with the equivalence principle?
Post by: timey on 24/04/2017 20:32:47
For the last time, ever, light is observed to be blue shifted when seen from a lower gravitational potential than its source. Clocks are seen to run faster when observed from a lower gravitational potential than the signal source. Same phenomenon, same process.

Yes - same phenomenon, yes- same process...
But the frequency of 'already emitted' light shifts to a LOWER frequency in the HIGHER potential, and the clock shifts to a HIGHER frequency in the HIGHER potential.
The maths specify this as being such, and why these phenomenon are viewed as they are in or from a location in the gravity potential is blatantly obvious.

Imagine a clock that generated a time signal by electrons moving between energy levels with a small energy difference. That would produce an electromagnetic (radiofrequency) wave whose  frequency defined time

The length of a second is already defined, so the frequency of the electromagnetic wave caused by electrons moving between energy levels would have to be synonymous to the length of an already defined second.
The frequency of the electron transitions between 2 certain ground states of the caesium atom was chosen because it is synonymous with the length of the 'already defined by history' length of second and keeps time to the tune of this length of second much better than any predecessor of time keeping did.
The quantum clock now keeps time 37 times better than the caesium atomic clock.  It operates at a frequency 100,000 times higher than the frequencies used in NIST-F1 and other similar time standards around the world.
All this means is that there are a lot more wave cycles within the time period length of a second than the atomic clock counterpart's frequency has.  Therefore a second can be more precisely broken down into fractions of a second and makes for a more precise measurement.
It does not mean that the length of a second has been altered.

Now imagine an electromagnetic wave generated by electrons moving between levels with a much larger energy difference - light. At what photon energy do you think the mechanism of blue shift  suddenly reverses, and why?

I don't think that the mechanism of blue shift reverses and have never said that.
The caesium atom 'emits' photons.  Blue shifting occurs for 'already emitted' photons.

Are you suggesting that the caesium atom is 'emitting' higher frequency photons in the higher gravity potential?
Observing from the lower potential the emitted photon can 'only' be viewed when it reaches the lower potential, and we know that the photon will have been blueshifted as it travelled from the higher potential to the lower potential...
Title: Re: Is there a discrepancy with the equivalence principle?
Post by: alancalverd on 24/04/2017 21:48:36
I don't know why I bother. Pehaps I care.

Gravitational redshifts and clock variations depend only on the relative positions of the source and observer, and they behave in exactly the same way.

There is no point in your trying to explain a phenomenon that does not occur.
Title: Re: Is there a discrepancy with the equivalence principle?
Post by: nilak on 24/04/2017 21:58:10
I think Timey wants to say that if we have a clock that has 1 Hz at the ground level and emmits a blue light and  then we place it much higher, its frequency will increase at say 2Hz. The observer will also see it flashing at 2Hz (or perhaps will see it higher) and expects the blue laser to emmit a higher frequency wavelength (say violet)  than it was on the ground because time runs faster. Furthermore, as the light travels down to the observer if the light was violet it gets even more blueshifted to say UV.
However, I'm not sure you can do that in relativity. You simply observe the light frequency and the clock frequency and assume that is how time runs there relative to you.
Title: Re: Is there a discrepancy with the equivalence principle?
Post by: jeffreyH on 24/04/2017 22:21:19
The problem is you cannot directly compare here with somewhere else. There has to be some sort of transformation involved. In the case of special relativity it is the Lorentz transformation. People who don't understand the mathematics of relativity can adopt erroneous notions.
Title: Re: Is there a discrepancy with the equivalence principle?
Post by: timey on 25/04/2017 00:01:43
I don't know why I bother. Pehaps I care.

Gravitational redshifts and clock variations depend only on the relative positions of the source and observer, and they behave in exactly the same way.

There is no point in your trying to explain a phenomenon that does not occur.

It is a prediction of General Relativity that a clock will run faster at elevation than it does at a lower elevation and that one's position in the gravitational field affects what one observes of the clock in the differing gravity potential.
(What have I said here that is not correct?)

Already emitted light will blueshift towards the gravitational field M and redshift away from the gravitational field M as per the diagram in this wiki link:

https://en.m.wikipedia.org/wiki/Gravitational_redshift

Yes - the observation of what one observes of redshifted or blue shifted light depends on the observers position in the gravity field, but the maths describe that the light is shifting whether anyone is observing or not.
(What have I said here that is not correct?)

I know that what I am saying is correct.

What I wish to discuss within these experimentally verified physical facts is that the frequency source (an atom, i.e. not m=0) that comprises the mechanism of the clock - a clock that is observed to run faster from the lower potential, or observed to run slower from the higher potential - this frequency source of the clock has a frequency that gets 'higher' in the higher potential...
... but light emitted in the lower potential will be observed to have a 'lower' frequency when it arrives 'in' the higher potential.

If one draws a graph of the maths of the frequency increases of the clock as it is placed in higher potentials (a perfectly reasonable thing to do because physics has basis within its maths of time running slower nearer to M), and then compares this to the graph of gravitationally shifted light in the gravity field as provided in the Wiki link, it is clearly obvious that while already emitted light will shift to a 'lower' frequency in the higher gravity potential, that a clock will shift to a 'higher' frequency in the higher gravity potential.

It is this difference in the shifting of frequency between what occurs for a clock and what occurs for already emitted light that I wish to discuss with a view to using an alternative interpretation of the observation and an alternative means of calculation to describe the observation...  Where modified mathematics is a common occurrence in physics.

MOND uses a modified calculation to describe galaxies.
The observation of the Galaxy remains the same but the maths describing the observation are differing from Newton's interpretation of the laws of motion and are attempting to describe Galaxy rotation without Dark Matter halo's.

My modifications of 'interpretation' are attempting to give physical cause to gravitational acceleration and this alternative interpretation 'can' be described mathematically (by someone versed in maths)

Gravitational acceleration is a known physical phenomenon with no known physical cause.
Title: Re: Is there a discrepancy with the equivalence principle?
Post by: alancalverd on 25/04/2017 20:34:55
Yes, there are several equations that give the correct answer. It just happens that the photon shift is more intuitively modelled by having the photon travel from source to observer, and the clock shift is more intuitively modelled by moving the clock away from the observer. However if you had a clock running in space and moved it closer to the earth, an earthbound observer would see it slow down, and if you moved your P&R mossbauer source closer to the earth, the blue shift would decrease. Now you will model both observations with the same GR equation because they are the same observation of the same phenomenon.

The best equation is the one that most accurately describes and predicts the observation.
Title: Re: Is there a discrepancy with the equivalence principle?
Post by: timey on 25/04/2017 21:04:39
Clearly...

But GR doesn't provide a description of physical cause for gravitational acceleration, nor does GR provide a description of physical cause for a clock ticking at a differing rate, nor does GR provide a description of physical cause for light shifting in the gravitational field, and GR is incompatable with a quantum description of light.

Therefore - I would like to discuss an alternative interpretation of the observation of the fact that, as per the graphs* show, a clock increases in frequency in the higher potential, and light emitted in the lower potential decreases in frequency in the higher potential.
(*where you may place observers at any location of gravity potential on that graph and work out what they observe if each other if you like, but this won't make an iota of difference to fact that the clock increases in frequency in the higher potential, and that light emitted in the lower potential decreases in frequency in the higher potential.)

Any chance we could have that discussion please?
Title: Re: Is there a discrepancy with the equivalence principle?
Post by: alancalverd on 26/04/2017 00:53:23
As you persistently misquote the experimental result, there is little point in discussing it.
Title: Re: Is there a discrepancy with the equivalence principle?
Post by: timey on 26/04/2017 03:02:45
Is there anyone else out there that can actually recognise that if one draws a graph of gravitationally shifted light that this shows that the frequency of light (any frequency) is shifted to a higher frequency direction towards M, and a lower frequency direction away from M?

Is there anyone else out there that can recognise that if one draws a graph of gravitationally shifted atomic clocks, that the frequencies of the elevated clocks, held relative to the ground clock, will be increased at each elevation, and that this is why physics states that time gets slower nearer to mass?

Honestly Alan I'm despairing of you now!
Title: Re: Is there a discrepancy with the equivalence principle?
Post by: alancalverd on 26/04/2017 08:23:38
Anyone could, but no physicist would. We are far too pedantic and bound to the truth. You might get someone to state that

"When observed from  a given point, electromagnetic waves and time pulses emitted from a higher gravitational potential appear to have a higher frequency than those generated by the same process  locally. When observed from the same gravitational potential as the source, there is no difference, regardless of the mechanism of the source or the reference, or the gravitational potential at that point. Therefore time is affected by gravitational potential. A mathematical model based on this hypothesis has predicted every experimental result to date."   

Anything less would have omitted important and relevant facts and conditions.
Title: Re: Is there a discrepancy with the equivalence principle?
Post by: nilak on 26/04/2017 08:28:11
Yes, I thought that's how it worked  according to relativity, and I even incorporated into my model, but I've recently realised (reading Alan's posts) that relativity says something a little easier but different, and therefore my hypothesis is probably incorrect.
If I got this right, you say that if a clock goes up against a gravitational field it's frequency increases. My model shows that if a massive particle does that it's wavelength decreases an I thought that should be a relationship between the clock rate and that wavelength. But this assumption is apparently not needed as the wavelength always depends on its velocity (therefor KE). Anyway the wavelength changes when travelling from higher to lower. In free fall the wavelength gets lower as the particle gains speed but when it stops at a lower level the wavelength goes probably back to where it was.
Your hypothesis is different. You say that energy level is responsible for the tickrate. When moving along the gravitational field, PE varies but KE gets back to zero when stopping at the height you want to do the measurement. That could mean something, but my opinion is PE is not intrinsic of the clock, that is why it is called potential, the clock doesn't  have it yet.
Title: Re: Is there a discrepancy with the equivalence principle?
Post by: nilak on 26/04/2017 09:40:21
The queasy now is, if your clock rate gets higher as it climbs against the gravitational field, and the frequency of light gets lower then the two effects combine and alter the final result, which doesn't seem to happen. The redshift alone slows the frequency of the pulses that come from the clock below, that is because gravity redshifts light but also reduces any pulses of light sent from below. The clock below sends pulses at the same rate but they get rarer as they climb up. If you think the clock that is lower ticks slower then it would further slow down the frequency of pulses.
According to relativity It sounds as if the two clocks remain at the same rate, but the observers see different rates.
Title: Re: Is there a discrepancy with the equivalence principle?
Post by: alancalverd on 26/04/2017 14:32:18
Again, postulating the unobservable and confusing source and observer position. The whole point of the experiment and all the arguments that derive from it is that the apparent frequency of any source depends on the relative gravitational potential of the source and observer, and the behavior is exactly the same whether you are looking at the energy of a photon or the rate of reproduction of rabbits. Once you understand that, there is no anomaly.   

Explaining how it happens is indeed a whole 'nother thing - indeed a Noether thing - but it's a lot easier to explain one actual phenomenon than two imaginary ones!
Title: Re: Is there a discrepancy with the equivalence principle?
Post by: timey on 26/04/2017 14:34:43
When moving along the gravitational field, PE varies but KE gets back to zero when stopping at the height you want to do the measurement. That could mean something, but my opinion is PE is not intrinsic of the clock, that is why it is called potential, the clock doesn't  have it yet.

Ok look Nilak - for the purpose of illustration let's look at a damn that is producing electricity.

A body of water is held at a higher gravity potential and within that water held at the higher potential there is the potential to produce x amount of electricity by converting potential energy into kinetic energy.

Energy is always conserved, so where in this scenario does the potential energy reside?
Title: Re: Is there a discrepancy with the equivalence principle?
Post by: timey on 26/04/2017 14:35:33
Anyone could, but no physicist would. We are far too pedantic and bound to the truth.

There is a graph on the gravitational shifted light wiki link that shows that light gravitationally shifts in the gravity field.

The rate of time for a BH is thought to run slower than on Earth because it is a bigger mass and although an observer at the BH would not consider his clock to be running slow, he will AGE slower than a person on Earth.

Physicists describing near Earth time dilations concerning depth to centre of Earth and radial centripetal motion differences of longitude use a mathematical system to determine that a clock runs faster at a higher gravity potential radius, but also runs slower due to centripetal motion at that radius, and that in the case of the extra depth due to equatorial bulge, 1 cancels the other so that clocks run at the same rates at sea level of any longitude, despite the extra 23 or so kilometres depth at equator.

The same mathematical system can be drawn as a graph, where the line of curve from sea level for the GR altitude effects will resemble the graph on this link:

https://en.m.wikipedia.org/wiki/Gravitational_potential

The mathematical system is itself representive of a graph...

You might get someone to state that

"When observed from  a given point, electromagnetic waves and time pulses emitted from a higher gravitational potential appear to have a higher frequency than those generated by the same process  locally. When observed from the same gravitational potential as the source, there is no difference, regardless of the mechanism of the source or the reference, or the gravitational potential at that point.

Isn't this because one is then measuring the scenario as per the clock in that potential rather than at the rate of time of some other potential?
What you are saying suggests that the clock viewed to be ticking at a differing rate from another potential is just a mirage.
If there is no 'actual' difference in the clock's rate of time then a person with a clock that 'appears' to be time dilated from another potential will age at the same rate as a person anywhere else, thus rendering the famous Twin Paradox as a load of old baloney.

Therefore time is affected by gravitational potential.

Sorry, but I thought you just said that the time 'in' a gravity potential runs the same at each potential...
If the time is running at the same rate at each potential then surely under this remit it is not 'the time' that is affected by the potential, it is 'the observation' of time that is affected...

You cannot say that time is occurring at the same rate in each gravity potential and then say that a person ages in keeping with their time dilated clock.  That is a contradiction.

A mathematical model based on this hypothesis has predicted every experimental result to date."

And on the basis that this mathematical model is only describing the observation of what is occurring, but fails to describe 'why' it is occurring, this leaves a little room for 'interpretation' of observation so long as the 'interpretation' does not deviate from the values of the mathematical model.

Anything less would have omitted important and relevant facts and conditions.

The mathematical model fails to describe what the physical mechanisms for the observations are...
This being a fact that is widely written upon by physicists in the books that I have been studying.

If we could just accept that these physical processes of altitude related time dilation and motion related time dilation are occurring whether they are being observed by anyone or not, then a graph will show that clocks held at rest with respect to the gravity field will increase in frequency at elevation, and that the increased frequency of the clock will be counterbalanced by a decrease in frequency caused by centripetal motion of radius of longitude, or orbital velocity motion.
At a certain radius from Earth the orbital velocity cancels out the altitude dilation and the clock on the craft orbiting at that radius of altitude, at that orbital velocity will be running at the same rate as the clock on Earth.

This mathematical system is a graph that shows that the frequency of a clock will increase in the higher potential.

The gravitational shift wiki link shows a graph where the frequency of light, be the light red shifting away from M, or blue shifting towards M, always has a lower frequency in the higher potential.

There is 'nothing' in these last 2 paragraphs above that contravenes the conventional view of physics or the mathematical system that describes it...
...and I would really care to start discussing the fact that the frequency of light is gravitationally shifted in the higher potential to a lower frequency in 'unconventional terms' that will describe the acceleration of gravity via a system of cause and effect mechanics that gives the acceleration/deceleration of gravity a physical cause.

Is that too much to ask?
Title: Re: Is there a discrepancy with the equivalence principle?
Post by: timey on 26/04/2017 17:06:39
Going back to my suggested 'actual' experiment of placing 2 identical clock's on the surface of the Earth at different locations of the same height of gravity potential, and at the same longitude in order to equalise GR altitude and SR centripetal motion effects, but at locations of significantly differing geological density, where the value of m and h are the same, but the value of g is differing - to define via experiment which clock runs faster...

Please see below a 'thought' experiment that results in equalised parameters.  This thought experiment can be calculated via the existing mathematical system but concludes in asking where it is that the factor for the clock on the bigger mass running at a slower rate compared to the clock on the smaller mass resides...

https://www.thenakedscientists.com/forum/index.php?topic=70240.msg512969#msg512969
Title: Re: Is there a discrepancy with the equivalence principle?
Post by: nilak on 26/04/2017 21:40:22
When moving along the gravitational field, PE varies but KE gets back to zero when stopping at the height you want to do the measurement. That could mean something, but my opinion is PE is not intrinsic of the clock, that is why it is called potential, the clock doesn't  have it yet.

Ok look Nilak - for the purpose of illustration let's look at a damn that is producing electricity.

A body of water is held at a higher gravity potential and within that water held at the higher potential there is the potential to produce x amount of electricity by converting potential energy into kinetic energy.

Energy is always conserved, so where in this scenario does the potential energy reside?
If it is potential, the energy doesn't reside within the object but it is somewhere else, like in the gravitational field.
The redshift/blueshift of signals already results in time dilation. If you say the clocks change their frequency, as their potential energy changes, then it would mean the redshift / blueshift of light and signals doesn't happen. That is because, if both phenomena happened then the effect would be doubled than what we see.
If you think your principle is correct then you could try to suggest more experiment that could confirm it.
Title: Re: Is there a discrepancy with the equivalence principle?
Post by: timey on 26/04/2017 22:30:29
If it is potential, the energy doesn't reside within the body but it is somewhere else, like in the gravitational field.

If the energy is in the gravitational field then the difference between the equation mgh where m equals zero and where m doesn't equal 0 is significant.

If the dam were not full of water there would be no potential energy to convert.

The redshift/blueshift of signals already results in time dilation. If you say the clocks change their frequency, as their potential energy changes, then it would mean the redshift / blueshift of light and signals doesn't happen.

The clock is placed at elevation so it does have a differing potential energy to the clock below it regardless.  Whether this is what is causing the increased frequency of electron transitions or not is the point of my suggested experiment.

Light is redshifted or blue shifted in the gravity field 'after' it has been emitted not 'before'...
But there is an argument for the frequency emitting source of the clock's frequency of electron transitions emitting higher frequency photons in the higher gravity potential...
Measured in the higher gravity potential by the remit of the rate of time of the clock in the higher gravity potential any photons emitted by the emitting source in the higher gravity potential would be measured as having an identical frequency to the frequency of photons emitted by the emitting source in the lower potential when the frequency is measured by the rate of time in the lower potential.

"Measuring by the remit of the rate of time of the clock in the potential one is measuring" being the defining factor.

I already suggested an 'actual experiment' to prove or disprove my theory, and it's a very simple and relatively inexpensive experiment.
Title: Re: Is there a discrepancy with the equivalence principle?
Post by: nilak on 26/04/2017 23:03:41
I may be wrong but what I understand is the clock run as if they were at the same frequency (if space and time were absolute), only the signal frequency gets altered, but that in relativity means the clocks actually run at different rates.
Title: Re: Is there a discrepancy with the equivalence principle?
Post by: timey on 26/04/2017 23:38:18
Nilak - I haven't got time to teach you about how clocks work.  Just read all the wiki links and read some books on the subject.  That's what I did.

What I will do is leave you with this bit of advice concerning frequency.

The frequencies of differing emitting sources, or the frequencies of differing matter waves are all held relative to the length of second as per the caesium standard.
All of these frequency rates of these differing emitting sources or differing matter waves, as measured in the same gravity potential are quite differing from each other, from very few wave cycles per second to a very high number of wave cycles per second.

If we were to observe a caesium atomic clock at elevation from the lower potential where the observed increase in the higher clocks frequency of electron transitions as compared to the clock in the lower potential, is determined as a shorter length second - and then we were to measure the frequency of all emitting sources and matter waves in the lower potential from the lower potential held relative to this shorter second - then that which we are measuring 'in' the lower potential 'from' the lower potential will have a decreased frequency as compared to measuring the same phenomenon via the standard second.
All we have done is decide to measure that which is in the lower potential by the remit of the shorter second observed of the higher potential.

Also don't forget that wavelength is always inversely proportional to both frequency and energy, but frequency and energy are always proportional to each other...

Good luck with your explorations.
Title: Re: Is there a discrepancy with the equivalence principle?
Post by: timey on 28/04/2017 00:16:51
Is this too much to ask?

Well it would seem that it is too much to ask, however this flies in the face of logic considering that each and every physics book that I read describes the incompatibility of quantum physics and general relativity, and most of the physics books I read describe the search for a unifying theory.

Here you have someone, this being me, who is stating that by making a few adjustments in how one interprets observation and experimentally verified observation, that the maths that describe general relativity can describe quantum as a continuum unifying the two theories while negating the necessity for the need for Dark Matter and Dark Energy, or any multiple world scenario.

Yet finding a person who is willing to make calculation of the alternative interpretation of observation seems to be an impossible mission.

Can anyone provide me with a plausible explanation for this paradox?
Title: Re: Is there a discrepancy with the equivalence principle?
Post by: alancalverd on 28/04/2017 06:46:54
Oh dear, here comes that boring old pedant again. Red shift is not measured with respect to a cesium clock, but the angle of diffraction of visible photons or the beat of the hydrogen 21 cm line mixed with one generated locally, or a dozen other things including the speed of the moving mossbauer receptor in the Pound-Rebka experiment, all of which predated the cesium clock. It just happens that atomic clocks are handy for precise measurements in near space.

However you measure time, it is the same for all systems at a given gravitational potential, but observably differs between points at different potentials.

Interpreting observation is indeed what it's all about, but you haven't produced an observation that suggests anything like inverted time dilatation.
Title: Re: Is there a discrepancy with the equivalence principle?
Post by: jeffreyH on 28/04/2017 07:52:42
If you travel one metre at a constant speed that is held relative to a longer or shorter 'variable' second.  Then the distance remains the same, and it just takes a longer or shorter amount of 'time' to travel that metre.

That is a very pertinent point expressed with clarity. No one has as yet detected length contraction. If you maintained that clarity you would make much better progress.
Title: Re: Is there a discrepancy with the equivalence principle?
Post by: Colin2B on 28/04/2017 08:56:19
If you travel one metre at a constant speed that is held relative to a longer or shorter 'variable' second.  Then the distance remains the same, and it just takes a longer or shorter amount of 'time' to travel that metre.

That is a very pertinent point expressed with clarity. No one has as yet detected length contraction. If you maintained that clarity you would make much better progress.
I thought length contraction had been observed with gold ions, I'll try to find a ref.
Timey's point is a valid one. Most people assume length contraction and time dilation are separate phenomena but they are really the same. As soon as you start travelling you measure the distance to your objective as shorter, but the reason is that the clocks in the 'rest frame' are no longer synchronised for you. Also an observer in the rest frame sees clocks at each end of your ship as unsynchonised, hence measures it as contracted in length. Our measurement of time and distance are inter-related. See also Schwarzschild.
PS I forgot to add that this is why GR treats spacetime as one item via the spacetime interval.

Title: Re: Is there a discrepancy with the equivalence principle?
Post by: jeffreyH on 28/04/2017 09:57:07
If it has been detected I would be very interested in the reference.
Title: Re: Is there a discrepancy with the equivalence principle?
Post by: Colin2B on 28/04/2017 13:05:06
If it has been detected I would be very interested in the reference.
You have to remember that all subatomic interactions are indirectly observed, but the contraction is necessary to explain some of the interactions:

https://www.bnl.gov/rhic/physics.asp

I think there might be a wiki ref to it, but I can't find it. If you search on RHIC you should find more material.
Title: Re: Is there a discrepancy with the equivalence principle?
Post by: timey on 28/04/2017 14:17:47

However you measure time, it is the same for all systems at a given gravitational potential, but observably differs between points at different potentials.

Interpreting observation is indeed what it's all about, but you haven't produced an observation that suggests anything like inverted time dilatation.

That is because you, within your state of pedant, are refusing to accept the fact that where m=0 there can be no observer.

Therefore any observation of a 3rd aspect of time dilation where m=0 occurring contra directionally to GR altitude time dilation will only be observed indirectly.

I am suggesting that an indirect observation of this potential 3rd aspect of the time dilation phenomenon can be observed in the acceleration and deceleration of motion observed in the g-field and also within the frequency changes that occur for light in the g-field.
Title: Re: Is there a discrepancy with the equivalence principle?
Post by: alancalverd on 28/04/2017 17:34:19
We observe electromagnetic radiation, for which m = 0.
Title: Re: Is there a discrepancy with the equivalence principle?
Post by: jeffreyH on 28/04/2017 17:44:41
@Colin2B I'll let you know what I find. Should be interesting.
Title: Re: Is there a discrepancy with the equivalence principle?
Post by: timey on 28/04/2017 17:50:24
We observe electromagnetic radiation, for which m = 0.

Yes we do!

And electromagnetic radiation reduces in frequency when gravitationally shifting 'away' from M.
Title: Re: Is there a discrepancy with the equivalence principle?
Post by: alancalverd on 28/04/2017 19:39:28
Or, as a physicist would say, when observed from a higher gravitational potential than the source. Just like a clock. 
Title: Re: Is there a discrepancy with the equivalence principle?
Post by: timey on 28/04/2017 21:42:56
What relevance does this have to the proposed theory?

There is a system of mathematics called GR that states that a clock will tick faster at each elevation from M.
There is a system of mathematics called SR that states that at the orbital speed of each elevation this increase in the rate of time will be reduced.

There is also a system of mathematics within GR that describes how light shifts in the gravity field.  The light will always have a lower frequency in the higher potential no matter which direction it is travelling.

Therefore it really doesn't matter about what an observer observes from where-ever.
The proposed theory does not suggest that anyone will observe anything differently to conventional physics.
The proposed theory is concerning the acceleration deceleration of gravity.  We know that it occurs whether there is anyone observing or not.
Title: Re: Is there a discrepancy with the equivalence principle?
Post by: alancalverd on 29/04/2017 07:29:26
AFAIK GR explains, with minimal, testable assumptions, why observers at different gravitational potentials see the signals from each others' sources differently. And since an object moving at constant speed cannot be aware of the fact (i.e. there is no absolute motion) , SR likewise explains what observers moving relative to each other will see. The relative positions and speed of the observers is always crucial because there is no absolute observer and an observer fixed close to the source does not observe any effect. That's why it is called relativity.

So the relevance to the proposed theory is via Occam: the only assumption in relativity is the constancy of c, which is independently derived by Maxwell and demonstrated by experiment. There is no point in introducing any new hypotheses unless they explain something we already know but can't explain. 

Don't get too hung up on "observer". It's simply a generalised term for separating cause and effect, or source and detector. Doesn't imply any sentient beings.
Title: Re: Is there a discrepancy with the equivalence principle?
Post by: timey on 29/04/2017 12:41:09
Don't get too hung up on "observer". It's simply a generalised term for separating cause and effect, or source and detector. Doesn't imply any sentient beings.

It is you who keeps on bringing up the observer factor.  I am quite clear on the fact that position or motion relative to the g-field will change that which an observer observes.
See:
https://www.thenakedscientists.com/forum/index.php?topic=70259.msg513174#msg513174

Quote
There is no point in introducing any new hypotheses unless they explain something we already know but can't explain.

Yes - this being the point of my model.

So without detracting from the relativity of relativity, can we now talk about light changing frequency in the gravitational field in terms of my hypothesis, and can we now talk about the acceleration and deceleration of the motions of m in relation to M in terms of my hypothesis?
Title: Re: Is there a discrepancy with the equivalence principle?
Post by: alancalverd on 29/04/2017 23:14:54
The conventional equations for gravitational redshift and gravitational attractive force are established to a reasonable level of experimental precision and do not invoke any new hypotheses.

What is the anomaly you are trying to resolve?
Title: Re: Is there a discrepancy with the equivalence principle?
Post by: timey on 30/04/2017 03:13:35
There is no anomaly with the conventional equations for gravitational redshift and gravitational attractive force.

But it is a known fact that there is no description of physical cause and effect mechanics for gravitational acceleration.

This is the anomaly I seek to resolve.
Title: Re: Is there a discrepancy with the equivalence principle?
Post by: alancalverd on 30/04/2017 12:27:51
Aha!

Something to consider is the effect of interposing a third mass. https://en.wikipedia.org/wiki/Cavendish_experiment describes the classic method for measuring G, but what happens if you place a slab of metal between the large and small balls? Does the gravitational field add or subtract? If it adds, (a) we are looking for particle or quasiparticle that originates in matter and can penetrate matter without attenuation, but can interact with matter in a sucking mode or (b) mass does something to spacetime.

I think that the existing Einstein model of spacetime warping gives the correct answer without invoking spooky particles, but I'd be interested in your thoughts.
Title: Re: Is there a discrepancy with the equivalence principle?
Post by: timey on 30/04/2017 14:43:25
I also think that the existing Einstein model of spacetime warping gives the correct answer without invoking spooky particles, but I do think that a few minor adjustments of interpretation of observation can be made in order to describe why.

My hypothesis is that the phenomenon of time is energy related.

Frequency is energy related where frequency and energy are proportional to each other.  What I am suggesting is that where frequency increases this is because there has been an increase in the rate of time due to an increase in energy, and that where wavelength is inversely proportional to frequency, the lesser length of the wavelength (as compared to the length of the previously lower energy, lower frequency wavelength) is time related rather than distance related.

I am suggesting that Debroglie matter waves, all emitting sources, and already emitted light shifting in frequency can follow this remit.

Before I put this into the context of both gravitational attraction and the accelerations and decelerations observed of the gravitational field, is there any part of this post that needs clarification for you to understand it? 
Title: Re: Is there a discrepancy with the equivalence principle?
Post by: alancalverd on 30/04/2017 19:23:05
As far as gravitational energy is concerned, yes there is a clear relation between gravitational potential difference and observed time. But the concept of energy is simply "the scalar which is conserved", so it isn't clear how the potential energy of a battery, for instance, has any effect on time inside or outside the battery. And during a mass-energy exchange such as pair production, positronium collapse and gamma emission, what happens to time?

You might consider the stability of neutrons, having a 15 minute halflife in free space but being apparently infinitely stable in some (but not all) nuclei. That might lead to a classical model of nuclear stability, and if it is predictive, you might have evidence for the value of your hypothesis.
Title: Re: Is there a discrepancy with the equivalence principle?
Post by: timey on 30/04/2017 22:59:57
Alan you may well be correct.  I am not all that up on the intimate details of how particles interact, all I can do here is tell you about the patterns that I have recognized as being a possibility.
I came upon this hypothesis of mine by thinking about what time is doing where there isn't any mass.

This led to my viewing the phenomenon where a clock's frequency is observed to change as being energy related, and viewing the fact that a person aging in keeping with their clock can be due to the same addition or subtraction of energy that is occurring at differing gravity potentials via the equation mgh - where there 'may' be a possible argument for potential energy being responsible for a change in energy level to cause the change in frequency of electron transitions for a clock observed to have a differing frequency from the clock in the potential one is viewing from.

This then led me to examining the fact that if one measures the clock observed in another potential by the rate of time one is observing occurring in that potential, then the clock in the other potential will be ticking at the same rate one's own clock is in the potential one is viewing the other clock from.

Turning this thought upon its head, now I go to the black body experiment and look at how Planck's h constant is a joules per second measurement.
E=hf.  E changes with changes in f, but h remains constant.
But there is a possibility to do things differently here.

By stating frequency as being indicative of the rate of time occurring for an emitting source, we can hold frequency as constant in the face of variable seconds.  Now Planck's h constant will be variable because joules per second is held relative to a variable second where the change in length of a second is indicated in the change of frequency.
This system has negated the quantum nature of energy levels.

Now that the frequency of the light being emitted from the black body has been linked to changes in energy that are not quantised, we can examine light shifting frequency with changes in gravitational field energy, where the equation mgh can describe the changes in energy for the shifting light because where m=0 the changes in energy are just due to changes in g and h.

If we now say that the gravitational field itself also is an m=0 scenario, we can attribute the gravitational field energy as causing a 3rd aspect to the time dilation phenomenon where m=0.  This 3rd aspect is in addition to the known time dilation's of altitude and motion where m doesn't =0.
This 3rd aspect of the time dilation phenomenon is related to the gravitational field energy which is greater nearer to mass.  This 3rd aspect of time dilation, that will never affect the rate of time for a clock or observer, has a faster rate nearer to mass.
Although this 3rd aspect of time dilation will never affect the rate of time for a clock or observer it will affect the rate of motion a body of m will experience in the gravitational field of M.  It will decelerate the motion of m away from M, and accelerate the motion of m towards M. i.e. gravitational acceleration.
This 3rd aspect of the time dilation phenomenon being the physical cause of gravitational acceleration.

This is just half the story because if potential energy is increasing the frequency of electron transitions of a clock in the higher potential then the frequency of the magnetic moments of the electrons will also be increased.
This being the physical cause for gravitational attraction.

Leading to a whole bunch of reinterpretations of observation that go on to result in my cyclic model of the universe.   
Title: Re: Is there a discrepancy with the equivalence principle?
Post by: nilak on 01/05/2017 00:31:18
For a photon isn't the potential energy Ep=E/c^2 gx=hf/c^2 gx?
x is the height. h-Planck's constant.
I have indicated that E could remain constant, because f is held constant (only time rate changes) , but you are saying we should vary h. I'm not sure about this.
Title: Re: Is there a discrepancy with the equivalence principle?
Post by: timey on 01/05/2017 02:42:55
I am referring to the Planck Einstein relation.

If one holds frequency constant in the face of variable seconds, this will result in joules per variable second and the h constant will as a result not be a constant of quantised energy packets, but will result in a continuum.
Title: Re: Is there a discrepancy with the equivalence principle?
Post by: nilak on 01/05/2017 10:47:38
I am referring to the Planck Einstein relation.
Me too. The above relation was only for potential energy.
Quote
If one holds frequency constant in the face of variable seconds, this will result in joules per variable second and the h constant will as a result not be a constant of quantised energy packets, but will result in a continuum.
Here is a thought experiment. A spaceship moves at constant speed and a light beam is sent from an outside source that hits the spaceship.
An outside observer will see the frequency relative to a stationary point (relative to the source and the observer), constant. The wavelength will also be constant. If we think classically, the amplitude is also constant. In QM the photons will have constant energy in the observer's frame. The observer can also measure the frequency that wave crests hit the ship. This frequency will be higher, but the amplitudes will be the same. For the outside observer, there will be a difference of velocity between the ship and the photons. That means the photons will hit with more momentum. An observer on board will measure a higher frequency photons, and will assume they have higher intrinsic energy. If he wants to define f constant, he will need to increase h. For a classical wave, it will also measure a higher amplitude.
This shows how a classical wave energy can appear to be proportional to frequency.
Title: Re: Is there a discrepancy with the equivalence principle?
Post by: alancalverd on 02/05/2017 23:23:50
You cannot view photons as not being quantised, because we can count them and they are!
Title: Re: Is there a discrepancy with the equivalence principle?
Post by: timey on 03/05/2017 00:09:22
Without detracting from the art of photon counting wot-so-ever, the system that I suggest results in a compatibility between quantum and GR.

The physics books I read say that 'we' can't currently do compatibility between quantum and GR, but that 'we' would very much like to be able to. 
Title: Re: Is there a discrepancy with the equivalence principle?
Post by: timey on 03/05/2017 17:11:41
Ok then, lets examine photon counting...

The first observation is that photon counting occurs via time based units.

Photon flux is measured per unit time where the unit of time is a standard second.
Photon intensitiy is a count per steradian per standard second.
Photon radiance is a count per square metre per steradian per standard second.
Photon irradiance is a count per square metre per standard second.
Photon exitance is a count per square metre per standard second

The energy of a single photon at wavelength λ is Qp = h⋅c / λ with h = Planck's constant and c = velocity of light.

Where:
The speed of light c is a distance held relative to a standard second.
Planck's h constant is a count of joules per standard second.
Wavelength is a 'spatial' measure of distance between wavecyles as determined by the frequency of wavecycles where frequency is a count of wavecycles per standard second.

Let's now look at the equation where:
The energy of a single photon at wavelength λ is Qp = h⋅c / λ

h and c have been defined, so to further examine wavelength:

 λ=h/p
p=h/ λ

Examining p further:

p=h*vbar
where:
vbar=va
where in the case of light:
v=c
but what is 'a' equal to?

If p=h/ λ, but can also be calculated as p=h*vbar, which can also be expressed as p=h*(v*a), then what we are looking at is a relationship between wavelength and (v*a).
So - because the dfference between the calculations amounts to a mulitlication and division of h, p=h/ λ is the inverse of p=h*(v*a), h*(v*a) must be represetative of frequency where E=hf

On the one hand wavelength is a 'spatial' measure of distance between wavecyles, as determined by the frequency of wavecycles, where frequency is a count of wavecycles per standard second.
On the other hand (v*a) is the measure of the speed of light c, where c is a distance held relative to a standard second, multiplied by 'a' where 'a' must be the measure of the change in frequency.

My hypothesis simply states that wavelength is a 'temporal' measure of distance between wavecyles as determined by the frequency of wavecycles where frequency is a count of wavecycles per variable seconds.
 
One can hold frequency constant in the face of variable time based units and as a result Planck's h constant will be variable, and a continuum...
Or alternatively one can state that 'a' is equal to a change in the rate of time, where λ=h/p and p=h*(v*a).

Going back to:  The energy of a single photon at wavelength λ is Qp = h⋅c / λ

Considering wavelength as a 'temporal' measure of distance between wavecyles where:
Instead of more or less distance being covered in a constant time, it is the same amount of distance being covered in faster or slower time...

This is now dividing the sum of h*c by the remit of a wavelength that is representative of a faster or slower rate of time where:

The point particle model of the photon and its wave-function will be compatible with GR mathematics.

Anyone investigating this representation of the maths will understand that this changes the remit of Hubble's velocity related red shift distance correlation where Einstein's equations of GR will be valid as per a contracting model.
Title: Re: Is there a discrepancy with the equivalence principle?
Post by: alancalverd on 03/05/2017 17:43:23
One here, one there...who mentioned time? One here, now another one....OK, so some time has elapsed but it doesn't matter how much: as long as the events are separated by space or time, we can count them. And they are, so photons are quantised. 

But tell me more about p = h*vbar and vbar = va. What do vbar and a represent?
Title: Re: Is there a discrepancy with the equivalence principle?
Post by: timey on 03/05/2017 21:21:07
You may read about p = h*vbar here...

https://en.wikipedia.org/wiki/Planck%E2%80%93Einstein_relation

Quote
Combining de Broglie's postulate with the Planck–Einstein relation leads to p=h*vbar
(the v has a bar over it that I cannot represent in this format)

A bar over v is indicative of v changing where the changes to v are caused by a, a being acceleration...
But light travels at c, c being a constant distance held relative to a standard second, so there is no change to v, and where there are changes to p via h*(v*a) they are related to changes in frequency, and therefore to changes in energy.

who mentioned time?

Physics did.  All the measurements of photon counting are held relative to the unit time period of a standard second.

OK, so some time has elapsed but it doesn't matter how much: as long as the events are separated by space or time, we can count them. And they are, so photons are quantised.

Electrons are quantised, where it does matter very much to know where an electron is and how fast it is moving.
We know that time has elapsed between positions and velocities where (it would seem to me) with regards to distance travelled, to matter very much that one undestand how much time has elapsed.
This being because under the current remit of calculating these matters it is only possible to know the probability of how events are separated by space or time.

Where it is noted that probability is calculated via perturbations of time.
Title: Re: Is there a discrepancy with the equivalence principle?
Post by: alancalverd on 04/05/2017 00:07:57
Mystery revealed. Your "vbar" is not a v with a bar, but "nu" bar - wavenumber. Nothing to do with velocity or acceleration at all. We sometimes use a bar to indicate an average, but not in this instance. 

When I see two spots on an x-ray film (i.e. most days) I know that there were two quantum events. Each time I hear a click from a geiger counter, I know there was a single quantum event. I can compare two radiation sources by counting the number of clicks produced by A in the time taken for B to produce 100 clicks - no time standards required, and if B is a gram of radium I have measured the activity of A in hectocuries. 

https://en.wikipedia.org/wiki/Photoelectric_effect gives a good account of the experimental investigations that demonstrated the quantum nature of light, with no reference to "standard seconds" or anything else assocated with the measurement of time.
Title: Re: Is there a discrepancy with the equivalence principle?
Post by: timey on 04/05/2017 02:58:31
Yes  Alan, that is correct...

https://www.google.co.uk/search?q=what+is+a+wave+number&oq=what+is+a+wave+number&aqs=chrome..69i57j0l5.8926j0j7&sourceid=chrome&ie=UTF-8

Quote
In the physical sciences, the wavenumber (also wave number) is the spatial frequency of a wave, either in cycles per unit distance or radians per unit distance.
It can be envisaged as the number of waves that exist over a specified distance (analogous to frequency being the number of cycles or radians per unit time).

The spatial frequency of a wave is dependent upon the number of cycles per unit of distance, i.e frequency related and therefore subject to acceleration deceleration of rate.
And the number of cycles per unit distance is dependent upon the number of cycles per unit time, i.e.frequency related and therefore subject to acceleration deceleration of rate
All changes to these parameters are dependent upon an acceleration or deceleration of frequency, and frequency is dependent on energy.
The unit of distance is dependent upon velocity in relation to the unit of time, and the unit of time being used is a second, specifically the standard second.
However, GR is quite clear on the fact that different rates of time are occurring, and that these different rates of time occur independently of each other and simultaneously to each other...
...And this is not reflected in the practice of measuring all phenomenon held relative to a static unit of time.

When I see two spots on an x-ray film (i.e. most days) I know that there were two quantum events. Each time I hear a click from a geiger counter, I know there was a single quantum event. I can compare two radiation sources by counting the number of clicks produced by A in the time taken for B to produce 100 clicks - no time standards required, and if B is a gram of radium I have measured the activity of A in hectocuries. 

And how does that measure up to the amount of time it takes for the cesium atomic clock that I'm placing in your laboratory to complete 9,192,631,770 wave cycles?

Lets get a mate of yours to compare clicks he hears on a geiger counter at a higher gravity potential to your lab, where you are watching your mate conduct these measurements.
As far as your mate is concerned the atomic clock that I have placed in his lab is completing 9,192,631,770 wave cycles and the measurements that he saw you compare to the wave cycles of your clock, when he was with you in your lower potential lab, when made by himself in the higher potential are no different to the proportions of the measurements you made.
You on the other hand are observing that your mates clock in the higher potential is completing 9,192,631,770 wave cycles, while your clock has only completed 9,092,631,770 wave cycles.  A 100,000 wave cycle difference.  Your mates geiger counter is making clicks that are closer together than the measurement you took in your lab.
Your mate looks down and observes that his atomic clock has completed 9,292,631,770 wave cycles by the time your clock has completed 9,192,631,770 wave cycles.  He asks you to measure your gieger counter again, and as you do he measures that there is more time between the clicks of your gieger than there was when he measured his. 
Then your mate has a thought and asks you to increase the electric voltage (energy) of the ionized events that your geiger counter is counting.  When your geiger counter is making clicks at the same rate as his he asks you to stop.
He asks you by what measure of energy did you increase your ionized events by...and wonders if the ionized events occurring in his higher potential have been increased in energy due to his position in the higher potential.

Thanks for the link.  It's basically a brief synopsis of the more than several books that I have read on the subject that have given description involving an estimated 800 or so pages, if not more.

Quote
: wiki link
In 1887, Heinrich Hertz[2][3] discovered that electrodes illuminated with ultraviolet light create electric sparks more easily.

Does this per chance mean that more sparks are observed held relative to a unit of time, as compared to the amount of sparks created by electrodes illuminated with lesser energy photons when held relative to the same unit of time?
Title: Re: Is there a discrepancy with the equivalence principle?
Post by: alancalverd on 04/05/2017 06:48:05
You can associate a wavenumber with a standing wave, the arches of a bridge, the teeth of a comb....time or speed is not involved. You can measure distance with a standard stick, a surveyor's chain, King Edward's arm, or anything else you like - the modern definition of the meter is purely for convenience.

Not sure how you can introduce acceleration into a discussion on photons, which travel at a constant speed, both theoretically and experimentally. And note that neither the theory nor the experiment refers to any external timebase.

Quote
And how does that measure up to the amount of time it takes for the cesium atomic clock that I'm placing in your laboratory to complete 9,192,631,770 wave cycles?

Not at all.

Quote
Does this per chance mean that more sparks are observed held relative to a unit of time, as compared to the amount of sparks created by electrodes illuminated with lesser energy photons when held relative to the same unit of time?


No. It means that a smaller electric potential gradient will produce sparks.
Title: Re: Is there a discrepancy with the equivalence principle?
Post by: timey on 04/05/2017 15:17:10
You can associate a wavenumber with a standing wave, the arches of a bridge, the teeth of a comb

Where in the case of a Chladni plate the standing wave is caused by frequency.  The arches of a bridge are defined by the frequency of placement of supporting struts, the number of teeth of a comb defines how many hairs it can re-position, and the teeth of a cog defines the frequency of motions that a cog imparts to a system.

You can measure distance with a standard stick, a surveyor's chain, King Edward's arm, or anything else you like - the modern definition of the meter is purely for convenience.

But if one is moving at a constant speed of 1 meter per second and is asked to make a marks on an unmarked tape that they reel out as they move along for every second that passes on their clock, the rate at which the clock is ticking will affect the distance of the measurement.
The 'spatial' distance between the marks will differ with differing tick rates, the tick rates are frequency based, and the marks are synonymous to wave number.

Not sure how you can introduce acceleration into a discussion on photons, which travel at a constant speed, both theoretically and experimentally. And note that neither the theory nor the experiment refers to any external timebase.
I'm quite sure that I explained in great detail several posts ago that it is an acceleration or deceleration of the frequency of a photon that I reffer to, which has a relation to the wave number. wavelength=h/p, p=h*vbar, and vbar=v*a.

Not at all.

So - you are saying that geiger clicks cannot be measured for frequency via the tick rate of a clock?

Quote
:physics.info
In 1905, Einstein realized that light was behaving as if it was composed of tiny particles (initially called quanta and later called photons) and that the energy of each particle was proportional to the frequency of the electromagnetic radiation that it was a part of. Recall from the previous section of this book that Max Planck invented the notion of quantized electromagnetic radiation as a way to solve a technical problem with idealized sources of electromagnetic radiation called blackbodies. Recall also that Planck did not believe that radiation was actually broken up into little bits as his mathematical analysis showed. He thought the whole thing was just a contrivance that gave him the right answers. The genius of Einstein was in recognizing that Planck's contrivance was in fact a reasonable description of reality. What we perceive as a continuous wave of electromagnetic radiation is in reality a stream of discrete particles.

I am making the suggestion that the length of a unit of time, i.e. the rate of time of that system, is inversely proportional to the energy of the system, i.e. more energy will increase the timing of the system.   An emitting source is a system.  A receiving source is a system.  Energy is transferred from the emitted light to the receiving system. 
If one considers that the frequency resulting from the energy of a system is indicative of the systems 'timing' then via this new means of mathematical analysis one will find that by holding frequency as constant in the face of variable units of time that the value of Planck's h constant will change with each change in length of time unit and becomes a variable continuum.   

This does not change any observation.  But it does change the current view of what time is and gives physical cause to the phenomenon.  This change to the current view of what time is becomes useful in describing phenomenon that in conventional physics have no cause and effect mechanics (such as the accelerative and attractive force of gravity) and my model of a cyclic universe.

There isn't really any point in my continuing this discussion with you, is there Alan?
You have a certain view of the phenomenon of time being a measurement imposed upon a system, and I am viewing time as being integral to a system as a reactive to energy.
The view I take is compatible with relativity where the remit of relativity results in the fact of differing rates of time occurring independently of each other and simultaneously to each other, where all I am doing is deepening that theory.
However, I don't seem to be able to persuade you to explore a differing view, so lets just call it a day.
Title: Re: Is there a discrepancy with the equivalence principle?
Post by: alancalverd on 05/05/2017 00:50:18
You seem to have lost the plot entirely, or to be speaking an alien language. Let's give it a rest.
Title: Re: Is there a discrepancy with the equivalence principle?
Post by: guest4091 on 05/05/2017 18:10:55
If it has been detected I would be very interested in the reference.
The electric field strength increases for fast electrons as they contract, and is detectable.
check this link..
https://conf-slac.stanford.edu/sssepb-2013
select this:
Lecture 1. Lecturer Zhirong Huang
Title: Re: Is there a discrepancy with the equivalence principle?
Post by: timey on 05/05/2017 22:13:39
That is good news indeed Alan.  I'm happy to have your agreement to call it a day, because by all logical deduction one might expect that in considering a 'New Theory' that one may encounter concepts that are alien for the reason that one hasn't heard these concepts before.
And to say so I'm not all that keen on being told that I seem to have lost the plot, especially by someone who's idea of 'mathematical help' incorporates introducing the arches of a bridge and the teeth of a comb into a conversation about juggling the maths of the Planck Einstein relation.  It leaves one with the feeling that one isn't really being taken seriously which would be detrimental to my notions of friendship and it pleases me that such detriment has now been averted.  I look forward to speaking with you elsewhere on subjects that do not incorporate introducing ideas that are foreign to the current status quo.

*

For anyone else who is actually interested in a potential means to unify the point particle model with the wave function model for a compatibility between quantum and GR...

E=hf, where E is the energy of the particle, h is Planck's h constant, this being a set number of joules per second, and frequency is the number of wave cycles that occur per second, where the distance between these wave cycles is known as wavelength.  The remit of the equation E=hf denotes that when f changes, E will change.

I am now going to show how this equation can be juggled so that the resulting value of E remains the same but Planck's h constant is rendered variable and can be described as a continuum of linear progression.  To put this proposed interpretation into the context of my model I will hold the measurements of E relative to the gravitational redshift equation and light travelling from our position of gravity potential away from Earth...
It is of little consequence to this description which frequency of light we start out with.  What is of consequence is that we have measured the frequency and therefore the distance of a wavelength by the remit of the clock that is in the gravity potential that the light source emitter is in, and for the purpose of this description we will say that these initial measurements of the frequency of the light when being emitted in our position of gravity potential have been held relative to the cesium standard. (this is just a convenience, we could hold frequency relative to any length of second and the differences in frequency due to gravitational shift would remain proportional)

In that the initial measurement is holding frequency relative to the cesium standard, and Planck's h constant is holding joules per second relative to the cesium standard, we can start out with the equation E=hf, but where the gravitational redshift equation is describing the reduction of the lights frequency in the higher potentials we will be holding frequency relative to seconds that are becoming longer than the cesium standard.
This being a 3rd aspect to the time dilation phenomenon that my model has added which applies only where m=0, and is added on the basis that we are not going to be using the concept of relativistic mass at-all in my model, or the concept of SR motion related time dilation and SR length contraction/space dilation to describe the propagation of light in space...
In order to know by how much a 3rd aspect time dilation second has become longer than the cesium standard all we have to do is hold the number of wave cycles as constant by extending the number of micro seconds of the standard second so that the number of wave cycles completed within the extended second of each elevation remains the same at each elevation.

The equation E=hf is now holding frequency as constant.  But this does not mean that we are considering E to be constant.  What we are looking for is that the energy decreases of the redshifted light remain inversely proportional to the increases in wavelength and that our values remain within the remit of conventional and observed physics.
Clearly we can accomplish this within the equation E=hf where f is now being held constant by making h the variable...

This remit results in a continuum of seconds becoming linearly longer, a continuum of E becoming linearly lesser, and a continuum of the value by which the number of joules linearly decreases as E decreases.
And the implications of this result are that wavelength has become constant as a distance, where it takes a longer amount of time for the red shifted wave cycles to travel this constant distance as E is reduced by gravitational shifts...
This does not have any impact on the structure of the conventional use of h in physics.  One may calculate in conventional manner using Planck's h constant, but in the understanding that in doing so one is juggling a function of time dilation...
The impact that this remit does have on the structure of the mathematics is that one will not need to use probability to calculate beyond the Uncertainty Principle.  This method of interpretation means that one can determine position and velocity simultaneously...
(I would care to discuss how an equation like this Qp = h⋅c / λ  can be altered for a description of a particle of mass, where under the remit of my proposal, dividing by the Debroglie wavelength of a particle is in as much as dividing by the rate of time of the particle, which should give position... in relation to how an decrease/increase in the energy/frequency of that particle will increase/decrease the particles wavelength)

Given that someone has understood what I have said above, it would then be possible for me to move on and explain how the equation pe=mgh is significant, and differing where m=0 and where m doesn't =0, in order to further describe why a 3rd aspect of the time dilation phenomenon 'may' be possible...
And in that a 3rd aspect of the time dilation phenomenon 'may' be possible, describe the 'necessary by default of the proposed addition' alteration to the interpretation of why the rate of time is predicted, and observed to be increased (i.e. shorter length of seconds) compared to the clock in the lower potential, for the clock and all m with that clock in the higher gravity potential.
Title: Re: Is there a discrepancy with the equivalence principle?
Post by: Ethos_ on 06/05/2017 00:40:19


E=hf, where E is the energy of the particle, h is Planck's h constant, this being a set number of joules per second, and frequency is the number of wave cycles that occur per second, where the distance between these wave cycles is known as wavelength.  The remit of the equation E=hf denotes that when f changes, E will change.



My friend, I'm not sure you understand dimensional analysis. Let's examine your simple equation: E=hf

In SI units E is measured in Joules: kg* m^2* sec^-2
In SI units h is expressed as kg * m^2 * sec^-1
and f is expressed in sec^-1

(kg * m^2 * sec^-2) = (kg * m^2 * sec^-1) * (sec^-1)

Defining this equation dimensionally correct demands that all values; kg, m, and sec, remain consistent on both sides of the equation. Allowing seconds to become shorter or longer depending upon which side of the equation they reside destroys the dimensional equivalence and undermines the accuracy of the formula.

The value of seconds must remain the same on both sides of the equation if you are to use this formula to validate your theory.
Title: Re: Is there a discrepancy with the equivalence principle?
Post by: timey on 06/05/2017 01:26:01
I'm sorry if I wasn't being clear enough...but in holding the gravitational shifted frequency constant by adding the necessary microseconds to the length of a second to ensure that the same number of wave cycles complete in the longer seconds of each elevation, the value by which the number of joules linearly decreases as E decreases is then due to being held relative to the longer second 'because' f is being held constant.
Therefore I think that hf can be of dimensional accuracy under the remit I propose, and this can be determined by the fact that completing the equation holding either h or f constant will result in the same value of E.
Title: Re: Is there a discrepancy with the equivalence principle?
Post by: Ethos_ on 06/05/2017 04:54:55
I'm sorry if I wasn't being clear enough...but in holding the gravitational shifted frequency constant by adding the necessary microseconds to the length of a second to ensure that the same number of wave cycles complete in the longer seconds of each elevation, the value by which the number of joules linearly decreases as E decreases is then due to being held relative to the longer second 'because' f is being held constant.
Therefore I think that hf can be of dimensional accuracy under the remit I propose, and this can be determined by the fact that completing the equation holding either h or f constant will result in the same value of E.
Timey,.....you can't change the value of time on one side of the equation and leave the other values for time as they were. If a kg is more or less than a kg, it's not a kg. The same goes for length and time, the equation will not be dimensionally balanced if you only change the value of time on one side.

If we rewrite the formula: (kg * m^2 *sec ^--2) = (kg * m^2 * sec^-1) * (.90 sec^-1) we have invalidated it because the value of time on one side must equal the value of time on the other.

If you are to provide evidence for your theory, you'll need to offer another equation in support of it because (E=hf) will not meet the test.


Title: Re: Is there a discrepancy with the equivalence principle?
Post by: timey on 06/05/2017 14:39:17
Firstly Ethos I wish to thank you because this is exactly the type of discussion that I wish to be engaged in, where I will be learning more about the construct of mathematics no matter if my hypothesis proves to be viable or not.

Timey,.....you can't change the value of time on one side of the equation and leave the other values for time as they were.

This is the exact reasoning that leads me to the notion of juggling the maths in the first place, where I am exploring the notion that the equation under the conventional interpretation actually does lead to a dimensional imbalance that results in the Uncertainty Principle, and that this imbalance "may' be due to holding h and E relative to the same length of second where the physical action of changes in frequency is being caused by rates of time that are not equal to the length of second that h and E are being held relative to.

The same goes for length and time, the equation will not be dimensionally balanced if you only change the value of time on one side.

Length is entirely dependent on the rate of time a constant speed is held relative to.  This can be evidenced by the fact of 'spatial' changes in wave length when wave cycles are held relative to a static length of second.  By holding frequency constant as a result of changing the length of a second, the 'spatial' wavelength remains a constant distance that wave cycles travelling at constant speed take a longer/shorter amount of time to complete, where the possibility then arises for a consideration that the 'spatial' changes that result from holding wave cycles relative to a static length of second are a function of time

If we rewrite the formula: (kg * m^2 *sec ^--2) = (kg * m^2 * sec^-1) * (.90 sec^-1) we have invalidated it because the value of time on one side must equal the value of time on the other.

Why must joules remain (kg*mass^2*sec^2)?  If the time is different on one side then it can be matched on the the other side can't it?

(I'm not understanding the change in the equation you have made.  First you have said (kg * m^2 *sec ^--2), but you are saying that this is equal to (kg * m^2 * sec^-1) * (.90 sec^-1).  can you explain to me what the difference is between * ^--2, and, sec^2 is?  My understanding is that *sec^2 is per second squared.  And can you explain to me what this sec^-1 means?)

Where light is concerned it does not have any rest mass, so kg*mass^2 is then purely related to E.  Conventional physics calculates a relativistic mass for light that clearly will be reducing as E is reduced.  The means of calculating the changes in E as changes in relativistic mass involve, for the red shifted light, a conversion of kinetic energy into potential energy where potential energy does not contribute to relativistic mass value. (could well be that my understanding here needs to be expanded)

Clearly if my hypothesis is going to be viable then there will be a means of making a mass energy equivalence calculation that is inclusive of a variable length second, although I'm not clear on how to work the premise of (kg*mass^2*sec^2) backwards from E, so perhaps you might help me out in understanding the construct...

If the E of a red shifted frequency must be accompanied by a (kg*mass^2*sec^2) equation, where the higher altitude is the cause of the gravitational red shifted light, does the *sec^2 part of the equation use the conventional physics 'shorter' length of second associated with the higher altitude?
Title: Re: Is there a discrepancy with the equivalence principle?
Post by: Ethos_ on 06/05/2017 16:38:16
Firstly Ethos I wish to thank you because this is exactly the type of discussion that I wish to be engaged in, where I will be learning more about the construct of mathematics no matter if my hypothesis proves to be viable or not.

I will be honored to help you timey, but I request one consideration of you to begin. I'm old and at present, very sick. Allow this discussion to proceed slowly because I'm not sure how much energy I'll be able to invest. If you would prefer, we could do this via private message or we can continue here in this thread, I'll leave it up to you.
Title: Re: Is there a discrepancy with the equivalence principle?
Post by: Ethos_ on 06/05/2017 17:28:09
Let's start with some basics:

The formula: E=hf is a simple proportion and represents the relationship that Energy has to Planck's constant and frequency. While it is true that factors can be juggled, the individual factors; (mass, length, and time) must retain their individual integrity. Thus, all masses are calculated in kg, all lengths are calculated in meters, and all seconds are calculated in what our frame has measured as the second. The equation: (kg * m^2 * sec^-2) represents Energy of one Joule.

BTW, ........your question about (^-2) ..............allow me to elaborate.

(kg * m^2 * sec^-2) is the same as: [kg * m^2 / (sec^2)] ............it's just easier to write using ^-2.

When you see (sec^-2) it's simply means that the former figures are divided by (sec^2).

The equation: E=hf can also be written as: E=(h/sec) because frequency is (1/sec).


Title: Re: Is there a discrepancy with the equivalence principle?
Post by: timey on 06/05/2017 18:40:53
Ok, thanks for the description...

Obviously I am understanding that E=hf is a simple proportion and represents the relationship that Energy has to Planck's constant and frequency, and am taking the situation a stage further by suggesting that frequency has relationship with time.
I also understand that the individual factors; (mass, length, and time) must retain their individual integrity, but note that all 3 are subject to variability under the remit of relativity...

In making the addition of a 3rd aspect to the time dilation phenomenon that applies where m=0, (i.e. open space and light propagating through open space), where relativity is already stating that time can be dilated/contracted by position in the gravity potential, (aspect 1(GR)), and that time can be dilated/contracted by relative motion to the gravitational field, (aspect 2(SR)), I am attempting to extend the remit of relativity in order to unify the wave function with the point particle model.

And - as  you have said, "it is true that factors can be juggled".

Thus, all masses are calculated in kg, all lengths are calculated in meters, and all seconds are calculated in what our frame has measured as the second.

Which does not reflect the fact that the rate of time is not the same in all frames of reference.  Conventional physics uses SR Lorentz transformations to calculate differences in rates of time and length.  GR uses equations related gravitational shift to calculate differences in the rate of time, and relativity uses the concept of relativistic mass to calculate changes in mass in order to upkeep the bookkeeping required to maintain the universal speed limit of c, the speed of light.

All of these equations are holding E, f, h, and c relative to a static length of second, as measured in our frame of reference, therefore conventional physics is already juggling different rates of time, interpretations of length, and values of masses, and these differences are appearing on both sides of an equation.

I don't see any reason not to juggle h and f under a differing remit, (and indeed will go on to explain further how the value of a linear progression of a continuum of joules can be obtained via the data of the ultra violet catastrophe).  The only criteria is that the end result remains the value of observation and experimental evidence while providing a deeper description of cause and effect.

The equation: E=hf can also be written as: E=(h/sec) because frequency is (1/sec).

I'm not sure that it can actually.  If h, being a constant, is divided by 1 second that is a constant, then E will always remain the same value.  It is the changes in frequency that are causing the value of E to differ.

I will be honored to help you timey, but I request one consideration of you to begin. I'm old and at present, very sick.  Allow this discussion to proceed slowly because I'm not sure how much energy I'll be able to invest. If you would prefer, we could do this via private message or we can continue here in this thread, I'll leave it up to you.

I am grateful that you will help me, and I am very sorry to hear that you are not very well.  Hope you get better soon.  I will leave it up to you whether or not you want to make your contributions privately or on the thread.  A lot of people view the thread.  It is my hope that some of the things I say may ring bells with thought processes that others are investigating, so I will carry on on the thread either way.
Title: Re: Is there a discrepancy with the equivalence principle?
Post by: Ethos_ on 06/05/2017 18:57:59


I'm not sure that it can actually.  If h, being a constant, is divided by 1 second that is a constant, then E will always remain the same value.  It is the changes in frequency that are causing the value of E to differ.
Allow me to explain:

Where the Joule  = (kg * m^2 / sec^2)
and h = (kg * m^2/ sec)
if we multiply (kg * m^2/sec)
by f = (1/sec)
(kg * m^2 * sec^-1) * (sec^-1)
we arrive at (kg * m^2 * sec^-2) which equals 1 Joule

Quote from: timey

I am grateful that you will help me, and I am very sorry to hear that you are not very well.  Hope you get better soon. 
Actually timey, my health is not expected to improve but I appreciate the well wishes anyway.
Title: Re: Is there a discrepancy with the equivalence principle?
Post by: jeffreyH on 06/05/2017 19:16:16
If it has been detected I would be very interested in the reference.
The electric field strength increases for fast electrons as they contract, and is detectable.
check this link..
https://conf-slac.stanford.edu/sssepb-2013
select this:
Lecture 1. Lecturer Zhirong Huang

That link doesn't work.
Title: Re: Is there a discrepancy with the equivalence principle?
Post by: Ethos_ on 06/05/2017 19:49:46

And - as  you have said, "it is true that factors can be juggled".

And BTW, the only juggling I was referring to was as following examples:

E=hf
E/h=f
E/f=h
E/hf=1
Title: Re: Is there a discrepancy with the equivalence principle?
Post by: timey on 06/05/2017 21:26:39
Actually timey, my health is not expected to improve but I appreciate the well wishes anyway.

In that case the fact that you are willing to spend some of your time engaged here with me has that much more significance to me... and I can promise you, on the basis that you are not going to get cross with me because I will examine everything you say to me upside down, inside out, and back to front, this being the way I approach all knowledge, and attempt to juggle mattters to the tune of my hypothesis as I do so, that I will employ due logical thought process at every juncture and try my level best to be entertaining, if only as an example of an interested mind.

Where the Joule  = (kg * m^2 / sec^2)

Ok here you are saying that kilogram, multiplied by mass squared, divided by second squared, equals Joule

Am I to take it that because kilogram is a measure of weight and is multiplied by a value of mass in the equation, that kilogram must be a measure of g?
Title: Re: Is there a discrepancy with the equivalence principle?
Post by: Ethos_ on 06/05/2017 23:34:37


Ok here you are saying that kilogram, multiplied by mass squared, divided by second squared, equals Joule

Am I to take it that because kilogram is a measure of weight and is multiplied by a value of mass in the equation, that kilogram must be a measure of g?
That assumption would be incorrect timey. The kilogram is a measure of mass and is not to be confused with a measure of weight. The notation k refers to (kilo), a thousand, and the g in this case to grams and is not referring to gravity.

The notation: (kg * m^2 * sec^-2) is the formula for the Joule. The kg in this refers to a kilogram and the m^2 denotes meters squared. And remember what I said about sec^-2. This could also be correctly written as: (Kilograms times meters squared divided by seconds squared.)

Frankly, I'm astonished that you appear to be confused by these facts. Please don't take offensive to that remark, no insult was intended.









Title: Re: Is there a discrepancy with the equivalence principle?
Post by: timey on 07/05/2017 01:28:18
Not at-all Ethos.  However, I am not confused by these facts, I just quite simply don't know these facts.  Where it says "self educated since age of 11" beside my forum name, this actually means that I have had no schooling at-all since age 11.  My maths education stopped short at long division.  Although I have been reading about physics for getting on for 10 years now, without having a basis in algebra at-all, the maths of these books have passed me by in favor of the written explanations.
When I came up with my hypothesis it was my understanding that I could just give someone an explanation in words of what the theory incorporates and it would be no problem for that person to calculate from my description in words.  I personally still think this would be possible, but recognize after 2 years of trying that it's improbable.  Not because my model is incalculable but because people who are interested in exploring the boundaries of physics are busy with their own interpretations, and people who aren't exploring the boundaries of their own interpretations aren't interested in pushing the boundaries of physics.
When I started posting 2 years ago I was a busy person with a lot going on in my life and didn't have time to venture into learning maths.  Now I am not a busy person, I don't have anything going on in my life nor the desire to even leave the house, but I do have a lot of time to venture into learning the maths, and I am trying to learn because nobody else is going to calculate my ideas.
Don't get me wrong, the many written descriptions of physics I have read have given me a good education into what proportions the maths are describing, it just the symbolism such as m for mass or m for a meter, that I need to become familiar with, where the more basic the equation the less I've had explained due to the advanced level of the books I've been reading assuming that the reader has had at least a high school education in maths... ya dig?

I don't think there is any embarrassment in not knowing something, and the transition from not knowing to knowing is merely an instant in time and not something to be made a big deal out of.

So we are talking about a weight of energy, multiplied by a meter squared, divided by a second squared = joule
And Planck's h constant is a 'set' weight of energy, multiplied by a meter squared, divided by a second squared, and constitutes a constant number of joules.
E=hf is describing the proportions of the relationship between changes of frequency in relation to Planck's h constant and results in knowing the E of the light or the mass one is calculating.

It would of course be possible to make all of these considerations using a longer or shorter second, and although this is what I wish to explore, my hypothesis should be able to use a division or multiplication of some of the conventional maths by wavelength to introduce the time dilation factor in much the same way as the photon counting equation divides by wavelength where: Qp=h⋅c / λ. 
Title: Re: Is there a discrepancy with the equivalence principle?
Post by: Ethos_ on 07/05/2017 03:42:34

E=hf is describing the proportions of the relationship between changes of frequency in relation to Planck's h constant and results in knowing the E of the light or the mass one is calculating.
When calculating photon energy, (E=hf) is used because the photon has zero proper mass and using E=mc^2 is not applicable in that case. Knowing that the value of h is: 6.626 *10^-34 (J*s) and supposing that we have a frequency of 10 Hz we can multiply 6.626 *10^-34 by 10 resulting in 6.626 *10^-33 joules for the energy of a photon at 10 Hz frequency.

Remembering that f =(1/sec) and h=(kg *m^2/sec) our calculation becomes (10/sec) * (kg * m^2/sec) = 10(kg * m^2/sec^2) or 6.626 *10^-33Joules.
Title: Re: Is there a discrepancy with the equivalence principle?
Post by: Colin2B on 07/05/2017 06:28:58
Remembering that f =(1/sec) and h=(kg *m^2/sec) our calculation becomes (10/sec) * (kg * m^2/sec) = 10(kg * m^2/sec^2) or 10 Joules.
I think this is what Timey is trying to do - if seconds pass at, say, twice the rate relative to our gravitational potential i.e. each is = half of our seconds, then the calculation becomes 20J (10/0.5), but, to use wavelength as the modifying factor.
Title: Re: Is there a discrepancy with the equivalence principle?
Post by: Ethos_ on 07/05/2017 12:29:13
Remembering that f =(1/sec) and h=(kg *m^2/sec) our calculation becomes (10/sec) * (kg * m^2/sec) = 10(kg * m^2/sec^2) or 10 Joules.
I think this is what Timey is trying to do - if seconds pass at, say, twice the rate relative to our gravitational potential i.e. each is = half of our seconds, then the calculation becomes 20J (10/0.5), but, to use wavelength as the modifying factor.
Sorry for the typo, instead of the final answer being 10 Joules, my answer should have read 6.626 *10^-33 joules which I have corrected in my last post.

And yes, I think you're right about her thought on the matter. But when we construct the math to calculate these equations, we must remember that the rules of math are subject to our personal frame of reference. And for any observer, their frame will obtain accurate calculations using what they view as their familiar second, not shortened or lengthened by any measure. I'm not sure how one could mathematically overcome this and provide her with the mathematical evidence she desires.
Title: Re: Is there a discrepancy with the equivalence principle?
Post by: Colin2B on 07/05/2017 13:59:35
when we construct the math to calculate these equations, we must remember that the rules of math are subject to our personal frame of reference.
We also need to make the maths believable. As you know, you can use maths to create any scenario, but like writing a program the systems analysis part is key or gigo.
Simple eg a car can be described in maths as travelling at 30mph with a particular rpm and wheel size. If in the maths we double the wheel circumference the car will travel at 60mph, but it isn't believable unless there is a clear mechanism for the wheel size to change.
Before you can do the maths there has to be a clear system analysis of the mechanisms linked to observations and that, in my perception, is what Alan has been trying to do. Although I accept that Timey has a different perception and would consider the analysis as complete. Their positions appear to be irreconcilable.
Anyway, 'nuff said, I'll keep out and wish you both luck.
Title: Re: Is there a discrepancy with the equivalence principle?
Post by: Ethos_ on 07/05/2017 14:45:49

We also need to make the maths believable.
Absolutely Colin...................I suspect the only help I can provide for timey is assistance in understanding and navigating through the math with her. This is the very reason I have asked for us to progress at a pace which will allow her to understand the math. Without the ability to grasp what the math is telling us, I think timey will fail to understand the complexity of the difficult task which stands before us. As you have noted, the math must also be believable and that believability is only achieved through the experience offered us by experiment and the confirmation we achieve when repeatability of those experiments are recorded.
Title: Re: Is there a discrepancy with the equivalence principle?
Post by: timey on 07/05/2017 15:24:38
One of the consequences of having nothing going on in my life is that I have very little resources resulting in no internet connection of my own, therefore I will have to wait until later to read the interim posts, but I wrote this this morning so I'll post it and catch up with the replies later:

*

If I could throw myself back in time I would tell Planck that he could iron out the additions of energy causing frequency change into a linear progression by shortening the length of a second as frequency increases, and I have no doubt that he would shake my hand in sheer delight and thank me.  Einstein had introduced the notion of different rates of time applied to physics and this idea is just an extension/symmetry of that notion 

Under the remit of this system Planck's h constant becomes a function of time dilation, where it is changes in energy level that are causing changes in the rate of time.   It takes h value of joules to initiate a change in the rate of time, and the change in the rate of time can be observed by the change in frequency.

Applying this remit to the structure of conventional physics maths only requires a shift in interpretaion.

Looking at this photon counting equation Qp = h⋅c / λ.

h is now indicative of the amount of energy that causes the length of a second to change.  Within the mathematical structure of h the value of a joule is being held relative to the static length second, and c is a set distance being held relative to the static length of second...
h multiplied by c is taking the amount of energy it takes to cause the length of a second to change and multiplying that by a speed that takes 1 static length second to cover a set distance.
We could simply say that the result of h*c is equal to 299 792 458m/second that is longer or shorter than the static length second, but by what value is the second longer or shorter?
Now we look to the mathematical structure of  λ.  As frequency changes the length of a wavelength changes.  Increases in frequency cause shorter wavelengths, and decreases in frequency cause longer wavelengths.  If it takes h value of joules to initiate a change in the rate of time, and the change in the rate of time can be observed by the change in frequency, then  λ is incorporating the frequency change within its mathematical structure, where frequency is held relative to the length of a standard second.   λ = h/p and p=h*vbar and vbar=va where v is wave number or frequency and a is indicative of change.
 
So - h*c is a multiplication of 2 values being held relative to the length of a static length second, but the value of the result can be thought to have incorporated a change in the rate of time via the use of h.   The the resulting value divided by  λ is a division of a value that incorporates a change in the rate of time by a value that incorporates a change in the rate of time.
Therefore there 'is' a reference to the change in the rate of time on both sides of the equation Qp = h⋅c / λ. (and the equation E=hf)

This has been a description that involves the rate of time for the free electrons of the emitting source black body increasing with increased energy to emit higher frequency photons, and also a description of light propagation in space where the energy of the already emitted light is changed by it's position in the gravity potential, where it looks very much to me as though quantum photon is unified with gravity via a physical cause for the wave function...in the face of adding a 3rd aspect to the time dilation phenomenon where the rate of time is decreased in the higher gravity potential for anything or anywhere where m=0.
But this remit can extend to matter waves that are subject to changes in energy. i.e: the rate of time for the free electrons and also bound electrons...where the possibility arises to bastardize the equation Qp = h⋅c / λ for electrons, by exchanging c for v (I think, scratches head) where v is the frequency, (need help with that one)...
And if one considers that potential energy is a property of mass that can be added to or subtracted from mass due to the mass's position in the gravity field this further unifies quantum with gravity where we observe that a clocks frequency is increased in the higher gravity potential and hold that any physical process where m doesn't =0, i.e. matter waves, are all proportionally increased in frequency in the higher gravity potential which is supported by the commonly held notion that a person will age in keeping with their time dilated clock.

I can now describe how this remit can give physical cause and effect mechanics to the accelerative/decelerative force of gravity, and how this remit can give cause and effect mechanics to the attractive/directional force of gravity.

Title: Re: Is there a discrepancy with the equivalence principle?
Post by: timey on 07/05/2017 21:02:02
Back online now...and have read the replies of which I shall address some of the content below, but first...

Reading through what I wrote in my last post, I feel that I have been a bit fuzzy in my description concerning this bit:
Quote
: timey
Under the remit of this system Planck's h constant becomes a function of time dilation, where it is changes in energy level that are causing changes in the rate of time.   It takes h value of joules to initiate a change in the rate of time, and the change in the rate of time can be observed by the change in frequency.

"It takes h value of joules to initiate a change in the rate of time" being the fuzzy bit.

To be more clear - it takes h value of joules times the frequency that is indicating a change in the rate of time to know the value of the Energy that is changing the rate of time.

I think this is what Timey is trying to do - if seconds pass at, say, twice the rate relative to our gravitational potential i.e. each is = half of our seconds, then the calculation becomes 20J (10/0.5), but, to use wavelength as the modifying factor.

First thing here Colin is that I am worried that you are basing this analysis on the fact that clocks run faster at a higher gravity potential, which does remain as a fact within my hypothesis, therefore your analysis does have a bearing on the energy, frequency, and wavelength of any m at a gravity potential that causes m to increase in frequency by twice the rate compared to the reference frame of the 'static length' second, this being our own frame of reference, which we could state as being sea level Earth because according to GR and SR considerations all clocks run at the same rate at the sea level of every longitude.

But here, at present we are talking about light travelling through changes in the gravitational field (energy), where the gravitational field reduces in strength (energy) as distance is increased from M.  I am suggesting that these changes in energy are causing the rate of time in space itself to be decreased, i.e: longer seconds, as field strength (energy) is reducing with distance from M.
Light, because it is m=0 will not increase in the potential energy that I am suggesting is the cause of the increase in frequency for a clock, where mgh is just gh for light.
Light red shifting away from sea level Earth will reduce in energy as the field strength (energy) reduces with distance from M because it is taking longer and longer amounts of time to complete a wave cycle as frequency is reduced.

Therefore the equation Qp=h⋅c / λ is multiplying a time dilation function h that has its basis in a static length second measurement, by a speed held relative to a static length second, and dividing by a time dilation function that has it's basis in a static length second measurement.

Far as I can tell this system, in both the scenario of point particles where m doesn't =0, and the scenario of point particle photons where m=0, has given the wave function a physical cause where a division by wavelength will identify position, and also unify quantum with gravity.

I suspect the only help I can provide for timey is assistance in understanding and navigating through the math with her.

That is the best that I could hope for and I very much appreciate your input.
Title: Re: Is there a discrepancy with the equivalence principle?
Post by: Colin2B on 08/05/2017 14:57:46
To be more clear - it takes h value of joules times the frequency that is indicating a change in the rate of time to know the value of the Energy that is changing the rate of time.
"...............
First thing here Colin is that I am worried that you are basing this analysis on the fact that clocks run faster at a higher gravity potential,


No need to worry, it was addressing what you have expressed above, but it works whether clocks run faster or slower at any potential, or a position in space that has a time dilation relative to another position. So it should work for your various scenarios. The simplest way is to express the time dilation as a ratio against the std second then you use that ratio to work out f. For E=hf you are on safe ground as this is what current physics says.
Title: Re: Is there a discrepancy with the equivalence principle?
Post by: Ethos_ on 08/05/2017 15:34:44


That is the best that I could hope for and I very much appreciate your input.
Regarding the formula associated with photon energy:

Qp = hc/wavelength
Please excuse my notation, I don't have latex.
Let's use the figure (w) to represent wavelength.

Qp=hc/w

Photon energy = Planck's constant times the speed of light divided by selected wavelength.

Joules = h * c * w^-1
           = (J*s) * c * w^-1
           = (kg * m^2 /sec) * (m/sec) / (m)
           = (kg * m^2/sec^2)

For a VHF wavelength of 1 meter, just to simplify, we plug in the following values:

Qp = (6.626 E^-34 * 2.9979E^8) / 1

Photon energy of one photon at 1 meter wave length ........... (1.9864 E^-25) Joules

BTW, the (E^-25) is a simpler way of notation representing: (times 10 raised to the -25)
Title: Re: Is there a discrepancy with the equivalence principle?
Post by: Ethos_ on 08/05/2017 16:54:00
Timey..........I have a thought associated with Lorentz contractions which may be of some relevance  when applied to your theory. I seem to remember you making questioning comments regarding the  accuracy of length contract at some time in the past, not sure if I'm remembering correctly about that however. At any rate, get back with me and detail what your stance is on the subject because our posture on the correctness of Lorentz contractions is vital in determining the viability of your theory. 
Title: Re: Is there a discrepancy with the equivalence principle?
Post by: timey on 08/05/2017 17:30:50
but it works whether clocks run faster or slower at any potential, or a position in space that has a time dilation relative to another position. So it should work for your various scenarios. The simplest way is to express the time dilation as a ratio against the std second then you use that ratio to work out f. For E=hf you are on safe ground as this is what current physics says.

Thanks Colin!
The difference between conventional physics and my train of thought being the addition of a 3rd aspect to the time dilation phenomenon that states that red shifted light in the gravitational field of space will be travelling at the speed of light 'moving through' rates of time that are decreasing in rate, i.e. longer seconds, as the gravitational field strength (energy) is reduced by distance from M.  And that the equation Qp=h⋅c / λ is locating the gravitationally shifted photon via dividing by the rate of time that is occurring at that gravity potential. i.e: this has given the wave function of the photon a physical cause.

Now we can take that system to the gravitationally shifted clock in the higher potential where that clock (according to the maths, or an observer in the lower potential) will be shifted to a higher frequency, as opposed to light in the higher potential where that gravitationally red shifted light (according to the maths, or an observer in the higher potential) will be shifted to a lower frequency...
Ignoring all of the particles that are comprising the physical structure of the clock (where my hypothesis is that all matter waves will be proportionally decreased in length in the higher potential) and just concentrating on the frequency of the electron transitions of the cesium atom (this being the definition of the tick rate of the clock), my hypothesis is stating the increased frequency of the electron transitions is being caused by additional energy and that this energy is being added to the clock due to its position in the gravity potential...

So the question are:
Can an mgh calculation for the electron account for the increase in frequency of electron transitions?
(The only reason in conventional physics not to state the potential energy as residing in the mass itself, that I can determine, (if anyone can add another?) is due to the fact that in order to change the value of relativistic mass a conversion of kinetic energy to potential energy is necessary where potential energy does not contribute to relativistic mass value.  If potential energy where considered to reside in the mass, the relativistic mass equation wouldn't change the value of relativistic mass.)
Relativistic mass is not necessary in my model, but we can get to that bit later.

Can we, on the basis of my hypothesis, construct an equation for the 'quantum' electron such as this : Qe=h⋅f / λ?  (Where the division by the wave length is dividing by the rate of the time dilation factor.)

Photon energy of one photon at 1 meter wave length ........... (1.9864 E^-25) Joules

So in this case Ethos, according to my hypothesis, we will say that this is the energy of the photon when it was emitted, and that the energy of the emitting source was such that it would emit a photon of that energy.  The photon red shifts away from Earth and is reduced in frequency by the gravitational field, where the extra length of wave length will be indicative of the red shifted light having taken a longer amount of time to complete a wave cycle.  To understand by how much the rate of time has decreased at each elevation, i.e: longer seconds, take the extra bit of length of the wavelength, as compared to 1 meter, and divide that by c the speed of light to obtain a measure of time that can be added to the length of a std second to determine by how much longer than a std second a second is at that elevation.

I am in admiration (or perhaps it's jealousy) of your capacity for the juggling of numbers.  Adding numbers to the proportions confuses me.  How I think about proportions is a matter of shapes and moving lines doing slippy slidey stuff, where I complete all my considerations inside my head.

Ethos - just saw your latest post.  Yes, my hypothesis does not use the Lorentz Transformations to describe light moving across space.  When I get to the stage where we are discussing the above in terms of acceleration/deceleration of gravity it will become clear why, but to say so the values of the Lorentz transformations will be needed to cross reference the alternative, and the remit of the Lorentz Transformation will be used in my model to describe the perception of time and space for m moving across space, but in a slightly altered format.   
Title: Re: Is there a discrepancy with the equivalence principle?
Post by: Ethos_ on 08/05/2017 17:58:35


I am in admiration (or perhaps it's jealousy) of your capacity for the juggling of numbers.  Adding numbers to the proportions confuses me.  How I think about proportions is a matter of shapes and moving lines doing slippy slidey stuff, where I complete all my considerations inside my head.
No need to be jealous timey, I judge your intellect to be very high indeed. And also no need to admire my juggling ability, math like many things takes practice and I just like the exercise.


Quote from: timey
Ethos - just saw your latest post.  Yes, my hypothesis does not use the Lorentz Transformations to describe light moving across space.  When I get to the stage where we are discussing the above in terms of acceleration/deceleration of gravity it will become clear why, but to say so the values of the Lorentz transformations will be needed to cross reference the alternative, and the remit of the Lorentz Transformation will be used in my model to describe the perception of time and space for m moving across space, but in a slightly altered format.   
Thanks for the "timely" reply "timey", just a little tug at humor no less, because I'll need to rest some before I can return. I really do not wish to discuss my health but I find it necessary to let you know why I may be absent for I wish not to appear in avoidance of your company. I'll return when I've regained some strength.

Hang in there my friend.....................................................

Title: Re: Is there a discrepancy with the equivalence principle?
Post by: jeffreyH on 08/05/2017 18:08:03
U


That is the best that I could hope for and I very much appreciate your input.
Regarding the formula associated with photon energy:

Qp = hc/wavelength
Please excuse my notation, I don't have latex.
Let's use the figure (w) to represent wavelength.

Qp=hc/w

Photon energy = Planck's constant times the speed of light divided by selected wavelength.

Joules = h * c * w^-1
           = (J*s) * c * w^-1
           = (kg * m^2 /sec) * (m/sec) / (m)
           = (kg * m^2/sec^2)

For a VHF wavelength of 1 meter, just to simplify, we plug in the following values:

Qp = (6.626 E^-34 * 2.9979E^8) / 1

Photon energy of one photon at 1 meter wave length ........... (1.9864 E^-25) Joules

BTW, the (E^-25) is a simpler way of notation representing: (times 10 raised to the -25)

For a 1 hertz frequency we have Qp = (6.626 E^-34 * 2.9979E^8) / 2.9979E^8 so that the energy is simply 6.626 E^-34 joules which is a value equivalent to h but with different units. This is a wavelength of 1 light second. This is important for your proposition.
   
Title: Re: Is there a discrepancy with the equivalence principle?
Post by: jeffreyH on 08/05/2017 18:30:42
This is a very good page to use when playing around with the numbers.
https://rechneronline.de/spectrum/
Title: Re: Is there a discrepancy with the equivalence principle?
Post by: Ethos_ on 08/05/2017 19:06:07
This is a very good page to use when playing around with the numbers.
https://rechneronline.de/spectrum/
Thanks Jeff, very good link. And you're correct, I failed to divide again by c where wavelength is (v/f). I do these calculations without the benefit of a program and often make mistakes, especially when done in haste. This link you provided should illuminate or at least reduce those mistakes so thanks again.

It would appear that I need more practice...............................
Title: Re: Is there a discrepancy with the equivalence principle?
Post by: timey on 09/05/2017 02:50:18
For a 1 hertz frequency we have Qp = (6.626 E^-34 * 2.9979E^8) / 2.9979E^8 so that the energy is simply 6.626 E^-34 joules which is a value equivalent to h but with different units. This is a wavelength of 1 light second. This is important for your proposition.

And this wavelength would be a distance of 299 792 458 metres?
If we started light out being emitted at a visible light frequency, and calculated it as gravitationally red shifted via the gravitational shift equation, what strength would the gravity field be when the light reached a wavelength of 1 light second?

I'll return when I've regained some strength.

Hope to see you back soon.  In the meantime I'll re-post the current questions in hand, maybe Colin or Jeff might be able to answer, but otherwise I'll await you feeling better...

*

The difference between conventional physics and my train of thought being the addition of a 3rd aspect to the time dilation phenomenon that states that red shifted light in the gravitational field of space will be travelling at the speed of light 'moving through' rates of time that are decreasing in rate, i.e. longer seconds, as the gravitational field strength (energy) is reduced by distance from M.
 And that the equation Qp=h⋅c / λ is locating the gravitationally shifted photon via dividing by the rate of time that is occurring for space itself at that gravity potential. i.e: this has given the wave function of the photon a physical cause.

Now we can take that system of interpretation to the gravitationally shifted clock in the higher potential where that clock (according to the maths, or an observer in the lower potential) will be shifted to a higher frequency, as opposed to light in the higher potential where that gravitationally red shifted light (according to the maths, or an observer in the higher potential) will be shifted to a lower frequency...
Ignoring all of the particles that are comprising the physical structure of the clock (where my hypothesis is that all matter waves will be proportionally decreased in length in the higher potential) and just concentrating on the frequency of the electron transitions of the cesium atom (this being the definition of the tick rate of the clock), my hypothesis is stating the increased frequency of the electron transitions is being caused by additional energy and that this energy is being added to the clock due to its position in the gravity potential...

So the questions are:
Can an mgh calculation for the electron account for the increase in frequency of electron transitions?

(The only reason in conventional physics not to state the potential energy as residing in the mass itself, that I can determine, (if anyone can add another?) is due to the fact that in order to change the value of relativistic mass a conversion of kinetic energy to potential energy is necessary where potential energy does not contribute to relativistic mass value.  If potential energy where considered to reside in the mass, the relativistic mass equation wouldn't change the value of relativistic mass.  Relativistic mass is not necessary in my model, but we can get to that bit later.)

And...
Can we, on the basis of my hypothesis, construct an equation for the 'quantum' electron such as this : Qe=h⋅f / λ?  (Where the division by the wave length is dividing by the rate of the time dilation factor to determine position.)
Title: Re: Is there a discrepancy with the equivalence principle?
Post by: Colin2B on 09/05/2017 08:47:18
There are a number of separate questions in the above post and it might be best to look at them separately. Some related topics have been discussed by Ethos, Jeff & myself and I'll pm Ethos to help him put something together. 

Some second thoughts. It might be worth starting with some common understanding and terminology e.g.  somewhere you use the term "gravitational field strength (energy)", the field strength is not energy.
It is also worth getting a common understanding of how current physics uses standard seconds.  These are not commonly used because of the inconvenience of trying to keep everything synchronised, it is far easier to carry a frequency/clock standard which you know will give the same result at any elevation.
If Ethos starts from there you can build up without getting confused by trying to deal with multiple issue at once.
Title: Re: Is there a discrepancy with the equivalence principle?
Post by: timey on 09/05/2017 16:11:26
The points you raise Colin are pertinent to the premise of my hypothesis, mist significantly where I have put energy in brackets next to gravitational field strength, I'll get to that, but first let's discuss the standard second.

A few posts ago I said that it doesn't matter what length of second one starts out with.  What is important is that it is a static length second that is being used as a basis for the structure of the mathematics, where h, f, c and wavelength are being held relative to a static length second.  We could use any length of second as a static length to comprise the structure of these mathematics, so long as we use the same static length for each component the proportions of the maths will remain equivalent, but conventional physics 'is' using the static length of a standard second as it's basis for the mathematical structure of h, f, c and wavelength, so it is logical to follow suit, where, because a clock will run at the same rate at the sea level of all longitudes of Earth, we can state sea level Earth as being the reference frame of the standard second, and also recognise that there will be some other places in the universe where the combination of GR position in the gravity potential, and SR motion relative to the  gravity field will result in a second that can be the same length as a standard second. i.e: there is an orbital speed at an altitude from Earth that will result in a second that is the same length as a standard second.

The fact that I have written gravitational field strength (energy) is related to my first question:
" Can an mgh calculation for the electron account for the increase in frequency of electron transitions?"

GR is a theory of gravity.  But while GR can tell you with almost perfect precision what gravity is doing, it doesn't tell you 'how' it is doing it.  My hypothesis aims to extend the premise of GR to include a description of 'how' gravity works...

My first question is followed by a consideration of the remit of potential energy regarding relativistic mass.  It asks if there is any other reason apart from the premise of relativistic mass not to consider potential energy as residing within the mass.
The logic of this question is based on the fact that kinetic energy and potential energy are not conserved individually, it is the sum total of both that is conserved.  If conventional physics considered potential energy as residing within the mass, then the sum total of the conversion from kinetic energy to potential energy, or visa versus, would mean that the value of relativistic mass would remain unchanging and this would break the premise of the conventional theory.
Therefore conventional physics has, despite the conservation of energy law stating that it is the sum total of potential and kinetic energy that must be conserved, has chosen to state potential energy as an energy that cannot be placed as residing anywhere.

My hypothesis results in there being no requirement for relativistic mass.  This is related to:
a) the fact that my model does not use SR to describe light travelling in space
and:
b) the fact that the alteration my model makes to the use of SR to describe m's experience of traveling through time and space in relation to my models description of how light travels through time and space ensures that anything of mass (rest mass in conventional physics) can never travel at the speed of light.

So going back to my first question:
" Can an mgh calculation for the electron account for the increase in frequency of electron transitions?"

I am now considering that potential energy resides, or perhaps more accurately is active within the mass.
Let's briefly consider that there is a difference between the 'locked in' energy of rest mass e=mc^2, and the sum total of kinetic and potential energy.  We can expand on this consideration later...
...Where plus potential energy is causing activities within mass to become more active, (more frequent/increased rate)...

...And minus potential energy is causing activities within mass to become less active, (less frequent/decreased rate)...

I am suggesting that the gravitational field itself has potential energy, where the mgh equation is just gh.  Or more precisely gr=GM/r^2.
Where at each elevation the energy is decreased.  If a graviton were to exist, it's activities would become less active, less frequent/decreased rate where a gravitons energy, frequency and wavelength would resemble the gravitational red shift equation.

My model doesn't need a graviton.  It just states that potential energy has a relationship with time and that at each elevation it is time that is less active, less frequent constituting a slower rate of time.

This premise accounts for gravitational acceleration and deceleration of m 'through' m=0 open space gravitational field, and also accounts for the gravitational directional attraction of m in the higher gravity potential via increased frequency/rate of 'magnetic moments'.  We can expand on that later.

I'm very glad Colin that you picked up on my addition of energy in brackets:
"Gravitational field strength (energy)"
... and hope that I have been clear in the description of my reasoning for the addition...
Title: Re: Is there a discrepancy with the equivalence principle?
Post by: Colin2B on 09/05/2017 23:02:16
Just some questions to help Ethos


conventional physics 'is' using the static length of a standard second as it's basis for the mathematical structure of h, f, c and wavelength, so it is logical to follow suit, where, because a clock will run at the same rate at the sea level of all longitudes of Earth, we can state sea level Earth as being the reference frame of the standard second,
Conventional physics is using local, ie variable seconds - if I understand how you are using standard seconds.
It is usual to state the geoid as the reference for standard seconds as this represents the contour of equi-gravitational potential - the gravitational potential of average sea level, although it does not track sea level over land.


If we take standard seconds at the geoid then if, say, at a certain altitude time is passing at say +10% as measured from the geoid then 1.1 standard seconds would pass for every standard second which passes at the geoid. In other words the clock at altitude is ticking faster. Obviously the observer at altitude measures using local seconds. Is that how you are using the term standard seconds.


Added: when you say "Can an mgh calculation for the electron account for the increase in frequency of electron transitions?" by "[/size]increase in frequency of electron transitions" do you mean the frequency at which electron transitions occur, or the frequency (light colour) of the photons produced by the electron transitions?[/color]
Title: Re: Is there a discrepancy with the equivalence principle?
Post by: timey on 10/05/2017 00:22:55
Quote
:timey
conventional physics 'is' using the static length of a standard second as it's basis for the mathematical structure of h, f, c and wavelength, so it is logical to follow suit

I am saying that conventional physics is using the standard second as the static length second that is the basis for the structure of maths that are inclusive of frequency, the speed of light, Pancks h constant, and the measure of energy that comprises a joule. (and that's not all).  E=fh has it's basis in the standard second.

We don't make a measurement of c using a local second*, (unless we are considering the standard second to be local, which we can, and can most conveniently do this by stating the standard second as the reference frame of sea level Earth due to all clocks running at the same rate at sea level).
If one tries to use a speed of light that is held relative to the local rate of time of a clock that is not ticking at the rate of a standard second, i.e GR in SR, then one runs into problems with distance/length contraction.  (Mike Gale explored this scenario on his thread relativistic correction to the SC).

I'm not fussed though, we can just refer to the second that is the static length second that comprises the structure of the maths as a static length second if you prefer...

*Perhaps you may further solidify my understanding though... The observer observes his time dilated clock to be ticking normally.  Does the observer measure the speed of light via his time dilated clock?

Added: I mean the frequency that electron transitions occur, but does the frequency that electron transitions occur at have a bearing on the frequency of the photon emissions?  I seem to remember you commenting in another thread...
Title: Re: Is there a discrepancy with the equivalence principle?
Post by: timey on 10/05/2017 15:11:54
In addition to my post above:

The factors involving the maths of pe=mgh and g(r)=GMearth/r^2 that I need to understand are:

Is r the distance from the center of Earth unless specified otherwise?
Is the (r) and r in g(r)=GM/r^2 saying the same thing?

What I'm trying to establish is the difference, if any, between an equation that is describing the value of g for various radius from sea level to center of earth, and an equation that is describing the value of g for various radius from sea level direction away from Earth...
The reason being that it would seem to me that if we added a significant amount of mass evenly distributed to Earth, this would increase the value of r from center of Earth to sea level*, that a clock at elevation from sea level would be subject to a gravity potential where g has increased in field strength, and this will cause the equations that predict how much faster that clock will tick at an elevation from sea level to predict that the clock in elevation to the bigger mass will run faster than the clock in elevation to the smaller mass due to an increased value of M and r in the g(r)=GMearth/r^2 equation.
Because the clock is ticking faster at the same elevation from the bigger M than the clock at the same elevation from the smaller M is, if we put both clocks on the ground of each mass, the maths will tell you that the clock on the bigger mass is ticking faster than the clock on the smaller mass is, which is contrary to the conventional physics notion of time running slower for the bigger mass.
(I'm quite sure that my logic is not flawed here, and would appreciate comment)


*As a side issue - mass is not evenly distributed from center of Earth to sea level.  The distance from center of Earth to sea level is greater at the equator than it is at the poles by a distance of 13.2 or so miles.  But clocks will run at the same rate at the sea level of all longitudes due to the SR motion related 'dilation' of time caused by the greater radius's increased centripetal motion cancelling out the increase in GR altitude related 'contraction' of time caused by the greater radius's increased altitude...
To further understand the relationship between GR and SR time dilation's, if we left the mass value of Earth as it is, but ironed out the equatorial bulge so that there is an equal distance from sea level to center of Earth at each longitude, we would find that there will still be differences between the centripetal speeds at sea level of each longitude.  Because r from centre of Earth is now equal for all longitudes and centripetal speed is differing for each longitude, clocks at sea level will run faster at the poles and tick at slower rates as one progresses towards the equator.
Add the equatorial bulge back in and the clocks at sea level of all longitudes will run at same rate again...
Quote
:wiki
The fact that the Earth's gravitational field slightly deviates from being spherically symmetrical also affects the orbits of satellites through secular orbital precessions.[2][3][4] They depend on the orientation of the Earth's symmetry axis in the inertial space, and, in the general case, affect all the Keplerian orbital elements with the exception of the semimajor axis. If the reference z axis of the coordinate system adopted is aligned along the Earth's symmaetry axis, then only the longitude of the ascending node Ω, the argument of pericenter ω and the mean anomaly M undergo secular precessions.[5]
Such perturbations, which were earlier used to map the Earth's gravitational field from space,[6] may play a relevant disturbing role when satellites are used to make tests of general relativity[7] because the much smaller relativistic effects are qualitatively indistinguisgable from the oblateness-driven disturbances.
By superimposing the consideration of the equatorial bulge over the consideration without the equatorial bulge, there will be a relationship between the changes in distance of radius, and the changes in centripetal speed, in relation to the changes in time dilation, where one could cross reference the differing values and then ask oneself, as a starting point, where the concept of SR length contraction applies when GR and SR time dilation's are used in combination.

https://www.thenakedscientists.com/forum/index.php?topic=70251.msg513077#msg513077

(again... comment would be appreciated)
Title: Re: Is there a discrepancy with the equivalence principle?
Post by: Colin2B on 10/05/2017 16:15:38
We don't make a measurement of c using a local second*, (unless we are considering the standard second to be local, which we can, and can most conveniently do this by stating the standard second as the reference frame of sea level Earth due to all clocks running at the same rate at sea level).
If one tries to use a speed of light that is held relative to the local rate of time of a clock that is not ticking at the rate of a standard second, i.e GR in SR, then one runs into problems with distance/length contraction.  (Mike Gale explored this scenario on his thread relativistic correction to the SC).
Ok, now we know how you are using term standard second. We were misinterpreting because we thought you had originally used it to describe the time used in the app you mentioned. If you are using it to mean that measured by a caesium clock at sea level (although I would use the geoid) then you don't need to use the qualifier 'standard' as this is generally assumed in physics, defined by SI, and measured by any local observer. Best to reserve a qualifier for dilated time, if needed.

We also explored this problem of using altitude adjusted seconds - what I thought at the time you meant by standard seconds - way back. As we pointed out it not only causes problems with length measurements, but changes the calculation of energy which is also frame dependent.


*Perhaps you may further solidify my understanding though... The observer observes his time dilated clock to be ticking normally.  Does the observer measure the speed of light via his time dilated clock?
This is what I would have referred to as local seconds, if you had used standard (altitude adjusted) seconds at altitude.
As we have agreed elsewhere (way back) the local observer sees exactly the same (local) seconds as an observer at any other GP and, in the close vicinity, uses those seconds to measure speed. It is only the remote observer who sees the seconds as dilated and makes different calculations of frequency, speed, distance, energy etc.

As an aside, you may remember in the NIST thread I mentioned that that 2010 experiment was a test of concept for further work which would resolve one of your experimental proofs. Mentioning the geoid reminded me. NIST are progressing that work by measuring the geoid at their Boulder site, so you might see the tests done soon - next few yrs.

Added: I mean the frequency that electron transitions occur, but does the frequency that electron transitions occur at have a bearing on the frequency of the photon emissions?  I seem to remember you commenting in another thread...
No, there is no relationship.
"frequency that electron transitions occur" affects the number of photons emitted/s which affects intensity.
"frequency of the photon emitted" defines the colour/wavelength of the photon.
E=mgh for an electron would tell you how much KE it could acquire if it fell through h, placing an atom at a greater h doesn't cause the number or rate of electron transitions to increase - we don't observe an increase in intensity.

I don't know how far Ethos wants to take this, but the clarification will help, thanks.
Title: Re: Is there a discrepancy with the equivalence principle?
Post by: timey on 10/05/2017 18:52:09
As we have agreed elsewhere (way back) the local observer sees exactly the same (local) seconds as an observer at any other GP and, in the close vicinity, uses those seconds to measure speed. It is only the remote observer who sees the seconds as dilated and makes different calculations of frequency, speed, distance, energy etc.

Yes, I remember, but also re-call that we came to a Mexican stand off scenario over the fact of a person aging in keeping with their clock.

Yes it is only the remote observer who sees the seconds as dilated and makes different calculations of frequency, speed, distance, energy etc.  The local observer sees exactly the same (local) seconds as an observer at any other GP and, in the close vicinity, uses those seconds to measure speed...
But what does this mean?  What exactly are we saying here?
Are we saying that seconds are the same length everywhere and they just look different from the remote observers point of reference?
...In which case a person with a clock that is observed remotely as being time dilated would age no differently to anyone at any other GP.
Or are we saying that seconds are of differing length at differing GP's but appear to be the same length as everywhere else from the local point of reference?
...In which case a person will age in keeping with their time dilated clock, and their clock appears to be ticking normally because all physical process is increased, or decreased proportionally with the increased, or decreased tick rate process of the clock.

Clearly one cannot state that a second is the same length in any GP and also state that a person will age in keeping with their time dilated clock.
How one answers this contradiction has a direct bearing on what you say here:

E=mgh for an electron would tell you how much KE it could acquire if it fell through h, placing an atom at a greater h doesn't cause the number or rate of electron transitions to increase - we don't observe an increase in intensity.

Where are we when we don't observe an increase in intensity?  Are we in the local frame, or the remote frame?

placing an atom at a greater h doesn't cause the number or rate of electron transitions to increase

An atomic clock is observed from the remote frame of the lower potential to have an increased frequency of electron transitions.  So you are saying that the number or rate of electron transitions is not increased...
Am I completely misunderstanding the premise of frequency here?  Frequency is the number of wave cycles, and the number of wave cycles is the rate of wave cycles occurring per static length second, where you have said this:

"frequency that electron transitions occur" affects the number of photons emitted/s which affects intensity.

Leading back to my question:
"Where are we when we don't observe an increase in intensity?  Are we in the local frame, or the remote frame?"

The same applies to the observation of the frequency of a photon.  Remembering that we can only view a photon in the remote frame, when it arrives "in' the remote frame and that the photon will have been gravitationally shifted by the change in GP.

"frequency of the photon emitted" defines the colour/wavelength of the photon.

What defines the frequency of the photon?
The frequency of electron transitions has a relationship with a 'ground state'.  A ground state has a relationship with Energy levels, therefore the frequency of electron transitions has a relationship with energy.  The energy to increase electron transitions must 'come' from somewhere...
Now we are back to the beginning of the post where it becomes important to decide whether we are stating that a second is the same length in any GP, or if we are stating that a person will age in keeping with their time dilated clock.

I am taking the view that a person ages in keeping with their time dilated clock and therefore the seconds in each GP are of differing length, and the energy levels causing electron transitions to have differing frequencies are differing due to being subject to an increase/decrease in gravity potential energy...
It is a contradiction to the conservation of energy law to state:
E=mgh for an electron would tell you how much KE it could acquire if it fell through h
...And then say that potential energy has no effect on the activities of the electron, when it is the sum total of potential energy and kinetic energy that must be conserved.

As mentioned a few posts ago - the only reason (that I can define, if anyone can add another) within conventional physics to not state potential energy as active within mass is due to the remit of only using the value of kinetic energy to calculate relativistic mass value.  (comment would be appreciated)
Title: Re: Is there a discrepancy with the equivalence principle?
Post by: guest4091 on 10/05/2017 20:18:52
If it has been detected I would be very interested in the reference.
The electric field strength increases for fast electrons as they contract, and is detectable.
check this link..
https://conf-slac.stanford.edu/sssepb-2013/lectures
select this:
Lecture 1. Lecturer Zhirong Huang

That link doesn't work.
sorry, link is corrected now
Title: Re: Is there a discrepancy with the equivalence principle?
Post by: alancalverd on 10/05/2017 21:06:19
What exactly are we saying here?
The simple answer is that the gravitational field warps the spacetime continuum. Remarkably, this result from GR seems to explain every experimantal result to date.
Quote
E=mgh for an electron would tell you how much KE it could acquire if it fell through h, placing an atom at a greater h doesn't cause the number or rate of electron transitions to increase - we don't observe an increase in intensity.

Where are we when we don't observe an increase in intensity?  Are we in the local frame, or the remote frame?


This is a prime example of why you need to use physicists' language, not your own. The "frequency of electron transitions " means the number of times electrons make quantum transitions in a second, and this defines the number of photons emitted per second, i.e. the intensity (quantity) of emission. The photon frequency associated with an individual electron transition defines the energy per photon - the quality of the emission.

Confusion arises when you talk about black body radiation. If you increase the temperature of a blackbody, you increase the quantity of emission and also the maximum photon energy that can be emitted, so the spectrum quality shifts also. But it is important to remember that blackbody radiation is a continuum. If you increase the temperature of a real body you may generate new spectral lines as sufficient energy becomes available to permit quantised electron transitions within individual atoms and molecules, but that has nothing to do with the idealised blackbody spectrum. A fine example is the spectrum of a molybdenum-anode x-ray tube. Most of the radiation is a blackbody spectral continuum with a maximum determined by the applied anode voltage (usually 30 kV, giving up to 30 keV photons), but there are a couple of large "characteristic" spikes around 10 - 15 keV that we use for breast imaging.
Title: Re: Is there a discrepancy with the equivalence principle?
Post by: timey on 10/05/2017 21:56:53
The simple answer is that the gravitational field warps the spacetime continuum. Remarkably, this result from GR seems to explain every experimantal result to date.
Therefore it is remarkable that while GR can predict and describe what will occur, it cannot describe 'why' it is occurring.  A fact that is widely remarked upon in the physics books I read.
The "frequency of electron transitions " means the number of times electrons make quantum transitions in a second and this defines the number of photons emitted per second, i.e. the intensity (quantity) of emission.
Thank you.  So the question stands with a slightly altered terminology:

"Where are we when we don't observe an increase/decrease in electron quantum transitions.  Are we in the local frame?"

The photon frequency associated with an individual electron transition defines the energy per photon - the quality of the emission.
Is the number of times electrons are making quantum transitions in a second defined by energy levels?
And does the number of times electrons make quantum transitions have a bearing on the energy level of the individual electron transitions that are defining the energy per photon?
Title: Re: Is there a discrepancy with the equivalence principle?
Post by: Colin2B on 10/05/2017 23:00:25
Yes, I remember, but also re-call that we came to a Mexican stand off scenario over the fact of a person aging in keeping with their clock.
Not me. I have no problem with the fact that a person ages differently at a GP relative to someone at a different GP. What I have never managed to understand is why you think they wouldn't according to current physics.


I think one problem is that highlighted by Alan, that we end up talking at cross purposes due to misunderstandings of terminology. It's one reason I tend to keep out of these threads. I thought by trying to clarify some terminology we might be able to make some progress.


As I say, I'll see how Ethos wants to take it forward. I'll help him with drafting if he needs it.







Title: Re: Is there a discrepancy with the equivalence principle?
Post by: timey on 11/05/2017 00:07:55
I have no problem with the fact that a person ages differently at a GP relative to someone at a different GP. What I have never managed to understand is why you think they wouldn't according to current physics.
Because every time I try to discuss here the fact of seconds being of differing length in differing GP's, I am consistently told that this is only the case when viewed from the remote frame, and it has then been very difficult to continue the discussion in terms of talking about differing length seconds!!!

When I have raised the notion of the change in frequency of electron transitions being caused by a change in energy level, I have been told that this cannot be the case because the frequency of electron transitions is measured as the same in each GP and therefore the energy level in each GP must be the same...
I have also been told that the frequency of a clock must be the same at each GP because if one measures a clock from a position of higher gravity potential, it will be running slower, and then measures the same clock from a position of lower gravity potential, it will be running faster, and that the clock cannot be running both slower and faster at the same time.
It is indeed quite clear to me, under the remit of conventional physics, that the higher potential position is measuring the clock in the lower potential by it's increased tick rate, and that the lower potential position is measuring the clock by it's decreased tick rate, and that it is all 3 clocks that are running at differing rates.
What I'd like to talk about is 'why'...

As I say, I'll see how Ethos wants to take it forward. I'll help him with drafting if he needs it.

Great news indeed!  Alan also seems to be back, and on form, which secretly pleases me, with this being not such a secret now though aye ;)
Title: Re: Is there a discrepancy with the equivalence principle?
Post by: alancalverd on 11/05/2017 00:21:32
Here's the answer to the ageing question.

Wherever you are in the universe, you will live for threescore years and ten. But you will appear to die earlier when seen from the surface of a bigger planet, because time goes slower at lower gravitational potentials. So if you were born on the moon, whilst your clock will have delivered 2.2 billion seconds during your lifetime, mine (on planet earth) will have delivered rather fewer ticks when I receive the news of your death. 

Title: Re: Is there a discrepancy with the equivalence principle?
Post by: timey on 11/05/2017 00:26:14
Arrrrgh, I'm dead...

The simple answer is that the gravitational field warps the spacetime continuum. Remarkably, this result from GR seems to explain every experimantal result to date.
Therefore it is remarkable that while GR can predict and describe what will occur, it cannot describe 'why' it is occurring.  A fact that is widely remarked upon in the physics books I read.
The "frequency of electron transitions " means the number of times electrons make quantum transitions in a second and this defines the number of photons emitted per second, i.e. the intensity (quantity) of emission.
Thank you.  So the question stands with a slightly altered terminology:

"Where are we when we don't observe an increase/decrease in electron quantum transitions.  Are we in the local frame?"

The photon frequency associated with an individual electron transition defines the energy per photon - the quality of the emission.
Is the number of times electrons are making quantum transitions in a second defined by energy levels?
And does the number of times electrons make quantum transitions have a bearing on the energy level of the individual electron transitions that are defining the energy per photon?

... Does anyone have an answer?
Title: Re: Is there a discrepancy with the equivalence principle?
Post by: alancalverd on 11/05/2017 00:28:12
The simple answer is that the gravitational field warps the spacetime continuum. Remarkably, this result from GR seems to explain every experimantal result to date.
Therefore it is remarkable that while GR can predict and describe what will occur, it cannot describe 'why' it is occurring.  A fact that is widely remarked upon in the physics books I read.
It is true that gravitation is an anomalous force. What is remarkable is its predictability and universality, on the one hand, and on the other hand, the fact that such a simple concept as GR can predict such a subtle effect as time dilatation with altitude, with such precision.
Quote
The "frequency of electron transitions " means the number of times electrons make quantum transitions in a second and this defines the number of photons emitted per second, i.e. the intensity (quantity) of emission.
Thank you.  So the question stands with a slightly altered terminology:

"Where are we when we don't observe an increase/decrease in electron quantum transitions.  Are we in the local frame?"
The question remains meaningless, or at least ambiguous.
Quote

The photon frequency associated with an individual electron transition defines the energy per photon - the quality of the emission.
Is the number of times electrons are making quantum transitions in a second defined by energy levels?
And does the number of times electrons make quantum transitions have a bearing on the energy level of the individual electron transitions that are defining the energy per photon?
No and no. Quality and quantity are entirely separate.
Title: Re: Is there a discrepancy with the equivalence principle?
Post by: timey on 11/05/2017 01:20:46
It is true that gravitation is an anomalous force. What is remarkable is its predictability and universality, on the one hand, and on the other hand, the fact that such a simple concept as GR can predict such a subtle effect as time dilatation with altitude, with such precision.
Therefore there is no need to ask 'why' these physical processes occur?  I thought you went to CERN last year and knew everything now... inclusive of the fact that billions of money's are spent in the search for the 'why's' of the universe...

Quote
:timey
"Where are we when we don't observe an increase/decrease in electron quantum transitions.  Are we in the local frame?"
The question remains meaningless, or at least ambiguous
The clock's tick rate is dependent on the frequency of electron transitions.  Why would the question be meaningless?
Here I have said:
Quote
:timey
If one measures a clock from a position of higher gravity potential, it will be running slower, and then measures the same clock from a position of lower gravity potential, it will be running faster
...and
Quote
:timey
It is indeed quite clear to me, under the remit of conventional physics, that the higher potential position is measuring the clock in the lower potential by it's increased tick rate, and that the lower potential position is measuring the clock in the higher potential by it's decreased tick rate, and that it is all 3 clocks that are running at differing rates.
Do you disagree?

And what I want to discuss here is my idea for a physical mechanism that causes these clocks to tick at differing rates, and I am discussing this on the basis that GR is not providing one.
Is this understood?

Quote
:timey
Is the number of times electrons are making quantum transitions in a second defined by energy levels?
And does the number of times electrons make quantum transitions have a bearing on the energy level of the individual electron transitions that are defining the energy per photon?
No and no. Quality and quantity are entirely separate.
So - we know 'why' photons increase in energy when they are blue shifted, or decrease in energy when red shifted...via a calculation that converts pe to ke, or ke to pe.
But 'why' do the number of electron transitions per second differ in the differing GP's?
Conventional physics states that this is because time runs at differing rates at differing GP's, where I am looking at 'why' time runs at differing rates at differing GP's.
So - can 'you' tell me 'why' time runs at differing rates at differing GP's?
Can GR tell me the physical cause and effect mechanics of 'why' time runs at differing rates at differing GP's?
Can SR tell me the physical cause and effect mechanics of 'why' time runs at differing rates at differing speeds, relative to each other?
Can GR or SR tell me the physical cause and effect mechanics for gravitational attraction/acceleration/deceleration?
Title: Re: Is there a discrepancy with the equivalence principle?
Post by: alancalverd on 11/05/2017 07:45:24
Don't confuse "how" (physics) with "why" (philosophy).

GR states that all atomic clocks tick at the same rate, but look different when seen from a different viewpoint. That's why it's called relativity and not absolutivity. Then we explain and calculate the difference, and it turns out to be correct. Bingo - physics!

GR explains the equivalence between a gravitational field and an accelerating reference frame, but doesn't attempt to elicit the mechanism of gravitation, which remains to be discovered. No big deal: GR doesn't explain how rockets or cyclotrons work, but the predictions of an accelerating frame turn out to be correct. 

You still seem confused about electron transitions. I repeat: the number of quantum transitions per unit time determines the intensity (quantity) of a photon source; the energy of each transition determines the spectrum (quality) of the source. If you use the language of physics, people will be able to understand and therefore answer your questions, but if you talk about the frequency of electron transitions you will just confuse yourself and everyone else.

An old colleague used to explain photon spectra, Planck's photoelectrons, and all that stuff, by comparing a pint of beer (refreshing) with a pint of whisky (lethal). Same quantity, different quality.
Title: Re: Is there a discrepancy with the equivalence principle?
Post by: timey on 11/05/2017 12:42:22
I have no problem with the fact that a person ages differently at a GP relative to someone at a different GP. What I have never managed to understand is why you think they wouldn't according to current physics.
Because every time I try to discuss here the fact of seconds being of differing length in differing GP's, I am consistently told that this is only the case when viewed from the remote frame, and it has then been very difficult to continue the discussion in terms of talking about differing length seconds!!!
GR states that all atomic clocks tick at the same rate, but look different when seen from a different viewpoint. That's why it's called relativity and not absolutivity. Then we explain and calculate the difference, and it turns out to be correct. Bingo - physics!

Do you see what I mean Colin?

One cannot state that:
all atomic clocks tick at the same rate, but look different when seen from a different viewpoint
...and also state that:
a person ages differently at a GP relative to someone at a different GP.

... Because that is a contradiction.

If a person ages differently at a GP relative to someone at a different GP, then atomic clocks not only look different when seen from a different viewpoint, they are actually physically ticking differently in differing GP's. 
Title: Re: Is there a discrepancy with the equivalence principle?
Post by: timey on 11/05/2017 14:32:09
You still seem confused about electron transitions. I repeat: the number of quantum transitions per unit time determines the intensity (quantity) of a photon source; the energy of each transition determines the spectrum (quality) of the source.
Quote
:wiki
An atomic clock is a clock device that uses an electronic transition frequency in the microwave, optical, or ultraviolet region[2] of the electromagnetic spectrum of atoms as a frequency standard for its timekeeping element.
Quote
: wiki
A caesium atomic clock is a primary frequency standard in which electronic transitions between the two hyperfine ground states of caesium-133 atoms are used to control the output frequency.
Quote
:wiki
That value was chosen so that the caesium second equalled, to the limit of human measuring ability in 1960 when it was adopted, the existing standard ephemeris second based on the Earth's orbit around the Sun
Quote
:wiki
general relativity gives us gravitational time dilation. Briefly, a clock in a stronger gravitational field (e.g. closer to a planet) will appear to tick more slowly. People holding these clocks (i.e. those inside and outside the stronger field) would all agree on which clocks appear to be going faster.
Quote
  :New Scientist
Anil Ananthaswamy

'OUR most accurate clocks are probing a key tenet of Einstein’s theory of relativity: the idea that time isn’t absolute. Any violation of this principle could point us to a long-sought theory that would unite Einstein’s ideas with quantum mechanics.

Special relativity established that the laws of physics are the same for any two observers moving at a constant speed relative to each other, a symmetry called Lorentz invariance. One consequence is that they would observe each other’s clocks running at different rates. Each observer would regard themselves as stationary and see the other observer’s clock as ticking slowly – an effect called time dilation.

Einstein’s general relativity compounds the effect. It says that the clocks would run differently if they experience different gravitational forces.

For two decades, comparing atomic clocks aboard GPS satellites with those on Earth have helped test the effect – and always confirmed it. But since any deviation from relativity would be very subtle, we might need a more precise instrument to find it.

Most atomic clocks rely on the frequency of the microwave radiation emitted when electrons in caesium-133 atoms change energy states.

I repeat:
"An atomic clock is a clock device that uses an electronic transition frequency as a frequency standard for its timekeeping element."
"electronic transitions between the two hyperfine ground states of caesium-133 atoms are used to control the output frequency."
"Most atomic clocks rely on the frequency of the microwave radiation emitted when electrons in caesium-133 atoms change energy states."

If you use the language of physics, people will be able to understand and therefore answer your questions, but if you talk about the frequency of electron transitions you will just confuse yourself and everyone else.

My use of the language 'a change in the frequency of electron transitions' to describe the tick rate of a time dilated clock is exactly the language of physics.  It is 'you' Alan who appears to be confused.

Don't confuse "how" (physics) with "why" (philosophy).
Do not confuse the 'why' of physics as 'being' philosophy.
Title: Re: Is there a discrepancy with the equivalence principle?
Post by: nilak on 11/05/2017 15:54:56
I understand Alan tries to make a distinction between the number of electrons per second that transition between energy levels and the microwave radiation frequency emmited by a single transition. You can have for example 100 electrons changing levels per second meaning a frequency of 100Hz, but the microwave radiation can be for instance 9 GHz which is the clock rate.
Title: Re: Is there a discrepancy with the equivalence principle?
Post by: alancalverd on 11/05/2017 16:42:16
Your language must be precise if you are to be understood, and it is a great help to your own understanding to be absolutely unambiguous as to what you are talking about.

The frequency associated with an electron quantum transition is what Wikipedia loosely calls  an  electronic transition frequency, though it is more properly (and less ambiguously) termed electron transition energy. 

The frequency of electron transitions means how many electrons change state per second, regardless of the energy of each transition. Quite a different thing.

Wikipedia half-correctly states  a clock in a stronger gravitational field (e.g. closer to a planet) will appear to tick more slowly. but unless you state to whom it will appear, and slower than what, the statement is meaningless. Once you complete the statement, you have the prediction of general relativity.

And Wikipedia isn't quite correct about the cesium clock. In practice the critical microwave frequency is absorbed, not emitted, by the hyperfine shift. Why do I care? Because the drummer in one of my former bands was the principal physicist responsible for developing the original Essen cesium clock into a primary standard - and he is an excellent timekeeper!
Title: Re: Is there a discrepancy with the equivalence principle?
Post by: timey on 11/05/2017 17:35:32
On the basis that you seem to understand that it is the number of electron transitions that is responsible for the tick rate of the clock, I really don't understand why I even have to discuss the matter at-all, let alone argue over which language that I use to describe the tick rate of the clock.  Just seems like a trite waste of time to me!
The maths that predict that a clock will run faster in the higher potential compared to the lower potential say that the number of electron transitions that occur in the higher potential will be greater, meaning that there will be more of them occurring within the time period of a length of a second as measured in the lower potential.

A person in the higher potential will age quicker in the higher potential compared to a person in the lower potential.
Do you have anything to say in answer to this post:
https://www.thenakedscientists.com/forum/index.php?topic=69800.msg514132#msg514132
Title: Re: Is there a discrepancy with the equivalence principle?
Post by: timey on 11/05/2017 17:57:18
Not really correctly defined here by live science because the atoms are pushed up a chamber where they are excited by a microwave beam, however...
Quote
:live science
But even the best mechanical pendulums and quartz crystal-based clocks develop discrepancies. Far better for timekeeping is the natural and exact "vibration" in an energized atom.

When exposed to certain frequencies of radiation, such as radio waves, the subatomic particles called electrons that orbit an atom's nucleus will "jump" back and forth between energy states. Clocks based on this jumping within atoms can therefore provide an extremely precise way to count seconds.

It is no surprise then that the international standard for the length of one second is based on atoms. Since 1967, the official definition of a second is 9,192,631,770 cycles of the radiation that gets an atom of the element called cesium to vibrate between two energy states.

Inside a cesium atomic clock, cesium atoms are funneled down a tube where they pass through radio waves . If this frequency is just right 9,192,631,770 cycles per second then the cesium atoms "resonate" and change their energy state.

A detector at the end of the tube keeps track of the number of cesium atoms reaching it that have changed their energy states. The more finely tuned the radio wave frequency is to 9,192,631,770 cycles per second, the more cesium atoms reach the detector.

The detector feeds information back into the radio wave generator. It synchronizes the frequency of the radio waves with the peak number of cesium atoms striking it. Other electronics in the atomic clock count this frequency. As with a single swing of the pendulum, a second is ticked off when the frequency count is met.

Correctly defined here, but with no reference to the fact of measuring the number of electron transitions 'in' a reference frame via the tick rate of a clock in that reference frame having a bearing on the measurement...
Quote
: educate/explore
Atoms as clocks

Every atom is composed of a nucleus, which contains the atom’s protons and neutrons (collectively known as nucleons). Orbiting that nucleus are the atom’s electrons, which occupy different orbits, or energy levels.

By absorbing or releasing exactly the right amount of energy, the electrons can ‘jump’ from one energy level to another. This is called a transition. The electrons absorb energy to move to a higher energy level (away from the nucleus), and release energy to move down an energy level (towards the nucleus).

The energy released or absorbed in these transitions takes the form of electromagnetic radiation (e.g. visible light or microwaves). The same amount of energy is released every time the same transition occurs, no matter where or how many times it is measured.

As with all waves, the radiation has a certain frequency (i.e., it completes a certain number of full waves in a second, similar to the way a pendulum completes a certain number of swings in a minute) and this frequency can be measured. This means that a clock can be based on the wave frequency of an electron’s transition energy in an atom, in a similar way to a clock based on the swinging of a pendulum.

Why do we use caesium?

The caesium atom defines the SI second. The second is 9 192 631 770 periods of the electromagnetic radiation emitted or absorbed by the ground state hyperfine transition of the caesium atom. This means that a second is the amount of time it takes for the radiation from this transition to complete 9 192 631 770 full waves.

As with all atoms, no matter where or how it is measured this number will never change, meaning that it’s a far more reliable method of timekeeping than the Sun’s movement in the sky.

Measuring the second in a caesium fountain atomic clock

An atomic fountain clock has three stages:

Six lasers placed at right angles to each other (aimed above, below, left of, right of, in front of and behind the target) are fired at a group of caesium atoms. This is known as an optical trap: the light from the six lasers pushes the caesium atoms closer together, stopping them moving to the point where they almost stop vibrating at all. As both a particle and a wave, light has momentum (just like any other object that is moving), and is able to push very small objects such as atoms. Since atomic vibrations are what we feel as heat, the caesium atoms become ultra-cold, reaching temperatures of around one microKelvin - a tiny fraction of a degree above absolute zero (-273.15 °C).
Once the atoms have been cooled down, the lasers above and below them are used to launch them upwards inside the fountain’s microwave chamber, and the atoms then fall back down under gravity. This launch-and-fall movement is why the clock is referred to as a ‘fountain’. The chamber uses microwave radiation to cause the caesium atoms’ electrons to move between two specific energy levels as they fly up and fall down through it.
Finally, once the atoms have completed their flight, the energy levels of the electrons can be measured through fluorescence – atoms with electrons in different energy levels will emit different radiation patterns when probed with a laser.
This whole process takes about a second, and is repeated over and over with different microwave frequencies until the frequency that causes the maximum number of caesium electrons to change energy levels is found. This frequency is the resonant frequency, and this is the frequency that is used to define the SI second. As caesium fountain clocks are improved, the microwave frequency can be more finely tuned and the SI second can be even more accurately defined.
Quote
:nist.gov
All clocks must have a regular, constant or repetitive process or action to mark off equal increments of time. Examples include the daily movement of the sun across the sky, a swinging pendulum or vibrating crystal. In the case of atomic clocks, the beat is kept by a transition between two energy levels in an atom.

Place the clock in a higher gravity potential and that beat is increased compared to the lower potential.

A person will age faster if they are with a clock that has a faster beat.

I am exploring where in the higher potential the energy comes from to cause the clock to beat faster, and the person to age faster.
This is a perfectly reasonable exploration because Einstein's relativity is not compatible with quantum mechanics.
Alan - if you are not interested in making an exploration of this type, could I ask you, with all due respect NOT to post here anymore?
Title: Re: Is there a discrepancy with the equivalence principle?
Post by: timey on 11/05/2017 22:52:40
In practice the critical microwave frequency is absorbed, not emitted, by the hyperfine shift.
This frequency is the resonant frequency of 9,192,631,770 cycles per second and this is the frequency that is used to define the SI second.

In the higher potential, there will be more wave cycles per second as per the length of a second in the lower potential, i.e. a resonant frequency that is higher than the frequency of 9,192,631,770 cycles per SI second, although if one measures in one's own frame of reference, one will always measure 9,192,631,770 cycles per the second of their position in the gravity field.

GR states that clocks run differently if they experience different gravitational forces.  People holding these clocks (i.e. those inside and outside the stronger field) would all agree on which clocks appear to be going faster.
Therefore while GR states that the local reference frame will notice nothing different about their own clock, GR also categorically states that time is physically running faster in the higher potential compared to in the lower potential.

Everything in this post so far is conventional physics.

What I want to talk about that is not conventional physics is attributing the higher resonant frequency in the higher gravity potential to an increase in energy, because an increase in frequency must be accompanied by an increase in energy.  No energy is being artificially added to the clock in the higher potential, so I am looking at the mgh equation to add energy to cause the change in the critical microwave frequency.
The atoms that this critical microwave frequency is responsible for resonating will also be changed in their energy levels, otherwise the higher frequency would not cause a jump between ground states.
My reasoning is that if the atoms are changed in their energy levels, then the emitting source of the critical microwave beam will also be changed in energy level and will be emitting a higher frequency microwave beam, which is what is observed when the critical microwave frequency is higher than 9,192,631,770 cycles per SI second...

Alan - I really do not know how I could be any clearer.  Conventional physics states that these physical occurances occur because time runs faster at elevation...
It is my intention to describe physical cause and effect mechanics for 'why' time runs faster at elevation, and this description is essential for my models description of 'why' gravitational attraction occurs.
GR explains the equivalence between a gravitational field and an accelerating reference frame, but doesn't attempt to elicit the mechanism of gravitation, which remains to be discovered
Precisely...
And... I actually thought that this is what you were going to be helping me with by applying your mathematical skills to my ideas to establish if my ideas are mathematically viable or not for a description of physical cause and effect mechanics for gravitational attraction/acceleration/deceleration.
This is the only 'question' that I am raising here.

As I say, I'll see how Ethos wants to take it forward. I'll help him with drafting if he needs it.

Thanks.  I appreciate the help.
Title: Re: Is there a discrepancy with the equivalence principle?
Post by: alancalverd on 11/05/2017 23:48:25
On the basis that you seem to understand that it is the number of electron transitions that is responsible for the tick rate of the clock, I really don't understand why I even have to discuss the matter at-all, let alone argue over which language that I use to describe the tick rate of the clock. 
It isn't. That's why we have to get the language straight, so I can work out what it is that you don't understand or have entirely misconceived, before we can make any progress. 

Quote
A person in the higher potential will age quicker in the higher potential compared to a person in the lower potential.
Not true. His ageing will match his clock, so he will live for 70 years, just like his colleague on earth. But his earth brother will see him die because space years as seen from earth are shorter than earth years. 
Quote
Do you have anything to say in answer to this post:
https://www.thenakedscientists.com/forum/index.php?topic=69800.msg514132#msg514132
Only that it is too imprecise to answer. But I live in hope.
Title: Re: Is there a discrepancy with the equivalence principle?
Post by: Colin2B on 12/05/2017 00:21:08

I notice Alan has posted while I was typing, but I'll post this anyway. Hope it helps.


. Because that is a contradiction.
It's not a contradiction, except in the way you are interpreting it.



On the basis that you seem to understand that it is the number of electron transitions that is responsible for the tick rate of the clock, I really don't understand why I even have to discuss the matter at-all, let alone argue over which language that I use to describe the tick rate of the clock.  Just seems like a trite waste of time to me!
The reason Alan is continuing this "trite waste of time" is that he does not "understand that it is the number of electron transitions that is responsible for the tick rate of the clock", neither has he said that it is.


This is the point I raised earlier, we continually seem to misunderstand each other due to differences of terminology and interpretation.
I do understand that you want to discuss your model, not conventional physics. However, at some stage you will need to present your theory to physicists working in this area and using incorrect terminology will cause your theory to be dismissed. Meaningful maths cannot be developed out of incorrect assumptions.


I will add one comment on the position of conventional physics:

Conventional physics states that these physical occurances occur because time runs faster at elevation...
It is my intention to describe physical cause and effect mechanics for 'why' time runs faster at elevation,
GR states that the time differences we observe are due to a deformation or curving of spacetime and this curving is also the reason for what we observe as the force of gravity.
You are aware that if we stress, say, a metal it will deform. Similarly stress will deform spacetime and conventional physics states that a number of of things stress spacetime including momentum, pressure and energy,  in particular energy in the form of mass - bearing in mind that mass can be viewed as a substantial container of energy.
Just a thought.

Title: Re: Is there a discrepancy with the equivalence principle?
Post by: timey on 12/05/2017 00:31:54
It isn't.
Yes - you are right, which is why I amended my descrription below 'back' to talking about the frequency of wave cycles that define the SI second, but you just ignore that... Just as you ignore the fact that a description of E=fh is holding frequency, Planck's h constant and the measure of energy that comprises a joule relative to an SI second, which is the mathematical description of stating changes 'as compared to'...

In practice the critical microwave frequency is absorbed, not emitted, by the hyperfine shift.
This frequency is the resonant frequency of 9,192,631,770 cycles per second and this is the frequency that is used to define the SI second.

In the higher potential, there will be more wave cycles per second as per the length of a second in the lower potential, i.e. a resonant frequency that is higher than the frequency of 9,192,631,770 cycles per SI second, although if one measures in one's own frame of reference, one will always measure 9,192,631,770 cycles per the second of their position in the gravity field.

GR states that clocks run differently if they experience different gravitational forces.  People holding these clocks (i.e. those inside and outside the stronger field) would all agree on which clocks appear to be going faster.
Therefore while GR states that the local reference frame will notice nothing different about their own clock, GR also categorically states that time is physically running faster in the higher potential compared to in the lower potential.

Everything in this post so far is conventional physics.

What I want to talk about that is not conventional physics is attributing the higher resonant frequency in the higher gravity potential to an increase in energy, because an increase in frequency must be accompanied by an increase in energy.  No energy is being artificially added to the clock in the higher potential, so I am looking at the mgh equation to add energy to cause the change in the critical microwave frequency.
The atoms that this critical microwave frequency is responsible for resonating will also be changed in their energy levels, otherwise the higher frequency would not cause a jump between ground states.
My reasoning is that if the atoms are changed in their energy levels, then the emitting source of the critical microwave beam will also be changed in energy level and will be emitting a higher frequency microwave beam, which is what is observed when the critical microwave frequency is higher than 9,192,631,770 cycles per SI second...

Alan - I really do not know how I could be any clearer.  Conventional physics states that these physical occurances occur because time runs faster at elevation...
It is my intention to describe physical cause and effect mechanics for 'why' time runs faster at elevation, and this description is essential for my models description of 'why' gravitational attraction occurs.
GR explains the equivalence between a gravitational field and an accelerating reference frame, but doesn't attempt to elicit the mechanism of gravitation, which remains to be discovered
Precisely...
And... I actually thought that this is what you were going to be helping me with by applying your mathematical skills to my ideas to establish if my ideas are mathematically viable or not for a description of physical cause and effect mechanics for gravitational attraction/acceleration/deceleration.
This is the only 'question' that I am raising here.

As I say, I'll see how Ethos wants to take it forward. I'll help him with drafting if he needs it.

Thanks.  I appreciate the help.
That's why we have to get the language straight, so I can work out what it is that you don't understand or have entirely misconceived, before we can make any progress.
I haven't misunderstood any damn thing.  I have spent 2 years where you completely refuse to talk about seconds that are differing in length compared to the length of an SI second.
His ageing will match his clock, so he will live for 70 years, just like his colleague on earth. But his earth brother will see him die because space years as seen from earth are shorter than earth years.
The Earth brother will see him die because space years compared to Earth years 'are' shorter. (where m doesn't =0 in my model)
What is your problem?
Title: Re: Is there a discrepancy with the equivalence principle?
Post by: timey on 12/05/2017 01:43:10
I notice Alan has posted while I was typing, but I'll post this anyway. Hope it helps.
. Because that is a contradiction.
It's not a contradiction, except in the way you are interpreting it.
It wouldn't be a contradiction if Alan was not refusing to talk in terms of seconds differing in length.


On the basis that you seem to understand that it is the number of electron transitions that is responsible for the tick rate of the clock, I really don't understand why I even have to discuss the matter at-all, let alone argue over which language that I use to describe the tick rate of the clock.  Just seems like a trite waste of time to me!
The reason Alan is continuing this "trite waste of time" is that he does not "understand that it is the number of electron transitions that is responsible for the tick rate of the clock", neither has he said that it is.
 This is the point I raised earlier, we continually seem to misunderstand each other due to differences of terminology and interpretation.
I do understand that you want to discuss your model, not conventional physics. However, at some stage you will need to present your theory to physicists working in this area and using incorrect terminology will cause your theory to be dismissed. Meaningful maths cannot be developed out of incorrect assumptions.
No - but he did refuse to talk in terms of it being to do with wave cycles earlier, which quite clearly it is.  My description was amended, please read post above.

I will add one comment on the position of conventional physics:
Conventional physics states that these physical occurrences occur because time runs faster at elevation...
It is my intention to describe physical cause and effect mechanics for 'why' time runs faster at elevation,
GR states that the time differences we observe are due to a deformation or curving of spacetime and this curving is also the reason for what we observe as the force of gravity.
You are aware that if we stress, say, a metal it will deform. Similarly stress will deform spacetime and conventional physics states that a number of of things stress spacetime including momentum, pressure and energy,  in particular energy in the form of mass - bearing in mind that mass can be viewed as a substantial container of energy.
Just a thought.
Yes - and what I am introducing as an addition to these concepts is a) that it is the other way round, where the deformation or curving of spacetime is due to the time differences, where the time differences are the 3rd aspect of time dilation that my model adds that give physical cause and effect mechanics for the acceleration/deceleration motions that are induced by this distortion of time (where we were talking about E=hf for light in the gravity field before) and b) that the attractive directional force is due to the GR time dilation that we are discussing at present, where electron 'action' within the atom is increased causing there to be more magnetic moments as compared to the lower potential. The discussion is only possible if we can talk in terms of seconds becoming longer or shorter, which Alan doesn't seem willing to do.  It would seem that he is too hung up on a notion of my having misinterpreted the remit of conventional physics to conduct an interesting discussion, or that he quite simply does not understand that GR states time as occurring at differing rates.
You are aware that if we stress, say, a metal it will deform. Similarly stress will deform spacetime
Of course I am, I was making direct reference to such as a side issue in this post that no one cares to discuss:
https://www.thenakedscientists.com/forum/index.php?topic=69800.msg514035#msg514035

With regards to a conversation that would also be of interest to me I will repost this reposted question that no-one seems to answer:
Quote
:timey
As mentioned a few posts ago - the only reason (that I can define, if anyone can add another) within conventional physics to not state potential energy as active within mass is due to the remit of only using the value of kinetic energy to calculate relativistic mass value.  (comment would be appreciated)

If one has a problem with the terminology of my descriptions, then why not provide me with a description of the scenario in the words you would use as an example, then I could repost in the terms that you are used to hearing.  Just a time saving thought...excuse the pun. 
Title: Re: Is there a discrepancy with the equivalence principle?
Post by: alancalverd on 12/05/2017 07:42:35
Imagine you can shine a laser upwards from an origin O on the surface of the earth. Not difficult. Now measure the wavelength received at different altitudes A and B where B>A. We know you will see a red shift S which increases with observer altitude. As far as B is concerned, the red shift between A and B is a shift in the wavelength of a photon whose source has no mass but is at a lower gravitational potential.   Mathematically, if

SAB = SOB - SOA

then the mass of the source is irrelevant.

Note that I am measuring  wavelength, using a diffraction grating, so no possible effect of gravitation on my measuring apparatus.

It's a simple experiment that will challenge your hypothesis.
Title: Re: Is there a discrepancy with the equivalence principle?
Post by: timey on 12/05/2017 12:54:46
I went through all that with Colin earlier this thread.  The light is red shifted as it gets further away from O.  It's wavelength gets longer.  My hypothesis is that where space is m=0 that the rate of time is getting slower further away from O.  The light was emitted at a certain wavelength in a frame of reference, which we will call reference frame A, where a second is of a certain length. We will call the second at the point of the lights emission second 0.   At elevation 1 divide the extra length in wavelength by the speed of light for a time period and add this time period to the length of second 0 in order to know the length of second 1.  At elevation 2 divide the extra length in wavelength as compared to elevation 1 by the speed of light for a time period and add this time period to the length of second 1 in order to know the length of second 2, and so on.

Place your light source emitter at elevation and point the laser at O.  Now you will observe from your position below that the light is blue shifted.  But is the light source emitter emitting light with the same wavelength as it was emitting when it was in reference frame A?

The atomic clock would suggest that the light emitter at elevation is not emitting light with the same wavelength that it was in reference frame A.  The atomic clock suggests that the light source emitter will be emitting light with a shorter wavelength in the elevated position compared to the wavelength of the light the light source was emitting in reference frame A.

Quote
:educate/explore
Atoms as clocks

Every atom is composed of a nucleus, which contains the atom’s protons and neutrons (collectively known as nucleons). Orbiting that nucleus are the atom’s electrons, which occupy different orbits, or energy levels.

By absorbing or releasing exactly the right amount of energy, the electrons can ‘jump’ from one energy level to another. This is called a transition. The electrons absorb energy to move to a higher energy level (away from the nucleus), and release energy to move down an energy level (towards the nucleus).

The energy released or absorbed in these transitions takes the form of electromagnetic radiation (e.g. visible light or microwaves). The same amount of energy is released every time the same transition occurs, no matter where or how many times it is measured.

As with all waves, the radiation has a certain frequency (i.e., it completes a certain number of full waves in a second, similar to the way a pendulum completes a certain number of swings in a minute) and this frequency can be measured. This means that a clock can be based on the wave frequency of an electron’s transition energy in an atom, in a similar way to a clock based on the swinging of a pendulum.

Why do we use caesium?

The caesium atom defines the SI second. The second is 9 192 631 770 periods of the electromagnetic radiation emitted or absorbed by the ground state hyperfine transition of the caesium atom. This means that a second is the amount of time it takes for the radiation from this transition to complete 9 192 631 770 full waves.

As with all atoms, no matter where or how it is measured this number will never change, meaning that it’s a far more reliable method of timekeeping than the Sun’s movement in the sky.

Measuring the second in a caesium fountain atomic clock

An atomic fountain clock has three stages:

Six lasers placed at right angles to each other (aimed above, below, left of, right of, in front of and behind the target) are fired at a group of caesium atoms. This is known as an optical trap: the light from the six lasers pushes the caesium atoms closer together, stopping them moving to the point where they almost stop vibrating at all. As both a particle and a wave, light has momentum (just like any other object that is moving), and is able to push very small objects such as atoms. Since atomic vibrations are what we feel as heat, the caesium atoms become ultra-cold, reaching temperatures of around one microKelvin - a tiny fraction of a degree above absolute zero (-273.15 °C).
Once the atoms have been cooled down, the lasers above and below them are used to launch them upwards inside the fountain’s microwave chamber, and the atoms then fall back down under gravity. This launch-and-fall movement is why the clock is referred to as a ‘fountain’. The chamber uses microwave radiation to cause the caesium atoms’ electrons to move between two specific energy levels as they fly up and fall down through it.
Finally, once the atoms have completed their flight, the energy levels of the electrons can be measured through fluorescence – atoms with electrons in different energy levels will emit different radiation patterns when probed with a laser.
This whole process takes about a second, and is repeated over and over with different microwave frequencies until the frequency that causes the maximum number of caesium electrons to change energy levels is found. This frequency is the resonant frequency, and this is the frequency that is used to define the SI second. As caesium fountain clocks are improved, the microwave frequency can be more finely tuned and the SI second can be even more accurately defined.

Do you have a problem with any of the terminology that has been used in this quote?

Because if not then I shall use the terminology in this quote for further description...
Title: Re: Is there a discrepancy with the equivalence principle?
Post by: alancalverd on 12/05/2017 16:07:12
I went through all that with Colin earlier this thread.  The light is red shifted as it gets further away from O.  It's wavelength gets longer.  My hypothesis is that where space is m=0 that the rate of time is getting slower further away from O. 

We know time slows down at a low gravitational potential, as predicted by GR and measured by GPS clocks, and you now say it also slows down at the maximum gravitational potential (deep space, gravitational potential = 0). Seems very unlikely. At what point is time running at its fastest?
Title: Re: Is there a discrepancy with the equivalence principle?
Post by: timey on 12/05/2017 18:15:21
I have simply made a distinction between what time is doing where m doesn't equal 0, i.e. GR and SR time dilations, and what time is doing where m does equal 0, i.e. the 3rd aspect of time dilation that my model adds that cannot be measured directly because no measuring device is m=0...
This 3rd aspect of time dilation can be measured indirectly via the wavelength of gravitationally shifted light, and via the rate that all m freefalls towards M, or via the rate that the motion of m direction away from M is decelerated.

As I asked in my last post, did you have a problem with any of the terminology used in the quote I provided?

The point where time will run at it's fastest will be just before my cyclic model's rendition of a Big Bang.
Title: Re: Is there a discrepancy with the equivalence principle?
Post by: guest4091 on 12/05/2017 19:06:14
timey quotes:
Quote
Therefore conventional physics has, despite the conservation of energy law stating that it is the sum total of potential and kinetic energy that must be conserved, has chosen to state potential energy as an energy that cannot be placed as residing anywhere.

A test object receives energy from the g-field, therefore the PE is the field yet is still part of the total energy of the mass M. (Unless you want to regress to 'action at a distance'). Remove M, remove the field!
There is no theory that explains how the energy is transferred from M to the surrounding space, and that's where I think you are underestimating the challenge.
Space will have to be an invisible structure capable of retaining energy, but different from the ether of 1900. or maybe a process totally foreign to our thinking.

GR uses a geometric deforming of spacetime to represent the behavior of a test object  but that just puts off the question/explanation. What is deformed if space is a nothingness between bits of matter?
As a hypothetical example, imagine a g-field composed of vectors, all the same length, all pointing to earth center, but increasing in density toward earth center, i.e. inverse square rule. The object absorbs an impulse and change of direction at an increasing frequency as it approaches. The space is not curved, but the path is. The perception is also altered in GR just as in SR, so your local clock will seem 'normal', and you only see differences when comparing your clock to a remote clock.
 
Quote
Under the remit of this system Planck's h constant becomes a function of time dilation, where it is changes in energy level that are causing changes in the rate of time.
 

You can't redefine h as a function. The explanation and predictions of 'black body radiation' and the 'photoelectric effect' depend on it being a constant. Changes in the rate of time depends on position in the g-field. The equivalence principle states acceleration is equal to a uniform g-field for small intervals of space and time. Imagine the clock at a random position in the g-field, in a box accelerating upward at g. The (internal) light component will require more time to ascend, i.e. red shift, as viewed by an occupant at the top of the box. This is equivalent to the box resting on the surface.
Title: Re: Is there a discrepancy with the equivalence principle?
Post by: Colin2B on 12/05/2017 19:47:25
I went through all that with Colin earlier this thread.  The light is red shifted as it gets further away from O.  It's wavelength gets longer.  My hypothesis is that where space is m=0 that the rate of time is getting slower further away from O. 
You may have stated that, but I don't recall agreeing that it is the case. I probably didn't bother to respond.
Title: Re: Is there a discrepancy with the equivalence principle?
Post by: timey on 12/05/2017 20:50:46
I went through all that with Colin earlier this thread.  The light is red shifted as it gets further away from O.  It's wavelength gets longer.  My hypothesis is that where space is m=0 that the rate of time is getting slower further away from O. 
You may have stated that, but I don't recall agreeing that it is the case. I probably didn't bother to respond.

I didn't state it.  I was asking for advice on the mathematical description of such, where you said that I was on safe ground with the equation E=hf because that is how conventional physics describes the energy of light.

Furthermore I don't expect anybody here to agree with what I am saying.  There is no agreeing or disagreeing on how the universe works.  That would be presumptuous.  All I that is required is to understand if the mathematics for the ideas that I am suggesting can be mathematically viable to describe observation.  If they are then it is only by experiment (and my model has it's suggested experiment to prove or disprove itself as is required of any serious proposal) that one will see if the universe agrees or not.

You did bother to respond, in that you were discussing the related maths with me.  Am I the only person here who can actually remember the details of previous conversations?  I can indeed remember each and every word that you have written in response to me Colin, this being because I can be bothered. It upsets me that you be so dismissive.
Title: Re: Is there a discrepancy with the equivalence principle?
Post by: alancalverd on 12/05/2017 21:12:54
Quote
The energy released or absorbed in these transitions takes the form of electromagnetic radiation (e.g. visible light or microwaves). The same amount of energy is released every time the same transition occurs, no matter where or how many times it is measured.

That's the key point that you disagree with.
Title: Re: Is there a discrepancy with the equivalence principle?
Post by: Colin2B on 12/05/2017 22:15:00
I didn't state it.  I was asking for advice on the mathematical description of such, where you said that I was on safe ground with the equation E=hf because that is how conventional physics describes the energy of light.

It upsets me that you be so dismissive.
I wasn't being dismissive, I was saying I didn't respond. The E=hf was in response to the energy of light, not your description of redshift.
Title: Re: Is there a discrepancy with the equivalence principle?
Post by: timey on 12/05/2017 22:44:08
timey quotes:
Quote
Therefore conventional physics has, despite the conservation of energy law stating that it is the sum total of potential and kinetic energy that must be conserved, has chosen to state potential energy as an energy that cannot be placed as residing anywhere.
A test object receives energy from the g-field, therefore the PE is the field yet is still part of the total energy of the mass M. (Unless you want to regress to 'action at a distance'). Remove M, remove the field!
Yes - if M is removed, the field is removed and the field then becomes associated with the test particle.
 
There is no theory that explains how the energy is transferred from M to the surrounding space, and that's where I think you are underestimating the challenge.
There is no theory that explains how the energy is transferred from the field to the test particle either...
My model does not attempt to explain how M transfers energy to the field, but it does attempt to explain how that energy accelerates m in the field.  My model also doesn't attempt to describe how the field transfers energy to the test particle, but it does attempt to describe the 'action' in the test particle that is the result of the energy being transferred.

Space will have to be an invisible structure capable of retaining energy, but different from the ether of 1900. or maybe a process totally foreign to our thinking.
My theory states the phenomenon of time as being a by-product of energy.  Differing rates of time are an invisible structure.
GR uses a geometric deforming of spacetime to represent the behavior of a test object  but that just puts off the question/explanation. What is deformed if space is a nothingness between bits of matter?
My model uses a temporal deforming of spacetime to represent the behavior of the test particle.

As a hypothetical example, imagine a g-field composed of vectors, all the same length, all pointing to earth center, but increasing in density toward earth center, i.e. inverse square rule. The object absorbs an impulse and change of direction at an increasing frequency as it approaches. The space is not curved, but the path is.
You are taking about the rate of acceleration on approach to body M.
The object absorbes an impulse and is decreasing in it's own frequency (decreasing rate of time) as it approaches if it is mass.
The object absorbes an impulse and is increasing in it's own frequency (increasing rate of time in space) as it approaches if it is light.
If one uses the gravitational red shift equation to work backwards from observation position and calculate the blue shift of the approaching light, (because a progression of blue shift is the reverse of a progression of red shift), where one divides each bit of length of increase in red shifted wavelength by the speed of light and add the resulting time period to the time period of the second before... (as described in this post https://www.thenakedscientists.com/forum/index.php?topic=69800.msg514211#msg514211) ... then these time differences in the gravitational field will describe, not the reduction in frequency (rate of time) observed of the mass as it approaches, but the acceleration of the rate of motion that the object of mass is falling at, i.e. gravitational acceleration/free fall.

The perception is also altered in GR just as in SR, so your local clock will seem 'normal', and you only see differences when comparing your clock to a remote clock.
Which is unchanged in my model, but my altered version of the remit of SR should give more accurate time meshing results.
Quote
:timey
Under the remit of this system Planck's h constant becomes a function of time dilation, where it is changes in energy level that are causing changes in the rate of time.
 
You can't redefine h as a function. The explanation and predictions of 'black body radiation' and the 'photoelectric effect' depend on it being a constant.
I think you need to throw back in time to the ultraviolet catastrophe and before Planck invented the h constant to 'iron out' his data.  The problem with the ultra violet catastrophe was (in brief and simplified) that it takes more energy to achieve the transitions of the higher frequencies of light than it does to achieve the transitions of the lower frequencies.  So Planck worked out that if he chopped the input energy into equal packages and multiplied by the number of wave cycles per second that he could achieve the data curve he was seeking.
As an alternative, (although Planck's system is more dexterous in it's applications inclusive of my model's mathematics interpreted as a time function), as frequency increases simply shorten the length of a second so that the wave cycles remain a constant number, and the energy additions will be rendered as a linear progression.
Title: Re: Is there a discrepancy with the equivalence principle?
Post by: timey on 12/05/2017 22:45:59
Quote
The energy released or absorbed in these transitions takes the form of electromagnetic radiation (e.g. visible light or microwaves). The same amount of energy is released every time the same transition occurs, no matter where or how many times it is measured.

That's the key point that you disagree with.

No - I don't disagree with anything in any of that quote.
Quote
The same amount of energy is released every time the same transition occurs, no matter where or how many times it is measured.

Absolutely, because where-ever one measures one will be measuring by the rate of time of the local clock, and any and all atom's 'action' in that local will be affected as the clock's action is.

Edit: Phyti - I answered your post in the post before this one.
Title: Re: Is there a discrepancy with the equivalence principle?
Post by: alancalverd on 13/05/2017 07:30:11
Compare 
Quote
any and all atom's 'action' in that local will be affected as the clock's action is.
with
Quote
The same amount of energy is released every time the same transition occurs, no matter where or how many times it is measured.

Note the contradiction between "will be affected" and "the same....no matter where", and recall that my thought experiment was measuring energy with a diffraction grating, not a clock.
Title: Re: Is there a discrepancy with the equivalence principle?
Post by: timey on 13/05/2017 17:00:40
Well it is an interesting thought experiment Alan, but mostly because I have been scratching my head for hours trying for the life of me to work out why you think this is relevant to the 'action' of an atom placed at differing GP's of Earth, or the 'gravitational' shift in light.

Clearly the relationship between frequency and wavelength is upheld within the diffraction grating assembly via length of a groove and the density of grooves, where the light is diffracted into its constituent wavelength components.
Clearly the spectrum of light is used in the study of stars where, in accordance with Wien’s displacement law, the star with the longest wavelength has the lowest temperature and vice versa where the spectrum gives information on the mass, density of each element, intensity of the magnetic field, and changes occurring in a star.

Quote
:brighthub.com
A spectrum also helps to prove the dynamic nature of universe. As seen earlier, each celestial object has its own characteristic spectrum. The spectral lines corresponding to an elements present in a galaxy can be identified by comparing it with the actual spectral lines recorded for each element on Earth. If the spectral lines observed from a galaxy shows a shift towards the red colour then it is called redshift.
Whereas, if the shift is towards blue, then it is a blueshift. According to Doppler Effect, if a source is moving away from an observer then there is an apparent increase in wavelength of the light emitted by that source, this results in a redshift in its spectrum. Likewise, in a blueshift the source is moving towards the observer, hence a shift towards shorter wavelengths. Spectral lines obtained from most of the galaxies show a red shift, whereas the number of galaxies which show a blue shift in its spectral lines are only few, Andromeda is one among them. Thus, this shift in spectral lines provides information on the movement of galaxies. Since most of the galaxies are showing shift towards red it is commonly pointed out that the universe is expanding.

What we see here is an explanation of red shift and blue shift, and a means of interpreting the spectral lines corresponding to elements present in a galaxy that states that the spectral lines corresponding to elements present in a galaxy can be identified by comparing it with the actual spectral lines recorded for each element on Earth...
But this description does not take into account the factor of gravitational influences, such as the 'gravitational' shift in light, or the fact that an atomic clock's tick rate is based on a resonant microwave frequency that will cause the atom to jump between energy levels, where the mathematics of GR time dilation state (in no uncertain terms) that clock's will be ticking faster at higher GP's, and the fact of the matter is that the physicality of a faster tick rate incorporates that an atomic clock be operating at a higher resonant frequency in the higher GP as compared to in the lower GP, which has been proven by experiment.
A higher resonant frequency means that the energy levels that the atoms are jumping between must also be occurring at a higher energy level, (otherwise the 'maximum number' of atoms being resonated will be reduced, and it isn't), which means that the microwave beam source is also 'jumped' to a higher energy level and is emitting a higher frequency microwave beam.

I will let you digest that and state your thoughts before putting the above into the context of my 'contracting' cyclic model.
Title: Re: Is there a discrepancy with the equivalence principle?
Post by: alancalverd on 13/05/2017 23:56:32
So far, all you have done is present a rather garbled version of what we already know frm GR and experiment. Nothing in what you say has any bearing on a cyclic universe, however sensible that model may be.

The point of using a diffraction grating is to show that red shift is measurable without measuring temporal frequency, so whatever mechanism you propose for the desynchronisation of clocks must also apply to the change in wavelength of a photon, which suggests that shifts in atomic energy levels is not the root cause.
Title: Re: Is there a discrepancy with the equivalence principle?
Post by: jeffreyH on 14/05/2017 02:00:55
If f = frequency and w = wavelength then c = fw. The speed of light does not change. The frequency, wavelength ratio varies. This could be interpreted as our time dilation being the cause of a perceived change in wavelength. Where the properties of the photon are always constant. This contradicts the effects on the speed of light in a strong gravitational field. Especially at the horizon of a black hole.
Title: Re: Is there a discrepancy with the equivalence principle?
Post by: timey on 14/05/2017 02:39:58
So far, all you have done is present a rather garbled version of what we already know frm GR and experiment. Nothing in what you say has any bearing on a cyclic universe, however sensible that model may be.

The point of using a diffraction grating is to show that red shift is measurable without measuring temporal frequency, so whatever mechanism you propose for the desynchronisation of clocks must also apply to the change in wavelength of a photon, which suggests that shifts in atomic energy levels is not the root cause.

You seem to have completely forgotten that my model redefines the observation of red shift/blue shift as being entirely due to gravitational shift.  The wavelength changes in the gravitationally shifted light are then due to a 3rd aspect of the time dilation phenomenon where m=0, i.e. open space, where the fact that light's wavelength is longer in the higher potential (via the maths of GR, or the observer being in the higher potential), is time related, and that this constitutes a complete re-interpretation of Hubble's velocity related red shift/distance correlation as being a temporally related red shift/distance correlation, because the length of the wave is time related.
And where the clock which is 'increased' in frequency (via the maths of GR, or the observer being in the lower potential) and has a shorter wavelength in the higher GP, my model is suggesting that this is due to gravity potential energy being active in mass.  Where if one wants to locate a quantum electron, one can know it's energy, and therefore know the related frequency and wavelength, where Qe = h*f/wavelength, because the length of the wave is time related.  (I think anyway, but I need some help with that equation)
So in my model GR increases the rate of time for mass in the higher GP, the 3rd aspect of time dilation decreases the rate of time in the weaker open space gravity field, and SR decreases or increases the rate of time for mass due to accelerated or decelerated motion relative to the gravity field. 

The spectrum analysis of spectral lines corresponding to elements present in a galaxy that can be identified by comparing it with the actual spectral lines recorded for each element on Earth, resulting for the most part in red shift, are now not describing an expanding universe.  They are describing that the gravitational fields between galaxy clusters are becoming weaker as the galaxies of galaxy clusters are drawn closer together under the influence of gravity.  Where our blue shifted results are observed of our own Milkyway galaxy and the Andromeda galaxy in our own galaxy cluster being drawn closer together, thus indicating a gravity field that is increasing in strength.

Also you say that a cyclic model is a sensible model, where I will now remind you 'again' that my cyclic model differs from the conventional bounce cyclic model, in that my model states the universe as slowly contracting at a progressively accelerating rate from the point that my models rendition of Inflation period (also differing from Alan Guth's conventional model of inflation) ends. i.e. a universe that is currently contracting.
Title: Re: Is there a discrepancy with the equivalence principle?
Post by: timey on 14/05/2017 02:57:14
If f = frequency and w = wavelength then c = fw. The speed of light does not change. The frequency, wavelength ratio varies. This could be interpreted as our time dilation being the cause of a perceived change in wavelength. Where the properties of the photon are always constant. This contradicts the effects on the speed of light in a strong gravitational field. Especially at the horizon of a black hole.

GR states that time runs at differing rates at differing GP's, but the speed of light is held relative to a static length second.  You saw what happened when Mike held the speed of light relative to GR time dilation.  He couldn't use SR length contraction and obtain sensible results.
If you want to locate a photon Qp = h*c/wavelength.  Hold the speed of light relative to a 3rd aspect time dilation second, and you can say Qp = c/wavelength.  If changes in gravitationally shifted light's wavelength are temporal rather than spatial, then one doesn't need to use the SR length contraction/space dilation concept to describe curvature of space.  It is the path through time that is curved and the geometry itself remains Euclidean.
Title: Re: Is there a discrepancy with the equivalence principle?
Post by: guest4091 on 15/05/2017 17:16:23
timey;
Quote
Under the remit of this system Planck's h constant becomes a function of time dilation, where it is changes in energy level that are causing changes in the rate of time.

Plank's constant is independent of time. You can introduce a Timey factor if you wish.

Quote
I think you need to throw back in time to the ultraviolet catastrophe and before Planck invented the h constant to 'iron out' his data.  The problem with the ultra violet catastrophe was (in brief and simplified) that it takes more energy to achieve the transitions of the higher frequencies of light than it does to achieve the transitions of the lower frequencies.  So Planck worked out that if he chopped the input energy into equal packages and multiplied by the number of wave cycles per second that he could achieve the data curve he was seeking.

It wasn't quite that simple. The original formulas by Wien and Rayleigh were based on the premise of equal probability for all frequencies. Experiment showed they weren't. While energy was accumulating, lower frequency quanta were forming because the conditions were sufficient. (The quanta forming process does not know what the observer is expecting.)

Quote
As an alternative, (although Planck's system is more dexterous in it's applications inclusive of my model's mathematics interpreted as a time function), as frequency increases simply shorten the length of a second so that the wave cycles remain a constant number, and the energy additions will be rendered as a linear progression.

If a second contained a constant number of wave lengths, how could the frequency change? Increased frequency was the resolution of the black body spectrum and the photoelectric effect. E=hf. The value of h has been verified and accepted in all of quantum physics.
A new theory has to agree with established facts, and work with established values, whether constants or variables. This does not prohibit the introduction of new terms.
Title: Re: Is there a discrepancy with the equivalence principle?
Post by: timey on 15/05/2017 23:10:21
phyti...

If you put yourself in the local frame with a clock, that clock will have the same frequency and wavelength in any GP that you place yourself and the clock in...  Calculate from the reference frame of the observer in a differing GP and the frequency and wavelength is not the same.

Looking at the black body emissions, I am making a parallel with the source of the emissions and the clock.  The source of the black body emissions 'the free electrons' are being increased in energy and emitting higher frequency, shorter wavelength light emissions as energy is increased.

If you calculate from the reference frame of the free electrons, the frequency of the emissions will remain the same as energy is increased.  Calculate from the reference frame of the static length second that Planck used, i.e. Planck's reference frame, and the frequency and wavelength of the light emissions change as energy is increased.

By calculating from the reference frame of the free electrons I am suggesting that the frequency will remain the same because seconds are becoming shorter.  By holding the frequency constant and using shorter seconds as energy is increased, there will be no quantum leaps... The energy increases will be a linear progression and Planck's constant is not necessary, but is useful for mathematical purposes as a function of time.

And drawing parallels between the linear progression of energy increases of this method of analyzing the black body, and the frequency changes observed of the clock from the non-local frame, the frequency changes observed of the clock in the differing GP's are then caused by the increase or decrease of potential energy as compared to the potential energy of the observers frame.

Now quantum is unified with gravity... 
Title: Re: Is there a discrepancy with the equivalence principle?
Post by: guest4091 on 16/05/2017 16:53:35

The only linear relation is f max=kT, max freq is proportional to absolute temp.
refer to:
www.en.wikipedia.org/wiki/Black-body_radiation
Title: Re: Is there a discrepancy with the equivalence principle?
Post by: guest4091 on 16/05/2017 17:01:21
timey quotes: the op:

Quote
On the one hand what I observe is that there is a school of thought that states that a caesium atomic clock placed at a higher gravity potential only 'appears' to have a higher frequency from the perspective of the lower gravity potential...
And that if one places oneself at the higher gravity potential with the clock, then the frequency of the clock will be the same as it was in the lower gravity potential, and that it will now 'appear' to you that the lower gravity potential clock has a lower frequency.
This is a direct consequence of the equivalence principle, and the concept that a caesium atom will be equivalent in each reference frame.
...and it would seem that the same school of thought is prevalent regarding SR time dilation...
This being that one's atomic clock aboard a rocket in relative motion will also be observed by oneself to be ticking 'normally', and it is the stationary rocket who's clock is observed to be running slow.
But if one were to place oneself on the stationary rocket, the stationary rocket's atomic clock would be ticking normally, and you would observe the rocket in relative motion's clock as ticking slow.
Then on the other hand - there is the school of thought that a person will age in keeping with their time dilated clock as described in the NIST link above, and the link below.
http://science.howstuffworks.com/humans-age-in-space.htm
Physically it just isn't possible for both concepts to reside in 'sensible' physics.
Of course there is a way of turning the equivalence principle on its head to describe why a person ages in keeping with their clock, that also retains the speed of light in each reference frame, but it requires that one take the school of thought that what one observes of the other reference frame is not just an appearance and that the other clock really is running at a differing rate.

The motion induced phenomena, time dilation and length contraction applies to all processes involving em energy transfers. This would include biological, which are chemical, the physics of atoms. If your sense of time rate changes with your local clock, atomic, mechanical, etc., then you aren't aware of the effect. If your ruler shrinks simultaneously with your space can, you still measure the same length inside.
This is the first postulate of SR that the description of physics is the same for any inertial frame. The first postulate is actually a  consequence of the second, the speed of light is constant and independent of its source. The td and lc effectively produce a scaled down absolute rest frame from any inertial frame. This eliminates having to reference the arf or the center of the universe, etc., before forming equations of physics. The contradictory behavior you and others think exists as mentioned above vanishes when considering the altered perception. Einstein emphasized the role of the observer, but not far enough to label relativity as a theory of perception , which is an essential part.
The examples mentioned above are cases where the observer participates in the same effects that act on the local clock, i.e. you adapt to that reference frame, and with altered perception you are able to observe the differences in the remote clocks that you left.
In the SR train scenario, the passenger perceives a straight vertical trajectory for a falling object in the train vs a curved one for the viewer standing outside the train. The trajectories are perceptions and observer dependent.
In the GR elevator scenario, while the box is accelerating upward, a light signal passes through holes in the box. The outside viewer Louie sees a horizontal path for the light, while the viewer inside the box Vinnie sees a curved path. The equivalence principle states that Vinnie can't determine if the box is accelerating upward or resting at a fixed position in a gravitational field. Therefore Vinnie should see a light signal follow a curved path in a strong g-field. (verified in 1919). In relativity, perception depends on the state of motion for the observer.
I don't see any discrepancy.
Title: Re: Is there a discrepancy with the equivalence principle?
Post by: timey on 17/05/2017 00:08:12
But I'm not talking about a discrepancy with the results of calculation other than the fact that conventional physics cannot explain the mechanics of how gravity does what it does, the mechanics of Big Bang and Inflation, and relies on Dark Matter and Dark Energy to describe the universe,.

The discrepancy I am suggesting exists is concerning the fact that time runs faster out in "space', where I am pointing out that the experimental evidence is only proving that time runs faster for mass out in space. i.e. m in M's gravity field.

If one takes the view that I have described in previous posts, this opens the door to a differing means of calculating and this means will unify quantum with gravity, explain the how of gravity and provide description of a fully described cyclic universe inclusive of Big Bangs and Inflation periods without introducing any unobserved phenomenon other than the 3rd aspect of time dilation that my model adds which can be indirectly observed as being the physical cause of the acceleration/deceleration that is observed of m in the open space of M's gravitational field.

The physics books I read are quite clear that a theory of everything will be a 'better' description of our universe than the one that currently exists, and that a 'better' description is 'required' to provide description of the mechanics of gravity, the mechanics of Big Bang, the mechanics of Inflation, and unify quantum with gravity, which is what my model does.

GR mathematics will describe a contracting universe, and a contracting universe such as my model describes does not need Dark Energy or Dark Matter to make the theory viable.

The only linear relation is f max=kT, max freq is proportional to absolute temp.

Not if one calculates that the free electron is mass and treats the mass of the free electrons of the black body as if behaving in the same way as the bound electrons of the cesium atom that comprise the mechanism of an atomic clock, where frequency changes in the light emissions of the black body and the changes of the resonant frequency of the microwave beam that is exciting the cesium atoms of the clock are considered as both being caused by changes in energy.

With regards to SR, if space is temporally dilated rather than spatially dilated then non-euclidean geometry is a function of time dilation, and length contraction is due to the time one measures the objects motion with.  However, calculating alternatively will result in the same values where the observer of a length/mass is calculating the acceleration of gravity and the dilation of space as being 2 issues that affect distance traveled, and my model states accelerated/decelerated motion caused by gravity and the dilation of space as being one and the same thing.

BTW, I may not 'speak' like a physicist, this being because I do not have a formal education as a physicist, but I have studied physics for getting on for 10 years now, and I am more than familiar with the link you provide, as well as having read 'Quantum', not to mention 5 books entirely dedicated to SR and GR, plus dozens of books that cover all 3 subjects as well as a whole host of related considerations inclusive of what is required to produce a theory of everything.
Title: Re: Is there a discrepancy with the equivalence principle?
Post by: alancalverd on 17/05/2017 08:15:42

The discrepancy I am suggesting exists is concerning the fact that time runs faster out in "space', where I am pointing out that the experimental evidence is only proving that time runs faster for mass out in space. i.e. m in M's gravity field.


Except that it is also experimentally true when m = 0. Gravitational lensing happens to photons, and is entirely consistent with GR.
Title: Re: Is there a discrepancy with the equivalence principle?
Post by: timey on 17/05/2017 15:13:50
Gravitational lensing is caused by light being bent towards mass.

Light under the remit of SR is being calculated via a time dilation factor.  Time is said to stop for anything travelling at light speed.  Or, it is said that light does not experience time.  Whichever way one chooses to approach the subject, it is not light's own experience of time that is causing it to be bent towards mass.

GR states that time 'in space' gets slower near mass, yet we observe that there is a phenomenon called gravitational acceleration that accelerates all bodies m near M.  If time gets slower nearer to M then why is m's motion accelerated as it gets closer to M?  Whichever way one chooses to approach the subject, it is not a GR time dilation of space that causes accelerated motion...

SR is used to describe light travelling in space, but time has no meaning for the light itself, so what is SR describing?  If time has no meaning for light itself then SR is just describing a dilation of space in which light is being what?  Is it describing that the 'length' of light is being contracted while describing that the 'length' of space is being dilated?

My model states that the light is bent towards the mass because gravitational acceleration is caused by a 3rd aspect of the time dilation phenomenon that affects 'open space' where m=0, and that this 3rd aspect time dilation where m=0 runs at a faster rate nearer to mass.  This gives physical cause to both gravitational acceleration near mass and the dilation of space where it is not the 'length' of space that is dilated, it is the 'time' in space that is being dilated.

This is an alternative description of spacetime that will be compatible with the mathematics of GR and results in my cyclic model of a contracting universe.
Title: Re: Is there a discrepancy with the equivalence principle?
Post by: guest4091 on 18/05/2017 17:23:12
GR has been successfully verified regarding its predictions via many and varied experiments. I think "discrepancy" is a poor choice of words.
The theory is "limited" as to how much detail it provides regarding behavior of physical entities, eg. how does the g-field work, ...but aren't all theories. Newton never explained how a dominant mass could affect the motion of a smaller distant mass. Knowledge is a continuing process of refinement.

Quote
Light under the remit of SR is being calculated via a time dilation factor.  Time is said to stop for anything travelling at light speed.  Or, it is said that light does not experience time.  Whichever way one chooses to approach the subject, it is not light's own experience of time that is causing it to be bent towards mass.

Time dilation results from motion. Time is just a process that is programmed into a device called a clock.

Quote
GR states that time 'in space' gets slower near mass, yet we observe that there is a phenomenon called gravitational acceleration that accelerates all bodies m near M.  If time gets slower nearer to M then why is m's motion accelerated as it gets closer to M?  Whichever way one chooses to approach the subject, it is not a GR time dilation of space that causes accelerated motion...

Space is not time dilated, processes are, which includes clocks. If you also accelerate in SR to a different speed, a clock runs slower.

Quote
SR is used to describe light travelling in space, but time has no meaning for the light itself, so what is SR describing?  If time has no meaning for light itself then SR is just describing a dilation of space in which light is being what?  Is it describing that the 'length' of light is being contracted while describing that the 'length' of space is being dilated?

SR describes the effects of motion on observers of physical events and provides coordinate transformations between observers based on their relative velocities. Light is a measuring tool.
Title: Re: Is there a discrepancy with the equivalence principle?
Post by: timey on 18/05/2017 20:40:35
phyti - it's pretty difficult to decide on a thread title that is a question.  At the time I opened this thread the thread title related to a part of what I was talking abut with Mike.
What this thread should be called now is "Hey hello there, I've got a theory of everything that unifies relativity and quantum, thus fully describing a cyclic universe that has been contracting since the moment that inflation ended, and I want some help with the maths"...

I am posting at this forum purely on the basis of Lee Smolin's book "The Trouble With Physics" where chapter 1 is named "The Five Great Problems in Theoretical Physics".

Quote
: Lee Smolin
Problem 1: "Problem of quantum gravity" - Combine general relativity and quantum theory into a single theory that can claim to be the complete theory of nature.

Problem 2: "The foundation problems of quantum mechanics" - Resolve the problems in the foundations of quantum mechanics, either by making sense of the theory as it stands or by inventing a new theory that does make sense.

Problem 3: "The unification of the particles and forces" - Determine whether or not various particles and forces can be unified in a theory that explains them all as manifestations of a single fundamental entity.

Problem 4: "The constants of the standard model" - Explain how the values of the free constants in the standard model of particle physics are chosen by nature.

Problem 5: "Cosmological mysteries" - Explain dark matter and dark energy.  Or, if they don't exist, determine how and why gravity is modified on large scales.  More generally, explain why the constants of the standard model of cosmology, including dark energy, have the values that they do.

My model answers, or potentially answers, all 5 of these questions.

Problem 1 - general relativity and quantum are combined by a re-interpretation of the time dilation phenomenon and the introduction of an additional 3rd aspect of time dilation where m=0.
Problem 2 - is solved by remit of the solving of problem 1, where a combination of making sense of the theory and inventing a new theory is employed.
Problem 3 - should be solved by remit of solving problem 1, but this is where a mathematician needs to be employed.
Problem 4 - will be solved if problem 3 can be mathematically resolved by remit of solving problem 1
Problem 5 - is solved by remit of solving problem 1, where dark energy and dark matter do not exist but the values they have will be pertinent to the remit of how problem 1 has been resolved.

SR describes the effects of motion on observers of physical events and provides coordinate transformations between observers based on their relative velocities. Light is a measuring tool.

So what of SR describing the effects of motion on the physical event of light travelling across space, gravitational lensing of light, and the coordinate transformations between observer of light and light itself?
Title: Re: Is there a discrepancy with the equivalence principle?
Post by: timey on 19/05/2017 14:31:52
Quote
The difference between the theories...

Special relativity was first (1905) and deals with how motion, the perception of time and velocity are relative not absolute and dependent on the relative velocity of the observers. This includes E=mc^2, the way time is experienced differently by different observers moving at different fractions of the speed of light, the way that velocities add and thus how no particle with mass can ever go (or exceed) the speed of light, etc.

General relativity (1915) is a theory of gravity which replaces Newton's universal law of gravity (and reduces to it for large distances) and is a mathematical framework that describes how space-time is curved and bent by the presence of mass and how this structure effects the motion of particles. It is called general relativity because its solution in flat space (i.e. ones with no masses around) reduces to the equations of special relativity, thus special relativity is a "special" case of general relativity.

Thus, if people are talking about: Atomic clocks on space-ships not experiencing the same time, the twin-paradox, the inability to exceed the speed of light, the contraction of an object as it approaches the speed of light, etc. They're talking about special relativity.

If people are talking about: Space-time curvature due to a star or a planet, the bending of light around a star, planet or galaxy (gravitational lensing), the expansion of the universe, the big bang, etc. They're talking about general relativity.

Reference https://www.physicsforums.com/threads/difference-between-sr-and-gr.568598/
I'll add - that if people are talking about Atomic clocks at differing GP's not experiencing the same time, then they are talking about general relativity, but when people are talking about Atomic clocks at differing GP's at differing longitudes on Earth moving at differing centripetal motions, they are talking about a combination of general relativity and special relativity.

I thought I would post the above to negate replies that seek to repeat the description.

*

My model seeks to take the matter a stage further in that where a combination of general relativity and special relativity is used to describe clocks at differing GP's at differing longitudes on Earth moving at differing centripetal motions, my model extends this combination of general relativity and special relativity to the remit of describing clocks on space ships not experiencing the same time, the twin paradox, and the fact that no spaceship can travel at the speed of light, and indeed also give explanation as to why trying to achieve such speeds would be contrary to arriving at one's destination in a 'shorter' amount of time.

Due to the addition of this 3rd aspect of the time dilation phenomenon where m=0, my models rendition of SR is altered from the conventional description, but there is no way anyone is going to understand the altered rendition until one understands the alteration that my model is making to GR, so I'm concentrating on GR at the moment...

Where the first thing to understand is that my model is using the GR mathematics to describe a universe that is currently contracting.  GR mathematics 'minus Einstein's cosmological constant' will describe a universe that is currently contracting.
(This incorporates a re-interpretation of Hubble's velocity related red shift/distance correlation which is realized by the addition of the 3rd aspect of the time dilation phenomenon where m=0)

The second thing to understand is that my model uses GR 'only' to describe the passage of light in the gravitational field.
(Again - this incorporates a re-interpretation of Hubble's velocity related red shift/distance correlation which is realized by the addition of the 3rd aspect of the time dilation phenomenon where m=0)

The third thing to understand is that by adding the 3rd aspect of time dilation where m=0 my model has given a physical cause to the phenomenon of gravitational acceleration/deceleration of the motions of m in the gravitational field* that gives explanation of why all value m accelerates at the same rate in free fall.
(*The mathematics for MOND could be significant in that they modify Newton's laws to describe galaxies without dark matter)

The fourth thing to understand is that by adding the 3rd aspect of time dilation where m=0, my model has redifined the understanding of clocks not experiencing the same time at differing GP's and equates this with electrons (free or bound) not experiencing the the same energy levels, where my model takes this thought to the black body and states an increase in frequency of light emissions as being due to an electron experiencing a 'shorter' second.
(This incorporates a re-interpretation of Planck's h constant as a function of time dilation where all the same maths can be used as is, but dividing by wavelength will be a division by the GR time dilation factor and should be able (need help with this) to be employed to locate an electron, where the necessity for probability maths will be negated.  And when applied to light in the gravitational field, changes in gravitationally shifted light's wavelength at differing GP's are then indicative of changes in the rate of time due to the 3rd aspect time dilation factor of the m=0 gravity field of space., where the strength of the gravity field at that GP dictates the rate of time.  When applied to electromagnetic considerations, this should unify gravity and quantum)

The fifth thing to understand is that by adding the 3rd aspect of time dilation where m=0, this has redefined the premise of Hubble's velocity related red shift/distance correlation as a temporal red shift/distance correlation where it is now possible to consider a contracting universe that can be described by GR mathematics minus Einstein's cosmological constant, and think upon how a contracting universe will proceed.

If these 5 things can be understood then I could move on to describing the alterations that my model then makes to SR and why my model makes them...
I can also expand upon any of these 5 considerations in more detail should anyone have any questions.
Title: Re: Is there a discrepancy with the equivalence principle?
Post by: timey on 20/05/2017 14:54:45
https://en.wikipedia.org/wiki/Tests_of_general_relativity

There have been a lot of tests of GR and they show that GR is correct to acceptable percentage. (which is good news for my model because my model uses GR mathematics)

However, I tell you that it would make no significant difference to the results of these tests of GR if we considered that time ran slow out in space for space itself. (not for mass in space)
You will respond by saying how can this be?  Surely if time ran slower out in space then the tests we make of GR, where GR is saying that time runs faster out in space would show that GR is not correct.
I tell you that this 'may' not be the case...

https://en.wikipedia.org/wiki/Curved_space

All my model is doing is suggesting that these deviations from Euclidean geometry are not spatial, and are in fact temporal, where the spatial dimensions of the geometry of space remain Euclidean.

So my question is - when the tests of GR set out in the 'tests of general relativity' link provided above are calculated, are the deviations from Euclidean geometry being calculated as spatial additions to the geometric distance between objects? 
Title: Re: Is there a discrepancy with the equivalence principle?
Post by: timey on 20/05/2017 18:36:45
Because I'm pretty certain that the answer to my question above is - yes they are.

Now we are venturing into the realm of the two body problem, and I will again refer back to my earlier comment concerning MOND.

If we are considering that deviations from Euclidean geometry are temporally derived, then the mathematics of MOND, modified Newtonian dynamics for a modification of Kepler's law, 'should' be significant.
Title: Re: Is there a discrepancy with the equivalence principle?
Post by: timey on 21/05/2017 16:12:17
https://en.wikipedia.org/wiki/Modified_Newtonian_dynamics

Quote
:wiki
Milgrom's law can be interpreted in two different ways. One possibility is to treat it as a modification to the classical law of inertia (Newton's second law), so that the force on an object is not proportional to the particle's acceleration a but rather to μ(a/a0)a. In this case, the modified dynamics would apply not only to gravitational phenomena, but also those generated by other forces, for example electromagnetism.[10] Alternatively, Milgrom's law can be viewed as leaving Newton's Second Law intact and instead modifying the inverse-square law of gravity...

And I am putting forward the idea that the structure of a non-euclidean geometry of space can be re-interpreted, where deviations from euclidean geometry are temporal.  This approaches the matter in reverse where it is distances that 'may' not be 'as interpreted'...
However, because the maths of MOND can describe motions without addition of dark matter the proportions that these maths are using 'should' be significant to both my proposal of temporally derived deviations from a euclidean geometry of space, and my proposal of potential energy being active 'in' mass.
Title: Re: Is there a discrepancy with the equivalence principle?
Post by: guest4091 on 22/05/2017 17:17:33
timey;
Are you debating with yourself?, and where did everybody go!

Whenever someone proposes a TOE, I respond with the same question.
How can we know if we have seen everything, or discovered everything?
Science publications and history are indicating science isn't there yet. In the constant refinement of knowledge, the historical record shows theories revised or replaced with 'new and improved' ones. The common response to new facts, "it's more complicated than we originally thought". This is not to discourage the inquisitive mind from theorizing or experimenting. A TOE is a gargantuan task, and in light of the uncertainties regarding the phenomena, you and many others, question, ...why not open a bakery
.
I can't offer any help in your quest. GR is more complex than SR, and more work than fun.
The most significant issue I have with your model is time.
Time is not a causative factor. Flipping a coin is a sequence of physical events, but the outcome is independent of time. It's a correlation/measuring tool.
It's easy to visualize time dilation in SR as resulting from motion, but in GR, without clock motion, one idea is the stress of gravitational acceleration (its own weight) on the internal clock components for a clock at rest in a g-field. Eg, in using a portable grinding tool on a surface, applying pressure to the surface will slow the tools rotation speed.
Observing other clocks slowing means they are self adjusting to motion, which preserves the rules of physics, a good thing. ('rules' are human interpretations of the 'laws'.

problem 3. 'Ideal' notions of the physical world, because the mind wants simple models that it can comprehend. Where are those perfect circular orbits, and perfect symmetries, when you want them. The negative side to generalization is, the elements lose their identity. Time is drawn as a line on paper with other lines representing dimensions. This provides the opportunity to interpret as moving in time, since there is nothing to distinguish one from the other. It's the basis for Briane Greens metaphorical moving in time, a common misinterpretation of space-time graphs as road maps!

Then there was the guy who worked for NIST who asked a female coworker to meet him for lunch at 12:00 at Freddies. Upon arriving, he didn't see her anywhere and wondered if she had changed her mind Then he remembered, she works on the floor below him.  Just then she walks in at 857 ms past 12, (he wears an atomic watch).
 
Title: Re: Is there a discrepancy with the equivalence principle?
Post by: alancalverd on 22/05/2017 17:35:32
That's one heck of a ceiling height!

When I worked in the national standards lab, the blokes in Time Section all wore Mickey Mouse watches.

And here's a thing, observed during a trip to Geneva. Competitive advertisements for mens wristwatches all had a theme of "speed": F1 cars, fighter jets, sprinters, skiers....but surely the whole point of a watch is that it goes at exactly the same speed as everyone else's?  And whilst I have great admiration for Kevin Pietersen's batting, only a fool would wear a wristwatch to open against the West Indies. Is it me, or the rest of the world? 
Title: Re: Is there a discrepancy with the equivalence principle?
Post by: timey on 22/05/2017 18:48:12
Ah yes - funny, funny...

The most significant issue I have with your model is time.
Time is not a causative factor. Flipping a coin is a sequence of physical events, but the outcome is independent of time. It's a correlation/measuring tool.

Time is not a causative factor...?

If you take a body of mass travelling at a constant speed through positions in space that are running at differing rates of time, the constant speed of the mass will be affected by the rate of time of the space it is travelling through.  As the rate of time in the space increases, shorter seconds, the constant speed of the mass travelling through the space will be accelerated.  As the rate of time in the space decreases, longer seconds, the constant speed of the mass travelling through the space will be decelerated.  This looks to me as though time, or more precisely differing rates of time, can be causative of motion.

Note that I am making a distinction between what time is doing in the space the mass is travelling through, and what time is doing in the mass that is being traveled through space.  I can expand, but lets avoid overload in this post...

In any case phyti - when I very first met you here on line at the forum talking to Box on his thread, I asked what you would say if I told you I thought I had a theory of everything, and you said that that you would say "show me the maths".  The maths are GR, GR describing a contracting universe via a re-interpretation of Hubble's interpretation of the red shift/distance correlation.  What can I say?
Title: Re: Is there a discrepancy with the equivalence principle?
Post by: guest4091 on 23/05/2017 20:29:03
Quote
In any case phyti - when I very first met you here on line at the forum talking to Box on his thread, I asked what you would say if I told you I thought I had a theory of everything, and you said that that you would say "show me the maths".

That is not something I would say, but have seen it used frequently. (math without an s, as an abbreviation for mathematics.)
Will study the rest of this.
Title: Re: Is there a discrepancy with the equivalence principle?
Post by: timey on 24/05/2017 15:04:09
"The Trouble with Physics" chapter 3 : The World as Geometry - pages:40, 41, 42,43
Quote
:Lee Smolin
We are not accustomed to thinking of space as an entity with properties of it's own, but it certainly is. Space has three dimensions and it also has a particular geometry, which we learn in school.  Called Euclidean geometry - after Euclid, who worked out its postulates and axioms more than 2000 years ago - it is the study of the properties of space itself.  The theorems of Euclidean geometry tell us what happens to triangles, circles, and lines drawn in space.  But they hold for all objects real or imagined.

A consequence of Maxwell's theory of electromagnetism is that light rays move in straight lines.  Thus it makes sense to use light rays when tracing the geometry of space.  But if we adopt this idea, we see immediately that Einstein's theory has great implications.  For light rays are bent by gravitational fields, which, in turn, respond to the presence of matter.  The only conclusion to draw is that the presence of matter affects the geometry of space.
In Euclidean geometry, if two straight lines are initially parallel, the can never meet.  But two light rays that are initially parallel can meet in the real world, because if they pass on each side of a star, they will be bent towards each other.  So Euclidean geometry is not true in the real world.  Moreover, the geometry is constantly changing, because matter is constantly moving.  The geometry of space is not like a flat infinite plane.  It is like the surface of the ocean - incredibly dynamic, with great waves and small ripples in it.

Thus, the geometry of space was revealed to be just another field.  Indeed, the geometry of space is almost the same as the gravitational field.  To explain why, we have to recall the partial unification of space and time that Einstein achieved in special relativity.  In this unification, space and time together make up a four-dimensional entity called spacetime.  This has a geometry analogous to Euclidean geometry, in the following precise way.
Consider a straight line in space.  Two particles can travel along it, but one travels at uniform speed, while the other is constantly accelerating.  As far as space is concerned, the two particles travel on the same path.  But they travel on diferent paths in spacetime.  The particle with a constant speed travels on a straight line, not only in space but in space time.  The accelerating particle travels on a curved path in spacetime.

Hence, just as the geometry of space can distinguish a straight line from a curved path, the geometry of spacetime can distinguish a particle moving at a constant speed from one that is accelerating.
But Einstein's equivalence principle tells us that the effects of gravity cannot be distinguished, over small distances, from the effects of acceleration.  Hence, by telling which trajectories are accelerated and which are not, the geometry of spacetime is therefore the gravitational field.

Thus the double unification given by the equivalence principle becomes a triple unification: All motions are equivalent once the effects of gravity are taken into account, gravity is indistinguishable from acceleration, and the gravitational field is unified with the geometry of space and time.  When worked out in detail, this became Einstein's general theory of relativity, which he published in full form in 1915.

Unfortunately while Einstein's theory tells us what the effects are, and is highly predictive to great precision, the theory does absolutely nothing to explain why these effects occur, i.e. does not describe the physical mechanics that are causing these effects.

Quote
:Lee Smolin
the geometry of space can distinguish a straight line from a curved path, the geometry of spacetime can distinguish a particle moving at a constant speed from one that is accelerating
Quote
:Lee Smolin
A consequence of Maxwell's theory of electromagnetism is that light rays move in straight lines
Quote
:Lee Smolin
But two light rays that are initially parallel can meet in the real world, because if they pass on each side of a star, they will be bent towards each other

Therefore light rays that are being bent towards each other are doing so under the effect of gravity, but what effect is this?

Quote
:Lee Smolin
Einstein's equivalence principle tells us that the effects of gravity cannot be distinguished, over small distances, from the effects of acceleration.  Hence, by telling which trajectories are accelerated and which are not, the geometry of spacetime is therefore the gravitational field.

The light rays are being bent by the geometry of spacetime, the geometry of spacetime being the gravitational field.  But what is the geometry of space time?
Lets go back...
Quote
:Lee Smolin
the geometry of space can distinguish a straight line from a curved path, the geometry of spacetime can distinguish a particle moving at a constant speed from one that is accelerating
But light rays are moving at a constant speed, aren't they?  So how is the geometry of spacetime distinguishing the constant speed of light?
Quote
:Lee Smolin
Einstein's equivalence principle tells us that the effects of gravity cannot be distinguished, over small distances, from the effects of acceleration.
So if the effects of gravity cannot be distinguished from the effects of acceleration, then light rays being bent by the geometry of spacetime, where the geometry of spacetime is the gravity field, are indistinguishable from particles being accelerated by the gravitational field.

Now we are looking at motion and acceleration.  And we are also looking, by default, at motion and deceleration.

Quote
:Lee Smolin
Einstein succeeded in unifying all kinds of motion.  Uniform motion is indistinguishable from rest.  And acceleration is no different to being at rest but with a gravitational field turned on.
...where I'll add that deceleration is no different to being at rest but with a gravitational field turned on.
Quote
:Lee Smolin
Notice here that, as in the successful unification's discussed earlier, more than one unification is happening at once.  Two different kinds of motion are being unified; there is no longer a need to distinguish uniform from accelerated motion.  And the effects of acceleration are being unified with the effects of gravity.
But while there is a unification between different types of motion, and the effects of acceleration are unified with gravity, there is no unification between motion and acceleration/deceleration, where again we go back...
Quote
:Lee Smolin
the geometry of space can distinguish a straight line from a curved path, the geometry of spacetime can distinguish a particle moving at a constant speed from one that is accelerating
Where the geometry of spacetime is the gravitational field...
But what 'is' the gravitational field? What in the gravitational field is causing acceleration/deceleration?  What causes uniform motion?

This is where my model adds, in addition to SR relative motion time dilation and GR gravity potential time dilation, a 3rd aspect of the time dilation phenomenon for where m=0, i.e. the gravitational field.  This 3rd aspect of time dilation is affected by the strength of the gravitational field, in that where the field is stronger there are shorter seconds, and where the field is weaker there are longer seconds.  This idea unifies the observation of motion with acceleration/deceleration and renders any spacetime deviations from Euclidean geometry as a function of the added 3rd aspect time dilation.

The light rays being bent towards a star are being bent because time in the 'space' near the star is running faster than the time in the 'space' between the background object emitting the light rays and the star that these light rays are being bent towards.

Under this remit the acceleration/deceleration of gravity is an increase or decrease in the local time of 'space' giving physical cause to changes in the motion of particles in the gravitational field.
This 'time related' geometry is compatible with electromagnetic and the reciprocal of this introduction of a 3rd aspect of the time dilation phenomenon gives cause for the directional changes that occur in a gravitational field.  This is the part that unifies quantum with gravity which I have discussed previously, but can expand on again if necessary...   
Title: Re: Is there a discrepancy with the equivalence principle?
Post by: alancalverd on 25/05/2017 14:52:06
If "light rays travel in straight lines" then a single photon cannot interfere with itself in the double-slit experiment. 
Title: Re: Is there a discrepancy with the equivalence principle?
Post by: timey on 25/05/2017 15:27:47
But Alan, I did not say that light is not bent from a straight line...

"The Trouble with Physics" chapter 3: Unification becomes a Science - pages 54 & 55
Quote
:Lee Smolin
After the idea of unifying all four fundamental forces failed, most theoretical physicists gave up on the idea of relating gravity to the other forces, a decision that made sense because gravity is so much weaker than the other three.  Their attention was drawn instead to the zoo of elementary particles that the experimentalists were discovering in their particle accelerators.  They searched the data for new principles that could at least unify all the different kinds of particles.

Ignoring gravity meant taking a step backward, to the understanding of space and time before Einstein's general theory of relativity.  This was a dangerous thing to do in the long run, as it meant working with ideas that had already been superseded.  But there was also an advantage, in that this approach led to a great simplification of the problem.  The chief lesson of general relativity was that there is no fixed background for space and time; ignoring this meant that you could simply choose the background.  This sent us back to a Newtonian point of view, in which particles and fields inhabit a fixed background of space and time - a background whose properties are fixed eternally.  Thus, the theories that developed from ignoring gravity are background-dependent.

However, it was not necessary to go all the way back to Newton.  One could work within the description of space and time given by Einstein's 1905 special theory of relativity.  According to it, the geometry of space is that given by Euclid, which many of us study in junior high school; however, space is mixed with time, in order to accommodate Einstein's two postulates, the relativity of observers and the constancy of the speed of light.  The theory cannot accommodate gravity, but it's the right setting for Maxwell's theory of the electric and magnetic fields.

Once quantum mechanics was fully formulated, the quantum theorists turned their attention to unifying electromagnetism with quantum theory.  As the basic phenomenon of electromagnetism are fields, the unification that would eventually result is called quantum field theory.  And because Einstein's special theory of relativity is the right setting for electromagnetism, these theories can also be seen as unification of quantum theory with special relativity.

Smolin then goes on to describe QED and gauge principle, where quantum is unified with electromagnetism, and the weak and the strong forces, but not with gravity.

But in light of what Smolin says here:
Quote
:Lee Smolin
because Einstein's special theory of relativity is the right setting for electromagnetism, these theories can also be seen as unification of quantum theory with special relativity.
This does rather imply that Einstein's general theory of relativity and Einstein's special theory of relativity are NOT unified!

If we examine this more closely, what we can see is that the general theory of relativity is a background independent theory, and the theory of special relativity is a background dependent theory...
Why is special relativity a background dependent theory?
...well I see this as being due to the fact that special relativity is holding an objects speed relative to the same time period as the speed of light is being held relative to, thus by default imposing a preferred frame of reference...

Let's examine this a bit more:
Quote
:Lee Smolin
However, it was not necessary to go all the way back to Newton.  One could work within the description of space and time given by Einstein's 1905 special theory of relativity.  According to it, the geometry of space is that given by Euclid.
OK - so special relativity is giving a geometry of space that is Euclidean, and general relativity is giving a geometry of space that is non-Euclidean.

General relativity derives non-Euclidean geometry from changes in motion...
Special relativity derives Euclidean geometry from changes in length/distance...

If one applies the distance/speed/time formula, one can calculate the general relativity deviations from Euclidean geometry as being deviations of the rate of time in space that then are the cause of the changes in motion, where these changes in motion do not incorporate differences in distance traveled because it takes a longer or shorter amount of time to travel the same distance, i.e. resulting in euclidean geometry.

If one applies the distance/speed/time formula, one can calculate the special relativity deviations from non-Euclidean geometry as being deviations of the rate of time in space that have been calculated as deviations from the length of 'the measuring stick', where if 'the measuring stick' were not to deviate in length, this would result in both a non-Euclidean geometry and deviations in rate of motion.
(to calculate SR under the remit of my altered GR one would need to incorporate the changes in the 3rd aspect time dilation, where GR and SR can be calculated as a combination by knowing the value of the field.  I can expand on this)

Can anyone see what has happened here?  One is now the upside down of the other, but what have I done?  Is general relativity now a background dependent theory where it is the mass, or the masses of the objects causing 'the field' that causes the 'time related' deviations from Euclidean geometry?
(I now refer you back to my earlier comments about MOND and the modification of Newtonian dynamics...)

By adding a 3rd aspect of the time dilation phenomenon to open space where m=0, this has unified changes in motion in the gravity field with acceleration/deceleration of gravity, and unified the concept of non-Euclidean dimensions in space with Euclidean geometry, where the curves, dynamic waves, and ripples in the gravity field are 3rd aspect time dilation related...
This takes us to the concept of vacuum energy, the concept of potential energy, and my intention to unify the phenomenon of time itself with energy for a unification of quantum with gravity.
Title: Re: Is there a discrepancy with the equivalence principle?
Post by: guest4091 on 25/05/2017 20:57:32
Timey;

Quote
If you take a body of mass travelling at a constant speed through positions in space that are running at differing rates of time, the constant speed of the mass will be affected by the rate of time of the space it is travelling through.  As the rate of time in the space increases, shorter seconds, the constant speed of the mass travelling through the space will be accelerated.  As the rate of time in the space decreases, longer seconds, the constant speed of the mass travelling through the space will be decelerated.  This looks to me as though time, or more precisely differing rates of time, can be causative of motion.

Note that I am making a distinction between what time is doing in the space the mass is travelling through, and what time is doing in the mass that is being traveled through space.  I can expand, but lets avoid overload in this post...

The 'rate of time in space' isn't any clearer than 'the gravitational energy in the space surrounding the mass that generates it'.

How would a clock in empty space change its rate if the rate in space is constant?
 
paraphrased from The Meaning of relativity, Albert Einstein, 5th edition 1956:
"In order to give physical significance to the concept of time, processes of some kind are required which enable relations to be established between different places. The type of process should be well understood. The propagation of light in space is the most qualified for this purpose"
Enter the light clock.
Guess I'm not the only one who interprets time this way.
Title: Re: Is there a discrepancy with the equivalence principle?
Post by: timey on 25/05/2017 22:24:09
How would a clock in empty space change its rate if the rate in space is constant?

It would seem that you have not understood what I am saying.
1 -  A clock put into the higher potential, empty space or otherwise, will tick faster than a clock in the lower potential as per GR.  Let's get it clear that I am NOT disputing this verified by experiment fact.
2 - What I am suggesting is that putting a clock into empty space will not be measuring what time is doing in empty space, but will be measuring what time is doing for m in relation to M at that potential.
3 - What I am suggesting is that the rate of time in space is NOT constant, that the rate of time in space will be running faster closer to mass M and slower further from mass M , but that you will NEVER be able to measure this time that is occurring in empty space with a clock because the clock is mass.
4 - What I am suggesting is that if you drop the clock that is in the higher potential open space, it will accelerate towards the lower potential, and that it is the time that is occurring in open space that causes this acceleration of motion because as the clock falls into the lower potential the rate of time in the open space of the lower potentials gets faster closer to M and this will cause a body in motion to experience accelerated motion.
5 - What I'm suggesting is that if you throw the clock from the lower potential into the higher potential, that the clock will be decelerated as it moves into the higher potential, and that it is the time that is occurring in open space that causes this deceleration of motion because as the clock rises into the higher potentials, the rate of time in the open space of the higher potentials gets slower further from M and this will cause a body in motion to experience decelerated motion.
6 - What I am suggesting is that the rate of time that is occurring for the clock is differing from the rate of time that is occurring in the open space the clock is moving through, and that this contributes to all values of m falling at same rate of acceleration towards M.
7 - What I am also suggesting is that the rate of time that the clock is experiencing (GR time dilation) will have a direct bearing on the number of magnetic moments occurring for the clock, or any particle of mass, where at higher potentials there will be more frequent magnetic moments, at lower potentials there will be less frequent magnetic moments, where this is also a contributing factor to all values of m falling at same rate of acceleration towards M, gives the gravity field directionality, and also describes WHY gravity is so weak but at the same time so far reaching.

I am quite sure that my interpretation satisfies all of Einstein's requirements...

Enter the light clock.

The cesium atomic clock is a light clock.  It is indeed a microwave beam that excites the atoms at a particular frequency, and it is this particular frequency that is observed to differ when observed from a non-local potential.
However I have been perusing the 'mechanical relativity' thread.  What type of light clock are you proposing, or more to the point, where in the gravity potential are the photon's going to be emitted from the emitting source?

Guess I'm not the only one who interprets time this way.
No you are not the only person.  Physics books agree with GR and SR interpretations, but also agree that there is no known physical cause for the acceleration of gravity, and that quantum and gravity have not been unified.  I am the one who is 'the only' person who is interpreting gravitational acceleration as 3rd aspect time dilation related.  This is part of my "New Theory", and it is my new theory that I aim to describe here...
Title: Re: Is there a discrepancy with the equivalence principle?
Post by: timey on 26/05/2017 14:02:29
How would a clock in empty space change its rate if the rate in space is constant?
This is indeed a pertinent question though...

The physics books do not give any description as to 'how' a clock changes it's rate (as per GR) in the differing potentials of space, they only describe that a clock's rate of time will change.

Clearly, from the nature of the question, you are assuming that the rate of time for a clock in the differing potentials of space is changing because the rate of time in the differing potentials of space changes.

The physics books do not give any description as to 'how' the rate of time (as per GR) in the differing potentials of space changes, they only describe that a clock's rate of time will change when it is placed in differing potentials.

What I have done is separate the notion of time in the differing potentials of space from the notion (and observation) of time for the clock in the differing potentials of space into two independent issues, where the clock "in' the gravity potential is ticking at a rate that is independent and differing from the rate of time in the space of that position of gravity potential, and stating that the notion of the rate of time in space can be observed as being responsible for the observed changes in the rate of motion mass experiences in the gravity field...
Now we have a situation where the phenomenon of time for mass (GR and SR) occurs as a separate issue from, but 'inside' the phenomenon of time for space.

Now to describe 'how' these rates of time that are independent of each other are being changed.

What I am saying is that +energy=shorter seconds, and that -energy=longer seconds...
GR time dilation can be described as gaining potential energy in the higher potential and losing potential energy in the lower potential.
SR time dilation can be described as losing, or gaining potential energy due to motion relative to the gravitational field.
3rd aspect time dilation of space, where m=0, can be described as vacuum energy related, where vacuum energy and gravitational field strength are one and the same thing and the stronger field can describe +energy, and the weaker field can be described as -energy.

This unifies the phenomenon of time with energy, and leads us from potential energy and vacuum energy to the doorstep of temperature energy and back to the black body to apply the +energy=shorter seconds remit to the data for a re-interpretation of the function of Planck's h constant...
... And the realization that under the remit that I propose, bigger masses will not be running at slower rates of time.  Bigger masses will be running at faster rates of time.

This opens the door onto my changes to the conventional concept of time running faster in space for clocks...

Where my idea is that as a body m moves further away from M, there will come a point where body m's own gravity field will outweigh body M's influence on the field at that distance, and body m will then have it's own field around it.  From the point that body m attains a field around itself that is stronger than body M's influence on the field at that distance, body m's rate of time will be decreasing instead of increasing as body M's field becomes weaker with body m's increased distance from M.
Just to be clear let's describe this in reverse...
Body m has it's own gravitational field.  As body m becomes captured by body M's gravitational field, it's rate of time will be increasing as it is influenced by body M's field.  It's rate of time will be increased up till the point of distance from M that M's gravitational field at that distance outweighs body m's own gravitational field.  When body M's gravitational field at that distance outweigh's body m's gravitational field, body m's rate of time will start to decrease as body m gets closer M.
My idea states that it is the 3rd aspect time dilation of space that is causing the increased motion of body m as it free falls towards M.
Now would be the time to add SR time dilation (but not length contraction because that mathematical necessity is already being described via the 3rd aspect time dilation) for orbital speeds and motion caused by free fall.
Title: Re: Is there a discrepancy with the equivalence principle?
Post by: guest4091 on 26/05/2017 18:34:14
timey;
Quote
General relativity derives non-Euclidean geometry from changes in motion...
Special relativity derives Euclidean geometry from changes in length/distance...

I have to disagree with the cause and effect order in SR.
SR environment is the 'principle of relativity' restricted to a Euclidean geometry, with constant uniform motion, and a speed limit of c, Motion induced changes, td and lc, result from the propagation speed of light being constant and independent of its source, which is based on experiment.
 
Knowing only the basics of GR:
Einstein's motivation for the GR environment is expanding the principle of relativity, to  remove the status of the inertial frame as preferential. Accomplished by including non-Euclidean geometry, via the equivalence of inertial and gravitational mass, Changes in motion result from acceleration or gravity or both.

If he could not find justification for absolute properties, he replaced them with relative ones.

Need more time to respond to your other posts. And yes, I don't fully understand some of what you propose, but will continue on.
Title: Re: Is there a discrepancy with the equivalence principle?
Post by: timey on 27/05/2017 01:00:45
Recalling the very famous second postulate of Special Relativity declared by Einstein (1905):

“The velocity c of light in vacuum is the same in all inertial frames of reference in all directions and depend neither on the velocity of the source nor on the velocity of the observer”

(But by who's rate of time are we holding the speed of light relative to?)

Albert Einstein himself emphasized in his paper in 1917:

“The results of the special relativity hold only so long as we are able to disregard the influence of gravitational fields on the phenomena”

(This renders the theory of special relativity as background dependent, in that it requires that the gravitational field be ignored, i.e. it requires a fixed, non-dynamic Euclidean background.  So in a universe where gravity is mostly weak, physicists can feel justified in ignoring the gravity field.)

Einstein had developed another theory called General Relativity that deals with gravitational fields and according to this theory the velocity of light appears to vary with the intensity of the gravitational field.

(This is known as a background independent theory because there is nothing fixed within the mathematical framework.  But the framework does break down where gravity is extreme)

Special relativity holds the speed of light as constant held relative to a static length second.  The motion of the object, as a speed held relative to the same static length second, is a percentage of the speed of light as held relative to this static length second.  The percentage of the speed of light that the motion is determines the degree of time dilation and length contraction that is observed of the moving frame, observed of the moving frame because the moving frame itself notices nothing different about their own time or length.

So you can see that there is a special relativity situation where in order to describe a flat Euclidean space and keep the speed of light constant held relative to a static length second, length of object and distance traveled are rendered variable...
And you can see that there is a general relativity situation where the dynamics of the acceleration of gravity are interpreted as being a geometry that is non-Euclidean.  This being because if the speed of light was varying as per the speed of light being held relative to GR time dilation, then light and the motion of m in the gravity field would slow down on approaching M, and the motions of m are observed to accelerate towards M, not decelerate, therefore it is the geometry of space that is non=Euclidean.

You have said concerning GR:
Accomplished by including non-Euclidean geometry, via the equivalence of inertial and gravitational mass, Changes in motion result from acceleration or gravity or both.
Acceleration and gravity are one and the same thing.

Special relativity is describing length contraction of the object in a flat Euclidean space.
General relativity is describing a non-Euclidean space of curves (waves and ripples) that an objects path through space must follow as spatial geometry.

What I am suggesting is that the non-Euclidean spatial geometry of general relativity is actually a temporal function due to a 3rd aspect of the time dilation phenomenon where m=0, and that this unifies changes in motion with acceleration/deceleration in the gravity field, where acceleration and gravity are already unified, giving acceleration/deceleration and the force of gravity a physical cause.
Title: Re: Is there a discrepancy with the equivalence principle?
Post by: timey on 27/05/2017 15:37:45
Quantum mechanics is compatible with special relativity, but it is not compatible with general relativity.  If it were compatible with general relativity then quantum would be unified with gravity.
Therefore because special relativity is a background dependent theory, so is quantum a background dependent theory.

To unify quantum with gravity surely special relativity just needs to be rendered as a background independent theory.

Special relativity can be rendered as a background independent theory, to be used in conjunction with my altered version of general relativity, if one holds the speed of light relative to the 3rd aspect time dilation for open space where m=0 that my model adds.  Simply plug in the value of M and understand that the gravitational red shift equation will be describing the value by which a light ray's wavelength is getting longer with distance from M, where the extra distance added to the wavelength with each gravitational shift can be divided by the speed of light (held relative to a static length second as per c)) for a time period that can then be progressively added to the length of the second one was holding the speed of light relative to.
This is just stating a mathematical means to say that the light 'always' travels 299 792 458 metre that are constant in length, in seconds that are becoming longer as the frequency of the light is shifted towards the red as it moves away from M.  This is the value of the 3rd aspect time dilation for open space where m=0...

Note: This is saying that if we hold the speed of light relative to the 3rd aspect time dilation seconds at higher potentials, that the light will always be travelling 299 792 458 metre per variant second of each potential, but when this is the case - if we hold the speed of light relative to a static length second as per c, 'less' distance will be traveled at each potential per static length second of c...
On the other hand, if we held the speed of light relative to the GR time dilated seconds at higher potentials, the light would always be 299 792 458 metre per variant second of each potential, but when this is the case - if we hold the speed of light relative to a static length second as per c, 'more' distance will be traveled at each potential per static length second of c.
...In either case, this being the case of stating time for space where m=0 to be running at faster rates in the higher potentials, or stating time for space where m=0 to be running slower in the higher potentials - if one were attempting to recover the speed of light from being a variable at each potential to being a constant at each potential as per the speed of light held relative to a static length second as per c at each potential, then jimmying around with lengths and distances will do the trick...
Now take heed - if one were to wrongly assume that time runs faster in space where m=0 based on the fact that time runs faster for clocks that are not m=0, when in reality time in space where m=0 actually runs slower, this would complicate matters somewhat with regards to jimmying around with lengths and distances.

Quote
General Relativity deals with gravitational fields and according to this theory the velocity of light appears to vary with the intensity of the gravitational field.
What is this saying?  Is it saying that the speed of light is being held relative to seconds being shorter in space?  If seconds are shorter in space then light will be moving faster further away from M, and slower closer to M.  Bodies of m in relation to M would slow down as they are accelerated towards M and speed up as they are decelerated away from M.  But acceleration is gravity, and gravity warps space, therefore we end up with non-Euclidean geometry.
The speed of light, as a constant held relative to the static length second as per c, is recovered via special relativity where again length and distance are compromised but this time as a means of achieving a Euclidean geometry.

My model changes things a bit...

General relativity time dilation is now for mass 'in' that potential, and 3rd aspect time dilation is for space 'at' that potential. General relativity mathematics are already inclusive of the concept of this 3rd aspect time dilation.  This being because changes in motion, i.e. acceleration/deceleration in the gravity field are the same thing as gravity, and I am stating the 3rd aspect of time dilation as being the cause of the acceleration/deceleration, meaning that the 3rd aspect time dilation and acceleration/deceleration are the same thing.  This negates the need to add non-Euclidean geometry because the cause of these curves, waves and ripples are 3rd aspect time dilation related.  This will describe space curvature.

To make special relativity a background independent theory, simply hold the speed of the craft as a percentage of the speed of light held relative to the 3rd aspect time dilation second at each potential. (this gets soooo interesting, I'm literally dying to talk to someone in these terms)
To make quantum a background independent theory, simply recognize Planck's h constant as a function of time dilation.

Is there anybody out there who can converse on these matters?

I'd continue into describing how the 5th force of the Kazula-Klien theory (that Einstein was initially excited about) can be equated with what I'm suggesting, where the theory's failure to the purpose of unification was brought about by a background dependency due to the non-dynamic nature of the addition, where my addition is actually dynamic.
Title: Re: Is there a discrepancy with the equivalence principle?
Post by: guest4091 on 27/05/2017 18:11:36
timey;
#767
Quote
So my question is - when the tests of GR set out in the 'tests of general relativity' link provided above are calculated, are the deviations from Euclidean geometry being calculated as spatial additions to the geometric distance between objects?

Spatial measurements in GR use 'curvilinear coordinates' developed from the study of surfaces by Gauss. The surface, Einsteins mollusc, is not a Euclidean environment unless you focus on a tiny portion, just as an area of 100 square meters on earth's  surface may be considered flat.

#774
Quote
But light rays are moving at a constant speed, aren't they?  So how is the geometry of spacetime distinguishing the constant speed of light?

Quote
That's light in free fall, considered equivalent to inertial motion and a straight line in Euclidean space.. In freefall you do not sense any acceleration.

...where I'll add that deceleration is no different to being at rest but with a gravitational field turned on
at rest in a g-field, the object is accelerating, (the field vectors all point to the center of mass), i.e. gravity is always ON, but an object, ground, floor, shelf, etc. is preventing its motion. You measure this acceleration with a weight scale!

Quote
The light rays being bent towards a star are being bent because time in the 'space' near the star is running faster than the time in the 'space' between the background object emitting the light rays and the star that these light rays are being bent towards

The rays are free falling in the g-field of the star. Time is not bending the rays, gravity is. Time does not cause anything. Viewing a distant clock rate, it will be different from your local clock, just as it is in SR.

There are em fields, also invisible, yet they predict em phenomena with a high degree of accuracy. If you accept these as causes, then why not accept g-fields.
The over simplistic inverse square rule for gravity leads to incomprehensible results at a distance of zero. Referring to the 'shell theorem', the rule outside the mass does not apply inside the mass. You don't need relativity to conclude that gravitation is zero at the center of mass. Considering a composition of  bodies, such as a galaxy, is inside the universe, maybe it's time to rethink the rule. (a rule is a guess, the law remains  unknown)
----------------------------------------
The reason we have theories, is because we can't know the nature/essence of the physical world. Models are built from mental images which we suppose correspond to reality outside the mind. It's a trial and error process.
Title: Re: Is there a discrepancy with the equivalence principle?
Post by: timey on 27/05/2017 23:03:43
Time is not bending the rays, gravity is

Sure - but what is gravity?

You cannot answer this because no-where in conventional physics does it tell one what gravity is...

My model says that gravity is a combination of electromagnetic (the value of which is dependent on the energy related frequency of a particle) that causes a directional attraction, and the 3rd aspect of time dilation in the gravity field (this being dependent on the vacuum energy of space, the value of which is linked to the strength of the gravitational field. i.e. more energy in the stronger field, less energy in the weaker field) that is responsible for causing acceleration/deceleration.

I repeat again... that if a body is in motion in a field that has progressively changing rates of time, that bodies motion will be accelerated where seconds become shorter, and decelerated where seconds become longer.  This is synonymous to a Doppler shift, except the Doppler shift is a time shift, not a distance shift...
This can give an exact description of gravitationally shifted light, and the analysis of the redshift spectrum.

The best you are going to get from conventional physics as an explanation for the phenomenon of gravity, is that gravity and acceleration are the same thing and that gravity warps the geometry of space.
The physics books that I read are saying that a deeper understanding is required.
You would be 'comfortable' talking to me about places where there are copies of ourselves living out the probabilities of other worlds.  You would be comfortable talking to me in terms of other dimensions with regards to string theory.  You would feel comfortable talking to me in terms of the framework of GR or SR time dilation.  All of these subjects you can find raised within the physics literature.  Talking about a 3rd dimension of the time dilation phenomenon is a subject that I am raising as a 'New Theory'.  In light of the other theories that I have mentioned, I don't think that my idea is that outlandish tbh...
Title: Re: Is there a discrepancy with the equivalence principle?
Post by: timey on 29/05/2017 12:59:22
String theory arose in the attempt to unify the forces.  It has sought to unify general relativity with quantum, but also runs into the problems of back ground dependency.  Some versions are partially reminiscent of the Kuluzar-Klien theory in the use of a higher dimensional general relativity, but again there is failure due to the non-dynamic nature of background dependency.  Many string theorists believe that the fact of the many versions of string theory that have emerged are pointing to the existence of a deeper more fundamental theory that has even been given a name, M-theory, despite the details of this theory remaining a mystery.

My alterations to general relativity (which can be considered as a higher dimensional general relativity) gives a background independent theory that retains the dynamic nature of space as a 3rd aspect time dilation phenomenon, resulting in a spatial geometry that under all circumstance emerges as Euclidean... All I am doing here is saying that particle frequency is time dilation related, and that the phenomenon of time itself is energy related.
I suspect that string theory might benefit immensely from employing my remit of +energy=shorter seconds to it's notion of string 'vibrations', where the result (I think, scratches head) would be a reduction in the necessity for extra dimensions, and a unification of 'some' of the differing string theories.
Title: Re: Is there a discrepancy with the equivalence principle?
Post by: timey on 30/05/2017 15:42:50
Ok - well it would seem no-one at this forum is willing to converse at the level of theoretical physics, so I shall continue with Lee Smolin.

"The Trouble with Physics" chapter 13: Surprises from the Real World - pages 204 ,205, 207 & 208.
Quote
:Lee Smolin
Let us start with the cosmological constant, thought to represent the dark energy accelerating the universe's expansion.  As discussed in chapter 10, this energy was not anticipated by string theory, nor by most theories, and we have no idea what sets its value.  Many people have thought hard about this for years, and we are more or less nowhere.  I don't have an answer either, but I have a proposal for how we might find one.  Let's stop trying to account for the cosmological constant's value in terms of known physics.  If there is no way to account for the phenomenon on the basis of what we know, then maybe this is a sign that we need to look for something new.  Perhaps the cosmological constant is a symptom of something else, in which case it might have other manifestations.  How are we to look for them, or recognize them?

The answer will be simple. because universal phenomenon are ultimately simple.  Forces in physics are characterized by just a few numbers - for example, the distance over which a force travels and a charge to tell us how strong it is.  What characterizes the cosmological constant is a scale, which is the distance scale over which it curves the universe.  We can call this scale R.  It is about 10 billion light years, or 10power27 centimeters.  What is weird about the cosmological constant is that its scale is huge compared with other scales in physics.  Scale R is 10power40 times the size of an atomic nucleus and 10power60 times the Planck scale (which is about 10power-27 times the size of a proton).  So it's logical to wonder whether scale R might reflect some totally new physics.  A good approach would be to look for phenomenon that happen on the same vast scale.

Does anything else happen on the the scale of the cosmological constant?  Let's start with cosmology itself.  The most precise cosmological observations we have are the measurements of the 'cosmic microwave background'.  This is the radiation left over from the Big Bang, which comes to us from all directions of the sky.  The radiation is purely thermal - that is, random.  It has been cooling as the universe expands, and it is now at the temperature of 2.7 degrees Kelvin.  The temperature is uniform across the sky to a high degree of precision, but at the level of a few parts in 100 000 there are fluctuations in it.  The patterns in these fluctuations gives us important clues to the physics of the early universe.

Over the last decades, the temperature fluctuations of the microwave background have been mapped by satellites, balloon-borne detectors, and ground based detectors.  One way to understand what these experiments measure is to think of the fluctuations as if they were sound waves in the early universe.  It is then useful to ask how loud the fluctuations are at different wavelengths.  The results give us a picture, which tells us how much energy there is at various wavelengths.

The picture is dominated by a large peak, followed by several smaller peaks.  The discovery of these peaks is one of the triumphs of contemporary science.  They are interpreted by cosmologists to indicate that matter filling the early universe was resonant, much like the head of a drum of the body of a flute.  The wavelength at which a musical instrument vibrates is proportional to its size, and the same is true of the universe.  The wavelengths of the resonant modes tell us how big the universe was when it first became transparent: that is, when the initial hot plasma developed, or 'de-coupled'. into separate realms of matter and energy some three hundred thousand years after the Big Bang, at which time the microwave background became visible.  These observations are extremely helpful in tying down the parameters of our cosmological models.

Another feature we see in the data is that there is very little energy in the largest wavelength.  This may be just a statistical fluctuation, because it involves a small number of pieces of data.  But if it is not a statistical fluke, it can be interpreted as indicating a cut-off, above which the modes are much less excited.  It is interesting that this cutoff is at the scale of R, associated with the cosmological constant.

The existence of such a cutoff would be puzzling from the point of view of the most widely accepted theory of the very early universe, which is 'inflation'.  According to the theory of inflation, the universe expanded exponentially fast during one extremely early period.  Inflation accounts for the observation that the cosmic background radiation is so nearly uniform.  It does this by ensuring that all parts of the universe we see now could have been in causal contact when the universe was still a plasma..

The theory also predicts the fluctuations in the cosmic microwave background, which are hypothesized to be remnants of quantum effects during the period of inflation.  The uncertainty principle implies that the fields dominating the energy of the universe during inflation fluctuate, and these fluctuations become imprinted on the geometry of space.  As the universe expand exponentially, they persist, causing fluctuations in the temperature of the radiation produced when the universe becomes transparent.

Inflation is believed to have produced a huge region of the universe with relatively uniform properties.  This region of the universe is thought to be many orders of magnitude larger than the observable region, because of a simple argument about scales.  If inflation had stopped just at the point where it created a region as large as we now observe, there must have been some parameter in the physics of inflation that selected a special time to stop, which just happens to be our era.  But this seems improbable, because inflation took place when the universe had a temperature ten to twenty orders of magnitude greater than the center of the hottest star today; thus the laws governing it must have been different laws, which dominate physics only in those extreme conditions.  There are many hypotheses about the laws that govern inflation, and none of them say anything about a time scale of 10 billion years.  Another way to put this is that there seems no way for the present value of the cosmological constant to have anything to do with the physics that caused inflation.

Thus if inflation produced a uniform universe on the scale that we observe, it likely produced a universe that is uniform on much larger scales.  This in turn implies that the pattern of fluctuations produced by inflation should go on and on, no matter how far you look.  If you could see beyond the present size of the observable universe, you should continue to see small fluctuations in the cosmic microwave background.  Instead, the data hint that the fluctuations may cease above the scale of R.

Indeed, as cosmologists have examined the large-scale modes in the microwave background, they have found more mysteries.  It's an item of faith among cosmologists that at the largest scales the universe should be symmetric - that is, any one direction should be like any other.  This is not what is seen.  The radiation in these large-scale modes is not symmetric; there is a preferred direction. (It has been called "axis of evil" by the cosmologists Kate Land and Joao Magueijo).  No one has any rational explanation for this effect.

These observations are controversial because they disagree profoundly with what we would expect on the basis of inflation.  Since inflation explains so much of cosmology, many prudent scientists suspect that there is something wrong with the microwave data.  Indeed , it is always possible that the measurements are just wrong.  A lot of delicate analysis is applied to the data before they're presented.  One thing that's done is to subtract the radiation known to come from the galaxy that we live in.  This may have been done incorrectly, but few experts familiar with the details of how the data are analyzed believe that to be the case.  Another possibility, as noted, is that our observations are just statistical anomalies.  An oscillation at a wavelength of the scale R takes up a huge part of the sky - about 60 degrees; consequently we see only a few wavelengths, and there are only a few pieces of data, so what we are seeing may just be a random statistical fluctuation.  The chances of the evidence for a preferred direction being a statistical anomaly have been estimated at less than 1 part in 1000.  But is may be easier to believe in this unlikely bad luck than to believe that the predictions of inflation are breaking down.

These issues are currently unresolved.  For the time being, it is enough to say that we went looking for strange physics on the scale of R and found it.

My contracting cyclic model gives physical cause and effect mechanics for it's Big Bangs and Inflation periods, and this data of a preferred direction will correspond to those mechanics.
Given that anyone can understand that a 'contracting' universe will eventually result in all the mass of the universe being in the same location, i.e. a super-massive black hole, and that a super-massive back hole with no counterpart gravitational force acting upon it will explode it's plasma in an outward trajectory in an inflation like manner, I will continue onto the next few pages of Lee Smolin's book concerning the R scale with respect MOND.
Title: Re: Is there a discrepancy with the equivalence principle?
Post by: guest4091 on 30/05/2017 16:56:42
timey;

#779

Quote
The physics books do not give any description as to 'how' a clock changes it's rate (as per GR) in the differing potentials of space, they only describe that a clock's rate of time will change.

From "The Meaning of Relativity", Albert Einstein,5 th edition, paraphrasing:
'the g-field imparts/transfers energy to the test mass moving through it, but without detailed explanation due to lack of understanding, and a similar state for em fields for composite structures. The energy of the mass and its g-field and em field are included in the energy tensor '

Quote
Clearly, from the nature of the question, you are assuming that the rate of time for a clock in the differing potentials of space is changing because the rate of time in the differing potentials of space changes.

I'm concluding a clock in a g-field runs slower due to stress. If F=gm, then a mechanical clock requires more energy for the moving parts. A light clock at the surface, or any fixed postion, loses energy moving upward and gains energy downward, but the loss is greater than the gain, similar to the SR scenario of the round trip reflection in the direction of motion.

Quote
the clock "in' the gravity potential is ticking at a rate that is independent and differing from the rate of time in the space of that position of gravity potential,
but its rate is determined by its position

As I see it, as m enters the g-field of M, it accelerates toward the center of mass M on a free fall path, its clock process slowing (longer seconds), which is opposite of your description.

#781
Quote
(But by who's rate of time are we holding the speed of light relative to?)
SR  1st postulate: physical processes behave in the same manner in all inertial frames. The propagation of light is such a process, therefore the measured speed of light is the same in all inertial frames.

Quote
(This renders the theory of special relativity as background dependent, in that it requires that the gravitational field be ignored, i.e. it requires a fixed, non-dynamic Euclidean background.  So in a universe where gravity is mostly weak, physicists can feel justified in ignoring the gravity field.)

It is relativity restricted to a Euclidean geometry but still vary much dynamic.

Quote
Einstein had developed another theory called General Relativity that deals with gravitational fields and according to this theory the velocity of light appears to vary with the intensity of the gravitational field.

It's an extension of the relativity principle to include gravitation and arbitrarily chosen frames of reference/coordinate systems, thus eliminating any preferred frames.

Quote
SR time dilation can be described as losing, or gaining potential energy due to motion relative to the gravitational field.

SR time dilation works without g-fields!

#784
Quote
I repeat again... that if a body is in motion in a field that has progressively changing rates of time, that bodies motion will be accelerated where seconds become shorter,

Consider the spherical surface of the earth radiating gravitational elements (gravel)  per square meter, which form a field extending outward without limit. As m enters the field on a straight line course that passes earth, its altitude above the surface decreases, with an increase in frequency of gravel encounters (greater density/sq meter). The field strength is accelerating m, and a clock with m would run slower (at a lower potential in GR terms)
I can equate your 'changing rates of time' to 'changing field strength.'

Time dilation results from light chasing a moving target, in constant or changing speeds.

I  think the 'many worlds' theory is nonsense. It's the meteorologist who compensates for his ineptitude by predicting all forms of weather probabilities, thinking one of them will be correct.

Ed Smolin;
"We are not accustomed to thinking of space as an entity with properties of it's own, but it certainly is".
If you can discover how a structured space, any type field, stores energy, and the process of releasing it to an object passing through it, you will answer many questions.
Title: Re: Is there a discrepancy with the equivalence principle?
Post by: timey on 30/05/2017 19:08:34
I can equate your 'changing rates of time' to 'changing field strength.'

Good!  Because changes in field strength can be equated with changes in energy, and changes in energy can be equated with changes in the rate of time.  +energy=shorter seconds being the crux of my theory.

The observation of GR time dilation is inclusive of changes in frequency, and changes in frequency are proportional to changes in energy.  You cannot get a change in frequency without a change in energy.
Clocks at greater elevation, according to the framework of GR, have a higher frequency, and therefore must have a higher energy.
Clocks in relative motion will have a decreased frequency compared to a stationary clock, and therefore must have a lesser energy.

SR time dilation works without g-fields!

Erm, nope it doesn't.  SR time dilation can only work in a g-field and has only been observed in a g-field.  There is no place in the universe that does not have a g-filed.  It is SR mathematics that ignore the g-field, but add the non-Euclidean geometry via length contraction.  If you fly hypothetical rockets around at differing speeds and trajectories with the intention that they meet at arbitrary locations, you cannot get the rockets to time mesh at the meeting points.   
What you can say is that where motion is being imparted to an object by the gravitational field, that SR mathematics adds non-Euclidean geometry via length contraction, but this completely ignores GR time dilation effects, and constitutes a really weird upside down attempt to reconcile SR Euclidean geometry with GR non- Euclidean geometry.

My model is better!  SR mathematics can be reconciled with GR mathematics under my remit, where the GR non-Euclidean geometry of space is exchanged for a Euclidean geometry in space and acceleration/deceleration is then a 3rd aspect time dilation phenomenon.  There is then no need to use the length contraction mathematics of SR because we are not expecting SR to describe any actual spatial changes of geometry, be that the geometry of space, or the geometry of the moving body.  We are stating that length contraction is merely a result of measuring a frame that is ticking at one rate of time, from a frame where the clock is ticking at a differing rate, via a speed of light that is being held relative to the tick rate of the frame that is doing the measuring.

Time dilation results from light chasing a moving target, in constant or changing speeds.

That is a really odd way of looking at things... Not one I've heard before.  Why would light chasing a target cause time dilation?

Quote
Lee Smolin
"We are not accustomed to thinking of space as an entity with properties of it's own, but it certainly is".
If you can discover how a structured space, any type field, stores energy, and the process of releasing it to an object passing through it, you will answer many questions.

I am saying that the structure of space is Euclidean, that the field is a field of time dilation, and that energy is the same thing as time.  This will indeed answer a lot of questions, as applying this remit to physics results in my model of a fully described contracting cyclic universe.  A model that provides a do-able experiment to prove or disprove itself.
Many highly intelligent people have worked on string theory for 40 years without being able to provide a do-able experiment to prove or disprove string theory...

As to the rest of your post - I'm not sure if you are trying to educate me on that which conventional physics states, or if this is your interpretation of how gravity works.
In the first instance, there is no need.  I am aware of what conventional physics states, and the fact is that conventional physics states that it does not know what gravity is, other than gravity being the same thing as acceleration, and that acceleration causes a dynamic non-Euclidean geometry of space.
In the second instance, well fair enough, but I am talking about my model of a contracting cyclic universe, and in my model, gravity, acceleration, energy and time are all the same thing, causing a dynamical background independent temporal structure of space within a Euclidean geometry of spatial dimensions.
Title: Re: Is there a discrepancy with the equivalence principle?
Post by: guest4091 on 01/06/2017 17:03:04
timey;
Theories require accepting some parts on faith, without proof. The field is one of those. Newtonian gravity produced measured results that agreed with prediction, thus was acceptable. Today, still with no understanding of a field, except it's association with mass, Relativity is successful. This shows we don't need a complete understanding of all elements of a theory.
Based on this latest post, I would question your current understanding of conventional physics. This has been a long bus ride, and this is where I get off.
Title: Re: Is there a discrepancy with the equivalence principle?
Post by: timey on 01/06/2017 21:50:11
I would question your current understanding of conventional physics.

So it would seem... but it would appear to be you that does not understand that if GR describes a non-Euclidean geometry of space, but SR can ignore the effects of gravity (perhaps you do not understand that there is nowhere in the real world universe that does not have a gravitational field), but calculate a trajectory as a Euclidean geometry of space correctly as per experiment in the real world, that the means by which SR changes length/distance must correspond with the way that GR describes a non-Euclidean geometry.

Relativity is successful. This shows we don't need a complete understanding of all elements of a theory.

This is a point of view that is not shared by the physicists who's books I read, and who's lectures I watch, nor shared by the experimentalists who conduct physics experiments. That you say this leads me to question your understanding of theoretical physics.  The entire point of experiments such as LHC, LIGO, etc, too many to mention, are to answer the questions that Relativity and Quantum Mechanics do not answer, which again are too numerous to mention, but the fact that GR and QM are incompatible is the usual starting point, whether you are a string theorist, or working on any other type of unification theory.

If you cannot discuss GR and SR as per why one results in a non-Euclidean geometry, and why the other is reliant on a Euclidean geometry, in terms of the speed/distance/time formula, then you won't be able to converse in terms of the R scale with respect to MOND.  Or my plans to venture the conversation to discussing DSR, (Doubly Special Relativity).  I asked you what you would say if I thought I had a theory of everything that described a contracting cyclic universe.  You said that you would say show me the mathematics, or something thereof.  I can't show you the mathematics other than point you to Einstein's equations of GR that will describe a universe that is contracting and then describe how this changes current view.  It would seem to me that either you question my rights(?) to change current view, or simply don't understand current view as well as you think you do.  It hasn't been a long road.  You didn't get on the bus.
Not to worry though, I wish you well.  ...And if you do ever start to have questions about why it is that physics two best current working theories are not compatible with each other, and why physics cannot explain so much about our universe, then I highly recommend Lee Smolin's book "The Trouble With Physics".  He is very concise...
Title: Re: Is there a discrepancy with the equivalence principle?
Post by: timey on 03/06/2017 14:32:47
Actually I think I'll discuss DSR first...

https://en.wikipedia.org/wiki/Doubly_special_relativity

Lee Smolin describing how himself and Jaoa Magueijo came to DSR 2.

Quote
:Lee Smolin
But at some point during that afternoon, we hit on a key factor that had evaded us for months, having to do with trading momenta for positions.  When we were done, we had invented a second version of DSR, much simpler than the one developed by Giovanni Amelino-Camelia.  Now it is known to experts as DSR 2.
This was what Joao had wanted.  In our version, photons that have more energy travel faster.  Thus, in the very early universe when the temperature was very high, the speed of light was, on average, faster than it is now.  As you go back further in time and the temperature approaches the Planck energy, the speed of light becomes infinite.  It took somewhat longer to show that this led to a version of a variable-speed-of-light theory that was also consistent with the principles of general relativity, but we eventually got there, too.  We call this theory "Gravity's Rainbow" after Thomas Pynchon's novel.
"Doubly special relativity' is a stupid name, but it has stuck.  The idea is an elegant one, by now much studied and discussed.  We don't know if it describes nature, but we know enough about it to know that it could.

I mention DSR because it has similarities to my hypotheses.

Smolin says:
Quote
In our version, photons that have more energy travel faster.

In my model - photons that have more energy also travel faster, but they have more energy due to the strength of the gravitational field, and travel faster due to a 3rd aspect of the time dilation phenomenon where m=0, because time where m=0, i.e.the g-field, is running faster in a stronger gravitational field (lower potential), and is running slower in the weaker gravitational field, (higher potential)
This describes the accelerative/decelaritive phenomenon of the g-field where m=0. (and can describe a directional force via the electromagnetic, where the electromagnetic can be easily described in Euclidean spaces)

All I have done is render the spatial distances of non-Euclidean space given by general relativity as temporally derived by an added 3rd aspect to the time dilation phenomenon.  Now general relativity is a five dimensional dynamical back ground independent theory from which Euclidean geometry emerges.  Gravitational fields, waves and ripples are areas, or bands of energy that manifest themselves as rates of time that are running faster than the rate of time in the lesser energy of the space they are moving through.

Mass can be considered in the same way.
Where m doesn't equal 0 we see GR and SR time dilation that cause the frequency of the clock to change.
A higher potential (weaker gravity field) causes the GR time dilated clock to increase in frequency. (increased tick rate, faster rate of time)  An increase in frequency requires an increase in energy.
A lower potential (stronger gravity field) causes the GR time dilated clock to decrease in frequency (decreased tick rate, slower rate of time).  A decrease in frequency requires a decrease in energy.

My model states that the rate of time anywhere, for both mass and for where m=0, is energy related.  Where m=0 it is the vacuum energy of space, and the gravitational field strength that applies.  Where mass is concerned we look to the conservation of energy law that states that it is the sum of both gravity potential energy and kinetic energy that is conserved.  (relativistic mass is not necessary in my model, which I can expand upon) Thus stating that it is potential energy changes that are causing the increase or decrease in the frequency (tick rate) of the GR time dilated clock.  This with a bit more juggling can be applied to the clock in relative motion to the gravity field, where gp is converted to ke, and describes the decrease in frequency of the SR clock in relative motion.

Smolin says:
Quote
Thus, in the very early universe when the temperature was very high, the speed of light was, on average, faster than it is now.  As you go back further in time and the temperature approaches the Planck energy, the speed of light becomes infinite.

In the very early universe, light did not shine at-all.  The temperature caused a plasma that was opaque.  We are to assume of course that all the mass, and energy, that we see 'out there' was part of this plasma before it got 'out there' by means of an inflation period.
This is getting to the heart of the Big Bang considerations.  Current physics cannot take us to the causation of inflation, or to the physics of the big bang.

My model makes the argument that the universe is cyclic.  It also makes the argument that the universe is currently contracting, and has been very slowly contracting at an accelerating rate since the moment that the inflation period ceased.
My model looks at what happens when great quantities of mass conglomerate, and makes the argument that blackholes fit the description of plasma. Light does not shine.  The temperature is observed to decrease inversely proportional to increase in mass.  More gravitational field, less energy can escape, unless that field is disturbed either by another equivalent field (see binary black holes), or by the sudden lacking of another field. (see black hole mass intake and superluminal jets)
My model proposes that the universe has been contracting very slowly at an accelerating rate since the moment that the inflation period ceased.  That this contraction will eventually result in all the mass of the universe being located in one area, this being a super massive black hole.  This will have occurred as a result of blackholes merging with one and another.  When there is only one, an Inflation Period is caused by there be no other gravitational equivalent acting upon the singular black hole, and this singular black hole shall disperse all of the plasma via it's superluminal jets, leaving a sea of cooling particles swirling in close proximity to each other across a vast tract of space. (dynamical emergent space)
My model says that all clumping development is the causation of the very slow contraction of the spacial dimensions of the universe, and is the causation of the red shift observations, in that the gravity fields between masses are becoming weaker as mass further clumps, and that distances between masses are slowly increasing as these clumps of mass further clump.
This gives us a causal description for galaxy clusters.
This gives us a causal description of red shift observation.
This gives us a causal description for Big Bangs.
This gives us a causal description for Inflation period.
This gives us a causal description for the 'axis of evil' and the observations of a preferred direction.
(It does not give us a causal description of how cyclic universes are created, but the change in interpretation could give some clues as to that)

I'll get onto how the scale of R relates to the concept of dark energy, and how the scale of R relates to MOND, and to how MOND relates to dark matter, in my next post...
Title: Re: Is there a discrepancy with the equivalence principle?
Post by: timey on 04/06/2017 18:03:59
Lee Smolin then goes on to describe work that has been undertaken in the field of quantum gravity and quantum loop gravity, culminating in:

"The Trouble With Physics" chapter 15 : 'Physics after String Theory' - pages 255, 256, & 257
Quote
:Lee Smolin
While there is today an exciting sense of progress among quantum-gravity theorists, there is also a strong expectation that the road ahead will bring at least a few surprises.  Unlike string theorists in the exhilarating says of the two super-string revolutions, few of the people working on quantum-gravity believe they have their hands on a final theory.  We recognize that the accomplishments of background-independent approaches to quantum gravity are a necessary step to finishing Einstein's revolution.  They show that there can be a consistent mathematical and conceptual language that unifies quantum theory and general relativity.  This gives us something that string theory does not, which is a possible framework in which to formulate the theory that solves all five of the problems I listed in chapter 1.  But we are also fairly sure that we do not yet have all the pieces.  Even with the recent successes, no idea has that absolute ring of truth.

When you look back at the history of physics, one thing sticks out:  When the right theory is finally proposed, it triumphs quickly.  The few really good ideas about unification appear in a form that is compelling, simple, and unique; they do not come with a list of options or adjustable features.  Newtonian mechanics is defined by three simple laws, Newtonian gravity by a simple formula with one constant.  Special relativity was complete on arrival.  It may have taken twenty five years to fully formulate quantum mechanics, but from the beginning it was developed in concert with experiment.  Many of the key papers in the subject from 1900 on either explained a recent experimental result or made definite prediction for an experiment that was shortly done.  The same was true of general relativity.

Thus, all the theories that triumphed had consequences for experiment that were simple to work out and could be tested within a few years.  This does not mean that the theories could be solved exactly - most theories never are.  But it does mean that physical insight led immediately to a prediction of a new physical effect.

Whatever else one says about string theory, loop quantum gravity, and other approaches, they have not delivered on that front.  The standard excuse has been that experiments on this scale are impossible to perform - but, as we've seen, such is not the case.  So there must be another reason.  I believe that there is something basic that we are all missing, some wrong assumption we are all making.  If this is so, then we need to isolate the wrong assumption and replace it with the new idea.

What could that wrong assumption be?  My guess is that it involves two things: the foundations of quantum mechanics and the nature of time.  We have already discussed the first.  I find it hopeful that new ideas about quantum mechanics have been proposed recently, motivated by studies of quantum gravity.  But I strongly suspect that the key is time.  More and more, I have the feeling that quantum theory and general relativity are both deeply wrong about the nature of time.  It is not enough to combine them.  There is a deeper problem, perhaps going back to the origin of physics.

Around the beginning of the seventeenth century, Descartes and Galileo both made a wonderful discovery:  You could draw a graph, with one axis being in space and the other being in time.  A motion through space then becomes a curve on the graph.  In this way, time is represented as if it were another dimension of space.  Motion is frozen, and a whole history of constant motion and change is presented to us as something static and unchanging.  If I had to guess (and guessing is what I do for a living), this is the scene of the crime.

We have to find a way to unfreeze time - to represent time without turning it into space.  I have no idea how to do this.  I can't conceive of mathematics that doesn't represent a world as if it where frozen into eternity.  It's terribly hard to represent time, and that's why there is aa good chance that this representation is the missing piece.

I do have an idea as to how to represent time without turning it into space.  It involves a combination of the mathematics of general relativity and special relativity, a rethink on the spatial nature of the non-Euclidean geometry that is the result of general relativity as being temporally derived, leading to the addition of a 3rd aspect to the time dilation phenomenon for where m=0 in the gravity field, and a relationship between the phenomenon of time with energy.  Simpy hold special relativity speed percentages relative to the speed of light as per the 3rd aspect time dilation of the gravity field, where this time dilation is responsible for changes in acceleration, and you will be describing the passage through time without turning that time into space.
When one applies the +energy=shorter seconds remit to quantum mechanics, this places time dilation into the realm of quantum mechanics, and gives quantum mechanics a background independent 'temporal' geometry from which 3 dimensions of space emerge as per Euclidean geometry, that is compatible with the altered arrangement of general relativity, that results in a temporally dynamic background independent theory from which 3 dimensions of space emerge as per Euclidean geometry.
Title: Re: Is there a discrepancy with the equivalence principle?
Post by: timey on 10/06/2017 00:18:50
http://www.spacedaily.com/reports/Model_Suggests_Pre_Big_Bang_Physics.html

Quote
:link
By combining quantum physics with general relativity, however, Ashtekar and colleagues report they have been able to develop a model that describes a transition from a previous universe, through the Big Bang to an expanding universe that exhibits physics similar to the one that exists today.

Reporting in the current issue of Physical Review Letters, the team said their calculations reveal that prior to the Big Bang, there was a contracting universe with space-time geometry otherwise similar to the current expanding universe.

As gravitational forces pulled this previous universe inward, it reached a point at which the quantum properties of space-time cause gravity to become repulsive, rather than attractive.

Here Ashtekar and team are using a combination of quantum physics and general relativity to develop a model where the calculations reveal that prior to the Big Bang, there was a contracting universe with space-time geometry otherwise similar to the current expanding universe.

My cyclic model suggests that our universe is 'currently' contracting as per these mathematics of a contracting space-time geometry, which will be compatible with my model's +energy=shorter seconds remit.
My model then gives cause and effect mechanics for the Big Bang and an Inflation period.  Where the 'axis of evil' implications of preferred direction are described by these cause and effect mechanics, leaving a universe comprised of a vast sea of particles/energy that starts contracting from the point the Inflation period ceases.
This cyclic model that I have invented states that the development of what we observe of our universe today has occurred during the contraction, where the gravitational clumping of mass 'is' the contraction.
Title: Re: Is there a discrepancy with the equivalence principle?
Post by: timey on 14/06/2017 13:25:55
"QED" Richard P Feynman - chapter 2: Photons: Particles of Light - page 55 & 56
Quote
: Feynman
This is an example of the 'uncertainty principle': there is a kind of "complementary" between knowledge of where the light goes between the blocks and where it goes afterwards - precise knowledge of both is impossible.
I would like to put the uncertainty principle in its historical place: When the revolutionary ideas of quantum physics were first coming out, people still tried to understand them in terms of old-fashioned ideas (such as, light goes in straight lines).  But at a certain point the old-fashioned ideas would begin to fail, so a warning was developed that said, in effect, "Your old-fashioned ideas are no damn good when..."  If you get rid of all the old-fashioned ideas and instead use the ideas that I'm explaining in these lectures - adding arrows for all the ways an event can happen - there is no need for an uncertainty principle!

However - there is still no model that explains why these quantised effects occur.

It is my suggestion that 'discreteness' is a consequence of not accounting for time dilation, where events are not occurring in sinc with the experiments inherent time base structure.

My suggestion, as well as incorporating an added 3rd aspect of time dilation for where m=0, is that a +energy = shorter seconds approach can be employed.  This gives causality to the QED adding up of arrows model that Feynman describes, where the path of least time is the most probable route, and Planck's h constant and the Hamiltonian can be mathematically re-interpreted as a functions of time dilation, where a division of wave length/wavefunction by energy, (via frequency, in some cases) will be informative of the rate of time for the particle, system or geometrical space.
(I think this notion may be compatible with non-commutative geometry)
Title: Re: Is there a discrepancy with the equivalence principle?
Post by: Quantum Antigravity on 15/06/2017 01:55:19
   
Then on the other hand - there is the school of thought that a person will age in keeping with their time dilated clock as described in the NIST link above, and the link below.
 

   
   
It would seem that a person will age in keeping with their clock. Right?

People age in time, obviously.
But is the passage of time the direct cause of aging?
How exactly the passing time causes aging of a person?
     
But first, let's define what we mean by: " aging person " .

Who/what is aging?  Person?  :)   

I would say that it is the body of the person that is aging. 

But how the body is aging? Body is made of atoms.
Are body's atoms aging? 

If atoms of the body are not aging with time, than how exactly the body is aging?
 
Well, in my humble opinion, there is an essential difference
between the animate and the in-animate matter.
 
Einstein's theory of relativity applies to the in-animate matter only.
   
In my opinion, we can say that under normal circumstances on Earth
there is some coherent correspondence between
clocks, time passing, and people aging. 
   
Clocks make sense only so far as they correspond to Earth's spin.

But if a person leaves Earth and travels in space,
the Cesium atoms of the quantum clock do not correspond
neither to Earth's spin, nor to Earth's gravity, so such "time"
from the point of view of a person aging becomes meaningless,
because there simply  isn't anymore any correspondence between
clocks, time passing, and people aging.   
 
A living person is not an atomic clock,
and body's atoms are not aging.
 
So, to make a long story short, Einstein's theory of relativity
does NOT apply to living beings (animate matter).   
   
ONE LAST THING :

A living person is not an atomic clock,
and a ticking clock is not an equivalent of time passing.
   
CLOCKS ARE NOT TIME. 

CLOCKS ARE NOT POWERED BY THE FLOW OF TIME
IN THE SAME WAY AS WATER WHEELS ARE POWERED
BY THE FLOW OF WATER.
 
SAME APPLIES TO CESIUM ATOMS. 
 
THERE IS NO PROOF THAT THE FREQUENCY OF A CESIUM ATOM HAS ANYTHING TO DO WITH TIME PASSING -- IT IS MERELY AN AXIOMATIC ASSUMPTION.
     
WE CAN EXPERIMENTALLY DIRECTLY MEASURE THE FREQUENCY OF A CESIUM ATOM BY COMPERING IT WITH ANOTHER FREQUENCY, BUT WE CANNOT EXPERIMENTALLY DIRECTLY MEASURE THE FREQUENCY, OR "SPEED", OF TIME PASSING.

THERE ARE NO EXPERIMENTAL DETECTORS THAT CAN DIRECTLY DETECT THE PHYSICAL EXISTENCE OF TIME, NOT TO MENTION DETECTING THE "SPEED" OF TIME.  :)


 
FUNCTIONING OF CLOCKS AND CESIUM ATOMS HAVE NOTHING TO DO WITH TIME  :) 
Title: Re: Is there a discrepancy with the equivalence principle?
Post by: timey on 15/06/2017 11:51:19
Red tower blocks in Bold!  Fancy!

Anyway - as I was saying...

http://www.spacedaily.com/reports/Model_Suggests_Pre_Big_Bang_Physics.html

Quote
:link
By combining quantum physics with general relativity, however, Ashtekar and colleagues report they have been able to develop a model that describes a transition from a previous universe, through the Big Bang to an expanding universe that exhibits physics similar to the one that exists today.

Reporting in the current issue of Physical Review Letters, the team said their calculations reveal that prior to the Big Bang, there was a contracting universe with space-time geometry otherwise similar to the current expanding universe.

As gravitational forces pulled this previous universe inward, it reached a point at which the quantum properties of space-time cause gravity to become repulsive, rather than attractive.

Here Ashtekar and team are using a combination of quantum physics and general relativity to develop a model where the calculations reveal that prior to the Big Bang, there was a contracting universe with space-time geometry otherwise similar to the current expanding universe.

"QED" Richard P Feynman - chapter 2: Photons: Particles of Light - page 55 & 56
Quote
: Feynman
This is an example of the 'uncertainty principle': there is a kind of "complementary" between knowledge of where the light goes between the blocks and where it goes afterwards - precise knowledge of both is impossible.
I would like to put the uncertainty principle in its historical place: When the revolutionary ideas of quantum physics were first coming out, people still tried to understand them in terms of old-fashioned ideas (such as, light goes in straight lines).  But at a certain point the old-fashioned ideas would begin to fail, so a warning was developed that said, in effect, "Your old-fashioned ideas are no damn good when..."  If you get rid of all the old-fashioned ideas and instead use the ideas that I'm explaining in these lectures - adding arrows for all the ways an event can happen - there is no need for an uncertainty principle!

(I think this notion may be compatible with non-commutative geometry)

Where my model states non-commutative/non-Euclidean geometry as a temporal phenomenon from which 3 dimensions of Euclidean 'space' geometry emerge.
Title: Re: Is there a discrepancy with the equivalence principle?
Post by: timey on 23/06/2017 13:47:44
"The Trouble with Physics" chapter 13: Surprises from the Real World - pages 204 ,205, 207 & 208.
Quote
:Lee Smolin
Let us start with the cosmological constant, thought to represent the dark energy accelerating the universe's expansion.  As discussed in chapter 10, this energy was not anticipated by string theory, nor by most theories, and we have no idea what sets its value.  Many people have thought hard about this for years, and we are more or less nowhere.  I don't have an answer either, but I have a proposal for how we might find one.  Let's stop trying to account for the cosmological constant's value in terms of known physics.  If there is no way to account for the phenomenon on the basis of what we know, then maybe this is a sign that we need to look for something new.  Perhaps the cosmological constant is a symptom of something else, in which case it might have other manifestations.  How are we to look for them, or recognize them?

The answer will be simple. because universal phenomenon are ultimately simple.  Forces in physics are characterized by just a few numbers - for example, the distance over which a force travels and a charge to tell us how strong it is.  What characterizes the cosmological constant is a scale, which is the distance scale over which it curves the universe.  We can call this scale R.  It is about 10 billion light years, or 10power27 centimeters.  What is weird about the cosmological constant is that its scale is huge compared with other scales in physics.  Scale R is 10power40 times the size of an atomic nucleus and 10power60 times the Planck scale (which is about 10power-27 times the size of a proton).  So it's logical to wonder whether scale R might reflect some totally new physics.  A good approach would be to look for phenomenon that happen on the same vast scale.

Does anything else happen on the the scale of the cosmological constant?  Let's start with cosmology itself.  The most precise cosmological observations we have are the measurements of the 'cosmic microwave background'.  This is the radiation left over from the Big Bang, which comes to us from all directions of the sky.  The radiation is purely thermal - that is, random.  It has been cooling as the universe expands, and it is now at the temperature of 2.7 degrees Kelvin.  The temperature is uniform across the sky to a high degree of precision, but at the level of a few parts in 100 000 there are fluctuations in it.  The patterns in these fluctuations gives us important clues to the physics of the early universe.

Over the last decades, the temperature fluctuations of the microwave background have been mapped by satellites, balloon-borne detectors, and ground based detectors.  One way to understand what these experiments measure is to think of the fluctuations as if they were sound waves in the early universe.  It is then useful to ask how loud the fluctuations are at different wavelengths.  The results give us a picture, which tells us how much energy there is at various wavelengths.

The picture is dominated by a large peak, followed by several smaller peaks.  The discovery of these peaks is one of the triumphs of contemporary science.  They are interpreted by cosmologists to indicate that matter filling the early universe was resonant, much like the head of a drum of the body of a flute.  The wavelength at which a musical instrument vibrates is proportional to its size, and the same is true of the universe.  The wavelengths of the resonant modes tell us how big the universe was when it first became transparent: that is, when the initial hot plasma developed, or 'de-coupled'. into separate realms of matter and energy some three hundred thousand years after the Big Bang, at which time the microwave background became visible.  These observations are extremely helpful in tying down the parameters of our cosmological models.

Another feature we see in the data is that there is very little energy in the largest wavelength.  This may be just a statistical fluctuation, because it involves a small number of pieces of data.  But if it is not a statistical fluke, it can be interpreted as indicating a cut-off, above which the modes are much less excited.  It is interesting that this cutoff is at the scale of R, associated with the cosmological constant.

The existence of such a cutoff would be puzzling from the point of view of the most widely accepted theory of the very early universe, which is 'inflation'.  According to the theory of inflation, the universe expanded exponentially fast during one extremely early period.  Inflation accounts for the observation that the cosmic background radiation is so nearly uniform.  It does this by ensuring that all parts of the universe we see now could have been in causal contact when the universe was still a plasma..

The theory also predicts the fluctuations in the cosmic microwave background, which are hypothesized to be remnants of quantum effects during the period of inflation.  The uncertainty principle implies that the fields dominating the energy of the universe during inflation fluctuate, and these fluctuations become imprinted on the geometry of space.  As the universe expand exponentially, they persist, causing fluctuations in the temperature of the radiation produced when the universe becomes transparent.

Inflation is believed to have produced a huge region of the universe with relatively uniform properties.  This region of the universe is thought to be many orders of magnitude larger than the observable region, because of a simple argument about scales.  If inflation had stopped just at the point where it created a region as large as we now observe, there must have been some parameter in the physics of inflation that selected a special time to stop, which just happens to be our era.  But this seems improbable, because inflation took place when the universe had a temperature ten to twenty orders of magnitude greater than the center of the hottest star today; thus the laws governing it must have been different laws, which dominate physics only in those extreme conditions.  There are many hypotheses about the laws that govern inflation, and none of them say anything about a time scale of 10 billion years.  Another way to put this is that there seems no way for the present value of the cosmological constant to have anything to do with the physics that caused inflation.

Thus if inflation produced a uniform universe on the scale that we observe, it likely produced a universe that is uniform on much larger scales.  This in turn implies that the pattern of fluctuations produced by inflation should go on and on, no matter how far you look.  If you could see beyond the present size of the observable universe, you should continue to see small fluctuations in the cosmic microwave background.  Instead, the data hint that the fluctuations may cease above the scale of R.

Indeed, as cosmologists have examined the large-scale modes in the microwave background, they have found more mysteries.  It's an item of faith among cosmologists that at the largest scales the universe should be symmetric - that is, any one direction should be like any other.  This is not what is seen.  The radiation in these large-scale modes is not symmetric; there is a preferred direction. (It has been called "axis of evil" by the cosmologists Kate Land and Joao Magueijo).  No one has any rational explanation for this effect.

These observations are controversial because they disagree profoundly with what we would expect on the basis of inflation.  Since inflation explains so much of cosmology, many prudent scientists suspect that there is something wrong with the microwave data.  Indeed , it is always possible that the measurements are just wrong.  A lot of delicate analysis is applied to the data before they're presented.  One thing that's done is to subtract the radiation known to come from the galaxy that we live in.  This may have been done incorrectly, but few experts familiar with the details of how the data are analyzed believe that to be the case.  Another possibility, as noted, is that our observations are just statistical anomalies.  An oscillation at a wavelength of the scale R takes up a huge part of the sky - about 60 degrees; consequently we see only a few wavelengths, and there are only a few pieces of data, so what we are seeing may just be a random statistical fluctuation.  The chances of the evidence for a preferred direction being a statistical anomaly have been estimated at less than 1 part in 1000.  But is may be easier to believe in this unlikely bad luck than to believe that the predictions of inflation are breaking down.

These issues are currently unresolved.  For the time being, it is enough to say that we went looking for strange physics on the scale of R and found it.

My contracting cyclic model gives physical cause and effect mechanics for it's Big Bangs and Inflation periods, and this data of a preferred direction will correspond to those mechanics.
Given that anyone can understand that a 'contracting' universe will eventually result in all the mass of the universe being in the same location, i.e. a super-massive black hole, and that a super-massive back hole with no counterpart gravitational force acting upon it will explode it's plasma in an outward trajectory in an inflation like manner, I will continue onto the next few pages of Lee Smolin's book concerning the R scale with respect MOND.

"The Trouble With Physics" - Chapter 13: Surprises from the Real World - page 208, 209
Quote
:Lee Smolin
These issues are currently unresolved.  For the time being, it is enough to say that we went looking for strange physics on the scale of R and found it...

Are there any other phenomenon associated with this scale?  We can combine R with other constants of nature to see what happens at scales defined by the resulting number.  Let me give an example.  Consider R divided by the speed of light: R/c.  This gives us a time, and the time given is roughly the present age of our universe.  The inverse, c/R, gives us a frequency - a very low note, one oscillation per lifetime of the universe.

The next simplest thing to try is c^2/R.  This turns out to be an acceleration.  It is in fact the acceleration by which the rate of the expansion of the universe is increasing - that is, the acceleration produced by the cosmological constant...

In my contracting model this 'acceleration' is the rate at which the universe is accelerated in its contraction.
(This does away with the necessity for Dark Energy - gravity is causing contraction.)

Quote
: Lee Smolin - page 209 continued...
...Compared to ordinary scales, however, it is a very tiny acceleration: 10-8 centimeters per second.  Imagine a bug crawling across the floor.  It manages to go perhaps 10 centimeters per second.  If the bug doubled it's speed over the lifetime of a dog, it would be accelerating as much as c^2/R, a very small acceleration indeed.

But suppose there is a new universal phenomenon that explains the value of the cosmological constant.  Just by the fact that the scales match, this new phenomenon should also affect any other kind of motion with an acceleration this tiny.  So anytime we can observe something moving with such tiny acceleration, we would expect to see something new.  Now the game starts getting interesting.  We do know things that accelerate this slowly.  One example is a typical star orbiting in a typical galaxy.  A galaxy orbiting another galaxy accelerates even more slowly.  So, do we see anything different about the orbits of stars with accelerations this tiny, compared to the orbits of stars with larger accelerations?  The answer is yes, we do, and dramatically so.  This is the problem of Dark Matter.

If one considers that the cosmological constant (that matches with c^2/R) is describing the acceleration of a gravitational contraction of the universe, one will find that Dark Matter is also unnecessary - more on this next post.
Title: Re: Is there a discrepancy with the equivalence principle?
Post by: timey on 26/06/2017 14:16:48
"The Trouble With Physics"  Chapter 13: Surprises from the Real World - pages 209, 210, 211
Quote
:Lee Smolin continued...
So, do we see anything different about the orbits of stars with larger accelerations?  The answer is yes, we do, and dramatically so.  This is the problem of dark matter...

As we discussed in chapter 1, astronomers discovered the dark matter problem by measuring the acceleration of stars in orbit about the center of their galaxies.  The problem arose because, given the measured accelerations, astronomers could deduce the distribution of the galaxy's matter.  In most galaxies, this result turned out to disagree with the matter observed directly.

I can now say a bit more about where the discrepancy arises. (For the sake of simplicity, I'll restrict the discussion to spiral galaxies, in which most stars move in circular orbits in a disk.)  In each galaxy where the problem is found, it affects only stars moving outside a certain orbit.  Within that orbit, there's no problem - the acceleration is what it should be if caused by visible matter.  So there seems to be a region in the interior of the galaxy within which Newton's laws work and there is no need for dark matter.  Outside this region, things get messy.

The key question is:  Where is the special orbit that separates the two regions?  We might suppose that it occurs at a particular distance from the center of the galaxy.  This is a natural hypothesis, but it is wrong.  Is the dividing line at a certain density of stars or starlight?  Again, the answer is no.  What seems to determine the dividing line, surprisingly, is the rate of acceleration itself.  As one moves further out from the center of the galaxy, accelerations decrease, and there turns out to be a critical rate that marks the breakdown of Newton'slaw of gravity.  As long as the acceleration of the star exceeds this critical value, Newton's law seems to work and the acceleration predicted is the one seen.  There is no need to posit any dark matter in these cases.  But when the acceleration observed is smaller than the critical value, it no longer agrees with the predictions of Newton's law.

What is this special acceleration?  It has been measured to be 1.2x10-8 centimeters per second per second.  This is close to c^2/R, the value of the acceleration of the cosmological constant!

This remarkable twist in the dark matter story was discovered by an Israeli physicist named Mordelhai Milgrom in the early 1980's.  He published his findings in 1983, but for many years they were largely ignored.  As the data have gotten better, however, it has become clear that his observation was correct.  The scale c^2/R characterizes where Newton's law breaks down for galaxies.  This is now called Milgrom's law by astronomers.

I want you to understand how weird this observation is.  The scale R is the the scale of the whole universe, which is enormously bigger than any individual galaxy.  The acceleration c^2/R occurs on this cosmological scale; as noted, it is the rate at which the universe's expansion accelerates.  There is no obvious reason for this scale to play any role in the dynamics of an individual galaxy.  The realization was forced upon us by the data.  I recall my amazement when I first learned about it.  I was shocked and energized.  I walked around for an hour in a daze, muttering incoherent obscenities.  Finally!  A possible hint from experiment that there is more to the world than we theorists imagine!

How is this to be explained?  Apart from coincidence, there are three possibilities.  There could be dark matter, and the scale of c^2/R could characterize the physics of the dark matter particles.  Or the dark matter halos could be characterized by the scale of c^2/R, because that is related to te density of dark matter at the time they collapsed to form galaxies.  In either case, the dark energy and dark matter are distinct phenomenon, but related.

The other possibility is that there is no dark matter and Newton's law of gravity breaks down whenever accelerations get as small as the special value of c^2/R.  In this case there needs to be a new law that replaces Newton's law in these circumstances.  In his 1983 paper, Milgrom proposed such a theory.  He called it MOND for "modified Newtonian dynamics."  According to Newton's law of gravity, the acceleration of a body due to a mass decreases in a specific way when you move away from that mass - that is, by the square of the distance.  Milgrom's theory says that Newton's law holds, but only until the acceleration decreases to the magic value of 1,2x10-8cm/sec^2.  After that point, rather than decreasing with the square of the distance, it decreases only by distance.  Moreover, while normally the Newtonian force is proportional to the mass of the body causing the acceleration times a constant (which is Newton's gravitational constant), MOND says that when the acceleration is very small, the force is proportional to the square root of the mass times Newton's constant.

If Milgrom is right, then the reason that the stars outside the special orbit are accelerating more than they should be is that they are feeling a stronger gravitational force than Newton predicted!  Here is brand-new physics - not at the Planck scale, and not even in an accelerator, but right in front of us, in the motions of the stars we see in the sky.

As a theory, MOND does not make much sense to physicists.  There are good reasons why the gravitational and electrical forces fall off with the square of the distance.  It turns out to be a consequence of relativity combined with the three dimensional nature of space.  I won't go into details here, but the conclusion is drastic.  Milgrom's theory appears inconsistent with basic physical principles, including those of special and general relativity.

I have explained in posts above how my theory makes adjustments to both special and general relativity, but to re-cap:
My theory adds a 3rd aspect to the time dilation phenomenon for where m=0, i.e. open space.
This renders the time dilation's of special and general relativity as the experience of the mass 'only', where changes in frequency are the indication of changes in the rate of time, or more specifically, the timing of that mass.
The 'timing itself' of any mass travelling 'in' the 3rd aspect time dilated open space will not be directly affected by the 3rd aspect time dilation, but the mass will be indirectly affected because the motion of the mass travelling through the time dilated space 'will' be affected as being accelerated towards the greater M, or decelerated away from the greater M, due to changes in the rate of this 3rd aspect time dilation.
So - where Einstein has shown that gravity and acceleration are the same thing, my theory says that acceleration and this 3rd aspect time dilation of open space are the same thing.

The interesting thing about Milgrom's theory MOND is that what we see is a transference of proportionality:
Quote
Milgrom's theory says that Newton's law holds, but only until the acceleration decreases to the magic value of 1,2x10-8cm/sec^2.  After that point, rather than decreasing with the square of the distance, it decreases only by distance.  Moreover, while normally the Newtonian force is proportional to the mass of the body causing the acceleration times a constant (which is Newton's gravitational constant), MOND says that when the acceleration is very small, the force is proportional to the square root of the mass times Newton's constant.
That which is normally proportional to the square of the distance is now proportional to distance only...
That which is normally proportional to the mass times Newton's constant is now proportional to the square root of the mass times Newton's constant.
 
I have explained in previous posts how my model describes causality for both the attraction of gravity (mass internal GR&SR timing), and the acceleration of gravity (3rd aspect time dilation of open space), but I will put this description into the context of galaxies and 'potentially' Milgrom's theory next post...
Title: Re: Is there a discrepancy with the equivalence principle?
Post by: timey on 30/06/2017 13:42:19
Firstly to make a reminder:
In adding this 3rd aspect to the time dilation phenomenon for where m=0, i.e: open space, to give a physical cause and effect description for the phenomenon of gravitational acceleration, my model has re-interpreted the red shift distance correlation, i.e. Hubble's flow, as being caused by Doppler shifts in 'time'.  And that where peculiar velocity deviates from Hubble's flow, this is also caused by Doppler shifts in 'time'. Where the 'time' being shifted is this added 3rd aspect to the time dilation phenomenon for where m=0, i.e: open space, and the 'shifting' of this time is caused by the changing strength of the gravitational field between masses, and strength of gravitational field is vacuum energy related where +energy=shorter seconds...

+energy=shorter seconds being the principle theory of my model...

This +energy=shorter seconds remit also applies to the frequency changes that we observe of a clock under the remit of both GR and SR frameworks of time dilation, and applies where I have said this in last post:
Quote
I have explained in previous posts how my model describes causality for both the attraction of gravity (mass internal GR&SR timing)
...and in adding this 3rd aspect to the time dilation phenomenon, this applies where I have finished the above sentence with:
Quote
and the acceleration of gravity (3rd aspect time dilation of open space)

It is important to note that my model makes a clear cut distinction between the internal timing of mass (GR and SR time dilation's) and the external timing (3rd aspect time dilation) of the space that mass moves through.  It is only by viewing the phenomenon of time in this 3 dimensional fashion that what I'm going to say about Milgrom's theory MOND in relation to DSR (doubly special relativity) will make any geometrical sense.

To go back to a post I made earlier this thread:
Quote
:post 49
If you travel one metre at a constant speed that is held relative to a longer or shorter 'variable' second.  Then the distance remains the same, and it just takes a longer or shorter amount of 'time' to travel that metre.

This is the basis of my model's temporal interpretation of non-Euclidean space.  In order to understand what I am going to say about MOND and DSR I need you to consider that a temporal interpretation of non-Euclidean space, (or noncommutative geometry-quantum) is emergent of 3 dimensions of Euclidean spacial geometry, and that this is achievable via a simple mathematical juggle with the speed distance time formula.

Now I can move onto describing galaxies under the remit of my model - which I will do next post.
Title: Re: Is there a discrepancy with the equivalence principle?
Post by: timey on 06/01/2018 11:35:40
I'm happy to say that it turns out that I managed to learn enough maths to describe my idea of a 3rd time dilation...
https://m.youtube.com/watch?t=4s&v=BBND2wuf7Bg
Title: Re: Is there a discrepancy with the equivalence principle?
Post by: guest4091 on 07/01/2018 19:22:55
L wondered where you went. You made things interesting and motivated us to think more deeply about stuff.
Have you put your ideas in a paper that can be shared?
Title: Re: Is there a discrepancy with the equivalence principle?
Post by: timey on 16/01/2018 00:22:46
Thank you so much for your kind words Phyti!
No I hadn't written a paper, but your post totally inspired me to do so.  However I am having issues submitting my paper.

I need to be endorsed by people within the community before I can submit, and as a self taught who is an outsider to physics, finding endorsers is difficult.

This is the paper that I wish to submit.

https://www.pdf.investintech.com/preview/3b253818-fa52-11e7-b174-0cc47a792c0a/index.html

I have posted the email they sent regarding endorsement in the hope that someone reading may be able to help...

help@arxiv.org
11:40 PM (21 minutes ago)

to me
(Vikki Ramsay should forward this email to someone who's registered as
an endorser for the gr-qc (General Relativity and Quantum Cosmology)
archive of arXiv.)

Vikki Ramsay requests your endorsement to submit an article to the gr-qc
section of arXiv. To tell us that you would (or would not) like to
endorse this person, please visit the following URL:

https://arxiv.org/auth/endorse?x=NTSKAM

If that URL does not work for you, please visit

http://arxiv.org/auth/endorse.php

and enter the following six-digit alphanumeric string:

Endorsement Code: NTSKAM
Title: Re: Is there a discrepancy with the equivalence principle?
Post by: jeffreyH on 17/01/2018 14:42:35
I was looking for the maths. I didn't find any!
Title: Re: Is there a discrepancy with the equivalence principle?
Post by: timey on 17/01/2018 17:03:35
This link to my mathematical description of the proposed spacetime structure is provided in the paper.

The mathematics for other proposed inverse type functions can be found in the author's papers via the links provided.
Title: Re: Is there a discrepancy with the equivalence principle?
Post by: timey on 17/01/2018 18:08:53
Actually Jeff to say so, thanks for pointing that out.  Some of the links do not work via clicking on the paper itself I notice. Probably due to the PDF hoster, b/c if you then copy and paste the link into the search browser, it works fine.
Very annoying, I'll have to look into it.
Title: Re: Is there a discrepancy with the equivalence principle?
Post by: jeffreyH on 17/01/2018 19:20:14
Then it isn't your maths but other people's. Where is your maths?
Title: Re: Is there a discrepancy with the equivalence principle?
Post by: timey on 17/01/2018 20:27:01
Jeff, The YouTube video that I have posted in post 804 is my lecture containing my maths.  Here it is again...



---------------------------------------------------------------------------------------------------------------------------------------

This is the paper that I wish to submit:
https://www.pdf.investintech.com/preview/3b253818-fa52-11e7-b174-0cc47a792c0a/index.html
(If the links do not work by clicking on the paper, then they will if you copy and paste them to your browser)

I need to be endorsed by people within the community before I can submit my paper for peer review and publishing.  But as a self taught who is an outsider to physics, finding endorsers is difficult.

This is the email they sent me:
"
help@arxiv.org
11:40 PM (21 minutes ago)

to me
(Vikki Ramsay should forward this email to someone who's registered as an endorser for the gr-qc (General Relativity and Quantum Cosmology)
archive of arXiv.)

Vikki Ramsay requests your endorsement to submit an article to the gr-qc
section of arXiv. To tell us that you would (or would not) like to endorse this person, please visit the following URL:

https://arxiv.org/auth/endorse?x=NTSKAM

If that URL does not work for you, please visit

http://arxiv.org/auth/endorse.php

and enter the following six-digit alphanumeric string:

Endorsement Code: NTSKAM
"

I am posting this here in the hope that someone reading may be able to help me, or may know someone who might...
Title: Re: Is there a discrepancy with the equivalence principle?
Post by: Colin2B on 18/01/2018 17:34:44
This link to my mathematical description of the proposed spacetime structure is provided in the paper.
I think @jeff was expecting to find the maths in the paper, which is the usual format, even if it is a graphical presentation of the maths.

In order to improve your chances of getting an endorsement, it would be worth looking at some of the papers on arxiv and using a similar layout, putting all references at the end and including your main arguments in the discussion section and a conclusions summary.

I was going to comment on the links problem but looks as though you are going to sort that.

A comment on page 6 where you write:
“placing 2 clocks in differing locations that share the same longitude (to equalize centripetal motion) and the same height from centre of earth (to equalize position in gravity field), but in locations of known significant difference in geological density”
You need to explain more about what you mean by “equalising the position in the gravity field” -  placing the clocks equal distance from the centre of earth doesn’t do that. The relationship between distance from earth centre and gravitational potential assumes an even distribution of mass, any local density will cause a variation of gravitational potential and the field gradient vector.
If near this “location of ... significant difference in ... density” the clock ticks faster then this implies that relative clock rate is not directly dependant on relative gravitational potential. This is a major conclusion and dwarfs any bounce theory, as such it should be clearly stated in the abstract and conclusions and the main body should make it clear why this occurs.

I wouldn’t bother showing examples of use of inverse functions. Anyone familiar with this area will understand how they are used, but will want to see you explain why and how you have used them in this particular case - this should be in main body.

Do you have a newer version that you have proof read? There are a number of typos etc.

Hope that’s helpful.
Title: Re: Is there a discrepancy with the equivalence principle?
Post by: timey on 18/01/2018 18:23:08
Yes that is really helpful! (I'm so glad you cannot see the emotion registering on my face at this moment)

But this being quite important, just to check, is it your advice to use the actual workingout sheets used in my YouTube lecture and the descriptions thereof in short?  And then only mention other inverse functions in passing? (noting references at end).

...and yes, it is exactly the fact that "any local density will cause a variation of gravity potential and the field gradient vector" that is the basis for a further testing of gereral relativity to gain experimental evidence that a greater density of mass will cause a slower rate of time as Einstein's general relativity predicted, but your comments about making this clearer are taken on board.

And then maybe ditch section 3 altogether as an examination of bounce models, and just present my bounce model, (putting references at the end), and then split the considerations in the statement of predictiion into a conclusions section with the statement of prediction following as a simpe statement at end perhaps?

Typo's: My son did the spell check while converting my file to PDF for me, (I am on his old broken laptop that doesn't function well), he clearly did it in a hurry, I'll tell him :) 
Title: Re: Is there a discrepancy with the equivalence principle?
Post by: Colin2B on 19/01/2018 15:49:58
But this being quite important, just to check, is it your advice to use the actual workingout sheets used in my YouTube lecture and the descriptions thereof in short? 
Yes, endorsers are busy people who dont want to sit through a lecture but will skim a paper skipping what is easily understood. You could include diagrams with enough description to explain what the methodology is.

And then only mention other inverse functions in passing? (noting references at end).
That depends. There is no point telling these people about the general principle or examples of use of inverse functions (they will know that), however, if someone has used inverse function in the same way as yourself and the method and conclusions support your case then that is relevant. What you need to do however is not just quote the link, you need to extract an explanation of the specific points as a summary then provide a reference number to the list of papers and links at the end.


...and yes, it is exactly the fact that "any local density will cause a variation of gravity potential and the field gradient vector" that is the basis for a further testing of gereral relativity to gain experimental evidence that a greater density of mass will cause a slower rate of time as Einstein's general relativity predicted, but your comments about making this clearer are taken on board.
Sorry, my post contained a typo due to typing quick notes, it should of course be “potential gradient vector”.or more correctly “the gradient vector field of the gravitational potential”.
As you know, current experiments show that lines of gravitational equipotential change in distance from earth centre in areas of higher density. That change is such that the line is further from earth centre, so relative to an area the same distance from earth centre, but over a lower density we would expect the clocks to run slower eg a clock over an iron deposit will tick slower relative to a clock over a hollow cave. We know that field and equipotential lines do not cross, so if your methodology predicts the opposite clock rate change then clearly clock rates do not vary with gravitational potential and you need to explain why this is so, and what you believe the true description should be.

And then maybe ditch section 3 altogether as an examination of bounce models, and just present my bounce model, (putting references at the end),
You could keep this as a discussion, either before your method or as an annex. However, you tend to just refer to these papers and leave the reader to go away and extract relevant points. It would be better if you summarised the key findings for each paper and again number in text to point to reference section. You need to make it easy for the reader to grasp the key points of your argument without having to do the leg work themselves. Think busy people, make it hard they will put it to one side for later ie never.

Title: Re: Is there a discrepancy with the equivalence principle?
Post by: timey on 19/01/2018 23:02:18
Thanks for your advice.  I have taken on board all your comments, and am on it for a re-write.

I think I can include pics of my worksheets and adequately describe in writing w/o getting too long, all of my video up to and including page 5, perhaps utilising some highlighting system to differentiate which bits I'm writing about. Same with pages 8, 9, 10, & 11, although these will be more difficult and long winded.

...But in page 6 I outline a consideration concerning wavelength at the R scale, and a suggested mathematical manipulation that is based on the speed/distance/time formula, that is left open without finalising the notation.  As you know I am quite green with the maths.

So what I was wondering Colin, if you have the time. and the inclination of course, is if you might be able to advise me as to the means to notate that consideration properly?
This would cut a lot of text explanation from the paper if that was possible.
Page 6 starts at time 46.38, and lasts for a duration of 5 mins.

Thanks again for the advice!
Title: Re: Is there a discrepancy with the equivalence principle?
Post by: Colin2B on 19/01/2018 23:33:54
[quote author=timey link=topic=69800.msg531508#msg531508 date=
So what I was wondering Colin, if you have the time. and the inclination of course, is if you might be able to advise me as to the means to notate that consideration properly?
[/quote]
I’m not going to have a lot of spare time for a while, we’ve had a close family death and I have a lot of arrangements to see to. When i surface I’ll have a look.
Title: Re: Is there a discrepancy with the equivalence principle?
Post by: timey on 19/01/2018 23:59:06
Sorry for your loss Colin...
I'll look out for your return.
Title: Re: Is there a discrepancy with the equivalence principle?
Post by: jeffreyH on 20/01/2018 14:51:45
I have now watched portions of the video. You complicate things in your posts with undecipherable language. The video made more sense. Not a lot of people will get what you are implying. I do. You need to clarify your ideas somewhat and make your language very precise. Carry on. You have a very profound idea.
Title: Re: Is there a discrepancy with the equivalence principle?
Post by: timey on 21/01/2018 02:28:17
That you have said these words means a great deal to me Jeff.  Thank you.

And yes, my lack of adept usage regarding terminology is a problem (not just in physics) and my spelling is also atrocious.
I would be very happy if you would care to add any additional comment to Colin's regarding my re-write when I post it.
Title: Re: Is there a discrepancy with the equivalence principle?
Post by: jeffreyH on 21/01/2018 15:03:18
To follow up on your Navier Stokes remarks.
https://www.comsol.com/multiphysics/navier-stokes-equations
Pipe flow is one of the things I worked on in hydrology. I could try to find some of the papers relating to Navier Stokes and give you my opinions.
Title: Re: Is there a discrepancy with the equivalence principle?
Post by: Colin2B on 21/01/2018 22:46:58
I agree with Jeff, easier to see your ideas developing. I’ll try to watch video in odd moments, but it will be slow.
@jeffreyH  I’ve got to page 2 and i can see what @timey  is trying to do, but i don’t think partial derivative is best way to do vertical axis. That would give a gradient with +ve & -ve values which means -ve axis. I think a better representation would be a function eg td3=f(tu,xu) where subscript d3 is dilation 3, and sub u is undilated t&x ie baseline. What do you think?
Also, need to think of term for what @timey calls a vector, really a locus or plot.
Title: Re: Is there a discrepancy with the equivalence principle?
Post by: jeffreyH on 22/01/2018 18:52:49
I wouldn't get into the technicalities of the maths yet. The best way to look at it is via a perfectly spherical mass with a perfectly spherical cavity at its centre. The will be a Lagrange point at the centre of mass which coincides with the centre of the cavity. If a smaller sphere rests at this point we can rotate the system around any axis that passes through the centre of mass and the symmetry will be maintained. Any movement away from the centre will move the object to the nearest interior surface of the cavity in a straight line geodesic. This then breaks the symmetry. Much like the symmetry breaking of the Higgs field. Although here the object acquires weight and not mass. So it is an analogy. The important point is that all parts of the field are directed inwards and focussed at the centre. The object has to gain potential energy to reach the Lagrange point. The value of this energy is the same no matter where on the inner cavity surface it starts from. This is an idealised system. The real universe is far less straight forward.
Title: Re: Is there a discrepancy with the equivalence principle?
Post by: Colin2B on 22/01/2018 22:26:21
@jeffreyH You are obviously much further on in the video than i am. I see what you are thinking, do you consider distant mass of universe to be the outer shell?
My previous post was just some jottings of things that might cause a reader to reject the paper early on without reading through, rather than maths. Ill keep these and similar in a notes file for later editing .
Title: Re: Is there a discrepancy with the equivalence principle?
Post by: jeffreyH on 22/01/2018 22:47:25
It is not so simple as having the distant mass being the outer shell. You are talking about miniscule contributions from each source. If the universe contains infinite mass then you are talking about an infinite series whose limit would need to be determined. The limit cannot be infinity. Since then gravity would have infinite strength everywhere. This has a bearing on renormalisation and determinations of vacuum energy.
Title: Re: Is there a discrepancy with the equivalence principle?
Post by: timey on 23/01/2018 16:51:57
I removed my last post and replaced it with the revised version.

And have now replaced the below text again with edit 2

---------------------------------------------------------------------------------------------------------------------------------------

Maybe it might help if I give a quick breakdown of the video...

Page 1 through to 5 illustrate how the geometric curvature of spacetime can be attributed to an additional temporal phenomenon, and how by adding this temporal phenomenon, this constitutes a renormalisation term to the GR distortion of the metric, resulting in Euclidean geometry.
 
Page 1 introduces the diagram that I use.  This structure of diagram, that conforms to pythagaros, is a representation of the speed/distance/time formula.
Time is on the y axis, distance is on the x axis, and the line (which I call a vector) is the speed, where a straight line describes a constant speed, and a curved line can decribe an acceleration or a deceleration.
(that x is marked as equal to constant metres becomes relevant when x is not held equal to constant metres later on)

Page 2 demonstrates that by saying that seconds get longer proportional to distance, or that seconds get shorter proportional to distance, (clearly by a specific rate) that an acceleration or deceleration in the speed of an objects vector can be represented as a straight line.

Page 3 demonstrates that seconds getting longer or shorter proportional to distance are 2 sides (partial dirivitives) of the same consideration, this being changes in time dilation 3 (dt3), and that these changes in time can be held relative to a partial dirivitive of changes in acceleration. (partial b/c there are other changes in acceleration to be considered, dirivitive b/c all the considerations added together are then the 'changes' in acceleration (da) )

The diagram of a quarter sphere + equatorial bulge is quite simply ruminating the fact that the rate that clocks tick at here on Earth, and on other spinning masses, is a combination of both position in the gravity potential, and centripetal motion.
And the fact that clocks tick at the same rate at sea level at any elevation of the equatorial bulge where a lesser centripetal speed causes a speeding up of time, and a lesser altitude from centre of earth causes a slowing of time, means that the speeding up effects on time due to lesser motion directly cancel out the slowing down effects due to the lesser altitude, and visa versa for the inverse.
And that clocks that are raised in altitude from sea level earth will tick a fraction faster as a combination of both centripetal motion and postition in gravity potential that is not quite cancelled out.
Having introduced an additional time phenomenon and it's changes as (dt3) in pages 2 and 3, I now state that changes in time due to motion will be refered to as (dt1), and that changes in time due to position in the gravity potential will be referred to as (dt2).

Page 4a repeats the introduction of (dt3) from page 3, and then introduces (dt2) as an identical but inversed consideration, where the changes in rate of time are occuring for the clock that is on that vector travelling the distance on x.  This diagram is saying that an accelerated clock rate will cause an object's (the clock's in this case) vector in the 'up' direction to be decelerated , and that a decelerated clock rate will cause an object's vector in the 'down' direction to be decelerated.
(there is more on this distinction between a clock held in position to the earth and a clock in free motion 'up' and 'down' with respect to the earths motion, in detail, in pages 9 & 10.)

Page 4b inroduces the changes in time due to centripetal* motion as (dt1) where the changes in time are occurring inversely to the changes in time dilation 2 (dt2)
Now there are 3 partial dirivitives of acceleration.
Partial dirivitives of acceleration (b) being equal to (dt2) & (c) being equal to (dt1) are for where m doesn't equal zero.
Partial dirivitive of acceleration (a) being equal to (dt3) is for where m does equal zero.

Added together these partial dirivitives of acceleration becomes 'changes' in acceleration (da), where (da) are held proportional to changes in gravity potential.
(*other types of motion are considered in pages 9 & 10)

And the changes in time dilation 1 (dt1) + the changes in time dilation 2 (dt2) in the gravity potential are equal but inverse to the changes in time dilation 3 (dt3).
(edit question: Is that inversely proportional?)
Where the changes in time of a clock in the gravity potential are those observed by the observer who we presume to be making the calculations.

Page 4b then starts introducing the mechanics of what occurs when changes in x (dx) are implemented.

---------------------------------------------------------------------------------------------------------------------------------------

I'll continue shortly in more detail about (dx) and what is going on in page 5.  I'm basically practicing for the paper, so if my use of the terminology 'vector', 'partial dirivitive', etc is at fault?

I have 'again' replaced the text with an editied version.

(main edit insert being that (dt1) + (dt2) are equal and oposite to (dt3)
(edit question: Is that inversely proportional?)

(edit 2 insert being a reference to the relativistic nature of the observation of clocks ticking at differring rates in relation to the addition of (dt3), and that all considerations are formed from the observers reference frame.
Title: Re: Is there a discrepancy with the equivalence principle?
Post by: Colin2B on 23/01/2018 17:26:45
@timey  “Maybe it might help if I give a quick breakdown of the video...”
Devil is always in the detail. From your summary I wouldn’t have noticed the terminology issues.
I need to go through in detail and ill keep a list of items worth changing.
Title: Re: Is there a discrepancy with the equivalence principle?
Post by: timey on 24/01/2018 15:55:39
Ok - this is the 3rd edit of the captions for pages up to 4b.  I think I have finally included all of the relevant points in order to go on and describe the rest of page 4b and move onto page 5.  I think that some of these descriptions can be condensed a little, but I am worried that in cutting explanation in order to be less long and potentially tedious, that this is at the risk of under-explaining and being potentially misunderstood.
--------------------------------------------------------------------------------------------------
Page 1 through to 5 illustrate how the geometric curvature of spacetime can be attributed to an additional temporal phenomenon, and how by adding this temporal phenomenon, this constitutes a renormalisation term to the GR distortion of the metric, resulting in Euclidean geometry.
 
Page 1 introduces the diagram that I use.  This structure of diagram, that conforms to pythagaros, is a representation of the speed/distance/time formula.
Time is on the y axis, distance is on the x axis, and the line (which I call a vector) is the speed, where a straight line describes a constant speed, and a curved line can decribe an acceleration or a deceleration.
(that x is marked as equal to constant metres becomes relevant when x is not held equal to constant metres later on)

Page 2 demonstrates that by saying that seconds get longer proportional to distance, or that seconds get shorter proportional to distance, (clearly by a specific rate) that an acceleration or deceleration in the speed of an objects vector can be represented as a straight line.

Page 3 demonstrates that seconds getting longer or shorter proportional to distance are 2 sides (partial dirivitives) of the same consideration, this being changes in time dilation 3 (dt3), and that these changes in time can be held relative to a partial dirivitive of changes in acceleration. (partial b/c there are other changes in acceleration to be considered, dirivitive b/c all the considerations added together are then the 'changes' in acceleration (da) )

The diagram of a quarter sphere + equatorial bulge is quite simply ruminating the fact that the rate that clocks tick at here on Earth, and on other spinning masses, is a combination of both position in the gravity potential, and centripetal motion.
And the fact that clocks tick at the same rate at sea level at any elevation of the equatorial bulge where a lesser centripetal speed causes a speeding up of time, and a lesser altitude from centre of earth causes a slowing of time, means that the speeding up effects on time due to lesser motion directly cancel out the slowing down effects on time due to the lesser altitude, and visa versa for the inverse.
And that clocks that are raised in altitude from sea level earth will tick a fraction faster as a combination of both centripetal motion and postition in gravity potential that is not quite cancelled out.
Having introduced an additional time phenomenon and it's changes as (dt3) in pages 2 and 3, I now state that changes in time due to motion will be refered to as (dt1), and that changes in time due to position in the gravity potential will be referred to as (dt2).

Page 4a repeats the introduction of (dt3) from page 3, and then introduces (dt2) as an identical but inversed consideration, where the changes in rate of time are occuring for the clock that is on that vector travelling the distance on x.  This diagram is saying that an accelerated clock rate will cause an object's (the clock's in this case) vector in the 'up' direction to be decelerated , and that a decelerated clock rate will cause an object's vector in the 'down' direction to be decelerated.
(there is more on this distinction between a clock held in position to the earth and a clock in free motion 'up' and 'down' with respect to the earths motion, in detail, in pages 9 & 10.)

Page 4b introduces the changes in time due to centripetal* motion in the gravity potential as (dt1), where the changes in time dilation 1(dt1) are occurring inversely to the changes in time dilation 2 (dt2)
(*other types of motion are considered in pages 9 & 10)

Now there are 3 partial dirivitives of acceleration.
Partial dirivitives of acceleration (b), (being equal to (dt2)), and (c), (being equal to (dt1)), are for where m doesn't equal zero.
Partial dirivitive of acceleration (a), (being equal to (dt3)), is for where m does equal zero.
(Note that none of these demonstrations of curve manipultation are representing any true values yet)

Added together, these partial dirivitives of acceleration become 'changes' in acceleration (da), where (da) are held proportional to changes in gravity potential.

And the changes in time dilation 1 (dt1) + the changes in time dilation 2 (dt2) in the gravity potential are equal but inverse to the changes in time dilation 3 (dt3).
(edit question: Is that inversely proportional?)
Where the changes in time of a clock in the gravity potential are those observed by the observer who we presume to be making the calculations.
(Note that no actual value has been attributed to the magnitude of: a 'change' in time, a 'change' in acceleration, or a description of within what distance in the gravity potential a 'change' occurs, as of yet)

Page 4b then starts introducing the mechanics of what occurs when changes in x (dx) are implemented.
Title: Re: Is there a discrepancy with the equivalence principle?
Post by: jeffreyH on 25/01/2018 12:49:59
Just a note on the spherical shell cavity. If we place a detector at the Lagrange point then any photon fired from the interior surface will be red shifted by the time it reaches the detector. If the system were large enough then the central mass could be a celestial object such as a planet. Then the field of the central body would be applying blue shift to the incoming photon. There would be two opposite time dilation effects.
Title: Re: Is there a discrepancy with the equivalence principle?
Post by: timey on 25/01/2018 18:30:07
Yes - a photon is particle m=0, moving through space m=0.  And it is possible to say that a change in the length of wavelength for both gravitational redshifted light, and gravitational blueshifted light, can be attributed to a change in the rate of time in the gravity field of space, rather than being attributed to any actual spacial change in the wavelength, and that these two opposites 'could' be regarded as partial dirivitives, or 'up'/'down' dirivitives of an additional time dilation phenomenon for where m=0.

Things to consider:

The detector (in your post) measuring the magnitude of the redshift, will be measuring that redshift phenomenon with a clock that is ticking faster than the clock on the mass that the photon was measured with when it was released from that position on that mass.
There is a school of thought that will say that the light has not changed frequency at-all, and that it is just the fact that the detector's time is running faster at that position than it would on the ground, that is the cause of the measurement of a longer wavelength.
(For anyone wishing to refer to the rules of relativistic observation, the 'far away' clock will agree that the ground clock is ticking slower than the elevated clock)

However, the possibiity exists that we can attibute half of the detector's measurement of a longer wavelength to the fact that the detector's clock is running faster, and we can attribute the other half of the detector's measurement of the longer wavelength to this additional time dilation for where m=0, where it is taking light moving into the weaker gravity fields longer amounts of time to travel the same unit distance, 'causing' a lower frequency.
(A blue shift towards gravitational M can be the above consideration inversed)

Is it then interesting that Einstein's GR states twice the curvature than Newtonian Mechanics?
Title: Re: Is there a discrepancy with the equivalence principle?
Post by: Colin2B on 25/01/2018 22:10:54
Then the field of the central body would be applying blue shift to the incoming photon. There would be two opposite time dilation effects.
I did wonder if this might be similar to what @timey  might be thinking. Obviously current physics treats this as the difference in GP between the 2 points, but i dont see why it shouldn’t be modelled this way. Not sure i would call it an inverse function, more a contra-directional one.
I will be interested to see how it produces the anomaly close to higher density.

@timey   Ive been through the video, but skipped the repetitive bits. In the paper you can start with the principle of longer/shorter seconds with distance and then not need to go over it each time.
Just to explain what i said earlier:
A vector has both direction and magnitude, so although on the graph of t vs x the line can be thought of as being in the x direction it doesnt have a specific magnitude. So i would refer to the line as a graph or plot.
Are you absolutely sure you know what partial derivative is, how it is used and what it implies? If so then do use it, but it gave me real problems understanding what you are trying to show. I think of the time axis more as varying with x, so it isnt a linear scale eg you will be familiar with frequency in a frequency response curve being shown log scale to match how we hear it.
Overall it is up to you to decide how best to present your ideas.
I will be interested to see next stage, the meat.
Title: Re: Is there a discrepancy with the equivalence principle?
Post by: timey on 26/01/2018 00:19:33
I think perhaps the shorter descriptions that I will give as captions to the worksheets will be more accessable.  I'll definately be working to that end anyway.

With regards to giving these considerations magnitude, the whole exercise of this spacetime structure is to describe a formula of proportionality in the same way that the left side of the GR equation does, where both sides of that GR equation are background independent, meaning that the proportionality expressed by that equation is valid for any magnitude, and any reference frame.
Page 8 gives a description of how this proportionality is established, thus giving magnitude to the graphs or plots, (bearing in mind that a more experienced mathematician could and would describe these graphs and plots as vectors in a far more eloquently expressed equation), and pages 9 & 10 describe the physical mechanics of this proportionality.
However GR breaks down at high density, ie: black holes.  My suggested spacetime structure won't.

Partial dirivitive definition: "A partial derivative of a function of several variables is its derivative with respect to one of those variables."
Partial dirivitives of a time dilation being directional 'up', 'down' variables of a singular function,  of 1 of any 3 time phenomenon.
Partial dirivitives of changes in acceleration (a), (b), & (c) being variables of a singular function of observed changes in acceleration.
Am I wrong in my assessment of the terminology?

But going back to page 6, this being my main concern, if you can find the time at some point Colin to help me properly formalise that R scale consideration with the appropriate mathematical notation, that would be really great.  In the meantime I'm just going to get my head down and re-write the paper, so will be missing for a bit.
Thanks for the advice so far.

Edit: diriv'a'tive ;)
Title: Re: Is there a discrepancy with the equivalence principle?
Post by: timey on 27/01/2018 17:58:17
Chuckle...
Title: Re: Is there a discrepancy with the equivalence principle?
Post by: pasala on 28/01/2018 15:31:19
Mr. timey
Well, here i am sharing mine ideas on gravitational time dilation.  Right from the starting we are looking at the one side of the coin only.  At present we are of the view that outside area is empty and in case if that is true then why don't we measure speed of light in open area.  It is true that certainly there is a influence.  Each photon released from the clock is not free from outside influence.  Unless a new photon is released and makes a way there is no pressure or force and this pressure causes movement of particles.  In a constant gravitational field, your watch, needles, photons and EMF everything is undergoing constant pressure.  When velocity is gained, everything is relaxed from gravity and the release of photons slows down.  Unless a new photon is released your watch fails to gain KE. 

"Time dilation is only a simple instrument to measure gravitation variations at two different places and nothing else and in fact it cannot tell you what exactly a gravity is". 

First of all we have to set our mind that open area is not free and there is strong EMF is present and it is influencing each and everything including functioning of atomic clocks.

Yours
Psreddy

Title: Re: Is there a discrepancy with the equivalence principle?
Post by: timey on 28/01/2018 17:45:42
Mr. timey
Well, here i am sharing mine ideas on gravitational time dilation.  Right from the starting we are looking at the one side of the coin only.  At present we are of the view that outside area is empty and in case if that is true then why don't we measure speed of light in open area.  It is true that certainly there is a influence.  Each photon released from the clock is not free from outside influence.  Unless a new photon is released and makes a way there is no pressure or force and this pressure causes movement of particles.  In a constant gravitational field, your watch, needles, photons and EMF everything is undergoing constant pressure.  When velocity is gained, everything is relaxed from gravity and the release of photons slows down.  Unless a new photon is released your watch fails to gain KE. 

"Time dilation is only a simple instrument to measure gravitation variations at two different places and nothing else and in fact it cannot tell you what exactly a gravity is". 

First of all we have to set our mind that open area is not free and there is strong EMF is present and it is influencing each and everything including functioning of atomic clocks.

Yours
Psreddy


Please excuse me that I do not engage with you in discussion regarding time dilation.  I am entirely focused on expressing my own ideas on this thread, bar the string theory v quantum loop gravity comedy interlude.
I simply do not have the time for a debate, and to be frank, unless you are telling me that your ideas result in a falsifiable prediction for a doable experiment, then I'm afraid my interest will not be much piqued...
If your ideas do result in a falsifiable predicition then please do feel free to post a link to 'your' thread, and the posts that describe your prediction, and I shall read with interest.  If not, then good luck to you all the same, and sorry again that I'm too busy to be conversational.
Title: Re: Is there a discrepancy with the equivalence principle?
Post by: timey on 30/01/2018 22:22:37
Well the good news is that I have discovered that I can describe that which is on page 5 far more clearly. (fun, fun)
The bad news is that I will have to put pen to paper and draw it out again. (sigh)
Title: Re: Is there a discrepancy with the equivalence principle?
Post by: Colin2B on 31/01/2018 14:19:11
bearing in mind that a more experienced mathematician could and would describe these graphs and plots as vectors in a far more eloquently expressed equation
My point is that they wouldn’t, and it’s nothing to do with ‘eloquently expressed equations’, just with the definition and usage of the term vector. They would reserve vectors for describing a specific magnitude and direction (eg of speed or acceleration) rather than using the term on a graph which shows all possible magnitudes.
At the end of the day you are responsible for final edit and how you say it. Most people will reinterpret what you've said and move on, so it’s not a show stopper.

The next one you really need to be clear on

Partial dirivitive definition: "A partial derivative of a function of several variables is its derivative with respect to one of those variables."
Correct, but it depends on a thorough understanding of derivatives.
Partial dirivitives of a time dilation being directional 'up', 'down' variables of a singular function,  of 1 of any 3 time phenomenon.

Partial dirivitives of changes in acceleration (a), (b), & (c) being variables of a singular function of observed changes in acceleration.
I don’t understand what you are saying in those 2 statements.

You need to be clear which one of the variables you are taking the derivative with respect to. This not clear from the graphs - the assumption would be wrt x as time is the dependant variable to x, however, derivative of t wrt x is not speed, neither is the partial differential of t divided by x an acceleration.
As it's the first time you use a derivative is page 2 diagrams 4 & 5, you might like to expand on what you mean and also how you arrive at eg deceleration = longer seconds.
Again, you know what you are trying to say and the editing is up to you, as long as you are clear how you are using the terminology.

But going back to page 6, this being my main concern, if you can find the time at some point Colin to help me properly formalise that R scale consideration with the appropriate mathematical notation, that would be really great. 
In equations of motion distance s=vt+at2/2
vt is the distance travelled if there were no acceleration, in this case =ct
So distance due to the acceleration = at2/2 so if a=c2/R
Then dist =c2t2/2R

So if you want to compare extra distance travelled to distance without acceleration you divide the extra dist by ct
so ratio = ct/2R

@jeffreyH can you think of any other ways of presenting this?
Title: Re: Is there a discrepancy with the equivalence principle?
Post by: timey on 31/01/2018 19:04:39
Thanks for the feedback Colin.
On the basis of your previous comments I had actually included in my 'latest" re-write an explanation as to my usage of the partial dirivative term as part of a prelude introduction to the worksheets.
On the basis of your most recent comments I've added again to that prelude - posted below, and I am hoping that my explanation is sufficient that I am not miss-understood?  This is a very different way of describing the familiar phenomenon of gravitational acceleration and attraction, and other types of motions taking place within the gravity potential, where magnitude is attributed via the magnitude of the gravitational mass...
...And I ask you to hold the thought that in General Relativity, mass tells space how to bend, and space tells mass how to move.

I am mulling over your page 6 postings.  This will take me some time, as I tranpose the meaning of your maths at 'baby steps' rate into words, shapes, and back to maths again in order to understand.  I'll get back when I've understood it.

-------------------------------

Introducing An Additional Time Dilation Phenomenon:

These following worksheets illustrate how the geometric curvature of spacetime can be attributed to an additional temporal phenomenon, and how by adding this temporal phenomenon, this constitutes a renormalisation term to the GR distortion of the metric, resulting in Euclidean geometry.

The descriptions below will ask you to consider that there are not 2 but 3 time dilation phenomenons.
Time dilation 1 being caused by motion, both centripetal and other.
Time dilation 2 being caused by postion of height in the gravity potential of M.
Note that this model states that time dilation 1 and time dilation 2 are for where m doesn't equal zero.
This model introduces an additional time dilation - time dilation 3 - this being for where m does equal zero. ie: for particles m=0, and for the open spaces or 'background space' between masses.

The considerations below are based on a simple idea that if an object is travelling in a background space that is inherent with it's own rate of time - a rate of time that the object must travel through - an object travelling at a constant speed over distances where seconds get progressively shorter, will be accelerated. And an object travelling at constant speed over distances where seconds get progressively longer, will be decelerated.

I have described in the worksheets below, (perhaps unusually, hence the explanation), the 'up' 'down' changes of each time dilation phenomenon as 'partial dirivatives' of that time dilation.  As well as signifying the plus minus changes in time, this notation is indicative of directional orientation in the gravitational gradient.  These 'up' 'down' dirivatives also become useful notation when a consideration of changes in acceleration contains both a plus and a minus of the same time dilation phenomenon.
The time changes in each of the 3 time dilations are then each considered as a partial dirivative of changes in acceleration, where when added together in the mannner suggested, these 3 partial dirivatives of changes in acceleration become the changes in acceleration that are proportional to changes in gravity potential.

-------------------------------------
Title: Re: Is there a discrepancy with the equivalence principle?
Post by: jeffreyH on 31/01/2018 20:57:17
@Colin2B I need to review the following first.
https://en.m.wikipedia.org/wiki/Modified_Newtonian_dynamics
Title: Re: Is there a discrepancy with the equivalence principle?
Post by: jeffreyH on 31/01/2018 21:19:20
@Colin2B I have read through the wikipedia article and I am unconvinced by MOND.
Title: Re: Is there a discrepancy with the equivalence principle?
Post by: Colin2B on 31/01/2018 22:33:41
@Colin2B I have read through the wikipedia article and I am unconvinced by MOND.
I agree, it seems to put forward a solution which has no evidence to suggest it might be the way to go.

@timey you say “Note that this model states that time dilation 1 and time dilation 2 are for where m doesn't equal zero.”
SR is motion related and is defined for where gravity = 0, or a very close approximation ie flat spacetime. Does time dilation 1 include SR?
Title: Re: Is there a discrepancy with the equivalence principle?
Post by: timey on 01/02/2018 00:50:44
I'm not really sure where MOND applies in the page 6 Rscale consideration other than via the cosmological constant.  What interests me is that Milgrom's simple mathematical variant can describe galaxy rotation without dark matter.

quote wiki:
"By itself, Milgrom's law is not a complete and self-contained physical theory, but rather an ad-hoc empirically motivated variant of one of the several equations that constitute classical mechanics. Its status within a coherent non-relativistic theory of MOND is akin to Kepler's Third Law within Newtonian mechanics; it provides a succinct description of observational facts, but must itself be explained by more fundamental concepts situated within the underlying theory."
Unquote

Special relativity is a motion related calculation that can only be applied where gravity can be negated, ie: flat spacetime.
Yes time dilation 1 is a special relativity related consideration, where I am asking the question "Does the fact that clocks all tick 'potentially' at the same rate at sea level of any elevation of the equatorial bulge on Earth, mean that a clock ticking at sea level is both a general relativity, and a special relativity consideration?"

And yes, it is important that special relativity is calculated in flat space time, because adding the addition time phenomenon 3 for background space means that curvature is a temporal phenomenon and the spatial geometry of space is 'not' curved by a gravitational mass.
Title: Re: Is there a discrepancy with the equivalence principle?
Post by: Colin2B on 01/02/2018 13:05:29
@timey “I am mulling over your page 6 postings.  This will take me some time, as I tranpose the meaning of your maths at 'baby steps' rate into words, shapes, and back to maths again in order to understand.  I'll get back when I've understood it.”

I could do some diagrams of the motion equations if that would help.
Title: Re: Is there a discrepancy with the equivalence principle?
Post by: timey on 01/02/2018 16:57:44
No it's ok, I think I am with you on your description below as to the outcome of extra distance travelled due to acceleration, but I do have some questions about the notation that I shall get into in a mo.
-----------------------
quote Colin:
"In equations of motion distance s=vt+at2/2
vt is the distance travelled if there were no acceleration, in this case =ct
So distance due to the acceleration = at2/2 so if a=c2/R
Then dist =c2t2/2R

So if you want to compare extra distance travelled to distance without acceleration you divide the extra dist by ct
so ratio = ct/2R"
--------------------------
Yes I do want to compare the extra distance travelled with the acceleration, to the original distance (wavelength Rscale) without acceleration, but I think in order to get the small amount of distance I'm looking for I need to divide the extra distance by the distance of wavelength Rscale.  And in order to get the small amount of time I'm looking for I need to divide the extra distance by the speed of light. (Edit: Oh I don't think that is quite finished. Then perhaps divide this time by the age of the universe? Noting that all measurements are made via the observers clock.)  In this manner the small acceleration is proportional to the small distance and the small time.  Am I right?

Ok, I get your notation right up until you say divided by 2 and divided by 2R.  I'm assuming that R is the Rscale wavelength, but could benefit from some clarification as to this, and the use of 2.
Title: Re: Is there a discrepancy with the equivalence principle?
Post by: Colin2B on 01/02/2018 18:15:04
Yes I do want to compare the extra distance travelled with the acceleration, to the original distance (wavelength Rscale) without acceleration, but I think in order to get the small amount of distance I'm looking for I need to divide the extra distance by the distance of wavelength Rscale.  And in order to get the small amount of time I'm looking for I need to divide the extra distance by the speed of light. (Edit: Oh I don't think that is quite finished. Then perhaps divide this time by the age of the universe? Noting that all measurements are made via the observers clock.)  In this manner the small acceleration is proportional to the small distance and the small time.  Am I right?

Ok, I get your notation right up until you say divided by 2 and divided by 2R.  I'm assuming that R is the Rscale wavelength, but could benefit from some clarification as to this, and the use of 2.
On page 6 of your video, second line down, you give c2/R = acceleration so I have just taken this as a in at2/2.


Title: Re: Is there a discrepancy with the equivalence principle?
Post by: timey on 01/02/2018 18:51:26
Yes, Lee Smolin says that where R is wavelength on Rscale:
R/c = age of universe
c2/R = acceleration (equal to the acceleration that the 'expanding universe' expansion is thought to be accelerating at)

...so the maths "at2/2" where a is c2/R, says acceleration multiplied by time squared, divided by 2.
What is, or why 2?
Title: Re: Is there a discrepancy with the equivalence principle?
Post by: Colin2B on 01/02/2018 22:23:11
What is, or why 2?
I’ll do a diagram
Title: Re: Is there a discrepancy with the equivalence principle?
Post by: timey on 02/02/2018 00:38:17
What is, or why 2?
I’ll do a diagram
Ok, great!  Probably for the best (chuckle)

In the mean time, does this work?

R/c=t (age of universe)
ct=R (distance)
c2/R=a (acceleration = cosmological constant)

at2/2/R= tiny length distance
at2/2/c/t= tiny length time
Title: Re: Is there a discrepancy with the equivalence principle?
Post by: alancalverd on 02/02/2018 07:26:12
...so the maths "at2/2" where a is c2/R, says acceleration multiplied by time squared, divided by 2.
What is, or why 2?

Welcome back to the asylum!

Acceleration is the change of velocity per unit time. If a body starts at rest and accelerates at a for time t, its final velocity is at. If we want to know how far it has travelled (s), we need to multiply its average velocity (at/2) by time t.

Generalising, for a body with initial velocity u and final velocity v

v = u + at

s = ut + ½at2    and for completeness

v2 = u2 + 2as.

Nothing clever about these equations - they just derive from the  definition of velocity and acceleration.


Title: Re: Is there a discrepancy with the equivalence principle?
Post by: Colin2B on 02/02/2018 15:00:53
Ok, what Alan said, what i said earlier, plus piccies.

Pic 1
This shows constant speed for a time t, easy to understand because distance travelled in time t = u1t

Pic 2
This shows a constant acceleration a starting at 0 and finishing at time t. Final speed u2 is = at
However, we cannot use this final speed as with pic 1 because the speed varies over time, but we can use the average speed which is ½at, multiply this by time to get distance and we get ½at2

Pic 3
This just combines the effects of constant speed and acceleration and is used where the starting speed is not 0.


* Speed graph resize.jpg (149.56 kB . 640x478 - viewed 3471 times)

As a little aside, you will notice that in each case the distance is the area under the graph - shaded area.
Title: Re: Is there a discrepancy with the equivalence principle?
Post by: timey on 02/02/2018 18:06:16
Thank you @alancalverd for the welcome.

Yes between yourself and @Colin2B 's diagrams, I am understanding now that the notation I questioned is concerning the mean average...

So...on the basis that it is the extra distance travelled due to the acceleration that I wish to work with, you said Colin in post 839 that:

"So distance due to the acceleration = at2/2 so if a=c2/R
Then dist =c2t2/2R"

So just to check - the distance that =c2t2/2R, that is the extra distance travelled due to the acceleration?
Title: Re: Is there a discrepancy with the equivalence principle?
Post by: Colin2B on 02/02/2018 22:48:06
So just to check - the distance that =c2t2/2R, that is the extra distance travelled due to the acceleration?
There are 2 situations
either universe is not expanding in which case light will travel distance ct
Or universe is expanding and accelerating so total distance due to accelerated expansion is c2t2/2R

If you want to find out the difference in distance between the 2 this
= c2t2/2R - ct
I’m assuming you are using t=age of universe. That’s a lot of seconds
Title: Re: Is there a discrepancy with the equivalence principle?
Post by: timey on 02/02/2018 23:10:14
Yes it sure is a lot of seconds!
Ok, I think I have what I need now hopefully...

R/c=t (age of universe)
ct=R (distance)
c2/R=a (acceleration = cosmological constant)

c2t2/2R-ct/R= tiny length distance
c2t2/2R-ct/c/t= tiny length time

That tiny length of time should be just a tiny fraction of a second, if it isn't then I'm not doing the equation correctly.

(As an asside that would really help me visually, I don't suppose you would convert the length of c2t2/2R-ct into a percentage of ct for me?)
Title: Re: Is there a discrepancy with the equivalence principle?
Post by: Colin2B on 03/02/2018 09:04:33
That tiny length of time should be just a tiny fraction of a second, if it isn't then I'm not doing the equation correctly.
Or the acceleration youve been given is wrong.

Why not use something like this from measurements in 2012
“expanding at a rate of 74.3 plus or minus 2.1 kilometers (46.2 plus or minus 1.3 miles) per second per megaparsec (a megaparsec is roughly 3 million light-years).”
A little exercise for the brain cels
Title: Re: Is there a discrepancy with the equivalence principle?
Post by: timey on 03/02/2018 12:47:18
The velocity of expansion is on as un-solid ground, regarding the discrepancy between galaxy recession and the CMB recession, as the rate of accelerated expansion is Colin, and why would Smolin print in his book an equation that is not the rate that the accelerated expansion is thought to be?

Are you saying that my equation of a tiny length of time is not resulting in a fraction of second?  Because if not then this will only be because I haven't finished the equation properly, in which case it's not about exercising brain cells, it's about needing some advice.

Edit: There is a chance that I might have dyslexic-ly given c2/R when it should be R/c2, and I will go back to the book and check it later. But with regards to the calculations, it will not work if I use your suggested method of calculation, b/c the whole point of using the Rscale observation is that the observation of the Rscale is relativistic. (Edit 3: besides - the acceleration has to be calculated for the entire length of the Rscale for the required tiny distance and the required tiny time to be proportional for the purpose I'm putting them to) - (Edit 2: By introducing numerical mph, etc, this renders the calculation as only valid for the observers reference frames clock rate. The calculations I seek will be valid from any reference frames clock) ...and I am 'not' going to be describing an expanding universe with the tiny length distance, and the tiny length time I wish to achieve with these calculations.

(Edit 4: I just checked the book and c2/R is the acceleration by which the rate of the universe is thought to be expanding - that is, the acceleration produced by the cosmological constant.)
Title: Re: Is there a discrepancy with the equivalence principle?
Post by: timey on 04/02/2018 01:24:43
OK, so my calculations (for better or worse) make that tiny distance 0.5 of a metre, and that tiny time 0.5 of a second.

If I try 0.5 seconds squared divided by 0.5 metres I get 0.5

Or if I try 0.5 metres divided by 0.5 seconds I get 1.

Not really sure about the construct of my method to say so, still running it through, and in all honesty could use some advice really.
(Edit: Actually my calculations may well be wrong b/c to get away from the big numbers I used:
c2x10 second2/2ct-ct/ct=0.5metre
And:
c2x10second2/2ct-ct/c/t=0.5second
Where it may be that c2x10second2/2ct is not proportional to c2xage of universe2/2R?)

The idea is that I want to attribute a magnitude to a change of time that will result in a change of acceleration to a constant speed travelling within a specific distance.
And that this change in distance constituting a change in position in the gravity potential, comes with changes in time that I'm attibututing to changes in acceleration, where the magnitude of distance (tiny length distance) relates to the magnitude (tiny length time) of changes in time, and the consequent changes in acceleration, via proportionality with the cosmological constant.
Title: Re: Is there a discrepancy with the equivalence principle?
Post by: alancalverd on 04/02/2018 06:59:53
R/c=t (age of universe)
ct=R (distance)
c2/R=a (acceleration = cosmological constant)

For the benefit of latecomers, what is R? Looks like the Schwarzchild radius but I'd like to be sure.

Then a= c2/R admittedly has the dimensions of an acceleration but it is an assertion, not an obvious consequence of physics: acceleration of what?  How derived from observation or known physics?

The assertion (or conclusion if it is such) that ∂2R/∂t2 = c2/R means that the expansion of the universe is slowing down, which is apparently counterfactual to the point of being interesting.
Title: Re: Is there a discrepancy with the equivalence principle?
Post by: Colin2B on 04/02/2018 10:36:25
Not really sure about the construct of my method to say so, still running it through, and in all honesty could use some advice really.
The formulae i quoted are standard for accelleration, velocity, speed, time relationships, but they are dependent on the input data in order to give correct answers.
The reason i said “or the acceleration you’ve been given is wrong” is that i have no knowledge of how c2/R has been derived and under what circumstances it is valid. I dont for example understand how it can equal the cosmological constant.
The reason i suggested you use the 2012 data is that this is a direct observation. My understanding was that your theory would confirm all current observations but give a different interpretation of, say, red shift and expansion, due to your inverse dilation effects.
Title: Re: Is there a discrepancy with the equivalence principle?
Post by: timey on 04/02/2018 12:52:18
@alancalverd
R is the radius of
the observable universe.  And yes you are correct that I am changing the outlook on the expanding universe, but this is not to say that the expansion of the universe is slowing, but that the contraction of the universe is speeding up by the rate of that acceleration, noting that in my model the universe contracts from the moment that inflation ceases.

@Colin2B
I think that the circumstances of c2/R are the observation of R, where dividing c2 by R gives an acceleration that matches that of the acceleration that the universe is thought to be expanding at, where this rate of acceleration is now thought to be associated with the cosmological constant that Einstein retracted from GR.

Yes I understood your idea of reducing the measurement which is what led me to using a 10second time and subsequent distance, (hoping this would be proportional).
But using a measurement that incorporates a redshift consideration based on recessional speed will not work for my calculations.

Yes my model will give a different mathematical description of redshift observations, but this current calculation, although linked, is not the equation that describes this alternative interpretation of the redshift observation.
Title: Re: Is there a discrepancy with the equivalence principle?
Post by: Colin2B on 04/02/2018 13:06:41
to get away from the big numbers I used
R is the radius of the observable universe.
It's probably worth plugging in the real numbers to see what happens.
I've got limited access to WiFi for a while, be interested to see what you've got when I'm back.
Title: Re: Is there a discrepancy with the equivalence principle?
Post by: timey on 04/02/2018 13:18:36
@Colin2B lol, I did try plugging in the big numbers and it fried my brain. :D
I didn't know what I was looking at with the 'powers of' and (e) notations, and subsequently couldn't assess if I'd done it correctly.
Title: Re: Is there a discrepancy with the equivalence principle?
Post by: alancalverd on 04/02/2018 20:51:58
If we consider a simple universe consisting of just two massive particles, and for simplicity give them both the same mass m, then the only way the universe can begin with a big bang and not collapse thereafter is if the particles move apart at greater than their mutual escape speed ve.

Having no other source of kinetic energy they will slow down asymptotically to a constant speed v > ve.

We can generalise this by symmetry to the case of a very large number of particles expanding away from an origin in 3 dimensions.

If the universe is fairly mature, we are moving at or close to v away from the point of origin, so the bits of the universe closest to us will be moving on near-parallel paths with a relative speed of near-zero, and those furthest from us will be on antiparallel paths with a relative speed of 2v. All of which seems fairly consistent with observation.
Title: Re: Is there a discrepancy with the equivalence principle?
Post by: timey on 04/02/2018 20:59:58
All of which seems fairly consistent with 'interpretation' of observation.  Please do not forget that the expanding universe and accelerated expansion are not 'factual' physics, but 'theoretical physics' as they cannot be tested experimentally.
Yes, you are describing an existing model of the universe, all of which seems fairly consistent with observation, (as it should do considering how the theory was created to explain these 'interpretations' of observation), except for the fact that the theory doesn't describe the mechanics, and contains many documented problems.

So... I am describing a different model that will also be consistent with observation.

Back to the maths:
Ok, I have since worked out that the results of my 10second calculation will indeed, (unless I am very much mistaken), be the same as the 'age of the universe' calculation.
 At first I thought that b/c the speed of light squared was being multiplied by a smaller number of seconds squared, that this would throw the proportions off.  But (of course), the result is then divided by 2 times the smaller distance that is the consequence of the reduced number of seconds.  So the resulting tiny distance and tiny time from either calculation should be the same. (Please, please, please someone correct me if I am wrong)
 
Title: Re: Is there a discrepancy with the equivalence principle?
Post by: timey on 05/02/2018 01:38:42
As an asside, until I hear back about the maths...
My model does bounce, but it doesn't bounce like the animation on this link...

https://www.quantamagazine.org/big-bounce-models-reignite-big-bang-debate-20180131/?utm_content=bufferd6943&utm_medium=social&utm_source=twitter.com&utm_campaign=buffer

In my model the outward trajectory would occur in a flash (miniscule fraction of 1 of our seconds), leaving a slightly anistropically distibuted sea of energy and individual particles, that start the clumping process as the outward trajectory stops (fizzles out), this being due to explosion finished.
 
And then the inward trajectory would occur imperceptably slowly, accelerating, as the clumping progress escalates, at the same rate that the universe is supposed to be accelerating in it's expansion at under the remit of the Big Bang theory...
... And this contraction wouldn't necessarily be ending everything up back in the same spot everything left from, like the animation from the link shows.

My model's contraction mechanism is occurring in front of our faces at this moment, as our observations of black holes merging, and galaxies of galaxy clusters colliding. This being the ongoing continuation of the clumping process.

That continues until 'everything' is clumped into one mass, (a singular black hole) which explodes, ejecting everything (via jets), leaving a slightly anistropically distributed sea of energy and individual particles - and on it goes...
Title: Re: Is there a discrepancy with the equivalence principle?
Post by: alancalverd on 05/02/2018 10:52:06
Don't worry about the numbers - they are arbitrary human choices between parsecs and microns, how to divide a circle, and various types of second. Let's see the physics. If the dimensional analysis makes sense, you may have a viable theory.

Accelerated expansion makes some sense in terms of my "generalised two particle universe". The bits that are travelling slower than the fastest, are effectively inside a sphere which is expanding faster than their radial speed, so g is decreasing, for all but the most distant particle, faster than 1/r^3 and the kinetic energy of the slower particles is therefore increasing.
Title: Re: Is there a discrepancy with the equivalence principle?
Post by: timey on 05/02/2018 12:09:50
When I get back to my laptop later I will post you some contracting scenario papers by professional physicists.

But if I make your 2 particle universe into a 30 particle universe and say that 3 groups of 10 particles start moving towards each other into 3 separate clumps, then g in the space that is occurring between the 3 separate clumping processes will be decreasing, and will be decreasing even if the overall 'sphere' or 'arena', in which we imagine the entire process to be occurring within, is also slowly shrinking as a result of the clumping.

Edit: This arrangement would result in a discrepancy between what physics calls 'galaxy cluster recessional speeds' and 'CMB recessional speeds', as 'is' observed.
Title: Re: Is there a discrepancy with the equivalence principle?
Post by: alancalverd on 05/02/2018 13:27:35
Well folks, that's astrophysics sorted and the 3-body problem solved. Anything else we can help with?
Title: Re: Is there a discrepancy with the equivalence principle?
Post by: timey on 05/02/2018 13:40:07
For sure! I was wondering if I might look in your mouth before asking you for the name of your dentist?
Title: Re: Is there a discrepancy with the equivalence principle?
Post by: timey on 05/02/2018 18:28:14
@alancalverd
Investigation into the concept of a contracting universe has been explored under various premise:

By Christof Wetterich
https://arxiv.org/abs/1303.6878

Here in the form of a pre-big bang contraction by Thorsten Battefeld, Robert Brandenberger
https://arxiv.org/abs/hep-th/0406180

And more recently here with regards to black holes by Jerome Quintin, Robert H. Brandenberger
https://arxiv.org/abs/1609.02556

In case, as a latecomer, you have not realised, I was asking about the mathematical consideration b/c I am, following both @Colin2B and @jeffreyH advice, currently improving on my paper that I am trying to submit with arXiv scientific journal.
Title: Re: Is there a discrepancy with the equivalence principle?
Post by: timey on 05/02/2018 18:54:35
But going back to post 861...

https://www.space.com/38496-hunt-for-hubble-constant-standard-model.html?utm_source=notification
quote link:
"Using the Hubble Space Telescope to measure the light from a number of type 1a supernovas, a 2001 study revealed an 8 percent discrepancy in the value for the Hubble constant when compared with the method using precise measurements of the CMB. In other words, the Hubble Space Telescope recorded a faster rate of universal expansion than CMB measurements did. For the Hubble constant to be a fundamental constant, it must be, well, constant, no matter how it's measured."

Of course this necessity for the observation to be a constant is the case for a universe that is supposed to be expanding, but is not the case for a universe that is contracting at an accelerated rate.
Title: Re: Is there a discrepancy with the equivalence principle?
Post by: alancalverd on 05/02/2018 19:35:32
There's no "supposed to be", nor any a priori reason why it should be constant: it is an experimental value derived from measurements of what is actually happening - or at least what happened several zillion years ago - and is quite likely to vary according to how and where it is measured.

Quote
However, the term "constant" is somewhat of a misnomer, the Hubble constant H0is the current value of the Hubble parameter H, which changes over time.

The only surprise would be if it turned out to be negative or oscillatory in the short term.
Title: Re: Is there a discrepancy with the equivalence principle?
Post by: timey on 05/02/2018 22:20:10
@alancalverd
Huh?  You have CMB radiation receding at one speed, and galaxy clusters receding at a speed that is 8% faster. 
That is the observation and the measurement, and these measurements of observation are not as the expanding theory demands, as said in the article.
My model will result in the CMB having a lesser redshift than galaxy clusters do, as 'is' observed.
My model's interpretation of the redshift observation is different from the expanding theory, as it states all (edit correction: all of the CMB, and the major part of galaxy cluster) redshift observations as being due to gravitational shift of the light in the changing gravity field.

I am not here to debate a contracting universe versus an expanding universe.  In fact your posts are causing me some distress here Alan.  I have no wish to have to defend my right to create an alternative model of the universe, or to persuade you as to the importance of dicerning that a measurement based on an interpretation of an observation is not 'factual physics', and therefore the expanding universe is 'theoretical physics', a fact I thought we cleared up many posts ago.

I am here asking for advice with my mathematical method in describing a contracting universe, and advice on the layout of the paper that I am busy writing. I would be most delighted if you helped me in this venture, but cannot and will not engage with you in arguments about whether the universe is 'supposed to be' expanding.
I provided you with 3 links to papers by professional theoretical physicists who have put forward contracting scenarios, and General Relativity can describe a contracting universe just as soon as it describes an expanding one.
Enough already!
Title: Re: Is there a discrepancy with the equivalence principle?
Post by: timey on 05/02/2018 22:35:34
@Colin2B
I have posted the maths again as they had got lost back there somewhere...

OK, so my calculations (for better or worse) make that tiny distance 0.5 of a metre, and that tiny time 0.5 of a second.

If I try 0.5 seconds squared divided by 0.5 metres I get 0.5

Or if I try 0.5 metres divided by 0.5 seconds I get 1.

Not really sure about the construct of my method to say so, still running it through, and in all honesty could use some advice really.
(Edit: Actually my calculations may well be wrong b/c to get away from the big numbers I used:
c2x10 second2/2ct-ct/ct=0.5metre
And:
c2x10second2/2ct-ct/c/t=0.5second
Where it may be that c2x10second2/2ct is not proportional to c2x age of universe2/2R?)

The idea is that I want to attribute a magnitude to a change of time that will result in a change of acceleration to a constant speed travelling within a specific distance.
And that this change in distance constituting a change in position in the gravity potential, comes with changes in time that I'm attibututing to changes in acceleration, where the magnitude of distance (tiny length distance) relates to the magnitude (tiny length time) of changes in time, and the consequent changes in acceleration, via proportionality with the cosmological constant.
----------------------------
Next post:
Yes had already tried the big numbers, they fried my brain b/c I didn't know what I was looking at with the 'powers of' and 'e' notations, therefore I could not assess if I had done it correctly.

-------------------------------
Next post:
Ok, I have since worked out that the results of my 10second calculation will indeed, (unless I am very much mistaken), be the same as the 'age of the universe' calculation.
 At first I thought that b/c the speed of light squared was being multiplied by a smaller number of seconds squared, that this would throw the proportions off.  But (of course), the result is then divided by 2 times the smaller distance that is the consequence of the reduced number of seconds.  So the resulting tiny distance and tiny time from either calculation should be the same. (Please, please, please someone correct me if I am wrong)
Edit correction: Please, please, please someone tell me if I am right.
Title: Re: Is there a discrepancy with the equivalence principle?
Post by: alancalverd on 06/02/2018 23:26:32
It's difficult to understand your equations. If you don't have access to superscripts,  please clarify by using the convention a^2 to indicate a2, and plenty of brackets so that  p/(p + q) is clearly different from (p/p) + q, rather than the ambiguous p/p+q.
Title: Re: Is there a discrepancy with the equivalence principle?
Post by: timey on 06/02/2018 23:48:24
Hmmm yes, I see my sloppiness now.  Hope this has fixed it...

c2^x10 second2^/2(ct)-(ct)/(ct)=0.5metre
And:
c2^x10second2^/2(ct)-(ct)/c/t=0.5second

Where it may be that c2^x10second2^/2(ct) is not proportional to c2^x age of universe2^/2R, where R is radius of observable universe.

Except that I now think that the reduced measurement calculation 'is' propotional to the big number calculation, and that the 0.5 metre, 0.5 second result will be the same for both the big number and the reduced measurement calculation, but would like to confirm that this is actually the case.
Title: Re: Is there a discrepancy with the equivalence principle?
Post by: Colin2B on 07/02/2018 10:42:09
@timey  I think you mean:

c^2x10 second^2/2(ct)-(ct)/(ct)=0.5metre
And:
c^2x10second^2/2(ct)-(ct)/c/t=0.5second

So the resulting tiny distance and tiny time from either calculation should be the same. (Please, please, please someone correct me if I am wrong)
Edit correction: Please, please, please someone tell me if I am right.
Just to be clear, none of what I have said or will say is aimed at whether your overall ideas are right or wrong. My only objective is to clarify what you are saying and give your ideas the best chance of being understood. With that in mind:

I’m trying to understand what you have done and why. Can you explain step by step. It looks as though you substituted for R in the 1st part of the equation, but why did you divide by ct in the 2nd part. Also i don't see how the 1st equation can give metres and the 2nd seconds. Please explain.

However, this may be irrelevant if i interpret this correctly
The idea is that I want to attribute a magnitude to a change of time that will result in a change of acceleration to a constant speed travelling within a specific distance.
And that this change in distance constituting a change in position in the gravity potential, comes with changes in time that I'm attibututing to changes in acceleration, where the magnitude of distance (tiny length distance) relates to the magnitude (tiny length time) of changes in time, and the consequent changes in acceleration, via proportionality with the cosmological constant.
As with your use of partial derivative, I find parts of this confusing. Are you talking about changes in acceleration (see bold), or are you talking about changing an acceleration to a straight line - what you are calling “a constant speed travelling within a specific distance”?
If it is the latter then working out the “tiny distance and tiny time” won’t give you what you are looking for. To explain, imagine a car accelerating from rest. Say in the 1st 100m it takes 100s ie 1m/s. Later it covers 100m in say 10s = 10m/s so as it accelerates it covers fewer seconds for each 100m, so in your model to keep a straight line graph the spacing between seconds has to get longer as it accelerates, and it is this relationship you are looking for. You will remember early on I talked about log scales etc.
So we can use the equation for a straight line graph y=mx+c. In this case there is no constant c, so we use y=mx
So if distance s=at^2/2=c^2t^2/2R we can see that c^2/2R is a constant equivalent to m in y=mx. Then we call dilated time td =t^2 (x in y=mx) and we have a straight line graph s=c^2td/2R which gives you the relationship between distance and dilated time.
Title: Re: Is there a discrepancy with the equivalence principle?
Post by: timey on 07/02/2018 13:57:38
Colin (chuckle), on my phone it took me a while to spot the difference.  Wrong placing of (^) noted ;)

What you have posted is highly useful to me in my further description of page 5 and the consideration of 'vanishing' curvature and 'vanishing' SR distortions in length.  So thanks!

But to say so, the page 6 consideration of the Rscale equation is for a different purpose.
The reason I am looking for this 'tiny length distance' and 'tiny length time' is for the purpose of describing a 'value' or 'magnitude' to a description of the 'change' in time when a change occurs, and - on the basis that distortions of geometry have been 'vanished' as suggested page 1 through to 5 - a 'specific' tiny length of distance to which I can pin the 'changes' as occurring within.

My reason for doing so is based on the fact that a second is a quantised unit of an hour, where any further breakdown of these units of time can also be considered as quantised units. And my intention is quantising a 'tiny length time' as the magnitude of a change in time, where the magnitude of a change in time causes the changes in acceleration that occur in that specific 'tiny length' distance.*
The reason why it is important for my model to ascertain this particular measure of changes in acceleration has its basis in the fact that there are several factors causing acceleration in my model, where the values must be separately described.

(*The fact that the consideration is based on the 'observation' of the R scale observable universe means that the mathematical structure is 'background independent' and will be proportional from any reference frame)
Title: Re: Is there a discrepancy with the equivalence principle?
Post by: Colin2B on 07/02/2018 14:45:57
Ok, but can you explain what you did to the original formulae in order to get what you believe is a small distance and small time? As i said before:
“I’m trying to understand what you have done and why. Can you explain step by step. It looks as though you substituted for R in the 1st part of the equation, but why did you divide by ct in the 2nd part. Also i don't see how the 1st equation can give metres and the 2nd seconds. Please explain.”

EDIT: I've read paras 3&4 in your last post a few times and I'm still not sure exactly what you are trying to do. Can you explain another way??
Also you are still talking about changes in acceleration. The formulae Alan & I gave you are for constant acceleration, changing speed so would not work directly with a changing acceleration.
Title: Re: Is there a discrepancy with the equivalence principle?
Post by: timey on 07/02/2018 16:21:16
Just saw your edit which will have to wait till.later but:

OK, what I have done in order to make a reduced measurement calculation that 'should' be proportional to the 'required' calculation of
c^2xage of universe^2/2R
where R is the radius of observable universe

...is substitute the age of the universe with 10seconds, and substitute R with the distance that c travels in 10seconds, ie: (ct).

In the 'required' equation I then minus R like this:
c^2x age universe^2/2R-R

So in the reduced equation this is:
c^2x10second^2/2(ct)-(ct)
where t is 10second

So this action of minus the original distance has resulted in the distance travelled due to the acceleration, and now to arrive at the 0.5 metre result I continue the equation:
c^2x10second^2/2(ct)-(ct)/(ct)=0.5metre

And to arrive at the 0.5second result:
c^2x10second^2/2(ct)-(ct)/c/t=0.5seconds

Where b/c the reduced equation is proportional to the required equation (that is the intention anyway), that the required equation will calculate the same result, like this:
c^2xage of universe^2/2R-R/R=0.5metres?
And:
c^2xage of universe^2/2R-R/c/t=0.5seconds?
where t is the age of the universe.
Title: Re: Is there a discrepancy with the equivalence principle?
Post by: alancalverd on 07/02/2018 17:12:43

Ltet's unravel
c2^x10 second2^/2(ct)-(ct)/(ct)=0.5metre

Assuming you mean (c2 x 102/2ct ) - ct/ct

ct/ct = 1 so we have ((c2 x 100)/2ct) - 1 = (50c/t) - 1 so it all depends on the value you have chosen for t, except that c/t is a length and 1 is dimensionless, so the equation is meaningless.

The radius of the observable universe is R= ct if t is the age of the universe, so what exactly are you trying to calculate?


Title: Re: Is there a discrepancy with the equivalence principle?
Post by: timey on 07/02/2018 19:11:12
@alancalverd
quote alan:
"Assuming you mean (c2 x 102/2ct ) - ct/ct"

No, I think I meant (c^2x10seconds^2/2(ct) - (ct)/(ct)
In words:
The speed of light squared times ten seconds squared, divided by two times the speed of light times ten seconds, minus the speed of light times ten seconds, divided by the speed of light times ten seconds. 

quote alan:
"The radius of the observable universe is R= ct if t is the age of the universe, so what exactly are you trying to calculate?"

Does c^2xage of universe^2/2R-R/R=0.5metres? Where R is the radius of the ovservable universe.

Does c^2xage of universe^2/2R-R/c/t=0.5seconds?
where t is the age of the universe.

(I can write these above 2 equations in words too if it helps)

I will explain how and why I want to use these resulting distance and time in the next post as a clarification requested by Colin.  But Alan, you do realise that Colin has watched my YouTube lecture?
Title: Re: Is there a discrepancy with the equivalence principle?
Post by: timey on 07/02/2018 22:21:29
EDIT: I've read paras 3&4 in your last post a few times and I'm still not sure exactly what you are trying to do. Can you explain another way??
Also you are still talking about changes in acceleration. The formulae Alan & I gave you are for constant acceleration, changing speed so would not work directly with a changing acceleration.

@Colin2B  Are you familiar with the set of 10 equations in Einstein's general theory of relativity that describe the fundamental interaction of gravitation as a result of spacetime being curved by mass and energy?

B/c it occurs to me that if you are, although it is simpler for me to think in terms of how I think about the considerations that I am making, that it would be a damn sight simpler for me to take you through these existing equations that distort the geometry (via DrPhysicsA's YouTube lecture "Einstein's Field Equations fo Beginners"), and describe my correction at source, than it would for me to explain to you the route that I am attempting.
Perhaps more interesting for you, and not so irritating for Alan, if we are on famiiar ground maybe?

But I do very much want to know if I'm right in thinking that the R scale equations* I am talking about do result in 0.5 metres and 0.5 seconds, so that I can carry on with my thinking in my own terms. (*Hope the walk through in post 874/876 clarified)
Alan quoted a comment earlier, that Hubble's constant changes over time.  In my model time changes over time, and the information I am seeking is associated (in part) with considering this factor.
Title: Re: Is there a discrepancy with the equivalence principle?
Post by: alancalverd on 07/02/2018 23:04:09
Does c^2xage of universe^2/2R-R/R=0.5metres? Where R is the radius of the ovservable universe.

This now looks like c2t2/(2R-R)/R, which we can rearrange more conventionally as Rc2t2/R = c2t2 = R2 since R = ct. Since R2 has dimensions of square meters (an area), it cannot equal 0.5 meters or any other distance.

Quote
Does c^2xage of universe^2/2R-R/c/t=0.5seconds?
Conventionally written as c2t2/(2R-R)/c/t, or rearranged as cR2/Rt or  cR/t, this has dimensions L x T-1 x L x T-1, i.e. (meters/second)2 - the square of a speed - and therefore cannot equal 0.5 seconds.

It doesn't matter if t is 10 seconds or the age of the universe: the numbers will change but the dimensions won't.
Title: Re: Is there a discrepancy with the equivalence principle?
Post by: timey on 08/02/2018 00:03:38
Not c^2t^2/(2R-R)/R

(you are dividing c^2t^2 by (2R-R), which mathematically breaks down to (1R) and as such the term (2R-R) is nonsensical)

Please excuse my lack of brackets in my last post, it's:

c^2t^2/(2R)-R/R
and
c^2t^2/(2R)-R/c/t

In words:
The speed of light squared times the age of the universe squared, divided by 2 times the radius of the universe, minus the radius of the universe, divided by the radius of the universe.
and
The speed of light squared times the age of the universe squared, divided by 2 times the radius of the universe, minus the radius of the universe, divided by the speed of light, divided by the age of the universe.
Title: Re: Is there a discrepancy with the equivalence principle?
Post by: alancalverd on 08/02/2018 08:52:07
c^2t^2/(2R)-R/R
and
c^2t^2/(2R)-R/c/t

1.   Since ct = R, c2t2/(2R) = R/2, which is a length.   
R/R = 1, dimensionless, so you can't subtract it from c2t2/(2R).

2.   R/c/t = Rt/c = (ct x t)/c = t2,

so c2t2/(2R) - R/c/t is a length minus time squared, which is also meaningless.

We've been here many times before. Ignore the numbers, which are mostly arbitrary human choices. Check the dimensions - that's where the physics resides.
Title: Re: Is there a discrepancy with the equivalence principle?
Post by: Colin2B on 08/02/2018 09:46:16
Are you familiar with the set of 10 equations in Einstein's general theory of relativity
........it would be a damn sight simpler for me to take you through these existing equations ......and describe my correction at source, than it would for me to explain to you the route that I am attempting.
Most of us are familiar with the field equations, but whether your correction is seen as valid will depend the logic of how you got there and you will have to show that in any paper. Alan beat me to making the same comment on your use of the basic motion equations, so whichever route you take in your explanation, you will still have to show us how you got there.
Title: Re: Is there a discrepancy with the equivalence principle?
Post by: timey on 08/02/2018 12:22:57
@alancalverd
Colin gave me the maths c^2t^2/2R = distanceR+distance extra travelled due to acceleration. (Or so I understood anyway)

If this equation does = distance R+extra distance travelled due to acceleration, then - R leaves me with the distance travelled due to acceleration, and I divide this extra distance by distance R. Which in my mind a distance divided by a distance is a distance.  And it is a distance that I'm looking for. (Which I then use for another consideration altogether)

@Colin2B If you already understood from page 1 to page 4b of my video, that an acceleration to a constant speed can be represented as a straight line by adding an additional time dilation for background space. Where the changes of this time in this background space - that occurs concurrently with, and as a separate phenomenon to the timing of clocks in the gravity potential - are inversely proportional in the gravity potential to the changes of the timing of a clock in the gravity potential, then no need to further explain...
Other than to say again, that the c^2t^2/2R equation, and the process that I want to apply to it, are not involved in making that description, and the description that I do want to make with it, I cannot make until I've received your advice as to whether or not my continuation to that equation results in 0.5 metres.

So b/c I am now thoroughly confused my next question is:
Does c^2t^2/2R = distanceR+distance extra travelled due to acceleration?
If not, then could you please show me the mathematical means to arrive at the extra distance travelled due to the acceleration if R is the radius of the universe, and c2/R is the acceleration?
Title: Re: Is there a discrepancy with the equivalence principle?
Post by: alancalverd on 08/02/2018 12:49:47
We know R = cT, by definition, if T is the age of the universe.

Now add an acceleration a= d2r/dt2. The position of a point previously at R, after a time interval Δt, will be R + a(Δt)2/2. Basic equation of motion.

So what? Well, if a(Δt)2/2 > cΔt, the object will have disappeared beyond the Schwarzchild radius, so the mass of the observable universe will have decreased and (assuming gravity travels at no more than c) the universe will continue to expand as g will have decreased at the periphery.

So the requirement for an expanding universe is  a(Δt)2/2 > cΔt, simplifying to aΔt > 2c. All we need to do is to find a cause for a, however small its value, because if we wait long enough we will satisfy the inequality.

Now if an object is already at escape speed and you reduce g, it will accelerate. So if just one particle leaves the Schwarzchild radius, the observable universe will expand at an ever-increasing rate. In summary: the universe expands because it does.
Title: Re: Is there a discrepancy with the equivalence principle?
Post by: timey on 08/02/2018 13:14:17
And could you please show me what is the mathematical means (when using R as the distance) to arrive at the extra distance travelled in 13.8 billion years, when travelling at speed c, when subject to acceleration c^2/R?
Title: Re: Is there a discrepancy with the equivalence principle?
Post by: alancalverd on 08/02/2018 13:47:04
s = at2/2, as we said earlier, so 13.8 x  c4/2R2  billion light years.

Problem occurs if you start with R = 0, giving an infinite initial acceleration, but you can use the above approximation for short times where s<<R and  R>>0
Title: Re: Is there a discrepancy with the equivalence principle?
Post by: timey on 08/02/2018 13:58:22
So:

c^2t^2/2R= extra distance travelled due to acceleration?
(Where t is the age of the universe.)
Title: Re: Is there a discrepancy with the equivalence principle?
Post by: Colin2B on 08/02/2018 14:41:52
So:

c^2t^2/2R= extra distance travelled due to acceleration?
(Where t is the age of the universe.)
No, total distance travelled, from rest, at a given acceleration of c^2/R
Extra distance due to acceleration depends what you are comparing it to eg constant speed ct

s = at2/2, as we said earlier, so 13.8 x  c4/2R2  billion light years.
@alancalverd Alan, did you mean c2/2R x 13.82 billion light years.

I’m still not sure how Smolin derived c2/R as being the acceleration.
Title: Re: Is there a discrepancy with the equivalence principle?
Post by: timey on 08/02/2018 15:06:11
So (assuming Smolins equation c^2/R is correct):

(c^2/R)t^2/2R= distance (ct)+extra distance travelled due to acceleration
(Where t is the age of the universe.)

And
(c^2/R)t^2/(2R)-R/R= 'a distance'
Title: Re: Is there a discrepancy with the equivalence principle?
Post by: alancalverd on 08/02/2018 15:38:27
Quote from: alancalverd on Today at 13:47:04

    s = at2/2, as we said earlier, so 13.8 x  c4/2R2  billion light years.

@alancalverd Alan, did you mean c2/2R x 13.82 billion light years.

yes indeed! Apologies.

Quote
I’m still not sure how Smolin derived c2/R as being the acceleration.

s = ½at2 (equation of motion)  R = ct  (definition of Schwarzchild radius) so at any time t, R = ct = ½at2, hence a = 2ct/t2 =2c/t or 2c2/R.

Did Smolin give a reason for dividing by 2? Or explain how to prevent a→∞ if R = 0?
Title: Re: Is there a discrepancy with the equivalence principle?
Post by: alancalverd on 08/02/2018 15:43:03
And
(c^2/R)t^2/(2R)-R/R= 'a distance'

No. R/R is dimensionless so you can't subtract it from 2R or anything with any dimension.
Title: Re: Is there a discrepancy with the equivalence principle?
Post by: timey on 08/02/2018 15:53:58
Smolin said that c^2/R = an acceleration that matches the acceleration that the universe's expansion (as per the expanding theory) is now found to be accelerating at. And this acceleration is now thought associated with the cosmological constant that Einstein retracted from his theory of general relativity.

...and It's not minus R/R, it's minus R =distance/R
Title: Re: Is there a discrepancy with the equivalence principle?
Post by: Colin2B on 08/02/2018 16:15:09
Did Smolin give a reason for dividing by 2? Or explain how to prevent a→∞ if R = 0?
I don't know. @timey found it in a book so I don't understand why it's double the calc
Title: Re: Is there a discrepancy with the equivalence principle?
Post by: timey on 08/02/2018 16:33:32
Smolin didn't mention anything about /2.

The only place /2 has entered into the discussion is via the /2R, that is in the equation that 'you' provided me as a means to calculate the extra distance travelled.
Title: Re: Is there a discrepancy with the equivalence principle?
Post by: alancalverd on 08/02/2018 17:52:58
...and It's not minus R/R, it's minus R =distance/R

That is completely meaningless!

Worth reminding you of the order in which expressions are calculated: BODMAS.

First, evaluate anything in Brackets

then "Of" e.g. "40% of ....." (you won't find this in most physics texts)

then Divisions.Multiplications, Additions and Subtractions, in that order

so ((4+5)/(1 + 4 -2)) x 6 + 2 = ((9)/(3)) x 6 +2 = 3 x 6 +2 = 18 + 2= 20. Note that some additions and subtractions were inside the brackets, so we tackled them first. Once we have evaluated the brackets and got rid of the division, the multiplication and addition were unequivocal.

So when proposing a calculation, first sort out your physics, then make sure your calcs are ordered by BODMAS, then check that the dimensions give you what you need  - speed,length, time, mass, whatever, without adding apples and pears. Finally, put in the numbers. Remember that the numbers don't matter much. Famous exchange from history:

Fred Hoyle  "And there we have the luminosity of the galaxy...... apart from a factor of 1026"

Student "Multiply or divide?"

Fred "Er...um....I don't think it affects the physics either way"
Title: Re: Is there a discrepancy with the equivalence principle?
Post by: timey on 08/02/2018 17:54:50
(Back on lap top now and can see much better what is going on, the phone is restricting)

Quote from: timey on Today at 13:58:22
So:

c^2t^2/2R= extra distance travelled due to acceleration?
(Where t is the age of the universe.)
No, total distance travelled, from rest, at a given acceleration of c^2/R
Extra distance due to acceleration depends what you are comparing it to eg constant speed ct

Oh... craps!  Well I don't want to calculate the distance of R travelled from rest.

I want to say that:distance R = radius of observable universe, travelled at speed c + the acceleration.
Then I want to subtract the distance travelled if travelling at c without the acceleration. ie: (ct) (where t is the age of the universe) from this c + the acceleration distance, leaving me with the distance travelled due to the acceleration.

Then I divide this distance by the distance (ct) and this will give me a small distance.

(edit corrections were made)
Title: Re: Is there a discrepancy with the equivalence principle?
Post by: jeffreyH on 08/02/2018 18:34:52
So:

c^2t^2/2R= extra distance travelled due to acceleration?
(Where t is the age of the universe.)
No, total distance travelled, from rest, at a given acceleration of c^2/R
Extra distance due to acceleration depends what you are comparing it to eg constant speed ct

s = at2/2, as we said earlier, so 13.8 x  c4/2R2  billion light years.
@alancalverd Alan, did you mean c2/2R x 13.82 billion light years.

I’m still not sure how Smolin derived c2/R as being the acceleration.

Well you have length^2/(time^2 x length) = c^2/R. That looks an awful lot like acceleration. Whether or not the value is valid is another matter entirely.
Title: Re: Is there a discrepancy with the equivalence principle?
Post by: timey on 09/02/2018 00:38:56
(Back on lap top now and can see much better what is going on, the phone is restricting)

Quote from: timey on Today at 13:58:22
So:

c^2t^2/2R= extra distance travelled due to acceleration?
(Where t is the age of the universe.)
No, total distance travelled, from rest, at a given acceleration of c^2/R
Extra distance due to acceleration depends what you are comparing it to eg constant speed ct

Oh... craps!  Well I don't want to calculate the distance of R travelled from rest.

I want to say that:distance R = radius of observable universe, travelled at speed c + the acceleration.
Then I want to subtract the distance travelled if travelling at c without the acceleration. ie: (ct) (where t is the age of the universe) from this c + the acceleration distance, leaving me with the distance travelled due to the acceleration.

Then I divide this distance by the distance (ct) and this will give me a small distance.

(edit corrections were made)

So is that going to be:
(R/t+c^2/R)xt^2/(2R)-R= distance travelled due to c^2/R  ?
where R is the radius of the observable universe, and t is the age of the universe.

Or is it:
R/t+(c^2/Rxt^2)/(2R)-R=distance travelled due to c^2/R  ?

Or on the basis that
s = ½at^2

is it:

½(c^2/R)t^2=distance travelled due to c^2/R  ?

And if not, then please, how do I notate it?
Title: Re: Is there a discrepancy with the equivalence principle?
Post by: timey on 09/02/2018 07:14:53
@alancalverd
Me: Do you know the way to the shop?
You: Yes
Me: Could you give me directions please?
You: Certainly. Buy a map and apply map reading skills.
Me: To buy a map, clearly I'd need to get to the shop.  Can't you just tell me how to get there?

On the basis that I think it important that people receive feedback on personal interactions I will now tell you that I have not slept a wink, feel quite ill with frustration, and have been crying a lot.  I do not understand your purpose in being glib, or continuously bring up expanding universe theory, when I have told you my consideration is contracting.
In giving you this feedback, I ask you if there is potential for a change in narrative, where it is my hope that your intention is not to be a deconstructive force, or cause upset.

@Colin2B
Having given the matter about 9 hours thought now, if that equation c^2t^2/2R = distance, and ct=R, then distance=c^2t^2/2R has to be the extra distance travelled due to the acceleration, in addition to distance R, doesn't it?
Title: Re: Is there a discrepancy with the equivalence principle?
Post by: alancalverd on 09/02/2018 07:49:04
My only concern is to make sure that your hypothesis is summarised in equations that make sense, so that others can see your point. The first thing any physicist will (or should) do when faced with a novel equation is to ask whether it is dimensionally balanced. If not, there may be a typo or a misunderstanding, and he won't be able to discern the author's intentions. So let's get the typos out of the way and ensure that your ideas are expressed unequivocally.

We have given you the basic equations of motion in standard form. We agree that R =ct by definition. I am unclear about the proposed value of "the acceleration" - where does this come from? And what exists at this "extra distance"? Happy to have a reference to an earlier post if you have specified these.

Late edit: I have now picked up the reference that Smolin says the observed acceleration is c2/R. Mea culpa.

So at time t + Δt we have R + ΔR = R + c2(Δt)2/2R +vΔt where v is the "initial" velocity at t

Hence ΔR = c2(Δt)2/2R + Δt ∫(c2t/R) t (I think!)

Will give it more thought later - off to work!
Title: Re: Is there a discrepancy with the equivalence principle?
Post by: Colin2B on 09/02/2018 10:33:22
(Back on lap top now and can see much better what is going on, the phone is restricting)
....
I want to say that:distance R = radius of observable universe, travelled at speed c + the acceleration.
Then I want to subtract the distance travelled if travelling at c without the acceleration. ie: (ct) (where t is the age of the universe) from this c + the acceleration distance, leaving me with the distance travelled due to the acceleration.

Then I divide this distance by the distance (ct) and this will give me a small distance.
Ok, good you can see the formulae clearly.
First your last line, dividing a distance by a distance doesn’t leave you with a distance so that is out.*

So “I want to say that:distance R = radius of observable universe, travelled at speed c + the acceleration.”
Problem is that we know R=ct, so how can  R= ct+ dist due to acceleration ?

This leads me to believe that the question you are asking might not be the one you want answered. I wonder if you are really wanting to ask how you convert an observed expansion into a nonexpansion, presumably by using an adjustment to observed time?
Is it worth us following through how you get to the correction to the field equations first? Or do you want to try and work this through first?

As ive said before, and @alancalverd  seems to have the same objective, we only want to make sure your ideas are presented in the best possible way and if we allow mistakes like * to appear in the paper your readers will walk away at that point.
It’s a painful process, i know ive been there, but it’s worth it.

EDIT:  just noticed Alan modified his post while i was writing. What he says about Smolin’s acceleration now makes sense, something to work with. Out today, will look later.
Title: Re: Is there a discrepancy with the equivalence principle?
Post by: opportunity on 09/02/2018 11:22:18
Apologies for doing this, but I'm using Ockham's Razor here.

Running a clock, any mechanism, to out-pace light from a defined source; maybe that's why light doesn't carry mass naturally, because the laws of space-time we're investigating disallow such? Nonetheless, to try that, carry mass at light speed, obviously the mass in question I'm thinking will try to be more light-like, and decay, "or" be bent from that trajectory of light its trying to out-pace? If any measuring instrument being used in that context is held as gold, no effect on its initial workings, I'm thinking maybe the experiment is already contaminated with the hyped-up process of natural decay of mass in play, interfering with the normal process of the designed atomic clock, slowing it?

Once again, layman at the helm here given the number of posts. But, if light is what it is without mass and we try to use mass as light will not nature intervene, a nature we haven't properly scientifically understood?
Title: Re: Is there a discrepancy with the equivalence principle?
Post by: timey on 09/02/2018 13:27:09
OK, (feeling better now btw).  I think I see where the confusion is.

I am not thinking upon R as being inclusive of (a) for this equation.
The observation is c times t.
ct=R
So
a=c^2/R
(c+a)t=R+distance extra travelled due to acceleration.
And I am looking to then minus R and 'do stuff' with the extra distance travelled.

'Do stuff' like:
What percentage of R is this extra distance?
If I divide this extra distance by R? (Like one might divide 10 miles, by 2 miles and end up with 5 x 2 mile markers)
If I divide this extra distance by R and then c, and then t (age of universe)? (Like one might divide 10 miles by 2 mile markers, and end up with 5 x 2 mile markers, where it takes time t to travel 2 miles.)

Start figuring out how to say that 13.8 billion years ago as per our clock rate at this moment in time, time ran x percentage slower, and is universally speeding up as the clumping process continues, where this speeding up of time is associated with the given acceleration.

And then start looking at the discrepancy between the 'recessional velocity' of the CMB and the 'recessional velocity' of galaxy clusters. (I have highlighted 'recessional velocity' b/c in my model this is not a discrepancy between recessional velocities, but simply an anomaly between redshifts observations that is predicted by the model)
Title: Re: Is there a discrepancy with the equivalence principle?
Post by: alancalverd on 10/02/2018 10:15:32
(c+a)t=R+distance extra travelled due to acceleration.


No. The "extra distance travelled" in a time interval  Δt is vΔt + a(Δt)2/2 where v is the initial velocity at time t.

The problem is that we don't have an expression for v that stands up to analysis at t=0, so we can't solve the equation theoretically*. So do we have any actual observed value for v(now)?


*hence the pigs-ear integral I proposed yesterday!
Title: Re: Is there a discrepancy with the equivalence principle?
Post by: alancalverd on 10/02/2018 10:19:16
PS if there really was a primal big bang that resulted in the dispersal of mass from a huge black hole, then time certainly did run slower in the infant universe - isn't that the consequence of everyday relativity?
Title: Re: Is there a discrepancy with the equivalence principle?
Post by: Colin2B on 10/02/2018 11:22:50
The problem is that we don't have an expression for v that stands up to analysis at t=0, so we can't solve the equation theoretically*. So do we have any actual observed value for v(now)?
*hence the pigs-ear integral I proposed yesterday!
I did post a value some time back https://www.thenakedscientists.com/forum/index.php?topic=69800.msg532208#msg532208

Quote
Why not use something like this from measurements in 2012
“expanding at a rate of 74.3 plus or minus 2.1 kilometers (46.2 plus or minus 1.3 miles) per second per megaparsec (a megaparsec is roughly 3 million light-years).”

PS, I didn't think the integral was too pig eared (I assumed dt) and looked like a good starter. Question might be whether variation of v is constant over time, Also @timey is suggesting light red/blue shift is affected by path through g anomalies. Probably best to start with a simple assumption and see if magnitude is approx ok.
Title: Re: Is there a discrepancy with the equivalence principle?
Post by: alancalverd on 10/02/2018 12:11:07
You have put your finger on part of the problem. If, by observation, we know that the expansion is accelerating now, then our best guess is that v has not been constant over time past!

So let's take the current rate of expansion v = 28 km/sec per million lightyears and multiply by the current value of R, say 14 000 000 000 lightyears, gives v at the periphery of the observable universe as 28 000 x 14 000 = 392 000 000 m/s which, sadly, is faster than c. This does not make sense: it implies that there is an infinite amount of energy available to accelerate every particle that we can't see!

Time for some beer and rugby.
Title: Re: Is there a discrepancy with the equivalence principle?
Post by: Colin2B on 10/02/2018 12:55:25
Time for some beer and rugby.
Are you participating in rugby or watching? I can guess for the beer.
Raining here so excellent rugby weather, enjoy  :)
Title: Re: Is there a discrepancy with the equivalence principle?
Post by: timey on 10/02/2018 14:32:44
You guys are still trying to fit expanding theory to my request for help with an equation for a contracting model.

There is no integral expanding velocity in my model. Everything already expanded in my model's inflation period, and has been contracting at a=c^2/R for 13.8 billion years. (By the rate of our clock at this moment in time)

There has never been a time=0 in my model.
Time will be running slow at end of inflation period where mass (in particle form) and energy are at maximum dispersement.
Time will be running at it's fastest just before inflation period when all mass and energy (in black hole form) are at minimum dispersement.
And time will be running at it's slowest in the open spaces between galaxy clusters, that are made up of galaxies. Galaxies that are converging inwards upon each other.

This is why my model makes the prediction, that while clocks in elevation to mass will tick faster than clocks at lesser elevation, that if placing 2 clocks in different locations at same longitude (to equalise centripetal speed), and same height from centre of earth, (so that both clocks are equal), but the locations are of know significant difference in geological density - that contrary to general relativity my model predicts that the clock in the denser location will tick faster.
This is a doable experiment when NIST get their portable clocks up and running.

The narrative out there on the theoretical physics circuit is that no existing theories of everything can make testable predictions for anything different. And that observations like the discrepancy between CMB and galaxy clusters 'recessional velocity' cannot be explained by any existing theory.
My model and theory of everything does both...

So no - time=0 doesn't exist in the equation, so nothing to be theoretically unexplained there.  And introducing recessional velocity is an 'expanding theory' consideration that will not apply to a contracting model.

Hope this clears up the points that you both have made.
Title: Re: Is there a discrepancy with the equivalence principle?
Post by: Colin2B on 10/02/2018 16:00:56
You guys are still trying to fit expanding theory to my request for help with an equation for a contracting model.
No, just looking at the fact your model still has to result in what is currently observed, accepting that your interpretation of what is observed will be different.

Thanks for the clarification of which stage your universe is at currently, i hadn’t picked that out before.

So, if you dont want us looking at the expansion, what do you mean by “extra distance travelled”. Please clarify in terms of the contracting model you have just outlined.
In your contracting model we could take t=0 as being 13.8bn yrs ago, this would be start point for the contraction, which i assume is accelerating from 0 velocity at the turnaround point.

I don’t think the formulae we have been using will work if time is not constant over the period. Do you have a model for how time varies between start of deflation and current time?
Title: Re: Is there a discrepancy with the equivalence principle?
Post by: alancalverd on 10/02/2018 16:14:29
if placing 2 clocks in different locations at same longitude (to equalise centripetal speed), and same height from centre of earth, (so that both clocks are equal), but the locations are of know significant difference in geological density - that contrary to general relativity my model predicts that the clock in the denser location will tick faster.

What does the clock know about its environment? Only g
Title: Re: Is there a discrepancy with the equivalence principle?
Post by: timey on 10/02/2018 16:59:23
OK, to start with it will be useful at all times to remember that 'observation' is relativistic.
(This being what makes these considerations 'background independent'. An important feature in a theory of everything)

The 'observation' is c times t. (where t is somewhat dubiously based on parallax and standard candles)
There is no consideration here where t=0 has any actual physical meaning with regards to the equation.

The velocity of the contraction of the entire universe starts from 0 at c^2/R.
(And this measurement of acceleration will be proportional from any location in the universe, and also at any point in the history of the universe, because c is 299 792 458 metres per second of the rate of time at that point in history.)
But the contraction mechanism is the gravitational clumping process, so light arriving at our location as a phenomenon that is part and parcel of the universes clumping process cannot be associated with any t=0 concept.

Yes I can tell you the process by which time changes.
At start of contraction (clumping process) time is slow but more or less uniformish across entire universe, matching the more or less uniformish distribution of particles and energy.
Where there is more density in the slight un-uniformity, time runs a bit faster than in the less dense areas...
By a process in which particles that are satellite to a greater mass are increased in their internal timing, they are electro magnetically drawn in the direction of the greater mass, and the faster rates of time surrounding the greater mass accelerate this gravitational attraction. ie: gravity @alancalverd.
(My modification to general relativity is adding this third aspect of time dilation to describe gravitational acceleration, and adding these 2 axioms: "+energy = shorter seconds" and "The speed of light cannot exceed the local rate of time")

My model states the universal contraction process proceeds from 0 clumping, and accelerates at c^2/R.

The extra distance 'could' be used to describe by how much the universe has contracted.  But not until it is know (I have figured out) by how much the universal rate of time has increased.
This being where the observations of redshifts anomaly between CMB and galaxy clusters at 8% difference will come in useful.
Title: Re: Is there a discrepancy with the equivalence principle?
Post by: Colin2B on 10/02/2018 17:37:29
There is no consideration here where t=0 has any actual physical meaning with regards to the equation.
But there is. t in " c times t" is the elapsed time from t=0, the start of the contraction

The velocity of the contraction of the entire universe starts from 0 at c^2/R.
Problem with this is that  c^2/R is from Smolin's view of an expanding universe. what makes you think it is now contracting at that rate?

The extra distance 'could' be used to describe by how much the universe has contracted.  But not until it is know (I have figured out) by how much the universal rate of time has increased.
It should really be a reduced distance if the universe is contracting.

EDIT: PS
You are saying that the outer edge of universe has been contracting for 13.8bn yrs, and is now at a distance of 13.8bn yrs, so it must have gone out to 27.6 bn yrs. How have you calculated that?
Title: Re: Is there a discrepancy with the equivalence principle?
Post by: timey on 10/02/2018 18:50:40
c times t is not the start time of the contraction though.
c times t is our 'current' observation of a universe that has already contracted by a certain degree.

c^2/R matches the acceleration that is thought to occur as per 'observation' of redshifts.
It is the interpretation of those redshifts that results in expansion, (that GR descibes) and more recently, accelerated expansion. (That GR plus a=c^2/R describes)
My model's interpretation of the observation of redshifts is a contraction (that GR can describe) plus an acceleration (that a contracting GR plus a=c^2/R describes)

Yes, the distance R that we observe as (ct), 'is' the reduced measurement for this equation. The idea is to work out how much bigger it was.
(8% maybe? But suspect more complicated, still thinking on)
Where the rate of universal changes in rate of time, (as opposed to individual masses differing rates of time changing in the gravity potential, and the inverse changes to the rates of time (that I added) in the background spaces between masses - where both will also be increasing at this rate), is also accelerating at a rate associated with c^2/R.
(Where our 'working with' value of c^2/R, is as per our present time clock rate, and current observation of R)

In reply to your edit:
That would be the case, apart from the fact we are calculating in 'our' years, and I am saying that time has been speeding up at a rate associated with c^2/R. So in slower time, as per comparison to the rate of our clock now, c would not cover 'as much distance as we would associate with 27.6 bn yrs', in these ever increasingly slower rates of time as we go back in the history.
However it is worth me stating at this point that my model will calculate the universe as being a hell of a lot older that 13.8 billion years, when it is considered how these slower rates of time convert into an equivalent number of years as per our current rate of time.
Title: Re: Is there a discrepancy with the equivalence principle?
Post by: alancalverd on 10/02/2018 19:46:12
The 'observation' is c times t.

No. Light travels at c, so the the distance of any object you can see must be cτ where τ is the time it took for the light to travel from there to here. If the universe has a finite age t then the farthest observable object is at R = ct,the Schwarzchild radius.

R is important as long as c is the limiting speed for the propagation of gravity, which seems to be the case.Nothing outside of R can affect anything inside. Or can it?

Imagine a static universe. If there is a massive body at R + ΔR, it will have a gravitational effect at R such that photons emitted towards us from a body at R will appear to have come from a more massive body and thus be redshifted.Which is what we observe!
Title: Re: Is there a discrepancy with the equivalence principle?
Post by: timey on 10/02/2018 20:40:22
But 'time' is dynamical and changing in relativity Alan.

... And in each reference frame of gravity potential light is measured as being 299 792 458 metres per 'second', and the 'far away' clock will confirm that there is a rate of time difference between 1 reference frame's clock compared to another reference frame's clock.

Therefore the speed of light when in one reference frame of gravity potential is travelling at a different proportion of speed per second, due to the 'length' of the second there, than it is in the next reference frame of gravity potential that has a different length of second.

...and as a result c times t in the gravity potential is complicated.

R is what we can see, where it is accepted physics theory that there is more universe that we can't see.
It is part and parcel of accepted physics theory that this is the 'observable universe' and the fact of Hubbles constant and the expansion theory requires that the outer edges of the 'observable universe' are expanding at an accelerated rate that exceeds the speed of light.  Where c is held relative to our 'length' of second.
(Experiment: Make c relative to a longer second than ours, compare it to a measurement of c as per our second with regards to distance travelled.)

"Imagine a static universe"...
Yes, that is correct... and if masses were starting out smaller and becoming bigger, then redshift is what we would observe.

How are they getting bigger?
Via the clumping process.
What drives the clumping process?
Gravity
What is gravity?
GR answer:
Space tells mass how to move and mass tells space how to bend (as a spacial distortion of geometry)
My model's answer:
Mass tells space what it's rates of time are. Mass also tells satelite masses what their rates of time are.  And these changes in both rates of time that ocurr as seperate issues, but concurrently with each other, cause mass to move the way they do in the gravity potential of the greater mass.

Why do we observe accelerated redshifts?
GR+positive cosmological constant answer:
Because the universes expansion (described by GR without cosmological constant) is accelerating at c^2/R.
My model + negative cosmological constant answer:Because the universes clumping process is causing a contraction, (that can be described by GR without cosmological constant), that is accelerating at c^2/R.
Title: Re: Is there a discrepancy with the equivalence principle?
Post by: alancalverd on 10/02/2018 23:56:28
Whatever the complication, R = c∫dt where the integral of time is from the birth of the universe to now. It may well be that the Schwarzchild "radius" is actually a bit bumpy, but space seems to be mostly empty and the universe sufficiently isotropic that the observation limit can be considered a sphere for all practical purposes.

 
Quote
And in each reference frame of gravity potential light is measured as being 299 792 458 metres per 'second'
No. The second is defined everywhere by an atomic transition and the metre is defined everywhere as 1/299 792 458 of the distance light travels in a second.

Quote
Mass tells space what it's rates of time are.
But we know by (very precise) experiment that it is the combination of mass and distance, i.e. gravitational potential,  that determines relative clock rates.
Title: Re: Is there a discrepancy with the equivalence principle?
Post by: timey on 11/02/2018 00:36:31
The description you put forward hasn't defined what the birth of the universe is, and if you say Big Bang, this is not a defined mechanism in currently held physics theories.
Further more, space seems to be mostly empty, but the description doesn't give the mechanics of 'why' it is empty, other than an unknown event caused an inflation period of unknown mechanics that slowed down, and then speeded up again as accelerated expansion for an unknown reason, while the universe developed into what we see today with regards to mass clumping.

The second is defined everywhere by an atomic transition.
But the atomic transition of the 'far away clock' will tell you that the atomic transitions in one observed reference frame are not only different from it's own, but that another reference frame close to, but at a different gravity potential to the first reference frame, has different atomic transitions to the first reference frame.

And to reply to your edit:
I said:
"Mass tells space what it's rates of time are."
You said
"But we know by (very precise) experiment that it is the combination of mass and distance, i.e. gravitational potential,  that determines relative clock rates."
Yes, that is gravitational mass telling satellite mass what it's time is. (in my model)
'Mass telling space what it's rates of time are' is me referring to my models added time for space between masses.  The changes of which are inversely proportional to the changes in the rate of time for clocks held at different elevations from mass.
And this added time phenomenon occurs concurrently with the timing of clocks in the gravity potential, and can describe a physical reason for gravitational acceleration of objects falling towards gravitational mass.
Title: Re: Is there a discrepancy with the equivalence principle?
Post by: alancalverd on 11/02/2018 00:57:51
The description you put forward hasn't defined what the birth of the universe is, and if you say Big Bang, this is not a defined mechanism in currently held physics theories.
It doesn't matter! If any or all stuff has been moving away from you for time t, the furthest stuff you can see is at ct.


Quote
Further more, space seems to be mostly empty, but the description doesn't give the mechanics of 'why' it is empty,
It is mostly empty because there is more space than stuff, and stuff tends to coalesce in clumps as a result of gravitation.
Title: Re: Is there a discrepancy with the equivalence principle?
Post by: timey on 11/02/2018 03:00:01
Quote Alan:
"If any or all stuff has been moving away from you for time t, the furthest stuff you can see is at ct."

But in my model stuff has not been moving away from us for time t.
Small bits of stuff have been moving towards other small bits of stuff, making bigger bits of stuff, for more years than the galaxies that emit the light we observe have existed for. And c times t is a measure of the observation of galaxies that are formed already, and are estimated to be x distance away from us.

The light is coming from galaxies of galaxy clusters, where the galaxies of those clusters are converging.
The galaxies of our galaxy cluster are moving away from other galaxy clusters, and closer to other galaxies of our galaxy cluster.  Other galaxies of other galaxy clusters are moving closer to the other galaxies of their own galaxy clusters.
But this is a type of back reaction recessional value, and there is no need to say that everything is moving apart, just that what is there is being drawn together, by gravity, at an accelerating rate.

Quote Alan:
" It is mostly empty because there is more space than stuff, and stuff tends to coalesce in clumps as a result of gravitation."

You have not described why there is more space than stuff, nor why stuff coalesces into clumps as a result of gravitation.

It's all about 'initial conditions' Alan.
My initial conditions for a contraction are a more or less uniform sea of energy and particles.  Spaces in between masses are caused by particles vacating their former positions in this sea to the clumping process called gravitation that is described in my model as the changes in rate of time for masses in the gravity potential, and the changes in the rates of time my model adds for the spaces between masses.

In any case I know that for my model the equations that I am seeking are valid, and that c times t can only be an observation of light from a galaxy x distance away, and that the age of the universe is based, in part, on redshifts observations and Hubble's constant, so despite the fact that my model will calculate that the universe is much older, I can still use the current theory age to calculate an observation of ct.
In physical cosmology, the age of the universe is the time elapsed since the Big Bang.
In my model the age of the universe is also the time elapsed since the Big Bang.  Except that  in my model the Big Bang and Inflation period occurred really quickly, and the contraction period kicked in after inflation by the physics of gravitational clumping, where the overall contraction of the universe occurs at an acceleration of c^2/R.

My model makes a better description of observations. It also makes a falsifiable prediction for something different to current theory, which is what all other theories of everything fail to do.
So is there any chance I could get a hand with formally describing the equation that will describe (ca)t-(ct)=extra distance travelled, and how to work out what percentage of (ct) this extra distance is?
Title: Re: Is there a discrepancy with the equivalence principle?
Post by: alancalverd on 11/02/2018 08:24:02
My initial conditions for a contraction are a more or less uniform sea of energy and particles.  Spaces in between masses are caused by particles vacating their former positions in this sea to the clumping process called gravitation

That is an entirely reasonable initial condition and a lot easier to work with than a big bang. It's familiar physics as fog and rain formation, and if we really understood it, we could even predict  snowflakes. Problem is that it still doesn't answer "why" the sea of particles existed in the first place. So you have to introduce a big bang to produce the uniformish sea, but that just moves "why" back a few microseconds and adds another unanswered question.

That said, all our models of .planetary formation seem to point to the clumping of heavy nuclides, and our best guess as to their origin is from collapsing clouds of light ones, so matter becomes its own hoover over time, resulting in more space than stuff, as observed.

So far, so good. And you might expect that near objects will indeed be clumping. But if we draw a "sphere of influence" around a gravitational attractor in a sparsely-filled universe, at some  point there will be more matter outside that sphere than inside, so some distant objects will be receding towards another attractor. We can even define the radius beyond which it is more likely that stuff is moving away. Let out local attractor have mass M

Then M/ρ2 = ∑ρm/r2

where m,r is every particle in the universe outside the critical radius ρ. If the universe has a finite age, just substitute ct for ∞.

Now you have a clumping universe with redshifts increasing with distance, using only known physics, even if we haven't answered "why". 


Title: Re: Is there a discrepancy with the equivalence principle?
Post by: Colin2B on 11/02/2018 10:07:33
@timey
“So is there any chance I could get a hand with formally describing the equation that will describe (ca)t-(ct)=extra distance travelled, and how to work out what percentage of (ct) this extra distance is?
....the overall contraction of the universe occurs at an acceleration of c^2/R.”

I dont think this is going to help. These are values for expansion and give odd results.
Accelerating contraction starting at v & t = 0 is c^2t^2/2R
 Subtract ct = c^2t^2/2R - ct
Substitute ct for R as before:
ct/2 - ct so we can see dist due to acceleration is 50% of ct
There is no difference in time because t is same for both equations.

Would be best to work with Alan’s idea which gives a logical reason for clumping and what we see as red shift, his equations are more complicated but more likely to give consistent results. 

But you need to determine the changes of time. Eg Im still not clear why you get time running fast nr a black hole and slow far from mass. You must have worked out the logic of that to be able to make that statement. If you can explain it, perhaps we can describe it in maths.
 


Title: Re: Is there a discrepancy with the equivalence principle?
Post by: timey on 11/02/2018 13:12:32
@alancalverd.
The trouble is is that if you now draw a sphere around any part of my model, NOTHING will end up outside of that sphere, b/c my model is CONTRACTING.

In my model:
The 'initial conditions' for contraction are an almost uniform sea of individual particles and energy.
The 'initial conditions' for inflation is a singular black hole that contains that entire sea of particles and energy.
The inflation period is the singular black hole exploding 'particles and energy' via superluminal jets into a 'sea of particles and energy'.
And the contraction period is very slowly accelerating, where all development of particles into what we see today occurs within the contraction.

Draw a sphere around the edge of this 'sea of particles'.  (Edit correction: actually light would not be emitted by a sea of particles, but you can get the idea).  In 13.8 billion of our rate of time years we will see that nothing has ended up outside the outer edges of that sphere, and in fact we can now draw a smaller sphere around the outer edges b/c of the contraction.
This means that an observation that arrives at us from c times (t=13.8billion years) started off in a universe that was as big as the first sphere we drew, and arrived to be observed by us in a universe that is the size of the second smaller sphere.

@Colin2B
My model states that atomic transitions are caused by energy, and that changes in atomic transitions are caused by changes energy.
My model also states that changes in energy are occurring with changes in gravity potential, and that changes in time are associated with changes in energy, where +energy=shorter seconds.
The bigger mass 'has' more energy, and will have more energy associated with it's gravity potential.
That is why my model makes the prediction it does for the given experiment.

So are you saying that the extra distance under the construct I suggest is 50% of R?
Title: Re: Is there a discrepancy with the equivalence principle?
Post by: alancalverd on 11/02/2018 14:41:37
Draw a sphere around the edge of this 'sea of particles'.  (Edit correction: actually light would not be emitted by a sea of particles, but you can get the idea).  In 13.8 billion of our rate of time years we will see that nothing has ended up outside the outer edges of that sphere, and in fact we can now draw a smaller sphere around the outer edges b/c of the contraction.
This means that an observation that arrives at us from c times (t=13.8billion years) started off in a universe that was as big as the first sphere we drew, and arrived to be observed by us in a universe that is the size of the second smaller sphere.

A very neat description.

The smaller universe being denser, this means that g is increasing with time, so light coming from the most distant objects will be blue-shifted (a) by the Pound-Rebka effect and (b) by the Doppler effect.
Title: Re: Is there a discrepancy with the equivalence principle?
Post by: timey on 11/02/2018 17:32:02
Well you are on the right track Alan.

The potential for g in the smaller universe sphere will be the same as the potential for g in the 'sea of particles and energy' sphere, only packed into a smaller space.

BUT - It is the fact of the increasing un-uniformity of gravity potential caused by the clumping process that causes the contraction.
In the bigger universe sphere, the potential for gravity was more or less uniformly distributed.
In the smaller sphere universe this is not the case. There is now disparity in the potential of g between objects of mass, and the spaces between these objects of mass.
The gravity potential within the contracting sphere is continuously becoming more concentrated into small areas of high density, and large areas of low density.

So light that is emitted 13.8 billion years ago by objects of mass that formed from the 'sea of particles and energy', will have to travel across spaces where the potential for g is much reduced, and is still reducing as mass further clumps.

So the universal contraction process causes an increase in the potential for g per unit of area, (as per a smaller sphere universe), but the contraction process itself causes the contrast between the potential, of g 'for mass' to increase, while the potential for g in the 'open spaces' is descreased.

This will cause an anomoly between the redshift observations of the CMB and the redshift observations of galaxy clusters.
The CMB is subject to the decrease in g caused by clumping, and the increase in g caused by the universal contraction.
The light emitted from the galaxy clusters is also subject to the decrease in g caused by clumping, and the increase in g caused by the universal contraction.
And the galaxies of galaxy clusters, themselves, are moving at velocity towards points of convergence.

The effects of the increase in g caused by universal contraction will be far lesser than the effects of the decrease of g in the open spaces between masses caused by contraction, and the 'observations' will be redshifts, where the CMB redshifts will be of a lesser magnitude than the galaxy cluster redshifts.
Title: Re: Is there a discrepancy with the equivalence principle?
Post by: Colin2B on 11/02/2018 23:53:38
My model states that atomic transitions are caused by energy, and that changes in atomic transitions are caused by changes energy.
My model also states that changes in energy are occurring with changes in gravity potential, and that changes in time are associated with changes in energy, where +energy=shorter seconds.
The bigger mass 'has' more energy, and will have more energy associated with it's gravity potential.
That is why my model makes the prediction it does for the given experiment.
Presumably somewhere in your write up you explain why this eventually gives the same result as observation ie black hole, slower time.

So are you saying that the extra distance under the construct I suggest is 50% of R?
What this says is that if your universe started contracting from v=0, at t=0, has continued contracting for time t at an acceleration of c^2/R, then it will have contracted a distance half that covered by light in the same time t.
Problem with this is that if acceleration is = c^2/R then at t=0 we assume a larger R hence a smaller acceleration. Acceleration would then increase until current radius. This is why Alan put an integral into his formula. Without bothering to try and work this out (because we dont know if change of acceleration is constant) it does tell us that the contraction will be less than if it was at c^2/R for the whole time. So use this understanding it’s limitations.
There is another problem with using this acceleration, it is based on an expanding universe (which you said you didn’t want to use) so we dont know if the same factors would apply eg an expanding universe such as ours will not move towards a uniform sea of particles.

Separate question. You talk about light passing through areas of higher or lower density,  what effect does that have on the light between leaving a star and reaching our detector on earth?
Title: Re: Is there a discrepancy with the equivalence principle?
Post by: timey on 12/02/2018 00:55:11
No - I specifically have stated that my prediction for a doable experiment states that the clock in the denser location/or on the bigger mass will run faster, and that this is the 'different prediction' that my model makes as per my modification of GR. 
Black holes in my model do not have slower time, and in physical reality, are not actually observed to have slower time, or even observed much at-all.
Given that a clock could be held in elevation to a black hole, it's time would still run faster than that of the clock placed closer to the black hole, but both the clocks placed in proximity to the black hole would be running faster than a clock on, or near Earth.

And:

No we can't assume any larger R.  The observation is of light, and light would not be present as such at the earlier larger R.
The age of the universe has been worked out in large part via the redshift observations and Hubbles constant, and more recently from the CMB data.  The other part being a consideration regarding a lower limit via estimated star/mass formation process.
My model also uses the resdshift observations to estimate the age of the universe.
So an acceleration that is based on the redshift observations indicating acceleration (under expanding theory) that matches c^2/R, where R is c times the current estimated age of the universe that has been calculated via these redshift observations, can be used for the purpose of my calculation of a contraction.

However, b/c my model interprets the redshifts differently, I then add my alterations to the result.

So - if the extra distance travelled due to c^2/R, where R=c times age of universe is 50% of R, let's put that to one side.
Now it must be considered that the rate of time 13.8 billion years ago was a percentage slower then than it is now, as compared to our rate of time on our clock.
The rate of time will have increased by the same rate as the universal contraction of the universe accelerating at c^2/R.
So how do I get to that equation?
*

That's right. An accelerated expanding universe, such as in expanding theory, moves towards a big freeze.
*

The places that the light is emitted from ie: mass, are the high density areas of g, and the spaces inbetween the mass are the low density areas of g.
The effect on light moving into low density areas of g in spaces between mass are redshifts.

Time runs faster in high density g, than in low density g, apart from for the m located inside of the gravity potential of the greater Mass. Where infact these changes in time, in the manner that I suggest, can give a physical description of the effects of g.
Title: Re: Is there a discrepancy with the equivalence principle?
Post by: Colin2B on 12/02/2018 14:46:49
No - I specifically have stated that my prediction for a doable experiment states that the clock in the denser location/or on the bigger mass will run faster, and that this is the 'different prediction' that my model makes as per my modification of GR. 
Yes, you have stated that in paper and video, but have not explained why.
However, i was not aware of this bit:
Black holes in my model do not have slower time, and in physical reality, are not actually observed to have slower time, or even observed much at-all.
Given that a clock could be held in elevation to a black hole, it's time would still run faster than that of the clock placed closer to the black hole, but both the clocks placed in proximity to the black hole would be running faster than a clock on, or near Earth.
So if you place 2 clocks at different elevations above earth the higher clock will run faster, but if you place 2 clocks on the moon they will run slower than the ones on earth?
This will mean a very different relationship, if any, between gravitational potential and time compared to current theory.

No we can't assume any larger R.  The observation is of light, and light would not be present as such at the earlier larger R.
So why are you comparing contraction with a light beam emitted from a larger R?

Now it must be considered that the rate of time 13.8 billion years ago was a percentage slower then than it is now, as compared to our rate of time on our clock.
The rate of time will have increased by the same rate as the universal contraction of the universe accelerating at c^2/R.
So how do I get to that equation?
If you assume they are in the same proportion you could assume the clock rate has doubled.
Title: Re: Is there a discrepancy with the equivalence principle?
Post by: timey on 12/02/2018 15:58:54
The reason why is associated with adding the separate time phenomenon for open space, and stating that the time phenomenon of clocks ticking faster when placed at elevation is a) part and parcel of a time drift associated with motion as well as position in gravity potential
And
b) has nothing to do with what is occurring for the background space at that gravity potential the clock is occupying.
*

No - the moon is satellite m to earth gravitational M.  The earth is satellite m to sun, and the sun is satellite m to centre of galaxy black hole M.
But the difference with the sun being satellite m to black hole M is that solar system m is of greater g than gravity potential g of black hole M at this distance from black hole M. (leading to galaxy rotational considerations)
*

Let's me rephrase.
A larger R is assumed for the initial conditions of the 'sea of particles and energy', the value of which is unknown.
There is a lower limit of 11billion years for the development of light producing mass (edit correction: for the development of the mass we see today) that might be of use here.
What has been derived from observations of redshifted light is an age for the universe, (based on speed of expansion) and a value for acceleration, which I can use, so long as I make my alterations to the result.
So getting a measurement based on a point in time from when mass was developed enough to produce light is the place to start, based on the fact that the observations of redshifts are the basis of the considerations.
*

Yes one could assume that the rate of time had doubled. :)
So if the rate of time was half as fast at t=0, than it is at t=13.8 billion years later, this being now, then the extra distance travelled due to the acceleration would be ? percentage shorter than 50%.
Title: Re: Is there a discrepancy with the equivalence principle?
Post by: Colin2B on 12/02/2018 16:53:04
Still 50% because they are both subject to the same amount of time t.
Title: Re: Is there a discrepancy with the equivalence principle?
Post by: guest39538 on 12/02/2018 17:08:23
The reason why is associated with adding the separate time phenomenon for open space, and stating that the time phenomenon of clocks ticking faster when placed at elevation is a) part and parcel of a time drift associated with motion as well as position in gravity potential
And
b) has nothing to do with what is occurring for the background space at that gravity potential the clock is occupying.
Time moves with the object , the object does not travel a distance in an amount of time.  You are correct in a time that is independent of the object, however Newton a long time ago beat us on that one and already had absolute time.   
Understand the twins do age differently but experience the same amount of absolute time.   Understand the dilation is due to field density and velocity and not a consequence of space. 
How far we see has nothing to with the speed of light, the  inverse of light and the visual dimensional transverse of objects that are receding, are how far we see.

Title: Re: Is there a discrepancy with the equivalence principle?
Post by: timey on 12/02/2018 18:35:17
@Colin2B
We made the calculation for acceleration using the value of c as per our clock rate now.
And we made the calculation for, not 'the' extra distance travelled, but a value of distance to work with, using the value of t=age universe on the basis of c as per the rate of our clock, times t.

BUT - between 13.8 billion years ago, and now, time was 50% slower and accelerated to the rate it is now.  So using c times t needs to be corrected, b/c we have held c relative to the rate of our clock.
Where the measurement of a 50% distance of contraction in t=13.8billion years, is our rate of c, times the t=13.8 billion years of our rate of time.

So, on the basis that the speed of light is constant in any reference frame, including the reference frames of our history, if the speed of light covers 299 792 458 metres per 50% longer length of second than ours 13.8 billion years ago, then our shorter second in comparison, (13.8 billion years ago) would only cover half of 299 799 458 metres, and at every point in time after would cover a little more than half of 299 799 458 metres, up until present time where the speed of light is held relative to our clock and the full 299 799 458 metres is covered in 1 second.
Title: Re: Is there a discrepancy with the equivalence principle?
Post by: guest39538 on 12/02/2018 20:18:28
@Colin2B
We made the calculation for acceleration using the value of c as per our clock rate now.
And we made the calculation for, not 'the' extra distance travelled, but a value of distance to work with, using the value of t=age universe on the basis of c as per the rate of our clock, times t.

BUT - between 13.8 billion years ago, and now, time was 50% slower and accelerated to the rate it is now.  So using c times t needs to be corrected, b/c we have held c relative to the rate of our clock.
Where the measurement of a 50% distance of contraction in t=13.8billion years, is our rate of c, times the t=13.8 billion years of our rate of time.

So, on the basis that the speed of light is constant in any reference frame, including the reference frames of our history, if the speed of light covers 299 792 458 metres per 50% longer length of second than ours 13.8 billion years ago, then our shorter second in comparison, (13.8 billion years ago) would only cover half of 299 799 458 metres, and at every point in time after would cover a little more than half of 299 799 458 metres, up until present time where the speed of light is held relative to our clock and the full 299 799 458 metres is covered in 1 second.
That is brilliant though, well done Timey I understood that post right away and you are absolutely correct about the content discussed. 

You are saying, my example :  that if the Universe was expanding at 0.5c it would be 27.6 billion years old which is a contradiction?

Title: Re: Is there a discrepancy with the equivalence principle?
Post by: timey on 12/02/2018 23:06:59
@Colin2B
We made the calculation for acceleration using the value of c as per our clock rate now.
And we made the calculation for, not 'the' extra distance travelled, but a value of distance to work with, using the value of t=age universe on the basis of c as per the rate of our clock, times t.

BUT - between 13.8 billion years ago, and now, time was 50% slower and accelerated to the rate it is now.  So using c times t needs to be corrected, b/c we have held c relative to the rate of our clock.
Where the measurement of a 50% distance of contraction in t=13.8billion years, is our rate of c, times the t=13.8 billion years of our rate of time.

So, on the basis that the speed of light is constant in any reference frame, including the reference frames of our history, if the speed of light covers 299 792 458 metres per 50% longer length of second than ours 13.8 billion years ago, then our shorter second in comparison, (13.8 billion years ago) would only cover half of 299 799 458 metres, and at every point in time after would cover a little more than half of 299 799 458 metres, up until present time where the speed of light is held relative to our clock and the full 299 799 458 metres is covered in 1 second.

And this (unless I'm very much mistaken) reduces the 50% of R distance, that we are working with, to 25%. right?
Title: Re: Is there a discrepancy with the equivalence principle?
Post by: Colin2B on 13/02/2018 09:39:02
The answer is always 50%.
Take a look at the equations and work through them, if you adjust the time in any way you still get 50%. You can also adjust c and R and get the same result.
The reason is that there is a fixed relationship between R and 2 of the given values you supplied in the equations ie R=ct and R=c^2/a.
Work through the equations and you’ll see what i mean.
Title: Re: Is there a discrepancy with the equivalence principle?
Post by: timey on 13/02/2018 14:49:50
Ah yes, sorry Colin, I looked at my explanation and my explanation doesn't make it clear enough, b/c I haven't stipulated that only the resulting distance of 50% of R needs correcting.

Looking at the equation:
I use both c^2 and c to obtain the answer 50% of R.

Technically, (in my mind anyway ;) ) I have replaced the use of c^2 with the re-lengthed seconds, that c is held relative to (in the slower rates of time), for the resulting distance of 50% of R, and the re-lengthed seconds mean that I can reduce this 50% of R distance by half.

I realise that this constitutes a weird transaction of values over R and c^2, (edit correction: over R and c) but I think it works. You have to keep in mind Colin that the point of these equations is based on time being dynamical, and that in all reference frames, current and historic, c is held relative to this dynamical time.  So c times t = R is never a fixed value equation, and the purpose here is to use this value (which is our current observation) to ascertain the 'other values'.

To be clear - there are a few more operations that need to be manipulated before it is calculated (for my model) by how much distance the universe has contracted, how old the universe is (as per our rate of time), and how long it will take (as per our rate of time) for it to complete the contraction.
Title: Re: Is there a discrepancy with the equivalence principle?
Post by: alancalverd on 13/02/2018 14:52:35
All the intervening regions of more or less g are irrelevant. Spectral shift depends only on the relative velocity of the sender and receiver, and the gravitational potential difference between the start and finish points. In any contracting universe, distant objects will have a blue shift. The fact that most of them seem to have a red shift is the reason that everyone else thinks the universe is expanding.
Title: Re: Is there a discrepancy with the equivalence principle?
Post by: timey on 13/02/2018 15:26:09
A contracting universe that has galaxies moving towards the observers position will be observed to blueshift.

In my contracting model, galaxies are converging into galaxy clusters and the spaces between the galaxy clusters are expanding due to back reaction.  The observations of shifted light will be the observations we observe.  And in a universe that is not expanding, all the intervening regions of more or less g are NOT irrelevant. 

And, actually, professional theoretical physicists think of theoretical physics, ie: physics that hasn't been experimentally proven, as theory, where the only reason the 'ley person' BELIEVES the universe to be expanding, is b/c that is the theory that has had the most money spent on it, and therefore it has not been financially viable for professional theoretical physicists to work on anything else, even though other ideas may have existed, and still do exist.
Title: Re: Is there a discrepancy with the equivalence principle?
Post by: Colin2B on 13/02/2018 18:23:09
Bear in mind that my only interest here is that your paper gets a good hearing and doesn’t stumble.

Ah yes, sorry Colin, I looked at my explanation and my explanation doesn't make it clear enough, b/c I haven't stipulated that only the resulting distance of 50% of R needs correcting.

Looking at the equation:
I use both c^2 and c to obtain the answer 50% of R.

Technically, (in my mind anyway ;) ) I have replaced the use of c^2 with the re-lengthed seconds, that c is held relative to (in the slower rates of time), for the resulting distance of 50% of R, and the re-lengthed seconds mean that I can reduce this 50% of R distance by half.
Ok, i dont understand why you are making a ‘correction’, so i cant really help with this, but at the end of the day it is your paper and your responsibility for final edit.

You have to keep in mind Colin that the point of these equations is based on time being dynamical, and that in all reference frames, current and historic, c is held relative to this dynamical time.  So c times t = R is never a fixed value equation, and the purpose here is to use this value (which is our current observation) to ascertain the 'other values'.
Yes the time is dynamic, but it is affecting both ct and the acceleration equally and for the same overall period, with the same changes, so it is unclear why you would make a correction. I did some rough calcs using dynamic time and it made no difference to the %. You will need to make your reasoning much clearer in the paper.
Title: Re: Is there a discrepancy with the equivalence principle?
Post by: jeffreyH on 13/02/2018 18:35:41
The 'observation' is c times t.

No. Light travels at c, so the the distance of any object you can see must be cτ where τ is the time it took for the light to travel from there to here. If the universe has a finite age t then the farthest observable object is at R = ct,the Schwarzchild radius.

R is important as long as c is the limiting speed for the propagation of gravity, which seems to be the case.Nothing outside of R can affect anything inside. Or can it?

Imagine a static universe. If there is a massive body at R + ΔR, it will have a gravitational effect at R such that photons emitted towards us from a body at R will appear to have come from a more massive body and thus be redshifted.Which is what we observe!

If the observable universe is surrounded in all directions by a much larger attractor then there may be no need for dark energy. The problem is what can attract mass out of an event horizon?
Title: Re: Is there a discrepancy with the equivalence principle?
Post by: timey on 13/02/2018 18:49:46
@Colin2B
Yes I will have to make the paper very clear, so in going through this here with you, it is my intention that this should result in my being able to make a clearer explanation, hopefully inclusive of mathematical equations.

No the historically slower rate of dynamic time that increases at the same rate of acceleration, will not have any mathematical effect on the acceleration.
This being b/c the dynamical time increasing is what is 'causing' the acceleration, and therefore in re-described that acceleration as time being 50% slower 13.8 billion years ago, I then calculate what distance of 50% of R my variable light speeds will cover. (variable light speeds b/c c held relative to a slower second than ours will not cover as much distance as c held relative to our second, which is how we calculated distance R. This being the correction I seek to make)

@jeffreyH Whatever it is that would attract mass outside of an event horizon, I can guarantee you it will not have anything to do with my contracting universe.
Title: Re: Is there a discrepancy with the equivalence principle?
Post by: alancalverd on 13/02/2018 20:10:08
The Schwarzchild radius is not an event horizon.
Title: Re: Is there a discrepancy with the equivalence principle?
Post by: alancalverd on 13/02/2018 20:12:07
the only reason the 'ley person' BELIEVES the universe to be expanding,
is because distant objects are red shifted. Nothing to do with any theory.
Title: Re: Is there a discrepancy with the equivalence principle?
Post by: jeffreyH on 13/02/2018 20:14:51
The Schwarzchild radius is not an event horizon.

It is the radius of the horizon. If I am wrong then please explain.
Title: Re: Is there a discrepancy with the equivalence principle?
Post by: timey on 13/02/2018 20:49:52
the only reason the 'ley person' BELIEVES the universe to be expanding,
is because distant objects are red shifted. Nothing to do with any theory.

Yes - light in a negligable gravity field is redshifted as a velocity related phenomenon.  B/c current theory states the gravitational fields between galaxy clusters as negligable, (on account of the expansion), the 'fact' that redshifts are observed (here on earth) to be velocity related in a negligable gravity field, is used to describe the expansion.

But - light is also redshifted by changes in g.  B/c my model is not expanding, it is contracting, there is no such thing as a negligable gravity field in the gravitational fields between galaxy clusters, so for the most part my model uses gravitational changes to desribe redshift observations.
Title: Re: Is there a discrepancy with the equivalence principle?
Post by: timey on 13/02/2018 21:52:33
@Colin2B
Yes I will have to make the paper very clear, so in going through this here with you, it is my intention that this should result in my being able to make a clearer explanation, hopefully inclusive of mathematical equations.

No the historically slower rate of dynamic time that increases at the same rate of acceleration, will not have any mathematical effect on the acceleration.
This being b/c the dynamical time increasing is what is 'causing' the acceleration, and therefore in re-described that acceleration as time being 50% slower 13.8 billion years ago, I then calculate what distance of 50% of R my variable light speeds will cover. (variable light speeds b/c c held relative to a slower second than ours will not cover as much distance as c held relative to our second, which is how we calculated distance R. This being the correction I seek to make)

Anyway - to finish the equation I'm going to say that the estimated age of the universe is based on CMB data, and that I now need to minus the 8% difference observed of galaxy cluster redshifts, from this distance that we are working with which is now potentially 25%, leaving me with a distance that is 17% of R=c(t=age of universe as per our rate of time). This would be the distance that the entire universe has shrunk by, but in how many years held relative to our rate of time did that 17% shrinkage occur in?

Now I want to go backwards from present time where our years are held relative to our clock, back to 13.8billion years ago where time was 50% slower, and I can say that there is half again as many of our years from 13.8 billion years ago until present time measurement. This amounts to a total of 20.7 billion of our years.
(this is useful for describing cosmologic development in too many ways to mention)

Then I can say let's look into the future.  In 13.8 billion of our years an acceleration of c^2 will shrink our universe by how much?
This time the seconds get shorter as we go into the future, and more distance will be covered than c times t in our present time reference frame.  I will have to add potentially* 25% to 50% for this equation.  However the recessional velocity of objects contracting towards each other in the universe will be greater than they are now, and will constitute a larger percentage than 8% (that is the tricky bit) which I must minus from 75% in order to find the pecentage of R=c(t=13.8 billion years of our rate of time).  The entire universe will have contracted by that percentage in half of 13.8 billion of our years, ie: 6.9 billion years.
(*I say potentially b/c that is the part of the equation that we were at when you didn't understand how I had transferred the acceleration into dynamical time that is the cause of the acceleration.
I hope that the explanation of intended direction has cleared that up, and we can go back to hopefully notate it)
Title: Re: Is there a discrepancy with the equivalence principle?
Post by: Colin2B on 13/02/2018 22:18:11
The Schwarzchild radius is not an event horizon.
It is the radius of the horizon. If I am wrong then please explain.
Only in the special case of a non-rotating object whose radius is smaller than its Schwarzschild radius.
Title: Re: Is there a discrepancy with the equivalence principle?
Post by: Colin2B on 13/02/2018 22:27:29
B/c current theory states the gravitational fields between galaxy clusters as negligable,
However, there are intense gravitational fields that light passes through.

But - light is also redshifted by changes in g. 
That is not observed when light passes through intense gravitational fields.

I mention this because you are going to have to explain this anomaly in your paper.
I think i said in an earlier post that current physics treats this as travelling along a road with hills between you and destination, it doesn’t matter how much up and down you do (potential energy gain and loss), just the difference between height at beginning and end. You will need to make clear how your alternative works.
Title: Re: Is there a discrepancy with the equivalence principle?
Post by: timey on 13/02/2018 22:36:42
Light cannot be observed passing through any gravitational field.  The only place the light can be observed is when it hits the detector at the observation point, by which time anything that happened to that light with regards to shift has already happened.

Yes - we can observe that the direction that light takes in space can be changed by an intense gravitational field, but this says nothing about what the gravitational field is doing to the light with regards to shifts.  In fact it is known that light does shift with gravitational changes, but in the expanding universe theory, these shifts are negligable to the velocity related phenomenon of expansion.

Edit: In reply to your edit.  So there isn't any anomoly to be explained here, right?
Title: Re: Is there a discrepancy with the equivalence principle?
Post by: Colin2B on 13/02/2018 23:01:25
Edit: In reply to your edit.  So there isn't any anomoly to be explained here, right?
So are you saying that in your theory light passing through variations in gravitational fields between galaxies is not shifted.
Title: Re: Is there a discrepancy with the equivalence principle?
Post by: timey on 13/02/2018 23:06:03
Not at-all.  I'm saying that all but 8% (at this present time) of those redshift observations from galaxies is gravitational shift.

Yes, the light will be blueshifted if it passes by a gravitational mass, but this will not result in a blueshifted observation.  It just minuses the redshift, but only very slightly.
Title: Re: Is there a discrepancy with the equivalence principle?
Post by: timey on 13/02/2018 23:25:34
@Colin2B
Yes I will have to make the paper very clear, so in going through this here with you, it is my intention that this should result in my being able to make a clearer explanation, hopefully inclusive of mathematical equations.

No the historically slower rate of dynamic time that increases at the same rate of acceleration, will not have any mathematical effect on the acceleration.
This being b/c the dynamical time increasing is what is 'causing' the acceleration, and therefore in re-described that acceleration as time being 50% slower 13.8 billion years ago, I then calculate what distance of 50% of R my variable light speeds will cover. (variable light speeds b/c c held relative to a slower second than ours will not cover as much distance as c held relative to our second, which is how we calculated distance R. This being the correction I seek to make)

Anyway - to finish the equation I'm going to say that the estimated age of the universe is based on CMB data, and that I now need to minus the 8% difference observed of galaxy cluster redshifts, from this distance that we are working with which is now potentially 25%, leaving me with a distance that is 17% of R=c(t=age of universe as per our rate of time). This would be the distance that the entire universe has shrunk by, but in how many years held relative to our rate of time did that 17% shrinkage occur in?

Now I want to go backwards from present time where our years are held relative to our clock, back to 13.8billion years ago where time was 50% slower, and I can say that there is half again as many of our years from 13.8 billion years ago until present time measurement. This amounts to a total of 20.7 billion of our years.
(this is useful for describing cosmologic development in too many ways to mention)

Then I can say let's look into the future.  In 13.8 billion of our years an acceleration of c^2 will shrink our universe by how much?
This time the seconds get shorter as we go into the future, and more distance will be covered than c times t in our present time reference frame.  I will have to add potentially* 25% to 50% for this equation.  However the recessional velocity of objects contracting towards each other in the universe will be greater than they are now, and will constitute a larger percentage than 8% (that is the tricky bit) which I must minus from 75% in order to find the pecentage of R=c(t=13.8 billion years of our rate of time).  The entire universe will have contracted by that percentage in half of 13.8 billion of our years, ie: 6.9 billion years.
(*I say potentially b/c that is the part of the equation that we were at when you didn't understand how I had transferred the acceleration into dynamical time that is the cause of the acceleration.
I hope that the explanation of intended direction has cleared that up, and we can go back to hopefully notate it)

So, on thinking it over further, whatever percentage 8% is of 25%, ?%=percentage we are looking for, will be that percentage of 75%.
Title: Re: Is there a discrepancy with the equivalence principle?
Post by: Colin2B on 14/02/2018 09:07:20
Yes, the light will be blueshifted if it passes by a gravitational mass, but this will not result in a blueshifted observation.  It just minuses the redshift, but only very slightly.
So, if white light passes by a gravitational mass it would be blueshifted.
You need to explain why and by how much. That would be another prediction of your theory.

I hope that the explanation of intended direction has cleared that up, and we can go back to hopefully notate it
I must confess i would need to go through it in detail to see exactly what happens as i’m not getting a clear picture from your description, would have been better to declare this at beginning and work from start as the 50% might change. I’m busy this week sorting out my mother in law’s paperwork, so best you write a section for your paper clearly detailing what you’ve done and why and calling it a working estimate, then if we can work out a methodology from that we can replace it later.

Also, editorial slip, you say change in time causes acceleration, in that case faster time should be in past - think Pound Rebka, blue shift - can’t remember if i mentioned but i think there is a similar slip in your video, 2 graphs are reversed acceleration & deceleration.
Title: Re: Is there a discrepancy with the equivalence principle?
Post by: alancalverd on 14/02/2018 10:47:42
Light will be blueshifted as it approaches a mass, but equally redshifted as it departs, so the net effect is zero for light passing through a stationary universe, and only dependent on the relative gravitational potentials of the  source and detector.

In the case of a contracting universe, all "incoming" light from distant objects must be blueshifted by both the motion of the source and the fact that g increases with time at the detector.
Title: Re: Is there a discrepancy with the equivalence principle?
Post by: guest39538 on 14/02/2018 11:31:48
Light will be blueshifted as it approaches a mass, but equally redshifted as it departs, so the net effect is zero for light passing through a stationary universe, and only dependent on the relative gravitational potentials of the  source and detector.

In the case of a contracting universe, all "incoming" light from distant objects must be blueshifted by both the motion of the source and the fact that g increases with time at the detector.
Alan, no.     The Universe could contract by the way you are describing and a blue-shift of the objects, but it can also contract another way , in that the already objects in expansion , expand out of visual range, the universe will then contract back to the furthest point we can see.
Title: Re: Is there a discrepancy with the equivalence principle?
Post by: guest39538 on 14/02/2018 11:40:20
I drew it ...


* contact.jpg (32.6 kB . 731x461 - viewed 3434 times)


Title: Re: Is there a discrepancy with the equivalence principle?
Post by: Colin2B on 14/02/2018 12:47:36
Light will be blueshifted as it approaches a mass, but equally redshifted as it departs, so the net effect is zero for light passing through a stationary universe, and only dependent on the relative gravitational potentials of the  source and detector
@timey - Alan’s description is the equivalent of my up hill and down dale analogy.

Light cannot be observed passing through any gravitational field. 
yes it can

Yes - we can observe that the direction that light takes in space can be changed by an intense gravitational field, but this says nothing about what the gravitational field is doing to the light with regards to shifts.
We use the same technique of observing before and after, but instead of position change we would see a spectral line shift. A field strong enough to bend light is strong enough to cause a shift. No shift has been observed so your paper will need to explain why, it will be an obvious question from any reader.
Title: Re: Is there a discrepancy with the equivalence principle?
Post by: timey on 14/02/2018 13:36:10
@Colin2B
If time was faster in the past and is slower now, then my contraction would be decelerating as mass further clumps. Mass accelerating in the clumping process, and decelerated velocities of contraction are not compatible.
(Please remember that these changes in time that I am stating as affecting everything universally are not 'the' changes in time we observe of clocks in the gravity potential, although all rates of time of 'everything' will be affected b/c 'everywhere' is)

I have actually worked out (as per my model) the age of the universe, how long it took to contract till now, how long it will take to contract from now till the point of next Big Bang, and what distance the universe was before it contracted, the working outs of which I will post later.

@alancalverd.
In a contracting universe where the objects emitting light are not moving towards the observation point, there will be no blueshift observation.
In a contracting universe where all mass sizes 'do not' get bigger, the increases in the g field caused by the contraction will cause light to be blueshifted by the increasing g of the field caused by the contraction...
But in my model mass sizes are increasing (this being what is causing the universal contraction), so the g field of the spaces in between the increasing masses will be reducing in g faster than the contraction is adding to g.  And the observation will be redshifts.

@Colin2B But Alan you are correct that redshifted light moving into and out of the gravitational field of a body of mass, will be blue shifted as it moves into the masses field, but will be redshifted as it moves away from the gravitational mass by an equal amount, and will constitute no change to a redshift observation.
So Colin, there is no difference to these mechanics in my model.  And the equations I am seeking to describe the increase of the universal increases in time due to contraction of entire universe, do not have anything to do with the up/down hill type geometry of GR that my model adds a correction term for in my video pages 1 through to 4b.
And Colin, one can observe light that has travelled through the gravitational field of a mass, but one only observes that light when it arrives in the reference frame one is observing from. (that being my point)

@Thebox. Look I appreciate the interest, and the support, but your posts are a bit confusing the issues here, and I haven't got the time or energy to engage with 3 people about issues that are confused.
So with all due respect, b/c Alan and Colin have better math skills I will stick to answering them.  Sorry, I don't mean to be rude, I'm just very focused at the mo. K?
Title: Re: Is there a discrepancy with the equivalence principle?
Post by: Colin2B on 14/02/2018 15:15:12
OK, glad to hear you are clear about your theory, but there are a lot of items you will need to cover in your paper. You can't rely on someone coming back with questions like we do, you need to cover any possible question first, remember busy people no time. You can put detail and discussion into appendices in order to keep body text succinct.

If time was faster in the past and is slower now, then my contraction would be decelerating as mass further clumps.
You will need to explain why move from slower to faster is not a deceleration.

(Please remember that these changes in time that I am stating as affecting everything universally are not 'the' changes in time we observe of clocks in the gravity potential, although all rates of time of 'everything' will be affected b/c 'everywhere' is)
I had assumed that, but it is still a change of state from time past to time present, and light + universe radius are moving through it, so you will have to explain why it doesn’t result in a deceleration. After all, you state it is the cause of the acceleration so it must have an overall effect.

one can observe light that has travelled through the gravitational field of a mass, but one only observes that light when it arrives in the reference frame one is observing from. (that being my point)
My (and @alancalverd ) point also. But you will need to make this clear in statements such as:

“Yes, the light will be blueshifted if it passes by a gravitational mass, but this will not result in a blueshifted observation.  It just minuses the redshift, but only very slightly.”

Otherwise the reader will assume you mean a shift is detected at the observing point.

Lot’s left to do, but you are making progress towards a paper.
Title: Re: Is there a discrepancy with the equivalence principle?
Post by: timey on 14/02/2018 23:31:05
@Colin2B.  Taking on board the peripherals that you identify for future reference:

Ok...
(please note that unless stated otherwise all (c) and (t), and therefore (R) notation is as per our present rate of time)

ct=R
c^2/R=a
(c (is it times or plus?) a)t=(R+half R) - R = half R (or 50% of R)

My notation (td) means (time changes)

(td)/R=c^2/R, is equal to (50% td) over R, which isn't equal to 50% of R.
(c^2/R is now superfluous b/c (td)/R is equal and now 'physically describes c^2/R acceleration as (td))

(50%td) over R = 25% of R.
(I anticipate that you may want explanation to understand the mechanics of this part of the equation, and can provide an explanation on request if needed after seeing the 'table')

CMB redshift observation is 8% lesser than Galaxy Cluster resdshift observation.

25-8=17% of R contraction.

8 is 33% of 25, and that is the ratio between the decrease in redshift due to galaxies converging into galaxy clusters causing the g fields between the galaxy clusters to be decreased in g, and the increase in the gravity field due to universal contraction, that is then the cause of the clumping of mass to be accelerated.

Please see my table of time changes using the (td)/R=50% acceleration, as I take the universe first back in time, and then forward.
Resulting in the total time as per our years that the universe takes to contract, what position we are at now in that contraction, and the total 'actual' distance that the inflation period expanded the universe to, before it then started contracting.

(https://i.imgur.com/K36jU34.jpg)

(I would be most grateful Colin if you would identify points where my explanation does not make itself clear.  Of course that table could be elloquently expressed by some type of summation of changes in time over 33%, or something like that anyway, but that type of higher degree of mathematical manouvers is 'so far' a bit beyond me tbh)
Title: Re: Is there a discrepancy with the equivalence principle?
Post by: guest39538 on 15/02/2018 10:04:38
My notation (td) means (time changes)

Δt  means time changes

p.s you could of put  t/x  ≠   t'/ 0.5x

But you didn't want my help ....
Title: Re: Is there a discrepancy with the equivalence principle?
Post by: Colin2B on 15/02/2018 10:18:03
I’ll have a think about the table. I’ll need go through it to think if there is a better way of notating the rest but at the moment i cant really understand why you’ve done what you have.
Can i suggest you write this up to go in an appendix and we move on, perhaps something further on will feed back to this and make it clearer for me.

EDIT: just to explain. You have an understanding of how your 3 different time systems work and they don’t work as per conventional physics, so Alan & i need more information before we can understand how all this works. Hopefully that will come as we progress and at some stage we can revisit this. For the moment you have something you are happy to use as a working hypothesis.
Title: Re: Is there a discrepancy with the equivalence principle?
Post by: timey on 15/02/2018 13:38:09
OK Colin.  Thanks.
I have completed (for my paper) the inclusion of my video considerations up to page 4b, these being concerning considerations of GR  curvature, inclusive of motion related SR considerations.
I will now go back to page 5 and provide a much better description of what are explanations of SR length distortion considerations, placed within the construct of the page 1 through to 4b considerations.

(EDIT CORRECTION: Don't consider b/c I got it wrong
***What I do need you to consider in that 'table', (b/c sleeping on it, I realise I should have included) is that:
When the 'distance due to (td)' is at 0.75% of R, and the 'percentage of R due to the velocity of 'objects' redshifts' is at 0.24% of R, and the 'percentage of R contracted by' is at 0.51%...
It goes back in history 1 more time of +50% (td) where the 'percentage of R due to velocity redshidts' ends up being 0.033% or so, and 'percentage of R contracted by' is 0.1%.
(Edit correction:  the 'percentage of R due to velocity redshidts' ends up being 0.1% or so, and 'percentage of R contracted by' is 0.033% (I had put numbers wrong way round))*** EDIT CORRECTION: see next post for correction)

Anyway that is the ratios, and in going back in history we are heading back to an almost uniform sea of particles and energy, where the slightly anisotropic nature of this arrangement can be considered as the almost uniform g field having more potential in some areas than others.  This is a GR consideration, so my alterations to these descriptions are contained elsewhere.

The universal increases in rates of time over history (and into future) are going to be describing the 'observation of the acceleration of redshifts' that is attributed (currently in the expanding universe theory) to the accelerated expansion of the universe at a rate of c^2/R.

GR can describe a contracting universe, (without cosmological constant).
I just described physical mechanics for the acceleration of a contraction, for a universe that is accelerating in it's contraction,  using a value know to be associated with the cosmological constant.
Title: Re: Is there a discrepancy with the equivalence principle?
Post by: timey on 15/02/2018 21:12:48
Ok, I said something wrong in last post regarding taking the universe back further in history.  But this is the last word on this contraction business before I move on.

You will see from the table, and the last 3 colomns, that I take the 'reduced distance due to (td)' and ask what 33% of that distance is.  This is the 'percentage of R due to velocity reshifts' (or the rate at which mass is clumping) which I minus from the first distance to result in 'percentage of R contracted by'.

Now to say so, I tried calculating, in addition to the 6 jumps I already made into the past, 5 more t=+50% jumps of R=ct into the deeper past, where the 'distance due to time dilation' at each jump is halfed, and 33% of that distance is minused to give the percentage of R contracted by, which after 5 further jumps into the past has left me with a contraction of 0.0313225% of R.
In order to get to a 0% 'the percentage of R contracted by' result, I'd have to jump back few(?) more times.   
Title: Re: Is there a discrepancy with the equivalence principle?
Post by: timey on 27/02/2018 18:13:26
I had to step off and take a break from this project for a while, but I am back on it now.
@Colin2B, I am wondering if you have found the time as of yet to have a look at the 'table', and if so, if you had diciphered what I am doing there?
Title: Re: Is there a discrepancy with the equivalence principle?
Post by: timey on 28/02/2018 18:37:53
I’ll have a think about the table. I’ll need go through it to think if there is a better way of notating the rest but at the moment i cant really understand why you’ve done what you have.
Can i suggest you write this up to go in an appendix and we move on, perhaps something further on will feed back to this and make it clearer for me.

EDIT: just to explain. You have an understanding of how your 3 different time systems work and they don’t work as per conventional physics, so Alan & i need more information before we can understand how all this works. Hopefully that will come as we progress and at some stage we can revisit this.

@Colin2B Are you and Alan still with this?  It is my intention for you guys to understand.  I have re-arranged the intro quite considerably to this purpose, trying to outline what I am trying to do, and where the differences that I am implementing are.
Title: Re: Is there a discrepancy with the equivalence principle?
Post by: alancalverd on 28/02/2018 22:56:15
As I understand things so far, the hypothesis is that the red shift of distant objects is due to their clumping.

So let's have two observers at distance D and 2D from a clumping object. The observer at D sees events that occurred t years ago, and the observer at 2D sees events that occurred 2t years ago.

As the object contracts, so its red shift increases because g is increasing. Velocity red shift is irrelevant because for every part that is moving away from the observer towards the center of the clump,  there will be a symmetric part moving towards him. Gravitational red shift is the only significant factor.

Thus red shift increases with time. So the red shift observed at D, being the result of a more recent emission than that observed at 2D, will be greater.Therefore the red shift of near objects will be greater than the red shift of distant objects, which is contrary to observation.
Title: Re: Is there a discrepancy with the equivalence principle?
Post by: timey on 01/03/2018 00:41:00
Ok, look @alancalverd, you understand that a contracting universe consideration, where light sources are moving towards the observation point, will result in a blueshifted observation.

Do you also understand that a contracting universe consideration, where the light sources are 'not' moving towards the observation point, and mass sizes do not get bigger (ie:no gravitational clumping action), that the observation will also be blueshifted? This being due to the g field inbetween the light sources increasing in g as the universe contracts. (although in this scenario there is no mechanics for a contraction)

Ok, so what I am describing is a contracting universe consideration, where light sources are not moving towards the observation point, but are converged into galaxies that are converging upon each other into galaxy clusters, and mass sizes are increasing due to gravitational clumping.
Under this construct the highly anistropic g field between lightsources that have converged into galaxy/galaxy clusters* will be decreasing in g due to the clumping action, faster than the contraction of the universe will be increasing this highly anistropic g field.
(*clearly this is a vast oversimplification of the physics of galaxy formation, I am just referring to the action of gravitational clumping here)

Ok - so let's look at what you have said:

Quote Alan:
"So let's have two observers at distance D and 2D from a clumping object. The observer at D sees events that occurred t years ago, and the observer at 2D sees events that occurred 2t years ago."

Ok well a 'clumping object' is not good visualisation for me, so let's say we are looking at a galaxy cluster from D and 2D, at events that took place at t and 2t

Quote Alan:
"As the object contracts, so its red shift increases because g is increasing. Velocity red shift is irrelevant because for every part that is moving away from the observer towards the center of the clump,  there will be a symmetric part moving towards him. Gravitational red shift is the only significant factor."

Ok, so I don't get the first sentence.  The galaxies of the galaxy cluster will be converging upon each other, and this motion will be 'away' from either observer who is observing from differring galaxy clusters at D and 2D. (my model says that this will be a velocity related redshift and that this velocity related redshift will constitute 8% of the observed redshift for both observers.)

I do not understand "there will be a symmetric part moving towards him".  Our only observations of light sources movng towards us involve galaxies of our own galaxy cluster.
But yes - apart from the 8% velocity related redshifts, all the rest of the redshift is attributed to gravitational shift due the g-field between masses decreasing in g, and this is caused by the clumping process. (the universal contraction process that the clumping action causes does not add to g in the field as fast as the clumping process reduces g in the field)

Quote Alan:
"Thus red shift increases with time. So the red shift observed at D, being the result of a more recent emission than that observed at 2D, will be greater.Therefore the red shift of near objects will be greater than the red shift of distant objects, which is contrary to observation."

So yes, the redshift will increase over time, and this is what we observe, in that lightsources that are more distant are more redshifted.

However, if we say that we are the observers at 2D from observation, and we observe redshift B from light source at observation, but we observe redshift A coming from D.  We can then work out that observer at D will observe a redshift A  from both the observation at 2D from us, and from our galaxy cluster, because observer at D is midway inbetween. (this is no different to the current expansion viewpoint)

Now it is the data of the CMB values, where the redshifts are 8% lesser than those of galaxy clusters, that becomes significant.

The contracting universe that I am proposing results in the observations that we observe Alan. I've spent over 10 years checking this now... and indeed I will state again that my model can explain observations that current theory cannot, such as the discrepancy between the CMB and galaxy cluster redshift measurements, the discrepancies of luminosity magnitude with respect to distance, and the quandry of the development of the recently discovered supermassive blackholes, etc.

Before moving on, is there any part of the above that needs clarifying?

Because if not, then I can start talking about how the 'table' that I provided earlier is essentially placing observers at D, 2D, 3D, 4D, etc, into the past, and at D, 2D, 3D, 4D, into the future, using D=R and R=c(t=13.8billion years) in order to make it's calculations.

Or alternatively I could start intoducing my revised paper as to the modification that my model makes to general relativity, and see if I can bring you to an understanding of what I am doing there, as @Colin2B suggested, and come back to the 'universal contraction' consideration afterwards.
Title: Re: Is there a discrepancy with the equivalence principle?
Post by: alancalverd on 01/03/2018 08:44:04
The "intervening fields" between source and observer are irrelevant. Gravitational redshift depends only on the difference of gravitational potential between the source and observer.

The bit about symmetry isn't terribly important, but if a galaxy far far away is condensing, to a first approximation that movement will be a spherical collapse towards the center of that galaxy, so there will be as much matter moving towards us as away from us if the center isn't moving. But the important part is that the gravitational redshift of that galaxy will increase with time as the local g increases. Thus redD> red2D

Now let's allow the distant galaxy to move towards us as part of the overall collapse of the universe. That movement will produce a velocity blue shift that will increase with time as the collapse accelerates. Hence bluD >blu2D.

So we have a balance between observed redshift due to local clumping of distant objects and blue shift due to overall contraction.Therefore

(a) there will be a value D0 where the clumping redshift equals the collapsing blueshift, and

(b) nearby galaxies will have a net blueshift and distant galaxies a net redshift.

Is either statement actually observed?
Title: Re: Is there a discrepancy with the equivalence principle?
Post by: timey on 01/03/2018 13:28:55
@alancalverd Can I ask please if you had a look at those 3 arXiv papers on contracting scenarios by professional theoretical physicists, that I posted to you earlier this thread?  I can assure you that the only difference that I am adding 'never heard before' is the time related cause and effect mechanics for gravity.

OK so...

The difference of gravitational potential between the source and observer IS the 'intervening field' surely?

And in a universe that is contracting under the influence of gravity, this intervening field of gravity potential between source and observer is not irrelevant.

Also, you haven't seemed to grasp that a galaxy cluster, where the galaxies of that galaxy cluster are converging on a point, will not produce any objects that are moving in any significant fashion 'towards' the observer.
(My models contraction occurs due to the influence of gravity. There is no gravitational reason for 'a part' of a distant galaxy cluster to move significantly towards another galaxy cluster.)

The observers point of observation will also be converging towards the point that the galaxies of his galaxy cluster are moving towards.
Both the observation and the observation point are moving away from each other, and the distance between, ie: the g field of gravity potential, will be lengthening in distance, and reducing in g.
And the redshifts observed (of galaxy clusters) are then a combination of velocity related redshifts (8%) and gravitational redshifts (92%). Where 8% is the discrepancy between galaxy cluster redshifts and the CMB redshifts.

Now it becomes important to consider that a universe contracting under the influence of gravity will contract at an accelerating rate.  That is what my 'table' is concerning, providing physical cause and effect mechanics for an accelerated contraction, via the physics of changes in the rate of time in the g field, also addressed as my modification to GR in the video worksheets. (currently being improved upon for my paper, hopefully with some mathematical guidance from yourself and @Colin2B?)
Title: Re: Is there a discrepancy with the equivalence principle?
Post by: Colin2B on 03/03/2018 11:27:34
@alancalverd Can I ask please if you had a look at those 3 arXiv papers on contracting scenarios by professional theoretical physicists, that I posted to you earlier this thread? 
I don’t think @alancalverd will be able to see them as they were embedded in the paper which has now disappeared.

The difference of gravitational potential between the source and observer IS the 'intervening field' surely?

And in a universe that is contracting under the influence of gravity, this intervening field of gravity potential between source and observer is not irrelevant.
Just to clarify. Did you agree in an earlier post that the uphill down dale effect wouldn’t affect your light beam.
The difference between gp of source and observer is not the intervening field in current physics, so you would need to offer a new methodology here.
What is relevant is the gp of the source at the time the light left. Could your theory suggest that this has changed and so what we observe might not be correct? Just a thought.
Title: Re: Is there a discrepancy with the equivalence principle?
Post by: timey on 03/03/2018 14:45:58
No - what I said was that light travelling into and out of a mass's gravitational field that is between the source and the observation point will not affect the magnitude of redshift.

'Hills and troughs' is an analogy of the general relativity spacetime geometry.  This analogy allows the visualisation of an object moving slower when it climbs a hill, and faster when it drops into a trough.  This is the general relativity "mass tells space how to bend, and space tells mass how to move".

My model doesn't change this outlook in the slightest.
What my model does is describe physical cause and effect mechanics for 'why' the object goes faster or slower, via the physics of changes in time. (and in doing so results a flat spacetime geometry)

So you say: "the difference between gp of source and observer is not the intervening field in current physics"
...and as such, is therefore not irrelevant but can be negated at present.
It is the g field between source and observation point that is NOT irrelevant to the redshifts observation.

The methodology I am proposing is that light that left a source 13.8 billion years ago will have been moving across a g field in-between source and observation. In the 13.8 billion years (our rate of time) that the light took to travel from source to observation, that g field will have changed due to 13.8 billion years of further clumping under the influence of gravity.  It will have gotten 'weaker', and that will cause a redshift.

Yes - I did notice that the company who web hosted my PDF deleted it, but I'm sure I actually posted those particular 3 arXiv papers in a post specifically addressed to Alan.  I will now go back and see if I can find it.
Title: Re: Is there a discrepancy with the equivalence principle?
Post by: timey on 03/03/2018 15:13:28
Here are the 3 arXiv papers

@alancalverd
Investigation into the concept of a contracting universe has been explored under various premise:

By Christof Wetterich
https://arxiv.org/abs/1303.6878

Here in the form of a pre-big bang contraction by Thorsten Battefeld, Robert Brandenberger
https://arxiv.org/abs/hep-th/0406180

And more recently here with regards to black holes by Jerome Quintin, Robert H. Brandenberger
https://arxiv.org/abs/1609.02556

In case, as a latecomer, you have not realised, I was asking about the mathematical consideration b/c I am, following both @Colin2B and @jeffreyH advice, currently improving on my paper that I am trying to submit with arXiv scientific journal.
Title: Re: Is there a discrepancy with the equivalence principle?
Post by: Colin2B on 03/03/2018 23:36:10
Yes - I did notice that the company who web hosted my PDF deleted it, but I'm sure I actually posted those particular 3 arXiv papers in a post specifically addressed to Alan.  I will now go back and see if I can find it.
Ok, i notice you found them, i couldn’t and remembered them in the paper.
Will your revised pdf be too big to post here?
Title: Re: Is there a discrepancy with the equivalence principle?
Post by: timey on 03/03/2018 23:46:50
I haven't actually revised the whole paper as of yet.  What I have done is take on board the non-understandings that I have encountered here, and I am re-arranging the paper accordingly.  I am now, after the abstract, starting with the diagram of the quarter sphere, giving some 'known' context to what is occurring in page 1, 2, 3, 4a, 4b, etc, afterwards.

If I could I would like to be able to run this past you portion by portion, to make sure that it is understandable.  Would that be ok?   
Title: Re: Is there a discrepancy with the equivalence principle?
Post by: timey on 04/03/2018 00:25:23
In any case here is the first portion...

Abstract:

This proposal for a modification of general relativity, providing cause and effect mechanics for the acceleration/attraction of gravity, results in fully described cause and effect mechanics for a very different cyclic bounce cosmology, where our current position, including the entire history of universal development into clumped mass occurs within the 'contraction' period.

General Relativity can already describe a contracting universe, but to modify general relativity, (bearing in mind that physics has no comprehensive theory of time), I add another entirely seperate time function for background space - where the changes in the rate of this time can be considered as the phenomenon from which the effects of gravitational acceleration (not gravitational attraction) are emergent.
This temporal interpretation of space time curvature constitutes a renormalisation term to the general relativity metric resulting in Euclidean geometry, or flat space.

My model incorporates both general and special relativity considerations into a singular space time structure that addresses the quantum/relativity time problem resulting a 'pilot theory' type scenario of position and momentum.

My model makes a definitive falsifiable prediction for a doable experiment.

My model predicts that galaxy clusters will have greater magnitude redshifts than the CMB. (as is observed)


Modifications to General Relativity are:
1/ An additional time phenomenon
2/ An additional axiom "+energy=shorter seconds"
3/ An additional axiom "The speed of light cannot exceed the local rate of time"


1/ Introducing An Additional Time Phenomenon:

So before I introduce my model's additional time phenomenon and it's changes in rate, I will quickly go through some known time dilation considerations - that it be clearer the contruct of what follows.

(https://i.imgur.com/7f966DH.png)

The diagam above is a quarter sphere (purple curve) plus equatorial bulge gradient (blocked green).
Clocks placed at sea level at any placement on that equatorial bulge gradient (red curve/big dots) will all run at same rate.
This is because, from pole to equator, the increase in centripetal speed caused by the increased hieght from centre of earth/planet due to the equatorial bulge gradient - where an increase in speed causes a decreased clock rate - exactly cancels out the increase in the clock rate that is expected of a clock when that clock is positioned at an increased height from centre of earth/planet.
(ie: elevated position in the gravity potential)

Therefore this model makes all it's considerations of the rate of time of 'clocks' on the basis that the observed clock rate is a combination of both general relativity(position in gravity potential) time dilation effects, 'and' special relativity (motion related) time dilation effects.

This model states that these general relativity and special relativity clock timing effects are occurring ONLY for where m does NOT equal zero, (ie: for mass), and states that there is another separate time phenomenon for where m does equal zero. (ie: open space)
That this additional time phenomenon for where m does equal zero occurs concurrently to the timing of clocks, but as a separate issue.
And the changes in the rate of this additional time phenomenon are inversely proportional (ie: change by the same value observed of the clock, but negatively) to the changes in the rate of time for clocks, (ie: where m doesn't equal zero), when clocks are observed in fixed position* with the centripetal motion of the earth/planet in the gravity potential.
(*as opposed to clocks/mass in free motion with respect to the centripetal motion of the earth/planet, in the gravity potential, this being a differently valued consideration, more on the difference here later)
And the specific rates of both time phenomenon are defined (for all but a very tiny part, this being a 'universal contraction' consideration) by the magnitude of the Gravitational Mass.


I then go on to introduce the worksheets outlining the modification, which I can post next, when the above is found to be understandable.
Title: Re: Is there a discrepancy with the equivalence principle?
Post by: Colin2B on 04/03/2018 09:35:33
@timey   “I then go on to introduce the worksheets outlining the modification, which I can post next, when the above is found to be understandable.”

Ok, i understand [or think I do  :). ] what you are saying.

I assume statements like:
“It is the g field between source and observation point that is NOT irrelevant to the redshifts observation”
Will be fully explained as we go forward.
As I say, I’m not concerned whether I agree with your analysis, just whether it is understandable by a reader.
Title: Re: Is there a discrepancy with the equivalence principle?
Post by: timey on 04/03/2018 14:04:50
OK good. But to give you an idea of my philosophy, I don't expect that anyone should agree with my model, in just the same way that I do expect everyone not to agree with the expanding model. This being because both are just models and hypothesis until such time as experimental agreement is attained.  My model provides a falsifiable prediction.  Expanding model doesn't.

But it does concern me that you haven't fully understood why the g field between source and observation is NOT irrelevant in a universe that is slowly contracting under the influence of gravity, so before I get back to my laptop later and post the next portion, I'll have another go at this.

It's about 'initial conditions' Colin.
The initial conditions of expansion are that an almost uniform sea of particles undergo an inflation period that slows down, and then speeds up again as an accelerated expansion.  The g fields between galaxy clusters observed today 'are' irrelevant in this expanding theory.
The initial conditions of my models contraction are an almost uniform sea of particles that entirely filled up all of the spaces between the clumped mass' we see today (and beyond).  The masses we see today are formed of the particles that used to be in those spaces.
What is occurring is a dividing of the gravity field into points of concentrated gravity (mass), and tracts of weakening gravity (fields) between.
Light that was emitted at a point in history ago, will have been emitted in a universe that was less clumped, where the gravity (fields) were of greater magnitude. When the light arrives at observation in present time, the universe has further clumped between then and now, into more concentrated points of gravity (mass), and the gravity (fields) are weaker now than they were when the light left the source.
Light that is travelling through a g field that is weakening in magnitude will be redshifted.

There is nothing in the following worksheets that is going to cover giving an explanation of why the g fields in-between mass will weaken as the clumping process is furthered.  I daresay I will have to describe this in the bounce cosmology chapter.  Is the above description understandable?
Title: Re: Is there a discrepancy with the equivalence principle?
Post by: Colin2B on 04/03/2018 17:38:56
But it does concern me that you haven't fully understood why the g field between source and observation is NOT irrelevant in a universe that is slowly contracting under the influence of gravity, so before I get back to my laptop later and post the next portion, I'll have another go at this.
Problem is with use of term g field. I’ll explain.
We have agreed that the intervening gravitational potential has no end to end influence, however, GP and g are strongly linked. I know I'm teaching granny etc, but for clarity: think of lines of equal GP as contours on a map, the gradient of those lines ie steepness of the slope is g. So if you talk about the g field folks will assume you mean static GP/g field.
I think you have to explain in basic terms, perhaps light passing through a changing mass density. You could also make some assumptions about the size of the volume and change necessary to give the effect you are looking at.
Title: Re: Is there a discrepancy with the equivalence principle?
Post by: timey on 04/03/2018 18:19:14
Ok - I think we are having terminology problems, and that is good, 'cos best I get it sorted now. :)

I have agreed that the difference between gravity potential of source and observation point is relevant but can be negated at present.

Now when I say the difference in gravity potential of source and observation point, what I am talking about is the mass ? of source, and the mass ? of observation point, and the difference in the gravity potential of those masses.

When I say the g field between source and observation point, I am talking about the anistropic nature of gravity in open space and the inverse square law by which gravity gets weaker with distance from mass.

So - as the anistropic nature of the 'g field' of open space becomes weaker due to the gravity potential of masses becoming greater due to the clumping process, light travelling from source, emitted at a point in history, will be travelling across the anistropic nature of the open space g field (between galaxy clusters) that is getting weaker as the light travels, b/c clumping of mass is still occurring while the light travels.
And when that light is viewed 'now' at the observation point, it has been 'gravitationally' redshifted by this ever weakening g -field of open space.

If you can think of any better way to express this terminology-wise I'd be grateful.
I, myself cannot think in terms of a static gravity field, b/c I cannot visualise a means by which a static gravity field may exist in our universe of many masses interacting, but that's just me, and I don't think like everyone else, so best I know about these things.
Title: Re: Is there a discrepancy with the equivalence principle?
Post by: alancalverd on 04/03/2018 18:55:19
Alas, you have got it all back to front. Gravitational redshift is a function of the gravitational potential difference between source and receiver, nothing else. More specifically, unless gravitational travels faster than c (and we have no reason to suspect it does) the shift depends on gsource, then - greceiver, now.
Title: Re: Is there a discrepancy with the equivalence principle?
Post by: timey on 04/03/2018 19:32:21
Alas, you have got it all back to front. Gravitational redshift is a function of the gravitational potential difference between source and receiver, nothing else. More specifically, unless gravitational travels faster than c (and we have no reason to suspect it does) the shift depends on gsource, then - greceiver, now.

Alas you appear to be talking about the expanding model again, where the gravity field of the 'void' of space between the light source and observer,( which we are saying is an observation of light from another galaxy cluster) is irrelevant and redshifts are considered to be the greater part velocity related.

A contracting model will be doing a lot of things back to front.

Again can I ask you please if you actually looked at the 3 arXiv papers that I posted addressed specifically to you @alancalverd

https://www.thenakedscientists.com/forum/index.php?topic=69800.msg535206#msg535206
Title: Re: Is there a discrepancy with the equivalence principle?
Post by: alancalverd on 04/03/2018 23:02:52
It has nothing to do with expansion, contraction, or the hokey-cokey universe. A photon leaves its source and arrives at a detector with a gravitational energy shift dependent only on the difference of g between source and detector. Motion shifts are something else.   
Title: Re: Is there a discrepancy with the equivalence principle?
Post by: timey on 04/03/2018 23:47:44
Do you mean the difference between the size of the masses of the emitting source and the observation point?

Or are you talking about the anistropic changes in the g field that exists in the distance of space that lies between the emitting source and the observation point that the light must travel across?
Title: Re: Is there a discrepancy with the equivalence principle?
Post by: alancalverd on 05/03/2018 07:42:55
KISS* I have only mentioned the difference in local g between the emitter and the receiver, and I don't intend to add any complications. Nothing else contributes to gravitational redshift. As you know, g = - GM/r at the surface of the emitter (or receiver) of mass M and radius r.

*not implying that you are S, but Keep It Simple And Don't Waste Your Genius On Irrelevance hasn't passed into the idiom yet!
Title: Re: Is there a discrepancy with the equivalence principle?
Post by: Colin2B on 05/03/2018 08:44:04
 




If you can think of any better way to express this terminology-wise I'd be grateful.
I, myself cannot think in terms of a static gravity field,
“Static”? Blame auto correct, not what i meant to say.

Problem is one I mentioned early on, that if your falsifiable experiment is correct then your ideas mean current relationship between time dilation and GP/g doesn’t hold so you need a replacement. You somehow have to find a way of describing it -I had assumed it came out of your 3rd time ‘inversion ‘.  As I said earlier, you have to build some scenarios and work them through because it seems to me that the clumping would have to take place at >c.
Title: Re: Is there a discrepancy with the equivalence principle?
Post by: timey on 05/03/2018 12:26:35
@alancalverd.  Unfortunately I am adding the complication of the 'voids' and multiple body masses. Yes it is true that the expanding model 'ignores' these complications, b/c the expansion renders the gravity of clusters to be local and isolated from other clusters by 'expanding' voids of 0 or near 0 g.

@Colin2B
No - my model states the current relationship between the time dilation of a clock and GP/g does hold.
Clocks will obey GR in all but the factor of the greater mass, where b/c clocks run slower nearer to mass, GR states that time runs slower on the bigger mass.
My model states that although all clocks do run slower nearer mass than they do at elevation from mass, that a clocks rate will be faster near a 'bigger' mass than near a 'smaller' mass. The opposite to GR, and this can be tested via the experiment I suggest.
There is no way to test for the separate time phenomenon that my model adds.  The only observation of this time is via its changes, that are inversely proportional to the changes observed in a clock at same position in the gravity potential, and are causing objects in free motion in the gravity potential to be accelerated/decelerated in their motion towards/away from the gravitational mass.

This is what pages 1 through to 3 are about. I will continue in portions later regardless of if Alan is on board with 'contracting' considerations.

But for the mo - how can clumping under the influence of gravity occur at c?
Clumping will occur at whatever rate of gravitational force the gravitational mass is exerting on the local field.
Near a black hole clumping might well occur at c, when c is held relative to our rate of time. (where orbitals will slow the process down somewhat)
But a smaller body mass will not have objects gravitating towards it at c.
An almost uniform sea of particles and energy contracting under the influence of its own gravitational anomalies will start clumping/contracting very slowly indeed, and will proceed at an accelerating rate.
Title: Re: Is there a discrepancy with the equivalence principle?
Post by: alancalverd on 05/03/2018 14:01:24
Unfortunately I am adding the complication of the 'voids' and multiple body masses. Yes it is true that the expanding model 'ignores' these complications, b/c the expansion renders the gravity of clusters to be local and isolated from other clusters by 'expanding' voids of 0 or near 0 g.
Unfortunate indeed, because by experiment we know that any model must ignore them, because they have no effect in reality. If your model is to be taken seriously, it must be consistent with what we already know.
Title: Re: Is there a discrepancy with the equivalence principle?
Post by: alancalverd on 05/03/2018 14:07:54
But a smaller body mass will not have objects gravitating towards it at c.
Classically, there is no upper limit to the speed of any two objects approaching under gravity. Relativity tells us that the upper limit is c, which defines the event horizon. Hence black holes and Hawking radiation.
Title: Re: Is there a discrepancy with the equivalence principle?
Post by: timey on 05/03/2018 14:42:47
And could you please describe 'why' a model that is slowly contracting will ignore the fact of voids and multiple masses?
By definition, to do so would be highly illogical.

And what exactly is it that you are stating that we already know?
Title: Re: Is there a discrepancy with the equivalence principle?
Post by: alancalverd on 05/03/2018 19:54:46
We already know that gravitational redshift depends only on gsource - greceiver. Same as the height difference between your house and mine doesn't depend on the mountains in between.
Title: Re: Is there a discrepancy with the equivalence principle?
Post by: timey on 05/03/2018 20:59:44
Which experimental test did this knowledge arrise from Alan?
Title: Re: Is there a discrepancy with the equivalence principle?
Post by: alancalverd on 05/03/2018 23:05:45
Every astronomical observation, every repeat and variation of the  Pound-Rebka experiment, GPS satellites, radiofrequency shift from deep space probes, and pure common sense.  Just like the hills between houses: if a photon is blueshifted as it approaches a gravitational well en route to a distant receiver, it is equally redshifted as it leaves that well, so the net shift depends only on the difference in gravitational potential between the source and the receiver.
Title: Re: Is there a discrepancy with the equivalence principle?
Post by: timey on 05/03/2018 23:18:33
Every astronomical observation is calculated under the contruct you put forward, but does not serve as experimental evidence that there is no relevant gravity in a void.

Unless I completely missed something and the Pound Rebka experiment was conducted in a 'void' between galaxy clusters, this also does not serve as experemental evidence that there is no relevant gravity present in a void.

Have they installed GPS in the void?

Have deep space probes probed the voids?

If there is gravity in a void, of which there is no experimental evidence that there isn't, then it will cause gravitational shifts.

In a universe that is slowly contracting under the influence of gravity, the gravity of the voids will be highly relevant with regards to gravitational shift.
Title: Re: Is there a discrepancy with the equivalence principle?
Post by: Colin2B on 06/03/2018 08:35:57
No - my model states the current relationship between the time dilation of a clock and GP/g does hold.
Clocks will obey GR in all but the factor of the greater mass, where b/c clocks run slower nearer to mass, GR states that time runs slower on the bigger mass.
As I’ve said near the beginning, this will need some significant explanation because the 2 statements appear to contradict each other. You cannot keep the current relationship and have clocks not follow that relationship nr a density anomaly.

An almost uniform sea of particles and energy contracting under the influence of its own gravitational anomalies will start clumping/contracting very slowly indeed, and will proceed at an accelerating rate.
As I’ve said before, if your assertion is that light enters an area of uniform sea of particles, becomes shifted and then leaves a void area which does not affect it’s shift, then you will have to provide some scenarios and details. But it seems to me that the clumping has to occur at a significant rate so that the changes can occur while the light  is passing through the clumping zone. Gravitational shifts are very small compared to doppler, so a scenario is important to prove the magnitude required.
You will also need to explain how your idea overcomes 2 objections you will meet as your paper is read.
1. If light enters a dense area that is expanding it will be subject to the same g forces as it enters, passes through and leaves that zone as the particles that are clumping, so it is still going to have the same uphill/downdale effect. How does your void idea actually work in practice?
2. Which sources you are observing. If we take starlight then the star must have already formed during clumping, and certainly if we can see it then the light hasn’t passed through a sea of particles or that would show up in absorption spectra.
As I’ve said, i had assumed the whole idea would be explained by the inverse time dilation rather than voids.
Title: Re: Is there a discrepancy with the equivalence principle?
Post by: alancalverd on 06/03/2018 12:09:19
Every astronomical observation is calculated under the contruct you put forward, but does not serve as experimental evidence that there is no relevant gravity in a void.
An observation is not a calculation. It is a fact that demands explanation by calculation.

Quote
Unless I completely missed something and the Pound Rebka experiment was conducted in a 'void' between galaxy clusters, this also does not serve as experemental evidence that there is no relevant gravity present in a void.
Of course there are gravitational fields in the space between things. They just don't happen to be relevant to the observed redshift between two points. If they were, you could extract energy from space!

Specifically, the PR experiment shows perfect antisymmetry between up and down photons, and exactly the calculated value of shift compared with the horizontal frequency, taking into account only the gravitational potential difference between source and receiver. As the gravitational field is divergent, you would get a different result if the intervening field was indeed significant.

Quote
Have they installed GPS in the void?
AFAIK the satellites whizz about between the earth and the moon, and the space between the satellite and the receiver is are also subject to the varying gravitational field of everything else in the universe, but the frequency difference between clocks only depends on the relative gravitational potential difference between them.

Quote
Have deep space probes probed the voids?
depends on your definition of void. Obviously if you are going from A to B, the space you are travelling in contains at least A and B. AFAIK there is no frequency shift ofthe transmitter caused by intervening stuff.

Quote
If there is gravity in a void, of which there is no experimental evidence that there isn't, then it will cause gravitational shifts.
Lensing, yes. Frequency shift, no.

Quote
In a universe that is slowly contracting under the influence of gravity, the gravity of the voids will be highly relevant with regards to gravitational shift.
"gravity of the voids" implies a source of graviation with no mass. Observation?

Title: Re: Is there a discrepancy with the equivalence principle?
Post by: timey on 06/03/2018 13:38:48
No - my model states the current relationship between the time dilation of a clock and GP/g does hold.
Clocks will obey GR in all but the factor of the greater mass, where b/c clocks run slower nearer to mass, GR states that time runs slower on the bigger mass.
As I’ve said near the beginning, this will need some significant explanation because the 2 statements appear to contradict each other. You cannot keep the current relationship and have clocks not follow that relationship nr a density anomaly.

An almost uniform sea of particles and energy contracting under the influence of its own gravitational anomalies will start clumping/contracting very slowly indeed, and will proceed at an accelerating rate.
As I’ve said before, if your assertion is that light enters an area of uniform sea of particles, becomes shifted and then leaves a void area which does not affect it’s shift, then you will have to provide some scenarios and details. But it seems to me that the clumping has to occur at a significant rate so that the changes can occur while the light  is passing through the clumping zone. Gravitational shifts are very small compared to doppler, so a scenario is important to prove the magnitude required.
You will also need to explain how your idea overcomes 2 objections you will meet as your paper is read.
1. If light enters a dense area that is expanding it will be subject to the same g forces as it enters, passes through and leaves that zone as the particles that are clumping, so it is still going to have the same uphill/downdale effect. How does your void idea actually work in practice?
2. Which sources you are observing. If we take starlight then the star must have already formed during clumping, and certainly if we can see it then the light hasn’t passed through a sea of particles or that would show up in absorption spectra.
As I’ve said, i had assumed the whole idea would be explained by the inverse time dilation rather than voids.

No - they don't contradict each other.  There is no contradiction in stating that clocks run slower nearer mass, and stating that the clock nearer the bigger mass runs faster.

All I am saying here is, that the electron transitions of the mass elevated in the field of the gravitational Mass will have greater degrees of freedom to tick faster in the weaker field, and that time in the field itself is separate and has nothing to do with what happens for clocks/mass.

And the 'almost uniform sea of particles' that are my models initial conditions (post big bang/inflation) for universal contraction will not be producing any light before it has clumped into light producing mass.

My assertion is that an observation of light that was produced in a distant galaxy cluster 'has' to cross a void before we can observe it.
In the expanding theory that void is considered to be 0, or near 0 g because the voids are expanding and gravitational redshifts is source gravity minus observation gravity, and the rest of the resdshift is velocity related.
But in a model that is slowly contracting from initial conditions into clumped mass under the influence of gravity, the voids between masses will not have 0, or near 0 gravity. Redshift observations will be 8% velocity related redshift, and 92% gravitational redshift.

This can be evidenced in the fact that observational data shows that the CMB has an 8% lesser redshift than galaxy clusters. (where this observational data does not support expanding theory)

1/ I don't understand why you are visualising a dense area expanding, in a model of a universe that is slowly contracting from the initial conditions of an almost uniform sea of particles.

2/ I have already answered above as to the starlight question.

No - sorry, but the system of changes in time that I propose only describes 'why' an object in space is accelerated/decelerated by what we call 'gravity', and 'why' via c^2/R, this will cause an accelerated universal contraction.
The time changes can be considered just as 'gravity'.

I cannot 'make' you visualise a universal contraction from initial conditions of an almost uniform gravity field, under the influence of gravity.
But I can assure you that under this remit, an observation of light from 10 billion years ago, that has travelled a void, or voids, has travelled in a universe where there has been 10 billion years of further dividing of the field into points of concentration, and widening tracts of weakening anisotropic gravity fields when it reaches us.
This will cause redshift. The further away the source, the greater the redshift.
Title: Re: Is there a discrepancy with the equivalence principle?
Post by: timey on 06/03/2018 13:48:30
@alancalverd
The point is Alan, is that there is no experimental evidence that there 'isn't' significant gravity in the voids.  And there is no experimental evidence that redshifts are for the greater part velocity related.
That's all that is relevant with regards to my model.

You said:
""gravity of the voids" implies a source of graviation with no mass. Observation?"

The mass of the galaxy clusters Alan.
Title: Re: Is there a discrepancy with the equivalence principle?
Post by: alancalverd on 06/03/2018 14:28:45
Of course there are gravitational fields everywhere. The universe couldn't clump, form planets, or have visiting comets otherwise. So by "gravity of the voids" I assume you just mean "gravitational fields", as understood by everyone else, i.e. gravity in the voids

I thought you were objecting to velocity redshift, the observation of which is consistent with an accelerating and expanding universe, being significant.
Title: Re: Is there a discrepancy with the equivalence principle?
Post by: timey on 06/03/2018 14:49:46
I'm not 'objecting' to anything.
I am simply proposing something completely different that also fits with observation...
...and I provide a prediction that can falsify/prove the proposal via suggested experiment. (that is a further test of general relativity that has so far not been conducted)
Title: Re: Is there a discrepancy with the equivalence principle?
Post by: Colin2B on 06/03/2018 17:08:01
@timey “I cannot 'make' you visualise a universal contraction from initial conditions of an almost uniform gravity field, under the influence of gravity.”

I’m not trying to. What i am doing is reading what you say as if i am reading your paper and asking myself where people will see sticking points, questions etc. You have a choice, you can anticipate and  explain these issues in your paper, or you can wait for the questions to be asked. But what if the questions, and hence the endorsement, never come.
Title: Re: Is there a discrepancy with the equivalence principle?
Post by: alancalverd on 06/03/2018 17:33:52
I'm not 'objecting' to anything.
I am simply proposing something completely different that also fits with observation...
...and I provide a prediction that can falsify/prove the proposal via suggested experiment. (that is a further test of general relativity that has so far not been conducted)

I refer the honorable lady to reply #966 above
Title: Re: Is there a discrepancy with the equivalence principle?
Post by: timey on 06/03/2018 18:22:33
@timey “I cannot 'make' you visualise a universal contraction from initial conditions of an almost uniform gravity field, under the influence of gravity.”

I’m not trying to. What i am doing is reading what you say as if i am reading your paper and asking myself where people will see sticking points, questions etc. You have a choice, you can anticipate and  explain these issues in your paper, or you can wait for the questions to be asked. But what if the questions, and hence the endorsement, never come.

@Colin2B And I am answering them aren't I?.
I have told you that I am not describing a redshift observation of light moving through a uniform sea of particles.
I have told you that I am not describing a redshift observation of light moving through a dense area that is expanding.
I have told you that light passing by an intermediate gravity concentration surrounding a body of mass will not be affected with respect to the observed magnitude of redshift. (Same as expanding theory)

I have told you that (in my model) light emitted from a distant galaxy 10 billion years ago will travel through voids that are weakening in gravity as matter further clumps 'while' the light is travelling, and this will cause a gravitational redshift to the travelling light.
The further away the source, the greater amount of time the light has been in a weakening field, the greater the redshift. That is what we observe.  I have interpreted the observed data differently.
The void will be weakening because the galaxies of galaxy clusters are converging and as they get closer together within each cluster, the voids between weaken in gravity.
Is that understandable?

Edit: I have also directed you to the fact that the observed data CMB redshifts are 8% lesser than the observed data galaxy cluster redshifts. (doesn't support the expanding theory)  And have explained how my model states the CMB redshifts as 100% gravitationally shifted, and that the galaxy clusters are 8% velocity shifted, and 92% gravitationally shifted.  These are figures, for my model, that are based on observed data.
Title: Re: Is there a discrepancy with the equivalence principle?
Post by: timey on 06/03/2018 18:25:43
I'm not 'objecting' to anything.
I am simply proposing something completely different that also fits with observation...
...and I provide a prediction that can falsify/prove the proposal via suggested experiment. (that is a further test of general relativity that has so far not been conducted)

I refer the honorable lady to reply #966 above

@alancalverd. What of post 966?  Are you reposting those questions that my following posts already answered?
Or should I re-phrase to make it clear that I am saying that I do not 'object' to the expanding theory, but am proposing something different?
Title: Re: Is there a discrepancy with the equivalence principle?
Post by: alancalverd on 06/03/2018 19:31:08
Very simply, you have to explain why the observed red shift of distant objects is greater than that of nearby objects of similar mass. As I have explained since #966, you can't blame anything (or even a vast expanse of nothing) between the source and the observer. Your model has to consider only the gravitational red shift of the source and its motion relative to the observer.
Title: Re: Is there a discrepancy with the equivalence principle?
Post by: timey on 06/03/2018 21:09:42
Very simply, you have to explain why the observed red shift of distant objects is greater than that of nearby objects of similar mass. As I have explained since #966, you can't blame anything (or even a vast expanse of nothing) between the source and the observer. Your model has to consider only the gravitational red shift of the source and its motion relative to the observer.

In my model that has been slowly contracting from an almost uniform sea of particles and energy:
Light emitted from a distant galaxy (already clumped) 10 billion years ago, will travel through voids that are weakening in gravity as matter further clumps (over 10 billion years) 'while' the light is travelling, and this will cause a gravitational redshift to the travelling light.

The further away the source, the greater amount of time the light has been in a weakening field, the greater the redshift.

That is pretty simple I think...
I started talking figures of magnitude in post 1002 if you would care to read it...
Title: Re: Is there a discrepancy with the equivalence principle?
Post by: alancalverd on 07/03/2018 00:32:21
Pretty simple, indeed, but nothing to do with physics.

And I'm afraid the numbers in #1002 do not make sense.

Quote
The largest observed redshift, corresponding to the greatest distance and furthest back in time, is that of the cosmic microwave background radiation; the numerical value of its redshift is about z = 1089

Quote
The most reliable redshifts are from spectroscopic data, and the highest confirmed spectroscopic redshift of a galaxy is that of GN-z11 with a redshift of z = 11.1

The "8%" you talk about is not a variation in redshift but the discrepancy between two values of the Hubble parameter  (it isn't really a constant) as calculated from two different measurements of redshift. That doesn't immediately imply anything about a model of the universe since the stellar data is derived from numerous events over a wide range of times and a fair bit of guesswork.
Title: Re: Is there a discrepancy with the equivalence principle?
Post by: timey on 07/03/2018 00:40:34
Yes - there is a descrepancy in the Hubble parameter, based on redshift observations of the CMB and galaxy clusters.
On the basis that we are talking about the expanding model now, this anomaly suggests that the CMB is expanding at an 8% lesser speed than the galaxy clusters are, and this anomoly does not fit with expanding theory.

Edit:
https://www.space.com/38496-hunt-for-hubble-constant-standard-model.html
"Using the Hubble Space Telescope to measure the light from a number of type 1a supernovas, a 2001 study revealed an 8 percent discrepancy in the value for the Hubble constant when compared with the method using precise measurements of the CMB. In other words, the Hubble Space Telescope recorded a faster rate of universal expansion than CMB measurements did."
Title: Re: Is there a discrepancy with the equivalence principle?
Post by: alancalverd on 07/03/2018 15:39:25
recorded a faster rate of universal expansion than CMB measurements did."
No it didn't. It simply
Quote
revealed an 8 percent discrepancy in the value for the Hubble constant when compared with the method using precise measurements of the CMB
. The "rate of universal expansion", indeed the entire concept of universal expansion,  is an inference from redshift, not  a measurement.

Don't confuse journalism with science. Remember that the Hubble "constant" is nothing more than the slope of a line through a number of measurements of stellar redshift. The only reason it is called a constant is because the experimental line is fairly straight. Its value has varied from around 500 (Hubble, 1929) to 55 (Sandage and Tallemann, 1970) and is currently estimated to be around 71 from telescope data. A variance of 8% from an independent measurement of CMB is not a big deal, and I would advise you not to get excited by numbers at this stage of the game.
.
The physics aspect is the fact that distant objects mostly seem to have larger redshifts than near ones, which suggests expansion, but the fact that some near ones (e.g.Andromeda) are blueshifted says that the expansion is not uniform.
Title: Re: Is there a discrepancy with the equivalence principle?
Post by: timey on 07/03/2018 17:46:58
For crying out loud Alan, universal expansion is inferred b/c redshifts measurements are being 'interpreted' as the greater part velocity related shifts.
My model simply infers that the universe is contracting via a re-interpretation of the redshifts observation as being the greater part gravitationally shifted.

I know perfectly well what the Hubble ladder is thank you, and far from getting excited about journalism, I could have posted 1 of a dozen or more arXiv papers (peer reviewed) that concern the CMB Hubble telescope discrepancy, but was trying to make the info accessible for those people who do not read the arXiv links I post.
Sorry, but you are wrong.  The CMB redshifts measurements 'inferred' that the recessional velocity of the CMB is 8% lesser (or as little as 2.4%, dependent on who's error margins we are talking about) than the recessional velocity of galaxy clusters.
This is a fact that has 'professional' theoretical physicists confused, as it does not support expanding theory.

At this stage in the game Alan, all I want to do is calculate. I know exactly what I want to calculate, and have a pretty good idea of the type of calculation required, but do not know how to formalise the notation, this being what I am asking for help with.

Now it matters not to my model if the observational discrepancy is a lesser percentage. The parameters are adjustable here and do not require a specific value.

What I would like to do now is go back to the 'table' I posted earlier @Colin2B and explain to you how this reinterpretation of the redshift observations being the greater part gravitational fits in with c^2/R being an observed acceleration value.
Title: Re: Is there a discrepancy with the equivalence principle?
Post by: alancalverd on 07/03/2018 18:29:10
The CMB redshifts measurements 'inferred' that the recessional velocity of the CMB is 8% lesser (or as little as 2.4%, dependent on who's error margins we are talking about) than the recessional velocity of galaxy clusters.

That would be entirely consistent with the notion of the observable universe expanding, driven by the gravitational attraction of all the matter outside the Schwarzchild radius. The CMB, having no mass,  is not subject to gravitational acceleration.
Title: Re: Is there a discrepancy with the equivalence principle?
Post by: timey on 07/03/2018 18:48:15
Yes well - if it is your fancy to disagree with the professional theoretical physicists, who consider that the discrepancy is a problem for the expanding theory, and work on these problems as part of their bread and butter, then that is your opinion and I will not argue with you on it.

However, your opinion on these matters has no bearing on calculating, or even being open to considering an alternative contracting model, does it?

The CMB is radiation, and radiation shifts, both gravitationally (in the gravity field) & velocity related. (edit: evidenced in the fact that a recessional velocity has been given to the CMB)
In the expanding theory, a nobel prize was awarded for the discovery that the cosmological observations of light from supernova's indicate that there is an acceleration of the expansion.
In my contracting model, there is an acceleration of the contraction, and the value of c^2/R is relevant.

Is there any chance that we ONLY discuss my contracting model from now on Alan?
Title: Re: Is there a discrepancy with the equivalence principle?
Post by: timey on 08/03/2018 01:37:45
Pretty simple, indeed, but nothing to do with physics.

Yes - you are abslolutely right, it's not physics...
And I'm not pretending that it is. What it is is a hypothesis of a model, in much the same way as the Cold Dark Matter Model is not physics but a hypothesis of a model.

It is only experimental evidence that will result in CDMM being 'physics', where experiments are being conducted.

It is only experimental evidence via the experiment that I suggest that will prove my model as being physics.

"Tis the mark of an educated mind to be able to entertain a thought without accepting it"

Isn't it?
Title: Re: Is there a discrepancy with the equivalence principle?
Post by: alancalverd on 08/03/2018 07:05:54
radiation shifts, both gravitationally (in the gravity field)
Wrong. As it has been since you first suggested it. Redshift depends only on the gravitational potential difference between source and receiver.

You can't pick and choose which experimental facts fit your model, and you certainly can't invent some that plainly aren't true. Well, you can, but nobody will take you seriously: this is astrophysics, not politics.

The expanding universe has a lot of associated problems, which is why I don't defend a simple version of it. But whatever else you propose must not deny the facts. By all means consider a contracting or clumping universe, but don't kid yourself that redshift will fit your model - it is the job of a model to fit reality.
Title: Re: Is there a discrepancy with the equivalence principle?
Post by: alancalverd on 08/03/2018 07:13:02

radiation shifts, both gravitationally (in the gravity field)
Wrong. As it has been since you first suggested it. Gravitational redshift depends only on the gravitational potential difference between source and receiver.

You can't pick and choose which experimental facts fit your model, and you certainly can't invent some that plainly aren't true. Well, you can, but nobody will take you seriously: this is astrophysics, not politics.

The expanding universe has a lot of associated problems, which is why I don't defend a simple version of it. But whatever else you propose must not deny the facts. By all means consider a contracting or clumping universe, but don't kid yourself that redshift will fit your model - it is the job of a model to fit reality.

Curiously, I've just been arguing with some transport planners who want to pour more concrete over the countryside "because the road [past my house] is congested". I pointed out that the peak rush hour traffic moves at an average of 55 mph so adding a bus lane won't help as buses can't go any faster, and that doesn't fit with any sane definition of congestion. "But the computer model says the road is congested". "Have you actually measured the traffic flow?" "No. We have a computer model."
Title: Re: Is there a discrepancy with the equivalence principle?
Post by: timey on 08/03/2018 12:32:02
If the zero mass CMB radiation can be velocity related shifted in an expanding near zero gravity field... (expanding theory)
...then the zero mass CMB radiation will also be shifted by a gravity field that is changing in magnitude as the universe contracts under the influence of gravity... (my contracting model)

That 'is' a fact, b/c if the first holds true, then the second must also hold true, and if I knew how to make a poll from my phone I'd put it to the rest of the forum.
@Colin2B
@jeffreyH
@evan_au
@chiralSPO
Can you add your opinion please?
Title: Re: Is there a discrepancy with the equivalence principle?
Post by: Colin2B on 08/03/2018 15:16:18
If the zero mass CMB radiation can be velocity related shifted in an expanding near zero gravity field... (expanding theory)
...then the zero mass CMB radiation will also be shifted by a gravity field that is changing in magnitude as the universe contracts under the influence of gravity... (my contracting model)

That 'is' a fact, b/c if the first holds true, then the second must also hold true,
The 2nd does not follow logically from the first as there is no causal connection. You have to demonstrate a mechanism for the 2nd statement as an independent premise. You may feel you have done that.
Title: Re: Is there a discrepancy with the equivalence principle?
Post by: timey on 08/03/2018 16:11:44
In the first statement, (expanding theory) the causal mechanism for the velocity related shift is the expansion of the field.

In the second statement, (my contracting model) the causal mechanism for gravitational related shift is that matter further clumping over time under the influence of gravity causes changes to the magnitude of gravity 'in' the field.

So yes I do feel that I have demonstrated a mechanism for the 2nd statement as an independent premise.
Title: Re: Is there a discrepancy with the equivalence principle?
Post by: alancalverd on 09/03/2018 01:35:57
But intervening gravitational fields have no effect on the perceived red shift of a distant object.
Title: Re: Is there a discrepancy with the equivalence principle?
Post by: timey on 09/03/2018 02:53:33
But intervening gravitational fields have no effect on the perceived red shift of a distant object.

So you keep saying, but what experimental test has been conducted in the intervening fields for you to state this as factual physics, Alan?

The answer is none.

Therefore, it is not proven that intervening gravitational fields have no effect on the perceived red shift of a distant object, and in posting your above post as fact, you are incorrect.

If you continue to make the above statement as an objection to my model you will be guilty of what is known as 'crank' behaviour.
Title: Re: Is there a discrepancy with the equivalence principle?
Post by: captcass on 09/03/2018 05:28:14
Having been analysing the situation, my analysis further compounded through having discussed these matters here at this site for last 2 years, my diagnosis is that there seems to be some confusion surrounding the interpretation of the equivalence principle.

One of the postulates of SR, upon which GR is built, is the EP. In SR we begin with 2 observers who have EQUAL tick rates and length of a meter. They are then separated and put in motion and the Lorentz contractions kick in and we get each observer seeing the other's tick rate as slower and meter as shorter.
 GR describes the relative motion based upon these contractions.
What gets confusing is that GR describes the actual reality we live in, even though we know it is the illusion. We know that it is the illusion, because we stipulate it is so as the EP postulate in SR. We are just stuck with the illusion as our reality.
This is why Einstein said, "Reality is just an illusion, albeit a darned persistent one.".
The EP is correct. each observer, no matter where in the universe, even at the event horizon of a black hole, experiences the same tick rate and length of a meter in their inertial frame as everyone else.
This very much implies that the "true motion" of objects can be determined from their relative motion, just as the "true motion" of a target on a radar screen can be determined by designating one's own motion as  "true motion" and then using the target's "relative motion" to your designated "true motion".to determine the target's "true motion".
I can't give you math on this yet. I have been working in that direction as a part of a larger concept and I expect the math to be complicated.
However, as it is stipulated in SR, and since our actual reality is built upon the relativistic postulates, we must accept that there is a universal tic k rate for all observers, a universal second.
Title: Re: Is there a discrepancy with the equivalence principle?
Post by: alancalverd on 09/03/2018 08:25:52
So you keep saying, but what experimental test has been conducted in the intervening fields for you to state this as factual physics, Alan?

https://www.uam.es/personal_pdi/ciencias/jcuevas/Teaching/GPS_relativity.pdf is a very  readable account of the errors affecting GPS timing.

The gravitational field of the moon is enormous, and enormously variable  - it causes all the water on the earth to move, every day! But it doesn't appear to concern the guys responsible for your satellite navigation system.

Interestingly, you can find plenty of maps of the gravitational geoid, which show that your proposed experiment (or at least its inverse) of clocks at the same gravitational potential and different radii from the centre of the earth is being conducted continuously by the GPS.
Title: Re: Is there a discrepancy with the equivalence principle?
Post by: timey on 09/03/2018 12:04:31
Interestingly, you can find plenty of maps of the gravitational geoid, which show that your proposed experiment (or at least its inverse) of clocks at the same gravitational potential and different radii from the centre of the earth is being conducted continuously by the GPS.

No it isn't.

The experiment that I suggest has not EVER been conducted.

But it will be conducted when NIST achieve their plans on getting their lab sensitive clocks portable.
It was actually around two years ago that NIST were saying that their portable clocks would be ready in 2 years.

Edit: There isn't any part of that PDF that my proposal changes.  My model is compatible with both special and general relativity principles.
Title: Re: Is there a discrepancy with the equivalence principle?
Post by: Colin2B on 09/03/2018 14:56:50
But it will be conducted when NIST achieve their plans on getting their lab sensitive clocks portable.
It was actually around two years ago that NIST were saying that their portable clocks would be ready in 2 years.
They are testing them at the site in Boulder. Various locations around the site are being accurately surveyed for height and gravimeters are being used to measure g and GP at that point. Haven’t heard any anomoly reports yet.

Have you thought of any other tests? I’m thinking places where light is passing through varying g field. How about heavy binary systems, there must be some where field is large and varies quickly.
Title: Re: Is there a discrepancy with the equivalence principle?
Post by: alancalverd on 09/03/2018 16:36:19
I’m thinking places where light is passing through varying g field.
Like the space between the moon and earth? Very well characterised, significant and highly predictable variation in gravitational field, with several very precise clocks flying through it, plenty of stars (including a very bright one with a spectrum that has been studied for over 400 years) outside it, and no apparent effect on the red shift of anything.

Exactly as one would expect, but don't let the facts get in the way of a good hypothesis.
Title: Re: Is there a discrepancy with the equivalence principle?
Post by: jeffreyH on 09/03/2018 16:47:36
Very simply, you have to explain why the observed red shift of distant objects is greater than that of nearby objects of similar mass. As I have explained since #966, you can't blame anything (or even a vast expanse of nothing) between the source and the observer. Your model has to consider only the gravitational red shift of the source and its motion relative to the observer.

In my model that has been slowly contracting from an almost uniform sea of particles and energy:
Light emitted from a distant galaxy (already clumped) 10 billion years ago, will travel through voids that are weakening in gravity as matter further clumps (over 10 billion years) 'while' the light is travelling, and this will cause a gravitational redshift to the travelling light.

The further away the source, the greater amount of time the light has been in a weakening field, the greater the redshift.

That is pretty simple I think...
I started talking figures of magnitude in post 1002 if you would care to read it...

Hold that thought.
Title: Re: Is there a discrepancy with the equivalence principle?
Post by: Colin2B on 09/03/2018 17:00:43
Like the space between the moon and earth? Very well characterised, significant and highly predictable variation in gravitational field,
I agree, but I was looking for something that might convince Timey.
Title: Re: Is there a discrepancy with the equivalence principle?
Post by: captcass on 09/03/2018 17:22:28
On the one hand what I observe is that there is a school of thought that states that a caesium atomic clock placed at a higher gravity potential only 'appears' to have a higher frequency from the perspective of the lower gravity potential...
And that if one places oneself at the higher gravity potential with the clock, then the frequency of the clock will be the same as it was in the lower gravity potential, and that it will now 'appear' to you that the lower gravity potential clock has a lower frequency.
There are 3 points of view to consider. The observer located AT the higher potential, the observer located AT the lower potential, and an outside observer.
In a gravitational dilation field, (not a motion induced field), all three observers will agree the higher potential clock has a higher tick rate. But all three experience the same tick rate in their own inertial frame. The observer always experiences a 1 s/s tick rate in his/her inertial frame. If it wasn't, c would not be a constant, and c has been confirmed over and over again.
It is important to note that ALL of this is observer dependent......there is no illusion without an observer..
Title: Re: Is there a discrepancy with the equivalence principle?
Post by: timey on 09/03/2018 18:23:58
But it will be conducted when NIST achieve their plans on getting their lab sensitive clocks portable.
It was actually around two years ago that NIST were saying that their portable clocks would be ready in 2 years.
They are testing them at the site in Boulder. Various locations around the site are being accurately surveyed for height and gravimeters are being used to measure g and GP at that point. Haven’t heard any anomoly reports yet.

Have you thought of any other tests? I’m thinking places where light is passing through varying g field. How about heavy binary systems, there must be some where field is large and varies quickly.


I had heard that NIST did get the geodesics team in to precisely measure g and gp at the Boulder site in order to narrow down error margins.  I had not heard that they are testing portable clocks yet.  Can you direct me to a link for that?

The gp aspect of general relativity is proven already by NIST' clocks.
With regards to the g aspect, although there may be slight g anomalies at the Boulder site, it would be quite hard to tell an anomaly from an error margin when the clocks are being measured for g differences on the same site.  Not impossible though.
However, you do realise that if NIST had detected any departure from general relativity in conducting such a test of a clock regarding g, that we would not hear of it until many tests had been conducted, where it is noted that NIST have also not released any statement saying that they have now proven the g aspects of general relativity with regards to the ticking of a clock.

Yes - there may be a way to test for g by placing a lab sensitive clock at one of the gravitational wave experiment sites.  The incoming gravity of the wave will be a +gravity for the duration of the wave.
It is not entirely clear to me if the +gravity of a wave would shift a clock, but the minus gravity of just a few inches of difference in height from a gravitational mass does shift a clock.  So if the +gravity of the incoming wave was of the same magnitude difference as the -gravity at few inches elevation, then a clock at a gravitational wave site would be shifted.

Similarly - the GPS system, in that it keeps a record of all previous data, was utilised to search for dark matter.
It didn't find any, however the computer that analysed the data was no doubt programmed to search for points in history where the clocks recorded slower time.
Again, it may be unlikely that the GPS system would be sensitive enough to detect shifts due to gravitational waves - but if it were then I would ask the computer to look for the dates and times that we know gravitational waves were passing, and include a remit for identifying a faster tick rate as well as a slower.
Title: Re: Is there a discrepancy with the equivalence principle?
Post by: captcass on 09/03/2018 20:15:28
It is not entirely clear to me if the +gravity of a wave would shift a clock, but the minus gravity of just a few inches of difference in height from a gravitational mass does shift a clock.  So if the +gravity of the incoming wave was of the same magnitude difference as the -gravity at few inches elevation, then a clock at a gravitational wave site would be shifted.

Similarly - the GPS system, in that it keeps a record of all previous data, was utilised to search for dark matter.
It didn't find any, however the computer that analysed the data was no doubt programmed to search for points in history where the clocks recorded slower time.
Again, it may be unlikely that the GPS system would be sensitive enough to detect shifts due to gravitational waves
A gravity wave is an acceleration in tick rate shifting through the continuum. Therefore clocks do exhibit an increase in tick rate while the gravity wave passed through. We would need a much more sensitive clock than we have today to measure the increase, however.
Cesium clocks can measure a tick rate down to about 2*10^-9 seconds while LIGO can measure down to 10^-21 seconds.
Title: Re: Is there a discrepancy with the equivalence principle?
Post by: captcass on 09/03/2018 20:43:58
Cesium clocks can measure a tick rate down to about 2*10^-9 seconds
Sorry, that should be 10^-14 seconds for cesium clocks.
Title: Re: Is there a discrepancy with the equivalence principle?
Post by: timey on 09/03/2018 21:16:40
@captcass Do you mind me asking you who you are and what your status is, if any, as to physics education?
It's just that you have said something that does interest me, but is also completely contrary to conventional physics theory.  You haven't clarified that you are making a change to conventional physics, so perhaps you are simply confused?

A gravity wave is an acceleration in tick rate shifting through the continuum. Therefore clocks do exhibit an increase in tick rate while the gravity wave passed through.
Title: Re: Is there a discrepancy with the equivalence principle?
Post by: captcass on 10/03/2018 00:03:08
Do you mind me asking you who you are and what your status is, if any, as to physics education?
Hi Timey. I started another thread in the Lighter Side forum that deals with the Hubble Shift and time dilation and Colin2B and another person considered whether it was like your idea.
So yesterday I was just checking out other threads for the first time and saw yours and thought I would drop in. When I saw you wonder if there could be some confusion about the EP I thought I might be able to help clarify.
I am a Cum Laude graduate of a federal academy, '72.  Since then I have studied relativity, cosmology and quantum physics, I can read Einstein in the original.
For the last several years I have been looking at things from the time aspect. It is, after all, spacetime. I don't want to hijack your thread as I am way off mainstream. I am too the point where I believe simple fluctuations in the rate of time might be able to explain it all, including the origin of mass and energy.
I haven't had time to read your whole 21 pages here to see what you folks have covered, but I thought I would try to help clarify the EP thing mentioned in theyour first post, and the GPS measuring thing in the last.
I know the standard current theories, and am fairly sure what I have added to the thread so far is correct from that aspect, though you might see some unorthodox coloring to it in the time aspect.
Sorry if I shouldn't have interjected. 
Title: Re: Is there a discrepancy with the equivalence principle?
Post by: timey on 10/03/2018 00:35:53
Sorry if I shouldn't have interjected.

I think I had a speed read over your thread a few days ago.  Colin's assessment was correct with regards to the difference between your idea and mine in his reply to Thebox. What I am doing is way more radical in that my model of a cyclic bounce cosmology universe is 'currently' contracting.

I daresay it would be some venture to try to read the whole of this thread, and I wouldn't recommend it tbh.
I am, at this stage, writing up a paper on my model to submit to arXiv scientific journal, where Colin, Alan, and Jeff are helping me out with a bit of feedback and hopefully some maths expertese.

The name of the thread is due to the nature of the conversation I was having with Mike at the time, and doesn't have much bearing on the content of the thread as such.
No problem for interjection, although I am very focussed here at the moment in my own task and do not wish to debate here.
Nice to meet you, and if you are at-all interested in my theory of time and modification of GR, here is the place where I start introducing the first portion of my revised paper.

https://www.thenakedscientists.com/forum/index.php?topic=69800.msg535242#msg535242

And here is my Youtube video that is now being superceded by the improvements in description via the paper I am writing.


Edit: btw, conventional relativity states that +gravity, ie: gravity wave passing, will equal slower time. ;)
Title: Re: Is there a discrepancy with the equivalence principle?
Post by: captcass on 10/03/2018 02:33:32

Edit: btw, conventional relativity states that +gravity, ie: gravity wave passing, will equal slower time. ;)
Thanks, Timey. I looked at your abstract and I have no business discussing that as I am off into time and am not even looking at expansion and contraction. Indeed, I am thinking there is none. I sure wish you luck with the paper, though.
Re rates of time in the gravity wave, as it is a stretch/compress phenomenon, there would be dilation with the stretch and acceleration with the compression. They successively alter the size of LIGO's antenna. In what I am working on, it is the acceleration/compression aspect that evolves events forward down gradient in a time dilation gradient. (See? Colored by time! :) )
Anyhoooowwww.. That being said, I'll bow out of this thread.
Have fun, folks! :)   
Title: Re: Is there a discrepancy with the equivalence principle?
Post by: captcass on 10/03/2018 04:46:22
I am, at this stage, writing up a paper on my model to submit to arXiv scientific journal,
Sorry, just one more comment. arXiv is usually reserved for previously published authors. To get someone who has been published on arXiv to recommend your paper is very difficult as, if thy do so, and you later post something completely off the wall, they will also lose their posting rights.
Just wondering, do you or Colin, Alan or Jeff have papers already in arXiv? I do not. 
Title: Re: Is there a discrepancy with the equivalence principle?
Post by: captcass on 10/03/2018 04:47:22
I do not.
For good reason, as I have not got it all together yet...... :)
Title: Re: Is there a discrepancy with the equivalence principle?
Post by: Colin2B on 10/03/2018 08:35:13
I had not heard that they are testing portable clocks yet.  Can you direct me to a link for that?
The photographs i saw were of a small kitchen cabinet sized unit, eminently portable. I’ll see if i kept the reference.

The gp aspect of general relativity is proven already by NIST' clocks.
With regards to the g aspect, although there may be slight g anomalies at the Boulder site, it would be quite hard to tell an anomaly from an error margin when the clocks are being measured for g differences on the same site.  Not impossible though.
This is one aspect of your idea that I’ve asked about before but have not yet understood. You say your idea follows relativity except that near a higher density anomaly the clock will run faster than it would if there were no anomaly. What happens to gp and g near that anomaly?

It is not entirely clear to me if the +gravity of a wave would shift a clock, but the minus gravity of just a few inches of difference in height from a gravitational mass does shift a clock.  So if the +gravity of the incoming wave was of the same magnitude difference as the -gravity at few inches elevation, then a clock at a gravitational wave site would be shifted.
I don’t understand the differentiation you are giving to +gravity and -gravity effect of a wave. Please explain.
Title: Re: Is there a discrepancy with the equivalence principle?
Post by: timey on 10/03/2018 11:29:20
@captcass I cannot speak for Colin, Alan or Jeff, but I have not published before and will be seeking endorsement from previously published authors.  And thanks, good luck to you too.

@Colin2B
The portable clocks that they were talking about in the radio program I listened to were sounding like they could be hiked up mountains, or placed at locations around Yellowstone as an immediately tangible reference to elevation changes.
In that they were also saying that new tech means they can move away from the 3 foot fountain chamber in order to reduce size, I had visualised a smaller portable apparatus than kitchen cabinet size.

When I say anomaly, I am talking about an anomaly in g, ie: the density of g differs with the geological density of the local terrain.
If g differs from 1 local to the next, the gp will also differ from 1 local to the next.
The Boulder site is composed of a certain geological density, and within the confines of that local there will not be much difference in geological density, so while it is possible to measure gp differences from any local, it would be hard to measure differences in g, (local density) without placing clocks at 2 separate sites of known significant difference in g, (local density).

An incoming gravity wave will plus it's gravity to the gravity of the local, as it passes by.
My reference to minus gravity was ill-conceived. What I mean is that there is a difference in gravity field between ground level and a few inches above ground level that shifts a clock.  The field is x amount weaker at that elevation from ground level (-gravity).  Should the +gravity of the wave be of a similar magnitude as the -gravity at few inches elevation, then a clock will be shifted by the gravity wave.
General relativity says that clock will tick slower.
My model says that clock will tick faster.  It says it will tick faster because that situation is a difference in g, as opposed to a difference in gp.

@jeffreyH I didn't miss your post and am holding that thought.
Title: Re: Is there a discrepancy with the equivalence principle?
Post by: alancalverd on 10/03/2018 12:43:01
Interesting scenario. Place a clock at A (source) on the left and and another at B (receiver) on the right, and have a gravity wave pass from left to right. A will initially slow down (red shift) as seen from B then blue shift as B's clock slows down, so the net effect will be zero.

This is exactly the same mechanism as a photon from a distant source passing through any gravitational field on its way to a receiver - i.e. no effect.

If you want to believe that the grav wave speeds up A, then it will also speed up B, so again no effect overall.

A focussed gravitational wave, that passed through A but not B, would be an interesting tool. Its effect is left as an exercise to the reader, but it doesn't represent the passage of a stellar photon towards an observer through an intervening gravitational field.
Title: Re: Is there a discrepancy with the equivalence principle?
Post by: Colin2B on 10/03/2018 14:50:04
When I say anomaly, I am talking about an anomaly in g, ie: the density of g differs with the geological density of the local terrain.
If g differs from 1 local to the next, the gp will also differ from 1 local to the next.
The Boulder site is composed of a certain geological density, and within the confines of that local there will not be much difference in geological density, so while it is possible to measure gp differences from any local, it would be hard to measure differences in g, (local density) without placing clocks at 2 separate sites of known significant difference in g, (local density).
That wasn’t the question I was asking - see below.
PS what is “the density of g”?

General relativity says that clock will tick slower.
My model says that clock will tick faster.  It says it will tick faster because that situation is a difference in g, as opposed to a difference in gp.
This was what I was asking.
“You say your idea follows relativity except that near a higher density anomaly the clock will run faster than it would if there were no anomaly. What happens to gp and g near that anomaly?”
In other words, your theory unlinks gp, g and time dilation so what is the relationship between them. We know you can always determine g from gp measurements & vice versa, so as dilation is no longer dependent on gp near the anomaly, what is the relationship.

PS I assume you've seen Alan's reply re gravitational wave. You have to remember that by the time it reaches us it is a plane wave front - albeit a rather unusual one - you need to observe the passage of the wave as a distant observer to measure the dilation as @alancalverd points out. There are suggestions for a satellite network of atomic clocks to detect g waves.
Title: Re: Is there a discrepancy with the equivalence principle?
Post by: timey on 10/03/2018 15:59:13
No - what I am saying is that g is the gravitational Mass, and that gp is the gravity potential for mass elevated in the space that surrounds that Mass.
Now general relativity is 'generally' calculated as gravitational mass M, but this does not take into account any 'anomaly' of g on M.  Geological density is not uniform on M, and these non-uniformities are what I am referring to as 'anomalies' of g.
So - what I am saying is that, as per observation, clocks do tick faster when elevated in the gp, but that a further test of general relativity can be conducted by comparing clocks at the same elevation, but under different circumstances of magnitude of g.

Under this remit of experiment, conventional general relativity states that the clock in the denser location/greater magnitude of g will tick slower.
This is where my model differs and states that the clock in the denser location/greater magnitude of g will tick faster.
My model also states that the clock in the denser location will tick faster when elevated, so there is no deviation at all from observation, or even that much departure from conventional general relativity equations.
And, best of all, the theory can be tested!

I didn't see Alan's gravitational wave post, I'll look for it now.
Title: Re: Is there a discrepancy with the equivalence principle?
Post by: timey on 10/03/2018 16:19:44
Here you have said it all:

Interesting scenario. Place a clock at A (source) on the left and and another at B (receiver) on the right, and have a gravity wave pass from left to right. A will initially slow down (red shift) as seen from B then blue shift as B's clock slows down,

On the basis that we are observing from the 'far away clock', this next statement is superfluous:

so the net effect will be zero.

We will see clock A being shifted by the wave, and shift back when the wave has passed, and we will see clock B being shifted by the wave, and shift back when the wave has passed.
Title: Re: Is there a discrepancy with the equivalence principle?
Post by: timey on 10/03/2018 18:11:00
@Colin2B In addition to post 1041

There are suggestions for a satellite network of atomic clocks to detect g waves.

In answer to the above, yes, I do know, (for anyone reading that doesn't, see link below) and did myself suggest that clocks could be used to detect gravitational waves, on a gravitational wave thread at this forum, at the time of the press release of the first gravitational wave.

But it would be far cheaper to do it from the ground, at the LIGO site, and better b/c direct comparisons could be made between the 'differing' (? chuckle) methodology.

https://phys.org/news/2017-03-gravity-atomic-clocks.html
Title: Re: Is there a discrepancy with the equivalence principle?
Post by: Colin2B on 10/03/2018 18:39:32
Now general relativity is 'generally' calculated as gravitational mass M, but this does not take into account any 'anomaly' of g on M.  Geological density is not uniform on M, and these non-uniformities are what I am referring to as 'anomalies' of g.
Yes we all know that, but you are talking about the difference between a spherical cow and a real one.
For classroom teaching it is very convenient to consider the mass of earth as uniform, but we all know that is not the real world. This is why everyone in the real world talks about the geoid and lines of equipotential, these lines vary in height depending on local density. This means that the calculation of gp includes density variations. However, this still doesn’t explain why your theory predicts that a clock will tick faster near an increase in g. It’s the why I’m interested in.
Title: Re: Is there a discrepancy with the equivalence principle?
Post by: timey on 10/03/2018 19:08:43
Yes we all know that, but you are talking about the difference between a spherical cow and a real one.
You were asking what I meant by anomolies in g, and that is my answer to that question.

For classroom teaching it is very convenient to consider the mass of earth as uniform, but we all know that is not the real world. This is why everyone in the real world talks about the geoid and lines of equipotential, these lines vary in height depending on local density.
Yes, but they also vary in g at differring hieghts and locations due to the density of the local geological composition, so lack of height does not necessarily mean lack of geological density, and visa versa.

This means that the calculation of gp includes density variations.
Yes - the calculation of gp will be inclusive of density variations, but it is ONLY variations in hieght, or speed that have been precision tested as of yet. Variations in g have not been tested.

However, this still doesn’t explain why your theory predicts that a clock will tick faster near an increase in g. It’s the why I’m interested in.
My theory predicts that a clock will tick faster near a bigger mass than it will near a smaller mass because of the added axiom of "+energy=shorter seconds", where the observation of a faster clock rate in the gravity potential is then calculated as per its frequency with an associated energy (that must accompany a frequency) of something like mgh/m=e

Edit: Where a bigger Mass (gravitational M creating the gp) will have a greater associated energy.
Title: Re: Is there a discrepancy with the equivalence principle?
Post by: timey on 11/03/2018 00:43:14
So - in addition to post above...

On these maps, from GOCE and GRACE, there will be elevations from sea level that are composed of different densities of rock, and the g will be greater in a denser area than it is in a less dense area.

http://www.esa.int/spaceinimages/Images/2011/03/New_GOCE_geoid

https://svs.gsfc.nasa.gov/3655

My experiment suggests placing a clock at an elevation from sea level on a denser location, and placing another clock at the same elevation from sea level but on a less dense location (taking into account centripetal speed differences of longitude) and measuring the difference between g. (this being a different experiment than measuring the difference in gp between 2 clocks where 1 is elevated above the other)

Are you understanding now @Colin2B?
Title: Re: Is there a discrepancy with the equivalence principle?
Post by: alancalverd on 11/03/2018 08:03:18
Not sure what you are trying to demonstrate here.

http://ngm.nationalgeographic.com/2011/09/now-next/img/geoid.jpg is a good example of the measurement of g over the entire planet as seen from a constant height* above sea level, so that experiment has already been done, and we know that the variation is due to the density of the intervening material because we have been using gravimetry for mineral prospecting for well over a  hundred years.

As gravitational potential is gh, we also know that the redshift in a radial gravitational field is δE/E = δ(gh)/c2 = gδh/c2 where δh is the height difference between source and detector (Pound-Rebka and many subsequent experimental proofs) at fairly constant g (i.e. when h<<Rearth).

And we know that if δh = 0 and g is constant, there is no redshift (Mossbauer)

And our GPS clocks are corrected for δ(gh) because g varies with h when h is commensurate with Rearth.

The experiment you describe is just a complicated inversion of the P-R experiment where δh = 0  and you vary g between stations. *Your experimental difficulty is to define h since the geographical geoid is as wobbly as the gravitational one. The theory applies to an idealised spherical attractor so the conventional experiment uses a radial (vertical) track, allowing us to measure δh without knowing h. 

Since the equation has been demonstrated true to a considerable degree of precision, is it really likely that rotating the experiment through 90° will produce a different answer? It's not just a matter of precision, but sign: you are predicting a blueshift where everyone else has observed a redshift, by measuring a shift (in whatever direction) where everyone else has observed zero.

Here's an analogy. It is 10 miles from A to B and I can run at 10 mph, so I say "it will take me an hour to get to B". You are saying "that's irrelevant. I know the distance and the time it takes you, but I want to measure your speed, and it won't be 10 mph according to my model of the universe." 

Oddly, I've just had the same argument with a local road planner who assures me that the road past my house is permanently congested and needs a bus lane "because the computer model says so", whereas I have actually measured the traffic speed in the rush hour and know that it is faster than a bus can go. And not just me - it's the preferred route for emergency services. I wondered briefly if you might be related, but she refuses to do any actual measurements!
Title: Re: Is there a discrepancy with the equivalence principle?
Post by: Colin2B on 11/03/2018 10:10:53
Are you understanding now @Colin2B?

On these maps, from GOCE and GRACE, there will be elevations from sea level that are composed of different densities of rock, and the g will be greater in a denser area than it is in a less dense area.
yes, I understand this, it is all part of the std geoid mapping process.

My experiment suggests placing a clock at an elevation from sea level on a denser location, and placing another clock at the same elevation from sea level but on a less dense location (taking into account centripetal speed differences of longitude) and measuring the difference between g. (this being a different experiment than measuring the difference in gp between 2 clocks where 1 is elevated above the other)
yes, i understand this as it is consistent with your previous descriptions of the experiment. Also measurements of gp and g based on the geoid take account of centripetal effect.

My theory predicts that a clock will tick faster near a bigger mass than it will near a smaller mass because of the added axiom of "+energy=shorter seconds", where the observation of a faster clock rate in the gravity potential is then calculated as per its frequency with an associated energy (that must accompany a frequency) of something like mgh/m=e

Edit: Where a bigger Mass (gravitational M creating the gp) will have a greater associated energy.
this is what i don’t understand.
I suppose i will have to wait until you have finished writing out your whole paper, and see everything laid out, in order to understand your thinking.

However, a few thoughts to add to what Alan says.
With the geoid, lines of equipotential represent lines of equal energy, so if you follow them you do no work (water on a surface which follows these equipotentials will not flow even though the line may appear to slope when viewed by accurate surveying equipment). So, if you are adding energy (from where?) it should be reflected in these lines.
Also, as I’ve said before, these lines also determine g so near a density anomaly your theory should predict that g decreases, which is not what is measured nor what you are saying.
Just trying to think through how the maths of your theory would work compared to current physics.



Title: Re: Is there a discrepancy with the equivalence principle?
Post by: timey on 11/03/2018 11:53:15
Actually @alancalverd the traffic planner sounds much more like she might be related to you. (chuckle) She has a model built on parameters that she refuses to consider beyond.

@Colin2B
General relativity has made an 'assumption' that b/c a clock runs faster at elevation, that time runs slower on a bigger mass, and from that assumption deduces that time runs faster out in space where there is no mass.  So GR states additions of g to gravitational mass as causing slower time, and the resulting change in gp of elevated clocks (ticking faster) will, as a result of additional g to the gravitational mass, run slower time.

My model does not make this assumption.  I see that it is just as possible to say that:
Additions of g to gravitational mass cause faster time, and the resulting change in gp of elevated clocks (ticking faster) will, as a result of additional g to the gravitational mass, run faster time.
This does not change any 'observation' that we observe, or change any near earth general/special relativity 'observer' calculations such as made by the GPS system.
(Additionally: Where GR then deduces from it's 'assumption' of slower time on the bigger mass, that time runs faster in space, my model deduces the opposite, and that time for m and M differs from time for where m=0)

Alan makes a good point where he says:
" The experiment you describe is just a complicated inversion of the P-R experiment where δh = 0  and you vary g between stations."

...This is an experiment that has the potential to confirm if the assumption that general relativity has made concerning what time does on the bigger mass is correct.
Every stone, and every nook and cranny, right?

When we get to pages 8, 9, & 10, this is where the description of plus and minus changes in the timing of clocks (m in relation to M) is, and I will elaborate on how that relates to energy there.
Title: Re: Is there a discrepancy with the equivalence principle?
Post by: alancalverd on 11/03/2018 13:07:24
additions of g to gravitational mass
What on earth (or in space) does this mean?

In physics, g is the resultant acceleration in free fall of a small test object towards a much larger one, or more precisely the mutual acceleration of two masses towards their barycentre. I don't see how you can add an acceleration (LT-2) to a mass (M).

But then we scientists are an unimaginative lot.
Title: Re: Is there a discrepancy with the equivalence principle?
Post by: timey on 11/03/2018 13:16:00
Sorry, but as we were talking about what occurs near a bigger mass, or a smaller mass, additions of g to a gravitational mass would follow additions of m. ie: increasing the gravitational mass making it bigger.

Edit: Or you might find lesser g in a location on earth that is composed of low density rock, and greater g in a location that is composed of high density rock.

Edit 2: Additionally, one might find that a tiny magnitude of g was added to the gravitational mass of Earth for the duration of a gravitational wave hit.
Title: Re: Is there a discrepancy with the equivalence principle?
Post by: Colin2B on 11/03/2018 15:45:09
When we get to pages 8, 9, & 10, this is where the description of plus and minus changes in the timing of clocks (m in relation to M) is, and I will elaborate on how that relates to energy there.
Ok. It is worth facing the questions Alan & I have raised head on, possibly referred out to annex for detail so they can be worked through step by step. At the moment i can’t see how the maths will work to match current to yours, it would mean a change to the way g/gp works.
We’ll have to wait until you get there as you’ve obviously thought it through in detail.
Title: Re: Is there a discrepancy with the equivalence principle?
Post by: alancalverd on 11/03/2018 17:13:19
Sorry, but as we were talking about what occurs near a bigger mass, or a smaller mass, additions of g to a gravitational mass would follow additions of m. ie: increasing the gravitational mass making it bigger.

So you mean adding mass. Much effort would have been saved by saying so.

Quote
Edit: Or you might find lesser g in a location on earth that is composed of low density rock, and greater g in a location that is composed of high density rock.
Never mind "might". Everybody knows it is true- it is how we do geological surveys for oil and minerals.

So the question is why would you expect an experiment that has been done thousands of times, i.e the measurement of gravitational potential, to give a different answer next time?
Title: Re: Is there a discrepancy with the equivalence principle?
Post by: timey on 11/03/2018 17:40:47
Sorry you have completely lost me Alan.

Clearly you understand the experiment in that you have said this:
"The experiment you describe is just a complicated inversion of the P-R experiment where δh = 0  and you vary g between stations."

It might save an awful lot of time if you post me evidence of the experiment I suggest having been conducted a thousand times.

Edit: Or are you confusing geological surveys that use a gravimeter to measure gravity as evidence of clock shifts?
Far as I know, NIST are the only people who have directly tested general relativity with precision clocks, and they have only tested relative speed, and changes in elevation (gp), but they have not tested for difference in g.
Title: Re: Is there a discrepancy with the equivalence principle?
Post by: jeffreyH on 11/03/2018 21:31:09
The universe may be either expanding, static or contracting. In the case of a static universe nothing changes globally. For both the expanding and contracting cases the value of all contributions to individual points in the gravitational field change over time. The sources are either moving apart or closer together. Over time these gradual changes must have an influence on the waves propagation through spacetime. It is not just the shift but the change in the rate of shift that is important to take into account. I have no idea if this effect is negligible or if it has been included in calculations.
Title: Re: Is there a discrepancy with the equivalence principle?
Post by: timey on 11/03/2018 21:58:35
The universe may be either expanding, static or contracting. In the case of a static universe nothing changes globally. For both the expanding and contracting cases the value of all contributions to individual points in the gravitational field change over time. The sources are either moving apart or closer together. Over time these gradual changes must have an influence on the waves propagation through spacetime. It is not just the shift but the change in the rate of shift that is important to take into account. I have no idea if this effect is negligible or if it has been included in calculations.

OK, so now we have jumped from talking about near earth experiments concerning g, and we have moved back to talking about if the intervening gravitational field that exists between galaxy clusters is neglibigle in a contracting universe with regards to redshift observations.

I just went through all that in great length wth Alan, over some 30 or so posts, before we started talking experiments.
The short answer is that the intervening fields between masses (galaxy clusters) are not negligible in my contracting model (they are negligible in expanding model), and this effect is paramount to my model's description of cosmological redshifts observations with regards to Hubble's ladder.

Expanding model: Redshifts are velocity related for the greater part, and (g.source minus g.receiver) gravitationally shifted.
(The CMB is purely velocity related shifts)

My contracting model: Redshifts are aprox. 92% gravitationally shifted by changes in the intervening field including (g.source minus g.receiver), and 8% velocity related shifted.
(The CMB radiation is 100% gravitationally shifted by changes in the intervening field)
Title: Re: Is there a discrepancy with the equivalence principle?
Post by: timey on 11/03/2018 23:03:44
As we have now jumped back to the contracting aspects, this article below (I could post arXiv papers instead) gives a good description, indirectly, of why it is that my model, that is much, much older than 13.8 billion years (our rate of time) 'could' be useful.

https://www.quantamagazine.org/earliest-black-hole-gives-rare-glimpse-of-ancient-universe-20171206?utm_content=buffera559a&utm_medium=social&utm_source=twitter.com&utm_campaign=buffer
Title: Re: Is there a discrepancy with the equivalence principle?
Post by: alancalverd on 11/03/2018 23:21:12
It might save an awful lot of time if you post me evidence of the experiment I suggest having been conducted a thousand times.

Gravitational potential difference is δ(gh). Red shift, calculated from δ(gh), turns out to have the calculated value. It doesn't matter whether you vary gh (by flying a GPS satellite in an elliptical orbit over the wobbly gravimetric geoid - see the Ashby paper I quoted earlier) or just h (by the P-R experiment), we always get the calculated red shift.  A clock has no means of knowing g, h, or gh,  unless you introduce a new and wholly unknown force that (a) is mass-dependent and (b) only affects clocks, or you send it a time signal from an identical clock at h = 0 - and even then, you can only determine the product gh from the red shift.

I think I'll give this a rest now. You are clever enough to understand elementary physics but apparently determined to pretend not to. Let's not spoil a friendship.
Title: Re: Is there a discrepancy with the equivalence principle?
Post by: timey on 11/03/2018 23:45:14
@alancalverd
The point of my suggested experiment is that h remains 'constant' and that it is ONLY a variation of g that is measured via the 2 clocks, FROM a 3rd clock ('far away clock') that is placed elsewhere.
The GPS, P-R experiment, and the NIST GR clock experiments are not inclusive of a measurement of ONLY a variation of g.

Placing clocks at gravitational wave sites would be 'just' a variation of g for the duration of the hit.
Or the experiment that I suggest as to placing clocks at different locations at same elevation, but of different geological density would also be 'just' a variation of g.

But by all means give it a rest Alan.  I am indeed very fond of you, warts and all, but I am also finding our exchanges frustrating, so no probs, and look forward to reading your always entertaining quips on someone elses thread.  All the best.
Title: Re: Is there a discrepancy with the equivalence principle?
Post by: Colin2B on 12/03/2018 05:06:51
Or the experiment that I suggest as to placing clocks at different locations at same elevation, but of different geological density would also be 'just' a variation of g.
And GP.

As I mentioned earlier, there is another prediction that should be made by your paper and that is that rapidly varying gravitational fields should cause varying red shifts of light passing through that field. So binary systems should show this effect for stars behind them. Check examples and you have your effect.
This is also the point @jeffreyH is making in #1055.
Title: Re: Is there a discrepancy with the equivalence principle?
Post by: alancalverd on 12/03/2018 09:44:13
Or a slowly varying field such as the moon.  Too big, too well characterised, too predictable, and too obvious.
Title: Re: Is there a discrepancy with the equivalence principle?
Post by: Colin2B on 12/03/2018 10:46:30
.. too obvious.
Too obvious to be accepted methinks!
Title: Re: Is there a discrepancy with the equivalence principle?
Post by: timey on 12/03/2018 12:23:59
Light leaves the sun and is redshifted away from the sun, and blueshifted towards earth, and reaches our detector. Hey presto, we have an observation of light.

Along comes an eclipse. The moon has moved between the earth and the sun.
We have a variation in the field.
The light is redshifted away from the sun, it is blueshifted towards the moon, it is redshifted away from the moon, and it is blueshifted towards earth, and reaches our detector.  Hey presto we have the same observation that we had before.

Nothing has changed!  Why is that?
Source gravity, plus moon gravity, minus moon gravity, minus receiver gravity = gravitational shift.

Let's try that again:
Light leaves source in galaxy cluster, travels into dust cloud, out of dust cloud, into another galaxy cluster, out of this other galaxy cluster, into our galaxy cluster, and reaches our detector on earth.
Source galaxy cluster gravity, plus dust cloud gravity, minus dust cloud gravity, plus other galaxy cluster gravity, minus other galaxy cluster gravity, minus our galaxy cluster gravity=gravitational shift.

So -  any variations of 'clumped mass' in the field that the light travels 'past', it can be seen that while these masses will affect the light, this will have no effect on the final redshift observation' at 'receiver'.
Alan posted this 'fact' already, many posts ago, I thought we all understood that my model is no different to expanding model in this respect.

The difference in my model is that the galaxies that make up the galaxy clusters have been, and still are moving closer together as time passes, and this will change the field that the light travels through. The entire field.  And it is these changes in the entire field that cause the greater part of the shift of redshift observations (in my model).

However, it may be that the GPS archived data could be used as evidence of variation of g with regards to the variations of gravity that the moon elicits on the field.
The GPS system will be making adjustments for this effect, and yes it is hard to imagine that it would be missed that an increase in g would cause anything but a slowing of time.
This same conundrum exists within the structure of the Shapiro experiment.

But I see a possibility (via curvature and length contractions) that these same maths can be modified (turned upside down and inside out), to describe my contracting model, and this will tally up with observation.
Yes - of course there is a high probability that I am wrong, but I do provide a testable prediction with which my model can be falsified.

@Colin2B
Yes - if mass is increased, g is increased, and gp is also increased.
Title: Re: Is there a discrepancy with the equivalence principle?
Post by: alancalverd on 12/03/2018 17:44:29
The difference in my model is that the galaxies that make up the galaxy clusters have been, and still are moving closer together as time passes, and this will change the field that the light travels through. The entire field.  And it is these changes in the entire field that cause the greater part of the shift of redshift observations (in my model).

Suppose the photon is travelling in direction x→ from a source at x0  . At a particular point x it enters an infinitesimal region Δx where the gravitational potential is lower than the source, so it is blueshifted from the point of view of an observer inside Δx, then reaches the other side at x1 = x + Δx where the gp is the same as at x0 and is thus seen by an observer at that point to have no shift. You can have as many versions of Δx in series as you like, but that fact remains that the shift observed at any point xn depends only on the difference in gp between the source and the receiver.
Title: Re: Is there a discrepancy with the equivalence principle?
Post by: timey on 13/03/2018 00:03:37
OK Alan, but while your consideration takes into account the gravitational mass clumps of source and receiver, it doesn't take into account the rest of the gravitational mass clumps, or the gravity fields between all of the galaxy clusters.

Light leaves source. The source of the light is a star in a galaxy, in a distant galaxy cluster that is 1billion light years away.  When the light left the light source, 1 billion years ago, the galaxies of that galaxy cluster were not as close together as they are today when the light arrives at our detector.  Furthermore, the galaxies of every galaxy cluster in the universe were not as close to each other 1 billion years ago as they are today when the light hits our detector.

Let's just say we concentrate on an area containing 10 galaxy clusters, where the galaxies are becoming closer together over time.  The tracts of space between the galaxy clusters will be increasing as the space between the galaxies of the galaxy clusters reduces.

What is the 'open space' field gravity doing in the tracts of space between galaxy clusters?
Is it:
a) increasing in gravity
or
b) decreasing in gravity
Title: Re: Is there a discrepancy with the equivalence principle?
Post by: alancalverd on 13/03/2018 01:06:43
It is a simple fact that whatever you see today is the result of what happened yesterday or 13 billion years ago, depending on where you look.

As you can see from my analysis in #1064, it doesn't matter what happened between the source and the detector: gravitational red shift depends only on the relative gravitational potentials at the time and place of the source and detector.

Energy is conserved. A photon starts its life with kinetic energy hf and an arbitrary potential energy dependent on the mass of its source. As it travels it may gain or lose potential energy by passing near other masses, but each such gain or loss is reflected by a change in f as seen by an observer at that point in space and is recovered as the photon leaves that point.

Imagine a frictionless ball rolling along a bumpy surface: it loses kinetic energy as it climbs a bump, and gains k.e. on the way down. Its final kinetic energy depends only on the initial k.e. and the height difference between start and finish.  The velocity of a photon is constant but its k.e. is its frequency.
Title: Re: Is there a discrepancy with the equivalence principle?
Post by: timey on 13/03/2018 02:07:52
But the source and the receiver are not the only masses affecting the gravity of the intervening field between.

Analogy: Place 3 galaxy clusters in a triangle shape with equal distance between.  Place 5 galaxy clusters equally placed around the triangle.  Now let gravity take its course.  The galaxies of the galaxy clusters are converging on points within their clusters.  So, the distances between the galaxy clusters will get longer.

What happens if gravitational masses get further apart from each other?  Well, we know that gravity reduces by the inverse square law in the field, so clearly a longer distance between galaxy clusters is going to result in a weaker field.

OK, so in the expanding universe, the distance between galaxy clusters is getting longer really quickly, and in fact the further away the galaxy cluster is, the faster it is moving away.  The gravity in the distance between galaxy clusters will be near 0, the greater part of the observed redshift will be velocity related, and gravity shift will be source minus receiver.

But in a universe that is slowly contracting under the influence of gravity, the distances between galaxy clusters will be getting longer only as a result of galaxies of galaxy clusters getting closer to each other. The gravity in the distance between will be constantly weakening, where the amount of years that a photon travelled in these ever weakening fields contributes to it's redshift.  Photons arriving from galaxy clusters that are further away will have greater redshifts.
Title: Re: Is there a discrepancy with the equivalence principle?
Post by: alancalverd on 13/03/2018 08:24:36
It seems that I cannot teach you anything about physics.
Title: Re: Is there a discrepancy with the equivalence principle?
Post by: Colin2B on 13/03/2018 09:04:28
But the source and the receiver are not the only masses affecting the gravity of the intervening field between.
Alan isn’t saying that the source and receiver are the only masses affecting the field.
You are imagining a large area of density with light passing through and the density changing before the light has exited. But that’s not the way it works.
Imagine a string of infinitesimally small points along the path of the light. Each one has its own gp, caused by nearby mass, which affects the light as it enters and leaves (uphill down dale) so at the end of this string the sum result is that it is only the start and end values which matter.
Yes, the gp at each of those points will change with time but the light is long gone by the time that happens.
As I said in an earlier post, if your remit was correct you would see varying shifts in areas of intense changing g fields and we don’t.
Title: Re: Is there a discrepancy with the equivalence principle?
Post by: timey on 13/03/2018 10:31:54
@alancalverd I don't need you to tell me how the expanding universe is calculated with regards to gravitational redshift.  The books I have read have informed me of this, and indeed it is b/c the expanding theory is calculated that way that a 'contracting' universe will be calculated oppositely.

@Colin2B. I already stated 10 times now that points of mass that the light moves past along its journey will not affect the redshift observation at receiver. Not in my model, nor anyone else's.
What those points of mass are doing with respect to each other will however affect the magnitude of the gravity field between 'all of' the masses.

Can I please ask if you understand that an almost uniform sea of particles/energy will divide under the influence of gravity into clumps of mass and tracts of open space anisotropic gravity field?
Title: Re: Is there a discrepancy with the equivalence principle?
Post by: alancalverd on 13/03/2018 10:58:57
We haven't got to the question of whether the universe is expanding, contracting or clumping. Until you understand the mechanisms of redshift, which is the principal observable of distant objects, you won't be able to propose an experimental test of any model.

But your latest reply to Colin suggests that you do understand it, so your model has to explain why more distant objects generally have larger redshifts, without referring to the irrelevant stuff between the source and the receiver. Let's try:

If a galaxy contracts towards its barycentre at a fixed distance from the observer, the observed gravitational redshift will increase with time. This is indeed observed but clumping can only be inferred and the "fixed distance" is highly unlikely. What else do we know?

Suppose our target galaxy is at distance 2n light years. If its gravitational redshift is increasing with time, an observer at n lightyears will see a more recent and therefore larger redshift than we do. We don't have the luxury of making such observations, but we do have plenty of objects at different distances, and a fixed local gravitational potential against which to measure them. Unfortunately it turns out that the nearer objects have smaller redshifts.

So if the distant galaxy is clumping, we need to add a receding Doppler component larger than the rate of increase in gravitational redshift to account for the observed general increase in redshift with distance.

In short, the observations suggest that local clumping is only consistent with an expanding universe.

Can we have local contraction and distant expansion? Yes, of course. Local condensation is what makes planets,  solar systems and even shrinking galaxies and colliding stars. But if there is more matter outside the observable universe, stuff further away from us will have both a local (clumping) and a general (expanding) motion. No new physics, just a realisation that the observable universe the observable portion of the universe, and just as Hawking radiation gives us a clue to the content of a black hole, so the Doppler shift of distant galaxies gives us an idea of how much stuff we can't see.
Title: Re: Is there a discrepancy with the equivalence principle?
Post by: timey on 13/03/2018 11:12:39
I do not understand why you think that light that is arriving from further away will have a lesser redshift than a nearer source.
A more distant object will have a greater redshift than a closer object, this being b/c the light has spent a longer time in the field than the light from the nearer object has.  If the light has spent more time in a weakening field, the redshift will be greater.

Edit: I do agree that your hypothesis is interesting, but it is not testable.  My contracting model is.
Title: Re: Is there a discrepancy with the equivalence principle?
Post by: Colin2B on 13/03/2018 14:13:09
As Alan says the following post indicates you understand what we are saying
@Colin2B. I already stated 10 times now that points of mass that the light moves past along its journey will not affect the redshift observation at receiver. Not in my model, nor anyone else's.

But this one indicates you have a different interpretation of the field.
What those points of mass are doing with respect to each other will however affect the magnitude of the gravity field between 'all of' the masses.
If light passes through our solar system we are agreed that passing through that field will not affect the shift of the light. However, at any point the magnitude of field it passes through is the sum of all the influences from all the bodies in the system. So it is no different to the case you are stating.

However, i think this is the real sticking point.
A more distant object will have a greater redshift than a closer object, this being b/c the light has spent a longer time in the field than the light from the nearer object has.  If the light has spent more time in a weakening field, the redshift will be greater.
You are thinking big field changing while light goes from one end to other. What Alan is saying is that as it moves from one infinitesimal point to another it is still only moving from one gp to another and so the intervening field variation has no effect.
Title: Re: Is there a discrepancy with the equivalence principle?
Post by: timey on 13/03/2018 14:49:51
But, unless the universe is expanding of course, this means that you cannot calculate grav.source minus grav.receiver to calculate the entire amount of gravitational shift in the field.  There are other bodies of mass affecting the field.
Title: Re: Is there a discrepancy with the equivalence principle?
Post by: alancalverd on 13/03/2018 18:37:36
I give up.
Title: Re: Is there a discrepancy with the equivalence principle?
Post by: timey on 14/03/2018 00:35:04
Ok - so Christof Wetterich's model differs from mine in many respects, but in this thinking below, he and I are on very similar ground. (marked with ***)

quote
"Wetterich's idea is that light emitted from an atom is governed by the mass of its particles—if that atom were to become larger in mass, the light that it emits would change in frequency as its electrons became more energetic. More energy would appear as light moving toward the blue spectrum, while less energy (an atom losing mass), would move toward the red spectrum. ***Thus, Wetterich reasons, if the mass of observable objects were once less, we would now see them with a redshift as they expand. If his line of reasoning is true, Wetterich says it's possible that the universe is actually contracting***."

Read more at: https://phys.org/news/2013-08-cosmologist-universe.html#jCp
Title: Re: Is there a discrepancy with the equivalence principle?
Post by: alancalverd on 14/03/2018 08:05:05
Quote
Unfortunately, Wetterich's theory can't be tested because of the relative nature of mass. Everything we are able to see has a mass that is relative in size to everything else. Thus if it's all growing, we wouldn't have anything to measure it against to see that it's happening.

Wrong.

1. F = Gm1m2/r2, so if either or both m is increasing, a Cavendish experiment or spring gravimeter will demonstrate an increase in F. No evidence to date, though you might need to wait a long time to see it.

2. If the mass of the receiver is increasing with time, then the light from distant objects will be increasingly blueshifted with distance. Counterfactual - and we have been observing for around 13 billion years..
Title: Re: Is there a discrepancy with the equivalence principle?
Post by: timey on 14/03/2018 12:03:09
Correct. (I too find Wetterich's reasoning flawed) But I note that he has, in his paper, managed to make a mathematical description despite the wrongness you mention, and that is Wetterich's model.

But ditching the idea of an atom size becoming bigger with time, if the galaxies of galaxy clusters get closer together over time, then in a universe that is 'contracting' under the influence of gravity, an observation of light from a distant galaxy cluster will be redshifted, and light arriving from more distant galaxy clusters will be further redshifted, b/c that light was emitted at a point in history that pre-dates the light emitted from the closer galaxy.

So (again) looking at 2 adjacent galaxy clusters that are both converging upon their barycentres, we have 2 separate areas of multiple masses that are becoming more gravitationally concentrated, and the tract of space between them will be becoming more gravitationally diffuse.  This will cause a redshift.
If Wetterich can mathematically describe his model, I see no reason why my model cannot be described mathematically...
...And, unlike Wetterich, my model 'can' be tested.  To say so, none of the other toe models, string theory, loop quantum gravity, holographic, etc, can be tested either.  Surely the fact that I provide a testable prediction counts for something? Doesn't it?
Title: Re: Is there a discrepancy with the equivalence principle?
Post by: alancalverd on 14/03/2018 15:45:38
You need to make up your mind as to whether decreasing a gravitational field increases or decreases redshift. It is difficult to convince you that it does neither whilst you believe it does both. Physics is not religion!

Quote
in a universe that is 'contracting' under the influence of gravity, an observation of light from a distant galaxy cluster will be redshifted, and light arriving from more distant galaxy clusters will be further redshifted, b/c that light was emitted at a point in history that pre-dates the light emitted from the closer galaxy.
I explained why this was exactly wrong a few posts ago.
Title: Re: Is there a discrepancy with the equivalence principle?
Post by: timey on 14/03/2018 21:53:23
Physics is not religion!

I think this says it all Alan.

Most theoretical physicists have the humility not to presume that a factor that includes a premise that is not experimentally tested is either right or wrong.

I'm certainly not going to waste my time repeating why and where you are making presumtions again and again.

So yes, by all means:
quote Alan:
If there is more matter outside the observable universe, stuff further away from us will have both a local (clumping) and a general (expanding) motion."

Great hypothesis, good luck finding Dark Matter and Dark Energy. Explaining the Big Bang, Inflation, and when we will start seeing the unobservable mass moving towards us.  Oh, and please @ me when you can think of an experiment to prove those velocity related redshifts.

I think we are done here on this subject Alan. It's clear that a 'contracting' model is not your cup of tea.
All the best.

@Colin2B  If you have not been completely put off by Alan's religous beliefs concerning the nature of the redshift observations, I would care to continue with you.  If not. then no matter, and thank you very much for the mathematical information that you did impart to me concerning c^2/R.  That was useful to me.
Title: Re: Is there a discrepancy with the equivalence principle?
Post by: alancalverd on 14/03/2018 23:09:46
I'm certainly not going to waste my time repeating why and where you are making presumtions again and again.
Unlike you I have not made a single presumption nor stated as fact anything that has not been experimentally proven -  in most cases by experiments that you have quoted but clearly not understood.
Your insult has not fallen on deaf ears. Goodbye.
Title: Re: Is there a discrepancy with the equivalence principle?
Post by: timey on 15/03/2018 11:51:32
For the sake of anyone else who is reading, Alan has made a very serious presumption about redshift observations.

It is not proven that the cosmolgical redshift observations are velocity related, therefore it is presumptuous to state as fact that grav.source minus grav.receiver is the total sum of gravitational shift.

I have this information from the mouths of professional physicists, who write books on the subject.

I certainly have not 'meant' to insult Alan, but I will not be told that grav.source minus grav.receiver is the sum total of gravitational shift for cosmological redshift observations, when the velocity related redshift for those observations is not proven.

Evidence for this fact can be observed in the article that I previously posted.  Clearly no science magazine, or scientific journal would entertain Wetterich's, or any other physicists reinterpretation of redshift observations if the matter had been proven!

https://m.phys.org/news/2013-08-cosmologist-universe.html#jCp
Quote:
"Wetterich's paper hasn't been peer reviewed yet, but thus far, comments by others in the field suggest openness to this new line of thinking"

(I personally see no shame in being wrong, and always admit it when I am. In this case, I am not wrong)
Title: Re: Is there a discrepancy with the equivalence principle?
Post by: Colin2B on 15/03/2018 12:21:23
@Colin2B ..... I would care to continue with you.  If not. then no matter, and thank you very much for the mathematical information that you did impart to me concerning c^2/R.  That was useful to me.
I am happy to continue proof reading and pointing out where your text is unclear, but would not comment on whether it would be acceptable to the physics community as correct. Might be some delay in reading and responding over next few weeks as quite busy.
Title: Re: Is there a discrepancy with the equivalence principle?
Post by: alancalverd on 15/03/2018 13:36:20
It is not proven that the cosmolgical redshift observations are velocity related,

I never said they were, though it would be presumptuous to state that there is no velocity component involved, especially when it can be distinguished in nearby objects (from laboratory to galaxy) whose velocities can be measured independently. The only observations I quote are that distant objects generally have larger redshifts than near ones, and the CMB redshift is apparently less than some distant galaxies, which suggests relative motion between them. If this finding can be attributed to Doppler or Hubble effects, I have suggested how it can be consistent with a locally contracting universe.

Quote
therefore it is presumptuous to state as fact that grav.source minus grav.receiver is the total sum of gravitational shift.
It is never presumptuous to state what is either a tautology or an experimental fact.

Title: Re: Is there a discrepancy with the equivalence principle?
Post by: timey on 15/03/2018 19:36:18
@Colin2B  Thanks, couldn't ask for more.  I am going to take a break for a bit (couple weeks maybe) and concentrate only on my paper.  I have a lot of considering to do as to which aspect to describe first, but your questions have given me clues as to how to proceed.

@alancalverd, I do not wish to ignore you as that would be rude under the circumstances, but to point out the obvious you saying in reply to me:

It is not proven that the cosmolgical redshift observations are velocity related,

I never said they were, though it would be presumptuous to state that there is no velocity component involved, especially when it can be distinguished in nearby objects (from laboratory to galaxy) whose velocities can be measured independently.
...does rather rely on a velocity related interpretation of redshift observation, and special relativity calculations.
And isn't there a discrepancy regarding galaxy rotational velocities?  That requires that there be this elusive property called dark matter in order to describe these decepancies of 'extra' velocities?

The only observations I quote are that distant objects generally have larger redshifts than near ones, and the CMB redshift is apparently less than some distant galaxies, which suggests relative motion between them.
Yes, my model describes that distant objects have larger redshifts than near ones.  Yes, my model states that the relative motion between the CMB and galaxy clusters is relevant.

If this finding can be attributed to Doppler or Hubble effects, I have suggested how it can be consistent with a locally contracting universe.
I do not disagree.  But this is not relevant to my contractig model.

therefore it is presumptuous to state as fact that grav.source minus grav.receiver is the total sum of gravitational shift.
It is never presumptuous to state what is either a tautology or an experimental fact.
It is presumptious to state that  - my saying that my model is based on the redshift observations of galaxy clusters being 92% gravitational shifted, and 8% velocity related shifted, and the CMB radiation being 100% gravitationally shifted - is wrong b/c gravitational shift is grav.source minus grav.receiver.

The redshift observations 'do' follow tautology as greater part velocity related via an expanding universe.
The redshift observations 'do not' follow tautology as greater part velocity related for a universe contracting under the influence of gravity..

And with respect to cosmological observations, neither the velocity related nor the gravitational shift aspects of these redshifts are proven as experimental fact.

Also, the experiment that I suggest has not been conducted a thousand times as you seem to believe.
The experiment suggested is quite clear that NIST's portable clocks should be placed under equal circumstance of elevation and longitude, to equalise hieght from centre of earth and centripetal speed considerations, and that it is only a difference in the cicumstances of g that should be measured.
This has never been done.
A gravitational wave hit would also change the circumstances of g, and a clock placed at a gravitational wave site should be shifted by the wave.
This has never been done.

I do not wish to argue with you about these matters that I know to be factual.  I do not proclaim my theory to be correct.  I simply provide a prediction that can prove or falsify my idea.
I'm off to concentrate on my paper now, but again state that I most certainly do not mean you any disrespect, but simply cannot spend any more time arguing my right to calculate differently where current theory premise is not experimentally proven.
Title: Re: Is there a discrepancy with the equivalence principle?
Post by: timey on 24/03/2018 13:19:46
OK - just to clear up my use of terminology here:

and the CMB radiation being 100% gravitationally shifted

When I say the CMB is 100% gravitationally shifted, this is wrong in that there are no spectral lines in blackbody radiation.
What I mean to say is that the measurement of the CMB blackbody radiation's cooling has been attributed to the spaces between mass' expanding. The spaces between mass' expanding has been attributed to an expanding universe, where the rate of the CMB recessional velocity has been found to be approx. 8% slower than galaxy cluster recessional velocity.
(A measurement that physicists report is perplexing for expanding theory. The CMB should expand at the same rate as galaxy clusters. It is for this reason that many physicists have been chopping away at error margins within the data)

So - my model, which contracts from it's post inflation initial conditions of an almost uniform sea of particles and energy, under the influence of gravity, states that the spaces we observe between the galaxy clusters are expanding b/c the galaxies of galaxy clusters are, over time, becoming closer to each other.
The Hubble's ladder redshift observations of galaxy clusters are (in my model) approx.8% velocity related, and approx.92% gravitationally shifted by the changes in 'the field' caused by the gravitational clumping.
And the CMB blackbody radiation is cooling due to the changes in the field.

My stating the CMB as gravitationally shifted was technically incorrect.  What I was trying to say is that the CMB (in my model) does not have a recessional velocity, and that the cooling is due to the changes in the field, where these changes are caused by the influence of the gravitational clumping of mass.

The considerations of my YouTube video are concerning giving physical cause and effect mechanics for the phenomenon of gravity, and a spacetime structure that incorporates both GR and SR considerations in a singular description.
I'll be posting a few more portions of this description later.
Title: Re: Is there a discrepancy with the equivalence principle?
Post by: timey on 26/03/2018 19:40:45
Abstract:

This proposal for a modification of general relativity, providing cause and effect mechanics for the acceleration/attraction of gravity, results in fully described cause and effect mechanics for a very different cyclic bounce cosmology, where our current position, including the entire history of universal development into clumped mass occurs within the 'contraction' period.

General Relativity can already describe a contracting universe, but to modify general relativity, (bearing in mind that physics has no comprehensive theory of time), I add another entirely seperate time function for background space - where the changes in the rate of this time can be considered as the phenomenon from which the effects of gravitational acceleration (not gravitational attraction) are emergent.
This temporal interpretation of space time curvature constitutes a renormalisation term to the general relativity metric resulting in Euclidean geometry, or flat space.

My model incorporates both general and special relativity considerations into a singular space time structure that addresses the quantum/relativity time problem resulting a 'pilot theory' type scenario of position and momentum.

My model makes a definitive falsifiable prediction for doable experiments.

My model predicts the discrepancy between the observations of galaxy cluster redshifts and the CMB. (as is observed)


Modifications to General Relativity:
a)  An additional axiom "+energy=shorter seconds"
b)  An additional axiom "The speed of light cannot exceed the local rate of time"
c)  An additional time phenomenon for where m=0.


a) Introducing An Additional Axiom:
"+energy=shorter seconds"

I shall give full details on the cyclic bounce cosmology aspects of this model in chapter ?, where this axiom, "+energy=shorter seconds", is paramount for describing my model's contraction mechanics.
But for now I will point out that this additional axiom results in a departure from general relativity, in that although all clocks tick faster in elevation to mass, as is experimentally proven, that a clock near a bigger mass will tick faster than a clock near a smaller mass.
I point out that - although precision testing with clocks has been conducted with regards to elevation from gravitational mass, and with regards to relative motion - no precision testing has been conducted with regards to a change in the gravitational mass the clock is in proximity to.

I will be giving full details of experiments with which only a change in g be measured via a clock in chapter ?.
But for now I will just say that my prediction for the outcome of such an experiment is contrary to GR, in that GR states the clock near the bigger mass as ticking slower*, and my model predicts that the clock near the bigger mass will tick faster*. (*as compared to a clock near a smaller mass)
This prediction is directly related to this added axiom of "+energy=shorter seconds"


b) Introducing An additional Axiom: "The speed of light cannot exceed the local rate of time"

This additional axiom, "The speed of light cannot exceed the local rate of time", is related to my model's addition of an extra dimension of time. The changes of this extra dimension of time being the physical cause and effect mechanics for the 'acceleration' of gravity*. (*Not the 'attraction' of gravity, this (in my model) being associated with the time dilations observed of clocks in elevation to gravitational mass, and relative motions. Gravitational 'attraction' (as per my model) will be described in chapter ?)

This additional axiom, "The speed of light cannot exceed the local rate of time", is paramount in describing (for my model) the passage of light across the cosmos, and is directly related to my model's addition of an additional dimension of time for where m=0.


c) Introducing An Additional Time Phenomenon:

So before I introduce my model's additional time phenomenon and it's changes in rate, I will quickly go through some known time dilation considerations - that it be clearer the contruct of what follows.

(https://i.imgur.com/kfPFjZp.png)

The diagam above is a quarter sphere (purple curve) plus equatorial bulge gradient (blocked green).
Clocks placed at sea level at any placement on that equatorial bulge gradient (red curve/big dots) will all run at same rate.
This is because, from pole to equator, the increase in centripetal speed caused by the increased hieght from centre of earth/planet due to the equatorial bulge gradient - where an increase in speed causes a decreased clock rate - exactly cancels out the increase in the clock rate that is expected of a clock when that clock is positioned at an increased height from centre of earth/planet.
(ie: elevated position in the gravity potential)

Therefore this model makes all it's considerations of the rate of time of 'clocks' on the basis that the observed clock rate is a combination of both general relativity(position in gravity potential) time dilation effects, 'and' special relativity (motion related) time dilation effects.

This model states that these general relativity and special relativity clock timing effects are occurring ONLY for where m does NOT equal zero, (ie: for mass), and states that there is another separate time phenomenon for where m does equal zero. (ie: open space)
That this additional time phenomenon for where m does equal zero occurs concurrently to the timing of clocks, but as a separate issue.
And the changes in the rate of this additional time phenomenon are inversely proportional (ie: change by the same value observed of the clock, but negatively) to the changes in the rate of time for clocks, (ie: where m doesn't equal zero), when clocks are observed in fixed position* with the centripetal motion of the earth/planet in the gravity potential...
(*as opposed to clocks/mass in free motion with respect to the centripetal motion of the earth/planet, in the gravity potential, this being a differently valued consideration, more on the difference here later)

...And the specific rates of both time phenomenon are defined (for all but a very tiny part, this being a 'universal contraction' consideration) by the magnitude of the Gravitational Mass/Mass'

...continued next post
Title: Re: Is there a discrepancy with the equivalence principle?
Post by: timey on 26/03/2018 19:43:47
continued from post above...

Chapter 1: Introducing An Alternative Spacetime Structure

These following worksheets illustrate how the geometric curvature of spacetime can be attributed to an additional temporal phenomenon, and how by adding this temporal phenomenon, this constitutes a renormalisation term to the GR distortion of the metric, resulting in Euclidean geometry. (or flat spacetime)

The descriptions below will ask you to consider that there are not 2 but 3 time dilation phenomenons occurring.
Time dilation 1 being caused by motion, both centripetal and other.
Time dilation 2 being caused by postion of height in the gravity potential of M.
Note that this model states that time dilation 1 and time dilation 2 are for where m doesn't equal zero.
This model introduces the additional time phenomenon and it's changes as time dilation 3 - this time being for where m does equal zero. ie: for particles m=0, and for the open spaces or 'background space' between masses.

The considerations below are based on a simple idea that if an object is travelling in a background space that is inherent with it's own rate of time - a rate of time that the object must travel through - an object travelling at a constant speed over distances where seconds get progressively shorter, will be accelerated.  And an object travelling at constant speed over distances where seconds get progressively longer, will be decelerated.

I have described in the worksheets below, (perhaps unusually, hence the explanation), the 'up' 'down' changes of each time dilation phenomenon as 'partial dirivatives' of that time dilation.  As well as signifying the plus minus changes in time, this notation is indicative of directional orientation in the gravitational gradient.  These 'up' 'down' dirivatives also become useful notation when a consideration of changes in acceleration contains both a plus and a minus of the same time dilation phenomenon. (more on this later)
The time changes in each of the 3 time dilations are then each considered as a partial dirivative of changes in acceleration, where when added together in the mannner suggested, these 3 partial dirivatives of changes in acceleration become the changes in acceleration that are proportional to changes in gravity potential.


(https://i.imgur.com/sEfOSxt.jpg)   

Page 1 introduces the diagram that I use to make my description.  This structure of diagram, that conforms to Pythagaros as a geometrical consideration, is (as I am using it) a representation of the speed/distance/time formula.
Time is on the y axis, distance is on the x axis, and the line (which I call a vector because all considerations are concerning the changes in acceleration of an object travelling on that vector) is the speed, where a straight line describes a constant speed, and a curved line can decribe an acceleration or a deceleration.
That these plots, or graphs, can be representative of speeds that are constant, accelerated, or decelerated, relies upon the fact that the length of a second remains constant, and that the length of a metre remains constant. as demonstrated in the 'key' in the bottom left corner.


(https://i.imgur.com/QWLIRQP.jpg)

Page 2 demonstrates that by saying that seconds get longer proportional to distance in diagram 4, or that seconds get shorter proportional to distance in diagram 5, (clearly by a specific rate), that an acceleration or deceleration in the speed of an objects vector can be represented as a straight line, where these changes in acceleration of the object's trajectory through space can be attributed to changes in a rate of time that occurs in this background space that the object is travelling through.
(That x is marked as equal to constant metres becomes relevant when x is held equal to metres that are not constant later on)


(https://i.imgur.com/CpHChVN.jpg)

Page 3 demonstrates that seconds getting longer or shorter proportional to distance in diagrams 4 & 5, are 'up' 'down' functions, (partial dirivitives) of the same consideration, this being changes in time dilation 3 (dt3), and that these changes in time dilation 3 can be held relative to a partial dirivitive of changes in acceleration.
Partial dirivitive because there are other changes in acceleration to be considered, where all the partial considerations added together in a suggested manner are then 'the' changes in acceleration, (da).


Having introduced an additional time phenomenon and it's changes as (dt3) in pages 2 and 3, I now state that changes in time due to motion will be refered to as (dt1), and that changes in time due to position in the gravity potential will be referred to as (dt2).

(https://i.imgur.com/Zn3okP9.jpg)

Page 4a repeats the introduction of (dt3) from page 3, and then introduces (dt2) as an inversed consideration, (please note these curve manipulations do not represent true values as of yet), but in this case the changes in rate of time are occuring for the clock that is on that vector travelling the distance on x.  This diagram is saying that an accelerated clock rate will cause an object's (the clock's in this case) vector in the 'up' direction to be decelerated , and that a decelerated clock rate will cause an object's vector in the 'down' direction to be decelerated.
(There is more on the distinction between a clock held in position to the earth with regards to elevation and centripetal motion, and a clock in free motion 'up' and 'down' with regards to the earth's gravitational field's relationship with centripetal motion, in detail, in pages 9 & 10.)

(https://i.imgur.com/4JTqpVH.png)

Page 4b introduces the changes in time due to centripetal motion* in the gravity potential as (dt1), where the changes in time dilation 1, (dt1), with regards to centripetal motion*, are occurring inversely to the changes in time dilation 2,(dt2).
(*other types of motion are considered with respect to changes in time dilation 1 in pages 8, 9, & 10)

Now there are 3 partial dirivitives of acceleration.
Partial dirivitives of acceleration (b), (being equal to (dt2)), and (c), (being equal to (dt1)), are for where m doesn't equal zero.
Partial dirivitive of acceleration (a), (being equal to (dt3)), is for where m does equal zero.

Added together, these partial dirivitives of acceleration become 'changes' in acceleration (da), where (da) are held proportional to changes in gravity potential.

And the changes in time dilation 1 (dt1), plus the changes in time dilation 2 (dt2), at any observed position in the gravity potential, are equal but inverse to the changes in time dilation 3 (dt3), where the changes in time of a clock in the gravity potential are those observed by the observer who we presume to be making the calculations.
(Note that no actual value has been attributed to the magnitude of a what a 'change' in time consists of, nor to the magnitude of the subsequent 'change' in acceleration, or a description of within what distance in the gravity potential a 'change' occurs, as of yet.)

Working diagrams (WD's) 1 & 2 demonstrate how by implementing changes in x where metres get shorter or longer proportional to the amount of time that passes, where changes in time are equal to constant seconds - that this can be viewed as the inverse to the consideration of seconds getting shorter or longer propotional to the amount of distance covered, where changes in distance are held equal to constant metres, as shown in WD's 3 & 4. (which are the same as the diagrams no: 3 & no: 4 shown previously)

What these working diagrams demonstrate is that changing the length of a second can provide the same results as changing the length of a metre.

Page 5 demonstrates how the additional time phenomenon I have added can be slotted into a singular description of General and Special Relativity as a spacetime structure, and does so without changing the ideas of the underlying Relativity mathematics*.
(*It is only the implications of the Relativity maths that become changed by my modifications, and alternate interpretations of cosmological observations)

____________________

@Colin2B, I will be back with a revised page 5 at some point soon. Am I getting it across so far though?
Title: Re: Is there a discrepancy with the equivalence principle?
Post by: Colin2B on 28/03/2018 22:55:50
@timey - “@Colin2B, I will be back with a revised page 5 at some point soon. Am I getting it across so far though?”

I’ll have a read through these and get back to you, might be a couple of days so keep posting if you want to.
Title: Re: Is there a discrepancy with the equivalence principle?
Post by: Colin2B on 04/04/2018 14:08:37
I’m not commenting on whether your ideas clash with std physics, nor on points I’ve made before.

I would suggest you shorten your abstract to maximum of 2 paragraphs of no more than 10 lines each.
Most of what you then call abstract could be in an Introduction.

Make sure if you introduce a term in the introduction that you give a brief explanation of what it is. For example:
b)  An additional axiom "The speed of light cannot exceed the local rate of time"
This sounds similar to “speed of light being same for every observer”. You don’t really give a fag packet description of what you mean by it, so reviewer will not really understand what you are saying.

Chapter 1 seems to be an accurate description of what you are saying in your video. I would word it all differently, but that is down to personal style and it’s important you keep your own.

When you finish your paper I would suggest you send a copy + your proposed experiment to Ruxandra Bondarescu who was at Department of Physics, University of Zürich, Switzerland. E-mail: ruxandra@physik.uzh.ch
She is the one who suggested using atomic clocks to detect subsurface anomalies and will be an endorser, but more importantly your experiment will be lodged in her mind when they do the measurements.
Title: Re: Is there a discrepancy with the equivalence principle?
Post by: timey on 04/04/2018 15:23:42
Wow - Thanks Colin!  Above and beyond!  Yes, I will follow your makes sense advice on the abstract and intro.

I still have to draw out page 5 again, there is a mistake on the video page 5, and I can make a better description.
But it is only when I include pages 8, 9, 10, and 11 that pages 1 through 5 make sense.  I'm having a break from it being as I am at grandma Joan's.  Back on it when grandma duty is done.
Title: Re: Is there a discrepancy with the equivalence principle?
Post by: Colin2B on 07/04/2018 06:55:56
Can I make a suggestion which is made to all authors? When you have finished your book/article etc, just let it go, move on.
Don’t expect any endorsement or acceptance, your ideas will have been registered and if correct then the test, when done, will confirm your prediction.
I know you won’t let it rest in your mind until you have given it the best chance, but I hope that once written up you will have some satisfaction of a job completed and be able to let it go. 
No matter what the outcome, remember, in the journey you have learnt a lot.
Title: Re: Is there a discrepancy with the equivalence principle?
Post by: timey on 08/04/2018 03:25:08
Good advice Colin.

Once your paper is published to a scientific journal, then any person who subsequently uses any part of that concept within their paper is then required to cite your work.

Easy to let go under those circumstance.
Title: Re: Is there a discrepancy with the equivalence principle?
Post by: Colin2B on 08/04/2018 08:20:52
Just bear in mind that you are unlikely to be published in a scientific journal.
You may have to be content with publishing here and also making a few key people aware. At some stsge you have to let it go its own way and live the rest of your life.
Title: Re: Is there a discrepancy with the equivalence principle?
Post by: timey on 08/04/2018 13:49:51
In a physics climate that spends billions of dollars in research projects to investigate theories such as string theory, loop quantum gravity, holographic universe, asymptotic gravity, etc, that cannot make any falsifiable predictions to prove or disprove the theory, one would have thought that a person who can provide a falsifiable prediction for a 'really cheap' experiment* that will prove or disprove their theory, might well get published.  Only time will tell, aye?
(*placing a precision clock, or clocks at LIGO would be cheap by comparison to usual experimental expenditure)

Anyway it is good to hear - should an experiment with clocks determine that a clock will not only tick faster in the higher potential near a GM, but will also tick faster when it is placed near a greater GM than it will if placed near a smaller GM - that you and the other moderators here will recognise that this is what I have been saying on the forum for years.
Title: Re: Is there a discrepancy with the equivalence principle?
Post by: Colin2B on 08/04/2018 14:54:55
Not sure about the LIGO idea, but currently the lead team are modelling Mt Etna. They want to be able to measure subsurface movements of mass eg magma chambers and so be able to predict eruptions. That will attract money, so I’m sure the tests will be done as soon as practicable.
Title: Re: Is there a discrepancy with the equivalence principle?
Post by: timey on 08/04/2018 18:36:46
Quoted from arXiv paper: The Potential of Continuous, Local Atomic Clock Measurements
for Earthquake Prediction and Volcanology

"In this proceeding we argue that atomic clocks could measure smooth alterations of the geopo-
tential of the Earth on the sub-cm level. Additionally, measurements by local clocks in conjunction
with measurements by quantum gravimeters in volcanic areas would determine whether the ground
uplift is correlated with mass redistribution, since the two scale differently with the distance from the
1source. A sequence of earthquakes that lead to a gradual change in elevation over a period of a few
days could be associated with magma movements underground"

Under this remit, if my theory is correct, then when the gravimeter says that a chamber is filling up with magma, the clock will tick faster, saying to the experimenters under GR remit, that the ground is rising when it isn't, or isn't rising as much as the data suggests.
And when the gravimeter says that a chamber is emptying, the clock will tick slower, saying to the experimenters under the GR remit, that the ground is subsiding when it isn't, or isn't subsiding as much as the data suggests.

Looking at how a gravimeter works:
https://www.newscientist.com/article/2142507-quantum-gravity-detector-will-use-atom-clouds-to-survey-for-oil/
"If two atom clouds fall at different speeds, it would indicate a change in the density of the ground below. This could be due to the presence of oil or certain minerals, for example:
“Essentially it relies on the fact that any mass will generate a gravitational field, which can be detected with a very precise gravity sensor,” says Kai Bongs at the University of Birmingham, who helped develop the device..."

"...That’s why the oil and gas industry could be particularly interested in the gravimeter. It could be a powerful tool to help map out valuable deposits of oil or minerals, because denser materials will have a stronger gravitational pull than open pockets beneath the earth"

So the atom cloud in the chamber will arrive at the bottom of the chamber quicker if the density of the ground is higher.

The clock's electron transitions are predicted by GR to occur at a lower frequency or slower tick rate if the gravitational pull is higher.

My theory predicts that the clock's electron transitions will occur at a higher frequency or faster tick rate if the gravitational pull is higher.

OK, so a clock at LIGO would shift when the gravity wave hits, for the duration of the hit, and then shift back again.  This constitutes a scenario where only a change in g is occurring for a specific amount of time, in a situation where the wave is being measured already by an alternative means.  Damn sight cheaper way of using clocks to measure gravitational waves than this suggestion:

https://phys.org/news/2017-03-gravity-atomic-clocks.html

It would the finest thing in the world to live to see a finalisation of my train of thought manifest in the result of this type of experiment that is testing only against changes in g, no matter which way the experiment is observed to shift.
My theory has been a journey of thought process for me, and as a private venture that has been the most enjoyable experience in the whole world, but contrarily, as an idea that I have been trying to share it has been the most painful thing in the whole world, and I'd just as soon know the result of the experiment and then be done with thinking about it.
Title: Re: Is there a discrepancy with the equivalence principle?
Post by: Colin2B on 08/04/2018 22:58:13
Under this remit, if my theory is correct, then when the gravimeter says that a chamber is filling up with magma, the clock will tick faster, saying to the experimenters under GR remit, that the ground is rising when it isn't, or isn't rising as much as the data suggests.
And when the gravimeter says that a chamber is emptying, the clock will tick slower, saying to the experimenters under the GR remit, that the ground is subsiding when it isn't, or isn't subsiding as much as the data suggests.

My theory predicts that the clock's electron transitions will occur at a higher frequency or faster tick rate if the gravitational pull is higher.
This is the bit I will be interested to see when you post it. I promise not to comment on whether I agree with it or not.
Title: Re: Is there a discrepancy with the equivalence principle?
Post by: timey on 09/04/2018 12:37:55
My golden rule in physics is never to be in agreement with anything that isn't experimentally verified. If it isn't experimentally verified then it is only a theory. In my opinion it would be unreasonable of me ever to expect anyone to agree with something that is only a theory.

All I would expect from anyone with regards to my theory is an interest in whether or not the theory can be mathematically viable, and - given that I can show that it be mathematically viable - that the theory be tested experimentally.

I have actually already shown you why my theory makes a contrary prediction to GR with regards to 'only' changes in g for a clock, when I gave you the 'table of contraction' concerning how time is uniform at end of inflation, and divides into faster pockets of time where mass has clumped, and slower tracts of time in the spaces that are caused by mass vacating former positions during the clumping process.
These time changes then actually being the physical cause and effect mechanics for the observed phenomenon of gravity.
(where clocks tick faster on the bigger mass, but also tick faster in elevation to the gravitational mass due to an increase in the frequency of electron transitions, that are occurring, not b/c 'time' runs faster in space, but as an m in relation to M phenomenon)

Anyway, hopefully I'll get that across in the paper.
Title: Re: Is there a discrepancy with the equivalence principle?
Post by: alancalverd on 09/04/2018 23:58:08
An additional Axiom: "The speed of light cannot exceed the local rate of time"
Speed being length divided by time, this defines the "rate of time" as being measured in meters per second, and clearly being a knowable quantity.

What is the local rate of time at some known place on earth? It must be measurable or calculable at, say, Teddington or Paris.
Title: Re: Is there a discrepancy with the equivalence principle?
Post by: jeffreyH on 10/04/2018 22:15:42
It is a simple fact that whatever you see today is the result of what happened yesterday or 13 billion years ago, depending on where you look.

As you can see from my analysis in #1064, it doesn't matter what happened between the source and the detector: gravitational red shift depends only on the relative gravitational potentials at the time and place of the source and detector.

Energy is conserved. A photon starts its life with kinetic energy hf and an arbitrary potential energy dependent on the mass of its source. As it travels it may gain or lose potential energy by passing near other masses, but each such gain or loss is reflected by a change in f as seen by an observer at that point in space and is recovered as the photon leaves that point.

Imagine a frictionless ball rolling along a bumpy surface: it loses kinetic energy as it climbs a bump, and gains k.e. on the way down. Its final kinetic energy depends only on the initial k.e. and the height difference between start and finish.  The velocity of a photon is constant but its k.e. is its frequency.

I gave up but can't leave this post unremarked. Alan has said here all that needs to be said.
Title: Re: Is there a discrepancy with the equivalence principle?
Post by: timey on 10/04/2018 22:44:06
It is a simple fact that whatever you see today is the result of what happened yesterday or 13 billion years ago, depending on where you look.

As you can see from my analysis in #1064, it doesn't matter what happened between the source and the detector: gravitational red shift depends only on the relative gravitational potentials at the time and place of the source and detector.

Energy is conserved. A photon starts its life with kinetic energy hf and an arbitrary potential energy dependent on the mass of its source. As it travels it may gain or lose potential energy by passing near other masses, but each such gain or loss is reflected by a change in f as seen by an observer at that point in space and is recovered as the photon leaves that point.

Imagine a frictionless ball rolling along a bumpy surface: it loses kinetic energy as it climbs a bump, and gains k.e. on the way down. Its final kinetic energy depends only on the initial k.e. and the height difference between start and finish.  The velocity of a photon is constant but its k.e. is its frequency.

I gave up but can't leave this post unremarked. Alan has said here all that needs to be said.
Erm, yes... A very good desciption of general relativity.

If we were talking exclusively about a theory of general relativity, then I agree, what more is there to say.

I am talking about theoretical physics here, and am introducing a new theory and model.  Therefore if I should wish to describe my model, then by all logical remit, the post that Alan has made is NOT all there is to say...A fact that is backed up by all those books that theoretical physicist write about MOND, DSSR, VLS, etc...

Hello?  Due logical process?

Whether or not the theory has any merit or not is another matter, but if my memory serves me well, you said you were starting to understand my theory. What happened?
Title: Re: Is there a discrepancy with the equivalence principle?
Post by: timey on 11/04/2018 01:07:20
An additional Axiom: "The speed of light cannot exceed the local rate of time"
Speed being length divided by time, this defines the "rate of time" as being measured in meters per second, and clearly being a knowable quantity.

What is the local rate of time at some known place on earth? It must be measurable or calculable at, say, Teddington or Paris.

OK - yes the speed of light is a speed, and you say
speed being length divided by time, this defines the "rate of time" as being measured in meters per second, and clearly being a knowable quantity.

Decription 1: In general relativity a second changes length with each change of position of difference in the gravity potential, (leading to considerations by some physicists as to possibility of variable speed of light).
But also in general relativity the speed of light is measured as covering 1 metre per second in each and every reference frame.

Description 2: In special relativity, (which can only be used when travelling at a 'constant speed', ie: flat space with no gravity accelerations/decellerations) a second is held as a constant length against a constant speed of light.
But also in special relativity a metre changes length, and the magntude of the amount by which the metre changes length is dependent on by how much of a percentage of the speed of light the object has as it's travelling speed.

The objective of the spacetime structure of my model is to describe both general and special relativity effects within one mathemetical structure.
With regards to my model's additional axiom: "The speed of light cannot exceed the local rate of time", one must remember that in my model there is a seperate dimension of time for where m=0.  The local rate of time for light at a position in the gravity potential of the gravitational mass, will not be the same as the rate of time for mass in the same position in the gravity potential of the gravitational mass.

This is all dependent on my models description of an anistropically distributed energy in space that is proportional to the distribution of the gravity potential of the gravitational mass, where the axiom "the speed of light cannot exceed the local rate of time", goes hand in hand with the other added axiom "+energy=shorter seconds" where I am attributing the clock's increase in frequency of electron transitions with an increase in energy.
(giving a physical causation for high energy physics at elevation. ie: lighting, and other electron/positron observations)

Now one must remember that the objective is to describe both general and special relativity effects in one mathematical structure, where the geometry of space is not spatially curved, but is curved by the changes in time that my model adds for where m=0)

Looking at how this might work:
If we take description 1, general relativity's remit of faster length seconds in the higher gravity potential, which GR uses to describe an extended hypotenuse for non Euclidean geometry/Riemannian geometry/curvature...where we consider that the speed of light can be held relative to those faster/shorter length seconds, (which is a known general relativity consideration)...
And then, going to description 2, consider (as a new venture) that special relativity is using a constant speed of light across this curved area of space that is the gravity potential of the gravitational mass. As per this remit, as a combination of both considerations, then from the special relativity point of view the speed of light is becoming lesser in the faster time. If not, then one would have to hold the speed of light relative to each faster ticking clock in the higher potential in order to maintain a constant speed for the light in each of the reference frames it is passing through as it travels into the higher potential of the gravitational mass.

Consideration 1: So my model says that the speed of light cannot exceed the local rate of time, and that the local rate of time for light in the higher potential comprises of longer length seconds.  These longer length seconds will be exactly as much longer in length as the amount by which the shorter seconds are shorter, as compared to the observers clock, that is in the referece frame with the observer of the faster ticking clock.

Now we can go back to description 2:
quote:
"But also in special relativity a metre changes length"

Under the remit of consideration 1, this is the same as saying that a reference frame with a shorter length second will measure llight travelling a shorter distance than 299 792 458 metres per second, ie: a shorter metre.

Consideration 2: And we can also say that an object that is travelling at a constant speed 'through' the longer length seconds that my model adds, will apear not to have travelled as far. ie: a shorter metre.

This all is however complicated by:
Description 2 quote:
"and the magntude of the amount by which the metre changes length is dependent on by how much of a percentage of the speed of light the object has as it's travelling speed."

Ok, well we can uncomplicate that somewhat, and say that any speed the object has is a consequence of the gravitational potential, and any acceleration or decelleration that an object experiences in the gravity potential is gravity related.

So - forget about Paris or Teddington, and remembering that these considerations are concerning an alternate spacetime structure that will fit observation, let's go back to clocks 1 on the ground, clock 2 elevated above clock 1, and clock 3 elevated above clock 2, and a light source radiating electromagnetic radiation that is escaping the gravity well.

The escaping electromagnetic radiation is observed by clock 2 as redshifted as compared to how clock 1 observes the radiation.  Clock 3 observes the radiation from the ground as more redshifted than clock 2 does.

Clock 3 observes that clock 2 is less redshifted than clock 1, but... (and please correct me if I am wrong) Clock 3 observes the same magnitude of redshift of clock 1 that it observes of the redshift of the electromagnetic radiation radiating from the light source on the ground.
The clock on the ground, clock 1, observes clock 3 to be blueshifted, and...(please correct me if I am wrong) clock 1 observes clock 3 to be blueshifted by the same magnitude as clock 3 observes clock 1 to be redshifted.

My model states that the electromagnetic radiation from the light source on the ground is measured as redshifted by clock 3 partly b/c clock 3 is running faster, and partly b/c it has lost frequency due to (this is where conventional physics say 'kinetic energy loss') the slower rate of time that is occurring in space.
(this being compatible with overlaying special relativity onto curved space via using a constant speed of light for where seconds are getting shorter, noting that this isn't taking into consideration any special relativity slowing of time for the light, or object. How light perceives it's time is not relevant, and for an object who's motions are purely gravity related, in my model 'motion related time' is included as part and parcel of the observation of a clock.  This will be fully described in pages 9, 10, and 11)

The clock is then ticking faster, not b/c 'time' runs faster in space, but as an m in relation to M phenomenon.
Now this is where the other axiom that I add comes in.  This being "+energy=shorter seconds", where the 'energy' of space is anistropically distributed proportional to the gravity potential of the gravitational mass, and m near M has more energy (higher frequency electron transitions) in the higher gravity potential.
(giving a physical causation for high energy physics at elevation. ie: lighting, and other electron/positron observations)

So - Clock 3 observes clock 2 as less redshifted than clock 1.  Clock 2 has an increased frequency of electron transitions as compared to clock 1.  However, clock 3 is observing clock 2 as a light signal, and what I'm going to say is that clock 2 is not ticking as fast as clock 3 thinks it is.
Clock 3 is observing a redshifting of the light released by clock 2.  Half of the magnitude of the shift is due to the amount by which clock 3 is ticking faster than clock 2, and the other half is due to the light travelling into the space of a lower energy higher gravity potential, ie: slower time where m=0. (the extra length in wavelength is a temporal addition instead of a spacial addition)
Clock 2 is observing a blueshift of the light released by clock 3.  Again clock 3 isn't ticking as fast as that blueshift observation suggests.  Half of that observation is due to clock 3 ticking faster, and half of that observation is due to the light travelling into the space of a higher energy lower gravity potential, ie:faster time where m=0. (the lesser length in wavelength is a temporal subtraction instead of a spatial subtraction)

Overlay this with the special relativity length contractions that increase with increased speed (from the observers perspective), and what my model does is either describe why length contractions occur (for special relativity) as a function of both dimensions of time and their time dilations, or re-describe the curvature of space (for general relativity) as a function of changes in a time that occurs where m=0.

That is what page 5, that I am working on now, is concerning.

I kind of recommend that you wait to see the entire paper before trying to understand how my model fits all of these aspects into a contracting scenario, although any help with presentation (and with maths, should you finally understand what I'm up to), would be greatly appreciated.
Title: Re: Is there a discrepancy with the equivalence principle?
Post by: Colin2B on 11/04/2018 05:34:10
I’m just going to pick up a few items of terminology in case you use this in your paper.

“leading to considerations by some physicists as to possibility of variable speed of light” - most physicists consider this to be the case as per Shaperio delay, but due to length contraction/time dilation

“But also in general relativity the speed of light is measured as covering 1 metre per second in each and every reference frame.” - no it isn’t measured. For convenience c is sometimes set at 1 to simplify calculations.

“In special relativity, (which can only be used when travelling at a 'constant speed', ie: flat space with no gravity accelerations/decellerations) a second is held as a constant length against a constant speed of light.” - SR can be used for acceleration. You need to say “flat spacetime”. Length of a second is dilated in SR.

Sorry if you feel this is a painful process, but if you had ever been involved in a full peer review process you would think you have got off lightly.
The process is there to ensure there are no obvious trip up points which would lead to instant rejection. The final step, which you will need to go through is a spelling, typo, grammar check and double check reference numbers and Fig numbers - easy for them to get separated during editing.

Title: Re: Is there a discrepancy with the equivalence principle?
Post by: alancalverd on 11/04/2018 07:26:21
The details of the model don't concern me yet. I'm just interested in the concept of "rate of time". If you assert that it is a meaningful quantity, then you should be able to state or calculate it for a location where we generate time signals and all the relevant variables (e.g. g) are known.

No need to worry about the numbers, but if you can give an expression for the rate of time in terms of whatever you consider affects it, we can do the calculation and demonstrate the validity of the basis of your model.
Title: Re: Is there a discrepancy with the equivalence principle?
Post by: timey on 11/04/2018 11:52:55
Colin - it's not these type of posts that you make here that are painful. This is constructive criticism, which is indeed helpful.

No I wasn't planning on including theses descriptions in my paper. I was going to be relying on the diagrams to show how length contractions or curvature can be partially described, or described by my added dimension of time for where m=0, noting that in the description of my post that at no point was the special relativity slowing of time used.

But with the Shapiro delay there lies the perfect opportunity to check and see if my theory can actually be mathematically viable.
Since you and Alan know maths, it would be possible for you to ascertain the viability of my idea via this Shapiro effect observation if you were up for it? If you/you's are up for it, then I could post how 'I' see the possibility of describing the Shapiro effect observation via my theory later on tonight?
Title: Re: Is there a discrepancy with the equivalence principle?
Post by: timey on 13/04/2018 13:35:01
OK, so I guess the idea of re-describing the precise magnitude of the Shapiro effect via the remit of my theory as a mathematical venture is a non-starter.
For the record, all that is required is an inversing of the curvature of the geometry.

It would help me a lot if someone could confirm exactly what percentage of the magnitude of observed redshift, of electro magnetic radiation escaping into the higher gravity potential, current physics attributes to the measurement of the observed redshift by the faster ticking clock in the higher potential - in comparison to the percentage of the magnitude of the observed redshift of the electro magnetic radiation that is attributed to kinetic energy loss in the higher potential...
(Remembering that the observation of light can only be observed when the light reaches the reference frame it is being observed from)
Title: Re: Is there a discrepancy with the equivalence principle?
Post by: Colin2B on 13/04/2018 18:45:50
OK, so I guess the idea of re-describing the precise magnitude of the Shapiro effect via the remit of my theory as a mathematical venture is a non-starter.
For the record, all that is required is an inversing of the curvature of the geometry.
Sorry, must have misunderstood, i thought you were going to post something for us to look at.

I’m going to be out for a while with min interaction/thinking time, but hopefully back after next week.
Title: Re: Is there a discrepancy with the equivalence principle?
Post by: timey on 13/04/2018 21:32:56
No - I was waiting to hear if you and Alan were up for attempting a re-describing of the Shapiro effect magnitude via the remit of my theory.  This would involve some participation on your part in identifying how the Shapiro effect is being calculated, where I can tell you at each manipulation of the calculation as to how my theory inverts the curve.

If at each point where curvature is calculated, the inverse (or negative) curvature is used as a replacement, the amount of Shapiro effect delay for the inversed (negative) curvatures will be, by due logical process, exactly the same as for the non-inversed (positive) curvatures.

Anyway, maybe after next week when you get more time.
Title: Re: Is there a discrepancy with the equivalence principle?
Post by: alancalverd on 14/04/2018 07:42:18
As I understand it, the Shapiro delay is the difference between the theoretical straight-line propagation of a signal and its actual geodesic path in the presence of an intervening gravitational field. If you invert the geodesic curvature, the "lensing" effect is inverted from "convex" to "concave" and the signal will not be received at all.

Apropos the calculation of redshift, the "clock rate" or "kinetic energy" calculations give identical results because they are alternative ways of describing the same thing. The good news is that the result is supported by experimental data.
Title: Re: Is there a discrepancy with the equivalence principle?
Post by: Colin2B on 14/04/2018 09:54:53
In addition to what Alan says about the lensing, inverting the curvature doesn’t give the same result for time delay.
The original calculation uses Schwarzschild metric and increased distance so the inverse calculation would be complex. However, rough idea - consider you model where light is passing through different rates of time. In current view light starts far away in fastest rate of time, approaches mass, spends short time there and goes on to spend longer in faster rate area. The inverse of this means light spend longest time in slow-time area and short time in fast-time area. Clearly the 2 are not mirror images. To get the same result as Shapiro would require a much more complex system than you are proposing.
Title: Re: Is there a discrepancy with the equivalence principle?
Post by: timey on 15/04/2018 00:51:32
As I understand it, the Shapiro delay is the difference between the theoretical straight-line propagation of a signal and its actual geodesic path in the presence of an intervening gravitational field. If you invert the geodesic curvature, the "lensing" effect is inverted from "convex" to "concave" and the signal will not be received at all.

Apropos the calculation of redshift, the "clock rate" or "kinetic energy" calculations give identical results because they are alternative ways of describing the same thing. The good news is that the result is supported by experimental data.

This being in reply to both @Colin2B 's post as well as @alancalverd 's

Ok - so 'inverting the geodesic curvature', as in 'light being bent away from an intermediary mass' is not the type of inversing that I am suggesting.  As Colin's post addresses, the matter of inversing the curves is more complex than this.
And it goes without saying that the fact that light is observed to be bent towards an intermediary mass to cause lensing is not invertable.

For the benefit of the lay person readers:
https://en.wikipedia.org/wiki/Shapiro_time_delay
Quote
wiki: "The time delay effect was first observed in 1964, by Irwin Shapiro. Shapiro proposed an observational test of his prediction: bounce radar beams off the surface of Venus and Mercury and measure the round-trip travel time. When the Earth, Sun, and Venus are most favorably aligned, Shapiro showed that the expected time delay, due to the presence of the Sun, of a radar signal traveling from the Earth to Venus and back, would be about 200 microseconds"

Alan, you say:
As I understand it, the Shapiro delay is the difference between the theoretical straight-line propagation of a signal and its actual geodesic path in the presence of an intervening gravitational field.

So first off, when you say "the theoretical straight-line propagation of a signal", it has to be taken into consideration that in bouncing a signal off Venus (or Mercury) the time increases between Earth and Venus, and the spacial curvature between the gravity wells of Earth and Venus are part and parcel of that 'initial' time measurement of the experiment.

Quote
wiki
"In a nearly static gravitational field of moderate strength (say, of stars and planets, but not one of a black hole or close binary system of neutron stars) the effect may be considered as a special case of gravitational time dilation. The measured elapsed time of a light signal in a gravitational field is longer than it would be without the field, and for moderate-strength nearly static fields the difference is directly proportional to the classical gravitational potential, precisely as given by standard gravitational time dilation formulas."

So - question 1:
When wiki say "and for moderate-strength nearly static fields the difference is..." ...does this mean that the following calculations are based on there being flat spacetime between Earth and Venus?

Moving down the wiki page:
Quote
wiki
"Shapiro's original formulation was derived from the Schwarzschild solution and included terms to the first order in solar mass (M) for a proposed Earth-based radar pulse bouncing off an inner planet and returning passing close to the Sun:"

...and then there is the first equation. The description of the equation goes on to say:

Quote
wiki:
"The right-hand side of this equation is primarily due to the variable speed of the light ray; the contribution from the change in path, being of second order in M, is negligible"

So firstly, here it is stating that "the right hand side of the equation is primarily due to the variable speed of the light ray", where presumably this variable speed is being caused by the influence of the intermediary mass (the closer sun).  This re-enforcing question 1, being the question of whether this calculation of a variable speed for the light is being calculated from the initial conditions of a flat spacetime between Earth and Venus?

And secondly it is stating:
Quote
wiki: "the contribution from the change in path, being of second order in M, is negligible"

Question 2:
Does this mean that it is only the time variants being calculated here, with the 'geodesic curvature' (that is observed via lensing in other experiments) being negligable?

Moving down the wiki page further, the maths appear to change to geometry related, and the Eulidean dot product is applied, arriving at the description:
Quote
wiki:
"which is the extra distance the light has to travel"

Question 3:
So as per you saying:
Apropos the calculation of redshift, the "clock rate" or "kinetic energy" calculations give identical results because they are alternative ways of describing the same thing.

Am I seeing here within the Shapiro Effect maths 2 differing means of arriving at the same results?

Laying out how my model makes it's approach:

My model's calculation of the Shapiro Effect would incorporate including the factor of Earth and Venus being in their own gravitational wells.  This is the first instance where the curve is inverted.
General Relativity states this curve as being comprised of slower time at ground level Earth and Venus, and faster rates of time inbetween.
My model states the exact opposite.  It states this curve as being comprised of faster time at ground level Earth and Venus, and slower time inbetween. (In my model it is only objects that are comprised of mass will register faster time in the higher potential, this being an m near M phenomenon)

Note that: The measurement of the amount of time it takes for light to arrive at and bounce back from Venus is determined via experiment and does not discount light taking longer to travel the distance.  In fact if light takes longer to travel the distance, then there is no need to add spatial curvature to the distance. This spatial curvature is already described as a temporal effect by the slower time.
(This is an important distinction.  Instead of describing the geometry as spatial hills and troughs that objects fall into and climb out of, I have introduced a spatially flat geometry where gravitational wells are comprised of rates of time that get progressively faster, and objects moving at constant speed are accelerated by these increases in time when moving into a gravity well, and are decelerated by the decreases in time when moving out of gravity well.  But this isn't to say that light does not bend towards a gravitational mass, as will be discussed in a mo)

Now we are moving the Sun closer and bouncing the light to Venus and back past the presence of the Sun.

General Relativity states that the Sun in closer proximity will cause time to go slower and that this will slow the faster rate of time... (or are the maths starting from a basis of flat spacetime between E and V?)  ...will slow the faster rate of time that is between Earth and Venus, causing a delay in the return of the signal.

My models states the opposite, and that the Sun is running slightly faster time than Earth and Venus...
(much like satelites of E, where we observe that clocks run slightly faster time above E, but clocks on satelites to E run slightly slower clocks than E, due to SR motion related time dilation)
...and that the closer proximity of the Sun will cause the slower time in the space between Earth and Venus to run slightly faster. It will also cause clocks on E and V to tick marginally faster*, where the latter affects the measurement of the former, (bur dirsnt cancel) leaving us with a faster return time than the measurement when the Sun was not so close... (contrary to experimental evidence)  However, b/c light has energy, and energy is attracted to mass/mass is attracted to energy, the blueshifted light will be curved towards the intermediary mass (Sun).  The amount by which the light is curved is an attraction inwards, accentuated into faster motion by the faster rate of time near the Sun.  The extended curve comprises an extra distance travelled by the light.  (This is not the same as saying that light tavelling from Earth to Venus and back takes longer than the speed of light as per Earth clock to travel, and therefore space must be spatially curved with extra distance. In this 'new description' light is curving across Euclidean geometry to travel extra distance)

Weighing the extra distance travelled against the amount by which the slower time between Earth and Venus has been sped up by the closer proximity of the Sun, in relation to the amount by which the closer proximity of the Sun has increased the tick rates of clocks on Earth and Venus*, one should arrive at the amount by which the light took longer to travel due to the extra distance travelled via the curve of 'gravitational attraction' towards the Sun.
(*Neither Earth or Venus are any closer to the Sun, so no change in gravity potential height from Sun = no slowing of time due to being closer to Sun. When the 3 bodies align with Sun in middle, this causes a temporary overall increase in g field and a very slight 'increase' in tick rate.)

This assembly relies on using a 'newly introduced' time and time dilation factor for where m=0, while using the concept of observed time dilation of clocks in relation to gravitational M, and states that the energy of light will be 'gravitationally attracted' towards the gravitational M via blueshifted energy increases, and this attraction is accentuated by the faster rates of time nearer the gravitational mass.

This will not cause a lensing effect to be concave rather than convex.  Light moving into a gravitational field will be blueshifted and move faster as it is curved inwards  via 'gravitational attraction' towards the intermediary mass, and redshifts as it leaves again having had it's direction slightly changed by the inward curve, ie: more light arrives at observation point Earth, ie: lensing.

So - in order to ascertain if my model's remit can produce the same time delay results as the Shapiro Experiment recorded - first it becomes important to know, via the equations that show this Shapiro result to be compatible with conventional GR and SR, whether or not it was a flat spacetime that was used between Earth and Venus in order to calculate the variations in time that the presence of the Sun was calculated to have effected.

Any idea?
Title: Re: Is there a discrepancy with the equivalence principle?
Post by: alancalverd on 15/04/2018 07:57:20
This re-enforcing question 1, being the question of whether this calculation of a variable speed for the light is being calculated from the initial conditions of a flat spacetime between Earth and Venus?

As I understand it, yes. And once again, two ways of calculating the same thing. In the Shapiro case I find geodesics in a plastic spacetime rather easier to visualise than time dilatation in a Minkowski spacetime. But then I am very old and feeble of mind.
Title: Re: Is there a discrepancy with the equivalence principle?
Post by: Colin2B on 15/04/2018 11:04:44
Quote
wiki:
"The right-hand side of this equation is primarily due to the variable speed of the light ray; the contribution from the change in path, being of second order in M, is negligible"
So firstly, here it is stating that "the right hand side of the equation is primarily due to the variable speed of the light ray", where presumably this variable speed is being caused by the influence of the intermediary mass (the closer sun).  This re-enforcing question 1, being the question of whether this calculation of a variable speed for the light is being calculated from the initial conditions of a flat spacetime between Earth and Venus?
As Alan says, plastic spacetime the curvature of which results in both time and distance changes, so not flat spacetime - else there would be no ‘influence of the intermediary mass’, but flat space ie no curvature of the path. But as Alan says you can separate the 2 to see individual effects.
This is the part due to time dilation. Close to the sun, the effective rate at which time passes is slowed - according to current theory - that slowing is dependant on the GP at the points it passes through, so it slows down both on way in and out (this is not the same as redshift/blueshift) and the time taken is the integral over the entire path. That integral will not be the same if you just invert the time dilations.

Elsewhere you say “Instead of describing the geometry as spatial hills and troughs that objects fall into and climb out of”. Current theory does not describe the geometry as spatial hills and troughs, they are time-dilation hills and troughs the same as you are proposing.

And secondly it is stating:
Quote
wiki: "the contribution from the change in path, being of second order in M, is negligible"

Question 2:
Does this mean that it is only the time variants being calculated here, with the 'geodesic curvature' (that is observed via lensing in other experiments) being negligable?"
The lensing itself is not negligible, what is negligible is the additional path length due to lensing. The lengthening of the path due to gravitational bending is only 2.26m but the delay measured by Shapiro was round 200μs equivalent to a distance of 60km.

It would be interesting to work through your scenario for inverted time, can’t promise anything, but it might be a useful exercise for times when I can do nothing but lie back and think.
Title: Re: Is there a discrepancy with the equivalence principle?
Post by: timey on 15/04/2018 16:37:29
Elsewhere you say “Instead of describing the geometry as spatial hills and troughs that objects fall into and climb out of”. Current theory does not describe the geometry as spatial hills and troughs, they are time-dilation hills and troughs the same as you are proposing.

Yes - except that GR is (in effect) pasting 'physically', what is in effect longer metres, onto space where there are shorter length seconds in the higher potential, via the geometry of the GR curvature. (light, at light speed per Earth second, travels further than a metre per Earth second in the higher potential)

My model is stating slower seconds* in the higher potential in relation to constant length metres that the light is taking longer to travel.
This construct gives the same result as the GR assembly, and the physical puzzle of curvature being comprised of extra spatial distance is conveniently negated.
(*faster ticking clocks/mass then being an m near M phenomenon)

Checking against SR, if we use SR to measure the light's round trip between Earth and Venus, we are using flat spacetime and length contractions.
GR is saying that seconds are getting shorter in space, and distances are longer via curvature.
So in SR, by using a constant speed of light across a flat space, this (in effect) is the same thing as stating seconds getting longer in the face of the GR remit of seconds getting shorter in the higher potential.
Where in GR remit my model negates the curvature (extra distance) via this addition of a concept of longer seconds in the higher potential stating that the light takes longer to travel a metre.

Now this is complicated, but b/c my model has negated the extra distance of curvature in this manner for GR remit, the SR length contractions calculated of an object inline motion can then be attributed as a product of calculating a constant speed of light over a flat distance, where in fact light speed is covering the metres of the distance in longer period of time, ie: longer seconds  than the speed of light held relative to an Earth second would.  This being due to slower/longer seconds in the space of the higher potential of the gravitational mass.
So this additional dimension of time that my model adds 'should' describe either a) a negative temporal curvature in flat spatial geometry as a replacement for the GR geometry - or b) a negative temporal curvature in flat spatial geometry, where when the speed of light is held relative to the local rate of spacetime for m=0 at each point of change in gravity potential, this negates the SR length contraction calculations.

These slower seconds (of space) then are used as a partial* description of the observed deceleration of an object moving away from a gravitational mass, and conversely, are used as a partial* description of the observed acceleration of an object moving towards a gravitational mass.
(*where the other part of the descriptions are related to the tick rate (frequency of electron transitions) of clocks/mass/object, ie: pages 9, 10, 11 of video, although I am quite sure I can make better description of those pages now, and the energy 'gravitational attraction' of electromagnetic radiation.

Really happy to hear you are going to be having a think about it.  Regardless of whether or not there is any merit to doing so, I've had great fun with it myself.

@alancalverd lol ...you are neither old, nor feeble minded!
Title: Re: Is there a discrepancy with the equivalence principle?
Post by: jeffreyH on 15/04/2018 22:25:37
@timey said, Whether or not the theory has any merit or not is another matter, but if my memory serves me well, you said you were starting to understand my theory. What happened?

You stopped listening and started to talk nonsense.
Title: Re: Is there a discrepancy with the equivalence principle?
Post by: timey on 15/04/2018 23:29:23
@timey said, Whether or not the theory has any merit or not is another matter, but if my memory serves me well, you said you were starting to understand my theory. What happened?

You stopped listening and started to talk nonsense.

When one is describing a 'new theory' one does not 'listen', one 'describes'.

And since you have never stated that which you purported to understand, and - unless what you're saying is that any alteration to GR is nonsense which is in direct disagreement with you saying you purport to understand something of the theory - you also never state that which you think is nonsense!

So basically Jeff, your posting here is simply blah, blah, blah about what I'm not sure.  Hopefully at least you saying 'you give up' will have some meaning and you won't post this thread anymore.
I'd be grateful...and anyway, you can converse on the matter of curvature versus length contractions here:
https://www.thenakedscientists.com/forum/index.php?topic=72759.20#lastPost

On your own thread.

I won't 'ever' be responding to you again. I find your posts as if purposed to be hurtful, and I've got absolutely zero respect for that vibe.  Goodbye.
Title: Re: Is there a discrepancy with the equivalence principle?
Post by: Colin2B on 16/04/2018 15:32:29
Ok, last from me for a while. Just some points of detail about current theory/terminology in case you inlude references in your paper. All of this is probably just differences of wording rather than misunderstanding.

Yes - except that GR is (in effect) pasting 'physically', what is in effect longer metres, onto space where there are shorter length seconds in the higher potential, via the geometry of the GR curvature. (light, at light speed per Earth second, travels further than a metre per Earth second in the higher potential)
Although we can calculate them separately the distance and time changes are both the effect of curvature, so it isn’t really “pasted onto”.
Easier to see if we consider SR:

Checking against SR, if we use SR to measure the light's round trip between Earth and Venus, we are using flat spacetime and length contractions.
“using length contraction and time dilation.
A moving rod is seen, by an observer at rest, to be length contracted, but that contraction is because the time at the front of the rod is out of sync with the time at the rear of the rod - time dilation. We might calculate them separately, but length contraction and time dilation are the same thing.
So:

GR is saying that seconds are getting shorter in space, and distances are longer via curvature.
Best to say, seconds are getting shorter and distances are longer via curvature. It is the same effect, curvature causes both.

I will read through the rest, will be interesting to see the write up if you think you can make a better description. Good writing and fun while you are doing it  ;).
Title: Re: Is there a discrepancy with the equivalence principle?
Post by: timey on 16/04/2018 20:13:18
@Colin2B  For when you have recovered...

You say:
Best to say, seconds are getting shorter and distances are longer via curvature. It is the same effect, curvature causes both.

OK - so let's examine this.
First off the light.  Light travelling through a position of higher gravity potential is (via remit of GR) travelling at 299 792 458 metres per second of that position of higher potential.  The metre 'does not have to' change length b/c of the shorter length seconds at this position, as it is physically possible that the light can cover a constant length metre in a lesser amount of time, ie: no added distance of curvature.

So now the distance:
The extra distance of curvature is a requirement of observation. If one bounces a light ray off Venus, Venus is x distance from Earth and the light arrives back at Earth within a specific time period.  From the length of this time period 'as per experiment', it is clear that light speed being increased by the rate of faster time in space cannot be the only factor, b/c light takes too long to get back. Hence the added distance of curvature. Add the extra distance of the curvature and the time period in which the light takes to bounce off Venus and back is 'now' compatible with the increases in rate of time out in space.

This is how I understand it from a General Relativity view.

Calculate via SR and:
Now the space is flat between Earth and Venus. The light travels at speed of light as per Earth second for whole distance there and back with no reference to GR time dilation.  Length contractions occur.
Using SR to calculated the light's round trip from Earth to Venus and back will give same results as using GR, and this result is confirmed by experimental evidence.
(Shapiro Experiment uses Schwartzchild Metric to calculate inclusion of closer proximity of Sun during measurement)

This is my understanding (as per the length of the text allows) of the conventional viewpoint.
If this understanding (you may add to it) is a basis we can converse from, then I can show where 'my' changes to this basis occur as laid out in posts 1113 & 1115, and why a calculation made on the basis of my changes will also give the same result and be compatible with already existing experimental results.

Hope you feel better soon.
Title: Re: Is there a discrepancy with the equivalence principle?
Post by: timey on 22/05/2018 00:01:16
@Colin2B, I am just making use of this thread here to further discuss under the freedom of 'New Theories' a subject that I started on Physics board that @Janus was participating in here:

https://www.thenakedscientists.com/forum/index.php?topic=73266.msg541684#msg541684

Where I am trying to get to grips with the equatorial bulge. (Back to Shapiro Effect with you as and when you are up for it)

----------------------------
So repeating the consideration from that thread:

+relative speed, or centripetal speed = time going slower.

-minus relative speed, or centripetal speed = time going faster

+centrifugal force = time goes faster
-centrifugal force = time goes slower

higher GP = +centrifugal force + centripetal speed = time going faster

lower GP = -centrifugal force -centripetal speed = time going slower

+mass = time goes slower
-mass = time goes faster

Equatorial bulge sea level - Conventionally:

1: +mass = time goes slower
2: +height = +centrifugal force = time goes faster
3: + mass = -centrufugal force = time goes slower**
4: +centripetal speed = time goes slower
= same rate of time at sea level each longitude.

=slower, faster, slower, slower

= 3 slower + 1 faster


without bulge

+height = +centrifugal force = time goes faster
+ centripetal speed = time goes slower
= time goes faster

So +mass of equatorial bulge = time goes slower cancelling time going faster = same rate of time at sea level each longitude

Here we are saying that +mass is the same as -centrifugal force.

------------------------


Now under my theory everything is the same except:

+mass = time goes faster
-mass = time goes slower

Equatorial bulge sea level:

1: +mass = time goes faster
2: +height = +centrifugal force = time goes faster
3: + mass = -centrifugal force = time goes slower**
4: +centripetal speed = time goes slower
= same rate of time at sea level each longitude.

= faster, faster, slower, slower

= 2 faster, 2 slower.

without bulge

+height = +centrifugal force = time goes faster + centripetal speed = time goes slower
= 'time goes faster'

So +mass = time goes faster and +mass = -centrifugal force = time goes slower
= 'time goes slower'.

Add the above two together and 'time goes faster' and 'time goes slower' cancel
= same rate of time at sea level each longitude.

Here I am saying that + mass and -centrifugal force are NOT the same thing!

Now because where there is more mass/gravity, 'time goes faster', this gives a physical cause and effect description of why GRACE moves faster over the mountain than over the valley.
(This theory can be tested by placing a precision clock at LIGO to see how the clock ticks when a gravity wave hits.)
Title: Re: Is there a discrepancy with the equivalence principle?
Post by: timey on 23/05/2018 19:09:19
So going up a mountain:

1: +height = +centrifugal force = time goes faster
2: +mass = - centrifugal force = time goes slower
3: + mass = time goes faster
4: + centripetal speed = time goes slower

= 2 faster, 2 slower

Time running faster up a mountain (compared to sea level) being (as per my theory) due to:
less additional mass per extra height = less -centrifugal force
Title: Re: Is there a discrepancy with the equivalence principle?
Post by: Janus on 26/05/2018 01:12:07
@Colin2B, I am just making use of this thread here to further discuss under the freedom of 'New Theories' a subject that I started on Physics board that @Janus was participating in here:

https://www.thenakedscientists.com/forum/index.php?topic=73266.msg541684#msg541684

Where I am trying to get to grips with the equatorial bulge. (Back to Shapiro Effect with you as and when you are up for it)

----------------------------
So repeating the consideration from that thread:

+relative speed, or centripetal speed = time going slower.

-minus relative speed, or centripetal speed = time going faster

+centrifugal force = time goes faster
-centrifugal force = time goes slower

higher GP = +centrifugal force + centripetal speed = time going faster

lower GP = -centrifugal force -centripetal speed = time going slower

+mass = time goes slower
-mass = time goes faster

Equatorial bulge sea level - Conventionally:

1: +mass = time goes slower
2: +height = +centrifugal force = time goes faster
3: + mass = -centrufugal force = time goes slower**
4: +centripetal speed = time goes slower
= same rate of time at sea level each longitude.

=slower, faster, slower, slower

= 3 slower + 1 faster


without bulge

+height = +centrifugal force = time goes faster
+ centripetal speed = time goes slower
= time goes faster

So +mass of equatorial bulge = time goes slower cancelling time going faster = same rate of time at sea level each longitude

Here we are saying that +mass is the same as -centrifugal force.

------------------------


Now under my theory everything is the same except:

+mass = time goes faster
-mass = time goes slower

Equatorial bulge sea level:

1: +mass = time goes faster
2: +height = +centrifugal force = time goes faster
3: + mass = -centrifugal force = time goes slower**
4: +centripetal speed = time goes slower
= same rate of time at sea level each longitude.

= faster, faster, slower, slower

= 2 faster, 2 slower.

without bulge

+height = +centrifugal force = time goes faster + centripetal speed = time goes slower
= 'time goes faster'

So +mass = time goes faster and +mass = -centrifugal force = time goes slower
= 'time goes slower'.

Add the above two together and 'time goes faster' and 'time goes slower' cancel
= same rate of time at sea level each longitude.

Here I am saying that + mass and -centrifugal force are NOT the same thing!

Now because where there is more mass/gravity, 'time goes faster', this gives a physical cause and effect description of why GRACE moves faster over the mountain than over the valley.
(This theory can be tested by placing a precision clock at LIGO to see how the clock ticks when a gravity wave hits.)
Sorry for taking so long to reply to this but I've been away on vacation. 
You cannot have +/- Relative speed If a clock is moving relative to you, you measure it as running slow. 
As far as tangential speed and "centrifugal force" go you can calculate time dilation using either one, but not both.  If you are calculating it from an inertial frame, then you use tangential speed, If you are calculating it from the non-inertial rotating frame, then you can use the potential due to the apparent centrifugal force.  What you cannot do is combine their effects, because they are just two ways of looking at the same thing.

If you want to work out the time dilation for a clock traveling in a circle all you need to know is its tangential speed.  (If you put two clocks in circular paths with different radii but traveling at the same tangential speed, they will run at the same rate even though one of them is undergoing a larger centripetal acceleration.

If you want to work gravitational time dilation in, you can do use the following for the total time dilation as measured from the inertial frame. 
T = t/sqrt(1-2GM/rc^2- w^2r^2/c^2)  where w is the angular velocity of the rotation.  If you want to do in the rotational frame you have to use the integral of the centrifugal force, and you end up with the same equation.

An oblate spheroid adds an additional complication.  First consider the scenario with no rotation.  This can be look at a like a spherical body with a world girdling "hill" around the equator.  In this situation, walking from either pole to the equator is walking "uphill" and you are moving to a higher gravitational potential, where clocks run faster than they do at the poles.  If the "hills" were mass-less then you could use the simply version of the gravitational time dilation equation to work out just how much faster. 
But since they aren't, it is a bit more complicated.  The way the mass is distributed will have an effect, and it would take a bit of additional calculation to work out just how much difference this will account for.

If the Body is rotating and had no gravity, then someone moving from pole to equator would be effectively moving "downhill" to a lower potential causing clocks to run slower at the equator than at the poles, since the tangential speed is greatest at the equator.

If, as we have with the Earth, both effects are in play and the degree oblateness is due to the combination of the Gravity of the body and its rotation due to the plasticity of the body, we get the case where these two time dilation effects combine in such a way that all clocks at mean surface "level" run at the same rate.  ( if the surface is bumpy), you'll get variations between depressions and rises.)
Title: Re: Is there a discrepancy with the equivalence principle?
Post by: timey on 26/05/2018 19:32:10
An oblate spheroid adds an additional complication.  First consider the scenario with no rotation.  This can be look at a like a spherical body with a world girdling "hill" around the equator.  In this situation, walking from either pole to the equator is walking "uphill" and you are moving to a higher gravitational potential, where clocks run faster than they do at the poles.  If the "hills" were mass-less then you could use the simply version of the gravitational time dilation equation to work out just how much faster. 
But since they aren't, it is a bit more complicated.  The way the mass is distributed will have an effect, and it would take a bit of additional calculation to work out just how much difference this will account for.

If the Body is rotating and had no gravity, then someone moving from pole to equator would be effectively moving "downhill" to a lower potential causing clocks to run slower at the equator than at the poles, since the tangential speed is greatest at the equator.

If, as we have with the Earth, both effects are in play and the degree oblateness is due to the combination of the Gravity of the body and its rotation due to the plasticity of the body, we get the case where these two time dilation effects combine in such a way that all clocks at mean surface "level" run at the same rate.  ( if the surface is bumpy), you'll get variations between depressions and rises.)

As you say, in adding the mass considerations of an oblate spheroid, that is also rotating (has in fact been caused by roatating), matters become more complicated than 2 time dilations cancelling each other out. And because our equitorial bulge consideration is oblate and rotating there would seem a little more finess to the reality that 'could' be examined.

My point here is that if one sets up a spinning wheel with 2 identical objects attached by string to the wheel - where 1 string is longer than the other - then the object on the longer string will travel at a greater speed, the outward force will be stronger, and the longer string will 'feel' a greater tension, than the shorter string.

GRACE is showing us that while our gravimeters 'on' the mountain register lower 'inward' gravitational pull, that gravity up a mountain is much stronger than in the valley.
Stronger gravity is supposed (not proven) to slow time down. Increased centripetal speed (relative motion) is proven to slow time down.
So at the top of a mountain there are 2 aspects (conventional physics) slowing time down.
It is proven that a clock ticks faster at the top of the mountain.
The 'conventional calculation' say that this is due to the weaker gravity that the clock feels at top of mountain (as per our gravimeters).

I am saying (as per GRACE) that the clock is not 'feeling' less gravity at the top of mountain. If it were feeling less gravity it would fly off the top of the mountain with the extra centrifugal force at that height.

So departing from 'how' conventional maths for time dilation are calculated...
What I see is the possibility that the clock/mass at top of mountain is 'feeling' a tension caused by two opposing forces. The inward pull of stronger gravity, and the outward push of stronger centrifugal force.

It would appear to me that with regards to our 'real life example' of time dilation on earth, which is  a non spherical/rotating example, that the 'conventional calculations' are considering that increased centrifugal force is the same thing as weaker gravity.
If the 'conventional calculations' factor in the total gravity at top of mountain, this slows time down, as well as the increase in centripetal speed slowing time down. It is then centrifugal force alone that speeds time up.

And, as I said in last post, adding gravity will decrease the outward centrifugal force, causing a 'minus centrifugal force' aspect to the calculation where added mass is causing height. (equatorial bulge)
This elicits 3 aspects that cause slower time versus 1 aspect that causes faster time, so 'conventional maths' cannot include the 'extra gravity' that the centrifugal force is opposing, because clocks will not run at same rate at sea level under a 3 aspects cause slower time and 1 aspect causes faster time remit.

That is why my theory suggests that adding gravity (adding mass) causes time to go faster, and minus centrifugal force causes time to go slower, where height is adding centrifugal force causing time to go faster, and increased centripetal speed at height causes time to go slower.
Now, with 2 aspects causing time to go faster, and 2 aspects causing time to go slower, these aspects 'can' cancel for same rate of time at sea level of every longitude of the equatorial bulge.

What I am suggesting @Janus is a different means of calculating the same observations.

My description (theory of +mass, or gravity = time goes faster) explores an alternative. An alternative that actually goes on to describe physical cause and effect mechanics (as an alternative to space geometry that is curved) for 'why' GRACE moves faster over the mountains...
...And provides a falsifiable prediction for a test of general relativity, where a precision clock can be placed at LIGO to undergo conditions (when a gravity wave hits) that do not incorporate a change in centrifugal force via hieght, and do not include a change in centripetal speed, but only include an addition of the gravity of the wave for the duration of the hit.

General Relativity predicts that clock will run slower.
My theory predicts that clock will run faster.

My theory is saying that the centrifugal force itself is having an effect on the clocks in addition to gravity having an effect on clocks.
Relativity considerations are saying that centrifugal force has an effect on gravity, and that gravity effects the clocks.
Relativity says that GRACE is travelling in slower time over the mountains in the stronger gravity, and that the acceleration is curvature related.
My theory is saying that GRACE is travelling in faster time over the mountains in the stronger gravity, and that the acceleration GRACE feels is due to the faster rate of time.

That is my theory.
(yes, this theory can describe grav.lens)
Title: Re: Is there a discrepancy with the equivalence principle?
Post by: timey on 26/05/2018 19:42:30
In addition to post above:

So - in order for an attempt at examining/calculating under differing remit, it is important to recognise all the aspects at play here:

1/ The GRACE experiment itself, in determining the lead craft as accelerating in the stronger gravity, is bouncing electromagnetic radiation off the leading craft. That electromagnetic radiation 'should' (according to GR) be moving at a slower rate, in the slower time, in the stronger gravity.
This, of course, would account for the electromagnetic radiation taking longer to return when it is bounced off the lead craft over the mountains. But I hardly think for a minute that the GRACE experimenters will have neglected to account for this factor in stating the lead craft as 'moving faster' over the mountains...?

2/ Gravimeters display gals per cm per second squared.

A second squared up a mountain is different to a second squared in the valley. An up a mountain shorter second squared may affect magnitude of gals per cm measurement.
Where GRACE finds stronger gravity over a mountain, a gravimeter at top of mountain is 'potentially' measuring via a shorter second squared, as compared to a measurement in the valley and this will amount to less gals per cm per shorter second squared.

('If' the gravimeter interpretation of a second 'is' affected by it's position in the gravity potential, this aspect should account for a percentage of the gravimeters measurement of weaker acceleration, and interestingly (I think) in the negative 'could' be used to account for an internal tension (in atom) that 'causes' a downward attraction, (ie: gravity)
This being because if one views the opposing tensions of gravity versus 'centrifugal force and centripetal speed' as a tensor for electron frequency, and electron frequency as a tensor for the frequency with which a magnetic moment occurs, (where a more frequently occurring magnetic moment causes a greater downward attraction) then all this description is lacking is a tensor between electron frequency and the strong and weak forces, where this approach may (I think), describe the internal timing of time crystals as well as the internal timing (electron frequency) of any atomic structure, incuding the different atomic structures used as time keepers in different types of precision clocks, where any atomic structure will then increase it's internal timing in the higher GP)
Title: Re: Is there a discrepancy with the equivalence principle?
Post by: timey on 02/06/2018 20:26:59
So - given that GRACE shows that gravity is stronger at top of mountain than in valley, can we all agree that despite how time dilation is conventionally calculated via weaker gravity at the top of mountain, the clock is actually 'feeling' stronger gravity at the top of the mountain than in the valley?
Title: Re: Is there a discrepancy with the equivalence principle?
Post by: Colin2B on 03/06/2018 21:14:16
So - given that GRACE shows that gravity is stronger at top of mountain than in valley,
GRACE doesn’t show this, it produces a Gravity anomaly map. Gravity anomaly maps show how much the Earth’s actual gravity field differs from the gravity field of a uniform, featureless earth surface with homogeneous density often called the ellipsoid. If you look at the theoretical calculation you will see that it assumes a homogeneous perfect sphere. If you are going to make comparisons with a real system you have to take into account uneven mass distribution, which is what GRACE does.
If we look at the Andes, on a uniform earth the value of g at sea level would be approx 9.8 and at a ht of 1km it would be 9.797, however if we take GRACE anomaly measurements into account the values would be less by 5x10-7 ie 9.7999995 and 9.7969995 respectively so the value of g at the top of the Andes is always less than that at the bottom so the tops are not feeling a stronger gravity than the valley.
However, it is still important to note what @Janus says, that the dilation does not depend on g but on the gravitational potential.

 
Title: Re: Is there a discrepancy with the equivalence principle?
Post by: timey on 03/06/2018 22:26:33
So where the GRACE anomaly map shows red at top of mountains, and it states that anywhere red is showing a stronger gravitational acceleration, are you saying that that the red areas are not showing stronger gravitational acceleration?

I am aware that gravity potential is calculated as a 'theoretical calculation' assuming a homogeneous perfect sphere, and @Janus has shown how there are 2 methods of 'conventionally calculating' time dilation, but that they cannot be used together.
He said:
"As far as tangential speed and "centrifugal force" go you can calculate time dilation using either one, but not both.  If you are calculating it from an inertial frame, then you use tangential speed, If you are calculating it from the non-inertial rotating frame, then you can use the potential due to the apparent centrifugal force.  What you cannot do is combine their effects, because they are just two ways of looking at the same thing."

But our Earth is both rotating and has gravity anomalies that differ from the conventional 'gravity potential'...
...And what I want to do is to calculate the rotational aspects of relative motion time dilation, and the gravity potential anomalies aspects of time dilation together, because that is what we observe occurring on our planet.
...And I want to do this as a modification of general relativity where the factor of the gravity that clearly holds a clock to the top of a mountain 'is' included. This being because this extra gravity holding the clock to the mountain against the centrifugal force 'is' a factor that exists, isn't it?

Edit: I just looked at the GRACE gravity map again, and it clearly show red at the top of mountains, and it show a graph that suggests that red is equal to stronger gravity. I can't be reading that wrongly, can I?

https://en.wikipedia.org/wiki/Gravity_Recovery_and_Climate_Experiment#/media/File:Geoids_sm.jpg
Title: Re: Is there a discrepancy with the equivalence principle?
Post by: Colin2B on 04/06/2018 07:17:56
Edit: I just looked at the GRACE gravity map again, and it clearly show red at the top of mountains, and it show a graph that suggests that red is equal to stronger gravity. I can't be reading that wrongly, can I?
If you look at the scale you can see it describes the diagram as showing gravity field anomolies, and the units are in milligals so it is clearly not an absolute gravity measurement.
Title: Re: Is there a discrepancy with the equivalence principle?
Post by: alancalverd on 04/06/2018 07:38:14
Looking down from 200 km to the top of a mountain, there is obviously more mass immediately below the satellite than at the same altitude over the ocean, so there will be a positive gravitational anomaly compared with an idealised uniform sphere.

F = GMm/r^2, as always, but at only 200 km above a 6000 km knobbly geoid, M, r and the net direction of F on m (remember Maskeleyne at Schiehallion) do not approximate to the ideal isotropic and point values that apply in the laboratory or astronomy.
Title: Re: Is there a discrepancy with the equivalence principle?
Post by: Janus on 04/06/2018 16:52:27
So where the GRACE anomaly map shows red at top of mountains, and it states that anywhere red is showing a stronger gravitational acceleration, are you saying that that the red areas are not showing stronger gravitational acceleration?

Edit: I just looked at the GRACE gravity map again, and it clearly show red at the top of mountains, and it show a graph that suggests that red is equal to stronger gravity. I can't be reading that wrongly, can I?

Yes, you are reading the map wrong, In at least one way.   You keep wanting to interpret the GRACE readings as the strength of gravity at ground level, When they represent the fluctuations from the mean value of gravity measured at the altitude of the Satellites.  To get how gravity changes at ground level, you would have to combine this information with exactly how far ground level is from the center of the Earth at the point of interest.   The gravity map supplied by grace doesn't include this second bit of information.
Also, your insistent claim that red is at the top of mountains isn't always true.  In the GRACE map:
(https://earthobservatory.nasa.gov/Features/GRACE/Images/geoids_sm.jpg)
There are areas where higher gravity is associated with mountains, bit also areas where it is associated with low spots, such as out in the North Atlantic. You weren't under the impression that the "bumpiness" in the Gravity map represented elevation of the ground at that point were you? It is just a way of "doubling up" on how they represented the gravity measurements. They are shown in both color and and relief.
If you want to look for a correlation between the gravity map and geological features, this tectonic plate map is of interest.
(https://parafaultlinecassandraurgl.files.wordpress.com/2015/06/plates-of-the-world.gif)

Note that there is some correlation between where plates meet and high gravity areas( including that region in the North Atlantic.)  This is likely due to how local density of the crust is effected.    In certain places, this will also correlate with mountian ranges ( West coast of the Americas and the Himalayas, for example), but that is because these meeting points between plates are subduction  zones, where mountains tend to be built.


Title: Re: Is there a discrepancy with the equivalence principle?
Post by: Janus on 04/06/2018 17:33:01


But our Earth is both rotating and has gravity anomalies that differ from the conventional 'gravity potential'...
...And what I want to do is to calculate the rotational aspects of relative motion time dilation, and the gravity potential anomalies aspects of time dilation together, because that is what we observe occurring on our planet.
...And I want to do this as a modification of general relativity where the factor of the gravity that clearly holds a clock to the top of a mountain 'is' included. This being because this extra gravity holding the clock to the mountain against the centrifugal force 'is' a factor that exists, isn't it?


You don't need a "modification" of GR.   You just use the GR equation
Sqrt(1 + 2u/c^2- w^2r^2/c^2)

Here u is the gravitational potential at the point you are interested in.
w is the angular velocity of the rotation
r is the radial distance from the axis of rotation for you point of interest.

Working out the exact value of u when taking into account the various fluctuations in Earth's gravity field is the finicky part.  But to be honest, it isn't really going to amount to much.   

Gravitational potential is, in effect, a measure of energy per unit mass needed  to move from one point relative to the gravity source to a point an infinite distance from the gravity source. 
Now as you first start lifting an object from the surface of the Earth, variations in local gravity will have some effect on the energy needed to lift the object a given height when you are close to the surface, But as you get further from the Earth, this becomes less and less of a factor and it gets washed out by the overall effect of the total mass of the Earth. (the further you get from the Earth the more it gravitational behaves as a single point source).   This means that the small local variations in surface gravity have a minuscule effect on the gravitational potential at any point when compared to, say, the difference caused by being a few kilometers further from the center of the Earth by being on a mountain top vs being at sea level.
Title: Re: Is there a discrepancy with the equivalence principle?
Post by: timey on 05/06/2018 13:25:11
@Janus. I think you misunderstand my purpose.

I am quite simply interested in calculating time dilation using both the 'relative motion' and the 'gravity potential' equations in order to arrive at a 'contant rate of time' for sea level at each longitude of the equatorial bulge. (this being on basis that the equatorial bulge constitutes both changes in height and changes in speed occurring simaltaniously)

That is all I am interested in. You have said that 'adding in' the 'extra mass' of each increase in the equatorial bulge would require some additional calculation.

I am saying that 'if you add in' that extra mass that contitutes an increase in height of the bulge from poles to equator, where an aspect of +mass=time goes slower (conventional GR remit) must now be added to the calculation - this will NOT, under the conventional means of calculating, result in a constant rate of time at sea level of each longintude of the equatorial bulge.

You may tell me that my understanding is amiss somehow and 'show me how' adding in the extra mass under the remit of +mass=time goes slower WILL result in a constant rate of time at sea level of any longitude...

***In which case I will then AGREE with you that there isn't any necessity for a modification of GR.***
Title: Re: Is there a discrepancy with the equivalence principle?
Post by: Janus on 05/06/2018 16:37:58
@Janus. I think you misunderstand my purpose.

I am quite simply interested in calculating time dilation using both the 'relative motion' and the 'gravity potential' equations in order to arrive at a 'contant rate of time' for sea level at each longitude of the equatorial bulge. (this being on basis that the equatorial bulge constitutes both changes in height and changes in speed occurring simaltaniously)

That is all I am interested in. You have said that 'adding in' the 'extra mass' of each increase in the equatorial bulge would require some additional calculation.

I am saying that 'if you add in' that extra mass that contitutes an increase in height of the bulge from poles to equator, where an aspect of +mass=time goes slower (conventional GR remit) must now be added to the calculation - this will NOT, under the conventional means of calculating, result in a constant rate of time at sea level of each longintude of the equatorial bulge.

You may tell me that my understanding is amiss somehow and 'show me how' adding in the extra mass under the remit of +mass=time goes slower WILL result in a constant rate of time at sea level of any longitude...

***In which case I will then AGREE with you that there isn't any necessity for a modification of GR.***
It has been already worked out.  There are two competing factors that determine the Earth's shape, its gravity and its spin.  If it weren't for gravity holding it together, its spin would tear it apart. As it is, a compromise is reached and we get an oblate spheroid.   The surface of this spheroid is a geoid, which means that local "vertical" is always perpendicular to the surface at any point of the surface. (which means that, unlike for the surface of a non-rotating spherical body, it doesn't always point towards the center of the body.).    This also means that the surface of this geoid is an equi-potential surface.  A good clue to this is that a fluid, such as water, will always settle to the lowest possible potential, and if the potential wasn't equal over the surface of the Earth, the oceans wouldn't follow its shape, but instead would pool up at either the poles or equator. 
The shift in mass towards the equatorial bulge in of itself factors in to the final shape. It is not an additional factor that has to be factored in later when doing calculations, it is already involved in causing the surface to be equi-potential.

Clocks at sea level tick at the same rate no matter how you try to calculate it.

Now if you wanted to, you could do all the math to determine the potential due to gravity alone at any point on the surface*, use that for u in the equation I gave in the my last post, along with angular velocity of the rotating Earth and the radial distance from the axis to get the time dilation that way, but you will get the same result.


* There is a formula for determining the gravitational potential over the surface of an oblate spheroid, but they assume a constant density throughout the body, and the Earth is not uniformly dense.
Title: Re: Is there a discrepancy with the equivalence principle?
Post by: timey on 05/06/2018 20:59:20
Ok - Yes I already know all of these aspects that you mention.

Clocks at sea level tick at the same rate no matter how you try to calculate it.

The point is that I am interested in calculating time dilation in another way.

I am interested in calculating time dilation in another way because SR is compatible with Quantum, but GR isn't. (And GR has it's own limitations)

Anyway, I thank you for your input. It's kind of you to take the time, but I'm not sure you understand what it is I am trying to do here.

I am going to sign off now but will include, for the record, my falsifiable prediction for a doable experiment, including the remit of the experiment and why this experiment constitutes a test of GR that has not yet been conducted.

FALSIFIABLE PREDICTION FOR A DOABLE EXPERIMENT

A further test of General Relativity can be conducted by comparing clocks that are ONLY experiencing a change in gravity.

So far precision testing of GR with clocks has been conducted concerning change in height in the gravity potential, where the clock is, in addition to a change in gravity, also experiencing a change in centripetal speed and centrifugal force.
And precision testing of GR has also been conducted concerning a change in relative motion, where the clock is not experiencing a change in gravity.

https://www.sciencealert.com/portable-atomic-clock-measures-gravity-first-time-relativistic-geodesy

In Febuary this year portable precision clocks were tested for the first time.

It is my suggestion that these portable clocks should be placed in circumstance where ONLY a change of gravity is occurring, in order to confirm that General Relativity is indeed correct in assuming that an increase in gravity slows time down.

This could be conducted by placing a clock at 2 different locations at the same longitude, and at the same height above sea level, where there is a known density difference in the geology of the locations, and thus compare how they tick.
This will constitute only a difference in gravity.

Or - much more simply, just place a clock at one of the gravity wave experiments and record how the clock ticks differently when a gravity wave hits, as compared to how it ticks normally.
This will also constitute only a change in gravity.

General Relativity predicts that a clock that experiences only a change (increase) in gravity will tick slower.

My modification of General Relativity predicts that a clock that experiences only a change (increase) in gravity will tick faster.*

*The consequences of the remit of my modification are, of course, extremely far reaching.

However I just cannot do this trying to make people understand what it is that I am saying anymore.
Now that these precision clocks are portable, I daresay that they are going to be used to further test the tenets of General Relativity.

I have voiced my suggestion, and will now say no more.

Thank you.
Title: Re: Is there a discrepancy with the equivalence principle?
Post by: timey on 14/08/2018 17:46:52
Here is my first paper, now published at Journal of Space Exploration...

http://www.tsijournals.com/articles/TSSE-18-2146.pdf
Title: Re: Is there a discrepancy with the equivalence principle?
Post by: timey on 25/08/2018 02:29:06
Here is my first paper, now published at Journal of Space Exploration...

http://www.tsijournals.com/articles/TSSE-18-2146.pdf

It's come to my attention that the link I provided is not working anymore, or maybe it never did and no-one told me (although it works ok for me, scratches head).

In anycase here is a link to the text version of my paper

http://www.tsijournals.com/articles/remixing-the-universe-by-vikki-ramsay-time-theory-13760.html


And here is the link where the PDF can be clicked on for the full version with diagrams:

http://www.tsijournals.com/journals/journal-of-space-exploration-current-issue.html
Title: Re: Is there a discrepancy with the equivalence principle?
Post by: Colin2B on 26/08/2018 07:06:50
Thanks, the link works.
I’ll have a read later.